Sunteți pe pagina 1din 416

c o l e c Ń i a

Referent ştiinŃific: prof. Cristian Alexandrescu

Autorii aduc mulŃumiri speciale SocietăŃii de ŞtiinŃe Matematice din România


pentru sprijinul acordat.

Lucrare elaborată în conformitate cu programele şcolare în vigoare şi avizată


de Comisia NaŃională de Matematică din Ministerul EducaŃiei şi Cercetării cu
nr. 25216/1999 pentru folosirea în clasă şi pregătirea suplimentară a elevilor.

Editor: Călin Vlasie

Corectură: autorii
Machetare, tehnoredactare & prepress: AR T C R EA TI V
Coperta colecŃiei: Andrei Mănescu

Descrierea CIP a Bibliotecii NaŃionale a României


Matematică : olimpiade şi concursuri şcolare : clasele IX-XII,
2009-2010 / Dan Brânzei (coord.), Gheorghe Iurea, Adrian Zanoschi, ... –
Piteşti : Paralela 45, 2010
ISBN 978-973-47-1020-1

I. Brânzei, Dan (coord.)


II. Iurea, Gheorghe
III. Zanoschi, Adrian

51(075.33)
371.384:373.3

 Copyright Editura Paralela 45, 2011


Prezenta lucrare foloseşte denumiri ce constituie mărci înregistrate,
iar conŃinutul este protejat de legislaŃia privind dreptul de proprietate intelectuală.
D AN B RÂNZEI ,
(coordonator)
G HEORGHE I UREA , A DRIAN Z ANOSCHI , P ETR U R ĂDUCANU ,
G ABRIEL M ÎRŞANU , A NDREI N EDELCU , I OAN Ş ERDEAN , C ARMEN B OTEA ,
R OXANA D IACONESCU , V ASILE B UL GĂREAN , C AMEL IA M AGDAŞ ,
G HE ORGHE M AIORESCU , I ONEL T RIFON , L UCIAN T UłESCU

matematică
olimpiade şi concursuri şcolare
clasele IX-
IX-XII

2009-
2009-2010
LISTA AUTORILOR DE PROBLEME

AdomniŃei C. Căiniceanu George GhiŃă RomanŃa Muşuroia N. René G.


Agnola Liana Chichirim Nelu GhiŃă V. Năchilă P. RovenŃa Ionel
Aldea A. Chiş M. Gimoiu Iuliana NăstruŃ D. Runceanu E.
Aleman I. Chiteş Costel Giugiuc Leonard Neagu Mihai Rusu Constantin
Alexandrescu Ciorăscu Gh. Mihai Necula G. Sandovici A.
Cristian Ciupală Cătălin Giurgi Vasile Nedeianu Dan Sârghie D.
Alexe Gheorghe Ciurea Raluca Gorcea V. Nedelcu Ion Sava Radu
Alistar C. Coandă Claudiu Gorgotă V. Negulescu A. Savu Ion
Ambrinoc Costică Constantinescu Grigore V. Negulescu Dan Săvescu Cristian
AmorăriŃei C. Gabriela GuiŃă Visilina NeŃu G. Schneider Cristian
Andrei Gheorghe Coravu Iuliana Gusta C. Nicolaescu Florin Schwarz Dan
Andrica Dorin Cosma Paul Guşatu I. Niculescu Liliana Sitaru Daniel
Andronache Marian Cuc I. Heuberger Cristian Olteanu Şerban Staicu Ion
Anghel Costel Cucoaneş M. Heuberger Dana Omniuc G. Stanciu Neculai
Antohe Florin Cucurezeanu I. Huiban V. Panaitopol L. Stavre Petre
Apostolescu C. Damian M. Ilie Romeo Pană Cătălin Stoleru C.
Apostolescu Cezar Danielescu Iulian Ionescu George Paponiu Dana Stretcu Daniel
Aron Roxana Daniilescu Gabriel Ionescu Marian Paşca A. Şerbănescu Dinu
Arventiev Dorin Demeter Ciprian Jinga Daniel Pau Monica Tămâian Traian
Baroni Marian Diaconu Ilie Lăzărescu Dragoş Păcurar Cosmin Teclici D.
Băcilă L. Dicu Florentina LobonŃ Gheorghe Perianu Marius Teler Marian
Băetu Ioan Dicu M. Luchian Dorel Peşca A. Teodorescu Dinu
Băiatu Paul Didraga I. Lupşor Viorel Petrică Dragoş Tivadar G.
Bălan Dumitru Doboşan Aurel Lupu Cezar Petriceanu Daniel Toloşi Marin
Bălan Rodica Doinaru Mihaiela Lupu T. Piscan Stelian Totolici Mihai
BătineŃu D.M. Dragomir Adriana Macrea M. Piticari Mihai Traian D.
BătrâneŃu Petre Dragomir Lucian Magdaş C. Pîrlog Elena TuŃescu Lucian
Băzăvan Eduard Drâmbe D. Manea Cosmin Pleşa Viorica łena Marcel
Bencze Mihaly Drăghici V. Marchitan B. Pop Alin łicuşi O.
Berghea V. Duma Iuliana Marchitan Gheorghe Pop Cristian łigăeru Angela
Bîrzescu Cătălin Dumbravă Vasile Marinache D. Pop Vasile łintea A.
Boeriu Gabriela Dumitrel Florian Marinescu Dan Popa Gabriel Ursărescu Marian
Bojor F. DuŃă Traian Maşca Ioana Popa Vasile Ursu Constantin
Borduş C. Eckstein Alfred Mărghidanu Dorin Popescu Călin Vâjâitu Viorel
Boroica G. Faynshteyn Oleh Miculescu Radu Popescu Dan Vasile Emil
Botea Carmen Găvan T. Mihalache D. Popoiu D. Velicu Andrei
Botea Viorel Georgescu Carmen Militaru C. Poroşniuc D. Vişan Dumitru
BourbăcuŃ Nicolae Georgescu Paul Mititelu Radu Prajea Manuela Vîrban T.
Burduşel Călin Gheorghe Iurea MoanŃă Cristian Purcaru Octavian Vlad Petru
Bursuc Ion Gherasim G. Monea Mihai Radu N. Zamfir Romeo
Caragea Constantin GhiŃă Gheorghe Monea SteluŃa Radu Vasile Zîrnă Cătălin
Cârmaciu M. GhiŃă Ioan Mortici Cristinel Rădulescu Sorin Zvonaru Titu

N.B. Autorii problemelor din această culegere sunt rugaŃi să contacteze serviciul de marketing-
management-resurse intelectuale al Editurii Paralela 45 şi să comunice adresele personale.
CLASA A IX-
IX - A

ALGEBRĂ

I. Elemente de logică matematică şi mulŃimi

1.1. Elemente de calcul propoziŃional


 2x + 3 
1. Se consideră funcŃia f: R → Z, f ( x ) =   , unde [a] reprezintă partea întreagă a numărului.
 4 
StudiaŃi valoarea de adevăr a următoarelor afirmaŃii:
a) Există a, b ∈ R – Z, a ≠ b, pentru care f(a) = f(b).
b) Pentru orice p ∈ Z, există m ∈ Z astfel încât f(m) = p.
c) A = {k ∈ Z | 1 + k = 2f(k2)} = ∅.
Etapa locală, Caraş-Severin, 2010
2. StabiliŃi valoarea de adevăr a propoziŃiei
 x + 1   x + 3   x + 3   x + 1
p: „(∀) x ∈ R, ecuaŃia  − = −  nu are soluŃii”.
 2   2   2   2 
Etapa locală, Vâlcea 2010, S. Valentin
3. Să se determine mulŃimile de adevăr ale predicatelor:
p1(x,y): |x + 5y – 17| + |3x – 4y + 6| = 0, x, y ∈ R;
 x − 1 x + 1
p2(x):   = 4 , x ∈ R.
 3 
Etapa locală, Ilfov, 2009
4. Dacă a, b, c, d ∈ R*, demonstraŃi echivalenŃa propoziŃiilor:
p1: a2 – b2 + c2 – d2 = 0 şi a – b + c – d = 0; p2: ak – bk + ck – dk = 0, oricare ar fi k ∈ Z.
Etapa locală, Teleorman, 2009

5
5. Să se determine valoarea de adevăr a propoziŃiei:
p: (∃ x∈ N*) ((–∞, 2x + 2008) ∩ N = {0, 1, 2, ..., 3x}).
Etapa locală, NeamŃ, 2009, prof. C. Botezatu
6. Fie M = {x ∈ Z | (∃) n ∈ Z, x + 5x + 4 = n }. Să se stabilească valoarea de adevăr a propoziŃiei
2 2

„M ⊂ [–5, 0]”.
Etapa locală, Călăraşi, 2009
7. Fie A, B, C, D patru puncte în plan şi pentru n ∈ N* considerăm propoziŃia P(n): AMn + CMn ≤
≤ BMn + DMn pentru orice punct M din plan, (∀) n ∈ N*. Să se arate că:
a) dacă există n ∈ N* pentru care P(n) este adevărată, atunci AB || CD şi AD || BC.
b) P(2) este adevărată dacă şi numai dacă AB || CD, AD || BC şi AC ≤ BD.
c) P(1) este adevărată dacă şi numai dacă A, B, C, D sunt coliniare, punctele A şi C sunt între B şi D şi
AB = CD.
Etapa locală, Cluj, 2009
8. Se dau predicatele ternare, de variabile a, b, c ∈ R:
p: „ax2 + bx + c ∈ Q, (∀) x ∈ R”; q: „a = b = 0, c ∈ Q”.
Să se arate că p ⇒ q.
Etapa locală, Braşov, 2009
9. Fie ecuaŃia ax + bx + c = 0, a, b, c ∈ R* distincte, cu soluŃiile x1 şi x2 şi propoziŃiile:
2

p: „ax1 + bx2 + c = 0”; q: „ 3 a 2 c + 3 ac 2 + b = 0 ”. Să se arate că p ⇔ q.


Etapa locală, Bihor, 2009
10. Se dau predicatele binare, de variabile x, y ∈ R:
1 1
p: „x2 + y2 + x + y + = xy ”; q: „xy + 3(x + y) < ”. Să se arate că p ⇒ q.
4 4
Etapa locală, Dolj, 2009
11.
11. Să se arate că următoarea propoziŃie este adevărată:
(∀) x ∈ R, (∀) m ∈ N*, (∃) p ∈ {0, 1, 2, ..., m – 1}, [mx] = m ⋅ [x] + p.
Etapa locală, Braşov, 2009
12. Să se determine valoarea de adevăr a propoziŃiei:
(∀) x ∈ R, (∀) m, n ∈ N*, (∃) q ∈ Z*, |q| ≤ max(m, n), [mx] – [nx] = (m – n)[x] + q.
Etapa locală, Bacău, 2009

1.2. Metoda inducŃiei matematice


1. ArătaŃi că orice număr natural n ≥ 2 poate fi scris sub forma n = 2k1 + 3k2 ∈ N.
Etapa locală, Mureş, 2010
2. Să se arate că numărul an = 3 + 2n + 1 este divizibil cu 4, oricare ar fi n ∈ N.
n+1

Etapa locală, Vrancea, 2010, D. Munteanu


3. Să se arate că pentru orice n ∈ N, există k ∈ N şi o alegere convenabilă a semnelor + şi – astfel
încât n = ± 12 ± 22 ± 32 ± ... ± k2.
Etapa locală, Gorj, 2010
6
n(4n 2 − 1)
4. a) Să se demonstreze că (∀) n ∈ N*, 12 + 32 + ... + (2n – 1)2 = .
3
b) Pentru n ∈ N*, să se determine x1, x2, ..., xn ∈ R care satisfac inegalitatea
3( x12 + x 22 + ... + x 2n ) − 6( x1 + 3x 2 + ... + (2n − 1) x n ) + n (4n 2 − 1) ≤ 0 .
Etapa locală, Maramureş, 2010, G. Boroica
5. Să se determine mulŃimea {a1, a2, ..., an, ...} ⊂ N*, astfel încât, oricare ar fi n ∈ N*,
a13 + a 32 + ... + + a 3n = (a 1 + a 2 + ... + a n ) 2 .
1 1 1
6. Dacă n ∈ N*, atunci 1 < + + ... + <2.
n +1 n + 2 3n + 1
Etapa locală, Ilfov, 2009
n
7. Să se calculeze ∑ k(k + 1) , apoi să se demonstreze rezultatul obŃinut prin inducŃie matematică.
k =1
Etapa locală, Bihor, 2009
8. a) Să se demonstreze că pentru orice n ∈ N* are loc egalitatea:
n (n + 1)(3n 2 + 7 n + 2)
1 + 2(1 + 2) + 3(1 + 2 + 3) + ... + n(1 + 2 + ... + n) = .
24
b) DemonstraŃi că n ⋅ (n + 1)(n + 2)(3n + 1)§24, pentru orice n ∈ N.
Etapa locală, Maramureş, 2009
2 ⋅1 2(1 + 2) 2(1 + 2 + 3 + ... + n )
9. CalculaŃi suma S = 4 + + ... + şi demonstraŃi rezulta-
1 + 2 ⋅13 + 12 2 4 + 2 ⋅ 23 + 2 2 n 4 + 2n 3 + n 2
tul obŃinut folosind metoda inducŃiei matematice.
Etapa locală, Alba, 2009, prof. C. Cipariu
10.
10. a) DemonstraŃi că (a + b) ≤ 2 (a + b ), (∀) n ∈ N, (∀) a, b ∈ R+.
n n–1 n n

b) DemonstraŃi că (a + b)n (an + bn) ≤ 2n(a2n + b2n), (∀) n ∈ N, a, b ∈ R+.


Etapa locală, Brăila, 2009, prof. E. Runceanu
n
n ( n + 1)(n + 5)
11.
11. DemonstraŃi inegalitatea
k =1
∑k +1
k ⋅ k! ≤
12
, (∀) n ≥ 1.

Etapa locală, NeamŃ, 2009, prof. A. Sandovici


12.
12. ArătaŃi că (9n2 – 24n + 11) ⋅ 4n + 16 ⋮ 27, (∀) n ∈ N.
Etapa locală, Sibiu, 2009, prof. V. Berghea
1 2 3  n   n   n + 1
13.
13. DemonstraŃi identitatea   +   +   + ... +   =   ⋅   , (∀) n ∈ N*, unde [a] repre-
2 2 2 2 2  2 
zintă partea întreagă a numărului real a.
Etapa locală, Sălaj, 2009
n
 n

14.
14. Să se demonstreze inegalitatea ∏
i =1

(1 + a i ) − 2 n −1  (a i − 1)  ≥ 2 n , unde ai ≥ 1, i = 2, n .
 i =1 
Etapa locală, Botoşani, 2009

7
1.3. Probleme de numărare. Alte tipuri de raŃionamente
1. Se consideră următorul tabel în care linia „n” conŃine „n” numere
1
3 5
7 9 11
....................... .
a) Să se determine câte linii ale tabelului au primul element un număr raŃional.
b) Să se calculeze partea întreagă a primului element din linia 2010.
Etapa locală, Tulcea, 2010
2. ÎmpărŃim un pătrat prin laturi paralele la laturi în n2 pătrate congruente.
a) DemonstraŃi că numărul pătratelor de diferite dimensiuni care apar în figura obŃinută este:
n (n + 1)(2n + 1)
6 .
b) Dacă ducem diagonalele în fiecare pătrat mic, câte triunghiuri dreptunghice isoscele de diferite
dimensiuni apar în figura formată?
Etapa locală, NeamŃ, 2010
3. Se consideră un pătrat de latură 1. Ducând 4 drepte paralele cu laturile sale îl împărŃim în 9 pătrate.
Pătratul din mijloc îl eliminăm. Cele 8 pătrate rămase le împărŃim în acelaşi mod în 9 pătrate
egale şi din nou eliminăm pătratul din mijloc. Repetând procedeul de 2010 ori, să se afle aria
pătratelor eliminate.
Etapa locală, Satu-Mare, 2010
4. Se consideră numărul a = (1 + 2 + ... + 6)(2 + 3 + ... + 7)(3 + 4 + ... + 8). Fie ak numărul din
paranteza k din scrierea lui a.
a) Dacă numărul a conŃine n paranteze, scrieŃi al n-lea factor.
b) Pentru k = 100, atunci arătaŃi că numărul a nu este pătrat perfect.
c) GăsiŃi paranteza care conŃine cel mai mic pătrat perfect din numărul a.
Etapa locală, BistriŃa-Năsăud, 2010
5. Fie A o mulŃime cu 1005 termeni în progresie aritmetică cu raŃia 2 şi B o mulŃime cu 1006 termeni
în progresie aritmetică cu raŃia3 . DeterminaŃi numărul minim de elemente ale mulŃimii A ∪ B.
Etapa locală, Giurgiu, 2010
6. O tribună a unui stadion se compune din 41 de rânduri de scaune şi pe fiecare rând se află cu 10
locuri mai multe decât pe rândul precedent. În ultimul rând sunt 500 de locuri. CâŃi spectatori pot
intra în acest stadion?
Etapa locală, Ilfov, 2009
7. Luni dimineaŃa la ora 8, două calculatoare sunt atacate, fiecare de câte un virus. La intervale de
10 minute, aceşti doi viruşi atacă, fiecare, câte trei calculatoare încă necontaminate. PrecizaŃi când
(ziua, ora şi minutul) vor fi contaminate exact 119800 calculatoare.
Etapa locală, Covasna, 2009

8
8. Punctele unui plan sunt colorate în roşu sau albastru. DemonstraŃi că există un dreptunghi având
vârfurile la fel colorate.
Etapa locală, Bihor, 2009
9. a) Câte progresii aritmetice de numere naturale cu primul termen 1 conŃine numărul 45001?
b) Câte progresii aritmetice neconstante de numere naturale au toŃi termenii pătrate perfecte?
Etapa locală, Botoşani, 2009
10. Pe o tablă scriem numerele naturale de la 1 la 4n – 1, cu n ∈ N*. Printr-o operaŃie înŃelegem că
ştergem două numere oarecare de pe tablă şi în locul lor scriem valoarea absolută a diferenŃei
celor două numere. Să se demonstreze că după 4n – 2 operaŃii pe tablă rămâne un singur număr
par.
Etapa locală, Mureş, 2009
11. Pe o tablă scriem numerele naturale de la 1 la 4n, unde n ∈ N* este număr impar. Printr-o
operaŃie înŃelegem că ştergem numerele a şi b şi în locul lor scriem numărul a – 3b sau numărul
b – 3a. Este posibil ca după câteva operaŃii toate numerele rămase pe tablă să fie 0?
Etapa locală, Covasna, 2009
12.
12. Fiecărui pătrat al unei table de şah îi corespund două numere. Primul număr ne indică numărul de
pe linie, iar al doilea număr ne indică numărul în coloană al pătratului respectiv. Aşezăm 8 turnuri
pe tabla de şah astfel încât două dintre ele să nu se taie. Numerele care corespund turnurilor le
înmulŃim şi adunăm cele 8 numere astfel obŃinute şi rezultă 120. Scoatem turnurile de pe tabla de
şah. Se pot determina pătratele pe care au fost aşezate aceste turnuri?
Etapa locală, Satu Mare, 2009
13.
13. La fabricarea unui ceas s-a greşit mecanismul său astfel că secundarul merge corect şi orarul sare
după fiecare a 12-a săritură a minutarului. Cadranul este împărŃit în 60 de părŃi egale. În timp de
90 de minute, de câte ori se suprapun secundarul cu minutarul? În trei zile, de câte ori se suprapun
cele trei ace la un ceas normal şi la ceasul nostru defect?
Etapa locală, Harghita, 2009
14.
14. Pe fiecare latură a triunghiului ABC luăm câte n puncte. Punctele situate pe laturile [AB] şi [AC]
le unim cu punctele de pe [BC]. Dacă dintre segmentele astfel obŃinute nu avem trei concurente,
atunci să se determine numărul punctelor de intersecŃie apărute.
Etapa locală, Cluj, 2009
15.
15. Pentru fiecare număr natural cel puŃin egal cu 4, arătaŃi că interiorul unui patrulater convex
oarecare se poate descompune ca reuniune de n plăci în formă de triunghiuri dreptunghice având
interioarele disjuncte.
Etapa locală, Iaşi, 2009, prof. Gabriel Popa

1.4. OperaŃii cu mulŃimi. Determinări de mulŃimi


1. Se consideră A = {x | ∃ a, b, c, d ∈ N* astfel încât x = a2 + b2 + c2 – d2}. ArătaŃi:
a) 1, 4 ∈ A;
b) A = N.
Etapa locală, Suceava, 2010

9
 2n + 1   2k 
2. Se consideră mulŃimile A = a n a n = , n ∈ N * şi B = b k b k = , k ∈ N * .
 2 n − 1   2 k − 1 
a) Să se arate că A şi B sunt mulŃimi disjuncte.
b) Să se arate că în A există trei elemente distincte în progresie aritmetică.
c) Să se arate că între bk+1 şi bk există două elemente ale mulŃimii A.
Etapa locală, Botoşani, 2010, G. Omniuc
 x +6
3

3. a) Să se determine mulŃimea M = x ∈ Z * ∈ Z .
 5x 
 4n + 1   3k + 1 
b) Să se arate că mulŃimile A =  x ∈ Q x = , n ∈ N  şi B =  y ∈ Q y = , k ∈ N  sunt
 4n − 3   3k − 2 
disjuncte.
Etapa locală, Covasna, 2010
 6k + 2
2

4. a) Să se determine elementele mulŃimii A =  x ∈ Z x = , k ∈ Z .
 2k + 5 
b) DeterminaŃi card B ştiind că avem card P(B) + card P(A) = 32.
Etapa locală, Olt, 2010, G. NeŃu
5. Fie mulŃimea M = {x + y | x, y ∈ Z}.
2 2

a) Să se demonstreze că 2011 ∉ M, 2048 ∈ M.


b) Să se demonstreze implicaŃia: (∀) p ∈ M, (∀) q ∈ M ⇒ p ⋅ q ∈ M.
c) Să se determine submulŃimile finite M' ale mulŃimii M, cu proprietatea (∀) p ∈ M', (∀) q ∈
∈ M' ⇒ p ⋅ q ∈ M'.
Etapa locală, GalaŃi, 2010, C. Ursu
6. Se consideră mulŃimea A = {n ∈ N | n = p [ n ] + 1, p ∈ N}.
a) DeterminaŃi mulŃimea pentru p = 2.
b) ArătaŃi că (∀) p ≥ 3, mulŃimea A conŃine exact două elemente.
Etapa locală, Braşov, 2009, prof. Traian D.
 x − 3x + 2
3 
7. Să se determine mulŃimea A = x ∈ Z ∈ Z .
 2x + 1 
Etapa locală, Tulcea, 2009
8. Fie M = {m ∈ Z | x2 – mx + m + 6 = 0 are cel puŃin o soluŃie în Z} şi
  4x + 1  2x − 1  5x − 4 
N = x ∈ R 

+
 6   3 
 = 3  . Dacă S1 =
 m∈M

| m | şi S2 =
x∈ N

x , arătaŃi că S1 = S2.

Etapa locală, Ilfov, 2009


9. Fie A ⊂ R o mulŃime cu cel puŃin trei elemente, având proprietatea că, pentru orice două elemente
distincte x, y ∈ A, rezultă că x + y ∈ Q. Să se demonstreze că A ⊂ Q.
Etapa locală, IalomiŃa, 2009
10. Fie A mulŃimea funcŃiilor f : N → N, astfel încât f ≠ 1N, pentru orice k = 1, 2009 , iar f[2010] ≠ 1N.
10. [k]

a) Să se arate că A este nevidă.


10
b) Să se stabilească dacă A este infinită.
c) DemonstraŃi că toate elementele lui A sunt funcŃii bijective.
(S-a notat cu 1A funcŃia identitate şi cu f[k] compunerea lui f cu ea însăşi de k ori).
Etapa locală, Prahova, 2009
1 1 1 19
11.
11. MulŃimile finite A şi B satisfac egalitatea + + = , unde
card(A ∩ B) card(A ∪ B) card P(A ∪ B) 24
P (M) este mulŃimea tuturor submulŃimilor lui M. Să se justifice egalitatea A = B.
Etapa locală, GalaŃi, 2009
12. Se consideră mulŃimile A = {x ∈ N | x ≤ 2009, x se divide cu 11, dar nu se divide cu 13} şi B =
= {x ∈ N | x ≤ 2009, x se divide cu 13 dar nu se divide cu 11}. Care mulŃime are mai multe
elemente?
Etapa locală, Gorj, 2009
13.
13. Se dau mulŃimile A = {x ∈ R | x – ax + b = 0} şi B = {x ∈ R | x – bx + c = 0}, unde a, b, c ∈ R.
2 2

a) DeterminaŃi mulŃimile A şi B pentru a = – 4, b = c = 2.


b) DeterminaŃi relaŃia dintre numerele a şi c, ştiind că 1 ∈ A, 2 ∈ B.
c) DeterminaŃi valorile numerelor a, b, c, ştiind că A ∪ B = {1, a, b, c}.
Etapa locală, Harghita, 2009
 n +2
2 
14. Considerăm mulŃimea A =  x n x n = 2 , n = 1, 2,3,..., 2008 .
 n −n+2 
a) Să se calculeze A ∩ N.
b) Să se determine numărul elementelor mulŃimii A.
c) Să se determine cel mai mare element al mulŃimii A.
Etapa locală, Covasna, 2009
15.
15. Fie a, b, c ∈ R* distincte două câte două şi mulŃimile A = {x ∈ R | ax2 + bx + c = 0}, B = {x ∈
∈ R | bx2 + cx + a = 0}, C = {x ∈ R | cx2 + ax + b = 0}. Să se arate că dacă A ∩ B ∩ C ≠ ∅,
atunci A ∪ B ∪ C are exact patru elemente.
Etapa locală, Ilfov, 2009

II. Numere reale

2.1. MulŃimea numerelor întregi


1
1. Se consideră x ∈ R* cu proprietatea că x + ∈ Z. DemonstraŃi că:
x
1
a) x n + ∈ Z, (∀) n ∈ N*;
xn
1
b) x 2010 + 2010
este un număr par.
x
Etapa locală, Sibiu, 2010

11
[nx](n + 1)
2. Fie n ≥ 2 un număr natural. Să se demonstreze că [x] + [2x] + ... + [nx] = , (∀) n ∈ N –
2
– {0, 1} dacă şi numai dacă x ∈ Z.

Etapa locală, Hunedoara, 2010


3. Fie ecuaŃiile 2x + 3 + 2 x − 5 = y şi z + z + 11 = t , unde x, y, z, t sunt numere naturale şi
x > 2. ArătaŃi că x2 + y2 ≤ z2.
Etapa locală, Bihor, 2010
4. Dacă a, b ∈ R, n ∈ N* şi dacă ecuaŃia x2 + ax + b + n = 0 are rădăcini întregi, atunci na2 + b2 este
număr natural compus.
Etapa locală, Olt, 2010, Dorin Mărghidanu, Corabia
5. GăsiŃi toate tripletele de numere prime a, b, c, care satisfac inegalitatea: abc < ab + ac + bc.
Etapa locală, Dolj, 2010
6. Să se arate că numărul N = [ n + 1] + [ n + 2 ] + ... + [ n + n ] este pătrat perfect.
2 2 2

Etapa locală, Ilfov, 2009


1 1 1
7. RezolvaŃi în N* ecuaŃia + = .
x y 41
Etapa locală, Hunedoara, 2009
8. Să se arate că (∀) n ∈ N, numărul A = 111 1 − 222
... ...
2 este pătrat perfect.
2 n cifre n cifre

Etapa locală, Covasna, 2009


9. Fie Sn = 5 + 7 , n ∈ N. ArătaŃi că:
n n

a) Sn+2 = 12Sn+1 – 35Sn, (∀) n ∈ N; b) Sn | Sn+1 ⇔ n = 0.


Etapa locală, Sibiu, 2009
10.
10. Fie a1, a2, ..., an ∈ N* numere naturale distincte care au ca divizori primi doar numerele 2 şi 3.
a) Să se indice 100 astfel de numere.
1 1 1
b) Să se arate că + + ... + < 3.
a1 a 2 an
Etapa locală, Botoşani, 2009
11. ArătaŃi că pentru orice număr natural n, există un număr natural m astfel încât m + m + n ∈ N.
Etapa locală, Dolj, 2009
12.
12. Fie n ∈ N, n ≥ 2 şi numerele naturale nenule a1, a2, ..., an. Să se arate că toate ecuaŃiile x2 – (ak +
+ 1)x + ak+1 = 0, k = 0, 1, 2, ..., n cu an+1 = a1, au rădăcinile în Z dacă şi numai dacă a1 = a2 = ... = an.
Etapa locală, BistriŃa-Năsăud, 2009
13.
13. Fie mulŃimea A = {1, 2, 3, ..., 98}. ArătaŃi că oricum am alege 50 de elemente ale lui A, există
printre ele două având suma cub perfect.
Etapa locală, Iaşi, 2009, prof. Titu Zvonaru
14.
14. RezolvaŃi în mulŃimea numerelor întregi ecuaŃia x6 + x5 + 4 = y2.
Etapa locală, ConstanŃa, 2009, prof. I. Cucurezeanu

12
15. Fie ecuaŃiile de gradul 2, x2 – ax + b = 0 şi y2 – by + a = 0.
a) Să se arate că există o infinitate de perechi (a, b) de numere întregi pentru care ambele ecuaŃii
au soluŃiile întregi.
b) Să se determine toate perechile (a, b) de numere naturale pentru care ambele ecuaŃii au soluŃii
naturale.
Etapa locală, Mureş, 2009

2.2. Numere raŃionale, numere iraŃionale


1. Fie ecuaŃia de gradul al doilea în necunoscuta x: 2nx2 – 2(n2 + 1)x – n2 – 1 = 0, n ∈ N*, arbitrar.
Să se arate că rădăcinile ecuaŃiei sunt reale, distincte şi iraŃionale.
Etapa locală, BistriŃa, 2010
2. ArătaŃi că există o infinitate de numere raŃionale a ∈ Q + *
pentru care ecuaŃia x = a − a + x să
aibă o rădăcină raŃională.
3. a) DemonstraŃi că există o infinitate de numere raŃionale x astfel încât x + 2009 şi x + 2010
să fie raŃionale.
b) DemonstraŃi că există o infinitate de numere raŃionale x astfel încât x + 2009 şi x + 2010 să
fie iraŃionale.
Etapa locală, IalomiŃa, 2010
2 x + 1 3x − 1
2 2
4. Să se determine numerele reale x pentru care numerele: şi sunt simultan raŃio-
x −3 3x − 5
nale.
Etapa locală, Călăraşi, 2010
1 1 1
5. AflaŃi prima zecimală a numărului a n = + + ... + , n ∈ N*.
n +1 n + 2 2n
Etapa locală, Bucureşti, 2010, Gazeta Matematică

6. Să se demonstreze că dacă p, q, r ∈ Q şi pq + qr + pr = 1, atunci (1 + p 2 )(1 + q 2 )(1 + r 2 ) ∈ Q.


Etapa locală, Tulcea, 2009
1 
7. Fie a ∈ R*+ cu proprietatea {a} +   = 1 . ArătaŃi că a ∉ Q. DaŃi exemplu de astfel de număr.
a 
Etapa locală, Arad, 2009
a 2 + 20082
8. DemonstraŃi că ∈ R / Q, (∀) a ∈ Z.
3
Etapa locală, Brăila, 2009, prof. A. Negulescu
9. Se consideră numărul natural a şi expresia E(a ) = { a } + { a }2 + { a }3 , unde {x} reprezintă par-
tea fracŃionară a numărului x. Să se demonstreze că E(a) este număr raŃional dacă şi numai dacă a
este pătrat perfect.
Etapa locală, Maramureş, 2009, prof. F. Bojor

13
ab
10.
10. Dacă a, b ∈ Q* şi c = , demonstraŃi că:
a+b
a 2 + b 2 + c 2 ∈ Q.
Etapa locală, Satu Mare, 2009
11. Se consideră a, b, c, d ∈ Q, m, n ∈ N*. Să se demonstreze că dacă (a + b 2 ) m = (c + d 3 ) n ,
atunci ab = cd = 0.
Etapa locală, Timiş, 2009
12. Fie a, b, c ∈ Q*. Să se demonstreze că:
3
1) dacă a 2 ∈ Q, atunci 3
a ∈ Q;
2) dacă 3
a ⋅ b ∈ Q, atunci
5 3
a şi 5
b ∈ Q;
3) dacă 3
a ⋅ b ⋅ c ∈ Q, atunci
5 7 3
a, 5
b, 7
c ∈ Q.
Etapa locală, Prahova, 2009
13. Pentru k ∈ N, k ≥ 2 şi a0 ∈ N fixate, se defineşte şirul (an)n≥1 prin recurenŃă: a n +1 = n 1 + a kn ,
(∀) n ∈ N. Să se arate că şirul (an)n≥0 conŃine o infinitate de numere raŃionale şi o infinitate de
numere iraŃionale.
Etapa locală, Mureş, 2009
14. Să se arate că n 2 − 3n + 2 + n 2 + n + 1 este iraŃional pentru orice n ∈ Z.
Etapa locală, Suceava, 2009
15. a) Să se arate că orice număr real nenul se poate scrie ca un produs de două iraŃionale diferite.
b) Să se arate că orice număr real se poate scrie ca suma a 2009 numere iraŃionale diferite.
Etapa locală, Vâlcea, 2009

2.3. OperaŃii cu numere reale


1. Fie a, b, c ∈ R* trei numere reale distincte două câte două, astfel încât:
a 2 b2 c2 a 2 b2 c2
+ + = + + . Să se arate că a3 + b3 + c3 = 3abc.
c a b b c a
Etapa locală, Suceava, 2010, C. AmorăriŃei
4 4 4
2. Fie numerele reale pozitive a1, a2, ..., a20 astfel încât a1 + a 2 = , a 2 + a 3 = , ..., a19 + a20 = şi
3 5 37
a + a2 a2 + a3 a + a 20
A= 1 + + ... + 19 . DemonstraŃi că A ≥ 19 .
a 1a 2 a 2a 3 a 19 a 20
Etapa locală, Prahova, 2010, N. Radu
3. Fie n ≥ 1 un număr natural. Să se determine numerele reale x1, x2, ..., xn, ştiind că:
n (n + 1)(2n + 1)
x12 + x 22 + ... + x 2n − 2( x1 + 2 x 2 + ... + nx n ) + ≤ 0.
6
Etapa locală, Hunedoara, 2010, Gazeta Matematică, 2/2009

14
4. a) Să se demonstreze că dacă n ∈ N, n ≥ 2 şi a < b < c < d sunt numere reale pozitive cu a + d = b +
+ c, atunci n
a +n d <n b +n c .
3+n 5+n 7
n n
6+n 7 +n 8
b) ComparaŃi numerele A = şi B = , unde n ∈ N, n ≥ 2.
n
4+n 5+n 6 n
5+n 7 +n 9
Etapa locală, Cluj, 2010, I. Diaconu
5. DeterminaŃi numerele reale pozitive x, y, z, t ştiind că x + y + z + t = 8 2 , iar xyzt = 16 + xy +
+ xz + xt + yz + yt + zt.
Etapa locală, Iaşi, 2010
6. Fie a, b, c, d ∈ (0, ∞) astfel încât este verificată egalitatea:
a 2 + b 2 ⋅ c 2 + d 2 + b 2 + c 2 ⋅ a 2 + d 2 = (a + c)(b + d) .
i) Să se arate că ac = bd.
ii) InterpretaŃi geometric rezultatul obŃinut.
Etapa locală, Braşov, 2009, Gazeta Matematică, 2/2008
1 1 1
7. Să se arate că + + >1.
2+2 2 4+2 6 6 + 2 12
Etapa locală, NeamŃ, 2009, prof. Gh. Ciorăscu, prof. A. Sandovici
8. Dacă a, b, c ∈ (0, ∞) şi a + b + c = 1, arătaŃi că:
(a + bc)(b + ac)(c + ab) = (a + b)2(b + c)2(c + a)2.
Etapa locală, Covasna, 2009
9. DemonstraŃi că pentru orice număr natural n are loc inegalitatea:
n +1 n+2 n+3 2n + 2 1 1 1 1
+ + + ... + < + + + ... + .
n+2 n+3 n+4 2n + 3 n+3 n+4 n+5 2n + 4
Etapa locală, Teleorman, 2009
1 ⋅ 3 ⋅ 5 ⋅ 7 ⋅ ... ⋅ 4015 ⋅ 4017
10.
10. Fie P = . Să se calculeze cu o aproximaŃie de o zecime P.
22010 ⋅ (2009!)
Etapa locală, Gorj, 2009
a b c
11. Fie numerele reale a, b, c, diferite între ele. Dacă + + = 0 , atunci:
b−c c−a a −b
a b c
a) + + =0;
( b − c) 2
(c − a ) 2
(a − b ) 2
a b c a+b+c
b) + + = .
( b − c) 3
(c − a ) 3
(a − b) 3
(a − b)(b − c)(c − a )
Etapa locală, DâmboviŃa, 2009, prof. C. Borduş
12.
12. Fie numerele reale a, b, c, d astfel încât a + b + c + d = 6 şi a2 + b2 + c2 + d2 = 12. DemonstraŃi că
a, b, c, d ∈ [0, 3].
Etapa locală, Tulcea, 2009

15
y2 y2
13.
13. Fie x, y, z ∈ R*+, care satisfac relaŃiile x 2 + xy + = 25, + z 2 = 9 , z 2 + xz + x 2 = 16 . Să se
3 3
calculeze valoarea expresiei xy + 2yz + 3xz.
Etapa locală, Harghita, 2009
14. Fiind date numerele reale a, b, c, d, e, f, care verifică condiŃiile:
a + b + c + d + e + f = 10
 . DeterminaŃi valoarea maximă a lui f.
(a − 1) + (b − 1) + (c − 1) + (d − 1) + (e − 1) + (f − 1) = 6
2 2 2 2 2 2

Etapa locală, Bacău, 2009


x1 x2 xn
15. Dacă x1, x2, ..., xn ∈ R – {1} şi E = + + ... + , să se
1 − x1 (1 − x1 )(1 − x 2 ) (1 − x1 )(1 − x 2 )...(1 − x n )
arate că (1 + E) ⋅ (1 – x1)(1 – x2) ... (1 – xn) = 1.
Etapa locală, Satu Mare, 2009

2.4. Numere reale. Modulul unui număr real


1. Să se rezolve în R ecuaŃia [x] ⋅ {x} = x ⋅ |x|, unde [a] reprezintă partea întreagă a numărului real şi
{a} reprezintă partea fracŃionară a numărului real a, iar |a| modulul său.
Etapa locală, ConstanŃa, 2010, Gh. Andrei
2. DeterminaŃi a ∈ N* şi b, c, d ∈ R astfel încât |ax3 + bx2 + cx + d| ≤ 2, oricare ar fi x ∈ [–2, 2].
Etapa locală, Satu-Mare, 2010
3. ArătaŃi că dacă a ∈ R şi b ∈ R nu sunt simultan nule, atunci are loc inegalitatea:
|a+b|+|a−b|
2≤ ≤ 2.
a 2 + b2
Etapa locală, Călăraşi, 2010
4. Se consideră expresia E(x) = |x – 1| + |x – 2| + ... + |x – 2008|, x ∈ R. DeterminaŃi min E( x ) , pre-
x ∈R
cum şi pentru ce valori ale lui x se realizează.
Etapa locală, Arad, 2009
1
5. ArătaŃi că dacă xk ∈ R, k = 1, 2, ..., n, atunci max{x12 − x 2 , x 22 − x 3 , ..., x 2n − x1} ≥ − .
4
Etapa locală, Braşov, 2009, prof. M. Bencze
6. Fie a şi b două numere reale. Să se demonstreze inegalitatea | 1 + ab | + | a + b |≥ | a 2 − 1 | ⋅ | b 2 − 1 | .
Etapa locală, MehedinŃi, 2009, prof. D. Nedeianu
7. Dacă a, b ∈ (1, ∞) şi m, n ∈ N* – {1}, atunci să se compare numerele A = m n a + n b şi B =
=n m
a +m b .
Etapa locală, Dolj, 2009, prof. M. Dicu

16
8. a) DeterminaŃi cel mai mic număr natural nenul α astfel încât pentru orice triunghi ABC, lungi-
a b c b c a
mile laturilor sale a, b, c să satisfacă inegalitatea E = + + − − − <α.
b c a a b c
b) Există un triunghi ABC în care expresia E este mai mare decât 0,81?
Etapa locală, GalaŃi, 2009, prof. C. Ursu
9. RezolvaŃi în R ecuaŃia |1 – x| + x|x| + |1 + x| = 3.
Etapa locală, Tulcea, 2009
10. Fie a, b ∈ R şi x ∈ R care verifică inegalităŃile: |a + x + b| ≤ 1, |4a + 2x + b| ≤ 1 şi |9a + 6x + 4b| ≤
≤ 1. Să se arate că |x| ≤ 15.
Etapa locală, Bihor, 2009
9
11.
11. ArătaŃi că min(2a + b – c2, 2b + c – a2, 2c + a – b2) ≤ , unde a, b, c ∈ R.
4
Etapa locală, Cluj, 2009
12.
12. Fiind date numerele a, b, c > 0, să se arate că există x, y, z ∈ R, astfel încât:
|x + a + b| + |y + b + c| + |z + c + a| + |x – a – b| + |y – b – c| + |z – c – a| = 3( ab + bc + ac ) .
Etapa locală, Botoşani, 2009

2.5. Parte întreagă. Parte fracŃionară


1. AflaŃi valorile lui n ∈ N pentru care [ 4n 2 + 1] + [ 4n 2 + 2 ] + ... + [ 4n 2 + 12n ] = 117 .
Etapa locală, Alba, 2010, prof. Cosmin Păcurar
2. RezolvaŃi în R ecuaŃia: 25{x}2 + 1 = 10x.
Etapa locală, Sibiu, 2010, prof. A. Peşca
3. RezolvaŃi în R ecuaŃia: [x2 – 4x + 4] = [–2x2 + 8x – 6].
Etapa locală, Prahova, 2010, prof. P. Năchilă
 1
4. a) Să se arate că [ x ] +  x +  = [2 x ] , oricare ar fi x ∈ R.
 2
 π 2010 + 1   π 2010 + 2   π 2010 + 2 n −1 
+  + ... +   = [π ] , oricare ar fi n ≥ 4020,
2010
b) DemonstraŃi că:  n
 2   4   2 
n ∈ N.
Etapa locală, Buzău, 2010
 n + n   n + 3n + 2   n + 5n + 6   n + 7n + 12 
2 2 2 2
5. DemonstraŃi că (∀) n ∈ N,  + + +  = ( n + 2) .
2

 4   4   4   4 
Etapa locală, DâmboviŃa, 2010
6. RezolvaŃi ecuaŃia {x} – {2009x}= x în mulŃimea numerelor reale.
Etapa locală, NeamŃ, 2009, prof. A. Sandovici, prof. Gh. Ciorăscu

17
a[a ] + c{c} − [b] ⋅ {b} = 0,16

7. Să se rezolve sistemul 4b[b] + 4a{a} − 4[c] ⋅ {c} = 1 , unde [x], {x} reprezintă partea întreagă,
c[c] + b{b} − [a ] ⋅ {a} = 0,49

respectiv partea fracŃionară a numărului real x.
Etapa locală, Prahova, 2009, prof. G. Necula, Plopeni
x − 2 x  2x − 1 
8. Să se rezolve ecuaŃia   − ( x − 2) = −  .
 2  2  4 
Etapa locală, Vaslui, 2009, prof. D. Teclici
9. Pentru fiecare n ∈ N* considerăm suma Sn = 1 + 2 + 3 + ... + n .
a) CalculaŃi partea întreagă a numerelor S1, S2, S3, S4.
b) Să se arate că [Sn] = n ⇔ n ∈ {1, 2}.
c) Să se rezolve ecuaŃia [x + S2009–x] = 2009, x ∈ N.
Etapa locală, Maramureş, 2009
10. a) DemonstraŃi că: [xy] ≥ [x] ⋅ [y], (∀) x, y ≥ 0.
n
1
b) Fie S(n ) = ∑[ k 4 + 4 k 3 + 5k 2 + 4 k + 1 ]
k =1
, n ∈ N*, unde [k] reprezintă partea întreagă a lui k.

DemonstraŃi că S(n) < 1, (∀) n ∈ N*.


11.. Să se rezolve ecuaŃia [–2x] + [–x] + [x] + [2x] + 1 = 0.
11
Etapa locală, Timiş, 2009, prof. A. Eckstein
12. Să se determine partea întreagă, respectiv prima zecimală a numărului n 2 + n , n ∈ N*.
Etapa locală, Covasna, 2009
x
13.
13. a) Să se arate că nu există numere reale x, pentru care   = x 2 + 1 .
2
y
b) Să se determine numerele reale y, pentru care   = y 2 .
2
Etapa locală, Caraş-Severin, 2009, prof. L. Dragomir
14.
14. Să se arate că pentru orice număr natural n ≥ 2 avem identitatea:
[ 1] + [ 2 ] + ... + [ n 2 − 1] + 12 + 22 + ... + n 2 = n 3 , unde [a] reprezintă partea întreagă a numărului a.
Etapa locală, Dolj, 2009
15.
15. Să se rezolve ecuaŃia [x + k] ⋅ {x + k} = xk, unde k ∈ Z.
Etapa locală, Iaşi, 2009
16.
16. Să se determine numerele reale x, care verifică relaŃia |[x – a]| + [|x – a|] = 1, unde a este un para-
metru real, iar [t] reprezintă partea întreagă a numărului real t.
Etapa locală, Teleorman, 2009

18
2.6. InegalităŃi clasice remarcabile
n! 1
1. Să se demonstreze inegalitatea: ≤ n , (∀) n ∈ N*, unde n! = 1 ⋅ 2 ⋅ ... ⋅ n.
(1 + 1 )(1 + 2 )...(1 + n ) 2
2 2 2

Etapa locală, Alba, 2010, Gazeta Matematică, 11/2009


a b c 3
2. DemonstraŃi inegalitatea: + + ≥ , (∀) a, b, c ∈ R*.
b+c a+c a+b 2
Etapa locală, Sibiu, 2010, prof. A. Pop
3. Se consideră n ∈ N* şi numerele naturale a1, a2, ..., an astfel încât { a1, a2, ..., an} = {1, 2, ..., n}.
n (n + 1)(2n + 1)
a) ArătaŃi că a1 + 2a 2 + 3a 3 + ... + na n ≤ .
6
a2 a2 a 2 n (n + 1)
b) ArătaŃi că: 1 + 2 + ... + n ≥ .
1 2 n 2
Etapa locală, Buzău, 2010
1 11 1
4. a) ArătaŃi că: ≤  +  , (∀) x, y > 0.
x + y 4  x y 
ab bc ca 1
b) DemonstraŃi că + + ≤ (a + b + c) , (∀) a, b, c > 0.
a + b + 2c b + c + 2a a + c + 2 b 4
Etapa locală, Vaslui, 2010, prof. V. Huiban
5. DemonstraŃi că pentru orice număr natural nenul n are loc inegalitatea:
n +1 n+2 2n + 2 1 1 1
+ + ... + < + ... + .
n+2 n+3 2n + 3 n+3 n+4 2n + 4
Etapa locală, Teleorman, 2010
6. Fie a, b > 0 astfel încât (ab)2009 + 1 = 2b2009.
a) ArătaŃi că a ≤ b.
 1 
b) ArătaŃi că 1 + n (1 + b ) ≥ 4 , pentru orice n ∈ N*.
n

 a 
Etapa locală, NeamŃ, 2009, prof. A. Sandovici
7. Să se demonstreze inegalităŃile:
 4 4  9 x 1 
2
9 16 y 2
a)  2 + + 4 z 
 + + 4  ≥ 9 , unde x, y, z > 0. Când avem egalitate?
2 
 9 x 16 y  4 9 4z 
b) 24x – 36x + 17 ≥ 0, unde x > 0;
3 2

c) 24x3 – 36x + 17 ≥ 0, unde x > 0.


Etapa locală, Bihor, 2009, prof. Viorica Pleşa
8. DemonstraŃi că dacă produsul a două numere pozitive este constant, atunci suma lor este minimă
când numerele sunt egale.
Etapa locală, Prahova, 2009

19
π
9. DeterminaŃi x, y, z ∈ R –  + kπ, k ∈ Z pentru care 2(tg x + tg y + tg z) – (tg2 x + tg2 y + tg2 z) ≥ 3.
2 
Etapa locală, Hunedoara, 2009
10. DemonstraŃi că dacă a, b, c, x, y, z sunt numere reale strict pozitive, atunci:
x y z ( x + y + z) 2 a + 2 b b + 2c c + 2a
a) + + ≥ ; +
b) + ≥3.
a b c ax + by + cz a + 2c b + 2a c + 2 b
Etapa locală, Cluj, 2009, prof. Ilie Diaconu
11. DemonstraŃi că (∀) a, b, c ∈ (0, ∞) au loc:
a+b a+c b+c
a) (a + b)(a + c)(b + c) ≥ 8abc ; b) + + ≥ 6.
c b a
Etapa locală, Alba, 2009
12.
12. Să se demonstreze inegalitatea:
x12 + x 22 + x 32 + x 22 + x 32 + x 24 + ... + x 2n −1 + x 2n + x12 + x 2n + x12 + x 22 ≥ 3( x1 + x 2 + ... + x n ) , (∀) n ∈
∈ N, n ≥ 3.
Etapa locală, Olt, 2009, prof. T. Vîrban
13.
13. Să se arate că pentru orice numere reale pozitive a, b, c are loc inegalitatea:
a+b b+c c+a 1 1 1
+ 2 2+ 2 ≤ + + .
a +b
2 2
b +c c +a 2
a b c
Etapa locală, Iaşi, 2009
14. Să se demonstreze inegalitatea lui Minkowski:
(a 2 + b1 ) 2 + (a 2 + b 2 ) 2 + ... + (a1 + b n ) 2 ≤ a12 + a 22 + ... + a 2n + b12 + b 22 + ... + b 2n , (∀) n ∈ N*.
Etapa locală, Bacău, 2009
1 1 1 1
15. Dacă a, b, c ∈ R*+, atunci 3 3 + + ≤ .
a + b + abc b 3 + c 3 + abc c3 + a 3 + abc abc
Etapa locală, BistriŃa-Năsăud, 2009

2.7. InegalităŃi condiŃionate


 a b c 
1. ArătaŃi că  + +  + ab + ac + bc ≥ a + b + c + 6 , pentru orice a, b, c ∈ (0, 1).
2 2 2

 b + c a + c a + b 
Etapa locală, Sibiu, 2010, prof. A. Pop
2. Se consideră numerele reale strict pozitive x, y, z cu proprietatea că x + y + z = 1.
a) Să se arate că x2y2 + x2z2 + y2z2 ≥ xyz.
1 1 1 3
b) Să se arate că + + ≥ .
x y z xyz
Etapa locală, Hunedoara, 2010

20
3. Fie a, b, c ∈ (0, ∞) şi a + b + c = 2010. Să se arate că:
ab + ac + ab + bc + ac + cb ≤ 3015 .
Etapa locală, Mureş, 2010
4. Fie a, b ∈ R cu a + b = 1 şi x > 0. Să se arate că:
2 2

 a b  a b  1
 +  +  < 1 + .
 x x + 1  x + 1 x x
Etapa locală, ConstanŃa, 2010, prof. G. Constantinescu
5. Fie a, b, c ∈ (0, 1) astfel încât a + b + c = 1. ArătaŃi că a2 + b2 + c2 – 6abc ≥ a3 + b3 + c3.
Etapa locală, Sălaj, 2010
6. Dacă a, b, c ∈ (0, ∞) şi a + b + c = 3, să se arate că:
a3 b3 c3
+ + ≥1.
a + 2 b b + 2c c + 2a
Când are loc egalitatea?
Etapa locală, Braşov, 2009, prof. Gabriela Boeriu
7. Fie numerele x, y, z ∈ R+ cu x + y + z = 1. Să se demonstreze că au loc inegalităŃile:
8
0 ≤ xy + yz + zx – xyz ≤ .
27
Etapa locală, Vaslui, 2009, prof. D. Mihalache

8. Dacă a, b, c sunt numere reale, a căror sumă este 0 sau 1, să se demonstreze inegalitatea:
3
ab + bc + ca < .
2
Etapa locală, Timiş, 2009
9. Dacă a, b, c ∈ R astfel încât a + b + c = 1, să se arate că sunt adevărate inegalităŃile:
2 2 2

a) |a + b + c| ≤ 3;
b) 1 − a 2 + 1 − b 2 + 1 − c 2 ≥ 2 a + b + c .
Etapa locală, Covasna, 2009
10. Să se demonstreze că dacă x, y, z ∈ (0, ∞) şi x ⋅ y ⋅ z = 1, atunci:
1 1 1 3 2
+ + < .
x+y+ z y+z+ x z+x+ y 4
Etapa locală, Caraş-Severin, 2009, prof. L. Dragomir
11.
11. Fie a, b, c, d ∈ (0, ∞) cu abcd = 1. DemonstraŃi că:
1 1 1 1 16
+ + + ≥ .
a 2 (b + c + d ) b 2 (c + d + a ) c 2 (d + a + b) d 2 (a + b + c) 3(a + b + c + d )
Etapa locală, DâmboviŃa, 2009, prof. C. Burdujel

21
5
12.
12. Dacă a, b, c sunt numere reale pozitive, astfel încât a2 + b2 + c2 = , să se arate că:
3
1 1 1 1
i) + − < ; ii) a – 2b + c ≤ 10 .
a b c abc
Etapa locală, Arad, 2009
1 1 1 3
13.
13. Dacă a, b, c > 0 şi abc = 1, arătaŃi că E = 2 + 2 + 2 ≥ .
a ( b + c) b (a + c) c ( b + a ) 2
Etapa locală, Bihor, 2009, prof. D. Drâmbe
2a a
14. a) ArătaŃi că ≤ , (∀) a, b ∈ (0, ∞).
ab + 1 b
b) Dacă a, b, c > 0, astfel încât abc = 1, să se demonstreze inegalitatea:
2a 2b 2c
+ + ≤a c +b a +c b .
ab + 1 bc + 1 ab + 1
c) Fie a, b, c > 0, astfel încât a + b + c = 1. Să se demonstreze inegalitatea:
a3 + b3 + c3 + 6abc ≤ a2 + b2 + c2.
Etapa locală, Călăraşi, 2009, prof. D. Marinache
15.
15. Fie a, b, c ∈ (–1, ∞) astfel încât să avem ab + bc + ca + 2abc = 1. Să se arate că:
1 1 1
+ + ≤1.
2+a +b 2+b+c 2+c+a
Etapa locală, ConstanŃa, 2009, prof. T. Lupu

III. FuncŃii

3.1. Şiruri de numere reale. Şiruri particulare


1. Şirul de numere reale (xn)n≥1 verifică x1 = 2, xn–1 = 1 + x n − x n −1 , n ≥ 2. Să se arate că:
a) xn = 1 + x1x2x3...xn–1, (∀) n ≥ 2;
1 1 1 1 1
b) + + + ... + + = 1 , (∀) n ≥ 2;
x1 x 2 x 3 x n −1 x n − 1
1 1 1 1
c) + + + ... + < 1 , (∀) n ≥ 2.
x1 x 2 x 3 xn
Etapa locală, Bihor, 2010, Gazeta Matematică, 5/2009
1 n +1  1
2. Fie şirul (xn)n≥1 definit prin x1 = şi x n +1 = ⋅  x n +  , (∀) n ≥ 1. AflaŃi x2010.
6 n+3  2
Etapa locală, Vâlcea, 2010, prof. M. Cucoaneş
3. Se consideră numerele x1 = 1, x2 = 1 şi x n +1 = nx n + x n −1 , (∀) n ≥ 2, n ∈ N.
a) Să se calculeze x3 şi x4.

22
b) Să se demonstreze că xn < n – 1, (∀) n ≥ 3, n ∈ N.
c) Să se demonstreze că xn ∉ N, (∀) n ≥ 3, n ∈ N.
Etapa locală, Braşov, 2010, prof. Romeo Ilie
n
4. a) DemonstraŃi că pentru orice n ∈ N*, prin împărŃirea cu rest a numărului 32 la 2n se obŃine un
cât par.
 3n 
b) DemonstraŃi că şirul (an)n≥1 dat de a n =   , (∀) n ∈ N*, are o infinitate de termeni pari, o
n
infinitate de termeni divizibili cu 3 şi o infinitate de termeni divizibili cu 5. (cu [ξ] s-a notat partea
întreagă a numărului real ξ).
***
5. DeterminaŃi şirul (xn)n∈N* cu termeni strict pozitivi, astfel încât
x13 + x 32 + x 33 + ... + x 3n = ( x1 + x 2 + ... + x n ) 2 , (∀) n ∈ N*.
Etapa locală, Sibiu, 2009
a (n + 2)
6. Se dă şirul (an)n≥1 cu a1 = 2 şi a1 + a 2 + ... + a n = n , (∀) n ∈ N*.
3
n
a
Să se arate că: ∑ k =1
[ a k ] = n , unde [a] reprezintă partea întreagă a numărului a.
2
Etapa locală, Olt, 2009, prof. I. Guşatu
7. Se dă şirul (an)n≥1 cu an > 0, n ∈ N*, pentru care a n < a n − a n +1 , (∀) n ∈ N*.
2

Să se arate că: [n ⋅ an] = 0, (∀) n ∈ N*.


Etapa locală, GalaŃi, 2009, prof. Iuliana Duma
a (a + 1)
8. Fie şirul (an)n∈N, an ∈ N*, n ∈ N*. Dacă termenii săi respectă condiŃia a1 + a 2 + ... + a n = n n ,
2
oricare ar fi n ∈ N*, să se determine termenii săi ak.
Etapa locală, BistriŃa-Năsăud, 2009
9. Se notează xn = 1 + 3n ⋅ (2n – 1). Să se arate că:
a) pentru orice n ∈ N, există yn ∈ N astfel încât xn+1 – 3 ⋅ xn = yn;
b) pentru orice n ∈ N, xn este divizibil cu 4.
Etapa locală, Caraş-Severin, 2009, prof. I. Didraga
m
10.
10. Fie (an)n≥1 un şir de numere reale cu proprietatea că a m − a n ≤ , (∀) m, n ∈ N*. DemonstraŃi că
n
şirul este constant.
Etapa locală, DâmboviŃa, 2009
(n + 1) a n
11.
11. Fie şirul cu termeni pozitivi (an)n≥1 cu a1 = 1, astfel încât a 1 + a 2 + ... + a n = ,
2
(∀) n ∈ N*.
a) Să se determine a2 şi a3.
b) Să se arate că an este număr natural pătrat perfect, (∀) n ∈ N*.
Etapa locală, Gorj, 2009

23
12.
12. Fie (xn)n≥1 un şir de numere reale cu proprietatea că xn+1 = x n − x 2n + x 3n − x 4n , (∀) n ∈ N şi x0 ∈
∈ (0, 1). Să se studieze mărginirea şi monotonia şirului (xn)n≥0.
Etapa locală, BistriŃa-Năsăud, 2009
13.
13. Să se determine şirurile de numere reale strict pozitive (an)n≥1 cu proprietatea:
1 + a12 + a1a 22 + a1a 2a 32 + ... + a1a 2 ...a n −1a 2n = a1a 2 ...a n (a n + 1) .
Etapa locală, Botoşani, 2009
14.
14. Fie a ∈ R, a > 2 (fixat) şi (xn)n≥0 un şir de numere reale cu x0 > 0, x1 – x0 > 1, care satisface
(a − 1) n − 1
recurenŃa xn+1 = axn + (1 – a)xn+1, (∀) n ∈ N*. Să se arate că x n > , (∀) n ∈ N*.
a−2
Etapa locală, Vaslui, 2009

3.2. Progresii aritmetice


1. Se consideră şirul (xn)n≥1 format din elemente strict pozitive. Să se arate că (xn)n≥1 este o progresie
1 1 1 2  1 1 1 
aritmetică dacă şi numai dacă + + ... + =  + + ... +  , pentru
x1x n x 2 x n −1 x n x1 x1 + x n  x1 x 2 x n 
orice n ≥ 3, n ∈ N.
Etapa locală, Buzău, 2010
1
2. Fie (an)n≥1 o progresie aritmetică cu raŃia , iar termenii a1 şi an sunt numere naturale nenule.
3
Ştiind că suma primilor n termeni ai progresiei aritmetice este 2010, se cere:
a) să se determine n;
b) să se determine numărul termenilor din sumă care sunt numere naturale.
Etapa locală, GalaŃi, 2010, prof. M. Cârmaciu
3. Fie A o mulŃime cu 1005 termeni în progresie aritmetică cu raŃia 2 şi B o mulŃime cu 1006
termeni în progresie aritmetică cu raŃia 3 . DeterminaŃi numărul minim de elemente ale mulŃimii
A ∪ B.
Etapa locală, Giurgiu, 2010, prof. Şerban Olteanu
1 1 1 Sn
4. Să se demonstreze că + + ... + ≤ , unde a1, a2, ..., an sunt numere pozitive în progre-
a1 a 2 a n a 1a n
sie aritmetică, iar Sn este suma lor.
Etapa locală, Gorj, 2010
5. Să se determine progresia aritmetică de 6 termeni astfel ca suma pătratelor primilor 4 termeni să
fie egală atât cu produsul primilor 3 termeni, cât şi cu produsul ultimilor doi termeni.
Etapa locală, Ilfov, 2009
6. FormaŃi o progresie aritmetică cu n termeni pozitivi, ştiind că produsul dintre primul termen şi
raŃie este a, iar suma celor n termeni ai progresiei este minimă.
Etapa locală, Prahova, 2009

24
7. Fie a1, a2, a3, ..., an, ... termenii pozitivi ai unei progresii aritmetice. Să se demonstreze inega-
a1 ⋅ a 3 ⋅ ... ⋅ a 2 n +1
litatea: ≤ a1 ⋅ a 2 n +1 .
a 2 ⋅ a 4 ⋅ ... ⋅ a 2 n
Etapa locală, Hunedoara, 2009
8. Se consideră o progresie aritmetică (an)n≥1 cu raŃia r ∈ [0, 1). DemonstraŃi că există n ∈ N*, astfel
încât [an] = [an+1]. (Prin [x] s-a notat partea întreagă a numărului real x).
Etapa locală, Sibiu, 2009, prof. Alin Pop
a1 + a 2 + ... + a n a n +1
9. Fie şirul (an)n≥1 şi k ∈ N, k ≥ 3, astfel încât = , (∀) n ≥ 1. DemonstraŃi că
n k
an
şirul (bn)n≥1, unde b n = este o progresie aritmetică.
n (n + 1)(n + 2)...( n + k − 3)
Etapa locală, DâmboviŃa, 2009, prof. C. Burduşel
10.
10. DemonstraŃi că şirul (an)n≥1, an ∈ (0, ∞), este progresie aritmetică dacă şi numai dacă:
1 1 1 n −1
+ + ... + = , pentru orice n ≥ 2.
a1a 2 a 2 a 3 a n −1a n a1a n
Etapa locală, Alba, 2009
11.
11. Să se demonstreze că numerele 2 , 3 şi 5 nu pot fi termeni ai aceleiaşi progresii aritmetice.
Etapa locală, NeamŃ, 2009
12.
12. Se consideră şirul de numere reale (xn)n≥0, definit prin x n +1 = x n + 1 + 4x n + 1 , (∀) n ≥ 0, x0 = 2.
a) Dacă y n = 4 x n + 1 , să se arate că şirul (yn)n≥0 este o progresie aritmetică.
b) Să se găsească formula termenului general al şirului (yn)n≥0 şi respectiv (xn)n≥0.
Etapa locală, Bacău, 2009
13.
13. Se consideră numerele reale a1, a2, ..., a2007, în această ordine, în progresie aritmetică şi S = a12 −
− a 22 + a 32 − a 24 + ... + a 22007 − a 22008 . Să se arate că:
1004 2
S= (a1 − a 22008 ) .
2007
Etapa locală, Ilfov, 2009
14.
14. Fie (an)n≥0 o progresie aritmetică. ArătaŃi că:
a) pentru oricare trei numere naturale m, n, p distincte două câte două, există un număr raŃional α,
astfel încât am = α ⋅ an + (1 – α)ap;
b) dacă 2 şi 3 sunt termeni ai progresiei, atunci am ∈ R / Q, (∀) m ∈ N*.
Etapa locală, Giurgiu, 2009

25
3.3. Progresii geometrice
1. Să se demonstreze că şirul (bn)n≥1 este o progresie geometrică dacă şi numai dacă există a, b ∈ R*
astfel încât să aibă loc relaŃia: b1 + b2 + ... + bn = a ⋅ bn – a, (∀) n ∈ N, n ≥ 3, b ≠ 1.
Etapa locală, Ilfov, 2010
2. Fie numerele reale pozitive a, b, c şi n ∈ N astfel încât numerele a , b , cn sunt în progresie
n n

aritmetică, iar numerele an+1, bn+1, cn+1 sunt în progresie geometrică. ArătaŃi că a = b = c.
Etapa locală, Sălaj, 2010
3. Să se calculeze suma S = 8 + 88 + 888 + ... + 88 ...
 8 .
n ori

Etapa locală, Vâlcea, 2010


4. Fie (an)n≥1 o progresie geometrică cu termeni strict pozitivi şi raŃia q ∈ R – Q, cu a1 + a3 + a5 + ... +
1 1 1 1
+ + + ... +
+ a4015 = 64, iar = 16 . Să se arate că a2009 ∈ R – Q.
a2 a4 a6 a 4016
Etapa locală, Braşov, 2009
5. Se consideră progresia geometrică (bn)n≥1. Să se arate că:
b12 − b 22 + b32 − b 24 + ... + b 22 n −1 − b 22 n ≥ b 22 n +1 − b 22 n + 2 + ... + b 24 n −1 − b 24 n , oricare ar fi n ≥ 1.
Etapa locală, Cluj, 2009, prof. I. Diaconu
6. Fie a1, a2, a3, ..., a12 o progresie geometrică crescătoare cu termeni pozitivi. DemonstraŃi că
a12 – a1 ≥ 11(a7 – a6).
Etapa locală, Dolj, 2009
7. Se consideră şirul (an)n≥1 definit prin a1 = 1, a2 = 2 şi an+2 = 3an+1 – 2an, n ≥ 1.
a) DeterminaŃi an.
b) ArătaŃi că şirul (bn)n≥1 definit prin bn = an+1 – an este o progresie geometrică.
Etapa locală, Alba, 2009
8. Fie an = 2 ⋅ 3 , n ∈ N*.
n 1–n

a) Să se arate că şirul (an)n≥1 este o progresie geometrică.


130
b) DeterminaŃi n ∈ N* astfel încât suma a1 + a 2 + ... + a n = .
27
Etapa locală, BistriŃa-Năsăud, 2009
9. Dorim să construim un gard din vergele metalice având lungimile în progresie geometrică crescă-
toare şi care să îndeplinească condiŃiile:
a) ultima vergea să fie de două ori mai lungă decât prima;
4
b) să existe două vergele astfel încât una să aibă lungimea egală cu din lungimea primei
3
vergele.
PrecizaŃi dacă este posibilă construcŃia.
Etapa locală, Mureş, 2009
10. Se consideră ∆BAC cu aria S şi punctele A1, B1, C1 mijloacele laturilor BC, AC, respectiv AB.
Dacă A2, B2, C2 sunt mijloacele laturilor B1C1, A1C1, respectiv B1A1 şi repetăm procedeul astfel
26
încât An, Bn, Cn sunt mijloacele laturilor Bn–1Cn–1, An–1Cn–1, respectiv Bn–1An–1, iar Sn este aria
triunghiului AnBnCn, se cere:
a) să exprimaŃi S2 în funcŃie de S;
b) să arătaŃi că S1, S2, ..., Sn formează o progresie geometrică;
S
c) să arătaŃi că S1 + S2 + ... + Sn < .
3
Etapa locală, Tulcea, 2009
1 a 
11. Fie a, x1 > 0 şi fie şirul (xn)n≥1 definit prin x n +1 =  x n +  , n ∈ N*.
2 xn 
a) DemonstraŃi că există λ ∈ R şi o progresie geometrică (bn)n≥1 de numere naturale astfel încât
xn − a
= λb n , oricare ar fi n ∈ N*.
xn + a
b) DeterminaŃi termenul general al şirului (xn)n≥1.
Etapa locală, DâmboviŃa, 2009
12.
12. Se consideră progresia geometrică (bn)n≥1 cu b1 > 0 şi raŃia q > 0. ArătaŃi că şirul (an)n≥1, an =
b n + b 2n +1
= , (∀) n ≥ 1, este progresie geometrică şi, în cazul a1 = 1, calculaŃi în funcŃie de q
1 + bn+2
valoarea sumei S = a1b1 + a2b2 + ... + a10b10.
Etapa locală, Sălaj, 2009
13.
13. Să se cerceteze dacă există progresii geometrice cu termeni naturali, raŃia nenulă şi pentru care
suma primilor n termeni este pătrat perfect, oricare ar fi n ∈ N*.
Etapa locală, Satu Mare, 2009

3.4. NoŃiunea de funcŃie. EcuaŃii funcŃionale

1. Se consideră funcŃia f: N* → R, f (n ) = 2 + 2 + 2 + ... + 2 + 2 cu n radicali.


a) Să se arate că funcŃia de mai sus este mărginită.
b) Să se studieze monotonia funcŃiei date.
Etapa locală, Vrancea, 2010
2. Să se determine toate funcŃiile f: R → R, cu proprietatea că f(x + y) ≤ f(xy), (∀) x, y ∈ R.
Etapa locală, Arad, 2010
3. FuncŃia f: R → R verifică relaŃia f(x + 2) + f(x – 2) = f(x), (∀) x ∈ R. DemonstraŃi că:
a) funcŃia f este periodică;
x x
b) f: R → R, f ( x ) = − are proprietatea din enunŃ.
12 12 
Etapa locală, Gorj, 2010

27
4. Se consideră funcŃia f: (0, ∞) → R şi a > 0 astfel încât f(a) = 1. Să se demonstreze că dacă f(x) ⋅
a a
⋅ f(y) + f   ⋅ f   = 2f ( xy) , pentru orice x, y ∈ (0, ∞), atunci f este constantă.
x y
Etapa locală, Bacău, 2010
5. Să se determine toate funcŃiile monoton crescătoare f: R → Z care satisfac simultan condiŃiile:
i) f(x) = x, (∀) x ∈ Z;
ii) f(x + y) ≥ f(x) + f(y), (∀) x, y ∈ R.
Etapa locală, Călăraşi, 2010
6. Fie f: N → N definită prin f(0) = 0 şi f(n + 1) = 2(n + 1 + f(n)), (∀) n ∈ N.
a) DemonstraŃi că f(n) < 2n+2, pentru orice n ∈ N.
b) AflaŃi n pentru care f(n) este putere (cu exponent întreg) a lui 2.
Etapa locală, Vaslui, 2009
 
1 2
7. Fie funcŃia f: R → R, cu proprietatea că f ( x ) + 2f   = x 2 + 3 + 2 , (∀) x ∈ R*. Să se calculeze
x a
2007
1
suma ∑k = 2 f (n ) − 2
.

Etapa locală, Sălaj, 2009


8. Să se determine funcŃia f: R → R astfel încât 2f(x) + 3f(1 – x) = 4x – 1, (∀) x ∈ R.
Etapa locală, Alba, 2009
9. Să se determine funcŃia f: N → N cu proprietăŃile:
a) f(m + n) = f(m) + f(n) + 2mn – 1;
b) f(n) ⋅ f(n + 1) = f((n + 1)2).
Etapa locală, Bihor, 2009, prof. I. Cuc

10. Să se studieze:
a) injectivitatea funcŃiei f: N → [0, 1], f ( n ) = { n } ;
b) surjectivitatea funcŃiei g: N → [0, 2), g (n ) = { n } + { n + 1} .
Etapa locală, Arad, 2009
11. Să se determine f: N* → N*, cu proprietăŃile:
i) f(n) ≤ 4n – 1, (∀) n ∈ N*;
ii) f(1) + f(2) + ... + f(n) este pătrat perfect, (∀) n ∈ N*.
Etapa locală, Argeş, 2009, prof. M. Ionescu
 1  2
12. Se consideră funcŃia f: R → R, f ( x ) = [ x ] +  x +  +  x +  − [3x ] .
 3   3
 1
a) VerificaŃi că f  x +  = f ( x ) , oricare ar fi x real.
 3
b) ArătaŃi că f(x) = 0, oricare ar fi x real.
Etapa locală, BistriŃa-Năsăud, 2009

28
f (x ) + 3
13.
13. Să se demonstreze că funcŃia f, cu proprietatea că există a ≠ 0, astfel încât f ( x + a ) = ,
1 − f (x ) ⋅ 3
3
f (x) ≠ , (∀) x ∈ R este periodică.
3
Etapa locală, Satu Mare, 2009
 − x + 2008, pentru x ∈ Q
14. Fie f: R → R, f (x) =  .
 2009 − x, pentru x ∈ R − Q
a) ArătaŃi că f este inversabilă şi precizaŃi inversa sa. b) CalculaŃi f(2008) – f (2009 − 2009 ) .
c) RezolvaŃi în R ecuaŃia f(x) = –1. d) RezolvaŃi în R ecuaŃia f(x) = a, a ∈ R.
Etapa locală, Suceava, 2009
15.
15. Fie f: R → R definită prin f(x) = {x} + {2x} + {3x}, unde {a} reprezintă partea fracŃionară a
numărului real a.
a) Să se arate că funcŃia f este periodică şi să se determine perioada principală.
b) Să se rezolve inecuaŃia f(x) ≤ 1.
Etapa locală, ConstanŃa, 2009

3.5. FuncŃia de gradul I şi funcŃia de gradul II


1. Fie f: R → R o funcŃie de gradul al doilea. Să se arate că numărul elementelor mulŃimii A = {x ∈
∈ R | f(f(x)) = x} este diferit de 3.
Etapa locală, Timiş, 2010
2. Fie a, b, c ∈ R, a ≠ 0. Să se arate că:
a) funcŃia f: R – Q → R, f(x) = ax2 + bx + c nu este injectivă;
b
b) funcŃia g: Q → R, g(x) = ax2 + bx + c este injectivă dacă şi numai dacă ∈ R – Q.
a
Etapa locală, Mureş, 2010
3. DeterminaŃi funcŃia de gradul I, f: R → R ştiind că f ( 3 ) = 3( 3 + 1) şi graficul lui f conŃine cel
puŃin două puncte cu ambele coordonate numere întregi.
Etapa locală, Tulcea, 2010
4. Există funcŃii f: R → R, f(x2) = ax + b, (∀) x ∈ R, unde a, b ∈ R, a ≠ 0?
Etapa locală, Covasna, 2010
5. Fie ecuaŃia 2(m + 2)2x2 – 2(m2 + 3m + 2)x – m = 0, m ∈ (0, ∞).
a) DemonstraŃi că soluŃiile ecuaŃiei sunt reale şi distincte.
1
b) Dacă x1 < x2, arătaŃi că − < x1 < x 2 < 1 , unde x1 şi x2 sunt rădăcinile ecuaŃiei date.
2
Etapa locală, Ilfov, 2009

29
6. Fie x1 şi x2 soluŃiile ecuaŃiei x2 – 6x + 3 = 0 şi Sn = x1n + x n2 , n ∈ N*.
a) Să se calculeze S2S4 − S32 .ed
b) Să se arate că: Sn ⋮ 3, (∀) n ∈ N*.
Etapa locală, Vrancea, 2009
7. Se consideră numerele de trei cifre A = a1a 2a 3 , B = b1b 2 b 3 , C = c1c 2c3 , cu cifrele a1, a2, a3
nenule. Să se arate că dacă ecuaŃia Ax2 + Bx + C = 0 are soluŃii reale, atunci cel puŃin una din
ecuaŃiile a i x 2 + bi x + ci = 0 , i ∈ {1, 2, 3} are soluŃii reale.
Etapa locală, Buzău, 2009
8. Se consideră a, b, c ∈ N*. Să se arate că dacă ecuaŃia ax + 2 b x + c = 0 are soluŃii reale şi dife-
2

rite, atunci ecuaŃia b x 2 + 2 a + c ⋅ x + b = 0 are cel mult o soluŃie reală.


Etapa locală, Giurgiu, 2009
9. Dacă a, b ∈ R, n ∈ N* şi dacă ecuaŃia x2 + ax + b + n = 0 are soluŃii întregi, atunci na2 + b2 este
număr natural compus.
Etapa locală, Teleorman, 2009
10. Se dă ecuaŃia x – (2m + 1)x + m = 0, unde m ∈ N, cu soluŃiile x1, x2, x1 < x2. Să se arate că
2 2

intervalul (x1, x2) conŃine cel mult un număr natural pătrat perfect.
Etapa locală, DâmboviŃa, 2009
11. Considerăm numerele reale a, b, c, pentru care ecuaŃia 3x + 2(a + b + c)x + a2 + b2 + c2 = 0 are
2

soluŃii reale. CalculaŃi valoarea minimă a sumei S = (a – 1)2 + (b – 2)2 + (c – 3)2.


Etapa locală, Arad, 2009
12.
12. a) ArătaŃi că există funcŃii f: R → R, f(x) = ax2 + bx + c, a ≠ 0, astfel încât f(f(k)) = k, (∀) k ∈
∈ {1, 2, 3}.
b) ArătaŃi că, dacă f este o funcŃie de la a), atunci numerele a, b, c nu sunt toate întregi.
Etapa locală, Iaşi, 2009, prof. Gh. Iurea
13. Fie numerele a, b, c, d ∈ N*. Să se rezolve ecuaŃia x – (a + b2 + c2 + d2 + 1)x + ab + bc + cd +
2 2

+ da = 0, ştiind că are o soluŃie întreagă.


Etapa locală, Satu Mare, 2009
14. a) Fie α ∈ Z astfel încât 1 – 4α este un număr prim natural. Dacă ecuaŃia x2 + x + α = y4 are o
soluŃie în Z, să se arate că –α este pătrat perfect.
b) Să se rezolve ecuaŃia x2 + x – 4 = y4 în numere întregi.
Etapa locală, Covasna, 2009

IV.
IV. Alte tipuri de probleme
1. Să se calculeze suma numerelor naturale impare cuprinse între n2 – 5n + 6 şi n2 + n, n ∈ N, n ≥ 4.
Etapa locală, MehedinŃi, 2010
2. Fie x şi y numere naturale distincte nenule.
a) Să se arate că numerele x2 + 2y şi y2 + 2x nu sunt ambele pătrate perfecte.

30
b) Să se determine perechile (x, y) pentru care numerele x3 + 3y2 + 3x + 1 şi y3 + 3x2 + 3y + 1
sunt ambele cuburi perfecte.
Etapa locală, NeamŃ, 2010
3. DeterminaŃi cel mai mare număr de numere naturale din mulŃimea M = {1, 2, ..., 2009} cu
proprietatea că suma oricăror două numere alese se divide cu 10.
Etapa locală, Brăila, 2010
4. Numerele 9, 25 şi 49 sunt termeni ai unei progresii aritmetice cu raŃia strict pozitivă. Să se arate
că numărul 2009 este de asemenea termen al acestei progresii.
Etapa locală, Satu Mare, 2010
5. Să se demonstreze că următoarele numere nu sunt prime:
a) A = 27 000 000 000 000 001;
b) B = 125 000 000 000 125 001 – 75 000 000.
Etapa locală, Alba, 2010
6. a) Să se demonstreze că (1 + a) ≥ 1 + na, (∀) n ∈ N, (∀) a ∈ (–1, ∞).
n

b) Fie (an)n≥1 o progresie aritmetică cu termeni pozitivi, iar (bn)n≥1 o progresie geometrică cu
termeni pozitivi, astfel ca a1 = b1 şi a2 = b2. Să se demonstreze că an ≤ bn, (∀) n ≥ 1.
Etapa locală, Botoşani, 2009

7. Se consideră un pătrat de latură 1. Ducând 4 drepte paralele cu laturile sale, îl împărŃim în 9


pătrate egale. Pătratul din mijloc îl eliminăm. Cele 8 pătrate rămase le împărŃim în acelaşi mod în
9 pătrate egale şi din nou eliminăm pătratul din mijloc. Repetând procedeul de 2008 ori, se cere
aria pătratelor eliminate.
Etapa locală, Călăraşi, 2009
8. Să se arate că oricare ar fi 2k + 1 numere prime mai mari ca 2, există cel puŃin două care au dife-
renŃa divizibilă cu 2k+1 (k număr natural nenul).
Etapa locală, GalaŃi, 2009, prof. P. BătrâneŃu
9. Scriem numerele naturale impare sub forma unui tabel, astfel:
1
(pe linia a n-a a tabelului se scriu n numere, oricare ar fi n ∈ N*).
3 5
Să se demonstreze că:
7 9 11
a) pe oricare din liniile impare ale tabelului, numărul din
13 15 17 19
mijloc este un pătrat perfect;
b) suma numerelor de pe fiecare linie a tabelului este un cub ... ... ... ... ...
perfect.
Etapa locală, Iaşi, 2009
10. Să se demonstreze că pentru orice număr natural n, n ≥ 2, există n numere naturale nenule şi
1 1 1 1
diferite x1, x2, ..., xn, astfel încât + + ... + = .
x1 x 2 x n 2009
Etapa locală, IalomiŃa, 2009
11. Se consideră mulŃimea M ⊂ N cu card M ≥ 3. Spunem că funcŃia f: M → M are proprietatea (P)
dacă (∀) A, B ⊂ M, avem f(A) = B sau f(B) = A.
a) Să se dea exemplu de funcŃie cu proprietatea (P) dacă M = {1, 2, 3}.

31
b) Dacă M este o mulŃime pentru care există funcŃii cu proprietatea (P), să se determine numărul
acestor funcŃii. Câte dintre acestea sunt bijective? Dar monotone?
Etapa locală, Maramureş, 2009
12. a) Fie A = {(x, y) | |x| + |y| ≤ 5}. ReprezentaŃi grafic mulŃimea, folosind ca unitate de măsură 1 cm.
b) Să se arate că oricum am lua 101 puncte distincte din mulŃimea A, există cel puŃin două dintre
acestea aflate la o distanŃă mai mică sau egală cu 1 cm unul de altul.
Etapa locală, MehedinŃi, 2009
13.
13. Fie a, b, c ∈ R+ variabile cu proprietatea că a + b + c = 18. Să se determine valoarea maximă a
produsului a2b3c4 şi să se determine a, b, c în acest caz.
Etapa locală, Teleorman, 2009

GEOMETRIE

V. Vectori în plan

5.1. OperaŃii cu vectori


1. a) Să se demonstreze că ABCD este paralelogram dacă şi numai dacă pentru orice punct P din
plan are loc relaŃia PA + PC = PB + PD .
b) Fie ABCD un patrulater şi punctele M, N, P, Q, R, S mijloacele segmentelor AB, CD, AN,
BN, CM, DM. Să se arate că P, Q, R, S sunt vârfurile unui paralelogram.
Etapa locală, Hunedoara, 2010
2. Se consideră hexagonul regulat ABCDEF de latură l. Să se calculeze modulul vectorului
AC + BD .
Etapa locală, Vrancea, 2010
3. Pe laturile DC, BC ale paralelogramului ABCD se consideră punctele M, respectiv N astfel încât
1 1
DM = DC , CN = CB . Fie X, Y, P mijloacele segmentelor BM, ND, AN. DemonstraŃi că
4 3
1
XY = PM .
2
Etapa locală, Vâlcea, prof. Claudiu Coandă
4. Fie A, B, C, D din plan pentru care AD = a BA + b BC + CD , unde a, b ∈ R cu a + b = 1. ArătaŃi
că punctele A, B, C sunt coliniare.
Etapa locală, Braşov, 2020, prof. A. Aldea
5. Dacă triunghiurile ABC şi A'B'C' au acelaşi centru de greutate, arătaŃi că lungimile AA', BB', CC'
pot fi laturile unui triunghi.
Etapa locală, Giurgiu, 2010, prof. Şerban Olteanu

32
6. Pe laturile AB şi AC ale triunghiului ABC se consideră punctele D şi respectiv E, astfel încât
DA + DB + EA + EC = 0 . Fie T intersecŃia dreptelor DC şi BE. Să se determine α ∈ R cu proprie-
tatea că TB + TC = αTA .
Etapa locală, Braşov, 2009, Gazeta Matematică, 9/2008

7. Fie E şi F mijloacele laturilor [AB] şi [CD] ale patrulaterului ABCD şi I mijlocul lui EF. ArătaŃi că:
a) IA + IB + IC + ID = 0 ;
1
b) dacă CJ = CD şi EJ ∩ CI = {M}, arătaŃi că 3AM = AE + AF + AC .
4
Etapa locală, Ilfov, 2009
8. Fie M mijlocul diagonalei [AC], iar N mijlocul diagonalei [BD] al patrulaterului ABCD. Să se
arate că:
a) AD + CB = AB + CD ; b) AB + CD = 2MN ; c) AB + AD + CB + CD = 4MN .
Etapa locală, Covasna, 2009
9. Fie ABC un triunghi echilateral şi M un punct în interiorul său. Paralelele prin M la AB şi AC
intersectează BC în P, respectiv Q.
a) Dacă MM1 ⊥ BC, M1 ∈ (BC), arătaŃi că 2MM1 = MP + MQ .

b) Dacă MM2 ⊥ AC, M2 ∈ AC şi MM3 ⊥ AB cu M3 ∈ (AB), atunci are loc:


MA + MB + MC = 2(MM1 + MM 2 + MM 3 ) .
Etapa locală, Alba, 2009
10. Fie ABCD patrulater convex.
5 4
a) ConstruiŃi punctele N pe (AD) şi Q pe (BC) astfel încât AN =
AD şi BQ = BC .
7 7
b) DeterminaŃi locul geometric al punctelor M din planul patrulaterului ABCD pentru care
modulul vectorului 2MA + 3MB + 4MC + 5MD are valoare constantă.
Etapa locală, Iaşi, 2009
11.
11. Se notează O1 şi O2 mijloacele diagonalelor [AC], respectiv [BD] ale patrulaterului convex
ABCD.
AD − BC
i) Să se arate că O1O 2 = .
2
ii) Să se arate că dacă AD + CB = 3O1O 2 , atunci ABCD este paralelogram.
Etapa locală, Teleorman, 2009
12. Fie [AB] şi [CD] două coarde perpendiculare ale unui cerc de centru O. Dacă {P} = AB ∩ CD, să
se demonstreze că PA + PB + PC + PD = 2PO .
Etapa locală, Sălaj, 2009

33
13.
13. În patrulaterul ABCD, M, N, P, Q sunt mijloacele laturilor (AB), (BC), (CD), respectiv (DA).
1 1
a) ArătaŃi că MP = (AD + BC) şi QN = ( AB + DC) .
2 2
b) ArătaŃi că MP + QN = AC .
Etapa locală, BistriŃa-Năsăud, 2009
14. Fie ABCD un patrulater fixat şi M un punct variabil în plan. Să se arate că vectorul legat MA +
+ MB + MC + MD trece printr-un punct fix.
Etapa locală, Tulcea, 2009
15. Fie paralelogramul ABCD şi punctele M, respectiv N, astfel încât 2BM = BC şi 3DN = DC .
 
Notăm AB = a , AD = b şi AM ∩ BN = {P}.
a) ExprimaŃi vectorii AM, BN, AN şi DM cu ajutorul vectorilor a şi b .
b) Dacă CM = 2b , demonstraŃi că punctele A, N, K sunt coliniare.
c) DeterminaŃi rapoartele în care punctul P împarte segmentul (AM).
Etapa locală, Harghita, 2009

5.2. Descompunerea unui vector


AM 1 AN 3
1. În triunghiul ABC considerăm punctele M ∈ (AB), N ∈ (AC) astfel încât = şi = . Fie
MB 5 NC 4
MP 2 BQ
P ∈ (MN) astfel încât = . Dacă AP ∩ BC = {Q}, să se determine valoarea raportului .
PN 7 QC
Etapa locală, Mureş, 2010
2. Pe laturile AB şi AC ale triunghiului ABC se consideră punctele D şi respectiv E, astfel încât
DA + DB + EA + EC = 0 . Fie T intersecŃia dreptelor DC şi BE. Să se determine α ∈ R cu
proprietatea că TB + TC = αTA .
Etapa locală, Mureş, 2010, Gazeta Matematică, 5/2009
3. Fie a , b, c trei vectori nenuli, astfel încât vectorii a + b + c, a − b + c, − a + b + c să fie coliniari. Să
se arate că vectorii a , b, c sunt coliniari.
Etapa locală, Sălaj, 2010
4. Fie un trapez ABCD (AB || CD). Fie E, F, M, N şi P mijloacele segmentelor AD, BC, DC, EF şi
AB. Să se arate că EF || AB şi M, N, P sunt puncte coliniare.
Etapa locală, Covasna, 2010
5. Fie triunghiul ABC. Să se arate că:
a) D ∈ BC ⇔ (∃) α ∈ R* astfel ca AD = α ⋅ AB + (1 − α)AC ;
b) D ∈ (BC) ⇔ (∃) α ∈ (0, 1) astfel ca AD = α ⋅ AB + (1 − α)AC ;
c) dacă D ∈ (BC) atunci AD ⋅ BC < AB ⋅ DC + AC ⋅ BD.
Etapa locală, Botoşani, 2010

34
MC
6. În patrulaterul convex ABCD se iau punctele M ∈ (BC), N ∈ (CD) astfel încât =3;
BM
CN
= 2 . Dreptele AM şi BN se intersectează în punctul P. Se cere:
ND
a) să se exprime vectorii BM, CN, AM, BN în funcŃie de vectorii AB, BC, CD .
AP BP
b) dacă patrulaterul ABCD este paralelogram, să se calculeze şi .
PM PN
Etapa locală, Ilfov, 2009
7. În triunghiul ABC cu 3AC = 2AB şi BC = 10 se duce bisectoarea (AD, D ∈ (BC). Să se exprime
în funcŃie de AB vectorii AD, BD şi IG , unde I este centrul cercului înscris în triunghi, iar G
este centrul de greutate al triunghiului.
Etapa locală, Vrancea, 2009
8. Considerăm punctele M şi N în planul triunghiului ABC şi p, q ∈ R*. Ştiind că AM = p AB + AC
1 1
şi CN = q CB , să se arate că A, M şi N sunt coliniare dacă şi numai dacă − = 1 .
q p
Etapa locală, Arad, 2009
3
9. În planul triunghiului ABC se consideră punctele P, Q, R, M definite astfel: AP = AC , 3AQ +
5
+ 2BQ = O , 31RA + 18RB + 6RC = O , AR ∩ BC = {M}.
a) Să se arate că punctele P, Q, R sunt coliniare.
| BM |
b) Să se afle valoarea raportului .
| CM |
Etapa locală, Botoşani, 2009
10.
10. Fie triunghiul ABC cu lungimile laturilor a, b, c, I centrul cercului înscris în triunghi, M şi N din
plan, astfel încât BM = m ⋅ MA şi CN = n ⋅ NA . ArătaŃi că dacă punctele M, N şi I sunt coliniare,
atunci mb + nc = a.
Etapa locală, Braşov, 2009
1
11.
11. Fie triunghiul ABC astfel încât M, Q ∈ (AB) şi N, P ∈ (BC) cu proprietăŃile că |BP| = |BC|,
3
1 1 1
|BN| = |BC|, |BQ| = |AB|, |BM| = |AB|. DescompuneŃi vectorii BC, AB, AC, NM după
2 4 2
vectorii u = BP şi v = BQ .
Etapa locală, BistriŃa-Năsăud, 2009
12.
12. Se consideră triunghiul ABC cu lungimile laturilor |AB| = c, |AC| = b, |BC| = a. Se notează cu P
intersecŃia dintre mediana BD, D ∈ (AC) şi bisectoarea CE a unghiului 'BCA, E ∈ (AB). Să se
descompună vectorul PA în funcŃie de vectorii u = PB, v = PC .
Etapa locală, Teleorman, 2009

35
13. Fie triunghiul ABC dreptunghic în A cu AB = c, BC = a, CA = b, AD ⊥ BC, D ∈ BC. Fie G
centrul de greutate al triunghiului. DescompuneŃi vectorii AD şi GD după vectorii u = AB ,

v = AC .
Etapa locală, Satu Mare, 2009
14.
14. Triunghiul ABC are laturile de lungimi AB = 6, BC = 8, AC = 5. Dacă AA' este bisectoarea
unghiului 'A, A' ∈ (BC), exprimaŃi vectorii AA', AI în funcŃie de direcŃiile vectorilor u = AB ,

v = AC , unde I este punctul de intersecŃie a bisectoarelor triunghiului.
Etapa locală, Giugiu, 2009
| AB | | AC |
15.
15. În ∆ABC, [AM] este mediană, iar D ∈ (AB), E ∈ (AC) astfel încât = m şi = n . Dacă
| AD | | AE |
AM ∩ DE = {F}, să se descompună vectorul AF după vectorii u = AB , v = AC .
Etapa locală, Botoşani, 2009

5.3. Coordonatele unui vector. Repere


1. În planul raportat la un sistem cartezian de coordonate se consideră punctele A(0, 3), B(4, 0) şi
respectiv C în cadranul I astfel încât BC = 2 3 şi semidreapta BC face cu semiaxa un unghi de
60°. Fie I centrul cercului înscris în triunghiul AOB. Dacă IC = αOA + βOB , să se afle α şi β.
Etapa locală, Teleorman 2010
2. Fie planul P raportat la un reper (O, i, j ) şi punctele A(–1, 1), B(1, 2) şi C(2, –1). DeterminaŃi |M|,
unde M = {G ∈ P | 3GA + GB − 2GC = 0 }, iar |X| este cardinalul mulŃimii X.
Etapa locală, Satu-Mare, 2010
AM CN
3. Fie ABCD un pătrat de latură 1, M ∈ (AB), N ∈ (BC) astfel încât = 7, = 2 . Notăm cu P
MB NB
intersecŃia dreptelor CM şi DN. Să se arate că 13AP = 12AB + 5AD .
Etapa locală, Bacău, 2010
1 1
4. Fie paralelogramul ABCD şi punctele E, F astfel încât AE = − AD, EF = BA . Să se demon-
2 2
streze că punctele C, A, F sunt coliniare în două moduri: a) vectorial; b) analitic, raportând la
reperul (A, AB, AD) .
Etapa locală, Tulcea, 2010
AM 1 AN 3
5. În triunghiul ABC considerăm punctele M ∈ (AB), N ∈ (AC) astfel încât = şi = .
MB 5 NC 4
MP 2 BQ
Fie P ∈ (MN) astfel încât = . Dacă AP ∩ BC = {Q}, să se determine .
PN 7 QC
Etapa locală, Mureş, 2010

36
6. Fie punctele A(–2, 0), B(0, 2), C(4, 2). Să se determine mulŃimea {D ∈ (ABC) | ABCD trapez}.
Etapa locală, Covasna, 2009
7. Fie punctele A(2, 1), B(6, 3). DeterminaŃi:
i) coordonatele punctului M pentru care AM = 3MB ;
ii) coordonatele mijlocului segmentului [AB].
Etapa locală, BistriŃa-Năsăud, 2009
8. Să se demonstreze că distanŃa de la centrul cercului circumscris unui triunghi oarecare la o latură
a sa este egală cu jumătatea distanŃei de la ortocentrul triunghiului la vârful opus acestei laturi.
Etapa locală, Botoşani, 2009
9. Considerăm în plan doi vectori u şi v . Să se demonstreze echivalenŃa afirmaŃiilor:
i) u = v ;

ii) a v + b v = b u + a v , (∀) a, b ∈ R*, cu a2 ≠ b2.


Etapa locală, Suceava, 2009
10.
10. Pe laturile AB, BC, CA ale unui triunghi de vârfuri A(4, 0), B(0, 3), C(1, 4) se iau punctele M, N,
MA NB PC 1
P astfel încât: = a, = − b, = − , a, b > 0.
MB NC PA ab
a) Să se arate că dreptele AN, BP, CM sunt concurente şi să se determine coordonatele punctului
H de concurenŃă.
b) Pentru ce valori ale lui a şi b, distanŃele lui H la laturile triunghiului ABC sunt egale?
Etapa locală, Satu Mare, 2009
11.
11. Fie A, B, C, D patru puncte distincte în plan, E şi F mijloacele bipunctelor (A, C), respectiv
     
(B, D). Notăm AB = a , BC = b, CD = c, DA = d . Fiecărei perechi de numere reale (α, β) îi
asociem vectorul s = α (a + c) + β(b + d ) .
a) ExprimaŃi pe s în funcŃie de α, β, EF . b) În ce condiŃii s = 0 ?
Etapa locală, IalomiŃa, 2009
12.
12. Se consideră dreptele a, b, d, d' astfel încât d ∩ d' ≠ ∅, a || b, a ∩ d = {A}, a ∩ d' = {A'}, b ∩ d =
= {B}, b ∩ d' = {B'}.
a) Să se demonstreze că A ' B' − AB = BB' − AA' .
b) Fie M ∈ d de abscisă x în reperul (A, AB) şi M' ∈ d' un punct de abscisă x', în reperul
(A ' , A ' B') . ArătaŃi că:
MM ' = x ⋅ (A ' B' − AB) + AA' + ( x '− x )A' B' şi
MM ' = x ⋅ BB' + (1 − x )AA' + ( x '− x )A ' B' .
c) DeduceŃi că MM' || a dacă şi numai dacă x = x'.
Etapa locală, Giurgiu, 2009

37
13.
13. Se consideră dreapta variabilă (dλ): 2x – y – 6 + λ(x – y – 4) = 0. Să se arate că nu există λ ∈ R,
pentru care distanŃa de la punctul A(3, –1) la această dreaptă să fie egală cu 3.
Etapa locală, Ilfov, 2009
14.
14. Fie A1, A2, ..., A6 vârfurile unui hexagon regulat cu centrul în O. Dacă xk ∈ R, k = 1,6 şi
x1 OA1 + + x 2 OA 2 + ... + x 6 OA 6 =0, să se arate că x1 + 2x2 + x3 = x4 + 2x5 + x6.
Etapa locală, Buzău, 2009
15. Fie A, B, C, D, E cinci puncte situate pe un cerc cu centrul în O, iar H1, H2, H3 ortocentrele triunghiu-
rilor ABC, ACD, ADE. Să se determine x ∈ R dacă (BD + CE )[(x − 2009) 2 + 1] = H1H 3 , H1 ≠ H3.
Etapa locală, MehedinŃi, 2009, prof. O. łicuşi

VI. Coliniaritate. ConcurenŃă. Paralelism

6.1. Vectorul de poziŃie al unui punct în plan


1. În paralelogramul ABCD avem AB = 4, BD = 3, BC = 2. Fie G centrul de greutate al triunghiului
ABD, I centrul cercului înscris în ∆BCD şi M ∈ BC astfel încât BM = 2MC. Să se arate că punc-
tele G, I, M sunt coliniare.
Etapa locală, Cluj, 2010, prof. C. Magdaş
2. Pe laturile AB şi AC ale triunghiului ABC se consideră punctele D şi respectiv E, astfel încât
DA + DB + EA + EC = 0 . Fie T intersecŃia dreptelor DC şi BE. DeterminaŃi α ∈ R cu proprie-
tatea că TB + TC = α ⋅ TA .
Etapa locală, Bacău, 2010
3. Fie ABCDE un pentagon şi M, N, P, Q, R, S mijloacele segmentelor AB, BC, CD, DE, MP şi
1
NQ. Să se arate că RS = AE .
4
Etapa locală, IalomiŃa, 2010
4. În triunghiul ABC mediana BD întâlneşte bisectoarea AF în punctul O. Raportul dintre aria
3 AC 1
triunghiului DOA şi aria triunghiului BOF este egal cu . ArătaŃi că = .
8 AB 2
Etapa locală, Timiş, 2010
5. Fie punctele M, N şi P pe laturile AB, BC respectiv AC ale triunghiului ABC.
a) Să se arate că centrul de greutate al triunghiului MNP aparŃine medianei din A a triunghiului
NB MA PC
ABC dacă şi numai dacă 2 = + .
BC AB AC

38
MA NB
b) Centrele de greutate ale triunghiurilor MNP şi ABC coincid dacă şi numai dacă = +
MB BC
PC
+ .
CA
MA
6. Fie M un punct situat pe segmentul [AB] care împarte segmentul într-un raport =k.
MB
1 k
DemonstraŃi că pentru orice punct P din plan are loc relaŃia: PM = PA + PB .
k +1 k +1
Etapa locală, Bihor, 2009
7. Fie A1, B1, C1 puncte pe laturile BC, CA, AB ale triunghiului ABC. Notăm cu G1, G2, G3, G, G',
G'' centrele de greutate ale triunghiurilor AB1C1, BC1A1, CA1B1, ABC, G1G2G3, A1B1C1 res-
pectiv. Să se arate că G, G', G'' sunt coliniare şi G ' ' G = 3G ' ' G ' .
Etapa locală, Vaslui, 2009, Gazeta Matematică, 7-8/2008
8. Fie x, y, z trei numere reale pozitive, cu x + y + z = 1, iar ∆ABC un triunghi oarecare. Pe laturile
AB z
acestuia se consideră punctele A1 ∈ (BC), B1 ∈ (CA), C1 ∈ (AB), astfel încât 1 = ,
A1C y
B1C x C1A y
= , = . ArătaŃi că:
B1A z C1B x
a) dreptele AA1, BB1, CC1 sunt concurente într-un punct S;
b) pentru orice punct P din plan are loc egalitatea: PS = x ⋅ PA + y ⋅ PB + z ⋅ PC .
Etapa locală, Timiş, 2009, prof. M. Chiş
9. Fie ABCDE un pentagon şi M, N, P, Q, R pe laturile AB, BC, CD, DE, respectiv EA, astfel încât
AM BN CP DQ RE
= = = = = k , k > 0. DemonstraŃi că pentagoanele ABCDE şi MNPQR au
MB NC PD QE RA
acelaşi centru de greutate.
Etapa locală, Alba, 2009
10. Fie punctele coliniare ordonate şi echidistante M1, M2, ..., Mn, O un punct arbitrar.
a) Să se exprime vectorii OM i , i = 1, 2, ..., n , în raport cu OM1 şi OM n .
n
b) Să se arate că OM1 + OM 2 + ... + OM n = (OM1 + OM n ) .
2
Etapa locală, Bihor, 2009
11.
11. Fie rA , rB , rC vectorii de poziŃie ai vârfurilor ∆ABC, iar a, b, c, lungimile laturilor BC, AC,
respectiv AB. Să se demonstreze că, dacă (b + c − 2a )rA + (c + a − 2b)rB + (a + b − 2c)rC = 0 , atunci
∆ABC este echilateral.
Etapa locală, Vaslui, 2009
12. Fie ABC un triunghi, D mijlocul lui AC şi M un punct astfel încât MA + a ⋅ MB = b ⋅ MD , unde
a, b ∈ R.

39
a) Dacă a = b = 1, arătaŃi că MD = BC .
b) Dacă MD = BC , arătaŃi că a = b = 1.
Etapa locală, Teleorman, 2008
13. Fie ABCD un patrulater convex, E mijlocul diagonalei AC şi F mijlocul celeilalte diagonale, BD.
DemonstraŃi că, dacă α EF = AD − BC , α ∈ R – {2}, atunci patrulaterul ABCD este parale-
logram.
Etapa locală, Timiş, 2008
14. Fie paralelogramul ABCD şi punctele P, Q, astfel încât DP = λ DC, DQ = µ DB , unde λ, µ ∈ R*.
ArătaŃi că punctele A, P, Q sunt coliniare dacă şi numai dacă λ – µ = λµ.
Etapa locală, Suceava, 2009, prof. I. Bursuc
15. Fie O punctul de intersecŃie al diagonalelor unui patrulater ABCD. Să se arate că ABCD este
paralelogram dacă şi numai dacă pentru orice punct M (din plan) are loc egalitatea:
4MO = MA + MB + MC + MD .
Etapa locală, BistriŃa-Năsăud, 2009

6.2. Centre de greutate. Baricentre


1. Se consideră triunghiul ABC şi punctele M, N, P, Q, R, S astfel încât BM = k ⋅ MC, CN = k ⋅ NA ,
AP = k ⋅ PB, AM = p ⋅ MQ, BN = p ⋅ NR , CP = p ⋅ PS . Să se arate că triunghiurile MNP şi QRS au
acelaşi centru de greutate.
Etapa locală, Maramureş, 2010, Gazeta Matematică, 11/2009
2. Fie triunghiul ABC şi punctele M, N, P astfel încât MA + 2MB + 3MC = 0 ; NB + 2 NC + 3NA =
= 0; PC + 2PA + 3PB = 0 . Să se arate că triunghiurile ABC şi MNP au acelaşi centru de greutate.
Etapa locală, Arad, 2010
3. În paralelogramul ABCD considerăm punctele P şi Q astfel încât AP = x AC şi AQ = y AC cu x,
y ∈ (0, 1). Paralela prin P la AB intersectează pe AD în L şi BC în K, iar paralela prin Q la BC
intersectează pe AB în M şi DC în N. Ce relaŃie există între x şi y dacă triunghiurile PMN şi QLK
au acelaşi centru de greutate?
Etapa locală, Vâlcea, 2010
4. Fie ∆ABC şi ∆A'B'C' într-un plan, având centrele de greutate G şi G'. ArătaŃi că:
a) AA' + BB' + CC' = 3GG ' ;
b) dacă A ' B = k A ' C, B' C = k B' A, C' A = k C' B , k ∈ R – {1}, atunci G şi G' coincid.
Etapa locală, Vrancea, 2010, prof. D. Munteanu
5. Dacă triunghiurile ABC şi A'B'C' au acelaşi centru de greutate, arătaŃi că lungimile AA', BB' şi
CC' pot fi laturile unui triunghi.
Etapa locală, Giurgiu, 2010, prof. Şerban Olteanu

40
   
6. Se consideră triunghiul MNP şi fie punctele T, S astfel încât NS = 3MS, iar PT = 3MT. Fie Q
  
punctul de intersecŃie al dreptelor PS şi NT. DemonstraŃi că QM = MN + MP.
Etapa locală, Iaşi, 2009, prof. Dorel Luchian şi prof. Gabriel Popa
7. Fie triunghiul ABC, punctele A' ∈ (BC), B' ∈ (AC), iar {P} = AA' ∩ BB'.
a) DemonstraŃi că oricare ar fi punctul M în planul triunghiului ABC, există α, β ∈ (0, 1) astfel
încât MP = α MA + βMB + γ MC .
A' C B' C
b) ArătaŃi că, dacă − = 1 , atunci punctul P este mijlocul segmentului (BB').
A' B B' A
Etapa locală, Hunedoara, 2009
8. În triunghiul ABC dreptunghic în A, avem AB = 3, BC = 5, G centrul de greutate, iar I centrul
cercului înscris. Să se demonstreze că GI || AC.
Etapa locală, Covasna, 2009
9. Pentru orice triunghi ABC se consideră AB = c, BC = a, CA = b, unde I este centrul cercului
înscris, iar G este centrul de greutate. Să se arate că:
1
a) GI = (a ⋅ GA + b ⋅ GB + c ⋅ GC) ;
a+b+c
b) dacă c = 3, b = 4, a = 5, atunci IG este paralelă cu AC.
Etapa locală, Caraş-Severin, 2009, prof. Gabriel Popa
10.
10. Fie A, B, C puncte coliniare. Să se arate că pentru orice punct M din plan există numerele reale α,
β, γ nu toate nule, astfel încât α ⋅ MA + β ⋅ MB + γ ⋅ MC = 0 , α + β + γ = 0.
Etapa locală, Teleorman, 2009
11. Fie ABC un triunghi, G centrul de greutate, O centrul cercului circumscris şi H un punct definit
prin OH = OA + OB + OC .
a) Dacă A' este mijlocul lui BC, demonstraŃi că AH şi OA' sunt coliniari.
b) DeduceŃi că AH este înălŃime în triunghiul ABC.
c) DemonstraŃi că H este ortocentrul triunghiului ABC.
d) Dacă M este un punct din plan, demonstraŃi că 3MG = MA + MB + MC .
e) DemonstraŃi că O, H, G sunt coliniare.
Etapa locală, Alba, 2008
12.
12. Fie ABC un triunghi şi MNR triunghiul său median. Dacă P este un punct situat în interiorul lui
MNR, iar A', B', C' sunt intersecŃiile dreptelor AP, BP, CP cu laturile BC, CA respectiv AB,
AP BP CP
notăm a = ,b = ,c= . Să se arate că, dacă a, b, c ∈ N*, atunci P coincide cu G, cen-
PA ' PB' PC'
trul de greutate al triunghiului ABC.
Etapa locală, Argeş, 2008, prof. M. Ionescu
13.
13. Fie G intersecŃia medianelor triunghiului oarecare ABC, iar O un punct oarecare în planul triun-
ghiului.
a) ArătaŃi că GA + GB + GC = 0 .

41
b) DemonstraŃi că OA + OB + OC = 3OG .
c) Dacă punctul O este mobil pe dreapta BC, determinaŃi poziŃia sa astfel încât OA + OB + OC să
fie minim.
Etapa locală, Bacău, 2008
14.
14. Fie triunghiul ABC şi punctele M, N, P astfel încât MA + 2MB + 3MC = 0 ; NB + 2 NC + 3NA =
= 0; PC + 2PA + 3PB = 0 . Să se arate că triunghiurile ABC şi MNP au acelaşi centru de greutate.
Etapa locală, Giurgiu, 2008, prof. S. Olteanu
15. Se consideră în plan punctele A, B, C, D, E şi fie G1, G2 centrele de greutate ale triunghiurilor
ABC, respectiv ABE. Să se arate că:
a) 3OG1 = OA + OB + OC , pentru orice punct O din plan;
b) dacă M şi N sunt mijloacele segmentelor [DE], respectiv [CD], atunci segmentele [G1M] şi
GG G G
[G2N] se intersectează într-un punct G cu proprietatea că 1 = 2 .
GN GM
Etapa locală, Gorj, 2008

6.3. Teorema bisectoarei. Teorema lui Sylvester


1. În patrulaterul convex ABCD notăm cu G centrul de greutate al triunghiului BCD şi cu H
ortocentrul triunghiului ACD. ArătaŃi că ABGH este paralelogram dacă şi numai dacă G este
centrul cercului circumscris triunghiului ACD.
Etapa locală, Buzău, 2010
2. Bisectoarele unghiurilor 'A, 'B, 'C ale triunghiului ABC intersectează laturile acestuia în D ∈
 
∈ (BC), E ∈ (AC), F ∈ (AB). ArătaŃi că triunghiul este echilateral dacă şi numai dacă AD + BF
+ CE = 0 .
Etapa locală, Braşov, 2010, prof. T. DuŃă
3. În planul triunghiului ABC cu AB = c şi AC = b, se consideră un punct D cu proprietatea AD =
= b ⋅ AB + c ⋅ AC . Să se arate că semidreapta [AD este bisectoarea unghiului 'BAC.
Etapa locală, Vrancea, 2010, M. Poll
4. În acelaşi plan se consideră triunghiurile ABC şi MNP astfel încât MA + MB + NA + NC + PB +
= PC = 0 . Dacă H şi O sunt ortocentrul respectiv centrul cercului circumscris triunghiului ABC,
arătaŃi că OM + ON + OP = OH .
Etapa locală, DâmboviŃa, 2010
5. Fie ABC un triunghi ascuŃitunghic şi punctele O, H, G centrul cercului circumscris, ortocentrul,
respectiv centrul de greutate al triunghiului. DemonstraŃi că:
a) OH = OA + OB + OC ;

42
b) punctele O, G, H sunt coliniare şi determinaŃi raportul în care punctul G împarte segmentul
[OH].
Etapa locală, Ilfov, 2010
6. Fie ABC un triunghi scalen cu AB < AC, (AD bisectoarea unghiului 'A, D ∈ (BC). Construim
BE || AC, E ∈ (AD), CF || AB, F ∈ (AD), BP ⊥ AD, P ∈ (AD), CQ ⊥ AD, Q ∈ (AD).
a) CalculaŃi AE şi AF în funcŃie de AB, AC şi de lungimile laturilor triunghiului ABC.
1
b) DemonstraŃi că PQ = EF .
2
Etapa locală, NeamŃ, 2009, A. Sandovici
7. Fie [AA' bisectoarea interioară a unghiului 'A al triunghiului ABC cu A' ∈ (BC) şi A'' simetricul
lui A' faŃă de mijlocul laturii (BC).
a) Să se exprime AA' ' în funcŃie de AB şi AC .
b) Să se arate că (b + c)AA' ' + (c + a ) BB' ' + (a + b)CC' ' = 0 , unde B'', C'' sunt analoagele lui A''.
Etapa locală, Timiş, 2009, prof. A. Doboşan
8. Fie ABCD un patrulater inscriptibil. Notăm Ha ortocentrul triunghiului BCD şi cu Ma mijlocul lui
AHa. În mod similar definim punctele Mb, Mc şi Md. DemonstraŃi că punctele Ma, Mb, Mc şi Md
coincid.
Etapa locală, Cluj, 2009, prof. C. Magdaş
9. Se consideră triunghiurile ABC şi A'B'C' înscrise într-un cerc de centru O. Dacă H1, H2, H3, H4, H5, H6
sunt ortocentrele triunghiurilor BAA', CBB', ABB', ACC' şi respectiv BCC', demonstraŃi relaŃiile:
a) OH1 = OA + OB + OA ' (RelaŃia lui Sylvester);
b) H1H 2 + H 3 H 4 + H 5 H 6 = 0 .
Etapa locală, Sibiu, 2009, prof. Petru Vlad
10.
10. Fie ABC un triunghi nedreptunghic, H ortocentrul său, O centrul cercului circumscris triun-
ghiului, iar Oa, Ob, Oc centrele cercurilor circumscrise triunghiurilor HBC, HCA, respectiv HAB.
a) Să se arate că HB + HC = HA + 2AO .
b) Să se demonstreze că, pentru orice punct M din planul triunghiului ABC, are loc relaŃia MA =
= MH + MB + MC − 2MO a .
c) Să se demonstreze că O este ortocentrul triunghiului OaObOc.
Etapa locală, DâmboviŃa, 2009
11. Fie ABCDE un pentagon înscris într-un cerc. Notăm cu H1, H2, H3, H4 ortocentrele triunghiurilor
ABC, BCD, CDE, respectiv ACE. Să se demonstreze că patrulaterul H1H2H3H4 este
paralelogram.
Etapa locală, Dolj, 2009
c c
12.
12. Fie I centrul cercului înscris în ∆ABC. Dacă IA + IB + IC = 0 , atunci ∆ABC este dreptun-
2a 2b
ghic isoscel.
Etapa locală, Argeş, 2008

43
13. Se consideră un triunghi ABC şi M, N, P puncte aparŃinând segmentelor (BC), (CA), (AB). Se ştie
BM CN AP
că = m, = n, = p . Ce condiŃie trebuie să îndeplinească numerele m, n, p astfel încât
MC NA PB
triunghiurile ABC şi MNP să aibă acelaşi centru de greutate?
Etapa locală, BistriŃa-Năsăud, 2009
14.
14. Fie ABCD un patrulater înscris într-un cerc şi H1, H2, H3, H4 respectiv ortocentrele triunghiurilor
ABC, CDA şi DAB.
a) Să se arate că H1H2DA, H2H3AB, H3H4BC, H4H1CD sunt paralelograme.
b) Să se arate că patrulaterul H1H2H3H4 este înscris într-un cerc congruent cu cercul circumscris
patrulaterului ABCD.
Etapa locală, Braşov, 2008
15.
15. Fie triunghiul ABC şi AA1, BB1, CC1 bisectoarele unghiurilor A, B, C. Să se arate că vectorii
AA1 , BB1 , CC1 pot forma un triunghi dacă şi numai dacă triunghiul ABC este echilateral.
Etapa locală, Mureş, 2008

6.4. Teorema lui Menelaus şi Ceva


1. Fie triunghiul ABC şi punctele M, N, P, Q astfel încât BM = α ⋅ MC, CP = β ⋅ PA, AQ = γ ⋅ QB ,
BN = m ⋅ CN , unde α, β, γ, m ∈ R*+. Se cere:
a) dacă α = β = γ, să se demonstreze că ∆ABC are acelaşi centru de greutate cu ∆MPQ;
b) să se determine relaŃia dintre α, β, γ astfel încât dreptele AM, CQ, BP să fie concurente;
c) dacă NQ ∩ AC = {S}, atunci să se determine raportul în care punctul S împarte segmentul AC.
Etapa locală, GalaŃi, 2010, prof. C. Ursu
2. Se consideră triunghiul ABC, M este mijlocul lui (AC), N ∈ (BM) astfel încât BM = 4BN şi P ∈
∈ (BC) astfel încât PC = −6PB .
a) DemonstraŃi că punctele A, N, P sunt coliniare.
b) Dacă Q ∈ (AB) astfel încât PQ || AC, demonstraŃi că dreptele AP, BM şi CQ sunt concurente.
Etapa locală, Giurgiu, 2010, prof. Ion Staicu
3. Fie D un punct oarecare pe latura BC a triunghiului ABC. Bisectoarea unghiului 'ADC intersec-
tează AC în E, iar bisectoarea unghiului 'ADB intersectează AB în F. Dacă BE ∩ CF = {P},
demonstraŃi că punctele A, D şi P sunt coliniare.
Etapa locală, Iaşi, 2010
4. Fie ABCD un patrulater convex, cu AB ∩ CD = {E}, AD ∩ BC = {F}, iar M, N, P mijloacele
segmentelor [AC], [BD], respectiv [EF].
a) ArătaŃi că există x, y ∈ R* unic determinate, astfel încât AC = x ⋅ AB + y ⋅ AD .
b) DeterminaŃi x1, y1, x2, y2 ∈ R* astfel încât AE = x1 ⋅ AB + y1 ⋅ AD ; AF = x 2 ⋅ AB + y 2 ⋅ AD .
c) ArătaŃi că punctele M, N, P sunt coliniare.

44
5. Fie ABC un triunghi oarecare şi M un punct în interiorul triunghiului ABC. Considerăm punctele
D, E şi F astfel încât AM ∩ BC = {D}, BM ∩ AC = {E} şi CM ∩ AB = {F}.
AF BD CE 1
ArătaŃi că: ⋅ ⋅ ≤ .
AB BC CA 8
Etapa locală, NeamŃ, 2009, prof. A. Sandovici
6. Fie triunghiul dreptunghic ABC (AB ⊥ AC), având catetele b, c şi ipotenuza a. Fie M, P, N
picioarele pe ipotenuză ale înălŃimii, bisectoarei, respectiv medianei din A. DemonstraŃi că:
2bc
a) MP = 2 PN ; b) MP ≤ PN.
a
Etapa locală, DâmboviŃa, 2009, prof. C. Burduşel
7. Fie triunghiul ABC şi A' ∈ BC, B' ∈ AC, C' ∈ AB astfel încât AA', BB', CC' sunt concurente în
M. DemonstraŃi că:
MA C' A B' A
i) = + ;
MA' C' B B' C
MA MB MC MA MB MC
ii) + + +2= ⋅ ⋅ ;
MA' MB' MC' MA' MB' MC'
MA MB MC
iii) + + ≥6.
MA' MB' MC'
Etapa locală, GalaŃi, 2009, prof. V. Dumbravă
8. Se consideră punctele D şi M în planul triunghiului ABC astfel încât 34MA + 36MB + 5MC = 0
18
şi AD = AB . Fie Q punctul de intersecŃie dintre dreptele AM şi BC.
35
a) DemonstraŃi că punctele C, M şi D sunt coliniare.
MA
b) AflaŃi .
MQ
Etapa locală, Iaşi, 2009
MB NC
9. În triunghiul ABC se consideră punctele M ∈ (AB) şi N ∈ (AC). Să se arate că + =1, dacă
MA NA
şi numai dacă G ∈ (MN), unde G este centrul de greutate al triunghiului ABC.
Etapa locală, Vrancea, 2009
10. Fie triunghiul ABC cu m('B) = 90°. Să se arate că există un singur punct D ∈ (AC) cu proprie-
tatea că în triunghiul ABD bisectoarea din A, mediana din B şi înălŃimea din D sunt concurente.
Etapa locală, Suceava, 2009, prof. B. Marchitan
11.
11. Fie cercurile C(O1, a), C(O2, b), C(O3, c) tangente exterior două câte două, unde {T1} = C(O1, a) ∩
∩ C(O2, b), {T2} = C(O2, b) ∩ C(O3, c) şi {T3} = C(O3, c) ∩ C(O1, a). Să se arate că dreptele O1T2,
O2T3, O3T1 sunt concurente.
Etapa locală, Tulcea, 2009

45
12. Fie ABC un triunghi oarecare şi O ∈ Int(ABC) un punct mobil. Notăm AO ∩ BC = {A'}, BO ∩
| C' D | | B' C |
∩ CA = {B'}, CO ∩ AB = {C'} şi B'C' ∩ AO = {D}. Să se arate că produsul ⋅ nu
| B' D | | C' B |
depinde de alegerea punctului O.
Etapa locală, IalomiŃa, 2009
13.
13. Se consideră un punct P în interiorul unui triunghi ABC şi notăm A', B', C' intersecŃiile dreptelor
AP BP CP
AB, BP, CP cu laturile BC, CA, respectiv AB, astfel încât are loc relaŃia + + = 2007 .
PA ' PB' PC'
AP BP CP
CalculaŃi produsul ⋅ ⋅ .
PA ' PB' PC'
Etapa locală, Giurgiu, 2009
14.
14. Fie G, I centrul de greutate şi centrul cercului înscris în ∆ABC şi M ∈ (AB), N ∈ (AC), astfel
încât MN ∩ BC = {P}.
| MB | | NC |
a) Dacă MN trece prin G, arătaŃi că + =1.
| MA | | NA |
| AC | | MB | | AB | | NC |
b) Dacă MN trece prin I, arătaŃi că ⋅ + ⋅ =1.
| BC | | MA | | BC | | NA |
Etapa locală, Botoşani, 2009

VII. Perpendicularitate
RelaŃii metrice. Produs scalar

1. Într-un pătrat ABCD se notează cu M şi N mijloacele laturilor BC respectiv CD. Să se demon-


streze că dreptele AM şi BN sunt perpendiculare.
Etapa locală, BistriŃa-Năsăud, 2003
2. În triunghiul [ABC] luăm D ∈ (BC) şi E ∈ (AC) astfel încât BD = DC şi AE = 2EC. Dacă 'B =
BC
= 60° şi AD ⊥ BE, determinaŃi raportul .
AB
Etapa locală, Tulcea, 2004
3. Într-un triunghi isoscel BCA, BC = BA, medianele AA' şi CC' fac între ele un unghi de 60°.
DeterminaŃi unghiul B, din vârful triunghiului isoscel.
Etapa locală, Covasna, 2005
4. Fie ABC un tringhi echilateral înscris în cercul C(O, R), iar M ∈ C. Să se arate că MA2 + MB2 +
+ MC2 = constant.
Etapa locală, Mureş, 2006
5. DemonstraŃi că o condiŃie necesară şi suficientă ca medianele AA' şi BB' ale unui triunghi ABC,
oarecare, să fie perpendiculare este: a2 + b2 = 5c2.
Etapa locală, MehedinŃi, 2006

46
6. DemonstraŃi că în orice triunghi ABC avem:
3 9
a) cos 2A + cos 2B + cos 2C ≥ − ; b) sin2 A + sin2 B + sin2 C ≤ .
2 4
Etapa locală, Harghita, 2004
3
7. DemonstraŃi că în orice triunghi ABC avem: cos A + cos B + cos C ≤ .
2
Etapa locală, Satu-Mare, 2005
8. Fie triunghiurile echilaterale ABC şi OBC. Fie un punct M pe cercul de centru O şi rază (OB). Să
se demonstreze că triunghiul având laturile de lungimi MA, MB, MC este dreptunghic.
Etapa locală, IalomiŃa, 2004
9. Să se demonstreze că:
a) în orice ∆ABC avem |b – c| < a < b + c;
b) pentru orice trei numere a, b, c > 0 pentru care |b – c| < a < b + c, există un triunghi ABC având
lungimile laturilor BC = a, AC = b, AB = c.
Etapa locală, Caraş, 2003
10. Fie ma, mb, mc lungimile medianelor ∆ABC de laturi a, b, c, 2p = a + b + c. Să se demonstreze că:
a) b + c – a < 2ma < a + b + c;
b) p < ma + mb + mc < 3p.
Etapa locală, Timiş, 2004

VIII. Transformări geometrice

8.1. TranslaŃia

1. În patrulaterul ABCD, AB = 3, BC = 3 , CD = 2 3 , 'BAD = 'CDA = 60°. DeterminaŃi măsu-


rile unghiurilor 'ABC şi 'BCD.
Etapa locală, Maramureş, 2006
2. În patrulaterul ABCD, AB = 6 3 , CD = 12, 'A = 60°, 'B = 150°, 'D = 90°. DeterminaŃi lun-
gimile laturilor BC şi AD.
Etapa locală, Arad, 2006
3. Să se arate că într-un patrulater care are două laturi opuse congruente, dreapta care uneşte mijloa-
cele celorlalte două laturi este paralelă cu bisectoarea unghiului format de primele două laturi.
Etapa locală, Argeş, 2006
4. Fie ABCD un paralelogram, în care se duc înălŃimile |BK| şi |BH|, K ∈ AD, H ∈ DC. Se dau
segmentele KH = a şi BD = b (b > a). Să se afle distanŃa de la punctul B la punctul de intersecŃie
al înălŃimilor triunghiului BKH.
Etapa locală, Braşov, 2005

47
5. Fie ABCD un pătrat de latură egală cu unitatea, iar în interiorul său o figură F. Să se arate că,
dacă distanŃa dintre oricare două puncte ale figurii F este diferită de 0,001, atunci aria figurii F
este mai mică de 0,34.
Etapa locală, NeamŃ, 2005
6. Într-un trapez ABCD cu baza mare AB = a, diagonalele au lungimi b, c, formează un unghi
ascuŃit de măsură θ. Să se calculeze lungimea laturii BC.
Etapa locală, Alba, 2005
7. Se consideră o dreaptă d şi două puncte A, B coplanare cu d. Să se determine linia poligonală de
lungime minimă AMNB, cu proprietatea că M, N ∈ d şi segmentul MN are o lungime dată a.
Etapa locală, Bihor, 2001
8. LocalităŃile A şi B sunt separate de două râuri. În ce locuri trebuie construite podurile MN şi PQ
peste aceste râuri dacă vrem ca drumul AMNPQB să fie cel mai scurt posibil? (Presupunem că
malurile fiecărui râu sunt două drepte paralele şi fiecare pod este perpendicular pe maluri.)
Etapa locală, Teleorman, 2004

8.2. Omotetia
1. Fie omotetia H(O,k) şi triunghiul ABC.
1
a) ConstruiŃi imaginea triunghiului ABC prin omotetia H(O,k) când O ≡ A şi k = sau când O
2
este centrul de greutate al triunghiului şi k = 2.
b) VerificaŃi dacă printr-o omotetie H(O,k) un triunghi isoscel se transformă în triunghi isoscel. Dar
un triunghi dreptunghic? Dar un triunghi echilateral?
Etapa locală, Vaslui, 2006
2. Fie patrulaterul convex ABCD şi funcŃia f: M ∈ (ABC) → M' ∈ (ABC), definită prin relaŃia
1
MM ' = (3AM + BM + CM + DM) .
5
a) Să se arate că există un punct O ∈ (ABC), cu proprietatea f(0) = O, adică O este punct fix pentru f.
b) Să se determine k, astfel încât f să fie omotetia de centru O şi de raport k.
Etapa locală, Sălaj, 2006
3. Fie f: E → E o funcŃie bijectivă a planului euclidian pe el însuşi şi k ∈ R – {0, 1} un număr real
cu proprietatea f(M)f(N) = k ⋅ MN , (∀) M, N ∈ E. Să se arate că f este o omotetie.
Etapa locală, Suceava, 2006
4. Fie triunghiul ABC şi G centrul său de greutate. Se consideră omotetiile H  3  (G ) = M ,
 A, 
 2
H 2 ( M) = P , H  3 (G ) = N . Să se arate că patrulaterul ABCD este paralelogram, unde D =
 A,   B, 
 5  2
= H(N,5)(P).
Etapa locală, NeamŃ, 2005

48
5. Fie triunghiul ABC şi M ∈ Int(∆ABC). E dau omotetiile H  4 ( M ) = N, H  3 ( N ) = P şi
 C,   B, 
 3  2
H ( A , 2) (P ) = Q . Să se arate că H ( G , 4 ) (M) = Q , unde G este centrul de greutate al triunghiului
ABC.
Etapa locală, Mureş, 2005
6. Fie ∆ABC ascuŃitunghic. Se consideră omotetiile H 4  (M) = B, H  3  (B) = N, H 2  (M) = P .
 C, −   A,   A, 
 3  4  3
Să se determine omotetia ce transformă punctul C în punctul Q, unde {Q} = NP ∩ AC.
Etapa locală, Bacău, 2005
AB
7. Fie punctele A1, B1, C1 pe laturile BC, CA respectiv AB ale triunghiului ABC, astfel încât 1 =
A1C
BC CA AB BC
= 1 = 1 = k şi A2, B2, C2 pe laturile B1C1, C1A1 respectiv A1B1 astfel încât 2 1 = 2 1 =
B1A C1B A2C1 B2 A1
C 2 A1 1
= = . Să se arate că triunghiurile ABC şi A2B2C2 sunt omotetice.
C 2 B1 k
Etapa locală, DâmboviŃa, 2004
8. Se consideră două cercuri concentrice C1(O, r1) şi C 2(O, r2), cu r1 > r2 > 0. Să se construiască o
coardă a cercului exterior, care să fie împărŃită de cercul interior în trei segmente congruente.
Etapa locală, Argeş, 2004

IX.1. AplicaŃiile calculului vectorial


vectorial în geometrie.
geometrie.
Demonstrarea unor teoreme celebre

1. a) (Teorema lui Menelaus). Fie un triunghi ABC. Dacă o dreaptă care nu trece prin vârfurile A,
MB NC PA
B, C taie dreptele BC, CA, AB în punctele M, N, P, atunci are loc relaŃia: ⋅ ⋅ = 1.
MC NA PB
b) (Reciproca Teoremei lui Menelaus). Fie un triunghi ABC şi trei puncte M ∈ BC, N ∈ CA şi
MB NC PA
P ∈ AB, diferite de vârfurile triunghiului. Dacă avem relaŃia ⋅ ⋅ = 1 , atunci punctele
MC NA PB
M, N, P sunt coliniare.
2. a) (Teorema lui Ceva). Fie un triunghi ABC şi trei puncte A' ∈ BC, B' ∈ CA şi C' ∈ AB, diferite
de vârfurile triunghiului. Dacă dreptele AA', BB', CC' sunt concurente, atunci avem relaŃia:
A' B B' C C' A
⋅ ⋅ = 1.
A ' C B' A C' B
b) (Reciproca Teoremei lui Ceva). Fie un triunghi ABC şi punctele A' ∈ BC, B' ∈ CA şi C' ∈ AB,
diferite de vârfurile triunghiului. Dacă:
i) două ceviene AA', BB', CC' sunt concurente;

49
A' B B' C C' A
ii) are loc relaŃia ⋅ ⋅ = 1 , atunci cevienele AA', BB' şi CC' sunt concurente.
A ' C B' A C' B
3. (Teorema lui Stewart). Dacă M este un punct pe latura BC a triunghiului ABC, atunci are loc
relaŃia: AB2 ⋅ MC + AC2 ⋅ BM = AM2 ⋅ BC + AB ⋅ AC ⋅ BC.
4. (Teorema lui Soons). Fie d o dreaptă în planul unui triunghi ABC şi A', B', C' respectiv,
proiecŃiile vârfurilor A, B, C pe dreapta d. Perpendicularele din A', B', C' respectiv, pe dreptele
BC, CA, AB sunt concurente într-un punct ω – ortopolul dreptei d faŃă de triunghi.
5. (Teorema lui Leibniz). Fie G centrul de greutate al triunghiului ABC. Pentru orice punct M din
AB2 + BC2 + CA2
planul triunghiului ABC este adevărată relaŃia: MA2 + MB2 + MC2 = + 3MG2 .
3
6. (Teorema lui Cannot). Fie triunghiul ABC şi punctele A1 ∈ BC, B1 ∈ AC, respectiv C1 ∈ AB.
Perpendicularele duse din punctele A1, B1, C1 pe laturile BC, AC, respectiv AB, sunt concurente
dacă şi numai dacă AC12 + BA12 + CB12 = BC12 + CA12 + AB12 .
7. (Teorema lui Euler-Apollonius). Fie A, B, C, D vârfuri consecutive ale unui paralelogram dacă
şi numai dacă AB2 + BC2 + CD2 + DA2 = AC2 + BD2.
8. (Teorema lui Pappus). a) Fie triunghiul ABC şi punctele M ∈ (BC), N ∈ (AC), P ∈ (AB) care
împart aceste segmente în acelaşi raport. Să se arate că triunghiurile ABC şi MNP au acelaşi
centru de greutate.
9. (Teorema lui Steiner-Lehmus). Un triunghi care are două bisectoare interioare egale (măsurate
de la vârf la latura opusă) este isoscel.
10. (Teorema lui Gauss asupra patrulaterului complet). Mijloacele diagonalelor unui patrulater
complet sunt coliniare.

IX.2. Probleme de algebră rezolvate


cu ajutorul calculului vectorial
2
a +b
1. Dacă a, b ∈ [–1, 1], să se arate că: 1 − a 2 + 1 − b 2 ≤ 2 1 −   .
 2 
Etapa locală, Vaslui, 2006
n +1 n +1 n +1
n
4 + 8 + 16
n n
6 + 8 + 12
2. Să se demonstreze că: > n +1
.
n
6 + n 8 + n 12 4 + n +1 8 + n +1 16
Etapa locală, Călăraşi, 2006
3. Fie a, b, c, d ≥ 0. Să se arate că: a + b + c + d + b + c + d + c + d + d ≥ a + 4b + 9c + 16d .
Etapa locală, ConstanŃa, 2005
 x + y + z = 14
2 2 2
4. Să se rezolve sistemul:  .
 x + 2 y + 3z = 14
Etapa locală, Tulcea, 2005

50
4 x − 6 y + 3z − t = 6

 x 2 + y 2 + z 2 + 2 t = 52
 81

5. Să se rezolve sistemul:  290 .
4 x + 5y + z − 3t = 81
2 2


3x 2 − 6 y + 2z + t 2 = 34
 9
Etapa locală, Argeş, 2005
6. Să se determine coeficienŃii a, b ∈ R ai ecuaŃiei: 2x4 – 12x3 + 27x2 + 5ax – b = 0, ştiind că ecuaŃia
are toate rădăcinile reale.
Etapa locală, Arad, 2005
7. Fie x, y, z, a ∈ R, astfel încât x + 2y + 3z = a. Să se afle a, astfel încât minimul sumei x2 + y2 + z2
să fie 14.
Etapa locală, Bihor, 2004
8. Fiind date numerele reale a, b, c, d, e, f care verifică relaŃiile:
a + b + c + d + e + f = 10
 .
(a − 1) + (b − 1) + (c − 1) + (d − 1) + (e − 1) + (f − 1) = 6
2 2 2 2 2 2

Să se determine valoarea maximă a lui f.


Etapa locală, Cluj, 2004

IX.3. Rezolvarea unor probleme de trigonometrie


cu metoda vectorială

1. Să se demonstreze vectorial următoarele identităŃi trigonometrice:


i) sin2 x + cos2 x = 1; ii) cos (x + y) = cos x ⋅ cos y – sin x ⋅ sin y, (∀) x, y ∈ R.
Etapa locală, Tulcea, 2002
2. Fie ABC un triunghi cu BC = a, AB = c, AC = b şi unghiurile notate 'A, 'B, 'C. Să se arate,
folosind metoda vectorială, că avem relaŃiile:
a b c
i) a = b ⋅ cos C + c ⋅ cos B; ii) = = .
ainA sin B sin C
Etapa locală, BistriŃa-Năsăud, 2002
3. Să se demonstreze vectorial că:
i) cos(x – y) = cos x ⋅ cos y + sin x ⋅ sin y;
ii) cos(x + y) = cos x ⋅ cos y – sin x ⋅ sin y;
iii) cos 2x = cos2 x – sin2 x, (∀) x, y ∈ R.
Etapa locală, Mureş, 2002

51
4. DemonstraŃi vectorial că în orice triunghi au loc inegalităŃile:
3 3
i) cos 2A + cos 2B + cos 2C ≥ − ; ii) cos A + cos B + cos C ≤ .
2 2
Etapa locală, Argeş, 2001
5. DemonstraŃi că pentru orice triunghi ABC pentru care avem condiŃia cos A + tg B + sin C > p,
a 2 + b2 + c2
atunci: ≤ cos 2 A + tg 2 B + sin 2 C , unde 2p = a + b + c.
3
Etapa locală, Arad, 2001
6. Să se demonstreze vectorial, că pentru orice α şi orice n > 2, n ∈ N, avem:
 2π   4π   2(n − 1)π 
cos α + cos α +  + cos α +  + ... + cos α +  = 0.
 n   n   n 
Etapa locală, Bihor, 2001
sin a
7. Fie arcul AM având măsura algebrică a. Să se arate, folosind o omotetie, că tga = , ctg a =
cos a
cos a
= .
sin a
Etapa locală, Covasna, 2000

IX.4. AplicaŃiile trigonometriei în rezolvarea


problemelor de geometrie
1. Să se arate că dacă într-un triunghi două bisectoare sunt egale, atunci triunghiul este isoscel.
Etapa locală, Braşov, 2005
2. Într-un cerc de rază R se consideră două coarde perpendiculare AC şi BD. Să se arate că AB +
+ CD > 2R.
Etapa locală, IalomiŃa, 2005
3. În figura următoare să se arate că:
A1C ⋅ B1D ⋅ C1E ⋅ D1A ⋅ E1B = A1D ⋅ B1E ⋅ C1A ⋅ D1B ⋅ E1C.
Etapa locală, Călăraşi, 2005
B1

A1 D E
C1
C A
B
E1
D1

52
4. GăsiŃi unghiurile unui triunghi, aflate în progresie aritmetică, dacă lungimile laturilor sunt:
a) în progresie aritmetică;
b) în progresie geometrică.
Etapa locală, Vaslui, 2005
π
5. Cel puŃin unul din unghiurile unui triunghi ABC poate fi cel mult dacă a2, b2, c2 sunt în pro-
3
gresie aritmetică, indiferent de ordine.
Etapa locală, Harghita, 2005
6. Să se arate că două triunghiuri care au o latură comună, au acelaşi perimetru, au aceeaşi arie sunt
congruente.
Etapa locală, Ilfov, 2005
7. Fie cercul C(O, R), un diametru AB şi un punct C ∈ (OA) şi M, N două puncte pe cerc, astfel
π
încât m('MCA) = m('NCA) = . Să se arate că MC = NC.
3
Etapa locală, Mureş, 2005
8. a) Să se arate că dacă A, B, C sunt măsurile unghiurilor unui triunghi oarecare, există triunghiuri
A B C
ale căror laturi să fie exprimate prin cos , coa , cos .
2 2 2
b) Dacă triunghiul ABC este ascuŃitunghic, atunci sin 2A, sin 2B, sin 2C pot exprima lungimile
laturilor unui triunghi.
Etapa locală, Prahova, 2004
9. Fie M un punct în interiorul triunghiului echilateral ABC de latură 1. Să se arate că MA ⋅ MB +
+ MB ⋅ MC + MC ⋅ MA ≥ 1.
Etapa locală, Argeş, 2003
10. Fie G centrul de greutate al triunghiului ABC. Ştiind că m('GAB) + m('BGC) + m('GCA) =
= 90°, să se arate că triunghiul este isoscel.

IX.5
IX.5. AplicaŃiile trigonometriei în algebră
1+ x 1+ y 1+ z
1. Să se arate că + + = 2 3 dacă şi numai dacă x = y = z = 1,
1+ x + x 2
1+ y + y 2
1 + z + z2
x, y, z ≥ 0.
Etapa locală, IalomiŃa, 2007
2. Dacă a, b ∈ [0, 2], să se demonstreze egalitatea:
2 2
a b 4 − a 2 − b2   4 − a 2 − b2 
1−  +  + 1−  a − b  = b.
2 2 a 2 + b2  2 2 a 2 + b2 
   
Etapa locală, Călăraşi, 2007

53
3. DeterminaŃi toate funcŃiile f: N* → Q* cu proprietatea 1 + f(n)f(n + 1) = 2n2 ⋅ [f(n + 1) – f(n)],
(∀) n ∈ N*.
Etapa locală, Maramureş, 2007
4. Se dă funcŃia f: R → R, f ( x ) = 1 + x − x . Să se demonstreze că dacă x > 0, y > 0, z > 0, xy +
2

+ yz + zx = 1, atunci: f(x) ⋅ f(y) + f(y) ⋅ f(z) + f(z) ⋅ f(x) = 1.


Etapa locală, Timiş, 2007
3 + 2 1− x 2
5. Să se determine imaginea funcŃiei f: [–1, 1] → R, f ( x ) = .
1+ x + 1− x
Etapa locală, Suceava, 2007
6. Se dă mulŃimea E = {x ∈ [0, 1] | 8x(1 – 2x2) ⋅ (8x4 – 8x2 + 1) = 1}. Să se determine card E.
Etapa locală, BistriŃa-Năsăud, 2006
7. Să se rezolve în mulŃimea numerelor reale pozitive sistemul:
x + y + z = xyz

 x +
y
+
z
=
3 3.
 2
 1+ x 1+ y 1+ z
2 2 2

Etapa locală, Braşov. 2007


1− a 1− b
2
1− c 2
3 2
8. Fie a, b, c > 0, ab + bc + ca = 1. Să se arate că + + ≤ .
1+ a 2
1+ b 2
1+ c 2
2
Etapa locală, Bacău, 2006
9. Să se arate că din oricare 13 numere pozitive, cel puŃin două numere x şi y satisfac relaŃiile:
x−y 2− 3
0≤ ≤ .
1 + xy 2+ 3
Etapa locală, Vrancea, 2006
2
10. Fie a, b, c ∈ R astfel încât abc + a + c = b. DeterminaŃi maximul expresiei E(a, b, c) = 2 −
a +1
2 3
− 2 + 2 .
b +1 c +1
Etapa locală, DâmboviŃa, 2006

X.1. Probleme care folosesc inducŃia matematică


în geometrie
1. Într-un plan se consideră o dreaptă d şi un punct O pe dreapta d, iar OP1 , OP2 , ..., OPn vectori
de lungime 1 în acest plan de aceeaşi parte a dreptei d. Dacă n este impar, să se arate că
OP1 + OP2 + ... + OPn ≥ 1 .
Etapa locală, Timiş, 2010

54
2. Se consideră şirul (an)n≥1 cu a1 = 3 şi an = n(an–1 – 1) + 2, n ≥ 2. Să se arate că dacă se consideră an
puncte din plan, oricare trei necoliniare şi se unesc între ele cu segmente de n culori, se formează
cel puŃin un triunghi cu laturile de aceeaşi culoare.
Etapa locală, MehedinŃi, 2010
3. Pe o dreaptă se consideră n ≥ 2 segmente astfel încât oricare două au un punct comun. Demon-
straŃi că toate cele n segmente au un punct comun.
Etapa locală, Bacău, 2010
4. Fie n ≥ 3 puncte necoliniare în plan. Să se arate că există cel puŃin n drepte distincte, astfel încât
fiecare să treacă măcar prin două din cele n puncte.
Etapa locală, Bihor, 2010
5. Fie 2n, n ≥ 1, puncte distincte situate în interiorul unui poligon convex. Atunci poligonul se poate
împărŃi în n + 1 poligoane convexe, astfel încât toate cele 2n puncte să se găsească pe frontierele
acestor poligoane.
Etapa locală, Vrancea, 2009
 1 1
6. Se consideră an = 1 + n! ⋅ 1 + + ... +  puncte în plan, oricare trei necoliniare şi se colorează
 2! n! 
arbitrar segmentele ce le unesc cu n culori. Să se arate că oricare ar fi n ∈ N* se formează cel
puŃin un triunghi cu laturile de aceeaşi culoare.
Etapa locală, Cluj, 2009
7. Să se arate că numărul părŃilor în care planul este împărŃit de n drepte, n ≥ 1, două câte două
n2 + n + 2
secante şi trei câte trei neconcurente, este Fn = .
2
Etapa locală, NeamŃ, 2009

X.2. Probleme de geometrie combinatorică

1. Punctele de pe suprafaŃa unui hexagon regulat de latură 1 se colorează cu 6 culori. Să se arate că


există două puncte de aceeaşi culoare între care distanŃa este cel puŃin 1.
Etapa locală, Dolj, 2010
2. Punctele unui cerc de rază 1 se colorează cu trei culori. Să se arate că pentru orice culoare există
două puncte de aceeaşi culoare între care distanŃa este d > 1,7.
Etapa locală, Braşov, 2010
3. Să se arate că oricum am partiŃiona planul în n mulŃimi, există patru puncte în aceeaşi mulŃime
care sunt vârfurile unui dreptunghi.
Etapa locală, Bacău, 2010
4. Vârfurile unui poligon regulat sunt colorate 2n în roşu şi 2n în negru. Să se arate că există două
poligoane cu n + 1 vârfuri, unul negreu şi unul roşu, congruente.
Etapa locală, BistriŃa-Năsăud, 2010

55
5. Se colorează punctele planului în 4 culori, folosind toate culorile. Să se arate că există o dreaptă
pe care se găsesc cel puŃin 3 culori.
Etapa locală, Sibiu, 2010
6. Fiecare faŃă a unui cub este colorată cu o culoare diferită dintre şase culori date. Câte colorări
distincte există?
Etapa locală, Bihor, 2010
7. În orice poligon convex cu 2n laturi există o diagonală ce nu este paralelă cu nicio latură.
Etapa locală, Satu-Mare, 2010

X.3. Probleme de extrem


1. Fie ABC un triunghi oarecare şi M, N, P trei puncte arbitrare pe laturile triunghiului, M ∈ (BC),
N ∈ (AC) şi P ∈ (AB). Prelungim semidreptele (AM, (BN şi (CP până ce ele intersectează cercul
circumscris triunghiului în punctele T, Q şi respectiv R.
AM BN CP
a) Să se arate că: + + ≥9.
MT QN PR
1 1 1
b) Notând S1 = σ(BTC), S2 = σ(CQA), S3 = σ(ARB), să se determine minimul sumei + + .
S1 S2 S3
Etapa locală, NeamŃ, 2010, prof. A. Sandovici
2. Fie ABCD un patrulater convex cu d1, d2 diagonalele sale, circumscris unui cerc de rază r. Să se
determine valoarea minimă a produsului d1 ⋅ d2.
Etapa locală, Caraş-Severin, 2010
a+b+c
3. Fie ABC un triunghi în care notăm a = BC, b = AC, c = AB, p = . Dacă M este un punct
2
arbitrar pe cercul circumscris triunghiului ABC, iar I este centrul cercului înscris, să se arate că
pMI < max{aMA, bMB, cMC}.
Etapa locală, Vâlcea, 2010
4. Fie un patrulater convex ABCD. Să se determine punctul M din planul patrulaterului astfel încât
suma MA + MB + MC + MD să fie minimă.
Etapa locală, Giurgiu, 2010
5. Fie patrulaterul convex ABCD. Să se determine un punct M în interiorul lui, astfel încât suma
MA2 + MB2 + MC2 + MD2 să fie minimă.
Etapa locală, Arad, 2010
6. Să se determine poziŃia punctului M astfel încât dacă M este situat în planul ∆ABC, suma MA +
+ MB + MC este minimă.
Etapa locală, IalomiŃa, 2010
7. Să se demonstreze că suma pătratelor distanŃelor de la un punct M din planul ABC la vârfurile
acestuia, este minimă dacă şi numai dacă M ≡ G, unde G este centrul de greutate al ∆ABC.
Etapa locală, Maramureş, 2010

56
X.4. Locuri geometrice

1. În triunghiul ABC pentru care lungimea laturii AB este diferită de lungimea laturii AC, consi-
derăm M mijlocul laturii BC. Fie P, Q puncte variabile pe AB respectiv AC, astfel încât BP = CQ
şi R mijlocul lui PQ.
a) Să se arate că RM este paralelă cu bisectoarea unghiului 'BAC.
b) Să se afle locul geometric al punctului R.
Etapa locală, Botoşani, 2010
2. Fie punctele O, A fixate în plan şi M un punct variabil. Se cere locul geometric al punctelor M
pentru care OA ⋅ OM = k .
Etapa locală, BistriŃa-Năsăud, 2010
3. Fie punctele fixe A, B în plan şi M mobil. Se cere locul geometric al punctului M astfel încât
MA ⋅ MB = k .
Etapa locală, Bihor, 2010
4. Fie dreptunghiul ABCD cu lungimea diagonalei a.
a) Fie m ∈ R* şi fie Gm baricentrul sistemului de puncte A(m), B(–1), C(1). Să se precizeze
poziŃia lui G1 şi să se determine mulŃimea punctelor Gm, m ∈ R*.
b) Să se determine mulŃimea punctelor M din planul dreptunghiului pentru care:
MA − MB + MC = a .
Etapa locală, Cluj, 2010
5. Fie în planul P pătratul ABCD de latură a şi fie transformarea f: P → P, f(M) = 3MA + MB −
− 2MC − 2MD . Să se demonstreze că f(M) este un vector fix şi să se determine mulŃimea punc-
telor M ∈ P, pentru care MA + MB + MC + MD = f (M ) .
Etapa locală, DâmboviŃa, 2010
6. GăsiŃi locul geometric al punctului M astfel încât MA = k MB , k ≠ 1.
Etapa locală, Gorj, 2010
7. Fie punctele A, B, C, D fixe, astfel încât oricare trei puncte sunt necoliniare şi M mobil în plan.
Se cere locul geometric al punctului M, astfel încât MA ⋅ MB + MC ⋅ MD = k , k ∈ R.
Etapa locală, IalomiŃa, 2010

X.5. Probleme de construcŃii geometrice


1. Se dă unghiul 'xOy. ConstruiŃi un unghi ce are o latură Uz dată şi este congruent cu 'xOy.
Folosind acest fapt, construiŃi printr-un punct exterior unei drepte date d, paralela d' cu dreapta d.
Etapa locală, BistriŃa-Năsăud, 2008

57
ab
2. i) Fiind date segmente de lungimi a, b, c, să se construiască segmenul x = (al patrulea pro-
c
porŃional).
ii) Fiind date segmente de lungimi a, b, să se construiască un segment y, astfel încât y2 = ab (y se
numeşte medie geometrică a segmentelor a şi b).
Etapa locală, Satu-Mare, 2010
3. Se dă o dreaptă d, un punct P şi două numere pozitive, a şi r. Să se construiască un cerc de rază r,
care conŃine punctul P şi care intersectează dreapta d în două puncte la distanŃa a.
Etapa locală, Bacău, 2010
4. Se consideră două cercuri C 1(O1, r1) şi C 2(O2, r2) şi un vector v , situate în acelaşi plan. Să se
determine un punct M ∈ C 1(O1, r1) şi un punct N ∈ C 2(O2, r2), astfel încât MN = v .
Etapa locală, Caraş-Severin, 2010
5. Fie cercurile coplanare C 1(O1, r1) şi C 2(O2, r2) şi o dreaptă d în planul lor. Să se construiască o
dreaptă paralelă cu d, care determină coarde congruente în cele două cercuri.
Etapa locală, Gorj, 2010
6. Se dă o dreaptă (d), un cerc C şi un punct A pe cerc. Să se construiască un cerc tangent cercului
C în A şi tangent dreptei (d).
Etapa locală, Mureş, 2010
7. Fie C (O, r) situat în interiorul unghiului 'ABC. Să se construiască un cerc înscris în unghi şi
tangent cercului dat.
Etapa locală, Tulcea, 2010

X.6. Alte tipuri de probleme


AM m
1. Fie ABC un triunghi şi punctele M, N pe laturile [AB] respectiv [BC], astfel încât = ,
MB n
BN n
= , unde m, n, p sunt numere reale strict pozitive, astfel încât p2 = mn. Notăm cu P inter-
NC p
secŃia dreptelor CM şi AN. ArătaŃi că n PA + m PB + p PC = 0 .
Etapa locală, Sibiu, 2010
2. În triunghiul ABC, fie P, Q, R punctele de tangenŃă ale cercului înscris în ∆ABC. DemonstraŃi că
AP + BQ + CR = 0 dacă şi numai dacă triunghiul ABC este echilateral.
Etapa locală, Suceava, 2010
3. Se consideră triunghiul ABC şi punctele D, Q, E cu proprietăŃile D ∈ (AB), Q ∈ (CD), C ∈ (BE).
Se mai consideră şi punctele F, M astfel încât CE || DF, DC || FE, DE ∩ FQ = {M}. Să se arate că
AB DE
2+ = .
AD DM
Etapa locală, Bihor, 2010, RMT

58
4. Fie ABC un triunghi, M un punct interior triunghiului ABC, astfel încât 'MAB = 10°, 'MBA =
= 20°, 'MCA = 30°, 'MAC = 40°. DemonstraŃi că triunghiul ABC este isoscel.
Etapa locală, Dolj, 2010
5. Cercul înscris în triunghiul ABC este tangent la laturile BC, CA şi AB în punctele D, E şi F. Fie
K punctul diametral opus lui D pe cercul înscris, iar {M} = DF ∩ KE. ArătaŃi că ∆AFM este
isoscel.
Etapa locală, MehedinŃi, 2010
6. În ∆ABC se duce bisectoarea AD, D ∈ BC şi M = prABD, N = prADM. Dacă 3MD = AD + 2ND,
să se arate că m('BAC) = 60°.
Etapa locală, Alba, 2010, Gazeta Matematică, 6/2009
7. În triunghiul ABC se consideră punctele D, E, F – mijloacele laturilor AB, BC, AC şi M ∈ (BE),
BM CN
N ∈ (CE). ArătaŃi că AE, DM, FN sunt concurente dacă şi numai dacă = ≠1.
ME NE
Etapa locală, Prahova, 2009, prof. I. Nedelcu
8. Fie triunghiul ABC şi punctele D – piciorul bisectoarei din A, I1 şi I2 – centrele cercurilor înscrise
în triunghiurile ABD, respectiv ACD. Se ştie că BC = 12, AD = 8 şi I1I2 || BC.
a) DemonstraŃi că AB = AC.
b) Dacă AI1 = u , AI2 = v , exprimaŃi AB şi AC în funcŃie de u şi v .
Etapa locală, Prahova, 2009, prof.
9. DemonstraŃi că segmentele ce unesc vârfurile triunghiului cu punctele de contact ale cercului
înscris cu laturile triunghiului sunt concurente.
Etapa locală, Alba, 2009
sin 53 − 2 cos 40  sin 13
10. Să se calculeze expresia E = .
cos 63
Etapa locală, GalaŃi, 2009, prof. V. Grigore
11.
11. Fie paralelogramul ABCD, E un punct pe diagonala (BD), diferit de centrul paralelogramului şi C'
simetricul lui C faŃă de E. Paralela prin C' la AD intersectează dreapta AB în punctul F, iar
paralela prin C' la AB intersectează AD în punctul G. Să se arate că:
a) EF || AC;
b) punctele E, F, G sunt coliniare.
Etapa locală, Vrancea, 2009, prof. D. Sârghie
12. Să se demonstreze că în orice triunghi dreptunghic ABC cu m('A) = 90°, are loc
1 1 1
+ ≥ .
1 + 3tgB 1 + 3ctgB 2
Etapa locală, Mureş, 2009
13. Fie |AB| = a şi omotetia h 1 : P → P, O ∉ AB. Dacă h 2 (A) = A' , h 2 (B) = B' , calculaŃi
O, O, O,
3 3 3
lungimea segmentului A'B'.
Etapa locală, Vrancea, 2009

59
14.
14. Pe un cerc considerăm punctele A, B, C, D. Fie P mijlocul coardei [AB] şi Q mijlocul coardei
[CD]. Să se arate că |AC| + |BD| ≥ 2|PQ|.
Etapa locală, Sălaj, 2009
3
15. a) DemonstraŃi că, oricare ar fi x ∈ R, sin3 x ⋅ cos 3x + cos3 x ⋅ sin 3x = sin 4x .
4
b) DeterminaŃi toate numerele naturale n pentru care expresia
n
E(x) = sinn xcos nx + cosn x ⋅ sin nx − sin nx
4
este independentă de x.
Etapa locală, Teleorman, 2008
30
16. Se consideră în plan 30 de discuri disjuncte D1, D2, ..., D30. Notăm cu D = ∪ D k şi fie S aria lui
k =1
1 1
D. Să se arate că < S < , ştiind că proiecŃiile mulŃimii D pe axe sunt două segmente având
160 4
suma lungimilor egală cu 1.
Etapa locală, NeamŃ, 2009

60
CLASA A X-
X-A

ALGEBRĂ

I. MulŃimi de numere

I.1. MulŃimea numerelor reale

I.1.1. PUTERI ŞI RADICALI

I.1.1.1. OperaŃii. Numere raŃionale şi iraŃionale. Puteri. IdentităŃi

1. DemonstraŃi că 3 7 − a 2 + 3 7 + a 2 = 2 dacă şi numai dacă a ∈ {–5, 5}.


Etapa locală, Hunedoara, 2010
2. Fie a, b ∈ R+ astfel încât a = 4, b = 3, m = min {a, b}, M = max {a, b}. Dacă H = {x ∈ Z | m <
3 4

< 6 x < M} , determinaŃi S = k.∑


k ∈H
Etapa locală, Vrancea, 2010
3. Fie a, b, c numere reale strict pozitive. ArătaŃi că dacă 2a = 35, 5b = 14, 7c = 10, atunci abc – (a +
+ b + c) = 2.
Etapa locală, Mureş, 2010
n
2k + 1
4. Să se determine partea întreagă a numărului real A = ∑ 2 k +1 , n ∈ N*.
k =1 2k − 1
Etapa locală, GalaŃi, 2010, prof. V. GhiŃă
5. DeterminaŃi a, b ∈ (0, 1) astfel încât 3 a − b = 3 b − a şi a + 1 = 33 a 2 (1 − b) .
Etapa locală, NeamŃ, 2010, prof. G. Ciorăscu
 1  1  1  1
6. a) CalculaŃi suma S = 1 −  +  2 −  +  3 −  + ... +  n −  .
 4  4  4  4
b) DemonstraŃi identitatea:
5 9 13 4n + 1 n (2n + 1) n (n + 1) n
−1 + −2 + − 3 + ... + −n + = + .
4 4 4 4 4 2 4
Etapa locală, Cluj, 2010
7. Să se afle suma S = x + y + z + u + v ştiind că x + y + 2 z − 2 + u + v = x + y + z + u + v .
Etapa locală, Tulcea, 2010

61
I.1.1.2. Ordonare. InegalităŃi

 π
1. Fie x ∈  0,  şi numerele reale m, n, nenule, astfel încât (cos x)m+1 = sin x şi (sin x)n+1 = cos x.
 2
ComparaŃi numerele reale m şi n.
Etapa locală, Sibiu, 2010, prof. I. Aleman, prof. M. Macrea
2.
n
Să se determine cel mai mic număr natural m pentru care 3 ⋅ 4 3 ⋅ 8 3 ⋅ ... ⋅ 2 3 < m , (∀) n ∈ N*.
Etapa locală, Caraş-Severin, 2010
ab + bc + ca a + b + c b c a
3. Fie a, b, c ∈ N – {0, 1}. DemontraŃi că ≥ a bc .
a+b+c
Etapa locală, Braşov, 2010, prof. G. Boeriu
4. Fie n ∈ N*. Să se compare numerele:
a n = ( 2 + 1)( 3 − 2 ) ⋅ ... ⋅ ( 2n + 2n − 1)( 2n + 1 − 2n ) ,
b n = ( 2 − 1)( 3 + 2 ) ⋅ ... ⋅ ( 2n − 2n − 1)( 2n + 2n ) .
Etapa locală, BistriŃa-Năsăud, 2010
5. Să se arate că pentru orice n ≥ 6, n ∈ N are loc inegalitatea n
2 + 3n > n 3 + 2 n .
Etapa locală, MehedinŃi, 2010
6. Să se arate că x 2 − 2 x sin z − 4 cos z + 5 + y 2 − 2 y sin z − 6 cos z + 10 ≥ 3 , (∀) x, y, z ∈ R.
Etapa locală, Suceava, 2010
7. Să se arate că 2n + m n 2 m ⋅ m 2 n ≤ n 2 + m 2 , (∀) m, n ∈ N.
Etapa locală, Bacău, 2010

I.1.2. LOGARITMI

I.1.2.1. OperaŃii. IdentităŃi cu logaritmi


x y z x y z
1. Să se arate că dacă x, y, z > 0, atunci lg3 + lg3 + lg3 = 3 ⋅ lg ⋅ lg ⋅ lg .
y z x y z x
Etapa locală, Satu-Mare, 2010
2. Să se arate că oricare ar fi x, y, z ∈ (0, 1) are loc egalitatea:
 x  y  z  x  y  z 
tg lg  + tg lg  + tg lg  = tg lg  ⋅ tg lg  ⋅ tg lg  .
 y  z  x  y  z  x
Etapa locală, Arad, 2010
3. CalculaŃi:
x lg y
a) E = , unde x, y ∈ (0, ∞) – {1};
y lg x

62
b c a
lg lg lg
b) P = a c ⋅b a ⋅c b , unde a, b, c ∈ (0, ∞) – {1}.
Etapa locală, Mureş, 2010
4. Să se arate că expresia:
1 1
E = logsin x cos x + log cos x tg x + log tgx + log ctgx + log 1 cos x + log 1 sin x , unde x ∈
cos x sin x sin x cos x

 π
∈  0,  , nu depinde de x.
 2
Etapa locală, IalomiŃa, 2010
5. Fie a, b, c lungimile laturilor unui triunghi dreptunghic în C. Să se arate că dacă c – b ≠ 1 şi c + b ≠
≠ 1, atunci: log c + b a + logc − b a = 2 log c + b a ⋅ log c − b a .
Etapa locală, Ilfov, 2010
6. Să se arate că (∀) a, b ∈ (0, ∞) – {1} avem:
log a b 3 log a b n log a b log b a 3 log 2b a n log nb −1 a
a +a + ... + a =b +b + ... + a .
Etapa locală, DâmboviŃa 2010
7. Să se arate că oricare ar fi n ∈ N*, există a1, a2, ..., an ∈ Q ∩ (1, 2] astfel încât lg(n + 1) = lg a1 +
+ lg a2 + ... + lg an.
Etapa locală, Botoşani, 2010
8. ScrieŃi log308 în funcŃie de log330 şi log30 5.
Etapa locală, Cluj, 2010
 n   n 
9. Fiind date n, k ∈ N, k ≥ 2, să se demonstreze că  =  , unde {x}
 log n (k + 1) − 1  1 − log k +1 k 
reprezintă partea fracŃionară a numărului real x.
Etapa locală, Brăila, 2010, prof. M. Damian
10. ArătaŃi că numărul log n (n + 1) + log n +1 n nu este întreg, (∀) n ∈ N, n ≥ 2.
Etapa locală, Sălaj, 2010

I.1.2.2. Logaritmi. Ordonare. InegalităŃi

1. Dacă a, b, c ∈ (0, 1), sau a, b, c ∈ (1, ∞), iar k ∈ (0, 2], atunci să se arate că:
1 1 1 2
+ + ≤ .
k + log a b k + log b c k + log c a k
Etapa locală, Buzău, 2010
2. Fie x1, x2, ..., x2011 ∈ (1, ∞) astfel încât x12010 > x 2 ⋅ x 3 ⋅ ... ⋅ x 2011 . Să se demonstreze că:
lg 2010 x1 > (lg x 2 )(lg x 3 )...(lg x 2011 ) .
Etapa locală, Arad, 2010
3. Să se compare numerele lg (5 + 35 ) şi lg(6 + 35 ) . 2

Etapa locală, Teleorman, 2010

63
4. Să se demonstreze că dacă a, b şi c sunt numere reale mai mari decât 1, atunci:
log a b log b c log c a 9
+ + ≥ .
a+b b+c c + a 2( a + b + c )
Etapa locală, Harghita, 2010
5. Dacă x1, x2, x3, ..., xn ∈ (1, 2], atunci: log x1 (3x 2 − 2) + log x 2 (3x 3 − 2) + ... + log x n −1 (3x n − 2) +
+ log x n (3x1 − 2) ≥ 2n .
Etapa locală, Argeş, 2010
6. DemonstraŃi inegalitatea log3(1 + a + b) ⋅ log3(1 + b + c) ⋅ log3(1 + c + a) ≤ 1, ştiind că a, b, c > 0
şi a + b + c = 3.
Etapa locală, Prahova, 2010, prof. M. Doinaru
2 log n ( n +1) (n − 1)(n 2 + 4n + 6)
7. Să se demonstreze că: 2
2 log 2 3
+3
2 log 3 4
+ ... + n < , (∀) n ∈ N, n ≥ 2.
3
Etapa locală, Cluj, 2010
8. 2 2
Să se arate că lg 5 + lg 7 > lg12.
Etapa locală, Vrancea, 2010
n −1 n −1
9. Să se arate că log a b + log b a ≥ loga b + log b a .
n n

Etapa locală, BistriŃa-Năsăud, 2010


10. Fie a, b, c ∈ N – {0, 1}. DemonstraŃi inegalitatea:
ab + bc + ca ab + bc + ca ab + bc + ca
log a + log b + log c ≥3.
a+b+c a+b+c a+b+c
Etapa locală, Braşov, 2010, prof. G. Boeriu

I.1.3. METODE NESTANDARD DE REZOLVARE A PROBLEMELOR

I.1.3.1. Metoda trigonometrică


3 + cos 2 A
1. Să se arate că în orice triunghi ABC, ≥ cos 2 B + cos 2 C .
2
Etapa locală, Harghita, 2010
 2π  3A 3B 3C
2. În triunghiul ABC avem A, B, C ∈  0,  . ArătaŃi că tg + tg + tg ≥ 3.
 3  4 4 4
Etapa locală, Vaslui, 2010, prof. C. Alistar
3. Să se arate că în orice triunghi ascuŃitunghic are loc următoarea inegalitate:
a2 b2 c2 a ( b + c ) b (c + a ) c (a + b )
+ + ≥ + + .
cos A cos B cos C 2 cos A 2 cos B 2 cos C
Etapa locală, NeamŃ, 2010, prof. A. Sandovici
27 r
4. Să se arate că în orice triunghi ABC există inegalitatea: sin A + sin B + sin C ≥ ⋅ .
2 p
Etapa locală, Sibiu, 2010
64
sin A sin B sin C
5. Să se demonstreze că în orice triunghi ABC avem: 3
+ 3 + 3 ≥4.
sin B sin C sin A
Etapa locală, Arad, 2010
6. Să se arate că dacă între unghiurile unui triunghi există relaŃia 2A + 3B = 180°, atunci între
laturile sale există inegalitatea 4(a + b) ≤ 5c.
Etapa locală, Tulcea, 2010
7. Într-un triunghi ABC inegalitatea a > b implică a + h a ≤ b + h b , unde n ∈ N. În ce caz există
n n n n

egalitate?
Etapa locală, Teleorman, 2010

I.1.3.2. Metoda vectorială

x 6 + y 6 + z 6 = 3

1. Să se studieze compatibilitatea sistemului şi să se rezolve: x 8 + y8 + z8 = 3 .
 7
x + y + z = 3
7 7

Etapa locală, Tulcea, 2010


2. Să se demonstreze că dacă t ∈ R, atunci max t 2 − 24 t + 169 − t 2 + 6t + 25 = 226 .
Etapa locală, Argeş, 2010
x + y = − y(z + x )
2 2


3. Să se rezolve sistemul: x 2 + x + y = −2 yz .
 2
3x + 8y + 8xy + 8 yz = 2 x + 4z + 2
2

Etapa locală, DâmboviŃa, 2010


4. Să se demonstreze că C kn −1 ⋅ C kn + C kn ⋅ C kn +1 < C kn ++11 .
Etapa locală, Harghita, 2010
5. Să se demonstreze că oricare ar fi numerele reale a1, a2, a3, a4, a5, a6, a7, a8 cel puŃin una din
expresiile numerice: a1a3 + a2a4; a1a5 + a2a6; a1a7 + a2a8; a3a5 + a4a6; a3a7 + a4a8; a5a7 + a6a8 este
nenegativă.
Etapa locală, Timiş, 2010
6. Se dau numerele pozitive a1, a2 astfel încât a1 + a2 = 4. Să se arate că:
4a1 + 1 + 4a 2 + 1 ≤ 6 .
Etapa locală, Tulcea, 2010
4 x + 4 y + 4z + 17 = 8x + 4 y + 16z
2 2 2
7. Să se rezolve sistemul:  .
3x − 4 y + 12z = 12
Etapa locală, Braşov, 2010

65
I.1.3.3. Alte metode

1. Fie n un număr întreg.


a) Dacă r este restul împărŃirii lui n3 la 9, arătaŃi că r ∈ {0, 1, 8}.
b) DemonstraŃi că [3 9n + 1] = [3 9n + 7 ] , unde [x] reprezintă partea întreagă a numărului real x.
Etapa locală, Prahova, 2010, prof. C. Apostolescu
2. Fie [x] + [3x] + [6x] + [9x] + [18x] + [36x] = 14305. RezolvaŃi ecuaŃia, x ∈ R.
Etapa locală, Călăraşi, 2010
3. Să se determine numerele naturale n pentru care 23n + 2 şi 32n + 1 sunt simultan divizibile cu 10.
Etapa locală, Vrancea, 2010
4. Să se arate că (∀) x ∈ R, (∃) m ∈ {0, 1, 2, ..., 2010}, astfel încât [2011x] = 2011[x] + m.
Etapa locală, Suceava, 2010
5. Fie a ∈ N* aşa încât 2 - 2 este divizibil cu a. Definim şirul (xn)n≥1 prin formulele: x1 = a, xk+1 =
a

=2
xk
− 1 . Să se demonstreze că x k divide 2 xk − 2 , (∀) k ∈ N*.
Etapa locală, Vaslui, 2010
6. Să se găsească numărul t şi cifra cu care trebuie înlocuită litera a în următoarea egalitate:
[3(1 + t)]1999 = 3⋅ (a 29)333 .
Etapa locală, Alba, 2010

I.1.4. IDENTITĂłI TRIGONOMETRICE. SUME ŞI PRODUSE. INEGALITĂłI


n
1 n
1. Să se arate că ∑ arg tg k
k =1
2
+ k +1
= arctg
n+2
, n ∈ N*.

Etapa locală, Arad, 2010


sin x + cos x sin x cos x
4 4
3 3
2. Fie A = 3 − . Să se arate că A ≤ .
4 2 2
Etapa locală, Cluj, 2010, prof. V. Gorcea
3. AflaŃi valoarea maximă pe care o putea lua expresia: E = sin x1 ⋅ cos x2 + sin x2 ⋅ cos x3 + ... +
+ sin xn–1 ⋅ cos xn + sin xn ⋅ cos x1, n ∈ N, n ≥ 2, când x1, x2, ..., xn variază în mulŃimea R.
Etapa locală, Gorj, 2010
n
1
4. Să se calculeze suma: Sn = ∑
k =1 cos x − cos( 2 k + 1) x
.

Etapa locală, Olt, 2010


4 4
sin x cos x 1
5. Ştiind că avem + = , pentru ab > 0, să se arate că:
a b a+b
sin 2011 x cos 2011 x a+ b
2011
+ 2011
= .
a b (a + b) 2011
Etapa locală, Vâlcea, 2010

66
3
6. Fie x, y, z ∈ R astfel încât sin x + sin y + sin z ≥ . Să se arate că:
2
 π  π  π
sin  x −  + sin  y −  + sin  z −  ≥ 0 .
 6  6  6
Etapa locală, Botoşani, 2010
1 1 1 1 cos1
7. Să se demonstreze că: + + + ... + = .
cos 0 cos1 cos1 cos 2 cos 2 cos 3 cos 88 cos 89 sin 2 1
Etapa locală, Teleorman, 2010

I.2. MulŃimea numerelor complexe

I.2.1. PROPRIETĂłI. OPERAłII

(r + z1 )(r + z 2 )(r 2 + z1z 2 )


1. Fie z1, z2 ∈ C* de modul r. Să se arate că numărul r 2 + este real
z1z 2
nenegativ.
Etapa locală, Suceava, 2010
z12 z 22 z 32
2. Fie z1, z2, z3 ∈ C* astfel încât |z1| = |z2| = |z3| = 1 şi + + = −1 . Atunci:
z 2z3 z1z 3 z1z 2
a) să se arate că z13 + z 32 + z 33 = (z1 + z 2 + z 3 )(z12 + z 22 + z 32 − z1z 2 − z1z 3 − z 2 z 3 ) + 3z1z 2 z 3 ;
b) să se determine valorile lui S = |z1 + z2 + z3|.
Etapa locală, Vâlcea, 2010
3. Fie z1, z2, z3 ∈ C cu z1 + z2 + z3 = 0 şi |z1| = a, |z2| = b, |z3| = c. Ştiind că a + b2 = c2, arătaŃi că:
2

b 2 z12 + a 2 z 22 = 0 .
Etapa locală, Arad, 2010
4. Fie (zn)n≥1 un şir de numere complexe de acelaşi modul, cu proprietatea că z k2 = z k −1 ⋅ z . Ştiind că
cel puŃin trei termeni ai şirului sunt numere reale, arătaŃi că mulŃimea termenilor şirului este finită.
Etapa locală, Prahova, 2010
 a  b
5. Se consideră a, b ∈ R cu a ⋅ b < 0, z1, z2 ∈ C*. Să se arate că dacă  z1 +  z 2 +  ∈ R, atunci
 z 2  z1 
există k ∈ R* astfel încât z 2 = k ⋅ z1 .
Etapa locală, Maramureş, 2010, prof. G. Gherasim
6. Trei numere complexe a, b, c se numesc fraŃi dacă a + b + c = abc. ArătaŃi că:
a) există trei fraŃi nenuli;
b) dacă unul dintre fraŃi este 0, atunci imaginile geometrice ale celor trei fraŃi sunt puncte
coliniare;

67
c) cel puŃin unul dintre fraŃi are modulul strict mai mic decât 2.
Etapa locală, Caraş-Severin, 2010
7. Se consideră perechile de numere complexe z1, z2 pentru care sunt satisfăcute relaŃiile:
i) |z1| = |z2| = a ∈ (0, ∞);
ii) | z12 + z1z 2 + z 22 |= a 2 .
z
a) Să se demonstreze că dacă 1 ∈ C – R, atunci z12010 + z 2010
2 ∈ R.
z2
z1
b) Să se demonstreze că dacă ∈ R, atunci z12009 + z 2009
2 = 0.
z2
z1
c) Să se demonstreze că dacă ∈ C – R, atunci z12009 + z 2009
2 = a 2009 2 .
z2
Etapa locală, Botoşani, 2010, prof. D. NăstruŃ
8. Se consideră numerele complexe z1, z2, z3 astfel încât z1 + z2 ∈ R, z 3 ⋅ z1 + z 3 ⋅ z 2 ∈ R şi z3 ∈ C – R.
Să se demonstreze că z12010 + z 2010
2 ∈ R.
Etapa locală, Mureş, 2010
9. Fie z1, z2, z3 numere complexe de modul 1.
z1 + z 2 + z 3 + z1z 2 + z1z 3 + z 2 z 3 + z1z 2 z 3
a) DemonstraŃi că numărul este număr real.
1 + z1z 2 z 3
b) Dacă, în plus, z1 + z2 + z3 = z1z2z3 = 1, arătaŃi că z12 + z 22 + z 32 = −1 .
Etapa locală, Iaşi, 2010
z |z|
10. Fie z un număr complex nenul. ArătaŃi că numărul + este real şi să se deducă că există a,
|z| z
b ∈ R astfel încât z2 = az + b.
Etapa locală, Olt, 2010

I.2.1. PROPRIETĂłI ALE NUMERELOR COMPLEXE. OPERAłII

1 3
1. Fie a, b, c ∈ R şi ω = − + i .
2 2
i) Să se arate că A ∈ R, unde A = (a + bω + cω2)(a + bω2 + cω).
ii) Să se arate că 2(aω2 + bω + c)(aω + bω2 + c) = (a – b)2 + (b – c)2 + (c – a)2.
Etapa locală, Mureş, 2010
2. Fie a, b, c trei numere reale distincte două câte două. Să se arate că expresia:
a 2 + ibc b 2 + iac c 2 + iab
E= + + nu depinde de a, b şi c.
(a − b)(a − c) (b − a )(b − c) (c − a )(c − b)
Etapa locală, Bacău, 2010

68
3. Fie E = (a + bω + cω2)(a + bω2 + cω), unde a, b, c ∈ R, distincte două câte două, iar
1 3
ω= − +i .
2 2
a) Să se arate că E = (2a – b – c)(2b – c – a)(2c – a – b).
b) Dacă a + b + c = 0, atunci E = 27abc.
Etapa locală, Suceava, 2010
1 3
4. Fie ω = + i şi P1 (z) = z 3 + ωz 2 + ωz + 1 , P2 (z) = z 3 + ωz 2 + ωz + 1 . Să se arate că există A ∈ C,
2 2
B ∈ C astfel încât P1(z2) = A ⋅ P2(z), iar P2(z2) = B ⋅ P1(z).
Etapa locală, Tulcea, 2010
5. Să se arate că dacă a + b + c = 0, a, b, c ∈ R, atunci E = (a + bi) + (b + ci)4 + (c + ai)4 este real.
4

Reciproca este adevărată?


Etapa locală, Covasna, 2010

I.2.2. INTERPRETĂRI GEOMETRICE. FORMA TRIGONOMETRICĂ

1. Să se determine toate numerele complexe care au partea reală, partea imaginară şi modulul
numere raŃionale.
Etapa locală, Dolj, 2010
1 1 1
2. Fie z ∈ C* pentru care Re z < − . Să se arate că z + + 2 ∈ R dacă şi numai dacă z ∈ R.
8 z z
Etapa locală, Iaşi, 2010
3. Fie zk ∈ C, k = 1, 3 pentru care | z k | = 1, k = 1, 3 şi Re z k ≥ 0, k = 1, 3 . Să se arate că |z1 + z2 + z3| ≥ 1
şi să se interpreteze geometric.
Etapa locală, Olt, 2010
4. Numărul z fiind conjugatul numărului complex z ≠ 0, să se calculeze în funcŃie de ρ şi de
argumentul θ a lui z, modulul şi argumentul numărului complex Z, astfel încât Z = z –
 π π
–  cos + i sin z .
 3 3
Etapa locală, ConstanŃa, 2010
5. Fie numerele complexe z1, z2, z3, ..., zn, n ∈ N, n ≥ 2 astfel încât z1z2...zn = 1, |z1| = |z2| = ... = |zn|.
Să se arate că partea imaginară a produsului (1 + z1)(1 + z2)...(1 + zn) este nulă.
Etapa locală, Harghita, 2010

I.2.3. MODULUL NUMĂRULUI COMPLEX. INEGALITĂłI

1. Fie z1, z2 ∈ C, astfel încât |z1| = |z2|, α ∈ R. DemonstraŃi că:


a) |αz1 + z2| = |z1 + αz2|;
b) n|z1 + z2| ≤ 2|(n – 1)z1 + z2|, (∀) n ∈ N, n ≥ 2.
Etapa locală, Sibiu, 2010, prof. A. Paşca

69
2. a) Să se demonstreze că pentru orice z1, z2 numere complexe, are loc relaŃia |z1 + z2|2 + |z1 – z2|2 =
= 2(|z1|2 + |z2|2).
b) Ştiind că z1, z2 sunt numere complexe astfel încât |z1| = |z2| = 1, iar | z1 − z 2 |= 2 , să se
calculeze |z1 + z2|.
Etapa locală, Teleorman, 2010
3. Fie a, b, c ∈ C cu a + b = c . Să se arate că |c – a| + |c + a| = |c – b| + |c + b|.
2 2 2

Etapa locală, Hunedoara, 2010


1 1
4. Să se demonstreze că dacă z ∈ C* şi z 3 + 3 ≤ 2 , atunci z + ≤ 2 .
z z
Etapa locală, Harghita, 2010
1 1
5. Să se arate că z ∈ C* şi z 3 − 3 ≥ 14 , atunci z − ≥ 2 .
z z
Etapa locală, Maramureş, 2010, prof. G. Tivadar
6. Să se arate că pentru orice z1, z2 ∈ C, de modul 1, are loc inegalitatea:
|z1 + 1| + |z2 + 1| + |z1z2 +1 | ≥ 2.
Etapa locală, Vrancea, 2010, prof. C. Stoleru
1
7. Fie a, b, c ∈ C şi α, β > 0, cu α + β = . Să se arate că:
2
|(1 – α)(a – b) – β(a – c)| + |(1 – β)(a – c) – α(a – b)| ≤ |a – b| + |a – c|.
Etapa locală, Brăila, 2010, prof. G. Daniilescu
8. Fie z1, z2, z3 numere complexe distincte de modul r. ArătaŃi că:
1 1 1 1
+ + ≥ 2.
| z1 − z 2 | ⋅ | z1 − z 3 | | z 2 − z1 | ⋅ | z 2 − z 3 | | z 3 − z1 | ⋅ | z 3 − z 2 | r
Etapa locală, Iaşi, 2010
9. Fie z1, z2, z3 ∈ C, |z1| = |z2| = |z3| = 1 şi z1 + z2 + z3 = 0. DemonstraŃi că pentru orice z ∈ C avem:
|z – z1| + |z – z2 + |z – z3| ≥ 3.
Etapa locală, DâmboviŃa, 2010, prof. Călin Burduşel
2
10. Se consideră numărul complex nenul z, cu |z| < . Să se demonstreze că:
2
 n
z 2k 
n
Re n −



k =1 | z |
2k


 , (∀) n ∈ N*.

k =1
Im(z k ) <
2
Etapa locală, GalaŃi, 2010, prof. M. Totolici

70
II. FuncŃii

II.1. ProprietăŃi generale ale funcŃiilor. OperaŃii

II.1.1. FUNCłII INJECTIVE

1. Să se arate că nu există nicio funcŃie injectivă f: (0, ∞) → R astfel încât f(3x) + f(5x) = 4, pentru
x ∈ (0, ∞).
Etapa locală, Buzău, 2010
2. Există funcŃii injective f: (0, ∞) → R, astfel încât f(x) + f ( x ) + f ( x ) = 2009 , (∀) x ∈ (0, ∞)?
2

Etapa locală, Sibiu, 2010


3. Să se precizeze dacă există funcŃii injective f: R → R astfel încât f(2 ) + f(x + 1) = x(x – 1) + 4.
x

Etapa locală, Teleorman, 2010


4. Se consideră funcŃia f: (0, ∞) → R cu proprietatea că f(x1x2) = f(x1) + f(x2), (∀) x1, x2 ∈ (0, ∞).
a) ArătaŃi că f(1) = 0.
b) ArătaŃi că dacă 1 este singura soluŃie a ecuaŃiei f(x) = 0, atunci funcŃia f este injectivă.
Etapa locală, Ilfov, 2010
5. Există funcŃii injective f: R → R cu proprietatea că: 3f (x – x + x ) – f (x – x4 + x3) ≥ f4(x) + 4.
3 5 4 3 2 5

Etapa locală, Vrancea, 2010, prof. C. Stoleru


6. ArătaŃi că nu există funcŃii injective f: R → R cu proprietatea că: f(3x) + f (log3 x ) = 1 , (∀) x > 0.
Etapa locală, Bacău, 2010
7. Să se arate că nu există funcŃii injective f: R → R cu proprietatea că f(f(x)) = [x], x ∈ R.
Etapa locală, Caraş-Severin, 2010

II.1.2. FUNCłII SURJECTIVE

1. Fie A şi B două mulŃimi finite având a, respectiv b elemente. Să se arate că a ≥ b dacă şi numai
dacă există o funcŃie surjectivă f: A → B.
Etapa locală, Olt, 2010
2. Fie a > 0. Să se arate că nu există funcŃii surjective f: N → N cu proprietatea că f(n) ≥ a ⋅ n2,
(∀) n ∈ N.
Etapa locală, GalaŃi, 2010
3. Să se determine toate funcŃiile f: R → [1, ∞), surjective, care verifică relaŃia:
f(x + f(y)) = f(x) + 2xf(y) + f2(y), (∀) x, y ∈ R.
Etapa locală, Argeş, 2010
4. Să se arate că orice funcŃie f: R → R poate fi scrisă ca sumă a două funcŃii surjective f = f1 + f2.
Etapa locală, Iaşi, 2010

71
5. Fie A o mulŃime finită de numere reale cu cel puŃin două elemente şi f: A → A o funcŃie astfel ca
|f(x) – f(y)| < |x – y| pentru orice x, y ∈ A, x ≠ y.
i) Să se arate că f nu este surjectivă.
ii) Să se demonstreze că (∃) x0 ∈ A, astfel încât f(x0) = x0.
Etapa locală, Suceava, 2010
6. a) Să se construiască o funcŃie surjectivă f: [–1, 1] → [–1, 1] care să ia fiecare valoare a sa de o
infinitate de ori.
b) Să se construiască o funcŃie surjectivă f: N → N care să ia fiecare valoare a sa de o infinitate de
ori.
Etapa locală, Gorj, 2010
7. Să se determine toate funcŃiile surjective f: N → N cu proprietatea f(n) ≥ n + (–1)n, (∀) n ∈ N.
Etapa locală, Harghita, 2010

II.1.3. FUNCłII BIJECTIVE

1. Fie funcŃiile f, g: R → R, astfel încât g ( x ) = x + 3 x şi f are proprietatea f ( x + 3 x ) ≤ x ≤ f ( x ) +


+ 3 f ( x ) , (∀) x ∈ R. DemonstraŃi că f şi g sunt funcŃii bijective.
Etapa locală, Vâlcea, 2010, prof. V. Gorgotă
2. Fie f: C → C, definită prin f(z) = 2z + z , unde z reprezintă conjugatul numărului complex z.
ArătaŃi că f este bijectivă şi determinaŃi-i inversa.
Etapa locală, Sălaj, 2010
x +3
3. Există funcŃii bijective f: R → R cu proprietatea că f (f ( x )) = , (∀) x ∈ R?
4
Etapa locală, Caraş-Severin, 2010
4. Fie f: Z → Z care îndeplineşte următoarele condiŃii:
i) f(f(n)) = n, (∀) n ∈ Z; ii) f(f(n – 2) – 2) = n, (∀) n ∈ Z; iii) f(1) = 0.
a) DemonstraŃi că f este bijectivă.
b) DeterminaŃi f şi f–1.
Etapa locală, Botoşani, 2010, prof. D. Poroşniuc
min ( t − 2 t + 2), x ≤ 1
2
 t≤x
5. Se consideră funcŃia f: R → R, f(x) =  .
max (1 − t ), x > 1
 t>x
a) Să se arate că funcŃia f este injectivă, dar nu este surjectivă.
f ( x ) + k , x ≤ 1
b) Să se determine k ∈ R astfel încât funcŃia g: R → R, g(x) =  să fie bijectivă.
f ( x ), x >1
c) Să se determine g–1.
Etapa locală, Braşov, 2010, prof. V. Drăghici
( x + 1)( x + a )
6. Să se arate că funcŃia f: [0, 1] → [0, 1], f(x) = , a ∈ R este bijectivă.
x2 +1
Etapa locală, Bihor, 2010
72
7. Fie a ∈ R, a ≠ 0. Să se arate că există funcŃii bijective f: R → R, astfel încât:
a2f2[(a2 + 1)x] – 2af(x2 + a2) + 1 ≤ 0 pentru orice x ∈ R, dacă şi numai dacă a ∈ {–1, 1}.
Etapa locală, Mureş, 2010

II.1.4. FUNCłII MONOTONE

1. Să se arate că nu există funcŃii strict monotone f: R → R cu proprietatea că f(f(x)) = {x}, (∀) x ∈


∈ [0, 1].
Etapa locală, Caraş-Severin, 2010
2. Fie funcŃiile f, g: R → R, f strict crescătoare, g strict descrescătoare. Să se studieze monotonia
funcŃiei g  f.
Etapa locală, Covasna, 2010
3. Fie a, b > 0 două numere reale. Considerăm funcŃia f: [–a, a] → R, f ( x ) = b 2 + (a − x ) 2 +
+ b 2 + (a + x ) 2 . Să se determine intervalele de monotonie ale funcŃiei f.
Etapa locală, Harghita, 2010
1− x
4. Fie funcŃia f: R – {–2} → R, f ( x ) = . Să se arate că f este strict descrescătoare pe
( x + 2) 2
intervalul [–1, 1].
Etapa locală, Tulcea, 2010
5. Fie a, b ∈ R, a < b şi f: [a, b] → R, f ( x ) = x − a + b − x . Să se studieze monotonia funcŃiei f.
Etapa locală, Olt, 2010
x
6. Să se studieze monotonia funcŃiei f: R → R, f(a) = a b , unde a ≥ 0 şi b > 0.
Etapa locală, BistriŃa-Năsăud, 2010
7. Fie f: N* → N* o funcŃie strict crescătoare pentru care f(2n) = f(n) + n, (∀) n ∈ N*. Să se
calculeze f(n).
Etapa locală, Giurgiu, 2010

II.1.5. FUNCłII INVERSABILE

1
 2x , x ∈ ( −∞,0)

1. ArătaŃi că funcŃia f: R → R, dată prin: f (x) = cos x, x ∈ [0, π] este inversabilă şi determinaŃi
 1
− ⋅ x, x ∈ (π, ∞)
 n
inversa ei.
Etapa locală, Gorj, 2010

73
2. Fie p un număr natural nenul. DeterminaŃi funcŃiile f: Z → Z care satisfac relaŃia următoare:
1
f ( x )f ( y) = − [ x + (−1) y p − 2] ⋅ [f ( x ) + f ( y)] = 2[ y + (−1) x p] , (∀) x, y ∈ Z. Să se determine func-
2
Ńiile f inversabile care verifică relaŃia dată, şi să se afle f–1.
Etapa locală, Brăila, 2010
1  1 
3. Se consideră funcŃiile f:  , ∞  → (0, 1) şi g:  , ∞  → (1, 2) astfel încât g(x) – f(x) = x ⋅ f(x) ⋅
 2  2 
⋅ g(x) = 1. Să se demonstreze că funcŃiile f şi g sunt inversabile şi că f–1(t) = g–1(t + 1), (∀) t ∈ (0, 1).
Etapa locală, Olt, 2010
4. Să se demonstreze că funcŃia f: [k, k + 1) → [0, 1), f(x) = {x}, k ∈ Z fixat este inversabilă şi să se
afle inversa.
Etapa locală, Vâlcea, 2010
5. Să se arate că dacă funcŃia f: R → R este impară şi inversabilă, atunci inversa ei este impară.
Etapa locală, Botoşani, 2010
6. Fie f: R → R o funcŃie strict crescătoare şi surjectivă. Să se arate că f este bijectivă şi f–1 este
strict crescătoare.
Etapa locală, Mureş, 2010
7. Să se arate că funcŃiile f: R → R şi g: R → R care satisfac relaŃiile:
2f (2 x + 3) + g( 2x + 1) = 2(6x + 7)
 , pentru orice x ∈ R, sunt inverse una alteia.
f (1 − 2 x ) − g( −8x − 7) = 7
Etapa locală, Caraş-Severin, 2010

II.1.6. FUNCłII PERIODICE


1. a) Să se demonstreze că dacă b1, b2 sunt numere reale pozitive diferite şi a1, a2 numere reale
pentru care a1sin b1x + a2sin b2x = 0, oricare ar fi numărul real x, atunci a1 = a2 = 0.
b) Să se demonstreze că funcŃia f: R → R, f(x) = a1cos b1x + a2cos b2x este periodică dacă şi
b
numai dacă 1 este număr raŃional.
b2
Etapa locală, Harghita, 2010
2. Fie f: R → R o funcŃie cu proprietatea că: f(x + 2) = {f(x + 1)} + [f(x)], (∀) x ∈ R.
ArătaŃi că funcŃia este periodică.
Etapa locală, Ilfov, 2010
a , x ∈ Q
3. Fie a, b două numere raŃionale distincte şi funcŃia f: R → R, f ( x ) =  .
 b, x ∈ R - Q
AflaŃi perioada funcŃiei f.
Etapa locală, Covasna, 2010

74
1
4. Fie f: R → R – {1} o funcŃie care verifică relaŃia: f ( x + 1) = , (∀) x ∈ R.
1 − f (x)
AflaŃi perioada funcŃiei f.
Etapa locală, Satu-Mare, 2010
5. Fie f: R → R care satisface relaŃia f(x + 1) + f(x – 1) = 2f ( x ) , (∀) x ∈ R. Să se arate că f este
periodică.
Etapa locală, Alba, 2010
6. Se dă a, b ∈ R+ şi f: R → [a, b], astfel încât f2(x – a) + f2(x + a) = b2, (∀) x ∈ R.
Să se arate că f este periodică.
Etapa locală, ConstanŃa, 2010
7. Fie f: R → R o funcŃie care satisface relaŃia: f(x + 1) ⋅ f(x) + f(x + 1) + 1 = f(x), (∀) x ∈ R.
Să se arate că f este periodică.
Etapa locală, Sibiu, 2010
II.1.7. FUNCłII CONVEXE

1. Fie A ⊂ R o mulŃime convexă. O funcŃie f: A → R este semiconvexă (convexă în sens Jensen)


x+y  y+z z+x
dacă şi numai dacă (∀) x, y, z ∈ A avem: f  +f +f  ≤ f ( x ) + f ( y) + f ( z ) .
 2   2   2 
Etapa locală, Alba, 2010
2. MulŃimea M = {f: I → R | f convexă} este convexă.
Etapa locală, BistriŃa-Năsăud, 2010
3. O funcŃie f: I → R, mărginită superior este convexă dacă şi numai dacă este convexă în sens
Jensen.
Etapa locală, Maramureş, 2010
4. Orice funcŃie f: I → R convexă care nu este monotonă are un punct de minim global.
Etapa locală, Iaşi, 2010
5. i) Fie f, g: R → R, f, g convexe şi g crescătoare. Atunci funcŃia h: R → R, h = g  f convexă.
ii) Fie f, g: R → R, f, convexă, g concavă, g descrescătoare. Atunci funcŃia h: R → R, h = g  f
concavă.
Etapa locală, Prahova, 2010
6. i) Fie f, g: R → R, f, g concave, g crescătoare. Atunci funcŃia h: R → R, h = g  f concavă.
ii) Fie f, g: R → R, f concavă, g convexă, g descrescătoare. Atunci funcŃia h: R → R, h = g  f
concavă.
Etapa locală, MehedinŃi, 2010
7. Dacă f: [a, b] → [c, d] convexă, crescătoare, inversabilă, atunci f–1: [c, d] → [a, b] concavă,
crescătoare.
Etapa locală, Buzău, 2010

75
II.1.8. ALTE TIPURI DE PROBLEME CU FUNCłII

1. Fie a, b ∈ C, a ≠ 2 şi funcŃia f: C → C, f(z) = az + b. ArătaŃi că dacă punctele de afixe z1, z2, z3


sunt coliniare, atunci şi punctele de afixe f(z1), f(z2), f(z3) sunt coliniare.
Etapa locală, Giurgiu, 2010
2. În planul complex C, raportat la reperul (O, i, j) , se asociază fiecărui punct M de afix nenul z
1
punctul M', mijlocul segmentului [MM1], unde M1 este punctul de afix , şi se consideră
z
aplicaŃia f: C → C, definită prin f(M) = M' dacă M ≠ O şi f(0) = 0.
i) Să se determine punctele M din plan pentru care f(M) = M.
ii) Să se determine imaginea prin f a cercului C(O, 1) cu centrul în origine şi de rază 1.
iii) Să se stabilească dacă f este injectivă.
Etapa locală, Dolj, 2010
  x + y  f ( x ) + f ( y) 
3. Fie numărul a ∈ (0, 1) ∩ Q. Se consideră mulŃimile A = f : R → R f  =  şi
  2  2 
B = {f: R → R | f(ax + (1 – a)y) = af(x) + (1 – a)f(y))}, (∀) x, y ∈ R, a ∈ (0, 1).
x+y  a ( x + y)   x+y
a) ArătaŃi că = a  (1 − a ) x +  + (1 − a ) ay + (1 − a )  , (∀) x, y ∈ R, a ∈ R.
2  2   2 
b) Să se demonstreze că A = B.
Etapa locală, Hunedoara, 2010
4. Fie f, g: R → R două funcŃii astfel încât (g  f)(x) = (f  g)(x), (∀) x ∈ R. Să se arate că dacă
funcŃia h: R → R, h(x) = x – f(x) este bijectivă, atunci există x0 ∈ R astfel încât g(x0) = x0.
Etapa locală, Arad, 2010
[ x + x + 1]
2
5. Fie f: R → R, f ( x ) = , unde [a] reprezintă partea întreagă a numărului real a. Să se
x2 +1
3
determine valoarea lui x pentru care f(x) = 0 şi să se arate că f(x) ≤ , (∀) x ∈ R.
2
Etapa locală, Cluj, 2010
6. DeterminaŃi funcŃiile f: C → C cu proprietatea că f(x) + f(εx) = x, (∀) x ∈ C, unde ε este rădăcina
primitivă de ordin trei a unităŃii.
Etapa locală, Iaşi, 2010
7. Să se determine f: C → C ştiind că f(z) + f(–z) = z – f(iz), (∀) z ∈ C.
2

Etapa locală, DâmboviŃa, 2010, prof. Călin Burduşel


z
8. Se consideră f: C → C, f (z) = . Să se arate că Im f = {z ∈ C | |z| < 1}.
1+ | z |
Etapa locală, ConstanŃa, 2010
9. Se consideră funcŃia f: R → R, f(x) = (x + 1)(x + 2)(x + 3)(x + 4). Să se determine f(R).
Etapa locală, GalaŃi, 2010, prof. C. Gusta

76
10. Fie funcŃia f: C → R, f(z) = |az2| + |bz + c|, (∀) z ∈ C cu a, b, c ∈ C, cu a ≠ 0.
i) Să se arate că pentru orice z1, z2 ∈ C şi orice s ∈ [0, 1], are loc:
f(sz1 + (1 – s)z2) ≤ sf(z1) + (1 – s)f(z2).
ii) Să se arate că pentru orice z1 ∈ C cu |z1| < 1, există z2 ∈ C, cu |z2| < 1, astfel încât f(z1) < f(z2).
Etapa locală, Sibiu, 2010

II.2. FuncŃii particulare

II.2.1. FUNCłIA EXPONENłIALĂ

1. Fie a ∈ (0, ∞) – {1}. Să se determine funcŃia f: R → R, cu proprietatea: f(x + y) ≥ f(x)f(y) ≥ ax+y,


(∀) x, y ∈ R.
Etapa locală, Alba, 2010
x
 (m − 1)a + m 
2
2. Fie funcŃia f: R → R dată prin relaŃia f ( x ) =   , a ∈ R, m ∈ R. Să se determine

 ma + a + m − 1 
2

m ∈ R astfel încât funcŃia să fie definită pe R, oricare ar fi a ∈ R.


Etapa locală, Sălaj, 2010
3. Să se determine funcŃiile f: R → (0, ∞) pentru care:
a) f(x) ≤ 2010x, (∀) x ∈ R; b) f(x + y) ≤ f(x)f(y), (∀) x, y ∈ R.
Etapa locală, Arad, 2010
4. Fie f: R → R o funcŃie astfel încât (f  f)(x) = 2 , (∀) x ∈ R.
x–1

Să se arate că {f(1), f(2)} = {1, 2}.


Etapa locală, Timiş, 2010
5. Să se determine funcŃiile f: R → R cu proprietatea că f(x + y) = a f(x) + f(y), (∀) x, y ∈ R, a > 0,
y

a ≠ 1.
Etapa locală, Călăraşi, 2010
6. Să se determine funcŃiile f: R → R, care verifică relaŃia x + y ≤ 2xf(x) + 2yf(y) ≤ f(x + y) ⋅ 2x+y,
(∀) x, y ∈ R.
Etapa locală, Teleorman, 2010
7. Să se determine funcŃiile f: Q → R + cu proprietăŃile:
*

i) f(0) = 1; ii) f ( x ) = f ( x + y) ⋅ f ( x − y) , (∀) x, y ∈ Q.


Etapa locală, Vâlcea, 2010

II.2.2. FUNCłIA LOGARITMICĂ

 2x − 1 − x 
1. DeterminaŃi valorile lui x ∈ R pentru care este definit algoritmul: log x 2010 − x  − 3  .
 2 +1
x

Etapa locală, Sibiu, 2010, prof. L. Băcilă, A. łintea

77
2. Să se determine funcŃiile f, g: (0, ∞) → R ştiind că satisfac simultan condiŃiile:
x x
a) f   + 2 ≤ log 3 x ≤ g ( x ) − 1 ;
b) g  ≤ log 3 x ≤ f ( x ) + 1 , oricare ar fi x ∈ (0, ∞).
3 3
Etapa locală, Maramureş, 2010
3. Să se determine parametrul m, astfel încât funcŃia f(x) = loga[mx2 – 2(m + 2)x + 9], a ∈ (0, 1) ∪
∪ (1, ∞) fixat, să fie definită, (∀) x ∈ R.
Etapa locală, Cluj, 2010
4. Să se determine funcŃia f: (0, ∞) → R, cu proprietatea f(2x) ≤ log2 x ≤ f(x) + 1, (∀) x > 0.
Etapa locală, IalomiŃa, 2010
ln( xy)
5. DeterminaŃi funcŃiile f: R* → R cu proprietatea f(xy) ≥ f(x) + f(y) ≥ , (∀) x, y > 0.
1+ | ln x ln y |
Etapa locală, Tulcea, 2010
6. Să se determine funcŃiile f: (0, ∞) → R, care pentru orice x, y > 0, verifică relaŃia:
xyln (xy) ≤ yf(x) + xf(y) ≤ f(xy).
Etapa locală, DâmboviŃa, 2010
7. DeterminaŃi funcŃiile f: (0, ∞) → R care satisfac condiŃiile: ln (xy) ≤ xf(x) + yf(y) ≤ xyf(xy).
Etapa locală, Mureş, 2010

II.2.3. FUNCłII TRIGONOMETRICE

1. DeterminaŃi f: R → [–1, 1] cu proprietatea: f(3x) + 4f(2x) + f(x) = 2sin 2x(2 + cos x), (∀) x ∈ R.
Etapa locală, Dolj, 2010
2. Să se determine funcŃiile f, g: R → R, care satisfac relaŃia: f(x) + f(y) + g(x) – g(y) = sin x + cos y,
(∀) x, y ∈ R.
Etapa locală, ConstanŃa, 2010
 π 2
 π 
3. Fie f: [0, 1] → − , 0 o funcŃie pentru care f(sin t) = t2 – πt, pentru orice t ∈  , π . Să se
 4  2 
arate că f(x) = (arcsin x)2 – π(arcsin x).
Etapa locală, Timiş, 2010
4. Fie a, b, c, d ∈ R şi funcŃia f: R → R, f(x) = asin x + bcos x + csin 2x + dcos 2x.
 2π   4π 
i) Pentru fiecare x ∈ R, să se calculeze f ( x ) + f  x +  + fx + .
 3   3 
ii) Să se arate că există o infinitate de valori ale lui x, pentru care f(x) ≤ 0.
Etapa locală, Satu Mare, 2010
5. Fie a, b, c, d ∈ R şi funcŃia f: R → R, f(x) = 1 + acos x + bsin x + ccos 2x + dsin 2x.
 2π   2π 
i) Să se calculeze, pentru fiecare x ∈ R, f ( x ) + f  x +  + fx − .
 3   3 
ii) Să se arate că dacă f(x) > 0, (∀) x ∈ R, atunci f(x) < 3, (∀) x ∈ R.
Etapa locală, Prahova, 2010
78
6. Fie x, y ∈ R. ArătaŃi că dacă mulŃimea An = {cos nπx + cos nπy | n ∈ N} este finită, atunci x ∈
∈ Q, y ∈ Q.
Etapa locală, Cluj, 2010
7. Fie M o mulŃime de numere reale cu proprietăŃile:
i) 0 ∈ M;
ii) x ∈ M ⇒ (sin x + cos x) ∈ M;
iii) (sin 2x + cos 2x) ∈ M ⇒ x ∈ M.
Să se arate că:

a) ∈ M;
4
b) M conŃine o infinitate de numere iraŃionale subunitare.
Etapa locală, Covasna, 2010

III.
II. EcuaŃii

III.1. EcuaŃii în numere întregi


1. Să se rezolve în Z ecuaŃia: x2000 + y2000 = 2000xy – 1998.
Etapa locală, Arad, 2010
2. Să se demonstreze că pentru orice număr prim p > 2001 ecuaŃia x 2000
+ 2000x = p nu are soluŃii
întregi.
Etapa locală, NeamŃ, 2010
x12+ + ... +
x 22 x 2n
3. Să se rezolve în mulŃimea numerelor întregi ecuaŃia =1.
x1 + x 2 + ... + x n
Etapa locală, Ilfov, 2010
x1 x 2 x n −1 x n
4. ArătaŃi că ecuaŃia + + ... + + = 2010 nu admite soluŃii numere naturale pentru orice
x 2 x3 xn x1
număr natural n ≥ 2011.
Etapa locală, IalomiŃa, 2010
5. Să se determine numerele reale a şi b astfel încât ecuaŃia x – ax + b = 0 şi x2 – bx + a = 0 să aibă
2

rădăcini numere naturale distincte.


Etapa locală, ConstanŃa, 2010

III.2. EcuaŃii iraŃionale

1. Să se rezolve ecuaŃia: x − x + x + [ x ] = 1 , unde [x] este partea întreagă a lui x.


Etapa locală, Vâlcea, 2010

79
2. Să se rezolve în mulŃimea numerelor reale ecuaŃiile:
a) x + x − 1− x = 1; b) 2 x +1 11− 4 x
(−2 x )3x = 3 x −1 7 x + 2 .
Etapa locală, Harghita, 2010
3. RezolvaŃi ecuaŃia: x + 2 ( x + 1) = x + 1 .
x

Etapa locală, Prahova, 2010, prof. G. Necula


4. Să se rezolve în R ecuaŃia 7 x + 18 + − x − x + 8 + x − 6 x + 1 = 3 .
3 3 2 3 2

Etapa locală, Maramureş, 2010, prof. G. Boroica


5. Fie n un număr natural mai mare sau egal decât 3. RezolvaŃi în mulŃimea numerelor reale ecuaŃia:
x +1 3 x + 2 x + n −1 4 2
+ + ... + n = x + 9 x + ... + n x .
2 3 n
Etapa locală, NeamŃ, 2010, prof. A. Sandovici

III.3. EcuaŃii în mulŃimea numerelor complexe


1. Fie numerele complexe a, b, c, ε cu a ≠ 0, ε ≠ 1, ε3 = 1 şi |a + bε + cε2| ≤ |a|. Să se arate că ecuaŃia
az2 + bz + c = 0 are cel puŃin o soluŃie cu modulul mai mic sau egal decât 2.
Etapa locală, Argeş, 2010
2. DeterminaŃi z ∈ C astfel încât z2 – 3z + 2 ≤ 0.
Etapa locală, Vrancea, 2010, prof. D. Popoin
z 2 − z1
3. Fie numerele complexe z1 ≠ z2 ≠ z3 ≠ z1 astfel încât Re = 0 . Să se rezolve inecuaŃia:
z 3 − z1
|z1 – z2|x + |z1 – z3|x ≤ |z3 – z2|x.
Etapa locală, Prahova, 2010
4. Fie ecuaŃia az2 + bz + c = 0, unde a, b, c ∈ C, a ≠ 0 şi arg a + arg c = 2arg b, |a| + |c| = 2|b|.
Să se arate că ecuaŃia are cel puŃin o rădăcină de modul mai mică sau egală cu 1.
Etapa locală, Bihor, 2010
5. Se consideră ecuaŃia az + bz + bz + a = 0 , a, b ∈ C*. Să se arate că ecuaŃia are cel puŃin o
3 2

rădăcină în modul mai mică sau egală decât 1.


Etapa locală, Satu Mare, 2010

III.4. EcuaŃii exponenŃiale şi logaritmice


1. Să se rezolve în R ecuaŃia: 343x + 512x + 729x = 3 ⋅ 504x.
Etapa locală, Giurgiu, 2010, prof. George Ionescu
2. Să se rezolve ecuaŃia [log2x] = [log3x], x > 0, unde [a] desemnează partea întreagă a numărului
real a.
Etapa locală, Dolj, 2010

80
log 3 x + log 2 y = 2
3. Să se rezolve sistemul:  .
3 − 2 = 23
x y

Etapa locală, Teleorman, 2010


4. RezolvaŃi ecuaŃiile:
a) log n ( x + n (n − 1)) = 1 + n log n x , n ∈ N, n ≥ 2;
1 1 8
b) ( x −1)( x − 2 )
+ ( x −1)( x − 3)
= ( x − 2 )( x − 3)
.
2 2 2
Etapa locală, Vâlcea, 2010
x2 x2 x2
5. Să se determine soluŃiile reale ale ecuaŃiei 3 + 4 =5 .
Etapa locală, Hunedoara, 2010
1 1

6. Să se determine numerele reale x, y cu proprietatea că: 16x + 16y + 2x + 2y = 8 .


Etapa locală, Argeş, 2010
7. a) Să se rezolve ecuaŃia x log 5 4 − 7 log 5 x = 2 − x .
b) Dacă a, b, c ∈ R*+ – {1} astfel încât b2 + c2 = a2, să se rezolve ecuaŃia: b log a x + c log a x = x în
acest caz.
Etapa locală, Argeş, 2010
8. Să se rezolve în mulŃimea numerelor reale ecuaŃiile:
a) (4 + 15 ) x + (4 − 15 ) x = 62 ; b) log 2 (log 4 (log8 x )) = log 4 (log 2 (log8 x )) .
Etapa locală, Harghita, 2010
9. Să se rezolve în R ecuaŃia log 3 (1 + sin x ) ⋅ log 3 (1 + cos x ) = 1 .
2 2

2 2
Etapa locală, Vaslui, 2010
10. Să se rezolve în mulŃimea numerelor reale ecuaŃia: log 3 (2 + 1) = log 2 (3 − 1) . x x

Etapa locală, GalaŃi, 2010

III.5. EcuaŃii trigonometrice


1. Să se determine valorile reale ale lui x şi y pentru care avem relaŃia:
cos 2 x + (sin y + cos y) sin 2x + 1 = 0 .
Etapa locală, Dolj, 2010
2. Să se rezolve ecuaŃia: tg(x + 5°) ⋅ tg(x + 10°) ⋅ tg(x + 15°) ⋅ tg(3x + 20°) ⋅ tg(3x + 40°) = − 3 .
Etapa locală, Tulcea, 2010
3. Să se rezolve ecuaŃia [sin x] = [tg x].
Etapa locală, Covasna, 2010
4. Să se demonstreze că ecuaŃia (1 – sin x)2 + sin2(1 – x) = 0 nu are soluŃii reale.
Etapa locală, Bacău, 2010

81
5. Să se arate că următoarele ecuaŃii:
3 x 1
i) sin x ⋅ sin 2x ⋅ sin 3x = ; ii) | sin x | ⋅ sin 2 = x 2 + 2 nu au soluŃii.
4 5 x
Etapa locală, Alba, 2010

III.6. EcuaŃii funcŃionale


 2010 x  x + 1
4
1. Să se găsească funcŃiile f satisfăcând relaŃia: f   = .
 x +1 
2
x2
Etapa locală, Buzău, 2010
2. Să se determine f: N → Q care verifică simultan condiŃiile:
1 2010
i) f ( x ) = f ( x + 1) − , (∀) x ∈ R; ii) f (2010) = .
x − 3x + 2
2
2011
Etapa locală, Ilfov, 2010
3. Să se determine funcŃiile f: R → R care au proprietatea că f(x – y) – xf(y) ≤ 1 – x, (∀) x, y ∈ R.
Etapa locală, DâmboviŃa, 2010
4. Să se determine funcŃiile f: R → R cu proprietatea că f(x) ⋅ f(y) = f(x) + f(y) + xy – 1.
Etapa locală, Prahova, 2010
5. Să se determine funcŃiile f: R → R cu proprietatea f(x + y) = max{f(x), y} + min{f(y), x}, (∀) x,
y ∈ R.
Etapa locală, Iaşi, 2010

III.7. EcuaŃii nestandard


1. Fie f: R*+ → R o funcŃie care verifică relaŃia f (− log 2 x ) + log 2 x ⋅ f 2 ( x ) ≥ 2 x − f ( x ) , (∀) x ∈ (0, ∞).
Folosind, eventual, că ecuaŃia x ⋅ 2x = 1 are soluŃie, să se arate că ecuaŃia f(x) = 2x are cel puŃin o
1 
soluŃie în intervalul  ,1 .
2 
Etapa locală, Suceava, 2010, prof. C. AmorăriŃei
2. Să se rezolve în R ecuaŃia: 11 = 3x + 2.
7–2x

Etapa locală, Dolj, 2010


3. Să se determine numerele x, y ∈ R astfel încât să fie adevărată egalitatea:
2 sin 2 x (1 + ctg 2 x )
log 4 2 = − y2 + 6y − 5 .
sin 2 x
Etapa locală, Harghita, 2010
2
− 2011x x − 2011x
2
4. RezolvaŃi ecuaŃia 2010 x + = 1.
2010 x
Etapa locală, Vaslui, 2010, prof. C. AdomniŃei

82
x
5. Să se rezolve ecuaŃia 3x + 3− x + 1 = 3 cos .
3
Etapa locală, Botoşani, 2010

GEOMETRIE

IV.1. AplicaŃiile numerelor complexe în geometrie


− 1+ i 3
1. a) Fie ε = şi z ∈ C. DemonstraŃi că (z – 1)2 + (z – ε)2 + (z − ε) 2 = 3z 2 .
2
b) Dacă ABC este un triunghi echilateral de centru O şi M este un punct în planul (ABC),
deduceŃi inegalitatea: MA2 + MB2 + MC2 ≥ 3MO2.
Etapa locală, Giurgiu, 2010, prof. Şerban Olteanu
2. Fie z1, z2, z3, z4 ∈ C distincte cu proprietatea că |z – zk| = |zk|, k ∈ {1, 2, 3, 4}, unde z = z1 + z2 +
+ z3 + z4.
a) Să se arate că z = 0.
b) Dacă |z1| = |z2| = |z3| = |z4|, atunci z1, z2, z3, z4 sunt afixele vârfurilor unui dreptunghi.
Etapa locală, Hunedoara, 2010
3. a) Pentru numerele complexe z1, z2, z3 avem |z1| = |z2| = |z3| = 1 şi z1 + z2 + z3 = 0. Să se
demonstreze că pentru orice z ∈ C avem egalitatea: |z – z1|2 + |z – z2|2 + |z – z3|2 =3(|z|2 + 1).
b) Să se demonstreze că dacă M este un punct mobil pe cercul înscris în triunghiul echilateral
ABC, atunci suma MA2 + MB2 + MC2 este constantă.
Etapa locală, Harghita, 2010
AM BN
4. Fie ABCD un patrulater şi M ∈ (AD), N ∈ (BC) astfel încât + = 1 . Să se arate că
AD BC
mijlocul segmentului (MN) aparŃine segmentului determinat de mijloacele diagonalelor.
Etapa locală, Vaslui, 2010, prof. T. Găvan
5. Fie AA'BB'CC' un hexagon convex înscris în cercul C(O, R) astfel încât AA' = BB' = CC' = R.
Atunci triunghiul MNP este echilateral, unde M, N, P sunt mijloacele laturilor A'B, B'C, respectiv
C'A.
Etapa locală, Vrancea, 2010, prof. D. Popoiu
6. Fie a, b, ∈ C* şi funcŃia f: C → C, f(z) = az + b. Să se arate că dacă punctele de afixe f(z1), f(z2),
f(z3) sunt coliniare, atunci şi punctele de afixe z1, z2, z3 sunt coliniare.
Etapa locală, Botoşani, 2010
7. Se consideră patrulaterul convex ABCD, iar E, F mijloacele diagonalelor [AC], [BD]. Să se arate
că are loc relaŃia: AB2 + BC2 + CD2 + DA2 = AC2 + BD2 + 4EF2.
Etapa locală, Bihor, 2010

83
IV.2. Alte probleme
1. Se consideră în spaŃiu o mulŃime de puncte cu proprietatea că distanŃa dintre oricare două puncte
este aceeaşi. Să se arate că mulŃimea are cel mult 4 puncte.
Etapa locală, Hunedoara, 2010
2. Fie n ≥ 4 puncte în spaŃiu cu proprietatea că oricare 4 sunt conŃinute într-o sferă de rază 1. Să se
arate că toate punctele sunt conŃinute într-o sferă de rază 1.
Etapa locală, Iaşi, 2010
3. Într-un cub de latură 1 se consideră n sfere cu suma ariilor suprafeŃelor lor egală cu 32. Să se arate
că există un plan care intersectează cel puŃin 11 sfere.
Etapa locală, Dolj, 2010
4. Într-un cub de latură 1 se află 1331 puncte. Să se arate că se pot alege dintre ele 5 puncte care să
1
poată fi înglobate într-o sferă de rază .
9
Etapa locală, Braşov, 2010
5. Se consideră în spaŃiu 9 puncte de coordonate întregi. Să se arate că cel puŃin pe unul din
segmentele ce unesc aceste puncte, se mai află un punct de coordonate întregi.
Etapa locală, Cluj, 2010

84
CLASA A XI
XI-A

ALGEBRĂ

1 2 3
1. Fie σ =   ∈ S3, unde S3 reprezintă mulŃimea permutărilor de gradul 3.
3 1 2
a) Să se rezolve ecuaŃia x2 = σ.
b) Să se demonstreze că oricare ar fi ordinea factorilor, produsul tuturor permutărilor din S3 este
permutare impară.
Etapa locală, Botoşani 2010
2. Se consideră numerele reale a1 < a2 < ... < an. DeterminaŃi permutarea σ ∈ Sn pentru care suma
n
Aσ = ∑ (a
i =1
i − a σ (i ) ) 2 este maximă.

Etapa locală, Iaşi 2010


1 2 3 4 5 6 7
3. Să se rezolve în S7 ecuaŃia x 2 =   .
 4 7 6 5 1 3 2
Etapa locală, Harghita 2010
4. Fie n ∈ N, n ≥ 3.
a) Dacă permutarea x ∈ Sn comută cu transpoziŃiile: (1 2), (2 3), ..., (n – 1 n), atunci x = e.
b) Dacă permutarea y ∈ Sn comută cu transpoziŃia (1 2) şi cu ciclul (1 2 3 ... n), atunci y = e.
Etapa locală, Bucureşti 2010, prof. Ion Savu şi Sorin Rădulescu
5. a) Să se ordoneze elementele mulŃimii A = {1, 2, 3, ..., n – 1, n}, n ∈ N, n ≥ 2, astfel încât
maximul dintre cele (n – 1) numere obŃinute prin însumarea a câte doi termeni alăturaŃi să fie cât
mai mic posibil.
b) Pentru n = 2k, k ∈ N, n ≥ 6, fie {a1, a2, ..., a2k} mulŃimea formată cu elementele mulŃimii A
scrise în ordinea găsită la punctul a). Dacă se notează cu Mi punctul de coordonate Mi(a2i–1, a2i), i
∈ {1, 2, ..., k}, să se arate că [M1M2] ≡ [M2M3] ≡ ... ≡ [Mk–1Mk] şi că punctele M1, M2, ..., Mk sunt
coliniare.
Etapa locală, Vrancea 2010
2 2
6. Fie A =   şi X(a) = I2 + aA, a ∈ N*.
 − 1 − 1
a) Să se calculeze X(1) + X(2) + ... + X(n) şi X(1) ⋅ X(2) ⋅ ... ⋅ X(n), n ∈ N*.
b) Să se determine n ∈ N* pentru care X(1) + X(2) + ... + X(n) = n ⋅ X(1) ⋅ X(2) ⋅ ... ⋅ X(n).
Etapa locală, Cluj 2010, prof. Viorel Lupşor

85
7. Se consideră mulŃimea M = {X ∈ M2(C) | X2 = I2}.
a b
a) DemonstraŃi că pentru orice matrice A =   ∈ M2(C) are loc egalitatea A2 – (a + d) ⋅ A +
c d
+ det(A) ⋅ I2 = 02 (relaŃia Cayley-Hamilton).
b) DemonstraŃi că mulŃimea M are o infinitate de elemente.
0 − i 0 − i
c) Există matrice X ∈ M2(C) astfel încât X ⋅   = −  ⋅ X ?
i 0  i 0 
d) DemonstraŃi că oricare ar fi α, β, γ ∈ C, există A, B, C ∈ M2(C) astfel încât (α ⋅ A + β ⋅ B +
+ γ ⋅ C)2 = (α2 + β 2 + γ2) ⋅ I2.
Etapa locală, Maramureş 2010, prof. Vasile Giurgi
 
1
 
8. a) Fie A ∈ M3(R) şi U = 1 ∈ M3,1(R). DemonstraŃi că suma elementelor de pe fiecare linie a
1
 
matricei A este egală cu 2 dacă şi numai dacă A ⋅ U = 2 ⋅ U.
3
b) Dacă B = (bij)1≤i, j≤3, B ∈ M3(R) are proprietatea că ∑b
k =1
ik = λ ∈ R, (∀) i = 1, 3 , arătaŃi că suma

elementelor de pe fiecare linie a matricei Bn este egală cu λn, (∀) n ∈ N*.


m11 + m12 = 5
c) Fie M = (mij)1≤i, j≤2, M ∈ M2(R), astfel încât  . AflaŃi suma elementelor matricei
m 21 + m 22 = 5
M 2.
Etapa locală, Sibiu 2010, prof. Liana Agnola
9. Se consideră A, B, C ∈ M2(R) astfel încât BC = I2 şi funcŃiile f, g: M2(R) → M2(R) pentru care
(f  g)(X) = A ⋅ g(X) + f(X) ⋅ B, (∀) X ∈ M2(R). Să se arate că:
a) există B, C ∈ M2(R), B ≠ I2, C ≠ I2, pentru care BC = I2;
b) dacă f este injectivă, atunci şi g este injectivă.
Etapa locală, Caraş-Severin 2010
1 1 0
 
10. Fie A =  1 1 0  ∈ M3(R) şi şirul de matrice (Xn)n≥0, unde X0 = I3 şi Xn+1 = AXn + A – I3, n ∈
 0 0 2
 
2n
 det X 
∈ N. CalculaŃi Xn şi lim 2 n + 2n  .
n →∞
 2 
Etapa locală, Giurgiu 2010, prof. Şerban Olteanu, Giurgiu
− 1+ i 3
11. Fie matricele A, B ∈ Mn(X), n ≥ 2, astfel încât AB + εA + ε2B = On, unde ε = . Să se
2
arate că AB = BA.
Etapa locală, Arad 2010

86
0 1 1
 
12. Fie matricea A =  1 0 1  . ArătaŃi că:
1 1 0
 
A + I3 2I − A
a) An = αnA + β nI3, αn, β n ∈ Z, n ∈ N*; b) A n = 2 n ⋅ + (−1) n ⋅ 3 .
3 3
Etapa locală, Gorj 2010
2 1 1
 
13. Fie matricea A =  1 2 1  . CalculaŃi An, n ∈ N*.
 1 1 2
 
Etapa locală, DâmboviŃa 2010
1 a − b
 
14. Fie matricea A =  b 1 0  ∈ M3(R). CalculaŃi An, n ∈ N*.
a 0 1 
 
Etapa locală, Sălaj 2010
 1 1 3
 
15. Se dă matricea A =  2 1 2  . Să se calculeze A2010.
 3 1 1
 
Etapa locală, Vâlcea 2010, prof.dr. Cătălin Pană
a − b c  a + b − c 
16. Se dau matricele A =  ; B =   , unde a2 = b2 + c2.
 c a + b  − c a − b
a) CalculaŃi AB, BA şi A + B . b) DeterminaŃi det(An + Bn).
2 2

Etapa locală, Iaşi 2010


 2 0 3
 
17. Să se calculeze A pentru A =  0 3 0  ∈ M3(R).
n

 3 0 2
 
Etapa locală, Harghita 2010
 1 − 3 0
  1
18. Fie A =  3 1 0  . Dacă B = A , să se calculeze B2010.
  2
0 0 2
 
Etapa locală, Teleorman 2010
19. Spunem că o matrice nenulă X ∈ M2(C) este nilpotentă, dacă există n ∈ N astfel încât Xn = O2
(matricea nulă). Fie A, B ∈ M2(C) două matrice nenule, nilpotente. Să se demonstreze că
matricea A + B este nilpotentă dacă şi numai dacă matricele A ⋅ B şi B ⋅ A sunt nilpotente.
Etapa locală, Braşov 2010, prof. Romeo Ilie

87
 1 2010 
20. Se consideră matricea A =  .
0 1 
a b
a) Să se arate că dacă X ∈ M2(R) cu AX = XA, există a, b ∈ R astfel încât X =   .
0 a 
b) Să se determine matricele Y ∈ M2(R) pentru care Y2010 = A.
Etapa locală, Hunedoara 2010
a a
21. Fie A ∈ M2(R), A =   , a ⋅ b ≠ 0.
 b b
a) CalculaŃi An. b) RezolvaŃi ecuaŃia X5 = A, X ∈ M2(R).
Etapa locală, 2010
 −1 0 
22. Să se rezolve în M2(Z) ecuaŃia X3 – 2X =   .
 10 4 
Etapa locală, Harghita 2010
a b
23. a) Fie a, b ∈ R şi A =   . CalculaŃi An, (∀) n ∈ N*.
0 a 
 2 6
b) Dacă A =   , rezolvaŃi în M2(R) ecuaŃia X5 + X = A.
 0 2 
Etapa locală, Cluj 2010, prof. Cristian Pop, ISI Cluj
4 6 
24. Să se rezolve, în mulŃimea M2(R), ecuaŃia: X 2010 =   .
 8 12 
Etapa locală, GalaŃi 2010, prof. Florin Antohe
 0 1 0 
 
25. Să se rezolve în M3(Z), ecuaŃia: X =  0 0 1  .
5

1 0 0
 
Etapa locală, ConstanŃa, 2010, prelucrare Gazeta Matematică
3 1
26. Să se rezolve ecuaŃia: X n =   , n ∈ N*, X ∈ M2(R).
 6 2
Etapa locală, Ilfov 2010
 2 0 0
 
27. Fie A =  0 − 1 0  . DemonstraŃi că doar pentru n impar ecuaŃia Xn = A are soluŃie, X ∈ M3(R).
 1 0 3
 
Etapa locală, Buzău 2010
 a ε2b   3 9
28. Se consideră matricele A =   , a, b, ε ∈ C, B = 
  4 3  şi n ∈ N*.
 b a   

88
 ( a + bε ) n + ( a − bε ) n ( a + bε ) n − ( a − bε ) n 
 ε
a) Să se demonstreze că: A = n  2 2 .
 ( a + bε ) n − ( a − bε ) n ( a + bε ) n + ( a − bε ) n 
 
 2ε 2 
b) Să se calculeze Bn.
c) Să se rezolve în M2(C) ecuaŃia Xn = B.
Etapa locală, Prahova 2010, prof. Cezar Apostolescu, Ploieşti
29. Fie ecuaŃia X + aX + bIn = On, X ∈ Mn(Z), unde a, b, n ∈ Z, n ≥ 2.
2

a) DemonstraŃi că în cazul n = 2 ecuaŃia are o infinitate de soluŃii, oricare ar fi a, b.


b) Pentru care valori ale lui n ecuaŃia are o infinitate de soluŃii, oricare ar fi a, b?
Etapa locală, 2010
30. Dacă A ∈ M4(R) cu proprietatea A = O4, atunci există B ∈ M4(R) astfel încât (I4 + A)3 = B2.
4

Etapa locală, ConstanŃa 2010, prof. Constantin Caragea


 1 2 

31. Se consideră în M2(R) ecuaŃia matriceală X =  1 p
, unde p ∈ N, p ≥ 3 este un număr
 1 
 2 
2 3 n
natural impar fixat. Fie Y(n) = X + X + X + ... + X , unde X este soluŃie a ecuaŃiei matriceale de
mai sus. ArătaŃi că există un şir de numere reale (an)n≥0 astfel încât Y(n) = anX, (∀) n ∈ N*.
Etapa locală, Vrancea 2010
 cos 2 x sin 2 x 
32. Fie matricea A =  2 2 
 , x ∈ R.
 sin x cos x 
 1 + cos n 2x 1 − cos n 2 x 
 
a) ArătaŃi că A n =  2 2  , pentru n natural nenul.
 1 − cos n 2 x 1 + cos n 2 x 
 
 2 2 
n

∑ det A
k =1
k

b) Să se calculeze lim .
n →∞  n 
det  ∑ A k 
 k =1 
Etapa locală, Teleorman 2010
1
33. Un determinant de ordinul al treilea cu elemente reale are elementele egale cu pe diagonala
2
principală, iar pe fiecare linie şi coloană suma elementelor este 1.
a) Să se arate că determinantul este un număr pozitiv.
b) Să se găsească valoarea minimă pe care o poate lua determinantul.
Etapa locală, Teleorman 2010

89
34. Considerăm următorii determinanŃi cu elemente numere reale:
a1 a 2 1 b1 b 2 1 c1 c 2 1 d1 d 2 1
D1 = b1 b 2 1 ; D 2 = c1 c 2 1 ; D 3 = d1 d 2 1 ; D 4 = a1 a 2 1 .
c1 c 2 1 d1 d 2 1 a1 a 2 1 b1 b 2 1
Se ştie că trei dintre ei sunt egali cu 1. DemonstraŃi că şi cel de-al patrulea determinant este egal
cu 1.
35. Fie A ∈ M2011(Z) având toate elementele în mulŃimea {–1, 1}. Să se arate că det(A) este divizibil
cu 22010.
Etapa locală, Arad 2010
36. Fie A ∈ M3(R) cu toate elementele egale cu 1 sau cu –1. Să se arate că 4 divide det(A).
Generalizare.
Etapa locală, Bihor 2010
cos α 1 0 ... 0 0
1 2 cos α 1 ... 0 0
0 1 2 cos α ... 0 0
37. Să se calculeze determinantul ∆n, ∆ n = .
... ... ... ... ... ...
0 0 0 ... 2 cos α 1
0 0 0 ... 1 2 cos α
Etapa locală, Harghita 2010
 a12 + p a1a 2 a1a 3 ... a1a n 
 
 a 2 a1 a 22 + p a 2a 3 ... a 2a n 
38. Fie a1, a2, ..., an, p ∈ Z şi A n =   ∈ Mn(Z).
 ... ... ... ... ... 
 a a a n a 2 a n a 3 ... a 2n + p 
 n 1
Să se arate că det(An) ⋮ pn–1.
39. Fie matricea A ∈ M3(R) cu det(A2 + I2) = 0. Să se calculeze det(A).
Etapa locală, Buzău 2010
40. Fie A, B ∈ Mn(R), n ∈ N, n ≥ 2.
a) Dacă A2 – B2 = AB – BA, arătaŃi că det(A + B) = 0 sau det(A – B) = 0.
b) Dacă A2 + B2 = AB + BA, arătaŃi că det(A – B) = 0.
Etapa locală, Argeş 2010
41. Fie A, B ∈ M3(R) cu A simetrică. Dacă det(AB – B A) = 8 – det(2B), unde Bt este transpusa
t

matricei B, să se calculeze det(B).


Etapa locală, Giurgiu 2010, prof. Paul Băiatu
42. Matricea A = (aij) ∈ M3(R), |aij| = 1. AflaŃi valoarea maximă a determinanŃilor matricelor posibile.
PrecizaŃi matricea corespunzătoare valorii maxime obŃinute.
Interpretare geometrică.
Etapa locală, Gorj 2010

90
43. În sistemul cartezian de coordonate xOy, se consideră punctele A(n + 1, n2 + 1), n ∈ N şi O(0, 0).
a) CalculaŃi aria triunghiului OAnAn+1.
b) Notăm cu f(n) aria triunghiului OAnAn+1. Să se determine n ∈ N*, pentru care f(n) este minimă.
Etapa locală, Argeş 2010
a b
44. Fie A ∈ M2(R), A =   , cu a + d ≠ 0. Să se arate că pentru o matrice B ∈ M2(R) avem
c d
AB = BA dacă şi numai dacă A2B = BA2.
Etapa locală, Harghita 2010
45. Fie X, Y, Z ∈ Mn(R), matrice care comută două câte două, astfel încât (X – Y) ⋅ (Y – X) = On. Să
se arate că det((X – Y)4 + (Y – Z)4 + (Z – X)4) ≥ 0.
Etapa locală, GalaŃi 2010, prof. Dumitru şi Rodica Bălan
46. Fie A ∈ M2(Z) astfel încât det(A2 + A + I2) = det2(A + I2). Să se arate că trA + detA ∈ {–8, 0}.
Etapa locală, Bucureşti 2010, Gazeta Matematică
47. Fie A ∈ M2(R) şi Tr(A) = 1. Să se arate că:
(detA)det(A3 + A + I2) ≤ 2det4A – 4det3A + det2A + detA + 1.
Etapa locală, Ilfov 2010
48. Fie A ∈ M2(R) astfel încât TrA = 1. Să se arate că det(A + A + I3) ≥ 3.
2

Etapa locală, Dolj 2010, Gazeta Matematică 7-8/2004, problema 25011


49. Fie A ∈ M2(R) astfel încât A ⋅ A = I2 şi tr(A) = 2. Să se arate că A = I2.
t

Etapa locală, Argeş 2010, prof. Marin Ionescu, Piteşti


50. a) Să se dea un exemplu de matrice A ∈ M2(Z) pentru care det(A2 + I2) = 2.
b) Să se arate că nu există matrice A ∈ M2(Z) pentru care det(A2 + I2) = 2 ⋅ det(A + I2).
Etapa locală, Caraş-Severin 2010, RMT 1/2009
2π 2π
51. Fie A ∈ M2(C), tr(A) = ε; det(A) = ε2, unde ε = cos + i sin , n ≥ 4. ArătaŃi că:
n n
n

∑ε
k =1
k
⋅ det(A − εI 2 ) = 0 .

Etapa locală, Sibiu 2010, Gazeta Matematică


52. Fie A ∈ M2(Q) astfel încât det(A – 2I2) = 0.2

a) Să se calculeze det A. b) Să se arate că A2 = 2I2.


Etapa locală, Teleorman 2010
53. Fie A ∈ M2(Z) astfel încât det(A + A + I2) = det (A + I2). Să se arate că trA + detA ∈ {–8, 0}.
2 2

(trA = urma matricei A).


Etapa locală, Mureş 2010, prof. Gheorghe GhiŃă
54. Fie A, B ∈ M2(R) cu proprietatea det(A – B) = 0.
a) Să se demonstreze că det(A + B) = 2det(A) + 2det(B).
b) Să se demonstreze că det(tA + (1 – t)B) + det((1 – t) A + tB) = det(A) + det(B), (∀) t ∈ R.
Etapa locală, Hunedoara 2010

91
55. Fie A ∈ M3(C). Să se arate că:
a) det(A – At) = 0; b) dacă A ≠ At, atunci rang(A) = 2.
Etapa locală, Botoşani 2010
56. a) Dacă există A, B ∈ Mn(C) astfel încât A ⋅ B = In, atunci A ⋅ B = B ⋅ A.
b) Dacă există p, q ∈ N* şi A, B ∈ Mn(C) astfel încât pA + qB = A ⋅ B, demonstraŃi că Ap ⋅ Bq =
= Bq ⋅ Ap.
Etapa locală, Braşov 2010, prof. Traian DuŃă
57. Fie n ∈ N, n ≥ 2 şi matricele A, B ∈ Mn(R) astfel încât A + B = In şi A2010 = A2011. Să se arate că
matricea In – A2009B este inversabilă şi să se găsească inversa ei.
Etapa locală, Suceava 2010, prof. Dan Popescu
58. Fie A ∈ Mn(R) cu proprietatea că A2007 + A2008 + A2009 = On şi fie B = A2 + A + In.
ArătaŃi că matricea In – AB este inversabilă.
Etapa locală, Sălaj 2010, Gazeta Matematică 1/2009
59. Fie A ∈ Mn(R), n ≥ 2 şi Ak – Ak+1 + Ak+2 = On, unde k este un număr natural impar, iar B = In –
– A + A2. Atunci avem:
a) In – AB, In + AB inversabile;
b) (I n − AB) −1 + ( I n + AB) −1 = 2[I n + ( AB) 2 + ( AB) 4 + ... + (AB) k −1 ] .
Etapa locală, Brăila 2010, prof. Gheorghe Alexe
60. Fie A, B ∈ Mn(R), astfel încât A3B = A3 – B. Să se arate că AB = BA.
Etapa locală, Olt 2010, prof. Florin Nicolaescu
 1 0 0
 
61. Considerăm matricea A =  0 2 0  .
 0 0 3
 
a) ArătaŃi că A este inversabilă şi calculaŃi A–1.
b) DeterminaŃi numărul soluŃiilor ecuaŃiei X2010 = A, X ∈ M3(C).
Etapa locală, Covasna 2010
 0 − 1 0 
   mn 

62. Fie matricea A =  0 0 1  şi M = {An, n ∈ N*}. RezolvaŃi ecuaŃia X ⋅  A i − A −1  = A 2010 ,
1 0 0  i =1 
 
unde m = card(M), n ∈ N*, A–1 este inversa matricei A.
Etapa locală, Olt 2010, prof. Elena Pîrlog, Florin Nicolaescu
63. Fie A, B ∈ M2n(C) matrici inversabile cu A ⋅ B–1 + A–1 ⋅ B = I2n.
Să se demonstreze că det(A – B)2 ⋅ det[(A–1)2 + (B–1)2] = 1.
Etapa locală, Bihor 2010, Gazeta Matematică
64. Fie A ∈ M3(R) inversabilă, astfel încât tr(A) = tr(A2) = 0. DemonstraŃi că det(A – A–1) = det(A) –
– det(A–1).
Etapa locală, DâmboviŃa 2010, prof. Călin Burduşel

92
 2[ n +1 ] 2[ n + 2 ] 2[ n + 3 ] 
 
65. Pentru fiecare număr natural considerăm matricea de ordinul 3: A n =  2[ n + 4 ] 2[ n + 5 ] 2[ n + 6 ]  .
 [ n +7 ] 
2 2[ n + 8 ] 2[ n + 9 ] 
 
Să se arate că există k ∈ N astfel încât (∀) n ≥ k, n ∈ N, matricea An nu este inversabilă (am notat
cu [a] partea întreagă a lui a).
Etapa locală, Prahova 2010, prof. Emil Vasile, Ploieşti

ANALIZĂ MATEMATICĂ

1. Să se determine numerele reale a şi b, astfel încât:


(5a 2 + 2b 2 + 2) n + 2008 n +1 − n
lim = lim .
n →∞ ( 2a + 6ab) n + 2009 n → ∞ n + 2009 − n + 2008

R.M. „Grigore Moisil”, Alexandria


2. DeterminaŃi a, b, c ∈ R pentru care lim n ( an + bn + c − 2n − 3) = 2010 .
2
n →∞
Etapa locală, Giurgiu 2010, Gazeta Matematică
1 1 1
3. Să se calculeze lim x n şi lim x n n
, unde x n = + + ... + ,
n →∞ n →∞ 1 2 +2 1 2 3 +3 2 n n + 1 + (n + 1) n
n ∈ N*.
Etapa locală, Ilfov 2010
4. a) ArătaŃi că există două şiruri de numere raŃionale, (an), (bn), n ≥ 1, astfel încât:
a
(2 + 5 ) n = a n + b n ⋅ 5 . b) CalculaŃi lim n .
n →∞ b
n
Etapa locală, Gorj 2010
5. În interiorul unui unghi ascuŃit 'XOY se consideră un punct dat P. Se construieşte şirul de puncte
(Pn)n∈N astfel încât P0 = P, P1 este proiecŃia lui P0 pe OX, P2 este proiecŃia lui P1 pe OY, P3 este
proiecŃia lui P2 pe OX, P4 este proiecŃia lui P3 pe OY etc. Să se calculeze lim OPn .
n →∞
Etapa locală, Hunedoara 2010
6. Fie (an) un şir de numere în progresie geometrică crescătoare şi p un număr natural.
1 1 1 1 1 1
Sn = p + p + ... + p şi S'n = p + p + ... + p .
a 2 + a1 a 3 + a 2
p p
a n +1 + a n
p
a 2 − a1 a 3 − a 2
p p
a n +1 − a pn
S
a) Să se arate că R = 'n nu depinde de n. b) Să se calculeze lim S n , lim S n' , lim R p .
Sn n →∞ n→∞ p →∞

7. Să se calculeze: L = lim (8 x 8 + x 7 + 1 − 9 ( x 9 − x 8 + 1) .
x →∞

93
8. Fie şirul x n = { n} + { n + 1} + { n + 2} , n ∈ N*. DemonstraŃi că există subşirurile ( x k n ) n ≥1 şi
( x jn ) n ≥1 ale şirului dat, astfel încât lim x k n = 0 şi lim x j n = 1 .
n →∞ n →∞
Etapa locală, Bucureşti 2010, Prof. Daniel Petriceanu
9. Fie (xn)n≥1, (yn)n≥1 două şiruri de numere reale nenule şi convergente la zero.
x 2n y n
CalculaŃi lim .
n → ∞ 3x 2 − 2 x y + y 2
n n n n
Etapa locală, DâmboviŃa 2010, prof. Călin Burduşel
a +n 1 + 4n − 3 1 + 4n + 1 
10. Se dă şirul (an)n≥1 cu a1 = 1, iar pentru n ≥ 3 are loc a n +1 = n şi a n ∈  ; .
an  2 2 
n
a  a 
CalculaŃi: a) lim (a n − n ) ; b) lim  n  ; c) lim  n  .
n →∞ n → ∞ n  n → ∞ n 

Etapa locală, Buzău 2010


1 a
11. Se dă şirul (an)n≥1 definit prin a1 = 2, a n +1 = n + ⋅ a n , (∀) n ≥ 1. Să se calculeze lim n .
n n →∞ n

Etapa locală, Arad 2010


12. Pentru fiecare n ∈ N* se defineşte funcŃia fn: R → R, fn(x) = x3 + n2 ⋅ x. Se admite cunoscut faptul
n
că toate funcŃiile fn sunt bijective. Se consideră şirurile (an)n≥1, a n = f n−1 (n ) şi (bn)n≥1, b n = ∑a
k =1
k .

1
a) DemonstraŃi că a n < , (∀) n ∈ N*.
n
b) DemonstraŃi că şirurile (an)n≥1 şi (n ⋅ an)n≥1 sunt convergente şi calculaŃi limita fiecăruia.
c) CalculaŃi lim b n , dacă această limită există.
n→∞
Etapa locală, Maramureş 2010,
Prof. Cristian Heuberger, Colegiul NaŃional „Gheorghe Şincai”
13. Fie (xn)n∈N un şir de numere reale, astfel încât |xn+1 – xn| ≤ 2010–n, (∀) n ∈ N.
Să se arate că şirul (xn) este convergent.
Etapa locală, Cluj 2010
14. Fie a, b ∈ R şi (an)n≥1, (b)n≥1 şiruri de numere reale convergente la a, respectiv b. Fie σ o
n

∑a b
k =1
k σ( k )
permutare a numerelor naturale {1, 2, ..., n}. Să se arate că lim = ab .
n →∞ n
Etapa locală, Dolj 2010
15. Fie a ∈ R şi (an)n≥1 un şir de numere reale convergente la a, astfel încât an ≠ a pentru orice n ≥ 1.
GăsiŃi funcŃiile f: N* → N* cu proprietatea că şirul (b)n≥1, unde b n = a f ( n ) , pentru orice n ≥ 1, este
convergent la a.
Etapa locală, Dolj 2010

94
 x n +1 = x n + 2 y n
16. Fie şirurile (xn)n∈N, (yn)n∈N date prin  , x0 = 1 şi y0 = 2. Să se determine xn şi yn
 y n +1 = −2 x n + 5 y n
în funcŃie de n.
Etapa locală, Harghita 2010
17. Se consideră şirul (an)n≥1 cu a1 = 5 şi care verifică relaŃia 2n ⋅ (an+1 – an) + 3an+1 – 5an = 8n2 + 32n +
an
+ 30, (∀) n ∈ N*. Să se calculze lim .
n →∞ n 2

Etapa locală, Maramureş 2010, Gazeta Matematică 11/2009


18. Se consideră şirul (an)n≥1 cu a1 = 1 şi care verifică relaŃia 2n2 ⋅ (an+1 – an – 1) + n(an+1 – 3an) + 2an =
a
 1  n
= n4 + 3n3, (∀) n ≥ 1. Să se calculeze lim 1 + 3  .
n → ∞ n 
Etapa locală, Braşov 2010, prof. Gabriela Boeriu
17 77 x
19. Se consideră şirul (xn)n≥1, în care x1 = ; x 2 = ; 2 x n + 2 = 7 x n +1 − 3x n . CalculaŃi lim n n+1 .
2 4 n →∞ 3

Etapa locală, Giurgiu 2010, prof. Ion Staicu


5x − 2 x n
20. Fie şirul (xn)n≥0, cu x0 = 1, x n + 2 = n +1 , pentru orice n ≥ 0 şi lim x n = 0 . Să se determine
2 n →∞

formula termenului general şi să se calculeze limita lim ( x 0 + 2x1 + 3x 2 + ... + nx n −1 ) .


n →∞
Etapa locală, Harghita 2010
an
21. Fie şirul (an)n≥1, cu a1 = 1 şi a n +1 = . Să se determine formula termenului general şi să se
an + 2
calculeze limita şirului.
Etapa locală, Harghita 2010
3x + 2 x −2
22. a) Fie şirul (xn)n≥0, unde x0 > 0 şi x n +1 = n , n ≥ 0. Fie y n = n , n ≥ 0.
xn + 2 xn +1
DemonstraŃi că (yn) este progresie geometrică şi deduceŃi că şirul (xn)n≥0 este convergent.
az + b
b) Fie a, b, c > 0 şi şirul (zn)n≥0, unde z0 > 0 şi z n +1 = n , n ≥ 0.
zn + c
ArătaŃi că (zn)n≥0 este convergent.
Etapa locală, Brăila 2010, prof. Vasile Radu
1
23. Fie şirul (xn)n≥1, dat prin x1 = şi x n +1 = 4 x 3n , n ≥ 1. Dacă y n = 4x 2n + 2 x n + 1 , n ≥ 1, calculaŃi
4
lim ( y1y 2 ...y n ) .
n →∞
Etapa locală, Olt 2010, prof. Dumitru Vişan
24. Se consideră şirul de numere reale (an)n≥0, definit prin: a0 = 0, a1 = 1, 13a n = 12a n −1 + 5a n −2 , (∀) n ≥
≥ 2. Să se arate că şirul (an)n≥0 este convergent şi să se determine lim a n .
n →∞
Etapa locală, Vrancea 2010

95
25. Se consideră şirul (an)n≥1, a1 ∈ (0, 1) şi a n +1 = a n (1 − a n ) .
a) ArătaŃi că an ∈ (0, 1), (∀) n ≥ 1.
b) DemonstraŃi că (an)n≥1 este convergent şi calculaŃi lim a n .
n →∞
n
c) DemonstraŃi că şirul (bn)n≥1, b n = ∑a
k =1
2
k este convergent superior de a1.

26. Fie (xn)n≥1 un şir de numere reale strict pozitive pentru care există k > 0, astfel încât x 2n ≤ k(xn –
– xn+1), (∀) n ∈ N*.
a) Să se arate că lim x n = 0 .
n →∞

x 2 x3 x
b) Să se demonstreze convergenŃa şirului (yn)n≥1 definit astfel: y n = x1 + + + ... + n , n ≥ 1.
2 3 n
Etapa locală, Bucureşti 2010, prof. Radu Miculescu
27. Fie λ > 0 şi (xn)n≥1 un şir astfel încât xn ∈ (0, 1 + λ] şi x n +1 ≤ x 2n − λx n , (∀) n ∈ N*. Să se arate că
şirul (xn)n≥1 este constant.
Etapa locală, Suceava 2010, prof. Ion Bursuc
28. Fie şirul (an)n∈N cu proprietăŃile:
a) an > 0, (∀) n ∈ N; b) a 2n +1 ⋅ a 2n = 27 + 2a n +1 ⋅ a 2n , (∀) n ∈ N.
Să se arate că lim a n = 3 .
n →∞

Etapa locală, Prahova 2010, prof. Octavian Purcaru, Ploieşti


n
29. ArătaŃi că şirul (an) cu a1 = 1 şi a n +1 = a n − , oricare ar fi n ≥ 1, este divergent.
n +1
Etapa locală, Gorj 2010
30. Fie şirul (xn)n≥1 de numere reale, ce îndeplineşte condiŃiile: (1) x 4n − x 3n ⋅ x n +1 ≥ x 2n − 6x n + 9 ,
(∀) n ∈ N*.
a) Dacă presupunem că şirul (xn)n≥1 este descrescător, arătaŃi că este convergent şi calculaŃi limita
sa.
b) Este posibil ca un şir (xn)n≥1 ce verifică relaŃia (1) să aibă toŃi termenii negativi?
Etapa locală, Sibiu 2010
31. Fie a > 0 şi b, c ∈ R, astfel încât (b – 1) < 4ac. Se consideră şirul de numere reale (xn)n≥1, cu x1 ∈
2

∈ [0, ∞) şi x n +1 = ax 2n + bx n + c , (∀) n ≥ 1.
a) Să se demonstreze că lim x n = ∞ .
n →∞

x n +1
b) Să se demonstreze că lim n =a.
n →∞ x1x 2 ...x n
Etapa locală, Suceava 2010, prof. Angela łigăeru

96
n
32. Fie a1 = 1 şi şirul (an)n∈N* definit prin a n +1 = , (∀) n ≥ 1. ArătaŃi că:
1+ an
a 2n
a) a n ≥ n − 1 , (∀) n ∈ N*; b) lim = 1.
n →∞ n

Etapa locală, Caraş-Severin 2010, RMT 1/2009


ln x n
33. Să se arate că există şiruri de numere reale (xn)n≥1 cu proprietatea că: x n +1 = , (∀) n ≥ 1.
x n −1
Gazeta Matematică 1/2009
34. Fie şirul (xn)n≥1 strict crescător cu proprietatea: x 2n ( x 2n − 1) ≥ x 3n ⋅ x n +1 + 2 x n + 1 , (∀) n ≥ 1.
Să se arate că xn < –1, (∀) n ≥ 1.
Etapa locală, Botoşani 2010, prof. Ioan Băetu
35. Fie (xn)n≥1 şi (yn)n≥1 ce satisfac: 4 x n = ( x n −1 + y n −1 ) 6 + ( x n −1 − y n −1 ) 2 ;
4 y n = ( y n −1 − x n −1 ) 6 + ( x n −1 + y n −1 ) 2 , pentru orice n ≥ 1.
ArătaŃi că cele două şiruri sunt periodice şi determinaŃi perioadele acestora.
Etapa locală, Braşov 2010, prof. Mihaly Bencze
36. Fie şirul (xn)n≥1 dat prin x n +1 = 1 + 2x n − x 2n , (∀) n ≥ 1 şi x1 ∈ (0, 2). Să se studieze convergenŃa
şirului.
Etapa locală, Harghita 2010
37. Considerăm a < b două numere reale strict pozitive. Fie (xn)n∈N şi (yn)n∈N două şiruri de numere
x n + yn
reale definite astfel: x0 = a, y0 = b, x n +1 = ; y n +1 = x n + y n , (∀) n ∈ N. StudiaŃi conver-
2
genŃa celor două şiruri şi în caz de convergenŃă, aflaŃi limitele celor două şiruri.
Etapa locală, Dolj 2010
1 + (12 + 2 2 ) + (12 + 2 2 + 32 ) + ... + (12 + 2 2 + ... + n 2 )
38. CalculaŃi lim .
n →∞ 13 + 33 + ... + (2n − 1)3
[ x ] + [2 p x ] + ... + [n p x ]
39. CalculaŃi lim , x ∈ R, p ∈ N* fixat.
→∞ n p +1
40. Fie progresia aritmetică a1, a2, ..., an, ... cu raŃia şi primul termen strict pozitive.
a) Să se calculeze limita şirului (an).
1  a1 a2 an 
b) Să se calculeze: lim  + + ... + .
n →∞ a  a a1 + a 2 a1 + a 2 + ... + a n 
n  1
Etapa locală, Teleorman 2010

97
41. CalculaŃi limitele următoarelor şiruri (p este un număr natural nenul dat):
1p + 2p + ... + n p  1 
a) a n = ; b) b n = n  a n − .
n p +1  p + 1 
Etapa locală, Iaşi 2010
n
 ln a 
42. a) ArătaŃi că 1 +  ≥ 1 + ln a , (∀) n ∈ N*, a ≥ 1.
 n 
b) DemonstraŃi că a – 1 ≥ ln a, (∀) a ≥ 1.
c) DemonstraŃi că lim ( n 1 + n 2 + n 3 + ... + n n − n ) = ∞ .
n →∞
Etapa locală, ConstanŃa 2010, prof. Nelu Chichirim
43. Fie şirurile (an)n≥0, (xn)n≥0 de numere strict pozitive, cu proprietăŃile:
a 
i) (xn)n≥0 este strict descrescător; ii) lim x n = l , l ≠ 0; iii) x n = ln n −1  , (∀) n ≥ 1.
n →∞
 an 
Să se arate că şirul (an)n≥0 este convergent şi să se calculeze lim a n .
n →∞
Etapa locală, Argeş 2010,
Prof. Cosmin Manea şi Dragoş Petrică, Piteşti
44. Fie (an)n≥1, (bn)n≥1 două şiruri de numere reale strict pozitive, cu următoarele proprietăŃi:
lim (a n +1 − a n ) = a > 0, lim b n = b > 0 , lim n ⋅ (b n +1 − b n ) = c > 0 .
n →∞ n →∞ n →∞

CalculaŃi lim (a n +1 ⋅ b n +1 − a n ⋅ b n ) .
n →∞
Etapa locală, NeamŃ 2010
Propusă de prof. Adrian Sandovici
45. Fie şirul (an)n∈N* de numere reale, cu proprietatea că a1 ∈ (0, 1) şi an+1 = an(1 – an), (∀) n ∈ N*.
ArătaŃi că lim na n = 1 .
n →∞
Etapa locală, Sălaj 2010
1
46. Fie (an)n∈N un şir astfel încât a1 > 0 şi a n +1 = a n + 2 , (∀) n ∈ N.
an + an +1
a) Să se arate că (an) este strict crescător şi nemărginit.
a
b) Să se calculeze lim 3 n .
n →∞ n

Etapa locală, Cluj 2010


2x n + 1
47. Fie şirul de numere reale (xn)n∈N, definit astfel: x0 > 1, x n +1 = , (∀) n ∈ N.
xn + 2
a) Să se studieze convergenŃa şirului (xn)n∈N.
b) Să se calculeze lim n 2 ⋅ ( x n − 1) .
n →∞
Etapa locală, GalaŃi 2010, prof. Mihai Totolici

98
48. Fie şirul (xn)n≥1, xn+1 = x n − x 4n , (∀) n ∈ N*, x1 ∈ (0, 1).
a) Să se arate că şirul (xn)n≥1 este convergent şi să se calculeze limita sa.
b) Să se arate că lim ( x14 + x 42 + ... + x 4n ) = x1 .
n →∞

1
c) Să se arate că lim 3 n ⋅ x n = 3
.
n →∞ 3
Etapa locală, Mureş 2010
1
49. Fie şirul (xn)n≥0, definit prin x0 > 0, x1 + x 2 + ... + x n = , n ≥ 0.
x n +1
Să se calculeze lim n 2 x 3n .
n →∞
Etapa locală, Prahova 2010, Gazeta Matematică 4/2009
1 1 1
50. Fie şirul x1 > 0, x n +1 = + + ... + , (∀) n ≥ 1.
x1 x 2 xn
xn
a) Să se studieze convergenŃa şirului xn. b) Să se calculeze lim
.
n n →∞ 3

Etapa locală, 2010, prof. Marian Ursărescu


51. Să se calculeze: lim (a x + 1 + b 4x + 1 + c 9 x + 1) , a, b, c ∈ R.
n →∞
Etapa locală, Teleorman 2010
x + x + 2 − ax
2
52. DeterminaŃi a, b ∈ R pentru care lim =b.
x →1 x 2 + 2x − 3
Etapa locală, Caraş-Severin 2010
53. Se consideră funcŃia f: R → R, unde:
f ( x ) = 3 x 3 + (a + 2) ⋅ x 2 + (b + 1) ⋅ x + c − 3 x 3 + (m + 1) x 2 + mx + 1 . Să se determine funcŃia, ştiind
că sunt îndeplinite condiŃiile:
i) lim (a ⋅ x 2 + ax + b ⋅ x 2 + bx − 2x ) = 1 ;
x →∞

1 + 22 + 33 + ... + n n
ii) c = lim .
n →∞ nn
iii) m = lim ( n + 3 ⋅ n + 1 − n + 5 ⋅ n + 1 ) .
n →∞
Etapa locală, Bihor 2010
ln(1 + ax + x )
2
54. Să se determine a, b ∈ R, astfel încât lim = 2.
x →0 x + b −1
Etapa locală, Teleorman 2010
ln(e + 2 x ) − ln(e + x )
2 2
55. a) CalculaŃi limita: lim .
x →0 x

99
x x
 x + 1 − x 
b) DeterminaŃi valorile lui a ∈ R – {1}, astfel încât lim 1 + = e2 .
x → ∞ 
 (a − 1) x + 1 
2 2

Etapa locală, Sibiu 2010


56. Să se calculeze limitele:
1 − cos x ⋅ cos 2x ⋅ ... ⋅ cos nx
a) lim , n ∈ N*, dat;
x →0 x2
1
 sin x  x −1
b) lim  ;
x →1 sin 1 
n
1+ x − m 1− x
c) lim , n, m ∈ N, n ≥ 2, m ≥ 2.
x →0 x
Etapa locală, Bihor 2010
57. Să se calculeze limitele:
(x + a x )x −1
a) lim , a, b ∈ (0, ∞) – {1}, b ⋅ e ≠ 1;
x → 0 (x + b x )x − 1
2
(x + a x ) x − b x
b) lim 2 , a, b ∈ (0, ∞) – {1}, b ⋅ e ≠ a.
x →0 (x + b x ) x − a x
Etapa locală, ConstanŃa 2010, prof. Gheorghe Andrei
1
58. a) Să se calculeze: lim[log b (loga x b )] x − a , a, b ∈ (0, 1) ∪ (1, ∞), a ≠ b.
x →a
b) Fie f: R → R o funcŃie surjectivă şi strict crescătoare. Dacă lim f ( x n ) = ∞ , atunci
n →∞
−1
lim f ( x n ) = ∞.
n →∞
Etapa locală, Buzău 2010
ctgx
 n

59. Să se calculeze lim 1 + ln ∏ (1 + k 2 x )  .
x →0
x >0  k =1 
Etapa locală, Vâlcea 2010, prof. Florentina Dicu
60. Dacă a ∈ R este punct de acumulare comun pentru domeniile funcŃiilor reale f şi g, cele două
f (x)
funcŃii nu se anulează într-o vecinătate a lui a, lim f ( x ) = lim g( x ) = 0 şi există lim , arătaŃi
x →a x →a x→ a g ( x )

ln(ln(e + f ( x )) f (x)
că lim = lim .
x →a ln(ln(e + g ( x )) x → a g(x )
Etapa locală, Iaşi 2010
n

∑ (n n
ek − k )
61. CalculaŃi limita şirului a n = k =1
.
n2
Etapa locală, Covasna 2010

100
n
 k 1


 ∑ a in 

62. CalculaŃi lim  i =1
 , unde ai > 0, (∀) i = 1, k .
n →∞ k
 
 
 
ex − x −1
63. Să se calculeze, fără a folosi regula lui l'Hospital, lim , ştiind că limita există şi este
x →0 x2
finită.
Etapa locală, Vrancea 2010
64.
64. Fie f: R → R o funcŃie cu proprietatea că f (p − q 3 ) = p − q 2 , (∀) p, q ∈ N. Dacă f are limită în
punctul x0 = 0, să se calculeze lim f ( x ) .
x→ 0
Etapa locală, Arad 2010
65. ArătaŃi că dacă a, b ∈ R şi |2x + 3sin x + ax + b| ≤ |x3|, (∀) x ∈ (–1, 1), atunci a = –ln 6.
Etapa locală, Sălaj 2010
66. Se consideră funcŃia f: R → (0, ∞), care verifică relaŃia f(x) ⋅ ln f(x) = e , (∀) x ∈ R.
x

f (x)
 ln x  x
CalculaŃi lim 1 +  .
x →∞
 f (x) 
Etapa locală, DâmboviŃa 2010
67. Fie f: R → R, f(x) = x + x. 3

f −1 ( x )
a) Să se verifice că f este bijectivă. b) Să se calculeze lim , n ≥ 2.
x →∞ n x

Etapa locală, Botoşani 2010, prof. Ioan Băetu


 1  1
68. Fie funcŃia f: R → R crescătoare, cu proprietatea lim f  x +  = lim f  x −  . DemonstraŃi că
n →∞  n  n →∞  n
pentru orice x0 ∈ R există lim f ( x ) şi lim f ( x ) = f ( x 0 ) .
x→x0 x→x 0

Etapa locală, Hunedoara 2010, Gazeta Matematică, nr. 7-8-9/2009


69. Fie funcŃia f: [0, 1] → [0, ∞) cu proprietăŃile:
a) (∀) a, b ∈ [0, 1] astfel încât a + b ≤ 1 ⇒ f(a) + f(b) ≤ f(a + b);
b) f(1) = 1.
Se cere:
1) să se demonstreze că funcŃia are limite laterale finite în orice punct x0 ∈ (0, 1);
2) să se construiască o funcŃie cu proprietăŃile a şi b).
Etapa locală, GalaŃi 2010, prof. Constantin Ursu
70. Fie f: R → R o funcŃie continuă astfel încât f(x) – f(y) = (x – y) ⋅ f(x) ⋅ f(y), (∀) x, y ∈ R.
Să se arate că funcŃia f este identic nulă.
Etapa locală, Mureş 2010

101
CLASA A XI
XII-A

ALGEBRĂ

I. Legi de compoziŃie

1. Pe R definim legea x  y = xy – 3x – 3y + 12.


a) Să se demonstreze că legea este asociativă şi admite element neutru.
b) Să se calculeze (2010)  (–2009)  (–2008)  ...  2009  2010.
Etapa locală, Hunedoara 2010
2. Pe R se defineşte legea de compoziŃie x  y = xy – 2008x – 2008y + 2008 ⋅ 2009.
a) Să se arate că x  x ≥ 2008, (∀) x ∈ R.
b) Să se calculeze valoarea numărului c, unde c = (–3000)  (–2999)  ...  (–1)  0  1  ...  3000.
c) Să se determine părŃile stabile finite ale lui R în raport cu legea de compoziŃie .
Etapa locală, Argeş 2010
3. Pe Q introducem legea „&” definită prin x & y = xy + 5x + 10y + a, a ∈ Q. Să se determine
valorile lui a pentru care intervalul (–5, +∞) este parte stabilă a lui Q în raport cu legea „&”.
Etapa locală, Botoşani 2010
4. Fie A o mulŃime pe care am definit o operaŃie, notată multiplicativ, cu proprietăŃile:
1) dacă a, b, c ∈ A şi ab = c, atunci ac = b;
2) aa = a, pentru orice a ∈ A.
Să se demonstreze că:
a) pentru orice a, b ∈ A ecuaŃia ax = b are o singură soluŃie pe mulŃimea A;
b) dacă A este finită şi ab ≠ b pentru orice b ≠ a, atunci A are un număr impar de elemente.
Etapa locală, Covasna 2010
5. Pe mulŃimea finită M considerăm definită operaŃia „”, care satisface următoarele proprietăŃi:
i) există e ∈ M astfel încât x  e = x, (∀) x ∈ M;
ii) (x  y)  (x  z) = z  y, (∀) x, y, z ∈ M.
Considerăm mulŃimea A ⊂ M, definită prin A = {x ∈ M | e  x = x}.
a) Să se demonstreze că x  x = e, (∀) x ∈ M.
b) Să se demonstreze că mulŃimea A este parte stabilă a mulŃimii M.
Etapa locală, Suceava 2010, prof. Angela łigăeru

102
II. Grupuri
6. Fie mulŃimea G = (1, ∞) şi legea de compoziŃie definită pe G prin x  y = x 2 y2 − x 2 − y2 + 2 ,
(∀) x, y ∈ G.
a) Să se arate că (G, ) este grup abelian.
b) Să se determine numerele reale m şi n, astfel încât funcŃia f: (0, ∞) → (1, ∞), f ( x ) = mx + n
să fie izomorfism între (R*+, ⋅) şi (G, ).
 ... x = 2 , unde x ∈ G.
c) Să se rezolve ecuaŃia x x
de 2010 ori
7. Fie M o mulŃime nevidă şi „&” o lege de compoziŃie pe M. Spunem că d ∈ M este destroyer
pentru operaŃia „&” dacă d & x = x & d = d, (∀) x ∈ M. Pentru a, b ∈ N* – {1} fixate, definim pe
R operaŃia „&” prin x & y = bxy + abx + aby + a2b – a, (∀) x, y ∈ R.
a) ArătaŃi că (G, &) este grup abelian, unde G = R – {–a}.
b) DemonstraŃi că „&” admite pe R element destroyer.
c) DemonstraŃi că (–2010a) & (–2009a) & ... & 0 & a & ... & (2010a) < 0.
Etapa locală, Alba 2010
8. Pe R se defineşte legea de compoziŃie: x & y = xy + 5x + 5y + 20.
a) Dacă G = (–5, +∞), să se arate că (G, &) are structura de grup abelian.
b) Să se arate că grupurile (G, &) şi (R*+, ⋅) sunt izomorfe.
c) Să se determine părŃile stabile finite, nevide ale lui R în raport cu legea „&”.
Etapa locală, Vrancea 2010
9. Definim pe mulŃimea G = (3, ∞) operaŃia: x & y = xy – 3x – 3y + 12, (∀) x, y ∈ (3, ∞).
ArătaŃi că (G, &) formează grup izomorf cu grupul (R, +).
Etapa locală, Teleorman 2010
10. Fie mulŃimea M = (a, ∞) şi legea x & y = xy – ax – ay. Să se determine a ∈ R astfel ca (M, &) să
aibă structură de grup.
Etapa locală, Argeş 2010
( x + y)(1 + a ) 2
11. Pe mulŃimea G = (–1 – a, 1 + a), a > 0, se defineşte operaŃia x & y = .
xy + (1 + a ) 2
a) ArătaŃi că (G, &) formează o structură de grup abelian.
1+ a
b) CalculaŃi α∗ α α , unde α =
∗ ... ∗ .
n ori 2
Etapa locală, Sibiu 2010, prof. Monica Pau
12. Fie G1 = (–a + b, a + b), G2 = (–1, 1), unde a, b ∈ R (a ≠ 0) şi operaŃiile z, S definite astfel:
bxy + (a 2 − b 2 )( x + y − b)
xzy= , (∀) x, y ∈ G1 şi
xy − b( x + y) + a 2 − b 2
2 n +1
 2 n +1 x + 2 n +1 y 
xSy=   , (∀) x, y ∈ G2, n ∈ N*.
 1 + 2 n +1 xy 
 
ArătaŃi că (G1, z) şi (G2, S) sunt grupuri abeliene şi stabiliŃi un izomorfism între ele.
Etapa locală, Braşov 2010, prof. Mihaly Bencze

103
13. Pe G = [0, 1] se defineşte legea de compoziŃie x  y = {x + y}.
a) Să se arate că (G, ) este grup abelian.
 1 2 n − 1
b) Dacă n ∈ N, n ≥ 2 şi H n = 0, , , ...,  , să se arate că (Hn, ) este subgrup al lui (G, ).
 n n n 
14. a) Să se determine toate funcŃiile continue f: R → R pentru care mulŃimea
 1 x f ( x )  
  
G =  0 1 x  x ∈ R  este parte stabilă a lui M3(R) în raport cu înmulŃirea matricelor.
 0 0 1  
 
b) Dacă G este mulŃimea determinată la a), să se arate că (G, ) este grup abelian.
Etapa locală, NeamŃ 2010, prof. Adrian Sandovici
n
15. Se consideră mulŃimea G = {fn: (2, ∞) → (2, ∞) | fn(x) = 2 + ( x − 2) 2 , n ∈ Z}. Să se arate că, în
raport cu operaŃia de compunere a funcŃiilor, G are o structură de grup abelian, izomorf cu (Z, +).
Etapa locală, Caraş-Severin 2010
 2 1
16. Fie A =   şi G = {Xa ∈ M2(R) | Xa = I2 + aA, a > –1}. DemonstraŃi că G este grup
 − 2 − 1
abelian în raport cu înmulŃirea matricelor, izomorf cu grupul aditiv al numerelor reale.
Etapa locală, Iaşi 2010
2 2
17. Se consideră matricea A =   şi mulŃimea G = {X(a) = I2 + a ⋅ A | a ∈ R – {–1}}.
 − 1 − 1
a) Să se demonstreze că X(a) ⋅ X(b) = X(a + b + a ⋅ b), (∀) a, b ∈ R – {–1}.
b) Să se demonstreze că mulŃimea G formează un grup abelian în raport cu înmulŃirea matricelor
din M2(R).
c) Să se calculeze X(1) ⋅ X(2) ⋅ ... ⋅ X(2010).
Etapa locală, Maramureş 2010
 a 0 bi  
  
18. Fie G =  0 0 0  a , b ∈ R, a + b > 0 .
2 2

 bi 0 a  
  
a) DemonstraŃi că (G, ⋅) formează grup, izomorf cu grupul (C*, ⋅).
1 0 i 
 
b) Dacă A =  0 0 0  , să se calculeze A2009.
 i 0 1
 
Etapa locală, Teleorman 2010

104
19. Se consideră mulŃimea G = {A ∈ M2(R) | A + I2 este inversabilă}, pe care se defineşte legea A 
 B = AB + A + B.
a) VerificaŃi că A  B = (A + I2) ⋅ (B + I2) – I2.
b) ArătaŃi că (G, ) este grup.
0 0 x 0
 
1 0 0 0
20. Fie A =  ∈ M4(R) şi G = {An | n ∈ N*}. AflaŃi valorile lui x, y ∈ [–1, 0) pentru
0 0 0 y
 
0 1 0 0
 
care (G, ⋅) este grup.
Etapa locală, Harghita 2010
21. Fie (G, ) un grup şi A = {f: G → G | f(x  y) = x  f(f(y))}; x, y ∈ G.
Să se determine cardinalul mulŃimii A.
Etapa locală, Olt 2010, prof. Florin Nicolaescu, Balş
22. Fie (G, ⋅) un grup cu proprietatea că există o funcŃie f: G → G injectivă, astfel încât:
x2f(x3(f  f)(y)) = f(xf(y))x2, (∀) x, y ∈ G. Să se arate că (G, ⋅) este grup abelian.
23. Fie a ∈ R şi mulŃimea Ma = {(a + i)n | n ∈ Z} ⊂ C. ArătaŃi că:
a) (Ma, ⋅) este grup comutativ;
b) grupurile (Ma, ⋅), (Z, +) sunt izomorfe;
c) determinaŃi intersecŃia grupurilor (M1, ⋅) şi (M2, ⋅).
Etapa locală, Sălaj 2010
k (a + b)2
24. Fie k > 0. Pe mulŃimea G = (–k, k) se defineşte operaŃia algebrică a  b = 2 .
k + ab
a) Să se arate că (G, ) este grup abelian.
t 1
b) DemonstraŃi că f: G → R, f (t) = ∫ dx este un izomorfism de grupuri de la (G, ) la
0 k − x2
2

(R, +).
Etapa locală, Cluj 2010
25. Fie (G, ⋅) un grup.
a) Să se arate că f: G → G, f(x) = axa–1 este automorfism, (∀) a ∈ G.
b) Dacă (G, ⋅) are un singur automorfism, atunci (G, ⋅) este abelian şi x2 = e, (∀) x ∈ G.
26. Fie (G, ⋅) un grup multiplicativ. Definim funcŃia fa: G → G, fa(x) = axa–1, a ∈ G.
a) Să se demonstreze că fa este automorfism al grupului (G, ⋅) pentru orice a ∈ G.
def
b) Să se demonstreze că mulŃimea Int(G) = {fa | a ∈ G} este subgrup al grupului (Aut(G), ).
c) Să se demonstreze că orice grup cu un singur automorfism este comutativ.
Etapa locală, Vâlcea 2010, prof. Roxana Aron

105
27. Fie (G, ⋅) un grup şi H o submulŃime a lui G, nevidă şi diferită de G, cu proprietatea: (∀) x ∈ H,
(∀) y ∈ G – H ⇒ xy ∈ G – H. Să se arate că H este un subgrup al lui G.
Etapa locală, Caraş-Severin 2010, Gazeta Matematică
28. Fie (G, ⋅) un grup şi H un subgrup propriu. Pentru p ∈ N, p ≥ 2, se consideră A = {x ∈ G |
xp ∈ H} şi B = {x ∈ G | xp+1 ∈ H}.
a) Să se arate că A ≠ ∅ şi B ≠ ∅.
b) Să se arate că (A – B) ∩ H = ∅.
Etapa locală, ConstanŃa 2010, prof. Gabriela Constantinescu
29. Fie (G, ⋅) grup multiplicativ şi f, g: G → G, f(x) = x2009, g(x) = x2010 morfisme de grup. Ştiind că f
este morfism surjectiv, să se arate că (G, ⋅) este grup abelian.
Etapa locală, Mureş 2010
30. a) Dacă în grupul (G, ⋅) elementele a, b ∈ G au proprietatea a ⋅ b = b ⋅ a, atunci am ⋅ bp = bp ⋅ am,
(∀) m, p ∈ N*.
b) Dacă în grupul (G, ⋅) cu elementul neutru e, elementele x, y ∈ G verifică egalităŃile x5 = e şi
y2 = x ⋅ y ⋅ x–1, atunci y31 = e.
Etapa locală, Sibiu 2010
31. Fie (G, ⋅) grup şi x, y ∈ G. ArătaŃi că:
a) (xyx-1)2009 = xy2009x–1;
b) dacă (yxy–1)4 = e şi xy = yx2, atunci x = e.
Etapa locală, Sălaj 2010, Gazeta Matematică B. 3/2009
32. Fie n ∈ N* şi (G, ⋅) un grup. Dacă există a, b ∈ G astfel încât ababa = babab, arătaŃi că an = e dacă
şi numai dacă bn = e.
Etapa locală, Suceava 2010, prof. Ion Bursuc
33. Fie (G, ⋅) un grup, a ∈ G şi Ha = {a | k ∈ Z}. ArătaŃi că:
k

a) Ha este subgrup al lui G;


b) Ha este finită dacă şi numai dacă există h, k ∈ Z, h ≠ k astfel încât ah = ak.
34. a) Să se arate că niciun grup nu poate fi scris ca reuniunea a două subgrupuri proprii ale sale.
b) Să se dea un exemplu de grup care poate fi scris ca reuniunea a trei subgrupuri proprii ale sale.
Etapa locală, Vrancea 2010

106
III. Grupuri finite

35. Pentru a ∈ R se definesc funcŃiile fa: R → R, fa(x) = x + a, ga: R → R, ga(x) = –x + a.


Fie G = {fa | a ∈ R}∪ {ga | a ∈ R}.
a) DemonstraŃi că (G, ) este grup necomutativ.
b) DeterminaŃi mulŃimea A = {n ∈ N* | (∃) h ∈ G, ord(h) = n}.
Etapa locală, Gorj 2010
ax = b
36. Fie (G, ⋅) un grup finit de ordin impar şi a, b ∈ G. Să se rezolve sistemul  în G.
bx = a
Etapa locală, Alba 2010, Gazeta Matematică 1/2009
1 2 3 4 5 6 7 1 2 3 4 5 6 7
37. Se consideră permutările σ =   , π =   din S7 şi
 2 3 1 5 6 7 4  4 3 7 5 6 1 2
H = {Gk | k ∈ N*}.
a) Să se arate că H este subgrup al lui S7 şi să se determine m ∈ N* pentru care H este izomorf cu Zm.
b) Să se determine o permutare θ ∈ S7 pentru care π = θσθ–1.
c) Să se determine cardinalul mulŃimii M = {xσ–1x | x ∈ S7}.
Etapa locală, Bucureşti 2010, prof. Costel Chiteş
38. Fie (G, ⋅) un grup finit cu proprietatea că există a ∈ G, a ≠ e, astfel încât x2 = e, (∀) x ∈ G – {a}.
DemonstraŃi că a2 = e, unde e este elementul neutru al grupului.
Etapa locală, Arad 2010
39.
39. (G, ⋅) este un grup în care mulŃimea formată din ordinul elementelor conŃine n numere naturale
consecutive (n ≥ 2). Să se arate că grupul este comutativ dacă şi numai dacă n = 2.
Etapa locală, Harghita 2010
40. Fie (G, ⋅) un grup cu 2n + 1 elemente, astfel încât f: G → G, o funcŃie cu proprietatea că f(xf(xy)) =
= yf(x2), (∀) x, y ∈ G. Să se demonstreze că G este grup comutativ.
Etapa locală, Hunedoara 2010, Gazeta Matematică 6/2009
41. Să se demonstreze că într-un grup G, următoarele afirmaŃii sunt echivalente:
a) Orice parte stabilă faŃă de operaŃia grupului este subgrup al lui G.
b) Orice element din G are ordin finit.
Etapa locală, Olt 2010
42. Fie n ∈ N, n ≥ 3 şi G un grup finit cu n elemente. Să se arate că n ∈ {3, 4} dacă şi numai dacă
există a, b ∈ G distincte, astfel încât G – {a, b} este subgrup.
Etapa locală, ConstanŃa 2010, prof. Cătălin Zîrnă
43. Fie (G, ⋅) un grup finit de ordin 6 cu element neutru e. Fie a, b ∈ G, a ≠ e, b ≠ e, astfel încât a3 = e
şi b2 = e. Notăm ab = c.
a) DemonstraŃi că c = ba sau c = ba2.
b) Să se calculeze ordinul elementului c ∈ G, unde c = ba, respectiv c = ba2.
Etapa locală, NeamŃ 2010, prof. Radu Mititelu

107
44. Fie un grup (G, ) cu 625 elemente în care toate elementele au acelaşi ordin p. DeterminaŃi
valoarea lui p şi daŃi exemplu de grup în condiŃiile problemei.
Etapa locală, Giurgiu 2010, prof. Şerban Olteanu
45. Fie (G, ⋅) un grup şi H o submulŃime a lui G. Spunem că H este generator de ordinul 2 pentru G,
dacă pentru orice element x ∈ G există două elemente a, b ∈ H, astfel încât x = a ⋅ b.
n 
a) Să se arate că orice submulŃime H a lui G cu cel puŃin   + 1 elemente este generator de
2
ordinul 2 pentru G.
b) Să se dea exemplu de grup finit cu cel puŃin 4 elemente care nu are generator de ordinul 2 o
n 
mulŃime finită cu   elemente.
2
Etapa locală, Braşov 2010, prof. Cătălin Ciupală
46. Fie (G, &) un grup şi H ≠ G un subgrup al său cu proprietatea că oricare ar fi a, b ∈ G – H, rezultă
a & b ∈ H. Să se arate că dacă (G1, ⋅) este un grup finit, atunci numărul morfismelor de grupuri
f: G → G1, astfel încât f(H) = {e1}, este egal cu numărul elementelor mulŃimii {y ∈ G1 | y2 = e1},
e1 fiind elementul neutru din grupul (G1, ⋅).
Etapa locală, Dolj 2010, Gazeta Matematică
47. Fie (G, ⋅) un grup abelian finit. Spunem că subgrupul H al lui G are proprietatea (A), dacă G ≠ H şi
produsul elementelor lui H este egal cu produsul elementelor din G – H. Să se arate că dacă G are
un subgrup cu proprietatea (A), atunci orice subgrup al lui G, diferit de G, are proprietatea (A).
Etapa locală, Dolj 2010

IV. Inele. Corpuri


48. Fie (A, +, ⋅) un inel cu proprietatea că x2 = x, (∀) x ∈ A. Să se arate că:
a) 1 + 1 = 0;
b) inelul este comutativ;
c) dacă A este corp, atunci A = Z2.
Etapa locală, Arad 2010
49.
49. Fie (A, +, ⋅) un inel astfel încât (∀) x, y ∈ A, x ⋅ (y – y) = (y – y) ⋅ x.
2 2

Să se demonstreze că inelul (A, +, ⋅) este comutativ.


Etapa locală, GalaŃi 2010, prof. Visilina GuiŃă
50. Fie A un inel şi funcŃia f: A × A → A definită prin f(x, y) = (xy)2 – x2y2.
a) CalculaŃi valoarea expresiei: E(x, y) = f(1 + x, 1 + y) – f(1 + x, y) – f(x, 1 + y) + f(x, y), unde 1
este elementul unitate al inelului A.
b) Dacă inelul A are proprietatea că x + x = 0 implică x = 0 şi dacă (xy)2 – 2(yx)2 = x2y2 – y2x2,
(∀) x, y ∈ A, atunci A este comutativ.
Etapa locală, Bihor 2010
51. Fie A un inel cu proprietatea că dacă x ∈ A şi x = 0, atunci x = 0. Fie a, b, c ∈ A, astfel încât a =
2

= ab, b = bc, c = ca. Să se arate că a = b = c.


Etapa locală, Prahova 2010, Gazeta Matematică 9/2008

108
52. Fie A ∈ M4(Z4), aij = i + j şi H = {kA | k ∈ N*}.
a) Câte elemente are mulŃimea H?
b) Să se demonstreze că (H, +) este grup comutativ.
Etapa locală, Covasna 2010
53. Fie A ∈ M2(Z3) astfel încât G = {1, A, A2} este un grup finit cu trei elemente în raport cu
înmulŃirea matricelor. DeterminaŃi toate grupurile G de acest fel.
54. Să se determine cel mai mic număr natural n, cu proprietatea că (∃) x, y, z ∈ Zn* distincte două
câte două astfel încât x + y = 2z şi xy = z2.
Etapa locală, Gorj 2010
 1̂ 5̂ 4̂ 
 
55. DeterminaŃi matricea A ∈ M3(Z7) ştiind că adjuncta sa este A =  4̂ 1̂ 5̂  .
*

 
 5̂ 4̂ 1̂ 
Etapa locală, Vâlcea 2010
 â b̂    
56. Se consideră mulŃimea: M = 
56.  â , b̂ ∈ Z 7  şi matricea A =  4̂ 3̂  ∈ M. Să se rezolve în M
  
 b̂ â    3̂ 4̂ 
ecuaŃia X6 = A.
Etapa locală, Mureş 2010

ANALIZĂ MATEMATICĂ

I. Primitive
1. CalculaŃi:
( x 2 − 1)dx
a) I1 = ∫x x 4 + 2x 3 + x 2 + 2x + 1
, x ∈ (0, +∞).

Prof. Gabriel Necula, Plopeni


9 x + 9mx + (18 + 2m ) x + 6m
3 2 2
b) I 2 = ∫ (3x 2 + 2mx + 5) n
dx , x ≥ 0, m ≥ 0, n ∈ N*, n ≥ 3.

Etapa locală, Prahova 2010, prof. Mihaiela Doinaru, Sinaia


2. Fie f: R → R, f(x) = (ax + b)sin λx, a, b, λ ∈ R. ArătaŃi că există A(x) = mx + b şi B(x) = px + q,
m, n, p, q ∈ R, astfel încât ∫ f (x )dx = A(x ) ⋅ sin λx + B(x) ⋅ cos λx + C , unde C ∈ R.
Etapa locală, Sibiu 2010
3x + 2 x + x − 2010
4 3 2
3. Să se calculeze: ∫ (x4
+ x 3 + x 2 + 2010) 2
dx , x ∈ R.

Etapa locală, Giurgiu 2010, prof. Paul Băiatu

109
2 ⋅ x 3 + 9 ⋅ x 2 + 17 ⋅ x + 12
4. Se consideră I n = ∫ dx , x ∈ R, n ≥ 1, n ∈ N.
( x 2 + 3 ⋅ x + 3) n
a) Să se calculze I1 şi I2.
b) Să se calculeze In, (∀) n ∈ N, n ≥ 3.
Etapa locală, GalaŃi 2010, prof. Romeo Zamfir
5. Fie f: R → R o funcŃie care admite primitivele F: R → R, care satisfac relaŃia f(x) + 2 ⋅ F(x) =
= sin2 3x, (∀) x ∈ R. Să se demonstreze că există o singură primitivă F care este mărginită pe R.
Să se calculeze această primitivă şi să se determine perioada ei principală.
Etapa locală, Brăila 2010, prof. Iulian Danielescu
 π
6. Se consideră mulŃimea D = 0,  şi funcŃia f: D → R, f(x) = sin x ⋅ cos 3x.
 2
 π
a) ArătaŃi că există a, b ∈ D – 0,  , pentru care f(a) ∈ Q şi f(b) ∈ R – Q.
 2
 π
b) DemonstraŃi că dacă F este o primitivă a funcŃiei f pentru care F(0) ∈ Q, atunci F  ∈ Q.
2
Etapa locală, Caraş-Severin 2010
 π 2 sin x + 3 cos x
7. Fie f:  0,  → R, f ( x ) = .
 2  3 sin x + 2 cos x
a) Să se demonstreze că există A şi B reale, astfel încât:
A (3 sin x + 2 cos x ) + B(3 cos x − 2 sin x )
f (x) = .
3 sin x + 2 cos x
b) Să se calculeze ∫ f (x )dx .
Etapa locală, Teleorman 2010
 1
 arctg , x ≠ 0
8. ArătaŃi că funcŃia f: R → R, f ( x ) =  x nu are primitive pe R.
 0, x=0

Etapa locală, Harghita 2010
sin x  π
9. a) Să se calculeze ∫ sin x + cos x dx , unde x ∈  0, 2  .
arcsin x
b) Să se calculeze ∫ x2
dx , unde x ∈ (0, 1).

Etapa locală, Hunedoara 2010


10. Să se calculeze I = ∫ sin x ⋅ ln(2 + sin 2 x )dx .
Etapa locală, NeamŃ 2010, prof. Marian Ursărescu

110
1
11. DeterminaŃi primitivele funcŃiei f: (0, 2π) → R, f ( x ) = .
3 + 2 cos x
Etapa locală, Iaşi 2010
1
12. Fie f: R → R, f ( x ) = .
3 + sin x + cos x
a) DeterminaŃi primitivele lui f pe [0, π ) . b) DeterminaŃi primitivele lui f pe [0, 2π).
Etapa locală, Sălaj 2010
1
13. Fie f: [0, 2π] → R, f ( x ) = . Să se determine o primitivă a lui f.
2 + sin x
Etapa locală, Vrancea 2010
 sin x + 1 sin x − 1  1  π π
14. Să se calculeze: I = ∫  3 +3  ⋅ 2
dx , unde x ∈  − ,  .
 cos x cos x  cos x  2 2
Etapa locală, Cluj 2010
ln x + 2009
15. Să se calculeze pe (0, ∞) integrala: ∫
( x + ln x + 2010) 2
dx .

16. Să se calculeze integralele:


x2  π
a) I1 = ∫ ( x cos x − sin x )( x sin x + cos x )
dx, x ∈  0,  ;
 2
x2  π
b) I 2 = ∫ ( x sin x + cos x ) 2
dx , x ∈  0,  .
 2
Etapa locală, Harghita 2010
sin x
17. a) DeterminaŃi primitivele funcŃiei f: (0, ∞) → R, unde f ( x ) = .
e + sin x + cos x
x

x11 x3
b) Fie I = ∫
x16 + 1
dx , x > 0; J =
x16 + 2 ∫
dx , x > 0. CalculaŃi I + J, I – J şi apoi determinaŃi I.

18. Să se calculeze:
x2 − x 2x + 1 5 −1
a) ∫ ex + x 2 + x + 1
dx , x ∈ R; b) ∫ (x − 1) ⋅ x ⋅ (x + 1)(x + 2) + a dx , x > 2
, a ∈ R.

Etapa locală, Argeş 2010


sin 2 x + sin 2 x + 1
19. Să se calculeze ∫ e − x + sin 2 x + 1
dx .

x − arctgx
20. DeterminaŃi primitivele funcŃiei f: (0, ∞) → R, f ( x ) = 2 ⋅ .
(1 + x 2 ) 2 (arctgx )3
Etapa locală, Bihor 2010

111
 1   π
21. Să se calculeze ∫ e x  2
+ ln cos x dx , x ∈  0,  .
 cos x   2
Etapa locală, ConstanŃa 2010, Gazeta Matematică
 2
sin , x ≠ 0
22. Să se arate că funcŃia f: R → R, f ( x ) =  x admite primitive.
0, x=0
Etapa locală, Argeş 2010
 2
sin , x ≠ 0
23. ArătaŃi că funcŃia f: R → R, f ( x ) =  x 2 admite primitive pe R.
0, x=0
Etapa locală, Botoşani 2010
 1
 sin  ln x , x > 0
24. Fie f: [0, +∞) → R, f ( x ) =  x  . ArătaŃi că f admite primitive pe [0, +∞).
0, x=0

Etapa locală, ConstanŃa 2010, prof. Dorin Arventiev
 1
ln x ⋅ cos , x > 0
25. Să se determine m ∈ R astfel încât funcŃia f: R+ → R, f ( x ) =  x să admită
m, x=0
primitive.
Etapa locală, Mureş 2010
26. Fie f: R → R o funcŃie care admite primitive, astfel încât f(0) = 1 şi F: R → R o primitivă a lui f
f (x)
care verifică relaŃia f(x) = 2010 ⋅ F(x), (∀) x ∈ R. CalculaŃi ∫f ( x ) + 2010
2
dx , x ∈ R.

Etapa locală, Brăila 2010, prof. Carmen Botea


27. FuncŃia f: (0, ∞) → (0, ∞) are o primitivă F care satisface condiŃia (x2 + 1)F(x) = xf(x), pentru
(∀) x ∈ (0, ∞). Să se determine funcŃia f, dacă f(1) = 1.
Etapa locală, Harghita 2010
28. a) Fie f: (0, ∞) → R o funcŃie derivabilă şi F o primitivă a sa, care verifică egalitatea:
x
F(x) = (2f (x) − x − 4) + 2arctgx , (∀) x ∈ (0, ∞). Să se determine funcŃia f.
2
1
b) CalculaŃi ∫x(x 4020
+ 1)
dx .

Etapa locală, Gorj 2010


29. Fie f: R → R o funcŃie derivabilă care satisface egalitatea (f'(x))2 = (f(x))2 + 4, (∀) x ∈ R.
a) DemonstraŃi că f este strict monotonă.
3
b) DeterminaŃi funcŃia f în cazul în care este strict crescătoare şi f (0) = − .
2
Etapa locală, Caraş-Severin 2010, RMT 1/2009

112
30. a) Să se determine funcŃia f: [0, ∞) → (0, ∞) care admite o primitivă F cu proprietatea:
ex ⋅ F(x) = f(x), (∀) x ≥ 0 şi f(0) = e.
b) Să se arate că nu există funcŃii f: R → R* care admit primitive, astfel încât pentru o primitivă
F a lui f să avem: F(x) ⋅ F(1 – x) = F(x2), (∀) x ∈ R.
31. Fie a ∈ R* şi f, g, h: R → R astfel încât g(x) = (x + a)f(x) şi h(x) = xf(x + a). Să se arate că dacă g
şi h admit primitive pe R, atunci şi f admite primitive pe R.
Etapa locală, Vrancea 2010
32. Fie I, J intervale şi f: I → R* o funcŃie cu primitiva F: I → J bijectivă.
F −1 ( x )
a) Să se calculeze: ∫ f (F −1 ( x ))
dx , x ∈ J.

ln(x + x 2 + 1)
b) Să se calculeze: ∫ x2 +1
dx , x ∈ R.

Etapa locală, NeamŃ 2010, prof. Radu Mititelu


33. Fie f: R → R o funcŃie care admite primitive pe R şi F o primitivă a sa pe R, pentru care:
sin 2 x
F( x ) f ( x ) ≤ , (∀) x ∈ R. Să se demonstreze că nu există lim F( x ) .
1 + sin 2 x x→∞
Etapa locală, Dolj 2010
34. Fie f: R → R o funcŃie primitivabilă şi F: R → R o primitivă a sa. DemonstraŃi că, dacă f(x) ≤
|x|
≤ , (∀) x ∈ R, atunci există un unic punct x0 ∈ R, astfel încât F(x0) = x0.
1+ | x |
Etapa local, Arad 2010
35. Fie f: R → R o funcŃie monotonă cu proprietatea că f admite primitive pe R.
2

a) Să se arate că f4 admite primitive pe R.


b) Să se dea un exemplu de funcŃie f cu proprietatea de mai sus, pentru care f3 nu admite primitive
pe R.
(prin fk înŃelegem funcŃia produs f⋅ f ⋅ ...
⋅ f , k ∈ N, k ≥ 2).
k ori
Etapa locală, Dolj 2010
36. a) Fie f: [a, b] → R strict crescătoare, care admite primitive şi fie F o primitivă a sa. ArătaŃi că
F( x1 ) − F( x 2 )
(∀) c ∈ (a, b), (∃) x1, x2 ∈ (a, b) astfel încât = f ( c) .
x1 − x 2

∫x (1 − x 3 ) 2010 dx , x ∈ R.
2
b) CalculaŃi
Etapa locală, Sălaj 2010
37. Să se determine grupurile de forma (∫ )
f ( x )dx ,  , unde f: R → R este o funcŃie care admite primi-
tive, iar „” este compunerea funcŃiilor.
Etapa locală, Bucureşti 2010, prof. Marcel łena

113
38. Fie (G, ⋅) un subgrup al grupului multiplicativ (R*, ⋅) şi F mulŃimea funcŃiilor f: R → R, deri-
vabile şi surjective, care satisfac condiŃia (∀) x ∈ R, f'(x) ∈ G. Să se demonstreze că mulŃimea F
în raport cu operaŃia de compunere a funcŃiilor este grup.
Etapa locală, GalaŃi 2010, prof. Marian Baroni

II. Integrale
3x 2 + 2 x , x ≤ 0
39. Se consideră funcŃia f: R → R, f ( x ) =  . DemonstraŃi că f este integrabilă pe orice
arctgx, x > 0
1
interval compact şi calculaŃi ∫ −1
f (x)dx .
Etapa locală, Iaşi 2010
a2 ln x
40. Fie a > 1. Să se calculeze ∫ 1
x (x + a)
dx .

Etapa locală, Olt 2010, prof. Dumitru Vişan


1 π
41. a) Să se arate că arctgx + arctg = , pentru orice x > 0.
x 2
a arctgx
b) Fie a > 1. Să se calculeze ∫ 1 dx .
a x
Etapa locală, Teleorman 2010
π
cos nx − cos (n − 1)x
2 2
42. Să se calculeze ∫ π
3
2
sin x
dx .

Etapa locală, Arad 2010


1
43. Să se calculeze ∫ −
2010
1
2010
x 2010 arccos 2010dx .

Etapa locală, Cluj 2010, prof. Ilie Diaconu


44. Să se calculeze următoarele integrale:
1 3x
a) ∫1 32x − 3x +1 + 3 dx ;
1 x
b) ∫ x 1− x dx .
0 3 +3 −3
Etapa locală, Maramureş 2010, prof. Gh. Szollosy, Gazeta Matematică 7-8-9/2009
π
45. Să se calculeze: I = ∫ 4 x 1 
+ ln(sin x ) dx .
πe  2
6  sin x 
Etapa locală, Bucureşti 2010, prof. Cristian Alexandrescu

114
46. Să se calculeze:
π 2 +1 ln x
a) ∫
0
arcsin(sin x )dx ; b) ∫ 2 −1 1 + x2
dx .
Etapa locală, Covasna 2010
5 2x − 5 4
47. Să se calculeze I = ∫ dx .
3 4 2 x − 5 + 11 − 2 x ⋅ e16 − 4 x
4

Etapa locală, Vâlcea 2010, prof. Cătălin Bîrzescu


2π x ⋅ sin x
2
48. CalculaŃi integrala I = ∫ dx .
0 1 + cos 2 x
Etapa locală, Suceava 2010, prof. Gheorghe Marchitan
1
49. Fie f: [0, 2π] → R, f ( x ) = .
3 + cos x
a) Să se determine primitivele funcŃiei f pe [0, π).
b) Să se determine primitivele funcŃiei f pe [0, 2π].

c) CalculaŃi ∫ f (x)dx .
0
Etapa locală, Alba 2010
arctgx 1
50. Se dă f: R → R, f (x) = ∫ π ln(1 + tg t)dt . Să se afle f'(x) şi ∫ f (x)dx .
2
0
4
Etapa locală, Olt 2010
51. Se consideră funcŃia g: R → R definită prin g(x) = x + sin x, (∀) x ∈ R, iar funcŃia f: R → R
verifică relaŃia f(x) + sin(f(x)) = x3, (∀) x ∈ R.

a) Să se calculeze xg' ( x )dx , x ∈ R.
b) Să se demonstreze că funcŃia g este inversabilă.
c) Să se demonstreze că funcŃia f admite primitive.
3
π
d) Să se calculeze ∫ 0
x 2 f ( x )dx .
Etapa locală, Maramureş 2010, prof. N. Muşuroia
52. Fie funcŃia f: [0, 1] → [1, 3], f(x) = x + x + 1. Se admite că funcŃia f are inversa g.
4 2

1 t (2t 2 + 1)
a) Să se calculeze ∫ 0 f (t )
dt .
1 3
b) Să se arate că ∫ f (x )dx + ∫ g( x)dx = 3 .
0 1
1 α
c) Să se demonstreze că, dacă α ∈ [1, 3], atunci are loc inegalitatea ∫0 ∫
f ( x )dx + g ( x )dx ≥ α .
1
Etapa locală, Botoşani 2010
53. Să se arate că dacă f: [–2, 2] → R este o funcŃie continuă, atunci există c ∈ (–2, 2) astfel încât
c  4 c

−2
f ( x )dx =  3 −  ⋅ f (c) .
c 4
Etapa locală, Cluj 2010, prof. Gheorghe LobonŃ

115
b
54. Se consideră funcŃia derivabilă f: [a, b] → R, astfel încât ∫ f (x )dx = (b − a)f (a ) . Să se arate că
a
există c ∈ (a, b) astfel încât f(c) – f(a) = f'(c).
Etapa locală, Vâlcea 2010, Gazeta Matematică
55. Fie mulŃimea F = {f: [0, 1] → [0, 1] | f continuă, f(0) = 0, f(1) = 1} şi aplicaŃia T: F → (0, 1),
1
T(f) = ∫ f (x)dx .
0
a) ArătaŃi că T este surjectivă. b) ArătaŃi că T nu este injectivă.
Etapa locală, Suceava 2010, Gazeta Matematică 12/2009
n x{x}
56. Fie şirul (In)n≥1, unde I n = ∫ dx . CalculaŃi:
1 [x]
In
a) I5; b) lim .
n →∞ n

 2n [x] 
57. Să se calculeze L = lim  ∫ dx − ln(an )  , unde n ∈ N*, a ∈ R*+, iar [x] reprezintă
n →∞
 0 [ x ] + 5[ x ] + 6
2

partea întreagă a lui x.
Etapa locală, Bucureşti 2010, Gazeta Matematică
 n [x] 
58. Să se calculeze L = lim  ∫
n → ∞ 0 [ x ] + 3[ x ] + 2

2
dx − ln(n + 1)  , unde n ∈ N*, iar [x] reprezintă partea

întreagă a numărului real x.
Etapa locală, Braşov 2010, Gazeta Matematică 12/2009
1 x
59. Fie f: [0, 1] → R o funcŃie continuă şi şirul (an)n∈N definit prin a n = f  dx , n ∈ N.
0 n ∫
a) Să se arate că lim a n = f (0) .
n →∞

b) Dacă f este derivabilă în 0, să se calculeze lim n (a n − f (0)) .


n →∞

Etapa locală, Hunedoara 2010


1
60. Fie şirurile de numere reale (an)n≥1 şi (bn)n≥1 definite astfel: a n = ∫ x n ⋅ ln( x + 1)dx , bn = a1 – a2 +
0
+ a3 – a4 + ... + (–1)n–1 ⋅ an, (∀) n ∈ N*.
Să se calculeze lim b n .
n →∞

Etapa locală, GalaŃi 2010, prof. Vasile Popa


2a + x a a 2a + x
61. Se consideră şirurile: I n = sin 2 nx ⋅ ln
−a ∫
2a − x
dx şi K n = sin 2 nx ⋅ ln
0 2a − x ∫
dx , unde a > 0,
fixat, n ∈ N*.
a) Să se arate că lim I n = 0 . b) Să se calculeze lim K n .
n →∞ n →∞
Etapa locală, Prahova 2010, prof. Octavian Purcaru, Ploieşti

116
x
62. a) StudiaŃi variaŃia funcŃiei f: [2, 3] → R, f ( x ) = .
ln x
3 x
b) AflaŃi partea întreagă a numărului real I =
2 ln x ∫
dx .
Etapa locală, Covasna 2010
π
sin 2009 x − cos 2 x 1
sin x
63. Să se arate că: ∫0
2
sin x + cos x + 1
2009 2009
dx < lim
n →∞ ∫ n
1
n +1 x
2
dx .

Etapa locală, Bihor 2010


1 ln(1 + 2x ) π 3 3
64. ArătaŃi că: ∫
0 1 + 3x 2
dx ≤
3 3
⋅ ln
e
.

Etapa locală, Braşov 2010, prof.dr. Ioana Maşca


x+2
65. Fie funcŃia f: [1, 2] → R, integrabilă, definită prin: f ( x ) = , unde r =
( 4 x + 2 x + 2) r + 2 r 2 x
2

(∫ ) (∫ ) < 16 .
2 2 2 2
= x 2 + x + 1 . Să se arate că: f (x) sin xdx + f (x) cos xdx
1 1

Etapa locală, Giurgiu 2010, prof. Stelian Piscan


1
1 ∫ 0 log2 f ( x )dx
66. Fie f: R → R + o funcŃie continuă pe R. Să se arate că: ∫ f (x)dx ≥ 2
*
0
.
Etapa locală, Gorj 2010

117
Concursul de Matematică „LaurenŃiu Panaitopol”, Bucureşti, 7 noiembrie 2009

CLASELE A IX-
IX-A – A X-
X-A

1. a) ArătaŃi că, dacă 0 < x < y < 90, atunci sin x° < sin y°.
b) Fie ABC şi A'B'C' două triunghiuri ascuŃitunghice astfel încât AB ≥ A'B', AC ≥ A'C', BC ≥
≥ B'C' şi aria(ABC) ≤ aria(A'B'C'). ArătaŃi că ∆ABC ≡ ∆A'B'C'.
L. Panaitopol
2. ArătaŃi că descompunerea în factori primi a sumei a două numere prime consecutive, diferite de 2,
conŃine cel puŃin trei factori (nu neapărat distincŃi; de exemplu, 7 + 11 = 18 = 2 ⋅ 3 ⋅ 3).
r−x
3. Fie b > a > 0 numere reale fixate. Pentru r ∈ [a, b] definim M (r ) = max .
x∈[ a , b ] x

DeterminaŃi r (funcŃie de a, b) pentru care M(r) este minim.


4. La „Balul Bobocilor” participă 100 de băieŃi şi 100 de fete. Cineva constată că băieŃii pot invita
fetele la dans astfel încât, în fiecare dintre cele 100 de perechi formate, să fie respectată regula:
(R) diferenŃa dintre vârstele dansatorilor este de cel mult o săptămână.
ArătaŃi că, dacă ordonăm crescător după vârstă atât băieŃii cât şi fetele, apoi primul băiat invită
prima fată, al doilea băiat invită a doua fată şi aşa mai departe, atunci perechile formate respectă
regula (R).

CLASELE A XI-
XI-A – A XII-
XII-A

1. Fie (an)n∈N un şir de numere naturale cu a0 > 0 şi cu proprietatea: „pentru orice n ∈ N, an+1 este un
divizor prim al lui an + 1”. ArătaŃi că, începând de la un anumit rang, şirul este periodic.
L. Panaitopol
1 − cos x
2. DeterminaŃi toate numerele reale m pentru care ecuaŃia arctg − mx = 2 nu are soluŃii.
1 + cos x
3. Fie m ∈ N* şi z1, z2, z3 numere complexe astfel încât z1p + z p2 + z 3p = 0 , pentru orice p ∈ {m, m + 1,
m + 2}. ArătaŃi că z1 = z2 = z3 = 0.
4. În plan considerăm un sistem de coordonate şi mulŃimea C a cercurilor având raza 0,01 şi centre
în punctele cu ambele coordonate întregi. ArătaŃi că orice dreaptă care trece prin originea axelor
de coordonate taie o infinitate de cercuri din C.

118
Concursul interjudeŃean de Matematică „Cristian S. Calude”, EdiŃia a X-a,
GalaŃi, 31 octombrie 2009

CLASA A IX-
IX-A

n
1. a) ArătaŃi că ∑ k(3k + 1) = n (n + 1)
k =1
2
, (∀) n ∈ N*.

1⋅ 4 2⋅7 3 ⋅10 25 ⋅ 76
b) ArătaŃi că + + + ... + > 16,8 .
1996 1986 1970 100
Elev Paul Cosma, GalaŃi
x
2. a) Să se rezolve în mulŃimea R – Z ecuaŃia − k 2 ⋅ [ x ] ⋅ {x} = 1 , unde k ∈ N, iar prin [x] şi
{x + k}
{x} s-au notat partea întreagă, respectiv partea fracŃionară a lui x.
1
b) Să se arate că dacă a, b, c ∈ (0, ∞) şi a + b + c = , avem inegalitatea:
2
1 1 1  1 1 1 
+ 2 + 2 ≥ 3⋅ + + .
a + ab + bc + ca b + ab + bc + ca c + ab + bc + ca a+b b+c c+a
2

Prof. Rodica şi Dumitru Bălan, GalaŃi


a+b b+c c+a 2( a + b + c )
3. a) ArătaŃi că în orice triunghi avem: + + ≥ , unde cu la şi ha
la + h b l b + h c lc + h a a 2 + b2 + c2
s-au notat lungimea bisectoarei şi respectiv a înălŃimii din vârful A.
b) Pe laturile patrulaterului convex ABCD se construiesc în exteriorul acestuia AE ⊥ AB, [AE] ≡
≡ [AB], CF ⊥ CB, [CF] ≡ [CB], CG ⊥ CD, [CG] ≡ [CD] şi AH ⊥ AD, [AH] ≡ [AD]. Dacă M, N,
P, Q sunt mijloacele segmentelor [EB], [HD], [DG] şi respectiv [BF], arătaŃi că MN2 + PQ2 =
= MQ2 + NP2.
Prof. Petre BătrâneŃu, GalaŃi

CLASA A X-
X-A

1. Fie triunghiul ABC ascuŃitunghic şi O centrul cercului circumscris triunghiului.


a) Să se demonstreze că: OA ⋅ sin 2A + OB ⋅ sin 2B + OC ⋅ sin 2C = 0 .
b) Să se determine mulŃimea punctelor M din plan care verifică egalitatea:
MA2 ⋅ sin 2A + MB2 ⋅ sin 2B + MC2 ⋅ sin 2C = k, unde k este o constantă pozitivă. DiscuŃie.
Prof. Iuliana Duma, GalaŃi
n
k +1
2. a) Să se determine partea întreagă a numărului: A = ∑ 2k + 2 , unde n ∈ N*.
k =1 k

119
 n
1 
b) Să se arate că mulŃimea X = n ∈ N *

∏ log
k =1
2 k +1 2 k <  este infinită.
2010 
Prof. Iuliana Duma, GalaŃi
3. Fie funcŃia f: A → A, unde A este o mulŃime finită de numere reale şi pentru orice x, y ∈ A,
x ≠ y, |f(x) – f(y)| < |x – y|. Să se arate că f nu este surjectivă şi are un punct fix (f are un punct fix
dacă există c ∈ A astfel încât f(c) = c).
***

CLASA A XI-
XI-A

1. a) Să se rezolve ecuaŃia: 2log3(tg x) = log2(sin x).


Oleh Faynshteyn, Leipzig, Germania
4 + 4 − 4 = 256
y z x t

b) Fie x, y, z, t ∈ R astfel încât:  . DemonstraŃi că x + 2y + 2z + 4t ≤ 0.


16 + 16 − 16 = 2
y z t x

Prof. Cristinel Mortici


x
2. a) Fie şirul (xn)n≥0 definit astfel: x0 = 4, xn+1 = 2 x 2n − 12x n + 21 . Să se calculeze L = lim 2nn .
n →∞ 2

Prof. Alfred Eckstein, Viorel Tudoran, Arad


b) ConstruiŃi m submulŃimi A1, A2, ..., Am (m oarecare) disjuncte şi nevide de numere reale, astfel
încât: A1 ∪ A2 ∪ ... ∪ Am = R şi (∀) α ∈ R, (∀) i ∈ {1, 2, ..., m} să existe un şir neconstant
(xn)n≥1 ⊂ Ai cu proprietatea că lim x n = α .
n →∞
Prof. Constantin Ursu, GalaŃi
3. Fie funcŃia strict crescătoare dată f: R → R şi definim g(x, y) prin egalitatea:
f ( x + y) − f ( x )
g ( x , y) = , x ∈ R, y ∈ R, y > 0.
f ( x ) − f ( x − y)
Pentru toŃi y ∈ R, y > 0 şi pentru x = 0 şi pentru toŃi y ∈ (0, |x|] cu x ≠ 0 avem g(x, y) < 2.
JustificaŃi afirmaŃiile:
a) Dacă h(x, y) = f(y) – f(x), atunci h(x + y, x + 3y) < 6h(x, x + y), pentru orice x ≥ 0 şi y > 0.
b) Dacă x ∈ R, y > 0, atunci g(x, y) < 14.
Prelucrare, Constantin Ursu, GalaŃi

CLASA A XII-
XII-A

1. a) Fie f: R → R o funcŃie ce admite primitive şi îndeplineşte condiŃia: f(x + 3) + f(x – 1) = f(x +


+ 1) + x, (∀) x ∈ R. Să se arate că ecuaŃia f(x) = 5 are cel puŃin o soluŃie în intervalul (0, 12).
b) Fie (G, ⋅) un grup şi a, b ∈ G astfel încât a3 ⋅ b = b ⋅ a2 şi a12 = e, unde e este elementul neutru al
grupului (G, ⋅). Să se arate că a = e.
Prof. Rodica şi Dumitru Bălan, GalaŃi

120
c) Fie M o mulŃime nevidă pe care se defineşte operaŃia „&”, care satisface proprietăŃile:
i) a & (b & c) = b & (c & a), (∀) a, b, c ∈ M;
ii) are loc implicaŃia a & b = a & c ⇒ b = c.
Să se arate că:
1) „&” este comutativă;
2) „&” este asociativă.
Prof. Vasile Dumbravă, GalaŃi
2. a) Fie (an)n≥1 un şir strict crescător de numere naturale nenule. Să se arate că:
1 1 1
1) şirul (xn)n≥1, x n = + + ... + , (∀) n ∈ N*, este convergent, iar limita sa
a1 a 1 ⋅ a 2 a1 ⋅ a 2 ⋅ ... ⋅ a n
este un număr iraŃional;
P(n )
2) nu există două funcŃii polinomiale P, Q, astfel încât x n = , (∀) n ∈ N*.
Q( n )
Prof. Vasile Popa, GalaŃi
b) Fie şirul (an)n≥1, an = 2 + 3 + 6 – 1, n ≥ 1. Să se determine elementele mulŃimii M = {m ∈ N* |
n n n

| (m, an) = 1, (∀) n ∈ N*}.


***
3. Fie funcŃiile f, g: R → R, g este surjectivă şi de două ori derivabilă, g'' continuă, iar A = {x ∈ R |
| g'(x) = 0}, cardA = 2 şi g''(x) ≠ 0, (∀) x ∈ A. Ştiind că f  g admite primitive, să se arate că
funcŃia f admite primitive.
Prof. Vasile Popa, GalaŃi

Concursul de matematică „UNIREA” EdiŃia a X-a, Focşani, 29 ianuarie 2010

CLASA A IX-
IX-A

 n2 
1. Fie a şi n numere naturale nenule, astfel încât a < 2n . Să se arate că numărul  2
este pătrat
a 
perfect dacă şi numai dacă n se divide cu a.
2. FuncŃia f: R → R are proprietatea f(x) + f(x + y) ∈ Q, (*), pentru orice x ∈ R şi orice y > 0. Să se
arate că f(x) ∈ Q, pentru orice x ∈ R.
3. Fie ABC un triunghi. Pe laturile BC, CA şi AB considerăm punctele A', B' şi C' astfel încât
segmentele AA', BB' şi CC' sunt concurente în punctul M.
AM AB' AC'
a) Să se demonstreze relaŃia lui Van Aubel: = + .
MA' B' C C' B
AM BM CM AM BM CM
b) Ştiind că ⋅ ⋅ = 2010 , să se calculeze + + .
MA' MB' MC' MA' MB' MC'

121
4. Fie triunghiul ascuŃitunghic ABC cu centrul de greutate G. Considerăm o dreaptă d care trece prin
G şi intersectează segmentele (AB) şi (AC) în punctele M şi respectiv N. Fie O intersecŃia
segmentelor [BN] şi [CM]. ArătaŃi că expresia 2[AMN] + [BOC] – [MON] este constantă, adică
nu depinde de alegerea dreptei d. (Prin [XYZ] am notat aria triunghiului XYZ.)

CLASA A X-
X-A

1. Fie n un număr natural nenul. Se notează cu an numărul submulŃimilor nevide ale mulŃimii {1, 2,
..., n} care nu conŃin numere consecutive. Să se arate că an+2 = an+1 + an + 1, pentru orice n ∈ N*.
2. Fie a, b, c, d ∈ C* afixele vârfurilor patrulaterului convex ABCD. Ştiind că:
a) ac = ac, bd = bd ; b) a + b + c + d = 0,
arătaŃi că ABCD este paralelogram.
3. FuncŃia f: Z → Z verifică relaŃia: f(x + y) + f(xy) = f(x) + f(y) + f(x)f(y), (*), pentru orice x, y ∈ Z.
a) ArătaŃi că f(1) ∈ {–1, 0, 1}.
b) DeterminaŃi toate funcŃiile care verifică relaŃia (*).
1
4. Fie ABC un triunghi ascuŃitunghic şi k ∈ R, 0 < k < . Pe laturile BC, CA, AB se consideră
2
punctele D, E, F, astfel încât BD = k BC, CE = k CA, AF = k AB . Să se stabilească dacă următoa-
rele două condiŃii sunt echivalente:
(1) Triunghiul ABC este asemenea cu triunghiul DEF; (2) Triunghiul ABC este echilateral.

CLASA A XI
XI-A

1. a) ArătaŃi că dacă a, b, c, d ∈ R, atunci:


1 1 1 1 1 1 1 1
a b c d a b c d
= .
a (a − 1) b(b − 1) c(c − 1) d (d − 1) a2 b2 c2 d2
a (a − 1)(a − 2) b(b − 1)(b − 2) c(c − 1)(c − 2) d (d − 1)(d − 2) a3 b3 c3 d3
b) Fie n ≥ 2 un număr natural şi a1, a2, ..., an numere întregi. ArătaŃi că numărul ∏ (a
1≤ i < j ≤ n
j − a i ) este

divizibil cu ∏ ( j − i) .
1≤ i < j≤ n

2. a) ArătaŃi că, dacă X ∈ M2(R), atunci 2det(X) = (tr(X))2 – tr(X2), unde tr(M) reprezintă urma
matricei pătrate M (suma elementelor de pe diagonala principală).
 a b 0
 
b) Se ştie că matricele A ∈ M32(R), B ∈ M23(R) au proprietatea AB =  c d 0  , unde a, b, c,
 0 0 0
 
d ∈ R. DemonstraŃi că det(BA) = ad – bc.

122
n
1
1
1 + + ... +
2
1

n = 1. b) CalculaŃi lim k =1 k k .
n
3. a) DemonstraŃi că: lim
n →∞ ln n n →∞ ln n
4. Fie două funcŃii f, g: R → R cu proprietatea |f(x) – f(y)| ≤ |g(x) – g(y)|, pentru orice x, y ∈ R.
a) DemonstraŃi că, dacă lim g ( x ) există şi este finită, atunci lim f ( x ) există.
x →∞ x →∞

b) Rămâne concluzia precedentă valabilă în cazul în care lim g ( x ) = ∞?


x →∞

CLASA A XII-
XII-A

1. Fie n ≥ 1 un număr natural şi (G, ⋅) un grup astfel încât funcŃia f: G → G, f(x) = xn să fie morfism
de grupuri.
a) ArătaŃi că, dacă n = 3 şi f este injectivă, atunci G este comutativ.
b) ArătaŃi că, dacă n = 3 şi f este surjectivă, atunci G este comutativ.
c) DaŃi exemplu de grup necomutativ pentru care funcŃia g: G → G, g(x) = x4 este automorfism de
grupuri.
2. Fie k ≥ 1 un număr natural şi G o submulŃime cu k elemente a lui Mn(R), n ≥ 2, care este parte
stabilă în raport cu înmulŃirea matricelor şi este grup în raport cu operaŃia indusă. Fie S = ∑ A şi
A∈G
t urma matricei S (suma elementelor de pe diagonala principală). Ştiind că t = nk, arătaŃi că S = kIn.
3. Se consideră funcŃia f: [0, 1] → R, derivabilă pe [0, 1] şi cu proprietatea f(0) = 0.
1 x

n →∞ 0 ∫
CalculaŃi lim n f  dx .
n
1
4. Fie o funcŃie continuă f: [0, ∞) → R, cu proprietatea |f(x)| ≤ , (∀) x ≥ 0 şi F o primitivă a
x +1
2

sa. ArătaŃi că F este mărginită şi lim F( x ) există.


x →∞

Concursul interjudeŃean de matematică „Grigore Moisil”, Urziceni,


EdiŃia a VI-a, 29-31 ianuarie 2010

CLASA A IX-
IX-A

1. DemonstraŃi că pentru orice număr natural n ≥ 2, are loc inegalitatea:


1  1 1 1  1 1 1 1 1 
⋅ 1 + + + ... +  > ⋅  + + + ... + .
n +1  3 5 2n − 1  n  2 4 6 2n 
2. AflaŃi al optulea termen al şirului 1440, 1716, 1848, ..., ai cărui termeni sunt produsul termenilor
corespunzători (primul cu primul, al doilea cu al doilea etc.) ai anumitor două progresiii arit-
metice.

123
3. Fie n ∈ N*. DemonstraŃi că mulŃimea {n, n + 1, n + 2, ..., n3 + n2 + n + 1, n3 + n2 + n + 2},
a) conŃine cel puŃin un pătrat perfect;
b) conŃine puterea a patra a unui număr natural.
4. Fie ABCDE un pentagon şi M, N, P, Q punctele de intersecŃie ale segmentelor ce unesc mijloa-
cele laturilor opuse în patrulaterele BCDE, CDEA, EABD, ABCE, respectiv. DemonstraŃi că
MNPQ este paralelogram dacă şi numai dacă ABCD este paralelogram.

CLASA A X-
X-A

1. Notăm cu X mulŃimea numerelor naturale mai mari sau egale cu 8 şi fie f: X → X o funcŃie cu
proprietatea că f(x + y) = f(xy), oricare ar fi numerele naturale x ≥ 4 şi y ≥ 4. Dacă f(8) = 9,
calculaŃi f(9).
z z +z z +z z
2. Fie z1, z2, z3 ∈ C, astfel încât: | z1 |=| z 2 |= 1 2 2 3 3 1 = 1 . DaŃi un exemplu de astfel de
z1 + z 2 + z 3
triplete cu z3 = 0. DemonstraŃi că dacă z3 ≠ 0, atunci |z3| = 1.
3. Fie f, g, h: Z → Z funcŃii periodice de perioade 7, 19, respectiv 31. DemonstraŃi că dacă funcŃia
f + g + h este constantă, atunci funcŃiile f, g şi h sunt constante.
(Fie T ≥ 1 un număr natural. Spunem că o funcŃie y: Z → Z este periodică de perioadă T dacă
y(x + T) = y(x), oricare ar fi x ∈ Z.)
4. RezolvaŃi ecuaŃiile:
a) log 0,5 (1 + tg 2 x ) ⋅ log 0,5 (1 + ctg 2 x ) = 1 ; b) log1,5 (1 + sin 2 x ) ⋅ log1,5 (1 + cos 2 x ) = 1 .

CLASA A XI-
XI-A

1. DemonstraŃi că şirul x n +1 = x n − x 2n + x 3n − x 4n , cu x0 ∈ (0, 1) este convergent şi apoi calculaŃi


limita şirului yn = nxn.
1 3 1
2. Fie A ∈ M2(Q) o matrice cu proprietatea că A 3 + A = I 2 + A 2 + A 4 . DemonstraŃi că există
4 2 4
o matrice B ∈ M2(Q) astfel încât B2 = A.
3. a) Fie (an)n≥1, (bn)n≥1, (cn)n≥1 şiruri de numere reale şi funcŃiile fn: (0, 1) → R definite pentru orice
x ∈ (0, 1) şi n ∈ N* prin fn(x) = anx2 + bnx + cn. ArătaŃi că dacă pentru orice x ∈ (0, 1) avem
lim f n ( x ) = 0 , atunci există un şir de numere reale (wn)n≥1 convergent la zero, pentru care |fn(x)| ≤ wn,
n →∞
oricare ar fi x ∈ (0, 1) şi n ∈ N*.
b) DaŃi un exemplu de funcŃii gn: (0, 1) → R, n ∈ N* astfel încât lim g n ( x ) = 0 , oricare ar fi x ∈
n →∞
∈ (0, 1), pentru care nu există niciun şir de numere reale (vn)n≥1 convergent la zero, care să
satisfacă relaŃia |gn(x)| ≤ vn, oricare ar fi x ∈ (0, 1) şi n ∈ N*.
4. Fie A ∈ Mn(C) cu detA = d ≠ 0. DemonstraŃi că pentru orice k ∈ N*, avem:
k
a) det((...(A*)*...)*)* = d ( n −1) (k semne „*”);

124
( n −1) k + ( −1) k −1
k
b) ((...(A*)...)*)* = d n ⋅ A ( −1) (k semne „*”). (X* este adjuncta matricei X.)

CLASA A XII-
XII-A

a dx
1. CalculaŃi: lim ∫ .
a →∞ 0 (1 + x )(1 + x 2010 )
2

2. Fie mulŃimea F = {f: [0, 1] → [0, 1] | f continuă, f(0) = 0, f(1) = 1} şi aplicaŃia T: F → (0, 1)
1
definită prin T(f) = ∫ f (x)dx , oricare ar fi f ∈ F. DemonstraŃi că:
0
a) T este surjectivă; b) T nu este injectiv.
3. DeterminaŃi toate funcŃiile f: [0, ∞) → [0, ∞) derivabile, cu derivata continuă, astfel încât oricare
x
ar fi x ∈ [0, ∞), f ( x ) = 2010 + ∫
0
(f 2 ( t ) + f '2 ( t ))dt .
4. Pe o mulŃime nevidă S se defineşte o lege de compoziŃie „&” astfel încât x & x = x şi (x & y) & z =
= (y & z) & x, oricare ar fi x, y, z ∈ S. ArătaŃi că legea „&” este comutativă.

Concursul interjudeŃean de matematică „Nicolae Coculescu”, Slatina,


EdiŃia a VI-a, 27 noiembrie 2009

CLASA A IX-
IX-A

1. Fie a, b, c ∈ (0, ∞). Să se demonstreze inegalitatea:


a + b b + c c + a 1  b   c   a  
2 2 2
+ + ≤ ⋅ 1 +  + 1 +  + 1 +   .
a + c b + a c + b 4  a   b   c  
Prof. Costel Anghel
2. Notăm cu pn al n-lea număr prim, (p1 = 2, p2 = 3, p3 = 5, ...). Să se determine partea întreagă a
1 1 1
numărului 12Sn, unde Sn = + + ... + , n ∈ N*.
p1p 2 p 2 p3 p n p n +1
Prof. Florian Dumitrel
3. Fie ABC un triunghi neisoscel, M un punct în interiorul său astfel încât 'ABM ≡ 'ACM, E
proiecŃia lui M pe bisectoarea interioară a unghiului 'A, F simetricul lui M faŃă de E şi Q mijlocul
lui [BC].
a) Să se arate că EQ || AF.
b) DemonstraŃi că punctele Q, E şi proiecŃia lui M pe bisectoarea exterioară a unghiului 'A sunt
coliniare.
Prof. Dan Marinescu şi Ciprian Demeter, Hunedoara

125
4. Într-un pătrat de latură 1 sunt trasate un număr finit de segmente cu suma lungimilor egală cu 100.
a) Să se arate că există o dreaptă care intersectează cel puŃin 50 de segmente.
b) Să se arate că există o dreaptă care intersectează cel mult 70 de segmente.
Prof. Vasile Pop

CLASA A X-
X-A

1. Să se rezolve ecuaŃia log 7 (10 x − 3) = lg((7 x + 3) lg 7 + 3) .


Prof. Cezar Lupu
2. Fie a, b, c ∈ R şi funcŃia f: R → R, f(x) = ax + bcos x + csin x.
a) Să se arate că dacă a2 ≥ b2 + c2, atunci funcŃia f este injectivă.
b) Să se determine a, b, c ∈ R astfel încât funcŃia g: R → [0, 1), g(x) = {f(x)} să fie periodică.
Prof. Vasile Pop
3. Fie f: R → R o funcŃie convexă şi a, b ∈ R, a < b.
a) Să se arate că dacă există t0 ∈ (0, 1) astfel încât f((1 – t0)a + t0b) = (1 – t0)f(a) + t0f(b), atunci:
f((1 – t)a + tb) = (1 – t)f(a) + tf(b), pentru orice t ∈ [0, 1].
 2ab 
b) Să se arate că dacă 0 < a < b şi (a + b)f   = bf (a ) + af (b) , atunci graficul lui f conŃine un
a+b
segment de lungime mai mare sau egală cu b – a.
prof. Marin Toloşi
4. Fie funcŃia f: R → R cu proprietatea că f(x3 + y3) + f(x3 – y3) = 2x(f2(x) + f2(y)) – 2yf(x)f(y),
(∀) x, y ∈ R. Să se demonstreze că dacă mulŃimea A = {x ∈ R | f(x) = 0} este finită, atunci
1
pentru orice x ∈ R există a, b ∈ Q astfel încât | a − b | ≤ şi f(a) < x < f(b).
2
Prof. Marius Perianu

CLASA A XI-
XI-A

1. Fie matricele inversabile A, B, C ∈ Mn(C) astfel încât 2A ⋅ (B–1 + C) = B ⋅ (A–1 + C). Să se arate
că 2(B–1 + C) ⋅ A = (A–1 + C) ⋅ B.
Prof. Vasile Pop
2. Fie A ∈ Mn(R) astfel încât A = 3A – 2In.
3

a) Să se calculeze det(A2 + A + In).


b) Să se găsească o matrice cu proprietatea din enunŃ, diferită de aIn, a ∈ R.
Prof. Florian Dumitrel
1 1
3. Să se studieze existenŃa limitei şirului (an)n≥1 definit prin a1 = 1 şi a n +1 = + , (∀) n ≥ 1.
n a 2n
Prof. Florian Dumitrel

126
1 1 1
4. Fie şirul a n = 1 + + + ... + , n ≥ 1.
3 5 2n − 1
ea n
a) Să se calculeze lim .
n →∞ n

 1 1 1 
b) Să se arate că (an)n≥1 conŃine un subşir (sn)n≥1 astfel încât lim  + + ... +  = ln 2 .
n ←∞ s
 n +1 s n + 2 s 2 n 
Prof. Florian Dumitrel

CLASA A XII-
XII-A

1. Fie A ∈ M2(C), A ≠ O2, A ≠ I2, n ∈ N* şi Sn = {X ∈ M2(C) | Xn = A}. Să se arate că următoarele


afirmaŃii sunt echivalente:
a) (Sn, ⋅) este grup izomorf cu Un. b) A2 = A.
Prof. Marian Andronache
x
2. Să se calculeze I = ∫ dx , x ∈ (0, 1).
x + 1− x
Prof. Florian Dumitrel
3. Fie p un număr prim şi (G. ⋅) un grup cu 2p elemente, al cărui element neutru este e. Să se
2

demonstreze că dacă H1 şi H2 sunt subgrupuri ale lui G care verifică simultan condiŃiile:
a) H1 ∩ H2 = {e};
b) oricare ar fi submulŃimea S ⊂ G, cu card S = p, rezultă card (S ∩ (H1 ∪ H2)) ≥ 2.
ArătaŃi că H1 ≠ G sau H2 ≠ G.
Prof. Dana Heuberger
4. Fie f: R → R o funcŃie continuă şi periodică de perioadă iraŃională, iar F o primitivă a sa. Să se
demonstreze că şirul (F(n) – n)n≥1 este convergent dacă şi numai dacă f(x) = 1 pentru orice x ∈ R.
Prof. Florian Dumitrel
Concursul interjudeŃean de matematică „Ion Ciolac”, 8 mai 2010
CLASA A IX-
IX-A

1. DeterminaŃi numerele reale x1, x2, ..., x2010 ∈ (1, ∞) cu proprietăŃile: x1 + x2 + ... + x2010 = 4020;
∑ ( x i − 1) ( x j − 1) = 2009 ⋅1005.
1≤ i ≺ j≤ 2010

Prof. Carmen Georgescu, Craiova


2. ArătaŃi că pentru orice numere reale a, b, c > 0 are loc inegalitatea:
3
a+b+c+ ≥ 43 abc .
1 1 1
+ +
a b c
Gazeta Matematică

127
3. În triunghiul ABC avem m('B) – m('C) = 60°. Notăm cu M, N, P mijloacele laturilor [BC],
[CA] şi [AB] şi cu D proiecŃia lui A pe BC. Bisectoarea unghiului 'A intersectează dreapta MP în
E şi dreapta MN în F. DemonstraŃi că ∆DEF ≡ ∆MNP.
Prof. Liliana Niculescu

CLASA A X-
X-A

1. Fie z1, z2, z3, z4 ∈ C diferite astfel încât |z1| = |z2| = |z3| = |z4| = r, r > 0 şi z1 + z2 + z3 + z4 = 0.
1 1 1 1
CalculaŃi suma n
+ n + n + n , pentru n ∈ N, n impar.
z1 z 2 z 3 z 4
Prof. Daniel Jinga, Gazeta Matematică
16n
2. DemonstraŃi inegalitatea: (C02 n +1 ) 2 + (C12 n +1 ) 2 + (C32 n +1 ) 2 + ... + (C2n n +1 ) 2 ≥ , pentru orice n ∈ N.
n +1
( x + y)3x − 2 y = 81
3. RezolvaŃi în R × R sistemul:  2 .
log 3 ( x + y) = x − y
Prof. Cristian Schneider, Craiova

CLASA A XI-
XI-A

1. DemonstraŃi că pentru orice matrice A, B ∈ M2(C) are loc egalitatea:


(AB – BA)2 = (det(AB + BA) – 4detA ⋅ detB) ⋅ I2.
Prof. Traian Tămâian, Carei, Satu Mare, Gazeta Matematică
2. StudiaŃi convergenŃa şirului (xn)n∈N*, ştiind că pentru orice n ∈ N au loc relaŃiile: xn > 0 şi
(1 + x n +1 ) n +1 2 1
≤ − .
(1 + nx n ) 2
1 + nx n (1 + x n +1 ) n +1
Prof.dr. Raluca Ciurea, Craiova
Revista Gh. łiŃeica
3. Fie f: [1, ∞) → R o funcŃie ce are proprietatea lui Darboux pe [1, ∞) şi f(1) = 0. Dacă există
F: [1, ∞) → [0, ∞) astfel încât F'(x) = f(x), (∀) x ∈ (1, ∞) şi (x – 1)f(x) ≥ F(x), (∀) x ∈ [1, ∞),
demonstraŃi că F' = f.
Prof. Iuliana Coravu, Craiova

CLASA A XII-
XII-A

âx + 3̂y = 2̂
1. RezolvaŃi în Z8 sistemul  , unde â ∈ Z8.
3̂x + ây = 2̂
Prof. Aurel Doboşan, Gazeta Matematică

128
2. f: R → R este o funcŃie continuă şi a, b ∈ R, a < b îndeplinesc condiŃia:
b b
∫a
f (a + b − x + t )dx ≥ ∫ f (x)dx , pentru orice t ∈ R. DemonstraŃi că f(a) = f(b).
a
Revista Gh. łiŃeica, Prof. Cristian MoanŃă
3. Dacă f: R → R este o funcŃie continuă şi concavă, demonstraŃi că:
4
3 2 − 4 + 12 x − 6 x 2 + x 3
2
f (1) + ∫
4 f ( x )dx ≥
3

1
3
(2 − x )3
⋅ f ( x )dx .

Prof.dr. Raluca Ciurea, Craiova

Concursul interjudeŃean de matematică „Argument”, Baia Mare, EdiŃia I

CLASA A IX-
IX-A

1. Printr-un punct din interiorul unui paralelogram de arie S şi perimetru P se duc paralele la laturile
sale, care împart paralelogramul în patru paralelograme mai mici. Să se arate că cel puŃin două
1 1
dintre ele au perimetrele cel mult P şi cel puŃin două din ele au ariile cel mult S .
2 4
2. Fie x, y numere reale care verifică relaŃia x(x + y) = (x + y + 1)(1 – y).
2 2

Să se arate că dacă x este număr natural, atunci y este număr iraŃional.


3. Să se determine numerele pozitive a, b astfel încât mulŃimea punctelor (x, y) din plan care verifică
relaŃiile: |x| < a, |y| < b şi |x| + |y| < 1 să fie punctele din interiorul unui octogon regulat.

CLASA A X-
X-A

1. Să se determine funcŃiile f: R → R care verifică relaŃia f(f(x) + y) = 3x + f(f(y) – 2x), pentru orice
x ∈ R şi y ∈ R.
2. Fie (xn)n un şir de numere reale nenule care verifică relaŃia: x 2n − x n +1x n −1 = a , n ≥ 1, unde a, x0,
x1, x2 sunt numere întregi.
a) Să se arate că există α, β ∈ R astfel ca xn+1 = αxn + βxn–1, n ≥ 2.
b) Să se arate că dacă x1 divide pe x2 + x0, atunci toŃi termenii şirului sunt numere întregi.
3. Fie ABCD un patrulater convex. Să se determine locul geometric al punctelor X din planul patru-
laterului, care verifică relaŃia:
XA2 + CB2 + CD2 = XB2 + XC2 + DA2 = XC2 + XD2 + AB2 = XD2 + XA2 + BC2.

CLASA A XI-
XI-A

a n +1 = a 2n + a n b n
1. Se consideră şirurile (an)n, (bn)n date prin relaŃiile de recurenŃă:  , n ≥ 0, a0 ∈ R,
b n +1 = b 2n + a n b n
b0 ∈ R. Să se studieze monotonia şi mărginirea celor două şiruri.
129
2. Fie z = x + iy un număr complex cu x, y reale.
a) Să se arate că dacă 2 ⋅ | z + 1 | =| z + i | + | z − i | , atunci |z| = 1 şi x ≥ 0.
b) Să se determine locul geometric al punctelor M din planul pătratului ABCD pentru care are loc
relaŃia: 2 max{MA, MC} = MB + MD.
3. Să se arate că pentru orice n ≥ 3 există o permutare σ a mulŃimii {1, 2, ..., n} astfel că pentru orice
i, j, k cu 1 ≤ i < j < k ≤ n să avem: σ(i) + σ(k) ≠ 2σ(j).

CLASA A XII-
XII-A

1. Se consideră funcŃiile f: R × R → R, f(x, y) = axy – x – y + 2 şi g: (–1, 1) × (–1, 1) → R,


bx + cy
g(x, y) = , unde a, b, c sunt numere reale fixate. Să se determine a, b, c astfel ca:
1 − xy
a) legea x & y = f(x, y) să fie lege de compoziŃie pe intervalul [0, 2];
b) legea x  y = g(x, y) să fie lege de compoziŃie pe intervalul (–1, 1);
c) în ce condiŃii legile de la a) şi b) admit element neutru?
2. Se consideră şirurile de numere reale (xn)n, (yn)n definite prin relaŃiile de recurenŃă:
x + 3y n 3x + 2 y n
x n +1 = n , y n +1 = n , n ≥ 0.
4 5
Să se arate că pentru orice x0, y0 şirurile sunt convergente şi să se determine limitele lor.
3. Să se arate că dacă există două matrice A, B ∈ Mn(R) astfel ca det(AB – BA) ≠ 0 şi A2 + B2 =
π
= ctg ( AB − BA) , unde k ∈ N*, atunci n este multiplu de k. Să se dea exemple de astfel de
k
matrice pentru n = 2010.

Concursul interjudeŃean de matematică „Petre Moroşan – TRIDENT”,


„Memorialul Mircea Ganga”, EdiŃia a VII-a, SecŃiunea A (M1), Brăila,
6-8 noiembrie 2009

CLASA A IX-
IX-A

1. Se consideră triunghiul ABC dreptunghic în A, BC = a, AC = b, AB = c. ÎnălŃimea din A,


bisectoarea interioară din B şi mediana din C sunt concurente dacă şi numai dacă b2 = ac.
Prof. Gheorghe Alexe, Brăila
2. Dacă numerele reale strict pozitive a, b, c satisfac relaŃia a2 + b2 + c2 + a + b + c = 2, demonstraŃi
1 1 1 9
că + + ≥ .
c + ab b + ca a + bc 2
Prof. Adriana şi Lucian Dragomir, OŃelu-Roşu
Revista de Matematică de Brăila nr. 4

130
x
3. Să se determine z şi y ∈ R ştiind că [x] + [x + yz] =   , (∀) x ∈ R, unde [x] este partea întreagă
z
a numărului x.
Prof. Gabriel Daniilescu, Brăila

CLASA A X-
X-A

1. Fie ABCDE un pentagon şi M, N, P, Q punctele de intersecŃie ale segmentelor ce unesc mijloa-


cele laturilor opuse în patrulaterele BCDE, CDEA, EABD, ABCE respectiv. Să se demonstreze
că MNPQ este paralelogram dacă şi numai dacă ABCD este paralelogram.
Prof. Traian Tămâian, Carei, Gazeta Matematică 4/2009
2. Dacă a, b, c > 0 astfel încât ab + ac + bc = 3, demonstraŃi că:
a 3 (a + b ) b 3 ( b + c) c 3 (c + a )
+ + ≥ 2.
a 2 + ab + b 2 b 2 + bc + c 2 c 2 + ac + a 2
Prof. Carmen şi Viorel Botea, Brăila
3. Să se determine funcŃiile f: Q × Q → R cu proprietăŃile:
f(x + y, z + t) = f(x, z) + f(y, t), (∀) x, y, z, t ∈ Q;
f2(x, y) = f(x2 + y2, 2xy), (∀) x, y ∈ Q.
Prof. Andrei Velicu, ConstanŃa, Gazeta Matematică 1/2009

CLASA A XI-
XI-A

1 2 ... n 
1. Fie permutarea σ ∈ Sn, σ =   , unde n = 4k + 2, k ≥ 1. Să se arate că:
 n n − 1 ... 1 
a) ecuaŃia x2a = σ, x ∈ Sn, a ∈ N* nu are soluŃii în Sn;
b) nu există permutările τ1, τ2, ..., τ2l+1 ∈ Sn, l ≥ 1 astfel încât τ1τ2 = τ2τ3 = ... = τ2lτ2l+1 = τ2l+1τl = σ;
c) produsul tuturor permutărilor din Sn este o permutare pară, indiferent de ordinea permutărilor
din produs.
Prof. Gheorghe Alexe, Brăila
 0 2009 0 
 
2. Să se determine A, B ∈ M3(N) care verifică egalitatea: AB = BA =  0 0 2009  .
 2009 2 0 0 

Prof. Gabriel Daniilescu, Brăila
1− n
3. Fie şirurile de numere reale (an)n, (bn)n. Să se arate că dacă: an + bn ≥ 0 şi a 3n + b 3n − 3a n b n ≤ ,
n
(∀) n ≥ 1, atunci şirurile sunt convergente şi să se afle limitele lor.
Prof. Dan Negulescu, Brăila

131
CLASA A XII-
XII-A

1. Fie (G, ⋅) un grup multiplicativ având elementul neutru e şi a un element al său astfel încât axa–1 =
= x3, (∀) x ∈ G. Să se arate că xax–1 = a3, (∀) x ∈ G.
Prof. Rodica şi Dumitru Bălan, GalaŃi
x
2. Să se determine primitivele funcŃiei f: R → R, f ( x ) = .
2x + 2− x + 2
Prof. Dan Negulescu, Brăila
3. Pentru funcŃia f: R → R considerăm Mf = {ϕ: R → R | ϕ continuă şi f  ϕ = f}.
a) ArătaŃi că Mf este monoid cu operaŃia de compunere a funcŃiilor.
b) În cazul f(x) = x2 – 2x, (∀) x ∈ R, demonstraŃi că Mf are patru elemente.
c) În cazul f(x) = cos x, (∀) x ∈ R, dacă ϕ ∈ Mf, demonstraŃi că ϕ este derivabilă pe R – {kπ |
k ∈ Z}.
DaŃi exemplu de o funcŃie ϕ ∈ Mf care să nu fie derivabilă în niciun punct de forma kπ, k ∈ Z.
Prof. Radu Vasile, Brăila

Concursul interjudeŃean de matematică „Dimitrie Pompeiu”, EdiŃia a X-a,


Botoşani, 14-16 mai 2010

CLASA A IX-
IX-A

1. Pe latura (AB) a triunghiului ABC se consideră punctul D astfel încât 5 ⋅ AD = 2 ⋅ DB. Punctul
M se găseşte pe segmentul (DC) astfel încât 3 ⋅ CM = 7 ⋅ DM. Dreapta BM intersectează dreapta
AC în punctul E iar dreapa AM intersectează dreapta BC în punctul F. GăsiŃi raportul dintre
A∆DEF şi A∆ABC.
2. Să se determine funcŃiile f: N* → N*, care satisfac condiŃiile:
a) f(1) = 1;
b) f(m + n + m ⋅ n) = f(m) + f(n) + f(m ⋅ n), oricare ar fi m, n ∈ N*.
1
3. Fie ecuaŃia x2 – x(a + b) + a2 + b2 = , unde a, b ∈ R.
2
1
a) Să se arate că ecuaŃia are cel puŃin o rădăcină întreagă dacă şi numai dacă a 2 + b 2 = .
2
b) Să se determine a şi b astfel încât ambele rădăcini să fie întregi.

Problema suplimentară
ArătaŃi că oricare ar fi triunghiul ABC cu mijloacele laturilor [BC], [CA], [AB], notate cu A', B',
C', există un triunghi DEF având EF = AA', FD = BB', DE = CC'.

132
CLASA A X-
X-A

1. Fie numerele complexe z1, z2, z3 astfel încât |z2 – z3| + |z3 – z1| + |z1 – z2| = 1. Notăm în acelaşi fel
afexele lor. Se notează z4 = z1 ⋅ |z2 – z3| + z2 ⋅ |z3 – z1| + z3 ⋅ |z1 – z2|. ArătaŃi că distanŃele de la z4 la
dreptele z1z2, z2z3, z3z1 sunt egale.
1
2. Fie x = log a (a + 1) şi y = log a +1 (a + 2) , unde a ∈ R, a > 1.
2
a) DemonstraŃi că 2x > y.
n n
b) ComparaŃi numerele A n = 2 2 x + 2 x + y şi B n = 2 y + 2 3x , unde n ∈ N*.
n n

3. Fie f: N → N o funcŃie bijectivă. ArătaŃi că există trei numere naturale a < b < c astfel încât f(a) +
+ f(c) = 2010f(b).

Problema suplimentară
Pentru segmentele d, e, f găsiŃi condiŃii pentru a exista un triunghi ABC având înălŃimile egale cu
segmentele date.

CLASA A XI-
XI-A

1. a) Să se arate că dacă A ∈ M4,3(R) şi B ∈ M3,4(R), atunci det(AB) = 0.


b) Dacă n, p ∈ N*, A ∈ Mn,p(R) şi B ∈ Mp(R), cu detB ≠ 0, să se demonstreze că rang(AB) =
= rangA.
c) Dacă A, B ∈ Mn(R) şi există a, b ∈ R*, astfel încât AB = aA + bB, să se arate că rangA =
= rangB.
1 1 2 x n + x n −1 + 1
2. Se consideră şirul (xn)n∈N cu x0 = 0, x1 = şi x n +1 = , pentru orice n ≥ 1.
2 2 xn
a) ArătaŃi că şirul (xn)n∈N este convergent şi lim x n = 1 .
n →∞

b) CalculaŃi lim 4 (1 − x n ) .
n
n →∞

x
3. Fie funcŃia f: R → R, f ( x ) = − arctgx . Să se arate că oricare ar fi punctul c ∈ (–2π, –π),
x2 +1
există a, b ∈ [–2π, π], astfel încât f(b) – f(a) = (b – a) ⋅ f'(c).

Problema suplimentară
DaŃi exemplu de triunghi ABC cu bisectoarele [AD], [BE], [CF] astfel încât să nu existe un
triunghi având lungimile laturilor cât aceste bisectoare.

133
Concursul interjudeŃean de matematică „Vasile Dumitrache”

CLASA A IX-
IX-A

1. Fie p ∈ N un număr prim. Să se determine numerele întregi k ∈ Z pentru care k 2 − pk să fie


număr natural.
2. Pe laturile triunghiului ABC se consideră punctele A1 ∈ (BC), B1 ∈ (CA), C1 ∈ (AB) care le
împarte în acelaşi raport pozitiv neunitar. Fie {A2} = BB1 ∩ CC1; {B2} = CC1 ∩ AA1; {C2} =
= AA1 ∩ BB1. ArătaŃi că:
a) triunghiurile ABC şi A2B2C2 au acelaşi centru de greutate;
b) vectorii AA 2 ; BB2 , CC 2 pot fi laturile unui triunghi.
3. Să se rezolve în R ecuaŃia: {x}n – [x]n = xn, unde {x}, [x] reprezintă partea fracŃionară, respectiv
partea întreagă a lui x, iar n ∈ N.

CLASA A X-
X-A

 y

1. ArătaŃi că  ∑ z k  − 1 divide 2010, unde x, y sunt soluŃiile naturale ale ecuaŃiei:
 k =x 
x 2 + y 2 −5 x 2 +1
2 − 19 ⋅ 2 − 2010 = 0 şi z = log2 y.
2. Fie ABCD un patrulater înscris într-un cerc de rază R. DemonstraŃi că AB2 + BC2 + CD2 + DA2 =
= 8R2, dacă şi numai dacă AC ⊥ BD sau una dintre diagonale este diametru în cercul circumscris
patrulaterului.
3. ArătaŃi că pentru orice m ∈ R – {1}, imaginile pe cercul trigonometric ale soluŃiilor ecuaŃiei
x 3x x 3x
cos cos + m sin sin = cos x sunt vârfurile unui triunghi echilateral.
2 2 2 2

CLASA A XI-
XI-A

1. Fie A şi B două matrice de ordin k mai mare sau egal cu 1. Să se demonstreze că dacă det(A +
+ nB) = det(nA + B) pentru cel puŃin k + 1 valori distincte ale numărului natural n, atunci detA =
= detB.
2. Fie f: R → R o funcŃie continuă şi mărginită. DemonstraŃi că există a, b ∈ R astfel încât a – b = 1
şi (a2 + 1)f(b) = (b2 + 1)f(a).
3. Fie (an)n≥1 un şir de numere reale monoton crescător, astfel încât an ≥ 1, (∀) n ∈ N*, iar (xn) şi (yn)
1 + k[a n ] 
n n
două şiruri de numere reale definite prin: x n = ∑ 
k =1  k 

 , y n = [a k ] , unde [,] reprezintă
k =1

134
partea întreagă. Dacă (zn) este un şir de numere reale strict pozitive cu proprietatea:
n (2a n + 1) x − yn
lim = 0 , calculaŃi lim n .
n →∞ zn n →∞ zn

Concursul interjudeŃean de matematică „LaurenŃiu Panaitopol”, Tulcea,


15 mai 2010

CLASA A IX-
IX-A

1. Să se afle valorile parametrului real m pentru care ecuaŃia x2 – 2mx – 25 = 0 are două soluŃii
întregi.
2. Să se determine funcŃiile strict monotone f: N → N care satisfac următoarele condiŃii:
a) f(2n) = n + f(n), oricare ar fi n natural;
b) f(n) este pătrat perfect dacă şi numai dacă n este pătrat perfect.
3. Fie un triunghi ABC. Un cerc de rază ρ ce trece prin B şi C taie AB şi AC în E şi D. Fie ρ' raza
cercului circumscris triunghiului ADE. Să se arate că există un triunghi cu laturile R, ρ, ρ' şi un
unghi al acestui triunghi este constant.
4. Pe laturile AB, BC, CD, DA ale paralelogramului ABCD se consideră punctele M, N, P, Q astfel
AM CN CP AQ
încât = l, = k, = m, = p , unde l, k, m, p > 0 şi AP + AN + CQ + CM = 0 . Să se
MB NB PD QD
arate că dreptele QN, PM şi AC sunt concurente.

CLASA A X-
X-A

1. Să se rezolve în mulŃimea numerelor reale ecuaŃia arcsin x = [2x].


2. Fie n ∈ N astfel încât n ∉ N şi A = { a + b n , a, b ∈ N, a2 – nb2 = 1}. Să se arate că funcŃia
f: A → N, f(x) = [x] este injectivă dar nu este surjectivă.
3. Fie z un număr complex cu proprietatea: pentru orice număr natural n, partea reală a numărului zn
este mai mare decât partea imaginară a numărului zn. Să se arate că z este număr real pozitiv.
n
4. Fie a1, a2, ..., an numere reale. Să se arate că: ∑ cos(a i − a j ) ≥ − .
1≤ i < j≤ n 2

CLASA A XI-
XI-A

1. Să se arate că, dacă an → 6 şi x n = 3 a n + 3 a n −1 + ... + 3 a 0 , atunci xn → 2.


2. Matricele A, B ∈ Mm(C) au proprietatea (A + B)2 = A2 + B2 şi (A + B)4 = A4 + B4. Să se arate că
(AB)2 = On.

135
3. Fie f: R → R o funcŃie de două ori derivabilă, cu f''(x) ≠ 0, (∀) x ∈ R, având proprietatea: pentru
orice număr real x există cel puŃin un număr real cx astfel încât f(x + 1) – f(x) = f'(cx) şi f(x + 1) –
– f'(x) = f''(cx). Să se arate că:
a) asocierea x → cx este o funcŃie injectivă;
b) funcŃia f este indefinit derivabilă şi, pentru orice n natural, f(n)(x + 1) – f(n)(x) = f(n+1)(cx).
4. Se consideră matricea A ∈ Mn(R), unde n ∈ N, n ≥ 2 este impar. Să se arate că dacă detA > 0 şi
A + A* = 2A–1, atunci A2010 + (A*)2010 = 2(A–1)2010, (A* este adjuncta matricei A, iar A–1 este
inversa matricei A).

CLASA A XII-
XII-A

1. Fie a, b ∈ R, a < b. Spunem că funcŃia f: [a, b] → R are proprietatea P dacă şi numai dacă pentru
orice şir (xn)n≥1 de puncte din [a, b] pentru care şirul (f(xn))n≥1 este convergent, rezultă că (xn) este
convergent.
a) DemonstraŃi că dacă f are proprietatea P şi f admite primitive, atunci f este integrabilă.
b) Dacă f are proprietatea P şi f este integrabilă, rezultă că f are primitive?
b
2. Fie f: R → R o funcŃie derivabilă cu proprietatea: există c ∈ R astfel încât ∫ f (x)dx ≠ (b − a)f (c) ,
a
oricare ar fi a, b ∈ R, a ≠ b. Să se arate că f'(c) = 0.
3. Se consideră polinoamele P şi Q cu coeficienŃi reali, care au proprietatea că mulŃimile {n ∈ N |
| P(n) ≤ Q(n)} şi {n ∈ N | P(n) ≥ Q(n)} sunt infinite. ArătaŃi că P = Q.
4. Fie A un inel pentru care există numerele naturale nenule n şi k, astfel încât xk = xn+k pentru orice
x ∈ A. Să se arate că există un polinom P cu coeficienŃi în A, de grad n(n + 1), astfel încât P(x) =
= 0, pentru orice x ∈ A.

Concursul interjudeŃean de matematică „Victor Vâlcovici”, Brăila,


15 mai 2010

CLASA A IX-
IX-A

1. DeterminaŃi toate funcŃiile f: R → R, cu proprietatea: f(2x + f(y)) = x + y + f(f(x)), (∀) x, y ∈ R.


Gazeta Matematică
 π
2. DemonstraŃi că (∀) x, y, z ∈  0,  avem:
 2
1 1 1
+ + ≥ 3.
( tgy + tgz ) cos x ( tgx + tgz) cos y ( tgx + tgy) cos 2 z
2 2

Prof. Carmen şi Victor Botea, Brăila

136
3. Fie ∆ABC ascuŃitunghic şi A' ∈ (BC), B' ∈ (AC), C' ∈ (AC) picioarele înălŃimilor. Presupunând
că triunghiurile ABC şi A'B'C' au acelaşi centru de greutate, calculaŃi:
aria (∆ABC)
.
aria (∆A' B' C' )
Prof. Carmen şi Victor Botea, Brăila

CLASA A X-
X-A

lg 2 a + lg 2 b
1. a) Fie s, t ≥ 0 şi a, b ≥ 1 numere reale. Să se arate că: a s ⋅sin x + t ⋅cos x ⋅ b s ⋅cos x + t ⋅sin x ≤ 10(s + t ) ,
(∀) x ∈ R.
b) Dacă a, b > 0 rămâne adevărată inegalitatea?
Prof. Ilie Diaconu, Gazeta Matematică
2. Fie n un număr natural nenul, A o mulŃime cu n elemente şi funcŃia f: A → A. Să se determine
numărul funcŃiilor f ce au proprietatea f(f(x)) = f(x), (∀) x ∈ A.
***
3. Fie ∆ABC ascuŃitunghic şi A' ∈ (BC), B' ∈ (AC), C' ∈ (AC) picioarele înălŃimilor. Presupunând
aria (∆ABC)
că triunghiurile ABC şi A'B'C' au acelaşi centru de greutate, calculaŃi .
aria (∆A' B' C' )
Prof. Carmen şi Victor Botea, Brăila

CLASA A XI
XI-A

a n +1 a n a an
1. Fie şirul (an)n≥1 dat astfel: a > 0, a1 > 0 şi = + , n ≥ 1. Să se calculeze lim .
a a an n →∞ n + a

Gazeta Matematică
2
2. Fie f: [2, ∞) → R o funcŃie derivabilă care satisface relaŃiile f(2) = 0 şi f ' ( x ) = 2 4 , (∀) x ∈
x + f (x )
∈ [2, ∞). Să se arate că lim f ( x ) există şi este cel mult egală cu ln 3.
x →∞

Prof. Gabriel Daniilescu, Brăila


3. Fie A ∈ Mn(R), n ≥ 3, o matrice singulară astfel încât adjuncta sa are exact p elemente nenule,
unde p ≥ 2 este prim.
a) ArătaŃi că An–1 ≠ On.
b) ArătaŃi că p ≤ n.
Prof. Cristian Săvescu, Focşani

137
Concursul interjudeŃean de matematică „Alexandru Myller”, 2010

CLASA A IX-
IX-A

25
1. Să se arate că pentru orice număr real x ≥ , intervalul [x, 2x] conŃine trei întregi distincŃi în
2
progresie geometrică.
2. Se consideră o mulŃime A cu n ≥ 2 elemente şi 2n–1 submulŃimi distincte ale lui A. Se ştie că
oricare trei dintre aceste submulŃimi au intersecŃia nevidă. Să se arate că intersecŃia tuturor
submulŃimilor date este nevidă.
3. Fie n ∈ N, n ≥ 2 şi o mulŃime {x1, x2, ..., xn} de numere reale cu proprietatea |xi – xj| ≥ 1 pentru
n (n − 1)(n + 1)
orice 1 ≤ i < j ≤ n. Să se arate că x12 + x 22 + ... + x 2n ≥ .
12
4. Fie M ∈ (BC), N ∈ (AC), P ∈ (AB) picioarele a trei ceviene concurente în triunghiul ABC.
Paralela prin N la AB taie dreapta PM în E, paralela prin M la AB taie dreapta PN în F, iar
dreptele MN şi CP se taie în T.
NT BC AN
a) Să se arate că = ⋅ .
TM AC BM
b) Să se arate că dreptele MN, EF şi PC sunt concurente.

CLASA A X-
X-A

1. Dacă x, y ∈ (1, 2], demonstraŃi că: logx(3y – 2) + logy(3x – 2) ≥ 4.


Prof. Radu Sava
13
2. Dacă z este un număr complex de modul 1, arătaŃi că: 3 ≤ |1 + z| + |1 – z + z2| ≤ . Când se
4
realizează egalităŃile?
***
3. DeterminaŃi numerele reale x care pot fi scrise sub forma:
1 a1 a2 a a
x= + + + ... + n − 2 + n −1 , unde n, a1, a2, ..., an sunt numere naturale
a1a 2 ...a n a 2a 3 ...a n a 3a 4 ...a n a n −1a n an
nenule cu a1 < a2 < ... < an.
Prof. Iurea Gheorghe
4. Fie P mulŃimea punctelor planului, iar f: P → P o funcŃie care duce dreptele în drepte şi astfel
încât orice patrulater convex este dus într-un patrulater convex de acelaşi perimetru. DemonstraŃi
că f este izometrie.
Notă: FuncŃia f: P → P se numeşte izometrie dacă oricare ar fi punctele M şi N din plan, seg-
mentele MN şi f(M)f(N) au lungimi egale.
Prof. Paul Georgescu şi Gabriel Popa

138
CLASA A XI-
XI-A

41x + 40
1. DaŃi un exemplu de funcŃie f: (0, ∞) → (0, ∞) cu proprietatea că f (f (f ( x )))) = , pentru
40 x + 41
orice x > 0.
2. Fie A şi B două matrice din Mn(C), cu proprietatea că AB este nilpotentă şi ABA ≠ On. ArătaŃi că
există o matrice C ∈ Mn(C), astfel încât ABAC = On şi AC ≠ On.
3. DeterminaŃi funcŃiile continue f: R → R cu proprietatea că 2f(x + 1) = f(x) + 4f(2x), oricare ar fi
x ∈ R.
Prof. Dinu Şerbănescu
4. Fie f: (0, ∞) → R o funcŃie având proprietatea lui Darboux, astfel încât pentru orice a ∈ R,
ecuaŃia f(x) = a are un număr finit de soluŃii. ArătaŃi că f are limită la infinit.
Prof. Dinu Şerbănescu

CLASA A XII-
XII-A

1. Fie f: [a, b] → R o funcŃie integrabilă cu proprietatea că pentru orice x ∈ [a, b], există y ∈ (x, b)
y b
astfel încât ∫
x
f ( t )dt > 0 . ArătaŃi că ∫ f (x )dx > 0 .
a
Prof. Călin Popescu
x
2. Fie f: [0, ∞) → R o funcŃie crescătoare. ArătaŃi că ∫ (2t − x)f ( t )dt ≥ 0 , oricare ar fi x ∈ R.
0
Prof. Marian Andronache
3. Fie n un număr natural, n ≥ 2 şi G ⊂ M3(C) un grup cu n elemente în raport cu înmulŃirea
matricelor, cu proprietatea că tr(AB) = tr(A) ⋅ tr(B), oricare ar fi A, B ∈ G. ArătaŃi că:
a) elementul neutru al lui G este diferit de I3;
b) G este izomorf cu (Un, ⋅), grupul rădăcinilor complexe de ordinul n ale unităŃii.
Prof. Dinu Şerbănescu
7

4. Fie A un inel finit comutativ cu cel puŃin trei elemente şi a = x , b = x 8 . ArătaŃi că cel
x∈A

x∈A
puŃin unul dintre elementele a şi b este neinversabil.
Prof. Marian Andronache

Concursul interjudeŃean de matematică „Alexandru Papiu Ilarian”,


EdiŃia a XIV-a, Târgu-Mureş, 2009

CLASA A IX-
IX-A

1. a) Să se determine primele zece numere naturale care sunt multipli de 2 sau 5, dar nu sunt multipli
de trei.

139
x x  x  x  x   x 
b) Să se rezolve ecuaŃia:   +   +   = 10 +   +   +   , x ∈ N.
 2   5   30   6  10  15 
Prof. Vasile Pop
2. Cercul înscris în triunghiul ABC este tangent laturilor BC, CA, AB în A', B', C'. Să se arate că:
AB'+ AC' + BC'+ BA' + CA '+ CB' ≤ BC + CA + AB .
Prof. Vasile Pop
3. Se consideră şirul (xn)n≥0 definit prin relaŃia de recurenŃă: x n +1 = 3x n + 8x 2n + 1 , n ∈ N, x0 = 0.
a) Să se arate că toŃi termenii şirului sunt numere naturale.
b) Să se determine numerele naturale n, pentru care xn + 1 se divide cu 6.
Prof. Vasile Pop
4. Fie M = {A1, A2, ..., An} mulŃimea vârfurilor unui poligon regulat.
a) Să se determine numărul triunghiurilor isoscele care se pot forma cu vârfuri din M.
b) Să se determine numărul minim de vârfuri care trebuie eliminate din mulŃimea M astfel încât
să nu se poată forma triunghiuri echilaterale cu vârfurile rămase.
Prof. Vasile Pop

CLASA A X-
X-A

1. Fie O, A, B, C patru puncte în plan astfel ca distanŃele dintre oricare două să fie numere raŃionale.
Să se arate că există numere raŃionale a, b, c astfel ca a ⋅ OA + b ⋅ OB + c ⋅ OC = 0 .
Prof. Vasile Pop
2. Fie a, b, c, x, y, z numere pozitive care verifică relaŃiile: a = cy + bz, b = az + cx, c = bx + ay. Să
3
se arate că 1 ≤ x + y + z ≤ .
2
Prof. Vasile Pop
3. Să se arate că numerele reale distincte a1, a2, ..., an sunt în progresie aritmetică dacă şi numai dacă
mulŃimea D = {ai – aj | i, j = 1, n } are 2n – 1 elemente.
Prof. Vasile Pop
4. Fie m, n numere naturale şi A = {a1, a2, ..., an} o mulŃime de numere întregi. Să se arate că:
a) dacă 1 ≤ m ≤ n, atunci există indici distincŃi i1, i2, ..., ik ∈ {1, 2, ..., n} astfel ca suma a i1 + a i 2 +
+ ... + a i k să se dividă cu m;
b) dacă n < m < 2n, atunci există i1, i2, ..., ik ∈ {1, 2, ..., n} şi o alegere a semnelor + şi –, astfel ca
numărul ± a i1 ± a i 2 ± ... ± a i k să fie divizibil cu m.
Prof. Vasile Pop

140
CLASA A XI-
XI-A

1. Să se determine numărul elementelor mulŃimii: A = {(a1, a2, ..., a10) | 1 ≤ a1 < a2 < ... < a10 ≤ 101 şi
ai+1 – ai ≥ 10, i = 1, 9 }.
Prof. Vasile Pop
2. Fie a, b numere reale pozitive. Definim şirul (xn)n prin relaŃia de recurenŃă: xn+1 = (a + b)xn – abxn–1,
n ≥ 1, cu x0 = 1 şi x1 ∈ R – {a, b}. Să se arate că şirul (xn)n conŃine o progresie aritmetică infinită
dacă şi numai dacă a = b = 1.
Prof. Vasile Pop
3. Să se arate că singura permutare σ a mulŃimii {1, 2, ..., n} care are proprietatea 1 + σ(1) < 2 +
+ σ(2) < ... < n + σ(n) este permutarea identică.
***
4. Fie P un polinom de grad n ≥ 2 cu proprietatea P(k) = k ⋅ 2 , pentru orice k ∈ {0, 1, ..., n}.
k–1

a) Să se arate că: C1k + 2C 2k + ... + kC kk = k ⋅ 2 k −1 .


b) Să se determine P(n + 1).
Prof. Vasile Pop

CLASA A XII-
XII-A

a 1 a b 
1. Pentru a ∈ R considerăm matricea X n =   şi notăm (X a ) n =  n n
 , n ≥ 1.
− 1 a  −
 nb a n

Să se arate că există a ∈ R pentru care b1 < a1, b2 < a2, ..., b2009 < a2009 şi b2010 > a2010.
Prof. Vasile Pop
2. Fie f: R × R → R o funcŃie şi (an)n un şir de numere reale care verifică relaŃia de recurenŃă an+1 =
= f(an, an–1), (∀) n ≥ 1. Să se arate că dacă mulŃimea A = {an | n ∈ N} este finită, atunci există
P( x )
două polinoame P, Q ∈ R[X] şi există lim (a 0 + a1x + a 2 x 2 + ... + a n x n ) = , (∀) x ∈ (–1, 1).
n →∞ Q( x )
Prof. Vasile Pop
3. Se consideră funcŃiile f: R → R, derivabilă şi g: R → R de două ori derivabilă cu proprietăŃile:
f'(x) – 2f(x) ≥ 0, (∀) x ≥ 0, f(0) = 1; g''(x) – 3g'(x) + 2g(x) ≥ 0, (∀) x ≥ 0, g(0) = 0, g'(0) = 1.
Să se arate că:
a) f(x) ≥ e2x, (∀) x ≥ 0; b) g(x) ≥ e2x – ex, (∀) x ≥ 0.
Prof. Vasile Pop
4. Fie A o matrice de ordin 2n, n ≥ 1 cu elementele numere naturale şi cu proprietatea:
(P): pentru orice două linii Li, Lj cu i ≠ j, suma lor Li + Lj conŃine n elemente numere pare şi n
elemente numere impare.
a) Să se arate că pentru orice două coloane Ci şi Cj cu i ≠ j, suma lor Ci + Cj conŃine n elemente
numere pare şi n elemente numere impare.
b) Să se arate că pentru orice k ≥ 1 există matrice de ordin 2k cu proprietatea (P).
Prof. Vasile Pop
141
Concursul interjudeŃean de matematică „Danubius”, Corabia, 2010

CLASA A IX-
IX-A

 x + 2 x + y = b
1. Se dă sistemul de ecuaŃii:  , cu b, c ∈ R, b ≠ c.
 x + y = c
a) Fără a rezolva sistemul, să se arate că soluŃiile reale (x, y) satisfac condiŃiile: 0 < x < b – c şi
y < c < b.
b) Să se rezolve sistemul în mulŃimea numerelor reale.
Prof. Petre Stavre
32
2. Să se arate că (1 + sin x ) cos 2 x ≤ , oricare ar fi x ∈ [0, π].
27
Gazeta Matematică, prof. Dan Nedeianu
3. Fie O, I, G, H centrele cercurilor circumscris, înscris, centrul de greutate, respectiv ortocentrul
2
triunghiului ABC. Să se arate că: OA + IB + GC = OH , dacă şi numai dacă ∆ABC este echila-
3
teral.
Prof. Lucian TuŃescu, Craiova, prof. Ion Nedelcu, Ploieşti
4. Dacă x, y, z ≥ 0, să se demonstreze că:
2
x 2 + [ y ] 2 + {z}2 + y 2 + [ z ] 2 + {x}2 + z 2 + [ x]2 + { y}2 ≥ ( x + y + z) .
3
Prof. Dorin Mărghidanu, Corabia

CLASA A X-
X-A

1. Fie ABCD un patrulater convex şi {O} = AC ∩ BD. Să se arate că există un singur punct P ∈
∈ (ABCD) astfel încât G1G3 ∩ G2G4 = {O}, unde G1, G2, G3, G4 sunt centrele de greutate ale
triunghiurilor PAB, PBC, PCD şi PDA.
Prof. Lucian TuŃescu, Craiova, prof. Ion Nedelcu, Ploieşti
(r + z1 )(r + z 2 )(r 2 + z1z 2 )
2. Fie z1, z2 ∈ C* de modul r. Să se arate că numărul r 2 + este real pozitiv.
z1z 2
Gazeta Matematică (Dan Nedeianu)
3. Fie f: R → R o funcŃie cu proprietatea că f(f(x)) = [x], (∀) x ∈ R. Să se arate că există a, b ∈ R,
a ≠ b cu |f(a) – f(b)| ≥ |sin a – sin b|.
Prof. Cristian MoanŃă, Craiova
4. Dacă a1, a2, a3, a4 sunt numere reale pozitive, să se demonstreze că:
44 a1a 2a 3a 4 − 33 a1a 2a 3 ≤ a 4 . Să se precizeze cazul de egalitate. Generalizare.
Prof. Dorin Mărghidanu, Corabia

142
Concursul interjudeŃean de matematică pe echipe „Gheorghe łiŃeica”,
EdiŃia a XXXII-a, Drăgăşani, 21-22 mai 2010

Proba individuală

CLASA A IX-
IX-A

1. RezolvaŃi în N ecuaŃia: n = 3[ n ] + 1 .
***
2. Să se determine toate funcŃiile f: R → R cu proprietatea: f(xf(x) + f(y)) = (f(x)) + y, ∀ x, y ∈ R, (1).
2

3. Fie ma, mb, mc lungimile medianelor unui triunghi iar a, b, c lungimile laturilor corespunzătoare.
 2
Dacă a + k ⋅ ma = b + k ⋅ mb = c + k ⋅ mc, unde k ∈  0,  , atunci triunghiul este echilateral.
 3
Gazeta Matematică

CLASA A X-
X-A

x +1 x
1 1
x
 3
1. Să se rezolve ecuaŃia   +   − 3  = 3 în mulŃimea numerelor reale.

3  12   12 
Gazeta Matematică, 6/2009
2. a) Fie z1, z2 numere complexe cu |z1| = |z2|. Să se arate că: 2010 ⋅ |z1 + z2| ≤ 2 ⋅ |2009z1 + z2|.
b) Trei puncte M1, M2, M3, care se găsesc pe o sferă de centru O, formează un triunghi echilateral.
Introducem punctele M13 pe segmentul M1M3, M32 pe segmentul M3M2 şi M21 pe segmentul
M M M M M M 1
M2M1 astfel încăt: 13 3 = 32 2 = 21 1 = .
M1M 3 M 3M 2 M 2 M1 2010
Să se arate că segmentele OM13, OM32 şi OM21 au lungimi egale.
c) Folosind, eventual, punctul b), daŃi o justificare geometrică a inegalităŃii de la punctul a).
3. Fie n ≥ 2 număr natural şi q > 1 număr real. GăsiŃi un set de condiŃii suficiente pentru numerele
reale pozitive x1, x2, ..., x2n+1 astfel încât să aibă loc dubla inegalitate:
2
 1 1   qn −1   2 1 
( x1 + ... + x 2 n +1 ) + ... +  ≥ 1 + 4n + 2n 2 + k > (2n + 1) 2 , unde k =  
  q + 2 n  .
 x1 x 2n +1   q −1   q 

CLASA A XII-
XII-A

1. Într-un inel A cu 0 ≠ 1, avem x + y = 1 + xy, ∀ x, y ∈ A – {0}. Să se arate că A ≅ Z2.


Gazeta Matematică

143
2. Fie f: [0, ∞) → [0, 1) o funcŃie continuă ce are limita 0 la infinit. DemonstraŃi că:
lim  f n ( x )dx + f ( x )dx  = 0 .
1 2 n

n →∞ ∫
0 ∫1
f n −1 ( x )dx + ... + ∫
n −1 
Gazeta Matematică
3. Să se determine funcŃiile derivabile f: [0, ∞) → R, cu proprietăŃile:
a) f(0) = 0;
1  x  2  2x 
b) f ' ( x ) = f '   + f '   , (∀) x ∈ [0, ∞).
3 3 3  3 
***
Proba pe echipe, clasele IX-X
1. Să se determine α minim cu proprietatea „În orice triunghi există două laturi a şi b astfel încât
a
1≤ < α ”.
b
2. Să se determine şirurile (an)n≥0 de numere naturale, ştiind că:
a n + 2 + a 2n +1 − a n +1 + 2a n +1a n + a 2n − a n = 0 , oricare ar fi n ∈ N.
Gazeta Matematică 2/2009

Proba pe echipe, clasele XI-XII


1. Să se arate că pentru orice matrice A ∈ Mn(C), având pe diagonala principală toate rădăcinile de
ordinul n ale unităŃii, există matricele B, C ∈ Mn(C), astfel încât A = B + C şi Bn = In, Cn = On.
Gazeta Matematică
2. Fie n ∈ N, n ≥ 1 şi numerele reale pozitive x1 ≥ ... ≥ xn > 0, y1 ≥ ... ≥ yn > 0, ce verifică:
y1 y 2 + ... + y n y1 + y 2 y 3 + ... + y n y + ... + y n −1 y n y1 + ... + y n
≤ , ≤ ,..., 1 ≤ , ≤ 1.
x1 x 2 + ... + x n x1 + x 2 x 3 + ... + x n x1 + ... + x n −1 x n x1 + ... + x n
ArătaŃi că dacă f: R+ → R este o funcŃie derivabilă şi convexă, cu f(0) = 0, atunci are loc inegali-
f ( y1 ) + ... + f ( y n ) f ( x1 ) + ... + f ( x n )
tatea: ≤ .
y1 + ... + y n x1 + ... + x n
Prof. Ionel RovenŃa

Tabăra NaŃională de matematică. Concursul naŃional de matematică


„RecreaŃii matematice”, Durău, 28 august 2009

CLASA A IX-
IX-A

1. RezolvaŃi în necunoscuta (x, y) ∈ Q × Q ecuaŃia: x2009 + y2009 = x2010 + y2010.

144
2. Într-un careu cu 41 linii şi 49 coloane se scriu la întâmplare 2009 numere reale distincte. Fie A
mulŃimea ce are ca elemente cel mai mic număr de pe fiecare linie, respectiv B mulŃimea ce are ca
elemente cel mai mare număr de pe fiecare coloană. DeterminaŃi probabilitatea ca cel mai mare
element din mulŃimea A să fie chiar cel mai mic element din mulŃimea B.
3. Fie D un punct în planul unui triunghi echilateral ∆ABC, astfel încât BD = DC, m('BDC) = 30°
şi dreapta BC separă A şi D. Dacă E ∈ (BD) şi m('BAE) = 15°, să se arate că CE ⊥ AC.

CLASA A X-
X-A

1. RezolvaŃi, în necunoscuta (x, y) ∈ N × N, ecuaŃia: x ⋅ (x + 2) ⋅ (x + 8) = 3y.


2. Fie triunghiul ∆ABC cu m('ABC) = m('ACB) = 80° şi P ∈ (AB) astfel încât m('BPC) = 30°.
ArătaŃi că AP = BC.
3. DeterminaŃi funcŃiile f: N → N, pentru care are loc egalitatea:
2 ⋅ f(n + 3) ⋅ f(n + 2) = f(n + 1) + f(n) + 1, (∀) n ∈ N.

CLASA A XI-
XI-A

 π 
1. Fie şirul (xn)n≥1 definit prin: x1 ∈  − , 0  , xn+1 = 2xn – tg xn, (∀) n ≥ 1.
 4 
Să se studieze existenŃa limitelor: lim x n şi lim n − x n .
n →∞ n →∞
RecreaŃii Matematice
2. Să se determine funcŃiile derivabile f: I → (0, +∞) şi intervalul I ⊂ R ştiind că f(0) = 1 şi f3(x) +
+ f'(x) = 0.
3. Fie funcŃia g: [0, 1] → R derivabilă pe (0, 1) cu g(0) = 0, iar f: [0, 1] → R+ o funcŃie cu
proprietatea f(x) = g'(x), (∀) x ∈ [0, 1]. Să se arate că există cel puŃin un punct c ∈ (0, 1) astfel
π π 
încât: ⋅ g(c) ⋅ cos c  < f (c) .
2 2 

Proba pe echipe, Durău, 26 august 2009, SENIORI


1. Fie ABCD un patrulater înscris într-un cerc de centru O. Notăm cu P punctul de intersecŃie al
diagonalelor patrulaterului şi cu R punctul de intersecŃie al segmentelor ce unesc mijloacele
laturilor opuse ale patrulaterului. Să se arate că: OP ≥ OR.
2. La un test de matematică participă 8 elevi. Fiecare problemă a testului a fost rezolvată de exact 4
elevi, iar numărul problemelor comune rezolvate de oricare doi elevi este acelaşi. Să se arate că
testul are cel puŃin 14 probleme.
1
3. Fie şirul (xn)n≥0 definit prin: x0 > 0 şi x 0 + x1 + x 2 + ... + x n = , n ≥ 0.
x n +1
Să se calculeze lim n 2 ⋅ x 3n .
n →∞

145
Concursul de matematică RMCS, 7 martie 2010, OŃelu-Roşu

CLASA A IX-
IX-A

1. Se consideră o mulŃime de numere reale care satisfac următoarele proprietăŃi:


x
a) 1 ∈ M; b)   ∈ M ⇒ x ∈ M.
4
Să se arate că: 1) 2000 + 2000 ∈ M; 2) 2 2000 + 2 2000
∈ M.
Prof. Lucian Dragomir
n n
1 1
2. Pentru orice număr natural nenul n, se notează A n = ∑ şi B n = ∑ . ArătaŃi că:
k =1 k ( k + 1)
k
k =1 2
40
a) B30 < ; b) Bn ≤ An < 1, (∀) n ∈ N*;
41
c) pentru orice număr natural n ≥ 2, există numerele naturale nenule a1, a2, ..., an cel mult două
1 1
egale, astfel încât: + ... + =1.
a1 an
Prelucrare Gazeta Matematică 12/2009
3. O tablă de şah 5 × 5, cu pătrăŃele colorate alternativ în alb şi negru, au colŃurile negre. Pentru
fiecare pereche de pătrăŃele colorate diferit, se desenează câte un vector cu originea în centrul
pătrăŃelului negru şi vârful în centrul pătrăŃelului alb.
a) CâŃi vectori au fost desenaŃi?
b) CalculaŃi suma tuturor vectorilor desenaŃi.
RCMS 29
4. Fie A, B, C, D patru puncte coplanare, oricare trei necoliniare. DemonstraŃi că oricare trei dintre
centrele de greutate ale triunghiurilor determinate de trei dintre punctele date sunt necoliniare.
Gazeta Matematică 1/2010

CLASA A X-
X-A

1. a) ArătaŃi că nu există numere întregi x, pentru care log3(x + 2) = log2(2x + 7).


b) DeterminaŃi numerele naturale x şi y pentru care 3x = 2y + 7.
R.M.C.S. prelucrare
2. a) RezolvaŃi ecuaŃia sin(3arcsin x) + cos(2arcsin x) = 1.
b) Dacă α, β sunt rădăcinile ecuaŃiei z2 + 2z + 4 = 0, arătaŃi că numerele A = α4 + β 4 şi B = (α +
+ 2)5 + (β + 2)5 sunt raŃionale.
R.M.C.S. prelucrare
3. DeterminaŃi funcŃiile strict crescătoare f: R → R cu proprietatea că: f(xf(y)) = f(x)y, (∀) x, y ∈ R.
Prelucrare Gazeta Matematică

146
4. La un turneu de şah, oricare doi participanŃi joacă o singură partidă. După ce au jucat câte două
jocuri, cinci participanŃi părăsesc competiŃia. La finalul turneului s-a constatat că numărul total de
partide jucate este egal cu 200. CâŃi şahişti au participat la turneu?
RMCS (Olimpiadă Moldova 2007)

CLASA A XI-
XI-A

1. Se spune că o matrice A ∈ M2(R) are proprietatea (Pn) dacă există n ∈ N, n ≥ 3, pentru care An +
+ An–1 + An–2 = O2. ArătaŃi că:
a) există A ≠ O2, A ∈ M2(Z), care are proprietatea (P3);
b) dacă A ∈ M2(R) are proprietatea (P2010) şi se notează B = A2 + A + I2, atunci matricea I2 – AB
este inversabilă.
Prelucrare Gazeta Matematică 1/2010
2. Se consideră şirurile (an)n≥1, (bn)n≥1 şi (cn)n≥1 definite prin: a1 = 2, a n −1 = 1 + a n − a n −1 , n ≥ 2, bn =
an −1 n
1
=
a n +1 − 1
, n ≥ 1 şi c n = ∑
k =1 a k
.

a) ArătaŃi că an = 1 + a1a2 ... an–1, n ≥ 2.


b) CalculaŃi limita şirului (bn)n≥1.
c) DemonstraŃi că (cn)n≥1 este convergent.
Prelucrare Gazeta Matematică 5/2009
3. Pentru orice matrice A ∈ M3(R) se notează Tr(A) suma elementelor de pe diagonala principală.
a) StudiaŃi dacă există matrice A ∈ M3(R), A ≠ I3 pentru care |detA| = |TrA| = 1.
b) ArătaŃi că, dacă A ∈ M3(R), A ≠ O3, atunci A2 = O3 dacă şi numai dacă rangA = 1 şi Tr(A) = 0.
RMCS 24/2008
1
xe x
4. CalculaŃi: lim 2 .
x → 0 tg x
x >0

Gazeta Matematică, 8/1974

CLASA A XII-
XII-A

1. Fie G un grup comutativ cu proprietatea că funcŃia f: G → G, f(x) = x3 este injectivă. ArătaŃi că,
dacă a, b, c ∈ G şi ab–1 = bc–1 = ca–1, atunci a = b = c.
Prof. SteluŃa şi Mihai Monea, Deva
2. Fie G un grup multiplicativ cu 2n + 1 elemente. ArătaŃi că, dacă există o funcŃie f: G → G cu pro-
prietatea că f(xf(xy)) = yf(x2), (∀) x, y ∈ G, atunci grupul este comutativ.
Gazeta Matematică, 2009

147
3. Se consideră funcŃiile f, g: R → R continue şi, pentru orice primitive F, G ale funcŃiilor f,
respectiv g, se notează A = {x ∈ R | F(x) = G(x)}. Să se demonstreze că, dacă A = {0}, atunci
f (c ) − g ( c) 1
există c ∈ (0, 1) astfel încât = .
F(c) − G (c) 1 − c
Prof. Lucian Dragomir
x  1
4. Se consideră funcŃia f: [0, 2] → R, f ( x ) = . Dreapta de ecuaŃie y = m, m ∈ 0,  inter-
1+ x 2
 2
 1
sectează graficul funcŃiei considerate în punctele A şi B. Se defineşte funcŃia g: 0,  → R prin
 2
1
g(m) = AB (distanŃa dintre A şi B). CalculaŃi: ∫0
2 g ( m )dm .

Prof. Lucian Dragomir


Concursul de matematică „Petre Sergescu”, 26 martie 2010,
Drobeta-Turnu Severin
CLASA A IX-
IX-A
3
1. Să se arate că pentru orice numere reale a, b, c > 0 avem a + b + c + ≥ 43 abc .
1 1 1
+ +
a b c
Gazeta Matematică
2. Să se rezolve în Z ecuaŃia x2 + 1 = 3y.
RMM 5
3. Fie ABCD un patrulater convex. Pe laturile AB, BC, CD, DA considerăm punctele M, N, P şi
AM BN CP DQ
respectiv Q astfel încât = = = = k , k ∈ R*+. Notăm cu E, F, G, H respectiv
MB NC PD QA
mijloacele segmentelor [AC], [BD], [MP] şi [NQ]. Să se arate că:
a) E, F, G, H sunt coliniare; b) [FH] ≡ [EG].
(Se cere soluŃie vectorială.)
Prof. Daniel Stretcu şi Eduard Băzăvan

CLASA A X-
X-A

1. Să se determine funcŃia f: (0, ∞) → R cu proprietatea că oricare ar fi punctele A(a, f(a)), B(b,


f(b)) de pe graficul funcŃiei, dreapta AB intersectează axa Oy în punctul de ordonată a2b + ab2.
Gazeta Matematică
2. Să se rezolve ecuaŃiile:
1
2
a) x log 2 x + log 42 x − 2 log 22 x = 1 ; b) 4 tg + 2 cos + 1 = 4 cos x .
x 2
x

Prof. Iuliana Gimoiu, Dana Paponiu

148
3. Se consideră triunghiul ABC având BC = a, CA = b, AB = c şi fie zA, zB, zC afixele vârfurilor
sale. Să se arate că ∆ABC este echilateral dacă şi numai dacă are loc relaŃia:
ab(zA – zB) + bc(zB – zC) + ca(zC – zA) = 0.
Prof. Manuela Prajea

CLASA A XI-
XI-A

1. Fie k ∈ N, k ≥ 2. Se consideră şirul (an)n≥1 definit prin a1 ∈ (0, 1) şi a n +1 = a n (1 − a kn ) pentru orice


n ∈ N*.
a) Să se arate că şirul (an)n≥1 este convergent.
b) Să se calculeze lim k na kn .
n →∞

Prof. Leonard Mihai Giugiuc


2. Fie f: R → R o funcŃie crescătoare cu proprietatea că:
 1  1
lim f  x −  = lim f  x +  , (∀) x ∈ R. Să se arate că f este continuă.
n →∞  n n → ∞  n
Gazeta Matematică
3. Fie n ∈ N, n ≥ 2 şi matricea reală (aij)1≤i, j≤n.
a) Să se determine rangul matricii ştiind că elementele sale sunt nenule şi că produsul
a1σ(1) ⋅ a 2σ( 2) ⋅ ⋅ ... ⋅ a nσ( n ) este constant pentru orice permutare σ a numerelor 1, 2, ..., n.
b) Să se determine numărul matricelor de ordinul n având rangul unu şi elementele din mulŃimea
{0, 1, 2}.
Prof. Manuela Prajea

CLASA A XII-
XII-A

 x y  
1. Fie M =   x ∈ R , y ∈ R  .
 − y x  
a) Să se arate că (M, +, ⋅) este corp comutativ.
b) Să se arate că (M, +, ⋅) este izomorf cu corpul complex (C, +, ⋅).
c) Să se rezolve în M ecuaŃia: A4 + 3A3 + 2A2 + 3A + I2 = O2.
Prof. Daniel Sitaru
π
2 cos
2
nx − cos (n − 1) x
2
2. Să se calculeze: ∫π
3
sin x
dx .

Gazeta Matematică, 2009


3. Fie a ∈ R*. Să se determine funcŃiile derivabile, cu derivata continuă u: R → R, care verifică
x
egalitatea: | u ( x ) | = ∫ [u ( t ) + (u ' ( t )) 2 ]dt + a 2 , (∀) x ∈ R.
2
0
Prof.dr. George Căiniceanu

149
Concursul de matematică „Chindia”, 9 mai 2010, Târgovişte

CLASA A IX-
IX-A

1. Fie p ∈ R astfel încât ecuaŃia de gradul al doilea x2 – px + p = 0 are două rădăcini x1, x2 ∈ R.
1 5
DemonstraŃi că |x1 + x2 – 2| + ≥ .
| x 1x 2 − 2 | 2
Prof. Dinu Teodorescu
ax + b, x < 1
2. ArătaŃi că o condiŃie necesară şi suficientă ca o funcŃie de forma f: R → R, f(x)=  ,
cx + d, x ≥ 1
cu a, b, c, d ∈ R, să poată fi scrisă sub forma f(x) = mx + n + p|x – 1|, cu m, n, p ∈ R, este ca
a + b = c + d.
Prof. Cristinel Mortici
3. DemonstraŃi că dacă înălŃimile AD, BE, CF ale ∆ABC satisfac 4AD + 19BE + 25CF = 0 , atunci
∆ABC are un unghi cu măsura de 60°.
Prof. Cristinel Mortici

CLASA A X-
X-A

1. Fie x, y, z > 0 astfel încât 16xyz = 1. DemonstraŃi că:


1
log 22 (2 x + y) + log 22 ( 2 y + z) + log 22 (2z + x ) ≥ .
12
Prof. Dinu Teodorescu
2. Fie a, b, c ∈ R şi mulŃimile A = {z ∈ C – R | z + az ∈ R}; B = {z ∈ C – R | z + b z + cz ∈ R}.
2 2

DemonstraŃi că A = B dacă şi numai dacă a + b = c.


Prof. Cristinel Mortici şi Dinu Teodorescu
3. DemonstraŃi că pentru orice număr natural n ≥ 5, avem:
2 1 1 1 1 4
2 + ≤ 0 + 1 + ... + n −1 + n < 2 + şi
n Cn Cn Cn Cn n
2 1 1 1 1 1
2+ 2
≤ 0 2 + 1 2 + ... + n−1 2 + n 2 < 2 + .
n (C n ) (C n ) (C n ) (C n ) n
Prof. Cristinel Mortici

CLASA A XI-
XI-A

1. Fie A, B ∈ M2(C) astfel încât (AB)2 = O2. DemonstraŃi că ABA = O2 sau BAB = O2.
Prof. Cristinel Mortici

150
1 1
2. Pentru fiecare număr natural k ≥ 1, definim a k = lim n (3 nk +5 nk − 2) .
n →∞

CalculaŃi L = lim k (3a k + 5a k − 2) .


k →∞
Prof. Dinu Teodorescu
3. ArătaŃi că există o infinitate de funcŃii f: R → R astfel încât f(x) – ef(x) = x – ex, (∀) x ∈ R.
Câte dintre acestea sunt continue? Câte sunt continue şi monotone?
Prof. Cristinel Mortici

CLASA A XII-
XII-A

1. DeterminaŃi numerele naturale n, pentru care mulŃimea G = { a + b3 4 + c3 n , a, b, c ∈ Q} – {0}


este grup în raport cu operaŃia de înmulŃire a numerelor reale.
Prof. Cristinel Mortici
2. DeterminaŃi funcŃiile continue f: R → R care admit o primitivă F cu proprietatea că pentru orice
x ∈ R şi ε > 0, avem: f((x – ε, x + ε)) ∩ F((x – ε, x + ε)) ≠ ∅.
π
1 1 1  sin x 2 cos x 2  π 1 1 
3. Fie a, b > 0. DemonstraŃi că:  + <
2π  a b  ∫
0
2  
 a + b dx < 2  a 2 + b 2  .
 
Prof. Cristian Mortici şi Dinu Teodorescu

Concursul interjudeŃean de matematică – Memorialul „Traian Lalescu”,


EdiŃia a XXIV-a, 26-28 martie 2010, Timişoara
CLASA A IX-
IX-A

1. Fie A o mulŃime nevidă de numere naturale şi f: A → A o funcŃie diferită de funcŃia identică pe A


având proprietatea că: f(m) – f(n) = m – n, pentru orice m, n ∈ A.
a) ArătaŃi că A este infinită.
b) Rămâne adevărat rezultatul de la a) în cazul funcŃiei identice? JustificaŃi răspunsul.
2x  2x 
2. Să se rezolve în mulŃimea numerelor reale, ecuaŃia: = .
2 − x  x 2 − x + 1
MA NB PC
3. Fie ABC un triunghi şi M ∈ (AB), N ∈ (BC), P ∈ (AC) astfel încât = = .
MB NC PA
a) DemonstraŃi că triunghiurile ABC şi MNP au acelaşi centru de greutate.
b) DemonstraŃi că pentru orice punct S din plan, avem: SM2 + SN2 + SP2 ≤ SA2 + SB2 + SC2.
4. Despre un număr natural nenul vom zice că este liber de pătrate dacă el nu se divide cu pătratul
nici unui număr prim. Fie n un astfel de număr (adică liber de pătrate) şi An mulŃimea tuturor
numerelor z = a + b n , unde a şi b sunt numere raŃionale şi a2 – nb2 = 1. ArătaŃi că:
a) numărul 2010 este liber de pătrate;

151
b) pentru n ≠ 2010, mulŃimea An ∩ A2010 este finită;
c) mulŃimea A2010 este infinită.

CLASA A X-
X-A

1. Să se determine funcŃiile surjective f: N → N pentru care f(m) + f(n) = f(f(m + n)), ∀ m, n ∈ N.


2. Fie numerele x1, x2, ..., xn > 0 (n ≥ 2) cu ∑ x i x j = 1 . ArătaŃi că putem înlătura un număr astfel
1≤ i < j≤ n

încât suma numerelor rămase să fie mai mică decât 2 .


3. Fie a, b ∈ (1, ∞). Se consideră ecuaŃia:
2 2 2
a 8 x + 8 y + b8 y + 8 x = log a log b , x, y ∈ R.
a b
b a
a) Să se rezolve ecuaŃia în cazul a = b.
b) Să se studieze dacă ecuaŃia are soluŃii în cazul a ≠ b.
AM BN
4. Fie patrulaterul ABCD. Fie M ∈ (AD) şi N ∈ (BC) astfel încât + = 1.
AD BC
a) Să se arate că MN ⋅ BC ⋅ AD ≤ BN ⋅ AC ⋅ AD + AM ⋅ BD ⋅ BC.
b) Să se arate că mijloacele segmentelor [AC], [BD] şi [MN] sunt coliniare.

CLASA A XI-
XI-A

1. Se consideră şirul de numere reale definit prin: x1 > 0, x n +1 = x n + x n , (∀) x ∈ N*.


xn
Să se calculeze: lim x n ; lim n x n ; lim .
n →∞ n →∞ n →∞ n 2

n
2. Se consideră şirul (xn)n≥1 cu x1 = 2010 şi x n +1 = , (∀) n ∈ N*.
x1 + x 2 + ... + x n
ArătaŃi că şirul (xn) este convergent şi calculaŃi lim x n .
n →∞

3. Fie A, B ∈ Mn(R) matrici inversabile astfel încât A–1 + B–1 = In. Să se arate că:
a) In = (In – A)(In – B); b) det[In – A3 – B3 + (AB)3] ≥ 0.
 1 lg 2 lg 3 lg 5 
 
 lg 2 1 lg 5 lg 3 
4. Se consideră matricea: A =  , unde lg x notează logaritmul în baza 10 al
lg 3 lg 5 1 lg 2 
 
 lg 5 lg 3 lg 2 1 
 
lui x. Să se arate că detA < 1.

152
CLASA A XII-
XII-A

 π x π f (x)
1. Fie f: 0,  → R, f ( x ) = ∫ ln(1 + tg x ⋅ tg t )dt . Să se calculeze: a) f   ; b) lim 3 .
 2 0 4 x →0 x

2. Se consideră funcŃia f: (0, ∞) → R, f(x) = x + ln x. Să se arate că pentru orice x ∈ R, ecuaŃia f(t) =


= x are o soluŃie unică t = ϕ(x). Să se arate apoi că ϕ: R → (0, ∞), x → ϕ(x) este derivabilă şi să
1+ e dx
se calculeze: I =
1∫ 1 + ϕ( x )
.

3. Fie (A, +, ⋅) un inel şi f: A → R o funcŃie cu proprietatea că f(x + y) + f(x – y) = 2(f(x) + f(y)),


pentru orice x, y ∈ A.
a) Să se demonstreze că f(0) = 0 şi f(x) = f(–x), (∀) x ∈ A.
b) Să se demonstreze că pentru orice n ∈ N* şi x1, x2, ..., xn ∈ A, există a1, a2, ..., an ∈ {–1, 1}
astfel încât: f(a1x1 + a2x2 + ... + anxn) ≤ f(x1) + f(x2) + ... + f(xn).
1
4. a) Fie G un grup finit şi A o submulŃime a lui G astfel încât card(A) > card(G). DemonstraŃi că
2
pentru orice g ∈ G există a1, a2 ∈ A, astfel încât g = a1a2.
b) Fie K un corp finit. DemonstraŃi că pentru orice x ∈ K, există u, v ∈ K astfel încât x = u2 + v2.

Concursul interjudeŃean de matematică „SperanŃe râmnicene”,


4 aprilie 2010, Râmnicu Sărat

CLASA A IX-
IX-A

 x x − y y = 63

1. Să se rezolve sistemul  .
 x x x + y y y = 129

Gazeta Matematică
2. Se consideră poligonul regulat P1P2...Pn, cu laturile de lungimi a. Fie M un punct arbitrar din
interiorul poligonului şi d1, d2, ..., dn distanŃele de la acest punct la laturile P1P2, P2P3, ..., PnP1. Să
se determine minimul sumei d12 + d 22 + ... + d 2n şi punctul în care se atinge acest minim.
Prof. Constantin Rusu, Râmnicu Sărat
3. Se dă ecuaŃia x 2 − 2 a 2 + b 2 + c 2 ⋅ x + ab + ac + bc = 0 , unde a, b, c ∈ R. Să se arate că ecuaŃia are
rădăcini reale, oricare ar fi numerele reale a, b, c.
Dacă ab + ac + bc < 0 şi 1 + ab + ac + bc > 2 a 2 + b 2 + c 2 , atunci ecuaŃia are o rădăcină reală
situată în intervalul (0, 1).
Prof. Mihai Neagu, Râmnicu Sărat

153
CLASA A X-
X-A

1. Considerăm numerele complexe distincte z1, z2, z3, z4 cu proprietatea că |z – zk| = |zk|, pentru orice
k = 1, 4 , unde z = z1 + z2 + z3 + z4. Să se arate că:
a) z = 0; b) dacă |z1| = |z2| = |z3| = |z4|, atunci z1, z2, z3, z4 reprezintă vârfurile unui dreptunghi.
Gazeta Matematică
2. Să se arate că log3 4 < log2 3. Generalizare.
Prof. Costică Ambrinoc, Râmnicu Sărat
2 1
3. RezolvaŃi ecuaŃia [ x ] + = .
x [x]
Prof. Gheorghe GhiŃă, Buzău

CLASA A XI-
XI-A

 1  1
1. Fie f: R → R o funcŃie crescătoare cu proprietatea că: lim f  x −  = lim f  x +  , (∀) x ∈ R.
n →∞ 
n →∞ n  n
Să se arate că f este continuă.
Gazeta Matematică
2. ArătaŃi că ecuaŃia: (1 + c + (log a x ) ) ⋅ log a x + bx (1 + c + b x ) = 0 are o singură soluŃie în
2 2 2

intervalul (0, 1), oricare ar fi a > 1, b > 0, c > 0.


Prof. Neculai Stanciu, Buzău
3. Fie A, B ∈ Mn(R) astfel încât AB = BA. Dacă există m, p ∈ N* – {1} astfel încât Am = Bp = On,
atunci det(In + (A + B) + (A + B)2 + ... + (A + B)m+p–2) ∈ R*.
Prof. D.M. BătineŃu, Bucureşti

CLASA A XII-
XII-A

1 x n −1 n
1 1 1 
1. Fie I n = ∫1
( x + 1) n
dx , n ≥ 1. Să se calculeze: a) lim nI n ;
n →∞
b) lim
n →∞
∑ k  2
k =1
k
− .
3k 
2
Gazeta Matematică
x5 + x
2. CalculaŃi ∫ x 3 + 1 dx , x ∈ (0, ∞).
Prof. Dragoş Lăzărescu, Râmnicu Sărat
3. Fie f ∈ C[X] un polinom de grad n ≥ 1, având toate rădăcinile nenule şi distincte x1, x2, ..., xn ∈ C.
Presupunem că există λ ∈ C – {0, 1} astfel încât {x1, x2, ..., xn} = {λx1, λx2, ..., λxn}. Să se arate că:
a) λn = 1; b) dacă n este minim cu proprietatea a), atunci există a, b ∈ C* astfel încât f = aXn + b.
Prof. Marcel łena, Bucureşti

154
Concursul naŃional de matematică aplicată „Adolf Haimovici”,
Etapa judeŃeană, 13 martie 2010
Filiera tehnologică: profilul servicii, resurse naturale şi protecŃia mediului

CLASA A IX-
IX-A

1. Un elev doreşte să cumpere 33 ciocolate dintr-un magazin, unde acestea sunt ambalate în cutii de
câte 6, 9 şi respectiv 20 bucăŃi, fără ca acestea să poată fi vândute la bucată.
a) În câte moduri poate elevul să realizeze acest lucru?
b) În aceleaşi condiŃii, poate elevul să cumpere 43 ciocolate?
2. Numerele reale pozitive a1, a2, a3, ..., a10 sunt, în această ordine, în progresie aritmetică de raŃie
r > 0 şi pentru care r ⋅ a1 = 18. DeterminaŃi termenii progresiei astfel încât suma termenilor
acesteia să fie minimă.
x − 2 y = a

3. Fie a, b, c ∈ R astfel încât a + b + c = 1 şi sistemul de ecuaŃii:  y − 2z = b . Să se arate că
2 2 2

z − 2 x = c

pentru orice soluŃie (x, y, z) a sistemului avem x + y + z ≤ 1.
2 2 2

4. Considerăm triunghiul ABC, având laturile AB = c şi AC = b. În planul acestuia se consideră


punctele M, N, D astfel încât: AM = b ⋅ AB, AN = c ⋅ AC, AD = b ⋅ AB + c ⋅ AC . DemonstraŃi că:
a) (AM) ≡ (AN);
b) patrulaterul AMDN este paralelogram;
c) (AD este bisectoarea unghiului 'BAC.

CLASA A X-A

1. Avem la dispoziŃie un număr nelimitat de jetoane pe care sunt scrise numerele 5, 7 sau 11. Un
număr n ∈ N* se numeşte norocos dacă găsim un număr de jetoane astfel încât suma numerelor
scrise pe ele să fie egală cu n.
a) DemonstraŃi că numărul 13 nu este norocos.
b) ArătaŃi că numerele 14, 15, 16, 17 şi 18 sunt norocoase.
c) DemonstraŃi că orice număr natural n ≥ 14 este norocos.

155
2. Fie a, b, c, A ∈ (0, ∞) – {1} şi α, β, γ ∈ R* astfel încât: (ab)α = (bc)β = (ac)γ = A2.
1 1 1 1 1 1
DemonstraŃi că: + + = + + .
log a A log b A log c A α β γ
3. Se dă funcŃia f: D ⊂ R → R, f ( x ) = log 3 −1
(7 − 2 x − x ) .
a) ArătaŃi că domeniul de definiŃie D = [0,9 − 4 2 ) .
b) GăsiŃi punctele de coordonate întregi situate pe graficul funcŃiei f.
4. a) DemonstraŃi că: 1 + sin x + 1 − sin x ≤ 2 , (∀) x ∈ R.
b) RezolvaŃi în R ecuaŃia: 1 + sin x + 1 − sin x = 2 x + 2 − x .

CLASA A XI-
XI-A

1. O matrice A ∈ M2(R) verifică condiŃiile: det(A – 3I2) = 4 şi det(A + 2I2) = 9, I2 fiind matricea
unitate de ordinul al doilea. DemonstraŃi că A2 = 2A – I2.
2. Un determinant de ordin n ≥ 2 are (n2 – n + 2) elemente egale. DemonstraŃi că determinantul este
nul.
1
3. a) Se dă funcŃia f: R – {0, 1} → R, f ( x ) = 1 . DeterminaŃi limitele laterale ale funcŃiei f în
2x − 2
punctele x0 = 0 şi x1 = 1.
b) CalculaŃi: l1 = lim e x ⋅ [e − x ] şi l 2 = lim e − x ⋅ [e x ] , unde [a] reprezintă partea întreagă a număru-
x →∞ x →∞
lui real a, iar e este baza logaritmului natural.
4. ArătaŃi că nu există polinoame P, Q, cu coeficienŃi reali, P(x) = a0xn + a1xn–1 + ... + an, a0 ≠ 0 şi
Q(x) = b0xm + b1xm–1 + ... + bm, b0 ≠ 0 şi m, n ∈ N*, astfel încât P( x ) = x x 2 + 1 ⋅ Q( x ) , (∀) x ∈ R.

CLASA A XII-
XII-A

1. Pe mulŃimea G = (–1, ∞) se defineşte legea de compoziŃie internă dată prin x & y = x + y + xy,
(∀) x, y ∈ G.
a) DemonstraŃi că (G, &) este grup abelian.
b) RezolvaŃi în G ecuaŃia x∗ x ∗
x ∗ ... ∗
x = 1 , n ∈ N*, n ≥ 2.
n ori

c) ArătaŃi că mulŃimea H = {a2 – 1 | a ∈ Q*} este subgrup al grupului (G, &).


1
1 1 dt dt
2. a) Folosind substituŃia t =
u
, să se demonstreze că
x 1 + t2 ∫
= x
1 1 + t2 ∫, (∀) x > 0.

1 π a arctgx
b) ArătaŃi că: arctgx + arctg = , (∀) x > 0. c) CalculaŃi: I(a ) = 1
x 2 a
x ∫
dx , a > 1.

156
3. CalculaŃi:
2 4 x 3 − 6 x 2 + 8x − 3
a) I = ∫
0 ( x 2 − x + 1) 3
dx ;
1
 1  x+
b) I(x) = ∫  1 + x −  e x dx , unde e este baza logaritmului matural, iar x ∈ (0, ∞).
 x
4. Un elev colorează puncte de coordonate întregi ale planului, raportat la reperul ortogonal (xOy).
Fiind colorate două puncte A şi B, elevul poate colora simetricul lui A faŃă de B şi simetricul lui
B faŃă de A. ArătaŃi că dacă iniŃial, în plan, erau colorate punctele O(0, 0); A(1, 0); B(1, 1) şi
C(0, 1), atunci elevul poate colora toate punctele de coordonate întregi ale planului.

Concursul NaŃional de Matematică aplicată „Adolf Haimovici”,


Etapa naŃională, 22 mai 2010

Filiera tehnologică: profilul servicii, resurse naturale şi protecŃia mediului

CLASA A IX-
IX-A

1. Pe o dreaptă se consideră 2010 puncte, care formează 2009 segmente, fiecare de 1 cm. Pe primul
segment, deasupra dreptei, construim un pătrat, pe al doilea segment, dedesubtul dreptei,
construim un triunghi echilateral, apoi trasăm segmentul de lungime 1 cm. Pe al patrulea segment
construim un triunghi echilateral, deasupra dreptei, pe următorul segment construim un pătrat,
dedesubtul dreptei şi din nou trasăm următorul segment de lungime 1 cm (ca în desenul de mai
jos). Repetăm procedeul până ajungem la ultimul punct. CalculaŃi lungimea liniei frânte obŃinute
de la primul până la ultimul punct.

an 2n 1
2. Considerăm şirul (an)n∈N*, a1 = 3 şi a n +1 = + + , n ≥ 1.
3 3 3 3
a) ArătaŃi că şirul (bn)n≥1, definit prin b n = a n 3 − n , este o progresie geometrică.
n −1
n 2 1
b) ArătaŃi că a n = + ⋅  , n ∈ N*.
3 3 3
n (n + 1)
c) Notăm Sn = a1 + a2 + ... + an. DemonstraŃi că Sn ⋅ 3 − < 3 , (∀) n ∈ N.
2
3. În triunghiul ABC cu BC = a şi înălŃimea AD = h, D ∈ [BC] se înscrie dreptunghiul MNPQ, M ∈
∈ [AB], N ∈ [AC], P, Q ∈ [BC]. Notăm MN = x şi NP = y.
a) DemonstraŃi că are loc egalitatea: hx + ay = ah.

157
b) ArătaŃi că aria dreptunghiului MNPQ este cel mult jumătate din aria triunghiului ABC.
c) Considerăm o bucată de carton în formă de triunghi cu laturile egale cu 30 cm, 40 cm şi 50 cm.
DeterminaŃi dimensiunile dreptunghiurilor de arie maximă care pot fi decupate din acest carton.
3 x 2 − 2x + 2
4. Fie funcŃia f : Q –   → R, f ( x ) = . DeterminaŃi punctele situate pe graficul funcŃiei
2 2x − 3
f care au ordonata un număr întreg.

CLASA
CLASA A X-
X-A

1. RezolvaŃi ecuaŃiile:
x+2 3− x 5
a) log4 (x + 12) ⋅ logx 2 = 1; b) + = .
3− x x+2 2
2. a) ArătaŃi că funcŃia f: R → R, f(x) = 2x + x este strict monotonă.
b) Fie a, b ∈ R astfel încât 2a – 2b = b – a. ArătaŃi că a = b.
2 2
c) RezolvaŃi ecuaŃia 2sin x − 2cos x = cos 2 x , x ∈ R.
3. O capră este legată cu un lanŃ având lungimea de 6 m, lanŃ care este fixat cu unul din capete în
punctul A. AG este un gard cu lungimea de 4 m pe care capra nu poate să-l sară. Pătratul ABCD
este un Ńarc cu latura de 2 m, în care capra nu poate intra (vezi figura). AflaŃi suprafaŃa maximă de
iarbă pe care capra poate paşte (gardul şi laturile Ńarcului au lăŃimi neglijabile).
B C
K* * *
6m

* * *
G 4m A 2m D

4. Un număr natural se numeşte bun dacă el poate fi scris ca suma a două numere naturale conse-
cutive, cât şi ca suma a trei numere naturale consecutive. DemonstraŃi că:
a) 2010 nu este bun, dar 2013 este bun;
b) produsul a două numere bune este bun;
c) dacă produsul a două numere este bun, atunci cel puŃin unul dintre ele este bun.

CLASA A XI-
XI-A

x y z
1. a) Fie x, y, z ∈ R. ArătaŃi că: z x y = ( x + y + z)( x 2 + y 2 + z 2 − xy − yz − xz ) .
y z x
b) Fie matricea A ∈ M3(R) având toate elementele pozitive cu proprietatea că elementele de pe
diagonala principală sunt egale între ele, iar produsul elementelor de pe fiecare linie şi de pe
fiecare coloană este egal cu 1. ArătaŃi că detA ≥ 0.
158
2. Spunem că o matrice are proprietatea (p) dacă toate elementele sale sunt egale cu –1 sau 1, iar
produsul elementelor de pe fiecare linie şi de pe fiecare coloană este egal cu –1.
a) DaŃi un exemplu de matrice A ∈ M3(Z) cu proprietatea (p).
b) DemonstraŃi că nu există nicio matrice B ∈ M2,3(Z) cu proprietatea (p).
16
3. a) DeterminaŃi intervalele de monotonie ale funcŃiei f: R → R, f ( x ) = x 2 + .
x
b) Se construieşte un rezervor din sticlă în formă de paralelipiped dreptunghic cu baza pătrat şi
fără capac, având volumul egal cu 4 m3. DeterminaŃi dimensiunile paralelipipedului astfel încât
suprafaŃa de sticlă utilizată să fie minimă.
a − 2 2 − x
 , x <1
 x −1
4. DeterminaŃi a, b, c ∈ R, astfel încât funcŃia f: R → R, f ( x ) = 1, x = 1 să fie con-
bx + c log(x − 1), x > 1


tinuă pe mulŃimea R.

CLASA A XII-
XII-A

1. a) Pe mulŃimea R definim legea de compoziŃie &: R × R → R prin x & y = xy – x – y + 2. ArătaŃi


că legea & este comutativă şi asociativă.
b) Pe tablă sunt scrise numerele –2010, –2009. ..., 0, 1, ..., 2010. Se şterg două numere a şi b şi, în
locul lor, se scrie numărul ab – a – b + 2. Se continuă acest procedeu până când pe tablă rămâne
un singur număr. Care este acest număr?
a dx
2. CalculaŃi: I = lim ∫
a → ∞ 0 ( x + 1)( 2 x 2 + 1)
.

3. Considerăm mulŃimea F = {f: [0, 1] → [0, 1], f continuă, f(0) = 0 şi f(1) = 1}.
0, x ∈ [0, a ]
x − a

a) Fie a, b ∈ [0, 1], a < b şi funcŃia f: [0, 1] → [0, 1], f ( x ) =  , x ∈ (a , b) . ArătaŃi că f ∈ F şi
 b − a
1, x ∈ [b,1]
1
calculaŃi ∫ f (x)dx .
0
1 1
b) GăsiŃi două funcŃii f1, f2 ∈ F, f1 ≠ f2, astfel încât ∫
0
f1 ( x )dx = ∫ f (x )dx .
0
2

1 1
c) GăsiŃi o funcŃie f ∈ F astfel încât ∫ f (x)dx = 2010 .
0

4. La un turneu de şah au participat 20 de elevi. Fiecare a jucat câte o partidă cu fiecare. După termi-
narea turneului s-a văzut că exact un elev s-a clasat pe locul al 19-lea, obŃinând 9,5 puncte.
a) Câte partide s-au jucat la acel turneu în total?

159
b) ArătaŃi că ultimul elev are cel mult 0,5 puncte.
c) Poate elevul clasat pe primul loc să-l depăşească cu un punct pe al doilea clasat?
(Se acordă 1p pentru victorie, 0,5p pentru meci egal şi 0p pentru înfrângere.)

Concursul NaŃional de Matematică aplicată „Adolf Haimovici”,


Etapa judeŃeană, 13 martie 2010

Profil real, specializarea ştiinŃele naturii

CLASA A IX-
IX-A

1
1. Fie N = ab un număr natural de două cifre (în baza 10). DeterminaŃi N, dacă numerele a, b şi ⋅
3
N sunt în progresie geometrică.
2. Se consideră triunghiul ABC cu m('A) = 90° şi m('C) = 30°. Fie D ∈ (BC) şi P ∈ (AB) astfel
BD 1 AP 3
încât = , = , iar E este piciorul bisectoarei din B. DemonstraŃi că dreptele CP, AD şi
DC 3 PB 2
BE sunt concurente.
Gazeta Matematică 7-8-9/2009
3. Doi brazi, unul cu înălŃimea de 28 m, iar celălalt cu înălŃimea de 15 m, se află unul faŃă de altul la
distanŃa de 13 m. Un iepure este situat la 35 m faŃă de vârful bradului mai înalt şi la 25 m faŃă de
vârful bradului mai scund. La ce distanŃă se află iepurele faŃă de dreapta care uneşte bazele celor
doi brazi?
4. Pe tablă este scris de douăzeci de ori numărul zecimal 1,1 şi de douăzeci numărul zecimal 1,11.
Lucică cel obraznic a şters câteva numere dintre cele patruzeci aflate pe tablă. StabiliŃi câte
numere a şters Lucică, ştiind că suma numerelor rămase pe tablă este 19,93.

CLASA A X-
X-A

1 1
1. Considerăm expresia E( x ) = − .
x − 1− x x + 1− x
a) DeterminaŃi valorile reale ale lui x pentru care este bine definită expresia E(x).
2 1− x  1 1 
b) DemonstraŃi că E ( x ) = , (∀) x ∈ 0,  ∪  ,1 .
2x − 1  2  2 
c) RezolvaŃi inecuaŃia E(x) ≤ 0.
2. CalculaŃi suma S = lg(tg 1°) + lg(tg 2°) + lg(tg 3°) + ... + lg(tg 89°).
3. a) DaŃi un exemplu de trei numere complexe nenule z1, z2, z3, astfel încât: z12 + z 22 + z 32 = 0 .
b) Fie w1, w2, w3 ∈ C astfel încât w1 + w2 + w3 ≠ 0, w12 + w 22 + w 32 = 0 şi |w1| + |w2| + |w3| = 1.
DemonstraŃi că |w1 + w2 + w3| = 2.

160
4. Un disc este împărŃit în şase părŃi egale prin trei diametre. În fiecare dintre sectoarele formate se
află câte un pion. La o mutare, alegem doi pioni, pe care îi deplasăm în sectoare vecine celor din
care pleacă. Există un şir finit de mutări în urma cărora toŃi pionii să ajungă într-un acelaşi sector?
JustificaŃi răspunsul.
Gazeta Matematică, 10/2009

CLASA A XI-
XI-A

a b
1. Se consideră matricea A =   ∈ M2(Z) şi funcŃia f: M2,1(Z) → M2,1(Z), f(X) = A ⋅ X, (∀) X =
c d
x 
=  1  ∈ M2,1(Z).
 x2 
a) Dacă detA ≠ 0, demonstraŃi că funcŃia este injectivă.
b) Dacă detA ∈ {–1, 1}, demonstraŃi că funcŃia este bijectivă.
2. Fie A ∈ M3(C) o matrice, At transpusa sa, iar B = A – At ∈ M3(C).
a) ArătaŃi că B = –Bt.
b) DemonstraŃi că detB = 0.
3. Se consideră şirul (Tn)n∈N de triunghiuri dreptunghice isoscele din figura de mai jos. Notăm cu si
şi pi aria, respectiv perimetrul triunghiului Ti, i ∈ N. Definim şirurile (Sn)n∈N şi (Pn)n∈N prin
n n
Sn = ∑s
i =0
i , respectiv Pn = ∑ p . AflaŃi limitele şirurilor (S )
i=0
i n n∈N şi (Pn)n∈N.

T3 T4
T0 T1 T2 .......
1m 1 1 1 1
m m m m
2 4 8 16
1 1 1 n
4. Se consideră şirul (an)n∈N* definit prin a n = + + ... + − , (∀) n ∈ N*.
1 2 n 2
a) DemonstraŃi că şirul (an)n∈N* este strict crescător.
b) ArătaŃi că există n ∈ N* pentru care an > 2010.
c) Este şirul considerat convergent? JustificaŃi răspunsul!
Gazeta Matematică, 12/2009

CLASA A XII-
XII-A

1  1  1   1 
1. Se consideră funcŃiile f, F: (0, ∞) → R, f ( x ) = 2
ln1 +  iar F( x ) = 1 +  1 − ln1 +  .
x  x  x   x 
a) DemonstraŃi că F este primitivă a funcŃiei f.

161
e
b) CalculaŃi ∫ f (x)F(x)dx .
1

2. Fie (G1, &) şi (G2, ) două grupuri abeliene. Pe mulŃimea produs cartezian G = G1 × G2, definim
operaŃia pe componente „⋅” prin: (x1, y1) ⋅ (x2, y2) = (x1 & x2, y1  y2), (∀) (x1, y1), (x2, y2) ∈ G.
a) DemonstraŃi că (G, ⋅) este grup abelian.
b) Dacă (G1, &) = (G2, ) = (Z2, +), stabiliŃi dacă grupurile (Z4, +) şi (Z2 × Z2, +) sunt sau nu
izomorfe.
3. Întrucât operaŃia de scădere pe Z nu este asociativă, nu are element neutru şi nu este comutativă,
vom spune că este de tip (A, E, C) . Dacă o altă operaŃie, definită pe o mulŃime M, ar fi asociativă,
cu element neutru şi necomutativă, am spune că este de tip (A, E, C) .
DaŃi exemple de operaŃii, pe mulŃimi alese corespunzător, care să fie de tip (A, E, C), (A, E, C) ,
(A, E, C) , (A, E, C) , (A, E, C), (A, E, C), (A, E, C) . În fiecare dintre cele şapte situaŃii, aduceŃi o
minimă argumentare în sprijinul afirmaŃiei făcute.
9 ln x
4. CalculaŃi
1∫ x ( x + 3)
dx .

Gazeta Matematică, 11/2009

Concursul NaŃional de Matematică aplicată, „Adolf Haimovici”,


Etapa finală, 22 mai 2010

Profil real, specializarea ştiinŃele naturii

CLASA A IX-
IX-A

1. Într-un zid având forma unui dreptunghi ABCD cu lungimea AB = 5 m şi înălŃimea AD = 6 m se


sparge o fereastră dreptunghiulară MNPQ, cu MN || AB, MQ || AD, astfel încât d(MN, AB) = 2 ⋅
⋅ d(NP, BC) = 2 ⋅ d(PQ, CD) = 2 ⋅ d(MQ, AD) (prin d(XY, UV) am notat distanŃa dintre dreptele
XY şi UV). Ştiind că suprafaŃa ferestrei este de 3 m2, determinaŃi dimensiunile acesteia.
2. Solubilitatea în apă a unei substanŃe în raport cu temperatura este dată de legea S(t) = at2 + bt + c,
unde a, b, c ∈ (0, ∞), S este cantitatea (în grame) de substanŃă care se poate dizolva în 1000 g de
apă, iar t ∈ [10, 60] este temperatura (în grade Celsius) la care se produce dizolvarea. Experi-
mental, s-au determinat valorile solubilităŃii la câteva temperaturi, anume S1 = 10, S2 = 15, S3 =
= 25, pentru temperaturile t1 = 20, t2 = 25, respectiv t3 = 30.
a) DeterminaŃi cantitatea de substanŃă care se poate dizolva în 1000 g de apă la temperatura de 50°.
b) ArătaŃi că există două temperaturi diferite la care solubilitatea substanŃei este aceeaşi.
3. Un elev scrie pe tablă 10 numere naturale consecutive, iar un coleg şterge unul dintre ele. Care
sunt numerele scrise de elev, dacă suma numerelor rămase este 2010? Ce număr a şters colegul?

162
4. Se consideră în plan patrulaterul ABCD.
a) DeterminaŃi punctul M din plan pentru care suma MA + MB + MC + MD este minimă.
b) DeterminaŃi punctul N din plan pentru care suma NA2 + NB2 + NC2 + ND2 este minimă.
c) DeterminaŃi punctul P din plan pentru care modulul vectorului PA + PB + PC + PD este minim.

CLASA A X-
X-A

1. DemonstraŃi că, pentru orice număr complex de modul 1, are loc inegalitatea |2 + iz| ≤ 3. Când se
atinge egalitatea?
2. Spunem că un grup format din şase persoane este aproape unit dacă orice persoană din grup are
exact doi cunoscuŃi în cadrul grupului. (Presupunem că, dacă A îl cunoaşte pe B, atunci şi B îl
cunoaşte pe A.)
a) ArătaŃi că există grupuri de 6 persoane aproape unite care pot fi împărŃite în două grupuri de câte
trei persoane, astfel încât în cadrul fiecărui grup de trei, toate persoanele să se cunoască între ele.
b) ArătaŃi că există grupuri de 6 persoane aproape unite care pot fi împărŃite în două grupuri de
câte trei persoane, astfel încât în cadrul fiecărui grup de trei să nu existe perechi de cunoscuŃi.
3. Pe tablă sunt scrise numerele 2, 0, 1 şi 0. La fiecare pas se măreşte cu 2 cel mai mic dintre nume-
rele aflate pe tablă. (Dacă, la un moment dat, pe tablă sunt mai multe numere egale, se măreşte cu
2 unul dintre ele, la întâmplare). După câŃi paşi apare scris pentru prima dată pe tablă numărul
2010?
4. Pentru fiecare număr real a, considerăm funcŃia fa: R → R, fa(x) = ax + 2 – a. Notăm cu Ma sime-
tricul originii faŃă de graficul funcŃiei fa.
a) ArătaŃi că graficele tuturor funcŃiilor fa au un punct comun.
b) DemonstraŃi că lungimea segmentului MaMb este cel mult egală cu 2 5 , oricare ar fi numerele
reale a şi b.

CLASA A XI-
XI-A

 − 10 * − 7 
 
1. Unui elev i se scrie pe tablă matricea A =  * − 2 *  . Profesorul îi cere să înlocuiască
 * * * 

astericsurile cu numere întregi astfel încât, după completare, sumele tuturor numerelor de pe
fiecare linie, fiecare coloană şi de pe cele două diagonale să fie egale. Este posibil acest lucru? În
caz afirmativ, ce matrice obŃine elevul?
2. Se consideră funcŃia f: R → R, f(x) = x3 – 6x2 + 9x + a, unde a ∈ Z. Graficul funcŃiei f intersec-
tează axa Ox în trei puncte distincte.
a) DeterminaŃi valorile posibile ale lui a.
b) Abscisele celor trei puncte de intersecŃie ale graficului cu Ox pot fi toate numere întregi?
3. Se consideră funcŃia f: M2(R) → R, f(A) = det(A2 + I2), (∀) A ∈ M2(R). DemonstraŃi că funcŃia f
nu este nici injectivă, nici surjectivă.

163
4. Două drumuri rectilinii şi perpendiculare se întâlnesc în O. Din O pleacă simultan două mobile A
şi B, fiecare pe câte unul dintre drumuri, cu viteze constante v1 = 80 km/h, respectiv v2 = 100
km/h. După două ore, ambele mobile îşi schimbă sensul de mers, îndreptându-se spre O pe
aceleaşi drumuri (şi eventual depăşind punctul O), A cu viteza v2 iar B cu viteza v1, şi merg astfel
timp de încă două ore. În ce moment (ulterior schimbării sensului de mers) distanŃa dintre cele
două mobile este minimă?

CLASA A XII-
XII-A

1. Fie polinoamele g, h ∈ Z10[X], g = X18 – 1̂şi h = (X − 1̂)(X − 2̂)(X − 3̂) ⋅ ... ⋅ (X − 18) .
a) DemonstraŃi că b̂18 = 1̂ , (∀) b ∈ Z*19.
b) ArătaŃi că g = h.
c) CalculaŃi sumele ∑ î ⋅ ĵ şi ∑ î ⋅ ĵ ⋅ k̂ .
1≤ i < j≤18 1≤ i < j< k ≤18
1
2. Se dă funcŃia β: N* × N* → R, β(p, q) = ∫ t p −1 ⋅ (1 − t )q −1 dt .
0

a) CalculaŃi β(2, 2) şi β(p, 1).


b) Folosind schimbarea de variabilă t = 1 – u, demonstraŃi că β(p, q) = β(q, p), (∀) p, q ∈ N*.
q −1
c) DeduceŃi relaŃia de recurenŃă β(p, q) = ⋅ β(p + 1, q − 1) , (∀) p, q ∈ N*, q ≥ 2.
p
3. Într-un vas de cultură sunt la momentul t = 0, b0 bacterii. S-a observat că funcŃia n: [0, ∞) →
→ (0, ∞) definită prin n(t) este numărul bacteriilor din vas la momentul t, satisface relaŃia n'(t) =
= k ⋅ n(t), (∀) t ≥ 0 (n' este derivata funcŃiei n, iar k este o constantă pozitivă, care depinde de
mediul de cultură ales). ArătaŃi că, dacă iniŃial sunt b0 = 300 bacterii, iar k = 0,1, atunci, pentru
t ≥ 20, numărul bacteriilor din vas este mai mare decât 2010.
4. Din trei scânduri egale cu lăŃimea a (cm) se face un jgheab a cărei secŃiune are forma unui trapez
isoscel ABCD, une AB + BC + CD = 3a (vezi figura de mai jos). La ce valoare a unghiului
'BAD, suprafaŃa secŃiunii jgheabului este maximă?

A D

a
a
B C

164
Concursul NaŃional de Matematică aplicată „Adolf Haimovici”,
Etapa finală, 22 mai 2010

Filiera tehnologică: profil tehnic

CLASA A IX-
IX-A

1. Se consideră funcŃia f: R → R, f(x) = 4x – x2.


a) DeterminaŃi coordonatele punctelor în care graficul taie axa Ox.
b) DeterminaŃi coordonatele vârfului parabolei (P) asociate funcŃiei.
c) Notând cu S aria mulŃimii din plan delimitate de parabola (P) şi axa Ox, arătaŃi că 8 < S < 16.
2. a) DeterminaŃi cel mai mare număr întreg k, pentru care 2x2 – 3x + 1 ≥ k, (∀) x ∈ R.
b) GăsiŃi numerele întregi x, pentru care (2x2 – 3x + 1)2 ≤ 14x2 – 21x + 1.
3. Nicu are plantaŃi în livadă n pruni, numerotaŃi distinct cu numerele 1, 2, 3, ..., n. Într-o zi, Nicu se
apucă de cules prune respectând următoarea regulă: din prunul cu numărul 1 culege două prune,
din prunul cu numărul 2, culege cinci prune, din prunul cu numărul 3, culege 8 prune, şi aşa mai
departe, culegând cu trei prune mai mult decât din pomul precedent.
a) Ce număr de prune a cules din copacul cu numărul zece?
b) CâŃi pruni ar trebui să aibă pantaŃi Nicu, pentru a fi sigur că, respectând regula indicată, la
sfârşitul zilei are culese cel puŃin 2010 prune?
4. Pe fiecare dintre laturile (AB), (BC) şi (CA) ale unui triunghi ABC se consideră câte trei puncte,
două colorate cu roşu şi unul cu albastru.
a) CalculaŃi câte triunghiuri au vârfurile printre cele nouă considerate.
b) DeterminaŃi câte dintre acestea au vârfurile colorate cu aceeaşi culoare.

CLASA A X-
X-A

1. a) RezolvaŃi în mulŃimea numerelor complexe ecuaŃia z2 – 2cos t ⋅ z + 1 = 0, t ∈ [0, 2π).


1 1 1
b) Dacă z ∈ C*, astfel încât z + = 2 , calculaŃi z 4 + 4 şi z 8 + 8 .
z z z
1 1
c) CalculaŃi z1024 + 1024 ştiind că z + = 2 .
z z
2. a) RezolvaŃi în mulŃimea numerelor reale ecuaŃia: 3
2 ⋅ 3 x − 7 + 3 3 x − 2 = 3 2 ⋅ 3x − 2 + 3 3 x − 7 ,
3 3 3
utilizând, eventual, formula (a + b) = a + b + 3ab(a + b).
n
 1
b) GăsiŃi n şi termenul care conŃine pe x–1 din dezvoltarea  x 2 −  , n ∈ N*, ştiind că suma
 x
coeficienŃilor primilor trei termeni este 36.
3. Într-un sistem de axe de coordonate se consideră punctele A(3, 0), B(0, 2), M(3, –3) şi N(–2, 2).
a) GăsiŃi ecuaŃia dreptei MB.
165
b) ArătaŃi că AN, BM şi perpendiculara din O pe AB sunt concurente.
4. Dintr-o urnă care conŃine mai mult de 10 bile colorate în verde sau roşu, un elev extrage bile până
când constată că, pentru prima dată, numărul de bile verzi extrase este egal cu numărul de bile
roşii extrase. Elevul constată că acest lucru este îndeplinit după a zecea bilă extrasă şi că nu există
trei bile de aceeaşi culoare extrase consecutiv. DemonstraŃi că:
a) dacă ultima bilă extrasă este verde, atunci şi penultima este tot verde, iar antepenultima este roşie;
b) a cincea şi a şasea bilă extrase au culori diferite.

CLASA A XI-
XI-A

1. Fie M = {A ∈ M2(R) | detA ∈ {–1, 1}}.


a) ArătaŃi că M conŃine măcar 2 elemente.
b) JustificaŃi faptul că în M există cel puŃin 2010 elemente.
c) DemonstraŃi că, dacă A, B ∈ M, atunci A ⋅ B ∈ M.
x 2 + 2x + a , x ∈ (−∞,1)

2. Fie f: R → R, f ( x ) = b, x =1 , unde a, b, c ∈ R.
 2
x + cx, x ∈ (1, ∞)
a) ArătaŃi că există numere a, b, c, astfel încât funcŃia de mai sus să fie continuă.
b) Să se determine toate valorile a, b, c, pentru care f este derivabilă pe întreg domeniul de definiŃie.
3. Se numeşte cod de lungime 9 o matrice M de tip 3 × 3 care are drept elemente doar cifre nenule,
astfel încât fiecare cifră apare o singură dată.
a) Să se arate că există măcar 2 coduri M de lungime 9, pentru care detM = 0.
b) DeterminaŃi numărul total de coduri de lungime 9.
c) Există coduri M de lungime 9 pentru care detM este nenul şi multiplu de 3?
4. Un drum este realizat de 2 constructori numiŃi A şi B. Constructorul A realizează porŃiunea de
drum numită f, iar B realizează porŃiunea de drum numită g. Se ştie că f: [0, 2π] → [–1, 1] şi
g: [2π, 10] → R, f(x) = sin x şi g(x) = ax + b, a, b ∈ R.
a) Un autovehicul se deplasează din punctul M(0, 0) către punctul N(2π, 0), pe drumul construit
de A. Ştiind că autovehiculul se deplasează cu viteză constantă, să se identifice punctul în care
asupra autovehiculului nu acŃionează forŃa centrifugă (forŃa care „împinge” autovehiculul în exte-
riorul curbei). ArgumentaŃi.
b) Să se determine a şi b astfel încât cei doi constructori să poată „lipi” cele două drumuri, astfel
încât o maşină să poată parcurge întregul drum astfel format.

CLASA A XII-
XII-A

 x 0 x 
   
1. Fie G = M ∈ M 3 (R ) M =  0 0 0 , x ∈ R  .
 x 0 x 
   

166
a) Să se arate că (∀) M1, M2 ∈ G, rezultă că M1 ⋅ M2 ∈ G.
b) Să se determine o matrice F ∈ G cu proprietatea că M ⋅ F = F ⋅ M = F, (∀) M ∈ G.
c) Să se determine o matrice E ∈ G astfel încât M ⋅ E = E ⋅ M = M, (∀) M ∈ G.
 x, x ∈ [0,1]
 4 
2. Fie f: 0,  → R, f ( x ) =  2  4 .
 3  x − 2 x + 2, x ∈ 1, 
  3
 4
a) ArătaŃi că f e continuă pe 0,  .
 3
b) CalculaŃi o primitivă pentru funcŃia f.
c) Să se calculeze volumul „clopotului” format prin rotirea graficului funcŃiei f în jurul lui Ox.
3. Fie polinomul P = X4 + aX2 + 1 ∈ R[X].
a) Să se arate că pentru a = –4, polinomul de mai sus are toate rădăcinile reale.
b) Să se descompună P în factori ireductibili, (în R[X]) pentru a = 1.
4. Fie f: [0, π] → [0, 1], f(x) = sin x.
a) Să se demonstreze că f este concavă.
b) Să se demonstreze că lungimea graficului lui f este mai mare decât π2 + 4 .

Concursul NaŃional de Matematică aplicată „Adolf Haimovici”,


Etapa judeŃeană, 13 martie 2010

Filiera tehnologică: profil tehnic

CLASA A IX-
IX-A

1. Se consideră mulŃimea M = {1, 2, 3, ..., 10}.


a) Câte progresii aritmetice cu trei elemente şi raŃia pozitivă se pot forma cu elementele mulŃimii M?
b) Câte progresii geometrice cu trei elemente şi raŃia supraunitară se pot forma cu elementele
mulŃimii M?
2. a) Să se arate că, dintre toate dreptunghiurile de perimetru constant, pătratul are aria maximă.
b) Să se arate că dreptunghiul de arie maximă înscris într-un cerc este pătratul.
3. Spunem că perechea de numere naturale (a, b) este ideală dacă a2 – 2b2 = 1.
a) DaŃi exemple de două perechi de numere naturale ideale.
b) ArătaŃi că, dacă perechea (a, b) este ideală, atunci şi perechea (a2 + 2b2, 2ab) este ideală.
4. S-au tăiat bârne pentru lemne de foc. S-au făcut 1510 tăieturi şi s-au obŃinut 2010 bucăŃi. Câte
bârne au fost la început?

167
CLASA A X-
X-A

1. Se consideră numerele a = log3 (3 100 − 3 19 ) şi b = log3 (3 10000 − 3 1900 + 3 361) .


a) CalculaŃi a + b. b) ArătaŃi că a ⋅ b ≤ 4.
− 1+ i 3
2. Se consideră ε = ; w = 1 + ε, iar H = {z = a + bε | a, b ∈ Z}.
2
a) ArătaŃi că w ∈ H.
b) Dacă z1 şi z2 ∈ H, verificaŃi că şi z1 ⋅ z2 ∈ H.
c) GăsiŃi toate numerele z ∈ H care au modulul egal cu 1.
x
3. Se consideră funcŃiile fn: R → R, f1 ( x ) = , iar f n ( x ) = (f n −1  f1 )( x ) , n ∈ N, n ≥ 2.
1+ x2
1
a) RezolvaŃi în intervalul (0, ∞) ecuaŃia f1 ( x ) + f1   = 2 .
x
b) RezolvaŃi în intervalul [0, 2π] ecuaŃia 2 ⋅ f1 ( tgx ) = 3 .
c) GăsiŃi fn(x).
4. Notăm cu dn numărul diagonalelor unui poligon convex cu n laturi.
n ( n − 3)
a) GăsiŃi d6. b) VerificaŃi dn+1 – dn = n – 1, n > 2. c) ArătaŃi că d n = , (∀) n ≥ 3.
2

CLASA A XI-
XI-A

 1 2 3
 
1. Se dă matricea A =  4 5 6  . Sunt permise următoarele două operaŃii:
 7 8 9
 
- Adunarea numărului 1 la fiecare dintre elementele oricărei linii;
- Scăderea numărului 1 din fiecare dintre elementele oricărei coloane.
a) Este matricea A inversabilă?
b) Plecând de la matricea A şi utilizând operaŃiile date, se poate obŃine transpusa acestei matrice?
JustificaŃi răspunsul.
2. În planul raportat la sistemul ortogonal de axe de coordonate se consideră punctele An(n + 1, n2 +
+ 2n + 4), n ∈ N*.
a) CalculaŃi aria triunghiului A1A2A3.
b) ArătaŃi că aria triunghiului An–1AnAn+1 este independentă de n.
sin x
3. a) Să se calculeze lim .
x →0 1 + x − 1

sin x + sin 2 x + ... + sin 2010x


b) Să se determine lim .
x →0 1+ x −1

168
2
 | m | 2 x2
4. a) Să se determine L(m) = lim  cos x − ⋅ x  , m ∈ R.
x → 0 2 
1
b) Să se arate că L(m1 ) ⋅ L(m 2 ) ≤ ⋅ L(m1 + m 2 ) , (∀) m1, m2 ∈ R.
e
CLASA A XII-
XII-A

x +1 1  3 1 
1. a) DemonstraŃi că =  +  , (∀) x ∈ R.
x2 − 4 4  x − 2 x + 2 
x2 + x − 3
b) Să se calculeze ∫ x2 − 4
dx , x ∈ (2, ∞).

x + 3, x <1
2. Fie f: R → R, f ( x ) =  2 , a ∈ R.
ax + x + 2, x ≥ 1
a) Să se determine a, astfel încât f să fie continuă pe R.
b) Pentru a determinat anterior, să se calculeze ∫ f (x )dx .
3. Fie (G, ⋅) un grup cu 5 elemente şi e ∈ G, elementul neutru al grupului.
a) Să se dea un exemplu de asemenea grup.
b) Se admite că G este comutativ şi că x5 = e, (∀) x ∈ G. Fie y ∈ G – {e}. Să se arate că G =
= {y, y2, y3, y4, y5}.
0 0 1
 
4. Fie A =  1 0 0  .
 0 1 0
 
a) Să se calculeze A2 şi A7. b) Să se arate că (G, ⋅) este grup comutativ, unde G = {An | n ∈ N*}.

Olimpiada NaŃională de Matematică,


Etapa judeŃeană şi a Municipiului Bucureşti, 13 martie 2010

CLASA A IX-
IX-A

1. O dreaptă care trece prin centrul I al cercului înscris unui triunghi ABC taie laturile AB şi AC în
PB QC
P, respectiv Q. Notăm BC = a, AC = b, AB = c şi = p, =q.
PA QA
i) ArătaŃi că a (1 + p)IP = (a − pb)IB − cpIC .
ii) ArătaŃi că a = bp + cq.
iii) ArătaŃi că dacă a2 = 4bcpq, atunci dreptele AI, BQ şi CP sunt concurente.

169
2. Se consideră şirul (xn)n≥0 dat prin xn = 2n – n, n ∈ N. DeterminaŃi toate numerele naturale p, pentru
care sp = x0 + x1 + x2 + ... + xp, este o putere cu exponent natural al lui 2.
Dan Nedeianu, Drobeta Turnu Severin
Gazeta Matematică
3. Fie x un număr real. ArătaŃi că x este număr întreg dacă şi numai dacă relaŃia:
n ([ x ] + [nx ])
[ x ] + [2 x ] + [3x ] + ... + [nx ] = are loc pentru orice n ∈ N*.
2
(Prin [a] s-a notat partea întreagă a numărului real a).
Nicolae BourbăcuŃ, Hunedoara
4. DeterminaŃi toate funcŃiile f: N* → N* cu proprietatea f(n) + f(n + 1) + f(f(n)) = 3n + 1, pentru
orice n ∈ N*.
Călin Burduşel, Târgovişte

CLASA A X-
X-A

1. DemonstraŃi următoarele egalităŃi de mulŃimi:


i) {x ∈ R | log2[x] = [log2 x]} = ∪[2 m
, 2 m + 1) ;
m∈N
[x]
ii) {x ∈ R | 2 = [2 ]} =
x
∪[m, log (2 2
m
+ 1)) .
m∈N
(Prin [a] s-a notat partea întreagă a numărului real a).
Călin Burduşel, Târgovişte

2. Fie a ∈ [–2, ∞), r ∈ [0, ∞) şi numărul natural n ≥ 1. ArătaŃi că r2n + arn + 1 ≥ (1 – r)2n.
Gazeta Matematică
3. DeterminaŃi funcŃiile f: N → N cu proprietatea 3f(f(f(n))) + 2f(f(n)) + f(n) = 6n, pentru orice n ∈ N.
Vasile Pop, Cluj-Napoca
1
4. Fie şirul a n = z n + n , n ≥ 1, unde z ∈ C* este dat.
z
a + a n +2
i) DemonstraŃi că dacă a1 > 2, atunci a n +1 < n , pentru orice n ∈ N*.
2
ii) DemonstraŃi că dacă există k ∈ N* astfel încât ak ≤ 2, atunci a1 ≤ 2.

CLASA A XI-
XI-A

a1x + b1 , x ≤ 1
1. ArătaŃi că orice funcŃie continuă f: R → R de forma f ( x ) =  , unde a1, a2, b1, b2 ∈ R,
a 2 x + b 2 , x > 1
poate fi scrisă sub forma f(x) = m1x + n1 + ε|m2x + n2|, pentru x ∈ R, unde m1, m2, n1, n2 ∈ R, iar
ε ∈ {–1, 1}.

170
2. Se consideră matricele A, B ∈ M3(C) cu A = –tA, B = tB. ArătaŃi că dacă funcŃia polinomială
defnită prin f(x) = det(A + xB) are o rădăcină multiplă, atunci det(A + B) = detB.
(Prin tX s-a notat transpusa matricei X).
Dan Nedeianu, Drobeta Turnu Severin
3. Fie f: R → R strict crescătoare astfel încât f  f este continuă. ArătaŃi că f este continuă.
Gazeta Matematică
4. DemonstraŃi că există şiruri (an)n≥0 cu an ∈ {–1, 1} pentru orice n ∈ N, astfel încât:
1
lim ( n + a1 + n + a 2 + ... + n + a n − n n + a 0 ) = .
n →∞ 2
RomanŃa şi Ioan GhiŃă

CLASA A XII-
XII-A

1. Fie S suma elementelor inversabile ale unui inel finit. ArătaŃi că S2 = S sau S2 = 0.
Mihai Piticari, Câmpulung Moldovenesc
2. Fie G un grup cu proprietatea că dacă a, b ∈ G şi a2b = ba2, atunci ab = ba.
i) Dacă G are 2n elemente, arătaŃi că G este abelian.
ii) DaŃi un exemplu de grup neabelian care are proprietatea din enunŃ.
Gazeta Matematică
3. Fie a < c < b trei numere reale şi f: [a, b] → R o funcŃie continuă în c. ArătaŃi că dacă f are
primitive pe fiecare dintre intervalele [a, c) şi (c, b], atunci f are primitive pe intervalul [a, b].
Mihai Piticari, Câmpulung Moldovenesc
1
4. Fie f: [0, 1] → R o funcŃie derivabilă, astfel încât f(0) = f(1), ∫ f (x )dx = 0 şi f'(x) ≠ 1, oricare ar fi
0
x ∈ [0, 1].
i) DemonstraŃi că funcŃia g: [0, 1] → R dată prin g(x) = f(x) – x este strict descrescătoare.
n −1
k 1
ii) ArătaŃi că pentru orice număr întreg n ≥ 1 avem: ∑ f  n  < 2 .
k =0
American Mathematical Monthly

Olimpiada NaŃională de Matematică

CLASA A IX-
IX-A

1. Se consideră triunghiul ABC şi punctele D, E, F în care bisectoarele unghiurilor 'BAC, 'ABC,


respectiv 'ACB ale triunghiului taie cercul său circumscris.
a) ArătaŃi că ortocentrul triunghiului DEF coincide cu centrul cercului înscris în triunghiul ABC.
b) ArătaŃi că, dacă AD + BE + CF = 0 , atunci triunghiul ABC este echilateral.
Prof. Marian Ionescu, Piteşti

171
2. DemonstraŃi că există o asemănare între un triunghi ABC şi triunghiul având ca laturi medianele
sale dacă şi numai dacă pătratele lungimilor laturilor triunghiului ABC sunt în progresie arit-
metică.
Prof. Marian Teler, Costeşti; prof. Marian Ionescu, Piteşti
x x x
3. Pentru orice număr natural n ≥ 2 notăm An mulŃimea soluŃiilor ecuaŃiei x =   +   + ... +   .
2 3 n
a) DeterminaŃi A2 ∪ A3.
b) ArătaŃi că mulŃimea A = ∪A
n≥2
n este finită şi determinaŃi maxA.

Prof. Dan Nedeianu, Drobeta Turnu-Severin


4. Se consideră mulŃimea F a funcŃiilor f: N → N care au proprietatea f(a2 – b2) = f2(a) – f2(b), pentru
orice a, b ∈ N, a ≥ b.
a) DeterminaŃi {f(1) | f ∈ F}. b) ArătaŃi că F are exact două elemente.
Prof. Nelu Chichirim, ConstanŃa
CLASA A X-
X-A

n
1. Se consideră şirul (an)n≥0 de numere reale strict pozitive, pentru care ∑C a a
k =0
k
n k n −k = a 2n , pentru

orice n ∈ N. Să se arate că şirul este o progresie geometrică.


Prof. Lucian Dragomir, OŃelu-Roşu
2. Fie v, w ∈ C*, distincte. Să se arate că zw + w ≤ zv + v , pentru orice z ∈ C, |z| = 1, dacă şi
numai dacă există k ∈ [–1, 1] astfel încât w = kv.
Prof. Dan Marinescu, Hunedoara
3. Se consideră în plan 100 de puncte, oricare trei necoliniare. Punctele se împart în 10 grupe, fiecare
având cel puŃin trei puncte. Oricare două puncte din aceeaşi grupă se unesc între ele cu un segment.
a) Să se determine pentru ce împărŃirea punctelor numărul de triunghiuri formate cu aceste seg-
mente este minim.
b) Să se arate că există o alegere a grupelor cu proprietatea că toate segmentele pot fi colorate cu
trei culori astfel încât să nu existe un triunghi având laturile colorate cu aceeaşi culoare.
Prof. Vasile Pop, Cluj-Napoca
4. În exteriorul triunghiului ABC se consideră triunghiurile asemenea ABM, BCN şi CAP, astfel
încât triunghiul MNP să fie echilateral. Să se determine măsurile unghiurilor triunghiurilor ABM,
BCN şi CAP.
Prof. Nicolae BourbăcuŃ, Sarmizegetusa

172
CLASA A XI-
XI-A

1. Se consideră numerele reale a, b, cu b – a2 > 0. DeterminaŃi toate matricele A ∈ M2(R), astfel


încât det(A2 – 2aA + bI2) = 0.
Prof. G. René, Otopeni
2. Fie matricele A, B, C ∈ Mn(R), astfel încât ABC = On şi rang(B) = 1. ArătaŃi că AB = On sau
BC = On.
Prof. Marian Andronache, Bucureşti
3. Fie f: R → [0, ∞). ArătaŃi că f satisface inegalitatea f(x + y) ≥ (1 + y)f(x) pentru orice x ∈ R şi
orice y ≥ 0, dacă şi numai dacă funcŃia g: R → [0, ∞), g(x) = e–xf(x) este crescătoare.
Prof. Nicolae BourbăcuŃ, Sarmizegetusa
1
4. Pentru un număr real pozitiv a, definim şirul de numere reale (xn)n≥1 prin x1 = a, x n +1 = x n − ,
n
n ≥ 1. ArătaŃi că şirul este convergent şi calculaŃi limita sa.
Prof. Viorel Vâjâitu, Bucureşti
CLASA A XII-
XII-A

x
1. Fie f: R → R o funcŃie monotonă şi F: R → R, F( x ) = ∫ f ( t )dt .
0
ArătaŃi că dacă F este derivabilă, atunci f este continuă.
Prof. Dorin Andrica, Mihai Piticari
2. Un inel A are proprietatea (P) dacă orice element nenul se scrie în mod unic ca suma dintre un
element inversabil şi unul neinversabil.
a) Dacă în inelul A, 1 + 1 ≠ 0, arătaŃi că A are proprietatea (P) dacă şi numai dacă A este corp.
b) DaŃi un exemplu de inel cu cel puŃin două elemente, care are proprietatea (P), dar nu este corp.
Prof. Dan Schwarz, Bucureşti
3. Fie G un grup finit cu n elemente, H = {x | x ∈ G şi x = e}, unde e este elementul neutru al lui G
2

şi p numărul elementelor lui H. ArătaŃi că:


a) card(H ∩ xH) ≥ 2p – n, oricare ar fi x ∈ G, unde xH = {xh | h ∈ H};
3n n 3n
b) dacă p > , atunci G este comutativ; c) dacă < p ≤ , atunci G este necomutativ.
4 2 4
Prof. Marian Andronache, Bucureşti
h
4. Fie f: [–1, 1] → R o funcŃie continuă, derivabilă în 0 şi I(h ) = ∫ f ( x )dx , h ∈ [0, 1].
−h
ArătaŃi că:
a) există M > 0, astfel încât |I(h) – 2f(0)h| ≤ Mh2, oricare ar fi h ∈ [0, 1];
n
1

b) şirul (an)n∈N*, definit prin a n =
k =1
k ⋅ I  este convergent dacă şi numai dacă f(0) = 0.
k
Prof. Călin Popescu, Bucureşti

173
CLASA A IX - A

ALGEBRĂ
I. Elemente de logică matematică şi mulŃimi

1.1. Elemente de calcul propoziŃional


1 3 1 3
1. AfirmaŃia adevărată. De exemplu a = , b = , a ≠ b, dar f   = f   = 1 ; b) Fie m = 2p; f(2p) =
2 4 2 4
 4p + 3   3 3  2k 2 + 3 
=   = p +  = p +   = p, afirmaŃia este prin urmare adevărată; c) Din 1 + k = 2   , găsim,
 4   4 4  4 
1 + k  2k 2 + 3 
rezolvând =  , k = 1, adică A = {1}. AfirmaŃia este falsă; 2. Avem succesiv, folosind x = [x] +
2  4 
 x + 1   x + 3  x + 3  x + 3   x + 1 x + 1  x + 1 x + 3  x + 1
+ {x},   −  = −  −  , de unde − = −  , ceea ce revine
 2   2  2  2   2  2  2  2  2 
la x + 1 = x + 3, adică 1 = 3, fals; 3. Pentru a determina mulŃimea de adevăr a lui p1(x, y), rezolvăm sistemul
x + 5y = 17 x = 2
 , cu soluŃia  . MulŃimea de adevăr a lui p1(x, y) este A = {(2, 3)}. Pentru calculul mulŃimii
3 x − 4 y = −6 y = 3
 x − 1 x + 1 x +1
de adevăr a lui p2(x) rezolvăm ecuaŃia   = , x ∈ R. Notăm = k , k ∈ Z. De aici, x = 4k – 1 (1).
 3  4 4
4k − 2
Folosind definiŃia părŃii întregi, găsim k ≤ < k + 1 (2). Rezolvând (2), găsim k ∈ {2, 3, 4}, de unde x ∈
3
∈{7, 11, 15}. Ultima mulŃime reprezintă mulŃimea căutată. 4. ImplicaŃia p2 ⇒ p1 este evidentă: cum p2 este
adevărată pentru orice k ∈ Z, pentru k = 1 şi k = 2, obŃinem p1. Demonstrăm acum că p1 ⇒ p2. Avem
a − b = d − c a − d = b − c a − b = d − c  a = b a = d 
 2 ⇔ ⇔  ⇔   sau   ⇔

a − b = d − c
2 2 2
( a − b )( a + b ) = ( d − c )( d + c ) ( a − b )( a + b − d − c ) = 0  c = d b = c 
 a k = b k a k = d k   a k + ck = d k + bk 
⇔   ⇒   2a k + c k − b k = 2d k + b k − c k
 c k = d k  k
sau sau sau
 b = c k   a k − bk = dk − ck 
 
2(a k − b k + c k − d k ) = 0 sau a k − b k + c k − d k = 0 , adică p2. 5. Pentru a fi verificată egalitatea (–∞, 2x + 2009) ∩
3x ∈ (−∞, 2x + 2009) 3x < 2x + 2008 x < 2008
∩ N = {0, 1, 2, ..., 3x}, este necesar ca  ⇔ ⇔ şi, cum
3x + 1 ∉ (−∞, 2x + 2009) 3x + 1 ≥ 2x + 2008 x ≥ 2007
x ∈ N*, rezultă că x = 2007. Cum este adevărat că (–∞, 2 ⋅ 2007 + 2008) ∩ N = {0, 1, 2, ..., 3 ⋅ 2008}, deducem

174
că p este adevărată. 6. Avem de rezolvat în Z ecuaŃia x2 + 5x + 4 = n2. Rezolvarea este clasică, în final ajungând
la divizibilitate. EcuaŃia este echivalentă cu 4x2 + 20x + 16 – 4n2 = 0, de unde (2x + 5)2 – 4n2 = 9, iar în final
avem (2x + 5 – 2n)(2x + 5 + 2n) = 9, cu soluŃiile x ∈ {0, –1, –5}, ceea ce arată că afirmaŃia este adevărată.
7. a) Dacă luăm punctele M pe mediatoarea mAB a segmentului AB, se obŃine condiŃia mAB || mCD, echivalent cu
AB || CD şi analog BC || AD; b) Din teorema medianei în triunghiurile BMD şi CMA se obŃine AC ≤ BD;
c) Luăm poziŃiile particulare M = B şi M = D şi din inegalitatea triunghiului rezultă AB + AD = BD, de unde
A ∈ (BD). 8. Deoarece ax2 + bx + c ∈ Q, (∀) x ∈ R, făcând x = 0, obŃinem că c ∈ Q şi f(x) = ax2 + bx ∈ Q,
(∀) x ∈ R. Avem că f(1), f(–1) ∈ Q, deci a + b ∈ Q, a – b ∈ Q, adică a, b ∈ Q. Cum f ( 2 ) ∈ Q, 2a + b 2 ∈
∈Q, de unde b = 0. Dar f ( 2 ) = a 4 ∈ Q, prin urmare a = 0; 9. (⇒) Din
3 3
ax 22 + bx 2 + c = 0 şi ax1 + bx2 +
c c c2 b
+ c = 0, rezultă că x1 = x 22 , deci x1 x 2 = x 32 = ⇒ x2 = 3 şi x1 = 3 2 . Din x1 + x 2 = − , găsim că
a a a a
c 3 c2 b 3
3 + + = 0 , de unde ca 2 + 3 c 2 a + b = 0 , (1); (⇐) ÎmpărŃim în (1) prin a ≠ 0 şi obŃinem
a a2 a
c 3 c2 b
3 + + = 0 ⇒ 3 x1x 2 + 3 ( x1x 2 ) 2 − x1 − x 2 = 0 ⇒ (3 x 2 − 3 x12 )(3 x 22 − −3 x 2 ) = 0 ⇒ x 2 = x12 ⇒ ax 22 +
a a2 a
+ bx 2 + c = 0 ⇒ ax1 + bx 2 + c = 0 . 10. Este evident că x2 + y2 ≥ 2xy, (∀) x , y ∈ R, de unde
1 1
xy − x − y − ≥ 2xy , adică xy + x + y + ≤ 0 (1). Mai mult, din x2 + y2 ≥ xy, (∀) x, y ∈ R, obŃinem că
4 4
1 3 1 2
x + y + ≤ 0 sau 3x + 3y + ≤ 0 (2). Rescriem (1) sub forma xy + 3( x + y) + ≤ 2(x + y) = ⋅ 3(x + y) ≤
4 4 4 3
2 3 1 1 1 3 1
≤  −  = − . De aici, xy + 3(x + y) ≤ − − = − < . 11. Avem că x = [x] + {x}, (∀) x ∈ R, cu 0 ≤ {x} <
3 4 2 2 4 4 4
< 1. De asemenea, m{x} < m şi atunci [mx] = [m[x] + m{x}] = m[x] + [m{x}] = n[x] + p, unde p = [m{x}] ∈ {0, 1,
2, ..., m – 1}, ceea ce arată că propoziŃia este adevărată. 12. Avem (a se vedea problema 9) că există p1 ∈ {0, 1, 2,
..., m – 1}, p2 ∈ {0, 1, 2, ..., n – 1} astfel încât [mx] = m[x] + p1; [nx] = n[x] + p2, de unde [mx] – [nx] =
= (m – n)[x] + p1 – p2 = (m – n)[x] + q, unde q = p1 – p2 ∈ Z., şi |q| = |p1 – p2| ≤ max{p1, p2} ≤ max{m, n}.

1.2. Metoda inducŃiei matematice


1. InducŃie matematică. P(2): 2 = 2 ⋅ 1 + 3 ⋅ 0 (k1 = 1, k2 = 0). Presupunem P(k), k = 2k1 + 3k2; P(k + 1): k + 1 =
= 2k1 + 3k2 + 1 = 2k1 + 3k2 + 3 – 2 = 2(k1 – 1) + 3(k2 + 1) = 2l1 + 3l2; 2. P(1): Cum a1 = 12, P(1) adevărată.
Presupunem P(n) adevărată şi demonstrăm P(n + 1), adică 4 | 3n+2 + 2(n + 1) + 1. Într-adevăr 3n+2 + 2(n + 1) +
+ 1 = 3n+1 + 2n + 1 + 2 ⋅ (3n+1 + 1). Cum 3n+2 + 1 ∈ 2N, afirmaŃia P(n + 1) este adevărată; 3. Celebra teoremă a
lui Endös-Sunauyi, enunŃată pe N şi inducŃia este specială, o inducŃie de tip Cauchy. Vom arăta că P(n) → P(n + 4),
(∀) n ∈ N. Deci: presupunem P(k) adevărată, adică n = ± 12 ± 22 ± ... ± k2. Cum 4 = –12 – 22 + 32 = (k + 1)2 –
– (k + 2)2 – (k + 3)2 + (k + 4)2. Prin urmare n + 4 = ± 12 ± 22 ± 32 ± ... ± k2 + (k + 1)2 – (k + 2)2 – (k + 3)2 + (k +
+ 4)2 ⇒ P(n + 4) adevărată; 4. a) InducŃie imediată; b) Din a) obŃinem forma echivalentă: 3( x12 + x 22 + ... + x 2n ) −
– 6(x1 + 3x2 + ... + (2n – 1)xn) + 3(12 + 32 + ... + (2n – 1)2) ≤ 0, de unde scrierea (x1 – 1)2 + (x2 – 3)2 + ... + (xn –
– (2n – 1))2 ≤ 0 cu soluŃiile xn = 2n – 1, (∀) n ∈ N. 5. a) Vom arăta că {a1, a2, ..., an, ...} = N*. Observăm că
pentru a1 = 1, a2 = 2, ..., an = n egalitatea din enunŃ este verificată. Fie deci propoziŃia P(n): „an = n”, pentru orice
n ∈ N*. Inductiv arătăm acest lucru. cum pentru n = 1, P(1), a13 = a12 , conduce la a1 = 1, presupunem ak = k,

175
n 2 (n + 1) 2
(∀) k ∈ {1, 2, ..., n}. Din a13 + a 32 + ... + a 3n + a 3n +1 = (a1 + a 2 + ... + a n + a n +1 ) 2 , deducem + a 3n +1 =
4
2
 n (n + 1) 
= + a n +1  , de unde efectuând calculele, găsim an+1 = n+1 sau an+1 = –n. Cum an ∈ N*, obŃinem an+1 =
 2 
= n + 1. 6. Dubla inegalitate se demonstrează prin metoda inducŃiei. Vom demonstra numai una din inegalităŃi,
1 1 1 1 1 1
anume că P(n): + + ... + < 2 , (∀) n ∈ N*. Etapa de verificare. P(1): + + < 2 (A). Etapa
n +1 n + 2 3n + 1 2 3 4
1 1 1 1
de demonstraŃie. Vom demonstra de fapt o inegalitate mai „tare”, anume că + + ... + <2 −
n +1 n + 2 3n + 1 n
1 1 1 1
(1). Într-adevăr, presupunând (1) adevărată pentru 1 ≤ k ≤ n, P(k): + + ... + < 2 − , arătăm că
k +1 k + 2 3k + 1 k
1 1 1 1 1 1 1 1
este adevărată şi P(k+1): + + ... + < 2− . Avem: + + ... + = +
k+2 k+3 3k + 4 k +1 k+2 k +3 3k + 4 k + 1
1 1 1 1 1 1 1 1 1 1 1 2
+ + ... + + + + < 2− − + + + =2− − +
k+2 3k + 1 3k + 2 3k + 3 3k + 4 k k + 1 3k + 2 3k + 3 3k + 4 k 3(k + 1)
1 1 1 1 1 1 1 1 1 6(k + 1) 1
+ + = 2− − + + + = 2− − + = 2− −
3k + 2 3k + 4 k + 1 k k + 1 3k + 2 3k + 4 k + 1 k (k + 1) (3k + 2)(3k + 4) k +1
n n n
3k − 8 n (n + 1)(2n + 1) n (n + 1)
∑ k(k + 1) = ∑ k + ∑ k =
1
− < 2− . 7. Avem că 2
+ =
k (k + 1)(3k + 2)(3k + 4) k +1 k =1 k =1 k =1
6 2
n (n + 1)(2n + 1) + 3n (n + 1) n (n + 1)( 2n + 4) n (n + 1)( n + 2)
= = = . 8. a) Folosind simbolul de sumare şi valorile
6 6 3
uzuale ale sumelor puterilor naturale ale şirului de numere naturale, obŃinem:
1  n 2 ( n + 1) 2 n ( n + 1)( 2 n + 1)  1
n n
k ( k + 1) 1 3n ( n + 1) + 2(2n + 1)
Sn = ∑
k =1
k
2
= ∑
2 k =1
(k 3 + k 2 ) = 
2  4
+
6
 = ⋅ n (n + 1) ⋅
 2 12
=

n (n + 1)(3n 2 + 7n + 2)
= . Acelaşi rezultat se poate demonstra şi folosind metoda inducŃiei matematice;
24
n (n + 1)( n + 2)(3n + 1)
b) Cum 3n2+ 7n + 2 = (n + 2)(3n + 1), din a) ∈ N, deci n(n + 1)(n + 2)(3n + 1) ⋮ 24.
24
9. Aplicăm metoda inducŃiei matematice. Etapa de verificare: Pentru n = 0, a0 = 4§4(A). Etapa demonstrativă:
Presupunem proprietatea adevărată pentru un 0 ≤ k ≤ n: ak = 3k+1 + 2k + 1, şi arătăm că are loc şi pentru un k + 1:
ak+1 = 3k+2 + 2k + 3. Într-adevăr, ak+1 = 3k+2 + 2k + 3 = 3 ⋅ 3k+2 + 3 ⋅ 2k + 3 – 6k + 2k = 3(3k+1 + 2k + 1) – 4k§4,
conform ipotezei inductive. 10. Suma S se poate scrie sub forma:
n n n n
2(1 + 2 + 3 + ... + k) k(k + 1) 1 1 
∑ ∑ k (k + 1) = ∑ k(k + 1) = ∑  k − k + 1  = 1 − n + 1 = n + 1 .
1 1 n
S= =
k =1 k 4 + 2k 3 + k 2 k =1
2 2
k =1 k =1
10. Fie a, b ∈ R+. Dacă n = 1, atunci a + b ≤ a + b, evident adevărată. Presupunem că (a + b)k ≤ 2k–1(ak + bk).
Rezultă: (a + b)k+1 ≤ 2k–1(ak + bk)(a + b) = 2k–1(ak+1 + bk+1 + abk + bak) (1). Dar abk + akb ≤ ak+1 + bk+1 (2). Într-a-
devăr, rescriem (2): bk(a – b) – ak(a – b) ≤ 0 ⇔ (bk – ak)(a – b) < 0 ⇔ –(a – b)2(ak–1 + ... + bk–1) ≤ 0, proprietate
adevărată. Folosind (2) în (1), găsim că (a + b)k+1 ≤ 2k–1(2ak–1 + 2bk–1) = 2k(ak + bk). 11. Etapa de verificare.

176
n
1⋅ 2 ⋅ 6 n (n + 1)(n + 5)
Pentru n = 1, avem 1!⋅1 =
12
(A); Etapa demonstrativă. Presupunem că ∑
k =1
k +1
k! k ≤
12
,n∈

n +1
(n + 1)(n + 2)(n + 6)
∈ N* şi demonstrăm ∑
k =1
k +1
k! k ≤
12
(*).

n +1 n
n (n + 1)(n + 5) n + 2 n (n + 1)(n + 5)
Într-adevăr, ∑
k =1
k +1
k!k =∑ k =1
k +1
k!k + n + 2 (n + 1)!(n + 1) ≤
12
+ (n + 1) n +1 (n + 1) =
12
+n+

(n + 1)( n + 2)( n + 6)
+1 < , adică (*). 12. i) Etapa de verificare. Pentru n = 0, se verifică; Etapa de demonstraŃie.
12
Presupunem că (9n2 – 24n + 1)4n + 16§27 şi demonstrăm că (9(n+1)2 – 24(n + 1) + 1)4n+1 + 16 ⋮ 27. Este
suficient să demonstrăm că diferenŃa lor este divizibilă cu 27. Dar diferenŃa este 4n(27n2 – 27n), care, evident,
este divizibilă cu 27. 13. Folosim proprietatea inductivă şi faptul că [x + n] = [x] + n, (∀) n ∈ N (1).
1 1
Etapa de verificare: dacă n = 1, obŃinem   =   (A); Etapa demonstrativă. Presupunem că:
2 2
1 2  k   k   k + 1 1 2  k   k + 1  k   k + 1 
 2  +  2  + ... +  2  =  2  ⋅  2  , (∀) k = 1, n şi obŃinem că  2  +  2  + ... +  2  +  2  =  2  +  2  ⋅
                     
 k + 1   k + 1 
  k   k + 1   k + 2 
⋅ =     + 1 =  ⋅  , ceea ce trebuia demonstrat. SoluŃie alternativă: Dacă n este par,
 2   2   2    2   2 
1   2  3 n  1  2 3   4   5     2k − 2   2k − 1   2k  
n = 2k, atunci   +   +   + ... +   =   +    +    +    +    + ... +   +  +    =
2 2 2  2  2  2 2   2   2    2   2   2 
 n   n + 1
= 0 + 2 ⋅ 1 + 2 ⋅ 2 + … + 2(k – 1) + k = k2 =   ⋅   . La fel dacă n este impar.
2  2 
14. Folosim metoda inductivă. Etapa de verificare. Pentru n = 2, avem că (a1 – 1)(a2 – 1) ≥ 0, deoarece ai ≥ 1.
n
 n 
Etapa demonstrativă. Presupunem că ∏ (1 + a i ) − 2 n −1  ∑ (a i − 1)  ≥ 2 n , (∀) n ≥ 2, unde ai ≥ 1. Atunci
i =1  i=1 
n +1   n   n 
∏ (1 + a i ) ≥  2 n + 2 n −1 
  ∑(a i − 1)  (a n +1 + 1) = 2n(1 + an+1) + 
  ∑
(a i − 1) (1 + a n +1 ) , de unde:

i =1   i =1   i =1 
n +1
 n +1   n
  n +1 
∏ (1 + a i ) − 2n  ∑ (a i − 1)  ≥ 2n (1 + a n +1 ) + 2n −1  ∑ (a i − 1)  (1 + a n +1 ) − 2n  ∑ (a i − 1)  = 2n + 2n a n +1 +
i =1  i =1   i =1   i =1 
 n
  n  n n+
+2n −1  ∑ (a i − 1)  (1 + a n +1 ) − 2 n 
 i =1   ∑
(a i − 1) + a n +1 − 1 = 2n + 2n a n +1 + 2n −1 ⋅ ∑ (a i − 1) + 2n −1 a n +1 ∑ (a i − 1) −

 i =1  i =1 i =1

n n
−2n −1 ∑ (a i − 1) − 2n −1 a n +1 ∑ (a i − 1) − 2n a n +1 + 2n = 2n +1 .
i =1 i =1
n n
SoluŃie alternativă: Fie bi = ai – 1 ≥ 0, i = 1, n, atunci ∏ (1 + a ) = ∏ (2 + b ) = 2n + 2n–1(b1 + b2 + … + bn) + …≥
i =1
i
i =1
i

n
≥ 2n + 2n–1(b1 + b2 + … + bn) = 2n + 2n–1 ⋅ ∑ (a
i =1
i
− 1) .

177
1.3. Probleme de numărare. Alte tipuri de raŃionamente

1. a) Linia n începe cu a n = n 2 − n + 1 . Cum n – 1 < n 2 − n + 1 < n , (∀) n ≥ 2, n ∈ N, evident că doar pentru


n = 1, a1 ∈ N. Deci numai o linie începe cu primul element un număr raŃional; b) Din a) [an] = n – 1. Pentru n =
= 2010 obŃinem că [a2010] = 2009; 2. a) Numărăm pătratele ce le putem construi începând cu colŃul din stânga
sus: putem construi astfel n2 pătrate de latură 1, (n – 1)2 pătrate de latură 2, (n – 2)2 pătrate de latură 3, ş.a.m.d.
până la 1 pătrat de latură n. Numărul posibilităŃilor se obŃine prin adunarea lor şi este 12 + 22 + ... + n2 =
n (n + 1)(2n + 1)
= ; b) Observăm că pentru fiecare pătrat de la a) avem două triunghiuri cu ipotenuza verticală,
6
două triunghiuri cu ipotenuza orizontală, două triunghiuri cu ipotenuza pe direcŃia unei diagonale, două
triunghiuri cu ipotenuza pe direcŃia celeilalte diagonale. Acestea sunt singurele poziŃii pentru ipotenuză de a
obŃine triunghiuri dreptunghice isoscele. Cum acest lucru este valabil pentru fiecare pătrat ce se obŃine, avem că
2010 2 2010 k
n (n + 1)(2n + 1) 4n (n + 1)(2n + 1) k −1 1 
∑  9 
1 8
numărul căutat va fi 8 ⋅
6
=
3
. 3. Calculăm suma: ∑8
k =1
 k =
3  8 k =1
=

2010 2010
8 8 8
  − 1−   2010
1 9 8
= ⋅  
9 1 9
= ⋅ =1−   . 4. a) Numărul an din paranteza „n” va fi: an = n + (n + 1) +
8 8 9 1 9
−1
9 9
+ (n + 2) + (n + 3) + (n + 4) + (n + 5) = 6n + 15 = 3(2n + 5); b) Avem a = a1 ⋅ a2 ⋅ a3 ⋅ ... ⋅ a100 = (3 ⋅ 7) ⋅ (3 ⋅ 9) ⋅
⋅ (3 ⋅ 11) ⋅ ... ⋅ (3 ⋅ 215) = 3100 ⋅ 7 ⋅ 9 ⋅ 11 ⋅ ... ⋅ 199 ⋅ 201 ⋅ 203 ⋅ 205. În această scriere, cum 199 prim, evident
199 | a1 ⋅ a2 ⋅ ... ⋅ a100, dar a nu poate fi pătrat perfect; c) Avem de rezolvat ecuaŃia diofantică 3(2n + 5) = t2, t ∈
∈ N*. Evident că va avea o infinitate de soluŃii cu cel mai mic n obŃinut pentru t = 3, n = 11. Cel mai mic pătrat
perfect va fi în a 11-a paranteză. 5. card(A ∪ B) = card A + card B – card (A ∩ B) = 2011 – card (A ∩ B).
Determinăm maximul de elemente din A ∩ B. Dacă x, y ∈ A ∩ B, atunci x – y = k 2 şi x – y = p 3 , cu k, p ∈
∈ Z*, deci k 2 = p 3 , fals. Rezultă că A ∩ B are cel mult un element. Prin urmare min(card(A ∪ B)) = 2010
(când cele două progresii au un element comun).
6. Notăm cu x1 numărul de locuri de pe rândul 1. Numărul de spectatori de pe rândul 41 va fi modelat de
termenul de rang 41 al unei progresii aritmetice de raŃie 10, anume x41 = x1 + 40 ⋅ 10, de unde x1 = 100. Numă-
( x + x 41 ) ⋅ 41 (100 + 500) ⋅ 41
rul de spectatori va fi dat de suma progresiei aritmetice: S41 = 1 = = 12300 .
2 2
7. La ora 8 sunt contaminate 2 calculatoare. La ora 8 şi 10 minute sunt contaminate 2 + 2 ⋅ 3 = 8 calculatoare. La
8 şi 20 minute sunt contaminate 2 + 2 ⋅ 3 + 2 ⋅ 3 = 14 calculatoare. Urmând acest raŃionament, observăm că
numărul calculatoarelor contaminate este egal cu suma termenilor unei progresii aritmetice cu primul termen
[2 ⋅ 2 + (n −1) ⋅ 6)]⋅ n
a1= 2 şi raŃia r = 6. ObŃinem că Sn = = n(3n −1) . Avem de rezolvat ecuaŃia n(3n – 1) = 119800,
2
cu singura soluŃie convenabilă n = 200. Prima contaminare a avut loc la ora 8, deci a două suta contaminare va
avea loc peste 199 ⋅ 10 = 1990 minute, adică peste 33 ore şi 10 minute, adică marŃi la ora 17 şi 10 minute.
8. Există şapte puncte coliniare pe dreapta d; conform principiului cutiei, patru t
S2 S3 S4
din ele au aceeaşi culoare şi să presupunem că ele sunt colorate în roşu. Fie R1, S1
R2, R3, R4 aceste 4 puncte. Fie s, t alte două drepte cu s || t || d. Fie S1, S2, S3, S4 R4 d
proiecŃiile pe drepta s a punctelor R1, R2, R3, R4 iar T1, T2, T3, T4 proiecŃiile lui R3
R1 R2
R1, R2, R3, R4 pe t. Dacă două puncte de pe s sau t sunt roşii, atunci avem s
dreptunghiul căutat. Dacă pe s şi t se află cel mult câte un punct roşu dintre cele
T4
T2 T3
T1
178
considerate, din cele şase puncte albastre, patru formează dreptunghiul căutat cu vârfurile la fel colorate.
9. a) Fie (an)n≥1 o progresie aritmetică cu raŃia r şi primul termen a1 = 1. Atunci an = a1 + (n – 1)r. Deci 45001 =
= 1 + (n – 1)r, de unde (n – 1)r = 45000. De aici urmează că r este unul din divizorii lui 45000. Dar 45000 =
= 23 ⋅ 32 ⋅ 54. Numărul divizorilor este 4 ⋅ 3 ⋅ 5 = 60. Prin urmare, există 60 de progresii. b) Fie (an)n≥1 o progresie
aritmetică neconstantă care respectă proprietatea. Urmează că există şirul (xn)n≥1 strict crescător, xn ∈ N, astfel
încât an = x 2n . Cum an+1 – an = r, găsim (xn+1 – xn)(xn+1 + xn) = r. Dar cum (xn)n≥1 este strict crescător, xn+1 – xn ≥
r −1
≥ 1, adică xn+1 ≥ 1 + xn. De aici r = (xn+1 – xn) (xn+1 + xn) ≥ xn+1 + xn ≥ 1 + 2xn, de unde x n ≤ , adică şirul
2
(xn)n≥1 este mărginit, absurd! Urmează că nu există niciun şir cu proprietatea cerută. 10. La orice pas k: fie Sk+1 =
= suma numerelor de pe tablă înainte de ştergere; Sk = suma numerelor de pe tablă după ştergerea numerelor x şi y.
2 x , x < y
Atunci Sk+1 – Sk = x + y – |x – y| =  . Urmează că Sk+1 – Sk este întotdeauna un număr par. Fie k = 4n –
2 y, x > y
– 1. Atunci 1 + 2 + ... + 4n – 1 = 2n(4n – 1), deci suma iniŃială este un număr par. Urmează că suma obŃinută
după ştergere va rămâne tot timpul un număr par. Cum ultimul număr de pe tablă este o ”sumă” cu un singur
termen, înseamnă că el va fi par. 11. La orice pas k (înainte de ştergerea numerelor a şi b) vom nota cu Sk+1 suma
numerelor de pe tablă după ştergerea numerelor a şi b şi înlocuirea lor cu numerele a – 3b sau b – 3a. ObŃinem că
Sk+1 – Sk este unul din numerele a + b – (a – 3b) sau a + b – (b – 3a), adică 4b sau 4a, deci un număr întreg
divizibil cu 4. Dar S4n = 2n(4n + 1) = 2(2n + 1)(8k +5) = 2(2s + 1), s ∈ N. De aici deducem că 2 | S4n dar 4 /| S4 n .
Urmează că şi S4n–1 are aceeaşi proprietate, adică 2 | S4n–1 şi 4 /| S4 n −1 . Atunci oricare din termenii şirului S4n–1
are această proprietate, deci nu poate apărea 0, căci ar trebui ca 2 | 0 şi 4 /| 0 absurd, deci niciunul din termenii
şirului S4n–1 nu poate fi zero. 12. PoziŃia turnurilor va fi caracterizată de coordonatele lor. Cum tabla de şah are
dimensiunile 8 × 8 şi două turnuri nu pot fi simultan pe o aceeaşi linie sau coloană, atunci cele 8 turnuri pot
ocupa următoarele poziŃii: (1, a1), (2, a2), (3, a3), ..., (8, a8), unde {a1, a2, ..., a8} = {1, 2, ..., 8}. Avem condiŃia
suplimentară 1 ⋅ a1 + 2 ⋅ a2 + 3 ⋅ a3 + ... + 8 ⋅ a8 = 120. Trebuie să găsim acele valori pentru ak astfel încât
8

∑k⋅a
i =1
k să fie minimă. Alegem atunci ak astfel încât a1 → 8, a2 → 7, ..., a1 → 1. Cu alte cuvinte coordonatele

turnurilor vor fi: (1, 8), (2, 7), ..., (8, 1). 13. Secundarul se suprapune o dată pe minut cu minutarul, deci de 90
de ori în cele 90 de minute. Atunci la momentul m ore, n minute şi k secunde, unghiurile făcute de fiecare

dintre cele trei limbi ale ceasului cu limbile care ar arăta ora 12 sunt: secundarul face k ⋅ , iar minutarul
60
 60n + k  2π  60n + k   61k 
 90  ⋅ 60 . Cele două arce se suprapun dacă  90  =  90  = k , lucru posibil dacă şi numai dacă k = 0.
     
În trei zile, firesc, din cele spuse până acum, de două ori va fi situaŃia când cele trei arce se suprapun.
14. Dacă unim B1 cu Ai, respectiv Ci, i = 1, n , nu se formează puncte de
A
C1 An
intersecŃie. Dacă unim B2 cu C1, C2, ..., Cn segmentul B2Ci, i = 1, n − 1
C2
intersectează segmentul B1Cj, j = 1, n − 1 , pentru că sunt diagonalele C3
patrulaterului B1B2CjCi, formându-se i – 1 puncte de intersecŃie. Analog [B2Ci], A3
A2
i = 1, n intersectează B1Aj în n + i – 1 puncte, iar [B1Ai] intersectează [B1Ai], Cn A1
i = 1, i − 1 în i – 1 puncte. În total, dintr-un punct Bl, l < k se formează 1 + 2 +
B B1 B2 B3 Bn C
+ ... + n + (n + 1) + ... + (n + n – 1) = n(2n – 1) puncte de intersecŃie.

179
Pentru că în această situaŃie sunt k – 1 puncte Bl, l < k, vom avea (k – 1) ⋅ n ⋅ (2n – 1) puncte de intersecŃie. În
n
n 2 (n − 1)(2n − 1)
concluzie există ∑
k =1
(k − 1) ⋅ n ⋅ (2n − 1) =
2
puncte de intersecŃie. 15. Trasăm diagonala AC şi, în

fiecare dintre triunghiurile ABC şi ACD, ducem înălŃimea din vârful care se opune celei mai mari laturi
(asigurându-ne astfel că înălŃimea trasată intersectează latura pe care cade în interiorul acesteia). Am obŃinut
astfel o descompunere în 4 triunghiuri dreptunghice. Presupunem inductiv că avem o descompunere în n plăci ca
în enunŃ; trasând înălŃimea din vârful unghiului în unul dintre triunghiurile dreptunghice, obŃinem o descom-
punere în n + 1 plăci, ceea ce încheie rezolvarea.

1.4. OperaŃii cu mulŃimi. Determinări de mulŃimi


1. a) Avem 1 = 32 + 82 + 172 – 192, de unde 1 ∈ A. Analog 4 = 32 + 22 + 42 – 52, de unde 4 ∈ A; b) Demonstrăm
că (2k – 2)2 + (3k – 1)2 + (6k – 1)2 – (7k – 2)2 = 2k + 2, (∀) k ∈ N. De aici, Ńinând cont de a), 2N ⊂ A, (1).
Analog (2k – 3)2 + (3k – 8)2 + (6k – 17)2 – (7k – 19)2 = 2k + 1, (∀) k ∈ N. De aici 2N + 1 ⊂ A, (2). Din (1) şi
a + ak
(2), evident A = N. 2. b) Avem de rezolvat a n = m . EcuaŃia în numere întregi (2m – 1)(2p – 1) = (2n –
2
– 1)(p + m – 1) are o soluŃie dacă m = 2, n = 3, p = 8; c) CondiŃia bk+1 ≤ ap ≤ bk revine la 2k ≤ p ≤ 2k + 1, deci p =
= 2k şi p = 2k + 1. 3. a) Cum 5x | x3 + 6, evident că x | 6, x ∈ {–6, –3, –2, –1, 1, 2, 3, 6}, de unde, după verificare
4n + 1 3k + 1
M = {–6, –1}; b) Presupunem că A ∩ B ≠ ∅. Prin urmare, (∃) n, k ∈ N astfel încât = , de unde,
4n − 3 3k − 2
1 3k(2k + 5) − 15k + 2
după efectuarea calculelor, obŃinem în final n – k = , imposibil căci n, k ∈ N. 4. a) x = =
12 2k + 5
15k − 2 30k + 75 − 79 79
= 3k − . De aici 2x = 6k – = 6k − 5 + . Evident 2k + 5 ∈ D79 = {–79, –1, 1, 79}, de
2k + 5 2k + 5 2k + 5
unde k ∈ {–42, –3, –2, 37}. Imediat 2x ∈ {–268, –112, 52, 208}, de unde M = {–134, –56, 26, 104}; b) Notăm
card B = n. Cum card A = 4, card P(A) = 24 = 16. Avem 2n + 16 = 32, de unde n = 4. 5. a) 2048 = 1024 + 1024 =
= 210 + 210 = (25)2 + (25)2 ∈ M. Folosim faptul că dacă a ∈ Z, a2 ∈ M4 ∪ M4 + 1. De asemenea, imediat a, b ∈ Z,
atunci a2 + b2 ∈ M4 ∪ M4 + 1 ∪ M4 + 2, adică a2 + b2 ∈ Z – M4 + 3. Cum 2011 = 4 ⋅ 502 + 3 ∈ M4 + 3, 2011
∉ M; b) Fie p = x12 + y12 , q = x 22 + y 22 . Avem p ⋅ q = (x12 + y12 )(x 22 + y 22 ) = (x1x 2 − y1y 2 ) 2 + (x1y 2 + x 2 y1 ) 2 ∈ M;
c) Presupunem că M' conŃine pe a ∈ N*, a > 1, atunci va conŃine şi a2, a3, ..., an, ... . Evident că acest şir nu este
periodic, căci dacă (∃) i, j ∈ N astfel încât ai = aj, ar urma a = 1, fals. În concluzie M' poate să conŃină numai pe 0
sau 1, adică M' = {0}, M' = {1}, M' = {0, 1}.
6. a) Pentru p = 2 avem n = 2[ n ] + 1 . Cum [ n ] ∈ N, atunci (∃) s ∈ N astfel încât n = 2s + 1, unde s = [ n ] .
Pe de altă parte, folosind definiŃia părŃii întregi, avem că s ≤ 2s + 1 < s + 1. Singurele numere naturale s care
verifică sunt s ∈ {1, 2} (calcul imediat). De aici, A = {3, 5}. b) Dacă p ≥ 3, p ∈ N, rezolvăm ecuaŃia n =
= p[ n ] + 1 . Notăm [ n ] = k ∈ N şi ecuaŃia revine la n = pk + 1. Folosind iarăşi definiŃia părŃii întregi, găsim
k ≤ n < k + 1 , de unde k2 ≤ pk + 1 < k2 + 2k + 1. Găsim k ∈ {p – 1, p}, de unde A = {p2 – p + 1, p2 + 1}.
7. Avem identitatea 8(x3 – 3x + 2) = (2x + 1)(4x2 – 2x – 11) + 27, prin urmare x ∈ A ⇔ (2x + 1) | 27, de
 4x + 1   2x − 1  5x − 4
unde x ∈ {–14, –5, –2, –1, 0, 1, 4, 13}. 8. EcuaŃia  +  = 3 se mai scrie sub forma
 6   3 
 2 x − 1 1   2 x − 1  5x − 4  2 x − 1  5x − 4
 3 + 2  +  3  = 3 , de unde, conform identităŃii lui Hermite, 2 ⋅ 3  = 3 . Astfel,
     

180
5x − 4 4x − 2 5x − 4 5x − 4  13 16 19 22 
≤ < + 1 şi ∈ Z , cu soluŃiile 2, , , ,  , de unde S2 = 16. Pe de altă parte,
3 3 3 3  5 5 5 5
x2 + 6 7 7
ecuaŃia din definiŃia primei mulŃimi se transcrie sub forma m = = x +1+ . Dar x + 1 + ∈ Z,
x −1 x −1 x −1
x ∈ Z, dacă x – 1 | 7, de unde x ∈ {–6, 0, 2,8}. De aici S1 = 16. În final, S1 = S2. 9. Fie a, b, c ∈ A trei elemente
distincte. Avem că a + b, b + c, c + a ∈ Q, de unde 2(a + b + c) ∈ Q, deci a + b + c ∈ Q. De aici, a = (a + b + c) –
– (a + b) ∈ Q. Cum x + a ∈ Q, (∀) x ∈ A – {a}, urmează că x ∈ Q, de unde A ⊂ Q. 10. a) Construim f: N → N
astfel: f(0) = 1, f(1) = 2, ..., f(2008) = 2009, f(2009) = 0 şi f(n) = n, (∀) n ∈ N – {0, 1, 2, ..., 2009}. Evident că
f ≠ 1N, f[2] ≠ 1N, ..., f[2009] ≠ 1N şi f[2010] = 1N. b) Fie a1, a2, ..., a2010 ∈ N cu a1 < a2 < ... < a2010. Considerăm f: N → N
definită astfel: f(a1) = a2, ..., f(a2009) = a2010, f(a2010) = a1, f(n) = n, (∀) n ∈ N – {a1, ..., a2010}. FuncŃia f satisface
cerinŃele din enunŃ. Cum numerele a1, a2, ..., a2010 pot fi alese într-o infinitate de moduri, deducem că mulŃimea A
este infinită. c) Să observăm că f ∈ A ⇒ f[2010] = 1N ⇒ f este inversabilă şi f–1 = f[2009] ; rezultă că f este bijectivă.
1 1 1 19
11. Notăm x = card(A ∩ B), y = card(A ∪ B). Atunci condiŃia se transcrie + + = , (1). Putem
x y 2 y 24
1 1 1 1 1 1 9 19
presupune că x ≤ y < 2y. Dacă x ≥ 4, + + y < + + = < . Rezultă că x ∈ {1, 2, 3}. i) Dacă x = 1,
x y 2 4 4 16 16 24
1 1 7
nu avem soluŃie. (Un membru supraunitar, unul subunitar); ii) Dacă x = 2, ajungem la ecuaŃia + y = , (2).
y 2 24
1 1 1 1 37 7
Dacă în (2) avem y ≥ 5, obŃinem + y ≤ + 2 = < , de unde y ∈ {1, 2, 3, 4}. Înlocuind în (2), nu
y 2 5 3 160 24
1 1 11
obŃinem soluŃii. iii) x = 3. În acest caz, avem de rezolvat + y = . Avem că y ≥ 5 nu convine şi găsim că
y 2 24
doar y = 3 este soluŃie. De aici, card(A ∩ B) = card(A ∪ B) = 3, de unde A = B. 12. A = {11k | k = 1,182 , k ≠ 0,
13, ..., 182}, iar B = {13k | k = 1,154 , k ≠ 0, 11, 22, ..., 154}. Atunci card A = 183 – 15 = 168, iar card B = 155 –
– 15 = 140. 13. a) A = {−2 − 2 , − 2 + 2 } , B = ∅. b) Cum 1 ∈ A, atunci 1 – a + b = 0. De asemenea, 2 ∈ B
impune 4 – 2b + c = 0. Scăzând membru cu membru cele două egalităŃi, se obŃine cerinŃa. c) Avem cazurile:
i) 1 ∈ A. Atunci a = b + 1. Cum, din Viete, 1 + x2 = a, urmează că x2 = b, deci A = {1, b} şi automat B = {a, c} =
= {b + 1, c}. Din Viete, avem că b + c + 1 = b, de unde c = –1. ObŃinem în acest caz că A ∪ B = {b + 1, 1, b, – 1},
cu b ∈ R* – {–1, 1}; ii) 1 ∈ B. Atunci b = c + 1 şi din x3 + 1 = b, deducem că x3 = c, deci B = {1, c}, de unde
A = {a, b}. Automat, din Viete, b = 0, c = –1. De aici A ∪ B = {a, 0, 1, –1}. 14. a) Observăm că
n
xn = 1+ 2 . Cum 1 < xn < 2, urmează că (∀) n = 1, 2008 , xn ∉ N, deci A ∩ N = ∅. b) Pentru a număra
n −n +2
n k
elementele lui A, căutăm acei n ∈ {1, 2, ..., 2008} astfel încât xn = xk. Atunci 1 + 2 = 1+ 2 . De
n −n+2 k −k+2
aici, (k – n)(nk – 2) = 0 şi, cum n ≠ k, obŃinem nk = 2, cu soluŃiile 1 şi 2. Urmează că x1 = x2, celelalte elemente
ale lui A fiind distincte. Astfel, card A = 2007; c) Calculăm xn – xn+1. Găsim că:
n2 + n − 2
x n − x n +1 = 2 > 0 , (∀) n ≥ 2. Urmează că xn+1 < xn, deci şirul este strict monoton
(n − n + 1)(n 2 + n + 1)
3
descrescător, (∀) n ≥ 2. Cel mai mare element este x1 = x 2 = . 15. Ipoteza A ∩ B ∩ C ≠ ∅ impune că există o
2
soluŃie reală α, comună celor 3 ecuaŃii. Scriind acest lucru şi adunând cele 3 relaŃii, obŃinem (a + b + c)(α2 + α +
+ 1) = 0. Cum α ∈ R, urmează obligatoriu a + b + c = 0, de unde α = 1, soluŃia căutată. Imediat, folosind relaŃiile

181
 c  a  b  a b c
lui Viete, obŃinem A = 1,  , B = 1, , C = 1,  , deci A ∪ B ∪ C = 1, , ,  . Pentru a demonstra
 a  b  c  b c a
cerinŃa card(A ∪ B ∪ C) = 4, arătăm că nu putem avea o soluŃie dublă pentru nicio ecuaŃie, iar ecuaŃiile nu mai
c
au altă soluŃie comună afară de 1. Dacă o ecuaŃie are o soluŃie dublă, urmează, de exemplu pentru A, că = 1 ,
a
de unde a = c. De aici găsim imediat b = –2. Folosind acest lucru în B, găsim c = –2, de unde c = a = b = –2,
adică obŃinem ecuaŃia x2 + x + 1 = 0, care nu are soluŃii reale. Dacă două ecuaŃii ar avea o soluŃie comună diferită
2
c a b b
de 1, găsim = , de unde a2 = bc. Cum a + b + c = 0, găsim (b + c)2 = bc, de unde   +   + 1 = 0 , ceea ce
a b c c
b
ar conduce la faptul că ∉ R.
c

II. Numere reale

2.1. MulŃimea numerelor întregi


1
1. a) Notăm a n = x n + . Avem a1 ∈ Z, a 2 = a12 − 2 ∈ Z, an+1 = a1 ⋅ an – an–1, (∀) n ≥ 2. Demonstrăm inductiv
xn
acest lucru. de aici, inductiv, an ∈ Z; b) Din a) avem a1 ∈ Z. Dacă a1 = 2l, l ∈ N* se arată inductiv că a2n este par.
Analog a1 = 2l + 1, l ∈ N se arată inductiv că a2n este par. 2. Presupunem x ∈ Z. Evident că atunci [x] = x şi
(n + 1) ⋅ nx (n + 1)[nx ]
relaŃia este imediată: [x] + [2x] + [3x] + ... + [nx] = x + 2x + ... + nx = = , (1). Reciproc:
2 2
dacă are loc (1), x ∈ Z. Pentru aceasta folosim metoda reducerii la absurd. Presupunem x ∈ R – Z. Atunci are
(n + 1)[nx ]
loc [x] + [–x] = –1, (2), (∀) x ∈ R – Z. ObŃinem [x] + [2x] + ... + [nx] = , (3) şi [–x] + [–2x] + ... +
2
(n + 1)[−nx ] n +1
+ [–nx] = , (4). Adunând membru cu membru (3) şi (4) şi Ńinând cont de (2) obŃinem –n = − ,
2 2
de unde n = 1, absurd. Urmează că x ∈ Z. 3. Notăm a = 2x + 3, b = 2x – 5. Cum x ∈ N, x > 2, a, b ∈ N. Cum a +
+ b ∈ N, a + b ∈ N, avem imediat a ∈ N, b ∈ N, de unde se obŃine x = 3, y = 4. Din condiŃia
2
z + z + 11 = t, z, t ∈ N, z + 11 = t − z . Notăm z + 11 = u , u ∈ N; z + 11 = u2, de unde z = u2 – 11, adică
u = t2 – u2 + 11, de unde 4u + 4u2 + 1 – t2 = 45, adică (2u + 1 –t)(2u + 1 + t) = 45. Analizând cazurile
2 u + 1 − t = 1 2 u + 1 − t = 3 2 u + 1 − t = 5
 , respectiv  sau  , de unde u ∈ {11, 4, 3}. Ultima valoare nu
2u + 1 + t = 45 2u + 1 + t = 15 2 u + 1 + t = 9
convine căci u ∈ N, z ∈ N, iar pentru u = 3, z = –2. De aici z ∈ {110, 5}. Evident x2 + y2 ≤ x2. 4. a = –(x1 + x2) ∈
∈ Z şi b = –n + x1x2 ∈ Z, deci na2 + b2 ∈ N. În plus na2 + b2 = ( x12 + n )( x 22 + n ) şi cum x12 + n ≥ 2, x 22 + n ≥ 2 ,
rezultă că na2 + b2 este compus. 5. Ordonăm numerele astfel încât a ≤ b ≤ c. Dacă a ≥ 3, atunci ab + ac + bc ≤
≤ bc + bc + bc ≤ 3bc ≤ abc, contradicŃie! Deci a < 3, a prim, obligatoriu a = 2. Din ipoteză 2bc < bc + 2(b + c),
1 1 1 1 1 2 1
adică + > . Dacă c ≥ b ≥ 5, atunci + ≤ < . Urmează că b < 5, de unde b ∈ {2, 3}. Dacă b = 2,
b c 2 b c 5 2
atunci c poate fi orice număr prim. Dacă b = 3, atunci c poate fi 3 sau 5.

182
6. Demonstrăm că [ n 2 + k ] = n , (∀) 1 ≤ k ≤ n, k ∈ N. Avem de arătat că n ≤ n 2 + k < n + 1 , ceea ce revine la
n2 ≤ n2 + k < n2 + 2n + 1, inegalitate evidentă pentru k = 1, n . Atunci N = n+
n + ...
  + n = n 2 , pătrat perfect.

n ori

1 x − 41
7. Cum = , atunci x > 41. Analog rezultă că y > 41. Fie prin urmare x = 41 + a, y = 41 + b, a, b ∈ N*.
y 41
1 1 1
EcuaŃia devine + = , echivalent, după efectuarea calculelor, cu ab = 412. Cum 41 este prim,
41 + a 41 + b 41
rezultă că 41 | a sau 41 | b. Dacă 41 | a ⇒ 41 ⋅ a' = a, a' ∈ N. De aici x = 41(1 + a') şi ecuaŃia
1 1 1 a ' = 1 a ' = 41
devine = − , de unde, după calcule, a'b = 41, deci  sau  . Se obŃin
41 + b 41 41(1 + a ' )  b = 41 b = 1
 x = 82  x = 1722  y = 1722
soluŃiile  ,  şi, în cazul în care 41 | b, soluŃia nouă  . 8. Avem că A =
 y = 82  y = 42  x = 42
2
1 2 1 2 1 1  10 n − 1 
= ⋅ 99 ... 9 − ⋅ 999 ...9 = (10 2n
− 1) − (10 n
− 1) = (10 2n
− 2 ⋅ 10 n
+ 1) = (10 n
− 1) 2
=  
9   9  9 9 9 9  3  , iar baza
2 n cifre n cifre  
pătratului este număr natural, egală cu 333...3 (n cifre). 9. a) Sn+2 = 5 ⋅ 5n+1 + 7 ⋅ 7n+1 = 12 ⋅ 5n+1 + 12 ⋅ 7n+1 – 7 ⋅
⋅ 5n+1 – 5 ⋅ 7n+1 = 12(5n+1 + 7n+1) – 35 ⋅ (5n + 7n) = 12Sn+1 – 35Sn, (∀) n ∈ N. b) Pentru n = 0, obŃinem S0 = 2 şi
S1 = 12 şi, cum 2 | 12, evident că S0 | S1. Vom arăta, folosind metoda reducerii la absurd, că Sn /| Sn +1 , (∀) n ≥ 1.
Presupunem pentru aceasta că Sn | Sn+1, (∀) n ≥ 1. Atunci există un k ∈ N astfel încât Sn+1 = k ⋅ Sn, adică, folosind
(a), 35Sn–1 = (12 – k)Sn. Cum 35 nu divide 12 – k, urmează că 35 | Sn, de unde 5 | Sn sau 7 | Sn, ceea ce este
imposibil, din definiŃia lui Sn. 10. a) De exemplu, putem considera a1 = 6, a2 = 62, ..., a100 = 6100. b) Avem că
n
1  n 1  n 1   1  3 1  3 1 1
∑ a <  ∑ 2i   ∑ 3i  =  2 − 2k   −
2 2⋅3  k 
< 2 ⋅ = 3 . (Am Ńinut cont că şirurile a i = i , b j = j , pro-
2
i =1 i  i =0   i =0  2 2
gresii geometrice, aplicând formula calculului sumei termenilor unei progresii geometrice). 11. Fie n dat, dar
arbitrar ales. Dacă m + n = n , atunci evident m + m + n ∈ N. De unde alegerea lui m: m = n2 – n. 12. În
primul rând este o problemă de dacă şi numai dacă. Presupunând a1 = a2 = ... = an, găsim x1 = 1, x2 = a1 cele două
soluŃii întregi. Reciproc: să presupunem că toate ecuaŃiile au soluŃii întregi. Atunci condiŃia necesară este ca
numărul ∆k = (ak + 1)2 – 4ak+1 să fie pătrat perfect pentru orice k = 1, 2, ..., n. Evident că (ak + 1)2 – 4ak+1 < (ak + 1)2.
Fie m = (ak + 1)2 – 4ak+1 şi n = (ak + 1)2. Cum m şi n au aceeaşi paritate, atunci (ak + 1)2 – 4ak+1 ≤ (ak – 1)2 < (ak + 1)2,
de unde ak ≤ ak+1, (∀) k = 1, n . De aici a1 ≤ a2 ≤ ... ≤ an ≤ an+1 = a1, prin urmare a1 = a2 = ... = an.
13. ÎmpărŃim cele 98 de numere astfel: A = {{98, 27}, {97, 28}, ..., {63, 62}, {26, 1}, {25, 2}, ..., {14, 13}},
Cum partiŃia făcută conŃine 49 de clase, dacă vom alege 50 de elemente ale lui A, două se vor afla ăn aceeaşi
clasă şi vor avea suma fie 125, fie 27, deci cub perfect. 14. EcuaŃia iniŃială se poate scrie sub forma E = 64x6 + 64x5 +
+ 256 = (8y)2. Arătăm că (8x3 + 4x2 – x)2 < 64x6 + 64x5 + 256 < (8x3 + 4x2 – x + 1)2, pentru aproape toate
valorile lui x. Avem: 64x6 + 64x5 + 8x3 + 9x2 – 2x + 1 < 64x6 + 64x5 + 256 < 64x6 + 64x5 + 8x3 + 9x2 – 2x + 1,
pentru |x| ≥ 3. Urmează că posibilele soluŃii se găsesc pentru x ∈ {–2, –1, 0, 1, 2}. SoluŃiile vor fi (x, y) ∈ {(–2, ±6),
(–1, ±2), (0, ±2), (2, ±10)}. 15. a) Trebuie să avem, condiŃie necesară, că a2 – 4b = k2, (1), respectiv b2 – 4a = l2,
(2), cu k, l ∈ Z. Din (1) şi (2) găsim a2 – b2 + 4(a – b) = (k – l)(k + l), adică (a – b)(a + b + 4) = (k – l)(k + l), (3).
O soluŃie a lui (3) este a = m, b = –m – 1, m ∈ Z, adică (a, b) ∈ {(m, –m – 1) | m ∈ Z}, ultima mulŃime
conŃinând o infinitate de elemente. b) Dacă x1, x2 sunt soluŃiile primei ecuaŃii, y1, y2 sunt soluŃiile celei de a doua,
din relaŃiile lui Viete, găsim x1 + x2 = y1y2 = a, (4); y1 + y2 = x1x2 = b, (5). Din (4) şi (5) obŃinem relaŃia
(x1 – 1)(x2 – 1) + (y2 – 1)(y1 – 1) = 2, (6). Cum x1, x2, y1, y2 ∈ N, din (6) observăm că avem cazurile x1 = 0 sau x2 = 0,

183
y1 = 0, y2 = 0, a = b = 0. Deci, dacă o soluŃie este nulă, toate sunt nule. Fie acum cazul când niciuna din soluŃii nu
este nulă. Atunci, din (6), avem situaŃiile: i) (x1 – 1)(x2 – 1) = 0 şi (y1 – 1)(y2 – 1) = 2 cu a = 6, b = 5; ii) (x1 –
– 1)(x2 – 1) = 1 şi (y1 – 1)(y2 – 1) = 1 cu a = b = 4; iii) (x1 – 1)(x2 – 1) = 2 şi (y1 – 1)(y2 – 1) = 0 cu a = 5, b = 6.

2.2. Numere raŃionale, numere iraŃionale


1. Avem ∆ = 4(n + 1)2(n2 + 1). Cum ∆ > 0 şi ∆ ≠ k2, cele două soluŃii ale ecuaŃiei sunt distincte şi iraŃionale.
2. Fie α soluŃie. atunci a – α2 = a + α . Cum α ∈ Q şi de asemenea a ∈ Q*+, a – α2 = a + α = p ∈ Q*+ ⇒ a +
+ α = p2 şi a – α2 = p. ObŃinem α = –p, nu convine şi α = p – 1, p ≥ 1 care convine. Imediat a = p2 – p + 1 cu p ≥
≥ 1, p ∈ Q. 3. a) Fie x 0 + 2009 = a şi x 0 + 2010 = b , cu a, b ∈ Q*+. De aici x0 + 2009 = a2, respectiv x0 +
m n
+ 2010 = b2. Dacă b > a avem cum b, a ∈ Q*+, b – a = , m, n ∈ N* şi b + a = , m, n ∈ N*, de unde b =
n m
2
m2 + n2 n 2 − m2  n 2 + m2 
= şi a = , n > m, n, m ∈ N*. Imediat x 0 =   − 2010 , unde n, m ∈ N*; b) Alegând

2mn 2mn  2mn 
x 0 = 10 p + 3 ⇒ x0 + 2010 are ultima cifră 3 şi respectiv x0 + 2009 are ultima cifră 2. Urmează că cele două
 5
numere sunt iraŃionale nefiind pătrate perfecte. 4. Evident pentru orice x ∈ Q – 3,  numerele sunt simultan
 3
2x 2 + 1
raŃionale. Căutăm x ∈ R – Q pentru care cele 2 numere sunt simultan raŃionale. Avem atunci = a ∈ Q şi
x −3
3x 2 − 1
= b ∈ Q cu soluŃia obŃinută din ecuaŃia 3x(a – 2b) = 9a – 10b + 5, (1). Cum x ∈ R – Q, atunci
3x − 5
 5
a − 2b = 0 a = − 4 − 5 ± 374
 cu soluŃiile  . De aici 8x2 + 5x – 11 = 0 cu soluŃiile x1, 2 = . În concluzie,
9a − 10b + 5 = 0 b = − 5 16
 8
  5    − 5 − 374 − 5 + 374 
x ∈  Q -  , 3  ∪  ,  . 5. a1 şi a2 au prima zecimală 5. Pentru n ≥ 3, 0,6 < a3 ≤ an ≤ 0,7 –
  3    16 16 
1
− (inducŃie). Rezultă că prima zecimală a numărului an, n ≥ 3 este 6.
4n
6. Avem 1 + p2 = pq + qr + pr + p2 = (p + r)(p + q). Analog se obŃine că 1 + q2 = (q + p)(q + r) şi
1 + r2 = (r + p)(r + q), prin urmare (1 + p 2 )(1 + q 2 )(1 + r 2 ) = (1 + p)2 (1 + q) 2 (1 + r)2 = (1 + p)(1 + q)(1 + r) , care
1 1  1 1 1
evident este din Q. 7. Avem [a ] +   + {a} +   = a + , de unde a + = [a] +   + 1 ∈ Z. Fie k ∈ Z astfel
a  a  a a a 
1 k ± k2 − 4
încât a + = k . De aici, a2 – ak + 1 = 0. Înseamnă că există a ∈ R, a = (1). Luând, de exemplu, k = 3,
a 2
3+ 5 1 
găsim a = , număr iraŃional cu proprietatea că {a} +   = 1. Arătăm în continuare că orice alt număr cu
2 a 
proprietatea din enunŃ este iraŃional. Cum a ∈ R, din (1) obŃinem k ∈ Z / {–1, 0, 1}. Dacă k ∈ {–2, 2}, obŃinem

184
1 
a ∈ {–1, 1}, care nu verifică {a} +   = 1 . Presupunem că a ∈ Q. Atunci (∃) l ∈ Z+ astfel încât k 2 − 4 = l , de
a 
unde k2 – 4 = l2, sau (k – l) ⋅ (k + l) = 4, prin urmare k ∈ {–2, 2}, caz analizat. Cu alte cuvinte există o infinitate
de valori pentru (1), a ∈ R – Q care verifică (1). 8. Plecăm de la observaŃia următoare: orice pătrat perfect este
de forma 3k sau 3k + 1, dar niciodată nu este de forma 3k + 2. Folosim metoda reducerii la absurd: presupunem
a 2 + 2008 2 p
că (∃) p, q ∈ N* astfel încât = , (p, q) = 1. Efectuând calculele, suntem conduşi la 3p2 = q2(a2 +
3 q
+ 20082). Cum (p, q) = 1, atunci 3 | a2 + 20082; dar 20082 = M3 + 1 şi ar rezulta că a2 = M3 + 2, imposibil!
9. Presupunem că a = n2, n ∈ ∈ N*. În acest caz E(n2) = {n} + {n}2 + {n}3 = 0, adică E(a) ∈ Q. Reciproc:
presupunem E(a) ∈ Q. Notăm cu k = [ a ] . Atunci E(a) = (–k + a + k2 – 3ak – k3) + a (3k 2 − 2k + a + 1) , (1).
Notăm α = –k + a + k2 – 3ak – k3, (2); β = 3k2 – 2k + a + 1, (3). În primul rând β ≠ 0. (Dacă β = 0, atunci
ecuaŃia 3k2 – 2k + a + 1 = 0 ar trebui să aibă soluŃii întregi, absurd, căci ∆ = –4 – 4a < 0.) Din această obser-
E (a ) − α
vaŃie, E(a) = α + a β , de unde a = ∈ Q, adică obligatoriu a = n2, n ∈ N. 10. a2 + b2 + c2 =
β
a 2b2 a 2b2 (a + b) 4 − 2ab(a + b) 2 + a 2 b 2 ((a + b) 2 − ab) 2
= a2 + b 2 + = a 2 + b 2 + 2 ab − 2 ab + = = =
(a + b ) 2
(a + b) 2
(a + b) 2 (a + b) 2
2
 (a + b)2 a ⋅ b 
= −  = (a + b – c) . De aici,
2
a 2 + b 2 + c 2 = (a + b − c) 2 =| a + b − c | ∈ Q. 11. Inductiv se arată
 a + b a + b 
că (a + b 2 ) m = a m + b m 2 şi (c + d 3 ) n = c n + d n 3 , cu cn, am, bm, dn ∈ Q; a m + b m 2 = cn + d n 3 ⇔
⇔ a m − cn + b m 2 = d n 3 ⇔ (a m − c n ) 2 + 2(a m − c n )b m 2 + 2b 2m = 3d 2n (1). RelaŃia (1) are loc doar dacă
(a m − c n )b m = 0 . De aici, am = cn sau bm = 0, de unde (a + b 2 ) m ∈ Q şi (c + d 3 ) n ∈ Q. Presupunem ab ≠ 0.
a = a ⋅ a m −1 + 2b ⋅ b n −1
Atunci ab > 0 sau ab < 0. Presupunem ab > 0. În acest caz,  m , de aici rezultând bm > 0,
b m = a ⋅ b m −1 + b ⋅ a m −1
absurd, căci am văzut că bm = 0. Urmează că ab = 0. Absolut analog se tratează cazul ab < 0. 12. i) Cum a ∈ Q*,
3
a 2 ∈ Q implică (3 a 2 ) 2 ∈ Q, de unde a 3 a ∈ Q şi, cum a ∈ Q, urmează că 3
a ∈ Q; ii) Din 3
a ⋅ 5 b ∈ Q,
rezultă ( 3 a ⋅ 5 b )5 ∈ Q, de unde b ⋅ a 3 a 2 ∈ Q, deci 3
a 2 ∈ Q şi, aplicând (i), avem 3
a ∈ Q. Revenind, cum
3
a ⋅ 5 b ∈ ∈ Q, 3
a ∈ Q, urmează că 5
b ∈ Q; (iii) Se procedează asemănător, ridicând la puterea a 7-a, apoi la
a 5-a. 13. Avem a1 = k 1 + a 0k , a 2 = 1 + a1k = k 1 + 1 + a 0k = k 2 + a 0k , iar din aproape în aproape, a n = k a 0k + n ,
k

(∀) n ∈ N. Pentru orice n ∈ {(a 0 + p) k − a 0k p ∈N}, avem an = a0 + p ∈ N, deci {an | n ∈ N} ∩ Q are o infinitate

de elemente. Dacă n ∈ N / { (a 0 + p) k − a 0k | p ∈ N}, atunci an ∈ R / Q, prin urmare şi {an | n ∈ N} ∩ (R / Q) are

n 2 − 3n + 2 + n 2 + n + 1 ∈ Q. Cum n 2 < n 2 + n + 1 < ( n + 1) ,


2
o infinitate de elemente. 14. Presupunem că

(1), urmează că n 2 + n + 1 ∈ R / Q. Atunci este iraŃional şi celălalt termen, n 2 − 3n + 2 . Fie


n 2 − 3n + 2 + n 2 + n + 1 = m , m ∈ Q; rezultă că n 2 − 3n + 2 = m − n 2 + n + 1 , adică n2 – 3n + 2 =
= (m − n 2 + n + 1) 2 , de unde ajungem la faptul că n 2 + n + 1 = s, s ∈ Q, absurd, conform cu (1). Urmează că

185
 x
n 2 − 3n + 2 + n 2 + n + 1 ∈ R / Q. 15. a) x > 0 şi x ∉ Q; atunci x =   ⋅ (2 x ) ; ii) x > 0 şi x ∈ Q;
 2 
 
x  | x | 
atunci x = ( 2 ⋅ x ) ⋅ ; iii) x < 0 şi | x | ∉ Q; luăm x =  − ⋅ (2 | x | ) ; iv) x < 0 şi | x | ∈ Q; luăm
2  2 

|x|
x = (− | x | ⋅ 2 ) ⋅ ; b) i) Dacă x ∉ Q, luăm x şi încă 1004 numere iraŃionale distincte şi opusele lor;
2
ii) Dacă x ∈ Q, luăm x + 3 2 , − 2, − 2 2 şi încă 1003 numere iraŃionale distincte, împreună cu opusele lor.

2.3. OperaŃii cu numere reale


1. Din condiŃie găsim a3b + b3c + c3a = a3c + b3a + c3b, relaŃie echivalentă cu (a – b)(b – c)(a – c)(a + b + c) = 0.
1
De aici a + b + c = 0. Concluzia este imediată Ńinând cont de relaŃia a3 + b3 + c3 – 3abc = (a + b + c)[(a – b)2 +
2
x+y 4 a1 + a 2 a 2 + a 3 a + a 20
+ (a – c) + (b – c) ]. 2. Din
2 2
≥ , (∀) x, y ∈ Q +, obŃinem că A =
*
+ + ... + 19 ≥
xy x+y a1a 2 a 2a 3 a19 a 20
 1 1 1  1 3 37 
≥ 4 + + ... +  . De aici, Ńinând cont de ipoteză, A ≥ 4  + + ... +  =
a
 1 + a 2 a 2 + a 3 a 19 + a 20   4 4 4
4[1 + 3 + ... + 37] n (n + 1)(2n + 1)
= = 361. De aici A ≥ 361 = 19 . 3. Cum = 12 + 22 + 32 + ... + n2, (∀) n ∈ N*,
4 6
relaŃia din enunŃ revine la (x1 – 1)2 + (x2 – 2)2 + ... + (xn – n)2 ≤ 0. De aici, imediat soluŃiile xn = n, (∀) n ∈ N*.
d−c b−a
4. a) Din n d − n c < n b − n a avem < . Cum
n n −1
n n −1
a n n−2
+ a n
⋅ c + ... + d ⋅ c n −2 n n −1
+ c d + ... + n c n −1
d – c = b – a, dp ⋅ cq > bp ⋅ aq, (∀) p, q ∈ N inegalitatea este evidentă; b) Vom arăta că A < 1 < B, de unde
n
3+n 5+n 7
A < B. Arătăm A < 1. Folosim pentru aceasta punctul a). Avem: < 1 este echivalentă cu
n
4 +n 5 +n 6
n
3 + n 7 < n 4 + n 6 . Notând a = 3, b = 4, c = 6, d = 7 şi aplicând a), obŃinem inegalitatea cerută. Analog B > 1.
5. Fie x, y, z, t > 0 o soluŃie, atunci xyzt = 16 + xy + xz + ... + zt ≥ 16 + 66 ( xyzt)3 , dci xyzt ≥
≥ 16 + 6 xyzt . Rezultă xyzt ≥ 8 . Dar 8 2 = x + y + z + t ≥ 44 xyzt , de unde 8 ≥ xyzt . Prin urmare
xyzt = 8 şi toate inegalităŃile sunt egalităŃi. Rezultă x = y = z = t = 2 2 .
6. i) Conform identităŃii lui Lagrange (a2 + b2)(c2 + d2) = (ad + bc)2 + (ac – bd)2, rezultă că are loc inegalitatea
CBS a 2 + b 2 ⋅ c 2 + d 2 ≥ ad + bc , cu egalitate pentru ac = bd. Analog, b 2 + c 2 ⋅ a 2 + d 2 ≥ ab + cd , cu
egalitate tot când ac = bd. Prin adunare, găsim că a 2 + b 2 ⋅ c 2 + d 2 + b 2 + c 2 ⋅ a 2 + d 2 ≥ ad + bc + ab + cd =
= (a + c)(b + d), cu egalitate pentru ac = bd, adică cerinŃa problemei. ii) Fie ABCD un patrulater ortodiagonal cu
AC ∩ BD = {O}. Dacă notăm OA = a, OB = b, OC = c, OD = d, relaŃia algebrică se transcriedub forma AB ⋅ CD +
+ AD ⋅ BC = AC ⋅ BD. ObŃinem celebra identitate a lui Ptolemeu, adică suntem conduşi la faptul că patrula-
terul este inscriptibil, iar în acest caz OA ⋅ OC = OB ⋅ OD (puterea punctului O faŃă de cerc). 7. Avem

186
1 1
2 + 2 2 < 3 + 2 2 = (1 + 2 ) 2 , de unde 2 + 2 2 < 1 + 2 , adică > . Analog obŃinem că
2+2 2 1+ 2
1 1 1 1 1 1 1
> , > şi, cum + + = 2 −1 + 3 − 2 +
4+2 6 2+ 3 6 + 2 12 3+ 4 2 +1 3+ 2 4+ 3
+ 4 − 3 = 2 – 1 = 1, rezultă inegalitatea din enunŃ. 8. Din relaŃia din enunŃ avem că a = 1 – b – c, de unde a +
+ bc = 1 – b – c + bc = 1 – b – c(1 – b) = (1 – b)(1 – c). Analog, c + ab = (1 – a)(1 – b) şi b + ac =
= (1 – a)(1 – c). ÎnmulŃind membru cu membru aceste egalităŃi, obŃinem (a + bc)(c + ab)(b + ac) = (1 – a)2(1 –
– b)2(1 – c)2 = (a + b)2(a + c)2(b + c)2. (Am Ńinut cont că a + b + c = 1). 9. Pare că ar trebui să folosim metoda
inductivă. De data aceasta... nu. Arătăm că fiecare din cei n + 1 termeni din membrul stâng este mai mic
n +1 n +1 n +1 n +1
decât cel corespunzător din membrul drept. Într-adevăr, = < = =
n+2 (n + 2) 2 2
(n + 2) − 1 (n + 1)(n + 3)
1 1 1 ⋅ 3 ⋅ 5 ⋅ ... ⋅ 4017
= şi analog se procedează pentru fiecare pereche de termeni. 10. ObŃinem că p = ⋅ ,
n+3 2 2 ⋅ 4 ⋅ 6 ⋅ ... ⋅ 4018
1 3 3⋅ 5 5⋅ 7 4015 ⋅ 4017 4017
de unde avem imediat că p 2 = ⋅ ⋅ ⋅ ⋅ ... ⋅ ⋅ . Arătăm că fiecare fracŃie de forma
4 22 4 2 6 2 4016 2 40182
(2k − 1)(2k + 1) (2k − 1)(2k + 1) (2k ) 2 − 1 1 3 4017 1
2
este subunitară. Într-adevăr, 2
= 2
< 1 . De aici p 2 < ⋅ ⋅ < ⋅
( 2k ) ( 2k ) ( 2k ) 4 4 40182 4
3 1 3 1 1  a b c  1
⋅ ⋅ = < . În final, 0 < p < , adică p = 0,0… . 11. a) 0 =  + +  +
4 2 ⋅ 2008 32 ⋅ 2009 10000 100  b − c c − a a − b  b − c
1 1  a b c a  1 1  b  1 1  c  1
+ + = + + +  + +  + +  +
c − a a − b  ( b − c ) 2 ( c − a ) 2 (a − b ) 2 b − c  c − a a − b  c − b  b − c a − c  a − b  b − c
1  a b c a b c a b
+ = + + − − − = + +
c − a  ( b − c) 2
(c − a ) 2
(a − b ) 2
(c − a)(c − b) (b − c)(b − a) (b − c)(c − a) (b − c) (c − a)2
2

c a (b − c) + b(c − a ) + c(a − b) a b c  a b c 
+ ⋅ = + +  
(a − b) 2
(a − b)(b − c)(c − a ) ( b − c) 2
(c − a ) 2
(a − b ) 2  (b − c) 2 + (c − a)2 + (a − b)2  ⋅
; b) 0 =
 
 1 1 1  a b c a  1 1  b  1 1 
⋅ + + = + + +  + +  + +
 b − c c − a a − b  (b − c) ( c − a ) 3 (a − b ) 3 ( b − c ) 2  c − a b − a  ( c − a ) 2  b − c a − b 
3

c  1 1  a b c c b
+  + = + + + + +
(a − b) 2  b − c c − a  (b − c)3 (c − a )3 (a − b) 3 (a − b)(b − c)(c − a ) (a − b)(c − a )(b − a )
c
+ . 12. Aproape automat ne gândim la folosirea unei inegalităŃi. Cum a, b, c, d sunt oarecare
(a − b)(c − a )(b − a )
din R, sigur că prima la care vom apela este CBS. Avem: (a + b + c)2 < (12 + 12 + 12)(a2 + b2 + c2), de unde
(6 – d)2 ≤ 3(12 – d2), deci 4d2 – 12d ≤ 0, adică d(d – 3) ≤ 0, ceea ce conduce la d ∈ [0, 3]. Analog rezultă că a, b,
y2 y2
c ∈ [0, 3]. 13. Notăm A = x2 + xy + = 25 , B = + z 2 = 9 , C = z 2 + xz + x 2 = 16 , D = xy + 2yz + 3zx.
3 3
3x y 2
Cum A – B – C = 0, rezultă că xy = x(2z + x), (1). Dar D = y(x + 2z) + 3xz, de unde D = ( +
z 3

187
(1)
3x 27x
+ z 2) = ⋅9 = . Pe de altă parte, D = xy + 2yz + 3zx = 2zx + x 2 + 2 yz + 3zx = x (2x + y + z) . Însă D =
z z
27x 27 x
= x(2x + y + z) = , de unde 2 x + y + z = 2 . Dar A – B + C = 32, de unde x(2x + y + z) = 32, deci 2x +
z z
32 27 x 2 x x 8 3
+ y + z = . Găsim că 32 = . Cum > 0 , din ultima relaŃie = , prin urmare D =
x 2z 2 2 z 9
x 8 3
= 27 ⋅ = 27 ⋅ = 24 3 . 14. Pregătim CBS. Pentru aceasta, observăm că a2 + b2 + c2 + d2 + e2 + f2 – 2(a + b +
2 9
+ c + d + e + f) = 0, de unde a2 + b2 + c2 + +d2 + e2 + f2 = 20, de unde a2 + b2 + c2 + d2 + e2 = 20 – f2. Dar,
conform cu CBS, (a + b + c + d + e)2 ≤ (12 + 12 + 12 + 12 + 12)( a2 + b2 + c2 + d2 + e2), de unde (10 – f)2 ≤
 10  10
≤ 5(20 – f2). În final, f(3f – 10) ≤ 0 de unde f ∈ 0,  . În concluzie, max f = şi se atinge pentru a = b = c =
 3 3
4 x1 1 − y1 1
= d = e = . 15. Notăm yk = 1 – xk, k = 1, 2, ..., n. Atunci = = −1 ;
3 1 − x1 y1 y1
x2 1 − y2 1 1 x3 1 − y3 1 1
= = − ; = = − . Adunând
(1 − x1 )(1 − x 2 ) y1 − y 2 y1y 2 y1 (1 − x1 )(1 − x 2 )(1 − x3 ) y1y2 y3 y1 − y2 y3 y1 y 2 ...y n −1
1
membru cu membru, obŃinem că E = − 1 , de unde (1 + E) ⋅ y1y2...yn = 1, adică (1 + E)(1 – x1)(1 – x2)
y1 y 2 ...y n
... (1 – xn) = 1.

2.4. Numere reale. Modulul unui număr real

1. Notăm p = [x] şi t = {x}, (1). Dacă x ≥ 0, p ⋅ t = (p + t)2, de unde p = t = 0 este singura soluŃie reală a ecuaŃiei
2
 t 3t 2
p +  + = 0 . De aici x = 0 este soluŃie. Dacă x < 0, ecuaŃia devine pt = –(p + t)2 care se reduce la ecuaŃia
 2 4
t −3− 5
omogenă p2 + 3pt + t2 = 0, (2). Notăm y = , p ≠ 0. De aici y2 + 3y + 1 = 0 cu soluŃiile y1 = , y2 =
p 2
−3+ 5 −3+ 5
= , de unde, revenind în (1), pentru t = p ⋅ , găsim, Ńinând cont că t ∈ (0, 1], că p ∈
2 2
 3+ 5  3− 5 1− 5
∈ − , 0 . Cum p ∈ Z, urmează că p = –1. Pentru p = –1, t = , de unde x = altă soluŃie.
 2  2 2

−3− 5
Analog pentru t = p ⋅ , găsim p = –2, de unde x = 1 − 5 . În concluzie soluŃiile ecuaŃiei sunt x ∈ {1 –
2
1 − 5 
− 5, , 0 . 2. Avem |f(2) – f(–2)| ≤ |f(2)| + |f(–2)| ≤ 4, de unde |4a + c| ≤ 1, (1). Analog |f(1) – f(–1)| ≤ 4
2 
rezultă |a + c| ≤ 2, (2). Din (1) şi (2) găsim 3a ≤ 3 şi cum a ∈ N*, avem a = 1. Din (1) c ∈ [–5, –3] iar din (2), c ∈
∈ [–3, –1], de unde c = –3. Pe de altă parte 4 = |f(1) – f(–1)| ≤ |f(1) + f(–1)| ≤ 4. De aici |f(1) + f(–1)| = |f(–1) –

188
– f(–1)|. Prin urmare f(1) = 2, f(–1) = –2 sau f(1) = –2, f(–1) = 2. Prin urmare b + d = 0. La fel 4 ≤ |f(2) – f(–2)| ≤
≤ |f(2) + f(–2)|, de unde f(2) = 2, f(–2) = –2 sau f(2) = –2, f(–2) = 2. Rezultă că 4b + d = 0. În final găsim a = 1,
2(a 2 + b2 ) + 2 | a 2 − b2 |
b = 0, c = –3, d = 0. Se verifică că |x3 – 3x| ≤ 2, (∀) x ∈ [–2, 2]. 3. Avem succesiv 2 ≤ ≤
a 2 + b2
≤ 4, de unde imediat se revine la inegalitatea evidentă 0 ≤ |a2 – b2| ≤ a2 + b2.
4. Folosim inegalitatea |x – k| + |k – x + 2009| ≥ 2009, (1) pentru k ∈ {1, 2, ..., 1004}. Rezultă că E(x) =
1004 1004
= ∑
k =1
(| x − k | + | k − x − 2008 |) ≥ 1004 ⋅ 2009 , egalitatea realizându-se pentru x ∈ ∩[k, 2009− k] = [1004, 1005].
k =1

Aşadar, min E( x ) = 2017036 . 5. Folosim metoda reducerii la absurd: presupunem că max{x12 − x 2 , x 22 − x 3 ,


x∈R
1 1 1
..., x 2n − x1} < −
. De aici obŃinem, făcând pe rând în (1), k = 1, 2, ..., n că: x12 − x 2 + < 0 , x 22 − x 3 + < 0 , …
4 4 4
1 1 1 1
x 2n − x1 + < 0 . Adunând membru cu membru, găsim că x12 − x1 + + x 22 − x 2 + + ... + x 2n − x n + < 0 , deci
4 4 4 4
2 2 2 2
 1 1  1  1 2 2 1
 x1 −  + (x 2 −  +  x 3 −  + ... +  x n −  < 0 , absurd. De aici avem max{x1 − x 2 , ..., x n − x1} ≥ − 4 .
 2 2  2  2
6. Aplicând proprietatea modulului, avem că |1 + ab| + |a + b| ≥ |1 + ab + a + b|, respectiv |1 + ab| + |a + b| ≥ |1 +
ab – (a + b)|. ÎnmulŃind membru cu membru, găsim că (|1 + ab| + |a + b|)2 ≥ |(1 + ab)2 – (a + b)2| = |a2 – 1| ⋅
⋅ |b2 – 1|, de unde |1 + ab| + |a + b| ≥ | a 2 − 1 | ⋅ | b 2 − 1 | . 7. Cazuri: i) a = b ; atunci A < B, A = B sau A > B,
b
după cum m > n, m = n, respectiv m < n; ii) a < b. Atunci, notând x = > 1 , problema se reduce la a compara
a
m
numerele A1 = m 1 + n x şi B1 = n 1 + m x . Avem: A1 < B1 ⇔ (1 + n x ) n < (1 + m x ) m ⇔ 1 + n x < (1 + m x ) n ⇔
1
⇔ [(1 + m x )m ] n − n x > 1 . Folosind formula an – bn = (a – b)(an–1 + an–1 ⋅ b + an–2 ⋅ b2 + ... + bn–2 ⋅ a + bn–1),
n −1 m −1
obŃinem ∑
k =0
m
(1 + m x ) m ( n − k −1) ⋅ x k < (1 + m x ) m − x = (1 + m x ) m − (m x ) m = ∑ (1 +
k =0
m
x ) m − k −1 m x n . Avem deci

m ( n − k −1) k k
inegalitatea finală (1 + m
x) n < (1 + ⋅xn m
x ) m − k −1 ⋅ x m
. Atunci avem concluziile: Dacă m = n, atunci
A = B, dacă m > n atunci A < B iar dacă m < n atunci A > B. 8. a) După un calcul sumar, expresia E se poate
|a − b |⋅| b −d |⋅| c − a |
scrie sub forma E = de unde, Ńinând cont de inegalitatea triunghiului |b – a| < c şi
abc
analoagele, găsim E < 1. De aici, α = 1. b) Vom considera triunghiul ABC de laturi a = ε, b = 1 + ε2, c = 1 + ε,
(∀) ε ∈ (0, 1). Avem de verificat că, astfel alese, numerele a, b, c sunt laturi de triunghi. Verificăm că are loc
inegalitatea triunghiului: a + b > c, de unde 1 + ε2 + ε > 1 + ε, inegalitate adevărată conform cu (1). Analog 1 +
2
1− ε  1− ε 9
+ 2ε > 1 + ε2 şi 1 + ε2 + 1 + ε > ε. În acest caz, după calcule ajungem la   > 0,81 , de unde > ,
1+ ε  1 + ε 10
1
care conduce la ε < . Putem alege ε într-o infinitate de moduri, deci există o infinitate de triunghiuri care
19

189
 − x 2 − 2 x , x < −1
 2
− x + 2, − 1 ≤ x < 0
respectă condiŃia dată. 9. Explicitând modulele, rezultă că |1 – x| + x|x| + |1 + x| =  .
2
x + 2, 0 ≤ x < 1
 2
x + 2 x, x ≥ 1
Rezolvând în fiecare caz şi analizând soluŃiile, obŃinem S = {1}. 10. Notăm 9a + 6x + 4b = u, 4a + b + 2x = v,
a + b + x = w şi căutăm α, β, γ ∈ R pentru care αu + βv + γw = x. De aici 9α + 4β + γ = 0, 4α + β + γ = 0, 6α +
+ 2β + γ = 1, de unde α = 3, β = –5, γ = –7 şi astfel |x| = |αu + βv + γw| ≤ |α| ⋅ |u| +
+ |β| ⋅ |v| + |γ| ⋅ |w| = 3|u| + 5|v| + 7|w| ≤ 15, de unde |x| ≤ 15. 11. Metoda reducerii la absurd. Presupunem că
9 9 9 9
min(2a + b – c2, 2b + c – a2, 2c + a – b2) > , de unde 2a + b – c2 > , 2b + c − a 2 > şi 2c + a – b2 > .
4 4 4 4
Adunând membru cu membru, ajungem la (2a – 3)2 + (2b – 3)2 + (2c – 3)2 < 0, absurd. Atunci are loc
relaŃia din ipoteză. 12. Folosind proprietăŃile modulului, găsim: |x + a + b| + |x – a – b| ≥ 2a + 2b; |y + b + c| +
a +b a+c b+c
+ |y – b – c| ≥ 2b + 2c; |z + a + c| + |z – a – c| ≥ 2a + 2c; 3  + +  ≥ 3( ab + ac + bc) ≥ 4(a + b +
 2 2 2 
+ c), de unde a + b + c ≤ 0; absurd, conform ipotezei a + b + c > 0.

2.5. Parte întreagă. Parte fracŃionară

1. Se arată că [ 4n 2 + k ] = 2n , k = 1, 4n , [ 4n 2 + k ] = 2n + 1, k = 4n + 1, 8n + 3 , [ 4n 2 + k ] = 2n + 2 , k =
= 8n + 4,12n , n ∈ N*. EcuaŃia se transcrie 2n ⋅ 4n + (2n + 1)(4n + 3) + (2n + 2)(4n – 3) = 117, cu soluŃia n = 2.
2. Fie x = [x] + {x}. EcuaŃia devine (5{x} – 1)2 = 10[x], (1). Deoarece {x} ∈ [0, 1), Ńinând cont de (1), 10[x] ∈
1 1 1
∈ [0, 16), de unde [x] ∈ {0, 1}. Dacă [x] = 0, avem (5{x} – 1)2 = 0, de unde {x} = şi x + = . Dacă [x] = 1,
5 5 5
10 + 1 10 + 1 6 + 10
avem (5 ⋅ {x} – 1)2 = 10, de unde {x} = , iar x = 1 + = . 3. Notăm k = [x2 – 4x + 4] =
5 5 5
x 2 − 4 x + 3 < k ≤ x 2 − 4x + 4
= [–2x + 8x – 6]. Folosind definiŃia părŃii întregi, găsim:  , de unde k ∈
− 2x 2 + 8x − 7 < k ≤ −2x 2 + 8x − 6

 1 2 4− 2 4+ 2 
∈  − ,  
 . Cum k∈Z, urmează k = 0. Găsim [x – 4x + 4] = [–2x + 8x – 6] = 0, cu soluŃia x ∈
2 2
,
 3 3  2
 2 

 π 2010 + 1   π 2010
4. a) Inegalitatea lui Hermite! (vezi orice manual clasa a IX-a); b) Folosind a), obŃinem:  = +
 2   2
1  π 2010   π 2010 + 2   π 2010 1   π 2010   π 2010   π 2010 + 2 n −1   π 2010
+  = [π 2010 ] −  ;  = + = −  ; ............. ;  = n +
2  2   4   4 2   2   4   2n   2
1  π 2010  π 2010   π2010 + 1   π2010 + 2 
+  = [ n −1 ] −  n  , (1). Adunând membru cu membru în (1) găsim că: 
2 +  + ... +
2  2   2   4 

190
 π 2010 + 2 n −1  2010  π 2010   π 2010 
+ n  = [π ] −  n  . Dar  n  = 0 , dacă n ≥ 4020, ceea ce demonstrează relaŃia cerută.
 2   2   2 
5. Rezolvarea se bazează pe următoarea idee: alegem n = 4k, n = 4k + 1, n = 4k + 2, n = 4k + 3. Apoi verificare
pentru fiecare caz în parte: n = 4k ⇒ 4k2 + k + 4k2 + 3k + 4k2 + 5k + 1 + 4k2 + 7k + 3 = 16k2 + 16k + 4 = (4k + 2)2.
6. EcuaŃia se scrie sub forma {2009x} = – [x]. Deoarece {2009x} ∈ [0, 1), rezultă că [x] ∈ (–1, 0], de unde [x] = 0,
k
adică x ∈ [0, 1). Astfel, {2009x} = 0, care conduce la x = , k ∈ Z. Deoarece x ∈ [0, 1), rezultă că k ∈ {0,
2009
k
1, 2, ..., 2008}. SoluŃiile vor fi de forma x = , k ∈ {0, 1, ..., 2008}. 7. Notăm [a] = m, [b] = n, [c] = p, {a} = x,
2009
(m + x )m + (p + z)z − ny = 0,16 (1)

{b} = y, {c} = z. Sistemul se rescrie sub forma (n + y)n + (m + x ) x − pz = 0,25 (2) . Adunând (1) cu (2), găsim
(p + z)p + (n + y) y − mx = 0,49 (3)

n2 + z2 + (m + x)2 = 0,41. Cum n ∈ Z şi 0 ≤ n2 ≤ 0,41, rezultă că n = 0. Absolut analog, adunând (1) cu (3)
obŃinem m2 + y2 + (p + z)2 = 0,65, de unde m = 0. În final, adunând (2) cu (3) ajungem la p2 + x2 + (n + y)2 =
z 2 + x 2 = 0,41

= 0,74, cu aceeaşi consecinŃă p = 0. Sistemul se rescrie sub forma  y 2 + z 2 = 0,65 , de unde, adunând membru
 2 2
x + y = 0,74
cu membru, ajungem la x2 + y2 + z2 = 0,90, cu soluŃiile posibile x2 = 0,25, y2 = 0,49; z2 = 0,16. Cum x, y, z ∈ [0, 1),
x − 2  2x − 1
găsim x = 0,5; y = 0,7; z = 0,4 iar a = 0,5; b = 0,7; c = 0,4. 8. Notăm  =α,   = β cu α, β ∈ [0, 1).
 2   4 
x − 2 2x − 1 x  3 5
EcuaŃia devine α + β = + − − x + 2 . Cum 0 ≤ α + β < 2, avem că x ∈  − ,  , (1). Pe de altă
2 4 2  2 2
 x − 2  2x −1 x x 2k + 4
parte  +  = + x − 2 , de unde 2 + x − 2 ∈ Z. De aici x = 3 , cu k ∈ Z şi, Ńinând cont de (1),
 2   4  2
3 2k + 4 5  4 2 2 4 
obŃinem − < ≤ , de unde k ∈ {–4, –3, –2, –1, 0, 1}, cu soluŃiile x ∈ − , − , 0, , , 2 . 9. a) [S1] =
2 3 2  3 3 3 3 
= [1]; [S2] = [1 + 2 ] = 1 + [ 2 ] = 1 + 1 = 2 ; [S3 ] = [1 + 2 + 3] = 4 ; [S4 ] = [1 + 2 + 3 + 4 ] = 6; b) (⇐) dacă
n ∈ {1, 2}, atunci, din a) avem S1 = 1, S2 = 2, adică Sn = n, n ∈ {1, 2}. (⇒) Dacă n ≥ 3, Sn =
(a )
= (1 + 2 + 3 ) + 4 + 5 + ... + n ≥ 4 + 1
+ + ...
1 + 1 = n + 1 . De aici, [Sn] > n, (∀) n ≥ 3. Cum [Sn] = n,
n − 3 ori

urmează că n ∈ {1, 2}; c) Deoarece x ∈ N, x + [S2009–x] = 2009, adică [S2009–x] = 2009 – x, (1). RelaŃia (1) legată
de punctul b) conduce la 2009 – x = 1 sau 2009 – x = 2, cu soluŃiile x ∈ {2007, 2008}. 10. a) Fie x ≥ 0, y ≥ 0.
Atunci [x] ≤ x, [y] ≤ y, de unde [x] ⋅ [y] ≤ xy, (1). Pe de altă parte, cum x ≤ y, x, y ∈ R ⇒ [x] ≤ [y], aplicând
acest lucru în (1) găsim [[x] ⋅ [y]] ≤ [xy], de unde [x] ⋅ [y] ≤ [xy]. Cu aceasta, proprietatea cerută este
demonstrată; b) Aplicăm a) la relaŃia k4 + 4k3 + 5k2 + 4k + 1 = (k2 + k + 1)(k2 + 3k + 1), de unde
1 1
≤ , (3). Pe de altă parte, cum k2 < k2 + k + 1 ≤ (k + 1)2, iar
k + 4k + 5k + 4k + 1 [ k + k + 1] ⋅ [ k + 3k + 1]
4 3 2 2 2

(k + 1)2 ≤ k2 + 3k + 1 < (k + 2)2, deducem că [ k 2 + k + 1] = k şi [ k 2 + 3k + 1] = k + 1 , de unde (1) se

191
1 1
retranscrie sub forma: ≤ , (2), (∀) k ∈ N*. Dând valori lui k şi sumând în (4)
[ k + 4 k + 5k + 4 k + 1 ]
4 3 2
k (k + 1)
1
membru cu membru, găsim S(n) ≤ 1 – , de unde concluzia: S(n) ∈ (0, 1). 11. Vom folosi următoarele
n +1
observaŃii: i) dacă t ∈ Z, [t] + [–t] = 0; ii) dacă t ∉ Z, [t] + [–t] = –1. Revenind la problema noastră, dacă x ∈ Z,
atunci [–2x] + [–x] + [x] + [2x] + 1 = 0 + 0 + 1 = 1 ≠ 0, adică ecuaŃia nu are soluŃii întregi. Dacă x, 2x ∉ Z,
atunci [–2x] + [–x] + [x] + [2x] + 1 = –1 – 1 + 1 = –1 ≠ 0. În fine, dacă 2x ∈ Z, dar x ∉ Z, atunci [–2x] + [–x] +
+ [x] + [2x] + 1 = 0 – 1 + 1 = 0. În concluzie, toate numerele x ∉ Z cu 2x ∈ Z verifică ecuaŃia. Prin urmare,
 2n + 1 
mulŃimea soluŃiilor este  n ∈ Z  . 12. Determinarea părŃii întregi este imediată dacă Ńinem cont de dubla
 2 
inegalitate n < n 2 + n < n + 1, (∀) n ∈ N*, de unde [ n 2 + n ] = n . Pentru a determina prima zecimală a

numărului n 2 + n , observăm că pentru n = 1, 2 = 1,4... , pentru n = 2, 6 = 2,4... , de unde intuim că (∀) n ∈ N*


rezultă că n 2 + n = n,4.... Pentru a demonstra că prima zecimală a numărului n 2 + n este 4, vom demonstra
că n + 0,4 < n 2 + n < n + 0,5. Într-adevăr, n 2 + n ≥ n + 0,4 conduce la n ≥ 0,8, adevărat pentru (∀) n ∈ N*;
n 2 + n < n + 0,5 conduce la inegalitatea evidentă 0,25 > 0, (∀) n ∈ N*. Cu aceasta am demonstrat că 4 este
x 
prima zecimală a numărului n 2 + n . 13. a) Parcurgem cu x întreaga axă reală. i) Dacă x ∈ (–∞, 0),   < 0 ,
2
x 
iar x2 + 1 > 0, deci nu există soluŃie în (–∞, 0); ii) x = 0 nu verifică; iii) Dacă x ∈ (0, 2),   = 0 , pe când x2 + 1 > 1
2
x
≠ 0, deci nici în acest caz nu avem soluŃie; iv) Dacă x ∈ [2k, 2k + 2), k ∈ N*,   = k şi x2 + 1 ≥ 4k2 > k, (∀) k ∈ N*,
2
aşadar ecuaŃia nu are soluŃii reale; b) Absolut analog, găsim y = 0 soluŃie unică. 14. Pentru k ≥ 1, avem
[ p ] = k ⇔ k2 ≤ p < (k + 1)2. ObŃinem că [ p ] = k , (∀) p ∈ [k2, (k + 1)2), deci există 2k + 1 numere naturale
( k +1) 2 −1 n 2 −1 n n −1 n n −1
p cu [ p ] = k şi atunci ∑ 2
[ p ] = k (2k + 1) . Astfel, S = ∑
p =1
[ p] + ∑
k =1
k2 = ∑
k =1
k (2k + 1) + ∑
k =1
k2 = ∑ 2k
k =1
2
+
p=k
n n −1 n n
n (n + 1)(2n + 1) n (n + 1)
+ ∑
k =1
k2 + ∑
k =1
k =3∑ k =1
k2 + ∑ k − 2n
k =1
2
− n = 3⋅
6
+
2
− 2n 2 − n = n(n + 1)2 – 2n2 – n = n3 +

+ 2n2 + n – 2n2 – n = n3. 15. EcuaŃia se transcrie: ([x] + k) ⋅ {x} = x ⋅ k. Fie acum x = n + α, cu n ∈ Z, α ∈ [0, 1).
ObŃinem că (n + k)α = (n + α)k, de unde n(k – α) = 0. Dacă n = 0, x ∈ (0, 1) este soluŃie. Dacă k = 0, x = 0. În
concluzie, soluŃiile sunt x ∈ [0, 1) ∪ Z, sau x ∈ [0, 1), k ∈ Z*; 16. Notăm y = x – a şi cum |[y]| ≥ 0, din |[y]| +
+ [|y|] = 1, avem că [|y|] ∈ {0, 1}. Dacă [|y|] = 0, rezultă că |y| ∈ [0, 1), de unde y ∈ (–1, 1). Din ecuaŃia |[y]| = 1
suntem conduşi la [y] = 1 sau [y] = –1, de unde y ∈ [1, 2) care convine, sau y ∈ [–1, 0) care conduce la soluŃia
finală y ∈ (–1, 0), (1). ii) [|y|] = 1; rezultă că |y| ∈ [1, 2), de unde y ∈ (–2, –1] ∪ [1, 2), (2), iar din ecuaŃia
|[y]| = 0, adică [y] = 0, de unde y ∈ [0, 1). łinând cont de (2) în acest caz nu avem soluŃie. În final, (1) va fi
soluŃia căutată, deci x ∈ (a – 1, a), a ∈ R.

192
2.6. InegalităŃi clasice remarcabile
k 1
1. Aplicăm pentru inegalitatea mediilor pentru 1, k2 ∈ N, k ∈ N*. ObŃinem 1 + k2 ≥ 2k sau ≤ , (1), k =
1+ k2 2
= 1, n , n ∈ N*. Făcând în (1), pe rând, k = 1, 2, ..., n, înmulŃind membru cu membru relaŃiile obŃinute, găsim
rezultatul dorit. 2. Este celebra inegalitate a lui Nessbitt. Căutăm o demonstraŃie: Notăm x = a + b, y = b + c, z =
x+z−y x+ y−z y+z−x x y
= c + a. Inegalitatea se transcrie, Ńinând cont că a = ,b = ,c = , sub forma + +
2 2 2 y x
x z y z m n
+ + + + ≥ 6 , inegalitate evidentă, căci + ≥ 2 , (∀) m, n ∈ R*+, din inegalitatea mediilor.
z x z y n m
3. a) Folosind inegalitatea lui Cauchy-Schwarz obŃinem: (a1 + 2a2 + 3a3 + ... + nan)2 ≤ (a12 + a 22 + ... + a 2n )(12 +
n (n + 1)(2n + 1)
+ 22 + ... + n2). Cum a12 + a 22 + ... + a 2n = , avem inegalitatea cerută; b) Folosind aceeaşi inegali-
6
a2 a2 a2 (a1 + a 2 + ... + a n ) 2 a2 a2 a 2 (a + a 2 + ... + a n ) 2 n (n + 1)
tate, 1 + 2 + ... + n ≥ , obŃinem 1 + + 2 + ... + n ≥ 1 = .
x1 x 2 x n x1 + x 2 + x 3 + ... + x n 1 2 n 1 + 2 + ... + n 2
1 11 1
4. a) Din ≤  +  obŃinem (x – y)2 ≥ 0, inegalitate evidentă; b) Folosind a) obŃinem:
x + y 4  x y 
ab ab 1 1  ab ab 
= = ≤  + . Procedând analog cu fiecare termen şi adunând
a + b + 2c a + c + b + c a + c + b + c 4  a + c b + c 
ab ab
membru cu membru inegalităŃile, obŃinem inegalitatea cerută. 5. Avem:
n +1 n +1 n +1 n +1 1
= < = = . Procedând analog cu fiecare termen al sumei
n+2 (n + 2) 2
(n + 2) − 12 (n + 1)(n + 3) n+3
din membrul stâng şi adunând aceste inegalităŃi, găsim inegalitatea cerută.
1
a 2009 + 2009 2009
1 b a
6. a) RelaŃia (ab)2009 + 1 = 2b2009 conduce la a 2009 + 2009 = 2 . Însă 1 = ≥   , de unde
b 2 b
2009
a
  ≤ 1 , (1). Cum a, b > 0, din (1) urmează că a ≤ b; b) Din a) avem a ≤ b, de unde rezultă că an ≤ bn, pentru
b
 1   1  1
orice număr natural nenul n. Atunci 1 + n (1 + b n ) ≥ 1 + n (1 + a n ) = 1 + 1 + a n + n ≥ 2 + 2 = 4 . (Am folosit
 a   a  a
1 4 9 2
faptul că x + ≥ 2 , (∀) x ∈ (0, ∞)). 7. a) SoluŃie 1. Notăm A = 2 , B = y , C = 4z 4 . Evident A, B, C > 0.
x 9x 16
1 1 1
Aplicând inegalitatea dintre media aritmetică şi cea armonică, găsim (A + B + C)  + +  > 9 , adică
A B C
4 9
inegalitatea de demonstrat. Egalitatea are loc pentru A = B = C, adică pentru = = 4z 4 . SoluŃia 2.
9x 2 16 y 2
2 3 3x 4y 1
Aplicăm inegalitatea CBS numerelor a1 = ,a2 = , a 3 = 2z 2 , b1 = , b2 = , b 3 = 2 . ObŃinem că
3x 4y 2 3 2z

193
(a12 + a 22 + a 32 )( b12 + b 22 + b 32 ) ≥ (a1b1 + a 2 b 2 + a 3 b 3 ) 2 . De aici inegalitatea cerută; b) 24x3 – 48x2 + 24x +
+ 12x2 – 24x + 12 + 5 = 24x(x2 – 2x + 1) + 12(x2 – 2x + 1) + 5 = 24x(x – 1)2 + 12(x – 1)2 + 5 > 0, (∀) x > 0;
a b c a b c
c) Inegalitatea dintre media aritmetică şi cea geometrică aplicată numerelor , , conduce la + + > 3 ,
b c a b c a
a 2 b 3 c 2 2 3 2
(∀) a, b, c > 0. Notăm = , = , = 2x . ObŃinem + + 2x ≥ 3 , de unde 8 + 9 + 24x3 ≥ 36x ⇔
b 3x c 4 x a 3x 4 x
⇔ 24x3 – 36x + 17 ≥ 0. 8. Fie x, y > 0 şi x ⋅ y = p = constant. Folosind inegalitatea mediilor, găsim că x + y ≥
≥ 2 xy = p + p , deci suma este minimă când avem x = y = p . (Evident, cum x, y > 0, p > 0).
9. Inegalitatea se scrie sub forma tg x – 2tg x + 1 + tg y – 2tg y + 1 + tg2 z – 2tg z + 1 ≤ 0, sau (tg x –
2 2

– 1)2 + (tg y – 1)2 + (tg z – 1)2 ≤ 0, (1). Inegalitatea (1) este posibilă doar pentru tg x = 1, tg y = 1, tg z = 1,
π π π
egalităŃi care conduc la soluŃia problemei: x = + k1π, y = + k2π, z = + k3π, k1, k2, k3 ∈ Z.
4 4 4
x y z
10. a) Rescriem inegalitatea din enunŃ sub forma  + + (ax + by + cz) ≥ ( x + y + z) 2 . Automat suntem
a b c
x y z
conduşi la ideea de a folosi CBS. Vom face notaŃiile convenabile m 2 = , n 2 = , p 2 = , iar α2 = ax,
a b c
x y c
β2 = by, γ2 = cz. De aici rezultă că  + + (ax + by + cz) = (m2 + n 2 + p 2 )(α 2 + β2 + γ 2 ) ≥ (mα + nβ + pγ) 2 =
a b z
 x y z 
= ⋅ ax + ⋅ by + ⋅ cz  = (x + y + z)2; b) Folosind punctul a) avem inegalitatea:
 a b c 
 
a + 2b b + 2c c + 2a (a + 2b + b + 2c + c + 2a ) 2 9(a + b + c)2
+ + ≥ = 2 2 2 =
a + 2c b + 2a c + 2b (a + 2b)(a + 2c) + (b + 2c)(b + 2a ) + (c + 2a )(c + 2b) a + b + c + 8ab + 8bc + 8ac
9(a 2 + b 2 + c 2 + 2ab + 2ac + 2bc)
= . Dar a2 + b2 + c2 ≥ ab + ac + bc, (1). De aici, 3(a2 + b2 + c2 + 2ab + 2ac +
a 2 + b 2 + c 2 + 8ab + 8bc + 8ac
+ 2bc) ≥ 3(a2 + b2 + c2 + 8ab + 8ac + 8bc). Într-adevăr, 3a2 + 3b2 + 3c2 + 6ab + 6ac + 6bc ≥ a2 + b2 + c2 + 8ab +
+ 8ac + 8bc, adică (1); a2 + b2 + c2 ≥ ab + ac + bc. 11. a) Folosind inegalitatea dintre media aritmetică şi media
geometrică, obŃinem: a + b ≥ 2 ab , b + c ≥ 2 bc , a + c ≥ 2 ac . ÎnmulŃind membru cu membru aceste
inegalităŃi, găsim (a + b) ⋅ (b + c)(a + c) ≥ 8abc; b) Rescriem inegalitatea de demonstrat sub forma
a c b c a b
+ + + + + ≥ 6 . Folosind de asemenea inegalitatea dintre media aritmetică şi cea geometrică, pe
c a c b b a
a c a c b c a b
rând, găsim că + ≥2 ⋅ = 2 şi analoagele + ≥2, + ≥ 2 . Adunând membru cu membru,
c a c a c b b a
obŃinem ceea ce dorim. 12. Folosind inegalitatea CBS, obŃinem (12 + 12 + 12) ( x i2 + x 2j + x 2k ) ≥ ( x i + x j + x k ) 2 ,
1
(∀) xi, xj, xk ∈ R, i ≠ j ≠ ≠ k. De aici x i2 + x 2j + x 2k ≥
( x i + x j + x k ) , (1). Aplicând inegalitatea (1) pentru
3
fiecare termen al inegalităŃii şi însumând, obŃinem inegalitatea cerută. 13. Pornim de la inegalitatea evidentă
1 1 a+b a+b 11 1
a2 + b2 ≥ 2ab, a, b ∈ (0, ∞). De aici 2 ≤ , de unde 2 ≤ =  +  . Scriem inegalităŃile
a + b 2 2ab a + b2 2ab 2a b

194
b+c 1 1 1 a+c 1 1 1
analoage 2 2
≤ ⋅  + , 2 2
≤  +  şi, prin sumare, ajungem la relaŃia de demonstrat.
b +c 2 b c a +c 2a c
14. Ridicăm la pătrat ambii membri: (a1 + b1)2 + (a2 + b2)2 + ... + (an + bn)2 = a12 + a 22 + ... + a n2 + b12 + b 22 + ... +
+ b 2n + 2 (a12 + a 22 + ... + a 2n )(b12 + b 22 + ... + bn2 ) ⇔ a1b1 + a 2 b 2 + ... + a n b n ≤ (a12 + a 22 + ... + a n2 )(b12 + b22 + ... + b n2 ).
Ridicând încă o dată la pătrat, ajungem la (a1b1 + a 2 b 2 + ... + a n b n ) 2 ≤ (a 22 + a 22 + ... + a 2n )(b12 + b 22 + ... + b 2n ) ,
adică la inegalitatea „celebră” CBS. Practic am arătat echivalenŃa celor două inegalităŃi! 15. Avem a2 + b2 ≥ 2ab,
de unde (a2 + b2)(a + b) ≥ 2ab(a+b), adică a3 + b3 ≥ ab(a + b), adică a3 + b3 + abc ≥ ab(a + b + c). Avem şi
analoagele b3 + c3 + abc ≥ bc(a + b + c) şi a3 + c3 + abc ≥ ac(a + b + c). Adunând membru cu membru, obŃinem
1 1 1 1 1 1 1 a +b+c 1
+ 3 3 + 3 3 ≤ + + = ⋅ = .
a + b + abc b + c + abc c + a + abc ab(a + b + c) bc(a + b + c) ac(a + b + c) abc a + b + c abc
3 3

2.7. InegalităŃi condiŃionate


 a b c 3  a 1  b 1
1. ObŃinem 4 + + −  + ab + ac + bc – a2 – b2 – c2 ≥ 0 sau 4 −  + 4 − +
b+c a +c a +b 2 b+c 2 a +c 2
 c 1  1 4a 4 b 4c 1
+ 4 −  − [(a − b) 2 + (a − c) 2 + (b − c) 2 ] ≥ 0 . Imediat −2+ −2+ − 2 − [(a – b)2 +
a +b 2 2 b+c a+c a+b 2
 2a − b − c 2 b − a − c 2c − a − b  1
+ (a – c)2 + (b – c)2] ≥ 0 sau 2  + +  − 2 [(a – b) + (a – c) + (b – c) ] ≥ 0, (1). Pe
2 2 2

 b + c a + c a + b 
2a − b − c a + b + a + c − 2(b + c) a + b a + c a + b a + c
de altă parte, cum = = + − 2 , (1) devine 2  + −2+
b+c b+c b+c b+c b + c b + c
b+c b+a c+a c+b  1 a+b b+c
+ + −2+ + − 2 − [(a – b)2 + (a – c)2 + (b – c)2] ≥ 0. Pe de altă parte + −2=
a+c a+c a+b a+b  2 b+c a+b
(a − c) 2  (a − c) 2 (a − c) 2   (a − b) 2
= şi de aici inegalitatea precedentă devine 2  −  + 2 −
(b + c)(a + b)  (b + c)(a + b) 2   (c + a )( b + a )
(a − b ) 2   ( b − c) 2 (b − c) 2  2 (a − c ) 2 2( a − b ) 2
−  + 2 −  ≥ 0 sau ⋅ [2 − (b + c)(a + b)] + ⋅ [2 − (a + c) ⋅
2   (a + b)(a + c) 2  (b + c)(a + b) (a + c)(b + c)
2( b − c ) 2
⋅ (b + c)] + [2 − (a + b)(a + c)] ≥ 0 , evidentă dacă a, b, c ∈ (0, 1), căci 2 – (a + c)(b + c) ≥ 0 şi
(a + b)(a + c)
analoagele. 2. a) Cum x + y + z = 1, avem x2y2 + y2z2 + z2x2 ≥ xyz(x + y + z), de unde 2x2y2 + 2y2z2 +
+ 2z2x2 – 2x2yz – 2xy2z – 2xyz2 ≥ 0, care este adevărată, fiind echivalentă cu (xy – xz)2 + (xy – yz)2 + (xz –
1 1 1  1 1 1  3 1 1 1
– yz)2 ≥ 0; b) Prin ridicare la pătrat se obŃine 2 + 2 + 2 + 2 + +  ≥ , de unde 2 + 2 + 2 +
x y z  xy yz zx  xyz x y z
2( x + y + z ) 3
+ ≥ , care conduce la a) Ńinând cont că x + y + z = 1. 3. Folosim inegalitatea dintre media
xyz xyz
b+a+c
geometrică şi media aritmetică, obŃinem ab + bc = b(a + c) ≤ = 1005 . Analog pentru celelalte şi
2

195
2
 a b  1 1 
adunând membru cu membru, obŃinem inegalitatea cerută. 4. Avem  x +  ≤ (a 2 + b 2 ) +
x +1   x x
=
+1

2
2x + 1 (x + 1)2 x + 1  a b  x +1
= ≤ = . Analog  +  ≤ . Aplicând inegalitatea dintre media pătratică şi
x(x + 1) x(x + 1) x  x + 1 x  x
2 2
 a b   a b 
 +  + + 
 a b  a b   x x +1   x +1 x
cea geometrică, suntem conduşi la:  +  +  ≤ ≤
 x x + 1  x + 1 x 2
1
≤ 1 + . 5. Din a, b, c ∈ (0, ∞) şi a + b + c = 1, deducem a, b, c ∈ (0, 1). Atunci a2 + b2 + c2 – 6abc ≥ a3 + b3 +
x
+ c3 este echivalentă cu şirul de inegalităŃi: a2(1 – a) + b2(1 – b) + c2(1 – c) ≥ 6abc, de unde a2(b + c) + b2(a + c) +
a c b c a b
+ c2(a + b) ≥ 6abc. ÎmpărŃind cu abc > 0 obŃinem + + + + + ≥ 6 , inegalitate imediată, folosind
c a c b b a
x y a3 b3 c3 a4 b4 c4
faptul că pentru x, y > 0, + ≥ 2 . 6. + + = = + + ≥
y x a + 2b b + 2c c + 2a a 2 + 2ab b 2 + 2bc c 2 + 2ac
(a 2 + b 2 + c 2 ) 2 (a 2 + b 2 + c 2 ) 2 (a 2 + b 2 + c 2 ) 2
≥ 2 2 2
= = . Pe de altă parte, folosind inegalitatea
a + b + c + 2ab + 2ac + 2bc (a + b + c ) 2 9

a 2 + b2 + c2 a + b + c (a 2 + b2 + c 2 ) 2
dintre media pătratică şi media aritmetică, ≥ , obŃinem că =
3 3 9
2
 a 2 + b2 + c 2  (4)  a + b + c  4
=  ≥  = 1 , de unde rezultatul dorit. 7. Plecăm de la identitatea (1 – x)(1 – y)(1 – z) =
 3   3 
=1 – x – y – z + xy + yz + zx – xyz, (1). Cum x + y + z = 1, (1) devine xy + yx + zx – xyz = (1 – x)(1 – y)(1 – z),
(2). Din x + y + z = 1, obŃinem x + y = 1 – z şi cum, în plus, x, y, z ∈ [0, ∞), obŃinem 1 – z ≥ 0, (3) şi similarele 1 – x
≥ 0, (4), 1 – y ≥ 0, (5). Folosind (2) şi (3), (4), (5), obŃinem prima inegalitate xy + yz + zx – xyz ≥ 0, (6). În (6)
egalitatea se obŃine dacă x = 1, y = z = 0 sau x = 0, y = 1, z = 0 sau x = y = 0, z = 1.
Pentru justificarea celeilalte inegalităŃi, folosim inegalitatea dintre media aritmetică şi media geometrică a
(1 − x)(1 − y)(1 − z) 3
numerelor pozitive 1 – x, 1 – y, 1 – z. ObŃinem că ≥ (1 − x )(1 − y)(1 − z ) , de unde
3
8
(1 − x )(1 − y)(1 − z) ≤ . Din (2) avem relaŃia dorită (am Ńinut cont că x + y + z = 1). Egalitatea se realizează
27
1
pentru x = y = z = . 8. Plecăm de la inegalitatea x2 – x + 1 > 0, (∀) x ∈ R. Într-adevăr, x2 – x + 1 =
3
2
 1 3
=  x −  + > 0 . Considerând x = a, x = b şi x = c şi adunând inegalităŃile obŃinute, găsim că a2 + b2 + c2 + 3 >
 2 4
> a + b + c. Pe de altă parte, cum a2 + b2 + c2 = (a + b + c)2 – 2(ab + ac + bc), avem: (a + b + c)2 – 2(ab + ac + bc) +
3 3
+ 3 > a + b + c. Dacă a + b + c = 0, rezultă că ab + ac + bc < , iar dacă a + b + c = 1, atunci ab + ac + bc < ,
2 2
adică rezultatul aşteptat în ambele ipoteze. 9. a) Din a2 + b2 + c2 ≥ ab + ac + bc obŃinem că 3(a2 + b2 + c2) ≥ (a +
+ b + c)2; cum a2 + b2 + c2 = 1, rezultă că |a + b + c| ≤ 3 . b) Din 2(a2 + b2) ≥ (a + b)2 rezultă că

196
|a+b| |b+c| |a +c| |a+b|
a 2 + b2 ≥ . Cum 1 − a 2 = b2 + c2 ≥ şi analoagele 1 − b2 ≥ , 1 − c2 ≥ ,
2 2 2 2
|b+c| |a+c| |a+b| 1
prin adunare membru cu membru, obŃinem că 1 − a 2 + 1 − b2 + 1 − c2 ≥ + + ≥ ⋅
2 2 2 2
⋅ | 2(a + b + c) |= 2 | a + b + c | . 10. Aplicând succesiv inegalitatea dintre media aritmetică şi cea geometrică,
1 1 1 2
găsim: x + y + z = x + y + ≥ 2 xy + ≥ 2 2 . De aici deducem că < şi analoagele
xy xy x+ y+ z 4
1 2 1 2
< , respectiv < . Adunând membru cu membru, obŃinem inegalitatea dorită.
y+z+ x 4 z+x+ y 4
(b ⋅ c ⋅ d) 2 (c ⋅ d ⋅ a) 2 (d ⋅ a ⋅ b) 2 (a ⋅ b ⋅ c) 2
11. Întrucât abcd = 1, obŃinem transcrierea egalităŃii sub forma E = + + + .
b+c+d c+d+a d+a +b a +b+c
x 2 y 2 z 2 ( x + y + z) 2 (bcd + cda + dab + abc) 2
Folosind inegalitatea CBS sub forma + + ≥ , obŃinem că E ≥ ,
m n p m+n+p 3(a + b + c + d )
2
(1). Pe de altă parte, inegalitatea x2 + y2 + z2 + t2 ≥ (xy + xz + xt + yz + yt + zt) conduce la (x + y + z + t)2 ≥
3
8 8
≥ (xy + xz + xt + + yz + yt + zt) şi atunci (bcd + cda + bad + abc)2 ≥ (ab + ac + ad + bc + bd + dc), (2). Din
3 3
8(ab + ac + ad + bc + bd + cd)
(1) şi (2) găsim că E ≥ , (3). Pe de altă parte, folosind inegalitatea dintre media
9( a + b + c + d )
aritmetică şi cea geometrică, obŃinem ad + bc ≥ 2 abcd , ab + cd ≥ 2 abcd , ac + bd ≥ 2 abcd . Revenind în
8⋅6 16
(3), găsim că E ≥ = . 12. i) Folosim substituŃia a = c ⋅ x, b = c ⋅ y, x, y > 0. Din
9(a + b + c + d) 3(a + b + c + d)
5 1 1 1 3( x 2 + y 2 + 1)
condiŃia iniŃială rezultă ( x 2 + y 2 + 1)c 2 =
, iar inegalitatea devine + −1 < 2 = . De aici,
3 x c c xy 5xy
efectuând calculele, găsim forma echivalentă a inegalităŃii 3x2 – 5(1 – y) x + 3y2 – 5y + 3 > 0. Aplicând
principiul trinomului, se impune ca ∆ = –11y2 + 10y – 11 < 0, (∀) y > 0, ceea ce este adevărat pentru orice x, y ∈ R,
cu atât mai mult pentru x ≥ 0, y > 0; ii) Aplicând CBS, obŃinem că (a – 2b + c)2 ≤ (a2 + b2 + c2)(1 + 4 + 1), de
1
unde − 10 ≤ a − 2b + c ≤ 10 . 13. Din abc = 1, obŃinem că 2 = b 2 c 2 . Atunci E se rescrie astfel: E =
a
b 2 c 2 a 2 c 2 a 2 + b 2 (ab + ac + bc)2 x 2 y 2 z 2 ( x + y + z) 2
= + + ≥ (am folosit inegalitatea + + ≥ ). Însă x2 + y2 +
b+c a+c a+b 2(a + b + c) m n p m+n+p
+ z2 ≥ xy + yz + xz, de unde (x + y + z)2 ≥ 3(xy + yz + xz). Astfel, obŃinem că (ab + bc + ac)2 ≥ 3abc(a + b + c)
1 1 1 3
deci 2 + + ≥ . 14. a) ÎmpărŃind cu a , inegalitatea devine ( ab − 1) 2 ≥ 0 ;
a ( b + c ) b 2 (a + c ) c 2 ( a + b ) 2
2a a 2b b 2c c
b) Folosind a), avem ≤ şi analoagele ≤ , respectiv ≤ . Adunând membru cu
ab + 1 b bc + 1 c ca + 1 a
2a 2b 2c  a b c 
membru, obŃinem că + + ≤ + + abc = a c + b a + c b ; c) Avem că

ab + 1 bc + 1 ca + 1  b c a 

197
(a + b + c)3 = a3 + b3 + c3 + 3[ab(a + b) + bc(b + c) + ac(a + c)] + 6abc, (1); (a + b + c)2 = a2 + b2 + c2 + 2(ab +
+ ac + bc), (2). Din (1) obŃinem 1 = a3 + b3 + c3 + 3[ab(a + b + c – c) + bc(b + c + a – a) + ac(a + c + b – b)] +
+ 6abc; 1 = a3 + b3 + c3 + 3ab + 3ac + 3 bc – 3abc, (3), iar din (2) că 1 = a2 + b2 + c2 + 2(ab + ac + bc), (4). Dacă
folosim (3) şi (4), inegalitatea de demonstrat revine la 1 – 3ab – 3ac – 3bc + 9abc ≤ 1 – 2ab – 2ac – 2bc, sau
1 1 1 1 1 1
9abc ≤ ab + ac + bc, sau + + ≥ 9 , sau binecunoscuta inegalitate adevărată (a + b + c) + +  ≥ 9 . (Am
a b c a b c
Ńinut cont că a + b + c = 1). 15. Există x, y, z ∈ (0, ∞) astfel încât x = a + 1, y = b + 1, z = c + 1, de unde condiŃia
din enunŃ se transcrie (x – 1)(y – 1) + (y – 1)(z – 1) + (z – 1)(x – 1) + 2(x – 1)(y – 1)(z – 1) = 1, de unde, după
1 1 1 4 1 1
calcule, suntem conduşi la xy + yz + zx = 2xyz sau + + = 2 , (1). Folosim inegalitatea ≤ + ,
x y z m+n m n
1 1 1
(2) şi, Ńinând cont de notaŃii, avem de arătat că + + ≤ 1 . Înlocuind în (2), m, n cu x, y, z (ciclic)
x+y y+z z+x
1 1 1 1  2 2 2  1  1 1 1  (1) 1
şi sumând membru cu membru, obŃinem: + + ≤  + +  =  + +  = ⋅ 2 = 1 .
x+ y y+z z+x 4x y z 2x y z 2

III. FuncŃii

3.1. Şiruri de numere reale. Şiruri particulare


1. a) Au loc relaŃiile: (xn–1 – 1)2 = xn – xn–1, de unde xn – 1 = xn–1 (xn–1 – 1) = xn–1 ⋅ xn–2(xn–2 – 1) .... = xn–1 ⋅ xn–2 ⋅
1 1 1 1 1 1
⋅ ... ⋅ x1(x1 – 1) = x1x2...xn–1, de unde xn = 1 + x1 ⋅ x2 ⋅ x3 ⋅ ... ⋅ xn–1; b) + + ... + + = + +
x1 x 2 x n −1 x n − 1 x1 x 2
1 1 1 1 1 1 + x1x 2 ...x n − 2 1 1 1 1 1
+ ... + + = + + ... + + = + + ... + + = +
x n −1 x1x 2 ...x n −1 x1 x 2 xn −2 x1x 2 ...x n −1 x1 x 2 x n − 2 x1x 2 ...x n − 2 x1
1 1 1 1 1 1 1 1 1 1 1
+ + ... + + = + + ... + = + = 1 ; c) Folosind b) găsim: + + ... + <
x2 x n − 3 x1x 2 ...x n − 3 x1 x 2 x1 x 2 x 1 x 1 x1 x 2 xn
1 1 1 2 3 n
< + + ... + = 1 . 2. InducŃie! Pentru n = 1, x 2 = , n = 2, x 3 = . Presupunem x n = şi arătăm
x1 x 2 xn −1 6 6 6
n + 1  n 1  (n + 3)( n + 1) n + 1 2010
x n +1 =  + = = . De aici x 2010 = = 335 . 3. a) x 3 = 3 şi x 4 = 3 3 + 1 ;
n +3 6 2 6(n + 3) 6 6
b) InducŃie (tip Cauchy): n = 3, x 3 = 3 < 2 . Presupunem xn–1 < n – 2 şi xn < n – 1 şi arătăm că xn+1 < n.
Într-adevăr, x n +1 = nx n + x n − 1 < n(n − 1) + n − 1 = n 2 − 2 < n ; c) Ideea: n – 2 < xn < n – 1, de unde xn ∉ N.
Cum xn < n – 1, din b) arătăm xn > n – 2. Tot inducŃie ca la b.) Presupunem xn–1 > n – 3 şi xn > n – 2 şi arătăm că
x n +1 = n ⋅ x n + x n − 1 > n (n − 2) + n − 3 = n 2 − 2n + n − 3 = n 2 − n − 3 > n − 1 . 4. a) Folosim inducŃia
n n +1 n
matematică. Pentru n = 1, 32 = 2 ⋅ 4 + 1. Presupunem că 32 = 2 n (2a n ) + 1 , atunci 32 = (32 ) 2 = 2 n +1 ⋅ 2 ⋅
n
−n
⋅ (2 n ⋅ a 2n + a n ) + 1 = 2 n +1 (2a n +1 ) + 1 ; b) Conform a), a 2 n este par. Apoi din 3n > n, a 3 n = 33 este divizibil cu 3.
n
Ca la a) 34 = 4 n (5a n ) + 1 , n ∈ N*, deci a n se divide cu 5, n ∈ N*.
34

198
5. Folosim metoda inductivă. Pentru n = 1, se obŃine x1 = 1. Vom presupune că xn = n şi arătăm că xn+1 = n + 1.
Conform ipotezei, x13 + x 32 + x 33 + ... + x 3n + x 3n +1 = (x1 + x 2 + ... + x n + x n +1 ) 2 , (∀) n ∈ N*, Atunci 13 + 23 + 33 +
n 2 (n + 1) 2 n 2 (n + 1) 2
+... + n3 + x 3n +1 = (1 + 2 + ... + n + xn+1)2 , echivalent cu + x 3n +1 = + x n +1n (n + 1) + x 2n +1 , de
4 4
unde x 2n +1 − x n +1 − n ⋅ (n + 1) = 0, cu soluŃiile xn+1 = n + 1 sau xn = –n, n ∈ N*. (Ultima soluŃie nu convine!) În
concluzie, şirul căutat va fi xn = n, n ∈ N*. 6. Procedeul clasic: Notăm Sn = a1 + a2 + ... + an, de unde Sn – Sn–1 = an.
a (n + 2) a n −1 (n + 1) n +1
Atunci an = n − , de unde a n = a n −1 . Înlocuind, pe rând, n = 2, 3, ... şi înmulŃind
3 3 n −1
3 ⋅ 4 ⋅ 5 ⋅ ... ⋅ (n + 1) 1
membru cu membru, obŃinem că a1 ⋅ a 2 ⋅ ... ⋅ a n = ⋅ a1 ⋅ a2 ⋅ ... ⋅ an–1, de unde ⋅ n (n + 1)a1 = a n .
1 ⋅ 2 ⋅ 3 ⋅ ... ⋅ (n − 1) 2
Cum a1 = 2, din ipoteză, găsim că an = n(n + 1). Imediat, cum n ≤ n 2 + n < n + 1 , obŃinem că [ n 2 + n ] = n ,
n
n(n + 1) a n
(∀) n ∈ N, de unde [ a n ] = n . Astfel, ∑[
k =1
a k ] = 1 + 2 +... + n =
2
=
2
, adică concluzia. 7. Avem că

1 1
a 2n < a n − a n +1 ⇒ a n (a n − 1) < −a n +1 < 0 , de unde an ∈ (0, 1). Atunci an(1 – an) > an+1, deci < ,
a n (1 − a n ) a n +1
n −1 n −1
1 1 1  1  1
∑ ∑  a
1 1 1
rezultând că < − . Însumând, obŃinem < −  = − . Cu alte cuvinte,
1 − a n a n +1 a n k =1
1− ak k =1 k +1 ak  a n a1
1 1 1 1 1
+ ... + < − . Cum 1 – ai ∈ (0, 1), rezultă că > 1 , (∀) i = 1, n − 1 şi cum a1 ∈ (0, 1),
1 − a1 1 − a n −1 a n a1 1− ai
1 1 1 1 1 1
urmează că şi > 1 , de unde > + + + ... + > 1 + (n − 1) = n . De aici, a n < , (∀) n ∈ N*,
a1 a n a1 1 − a 2 1 − a n −1 n
(1). În plus an > 0, (2). Din (1) şi (2) găsim că 0 < nan < 1, de unde concluzia [n ⋅ an] = 0. 8. Vom folosi metoda
a (a + 1)
inductivă. Pentru n = 1 avem a1 = 1 1 , cu soluŃia a1 ∈ {0, 1}. Cum a1 ∈ N*, soluŃia admisă este a1 = 1.
2
a (a + 1)
Pentru n = 2, a 2 + 1 = 2 2 , cu soluŃia a2 ∈ {–1, 2}. ObŃinem a2 ∈ N*. Intuim că an = n. Presupunem acest
2
a (1 + a n +1 )
lucru şi ar trebui să avem an + 1 = n + 1. Într-adevăr, 1 + 2 + ... + n + an+1 = n +1 , cu soluŃiile an ∈ {n + 1, –n},
2
singura soluŃie din N* fiind an + 1 = n + 1, ceea ce încheie demonstraŃia. 9. a) Este clar că xn + 1 – 3xn = 1 + 3n +1 ⋅
⋅ (2n + 1) – 3 – 3n+1(2n – 1) = 2 ⋅ 3n+1 – 2, de unde yn = 2 ⋅ 3n+1 – 2, (∀) n ∈ N*; b) Cum 3n+1 = (2m + 1)n+1 = 2k + 1,
k ∈ N, obŃinem că yn = 2(2k + 1) – 2 = 4k, (∀) k ∈ N, de unde yn ⋮ 4. Demonstrăm, prin inducŃie, că xn ⋮ 4,
(∀) n ∈ N*. i) Pentru n = 1, x1 = 4 ⋮ 4; ii) Presupunem că xn ⋮ 4. Atunci xn+1 = 3xn + yn, (∀) n ≥ 1, de unde, într-a-
devăr, xn+1 ⋮ 4. 10. Procedăm prin reducere la absurd! Presupunem că şirul este neconstant: urmează că există
p k p+k
k, p ∈ N*, k ≠ p şi ak ≠ ap. Atunci 0 < |ak – ap| = |ak – an + an – ap| ≤ |ak – an| + |an – ap| ≤ + = , (∀) n ∈
n n n
k+p
∈ N*. De aici ar urma că n ≤ , (∀) n ∈ N*, fals, căci N* este nemărginită! (axioma lui Arhimede!).
| ak − ap |
3 a2
11. a) Avem că a1 + a 2 = , de unde a 2 = 2 , adică a2 = 4. Pentru n = 3, obŃinem că a1 + a 2 + a3 =
2

199
4 a3
= , de unde a 3 = 3 , adică a3 = 9; b) Folosim procedeul inductiv. În a) avem o „etapă de verificare”.
2
Presupunem că (∀) k ∈ N*, ai = i2, (∀) i ∈ {1, 2, ..., k}. Arătăm că ak+1 = (k + 1)2. Conform ipotezei avem
(k + 2) a k −1 k (k + 1)
k a k +1
a1 + a 2 + ... + a k + a k +1 = , de unde găsim =, adică a k +1 = k + 1 , deci
2 2 2
a k +1 = (k + 1) 2 . În concluzie, an = n2, (∀) n ∈ N*, ceea ce rezolvă problema. 12. a) Vom demonstra inductiv
mărginirea. Fie n = 1. Avem că x1 = x 0 − x 02 + x 30 − x 04 = x 0 (1 − x 0 ) + x 30 (1 − x 0 ) > 0 , cum x0 ∈ (0, 1), (1). Pe de
aptă parte, 1 – x1 = 1 − x 0 + x 02 − x 30 + x 04 = (− x 0 + x 02 (1 − x 0 ) + x 04 ) > 0 , de unde x1 < 1, (2). Din (1) şi (2) avem
x1 ∈ (0, 1). Acum presupunem că xk ∈ (0, 1), k ∈ N şi arătăm că xk+1 ∈ (0, 1). Într-adevăr, xk+1 = xk –
− x 2k + x 3k − x 4k = x k (1 − x k ) + x 3k (1 − x k ) > 0 , xk ∈ (0, 1), adică xk+1 > 0, (3). Avem 1 – xk+1 = 1 – x k + x 2k −
− x 3k + x 4k = 1 − x k + x 2k (1 − x k ) + x 4k > 0 , adică xk+1 < 1, (4). RelaŃiile (3) şi (4) demonstrează mărginirea şirului
dat; b) Pentru monotonie observăm că x n +1 − x n = − x 2n ( x 2n − x n + 1) = − x n2 ( x n2 + 1 − x n ) , de unde xn+1 – xn < 0,
adică xn+1 < xn, (∀) n ∈ N. 13. Procedăm prin inducŃie. Pentru n = 1, obŃinem a1 = 1, iar pentru n = 2, a2 = 2.
Presupunem că ak = k, (∀) k = 1, n şi arătăm că an+1 = n + 1. Scriem relaŃia pentru n + 1: 1 + 1! ⋅ 1 + 2! ⋅ 2 + ... +
n n n
+ n!n + n! a 2n +1 = n!a n +1 (a n +1 + 1) , adică 1 + ∑ k ⋅ k! = n!a
k =1
n +1 . Dar ∑ k ⋅ k! = ∑ [(k + 1)!−k!] = (n + 1)!−1 .
k =1 k =1
De aici n!an+1 = (n + 1)!, de unde an+1 = n + 1. 14. Din relaŃia de recurenŃă, xn+1 – xn = (a – 1)(xn – xn–1) = (a –
– 1)2(xn–1 – xn–2) = ... = (a – 1)n(x1 – x0), adică xk+1 – xk = (a – 1)k(x1 – x0). Înlocuind k cu 0, 1, 2, ..., n – 1 şi
(a − 1) n − 1 (a − 1) n − 1
sumând relaŃiile, obŃinem x n = x 0 + ( x1 − x 0 ) ⋅ > .
a−2 a−2

3.2. Progresii aritmetice


1. Presupunem că şirul (xn)n≥3 este progresie aritmetică. Atunci x1 + xn = x2 + xn–1 = ... = xk + xn–k + 1, k = 1, n .
x1 + x n x 2 + x n −1 x + xn  1 1 1 
Atunci + + ... + 1 = 2 + + + ... +  , de unde egalitatea evidentă:
x1 x n x 2 x n −1 x1 + x n  x1 x 2 x n 
 1 1   1 1   1 1   1 1 1 
 +  +  +  + ... +  +  = 2 + + ... +  . Vom folosi inducŃia pentru a demonstra
x
 n x x
1   n −1 x 2  x
 1 x n  x
 1 x 2 x n 

2 1 2  1 1 1 
implicaŃia reciprocă. Pentru n = 3 obŃinem + 2 = ⋅ ⋅  + +  , de unde găsim că x1 + x3 =
x1x 3 x 2 x1 + x 3  x1 x 2 x 3 
= 2x2, ceea ce arată că x1, x2, x3 sunt termenii şirului în progresie aritmetică. Presupunem că x1, x2, ..., xn
alcătuiesc o progresie aritmetică şi vom arăta că xn+1 este următorul termen al progresiei. Avem succesiv relaŃiile:
2 1 1 1 2  1 1 1 
+ + + ... + =  + + ... + ;
x1x n +1 x 2 x n x 3 x n −1 x n x 2 x 1 + x n +1  x 1 x 2 x n +1 
2 2  1 1  2  1 1  1  1
+  + ... + =  + ... + ; − +
x 2 x n +1 x 2 + x n  x 2 x n  x1 + x n +1  x1 x n +1  x1x n +1  x1 ( x1 + x n +1 )

200
1   1 1  1 1  1 1 1 1
+  =  + ... +  −  . Cum + + ... + ≠ 0 şi atunci −
x n +1 ( x 1 + x n +1 )   x 2 x n  x1 + x n +1 x 2 + x n  x2 x3 xn x1 + x n +1
1
− = 0 , rezultă xn+1 = xn + x2 – x1 = xn + r, ceea ce încheie demonstraŃia. 2. a) Fie S = a1 + a2 + ... + an.
x2 + xn
4020 n −1
ObŃinem a1 + a n = , (1). Cum an = a1 + (n – 1)r, n ≥ 1, obŃinem an – a1 = , (2). Din (1) şi (2) găsim
n 3
2010 n − 1 n − 1 2010
a1 = − ,an = + . Cum a1, an ∈ N obŃinem n ∈ {1, 4, 67}; b) Din a) găsim imediat: n = 1,
n 6 6 n
suma are 1 termen natural; n = 4, suma are 2 termeni naturali; n = 67, suma are 23 termeni naturali. 3. Avem
card(A ∪ B) + card(A ∩ B) = card A + card B = 2011. Pentru a afla numărul lui card(A ∪ B) trebuie găsit
maximul lui card(A ∩ B). Fie x ≠ y, cu x, y ∈ A ∩ B. Imediat x – y = k 2 , k ∈ Z* respectiv x – y = l 3 , l ∈
3 q
∈ Z*. De aici, dacă (∃) x, y, x ≠ y ∈ A ∩ B, atunci = ∈ Q, absurd. Urmează că A ∩ B are cel mult un
2 l
termen, de aici min card(A ∪ B) = 2010, care este atins când cele două progresii au un element comun. 4. Avem
1 1 2 1 1 
imediat + ... + =  + ... +  , (1). În plus, ştim că a1an ≤ ak ⋅ an–k+1, (2) (ExerciŃiu!). Folosind (1),
a1a n a n a1 a1 + a n  a1 an 
a1 + a n  1 1  1 1 1 1 1 1 a1 + a n  1 1
găsim  + ... +  = + + ... + , de unde + + ... + =  + + ... +
2  a1a n a n a 1  a1 a 2 an a1 a 2 an 2  a1a n a 2 a n −1
1  a1 + a n n S
+ ≤ ⋅ = (am folosit (2)!). 5. Fie x1, x2, x3, x4, x5, x6 termenii progresiei aritmetice.
a n a1  2 a1a n a1a n
CondiŃiile problemei conduc la relaŃiile: x13 + (3r −1)x12 + + (2r 2 − 9r ) x1 − 20r 2 = 0 , (1) şi x12 − rx1 − 2r 2 = 0 ,
(2), unde r reprezintă raŃia progresiei, iar x1 primul termen al progresiei căutate. RelaŃia (2) reprezintă o ecuaŃie
2
x  x x
omogenă:  1  − 1 − 2 = 0 . Notând cu t = 1 , soluŃiile acestei ecuaŃii conduc la relaŃiile x1 = –r, respectiv x1
 r r r
= 2r. Dacă x1 = –r, înlocuind în (1), obŃinem r = 0, deci „progresia” 0, 0, ..., care nu convine; dacă x1 = 2r, găsim
7 7 7 7 14 35 49
r = , de unde x1 = . Progresia căutată va fi deci , , , , 7, . 6. Pentru a determina progresia
6 3 3 2 3 6 6
aritmetică a1, a2, ..., an, trebuie să determinăm primul termen a1 şi raŃia r în funcŃie de a şi de n, unde n reprezintă
numărul termenilor. Avem că a1 ⋅ r = a, iar Sn este de valoare minimă. Aplicând condiŃia, obŃinem
n n
Sn = [2a1 + r (n − 1)] ≥ ⋅2 ⋅ 2a1 ⋅ r (n − 1) = n 2a (n − 1) , cu egalitate dacă 2a1 = r(n – 1), de unde
2 2
2a
2a1 ⋅ r = r2(n – 1). De aici r = , unde n ≥ 3, căci progresia este cu termeni pozitivi. Imediat,
n −1
2a (n − 1) 2 a (n − 1)
a1 = = . 7. Cum ai sunt termenii pozitivi ai progresiei aritmetice, găsim o dată că
4( n − 1 2
a 2i −1 + a 2i +1
a 2i = , (1) şi folosind inegalitatea dintre media aritmetică şi media geometrică, obŃinem că
2
a 2i −1 + a 2i +1 1 1
≥ a 2i −1 ⋅ a 2i +1 , (2). Din (1) şi (2) deducem că ≤ , inegalitate echivalentă cu
2 a 2i a 2i −1 ⋅ a 2i +1

201
a 2 i +1 a 2 i +1
≤ , (3), pentru i = 1, n . ÎnmulŃind membru cu membru în inegalitatea (3) prin parcurgerea lui i de la
a 2i a 2i −1
a3 a5 a a 2 n +1
1 la n, suntem conduşi la ⋅ ⋅ ... ⋅ 2 n +1 ≤ , de unde inegalitatea dorită. 8. Fără a restrânge
a2 a4 a 2n a1
generalitatea, putem alege a1 ∈ [0, 1]. Cum r ∈ [0, 1), să presupunem că an ≥ n – 1, (∀) n ≥ 1. Astfel am ajunge,
a +1− r
folosind faptul că an = a1 + (n – 1)r, la concluzia că n ≤ 1 , (∀) n ≥ 1, absurd, căci ar însemna că
1− r
mulŃimea numerelor naturale este mărginită superior. Urmează că există n ∈ N astfel încât an < n – 1.
Considerând intervalele [0, 1), [1, 2), [2, 3), ..., [n – 2, n – 1), din cei n termeni ai şirului, vor exista 2 termeni
consecutivi într-un singur interval (conform principiului lui Dirichlet), în concluzie, având aceeaşi parte întreagă.
n
9. Folosind notaŃia n! = 1 ⋅ 2 ⋅ ... ⋅ n, n ≠ 0, 0! = 1, obŃinem că a1 + a 2 + ... + a n = a n +1 , (∀) n ≥ 1, a1 + a2 + ... +
k
n +1
+ an+1 = a n + 2 , (∀) n ≥ 1, de unde (n +k – 1)an = nan+1, (∀) n ≥ 1, (1). Înlocuind în (1) pe rând n cu 1, 2, ...,
k
n, găsim ka1 = a2; (k + 1)a2 = 2a3; ....; (n + k –2)an–1 = (n – 1)an, (2), care, înmulŃite membru cu membru, conduc
(n + k − 2)!
la k(k + 1)...(n + k – 2) = 1 ⋅ 2 ⋅ ... ⋅ (n – 1)an, de unde a n = a1 , (3). Din (3) obŃinem că bn =
(n − 1)!(k − 1)!
(n + k − 2)! n+k−2
= a1 = a1 . Arătăm că şirul (bn)n≥1 este o progresie aritmetică. Pentru aceasta folosim
(n + k − 3)!(k − 1)! (k − 1)!
b + b n +1
condiŃia necesară şi suficientă ca un şir (bn)n≥1 să fie o progresie aritmetică: b n = n −1 , adică
2
n +k −3 n + k −1
a1 + a1
n+k−2 (k − 1)! (k − 1)! 2(a + k − 2)a1 2(n + k − 2)a1
a1 = = = . 10. a) (⇒) Presupunem că (an)n≥1
(k − 1)! 2 (k − 1)! (k − 1)!
n −1 n −1 n
1 1 1 1  1 1 
∑  a
1 1
este progresie aritmetică. Fie r raŃia sa. Avem că ∑a a =∑ = −  =  −  .
i =1 i i +1 i =1 a i (a i + r) r i =1 i a i +1  r  a 1 a n 
n −1
n −1
∑a a
1
Cum an = a1 + (n – 1)r, efectuând calculele, ajungem la = ; (⇐) Se poate proceda prin inducŃie.
i =1 i i +1 a1an
11. Folosim metoda reducerii la absurd. Presupunem că (∃) k, m, p ∈ N, distincte, astfel încât a k = 2 ,
a m = 3 , a p = 5 sunt termenii aceleiaşi progresii aritmetice. De aici, folosind formula termenului general, an =
m−k 3− 2
= a1 + (n – 1)r, ajungem la = . Cum numărul din stânga este raŃional, iar cel din dreapta
p−m 5− 3
iraŃional, se ajunge la o contradicŃie. 12. a) Vom arăta că diferenŃa yn+1 – yn = constant, (∀) n ∈ N. Într-adevăr,
y n +1 − y n = 4 x n +1 + 1 − 4 x n + 1 = 4 x n + 1 + 4 4 x n + 1 + 4 − 4 x n + 1 = ( 4x n + 1 + 2) 2 − 4x n + 1 =
= 4x n + 1 + 2 − 4x n + 1 = 2 , de unde şirul (yn)n≥0 este progresie aritmetică de raŃie 2; b) ObŃinem, folosind a)
că yn = 3 + 2n, (1). Cum y n = 4x n + 1 , găsim, folosind (1), (3 + 2n)2 = 4xn + 1, de unde xn = n2 + 3n + 2, (2).
ObservaŃie. Se vede că cele două şiruri (xn)n≥0, respectiv (yn)n≥0 sunt şiruri cu termeni numere naturale. 13. Este
clar că S = (a1 − a 2 )(a1 + a 2 ) + (a 2 − a 3 )(a 2 + a 3 ) + ... + (a 2007 − a 2008 )(a 2007 + a2008) = –r(a1 + a2 + ... + a2008) =

202
a1 − a 2008
= –r(a1 + a2008) ⋅ 1004, (1). Dar a2008 = a1 + 2007r, de unde − r = , (2). Din (1) şi (2) găsim că
2007
1004 2
S= (a1 − a 22008 ) . 14. a) Fie am, an, ap termenii progresiei aritmetice. Atunci se ştie că am = a1 + (m – 1)r,
2007
an = a1 + (n – 1)r, ap = a1 + (p – 1)r. Dacă r = 0, atunci (∀) α ∈ Q avem relaŃia: am = α ⋅ an + (1 – α)ap, (1). Dacă
m−n
r ≠ 0, înlocuind în (1), găsim: a1 + (m – 1)r = α[a1 + (n – 1)r] – (1 – α)[a1 + (p – 1)r], deci α = ∈ Q;
n−p
b) Dacă a n = 2 , a p = 3 , din punctul a) deducem că am = α ⋅ 2 + (1 − α) 3 = α( 2 − 3 ) + 3 ∈ R – Q, α ∈ Q.

3.3. Progresii geometrice


1. Presupunem că b1, b2, ..., bn în progresie geometrică, de raŃie q ≠ 1 şi b1 ≠ 0. Atunci b1 + b2 + ... + bn = b1 ⋅
qn −1 b
⋅ , de unde luăm a = 1 şi b = q. Reciproc. Pentru aceasta folosim metoda inductivă: n = 1, b1 = a(b – 1).
q −1 b −1
Pentru n = 2 găsim b2 = b1 ⋅ b iar pentru n = 3, b3 = b1 ⋅ b2, ceea ce arată că b1, b2, b3 sunt în progresie geometrică.
Presupunem b1, b2, ..., bn în progresie geometrică. Atunci bn+1 = a ⋅ bn+1 – a – abn + a = abn(b – 1) = a(b – 1)bn =
= b1 ⋅ bn, ceea ce încheie demonstraŃia. 2. Din 2bn = an + cn, (1) şi b2n+2 = an+1 ⋅ cn+1, (2), deducem, folosind (2) că
b2 = ac, de unde b = ac . Înlocuind în (1) obŃinem că 2 a n ⋅ c n = ( a n ) 2 + ( c n ) 2 care conduce la ( a n −

− c n ) 2 = 0 , de unde a = c. Imediat a = b = c. 3. Notăm A = 1 + 11 + 111 + 1111 + ... + 111


...
1 ; 9A = 9 + 99 +
n ori

10 n +1 − 1
+ 999 + 9999 + ... + 99
 9 = (10 – 1) + (10 – 1) + ... + (10 – 1) = 10 + 10 + ... + 10 – n = 10 ⋅
... 2 n 2 n
−n=
n ori
10 − 1
n +1
10(10 − 1) 10(10 n +1 − 1) 80(10 n +1 − 1) 80 n +1
= . De aici A = . Cum S = 8A, obŃinem 8A = , adică S = (10 − 1).
9 81 81 81
(q 2 ) 2008 − 1
4. Notăm S1 = a1 + a3 + ... + a4015. Suma S1 este o progresie geometrică de raŃie q2. ObŃinem S1 = a1 ⋅ .
q2 − 1
1
Analog se calculează suma S2, Ńinând seama de faptul că este o progresie geometrică de raŃie . Se ajunge la
q2
S1 a2 a2
= a 22 ⋅ q 4015 = a 2008 ⋅ a 2009 = 2009 . De aici, 2009 = 4 , de unde a 2009 = 2 q . Cum q ∈ R / Q, evident
S2 q q
că 2 q ∈ R / Q. 5. Rescriem inegalitatea sub forma b12 − b 22 + b 32 − b 24 + ... + b 22 n −1 − b 22 n + b 22 n + 2 − b 22 n +1 +
+ b 22 n + 4 − b 22 n + 3 + ... + b 24 n − b 42 n −1 ≥ 0 , (1). Fie q raŃia progresiei date. Inegalitatea (1) devine
8n − 2 8n − 4 4n − 4 4n − 2
b12 (1 − q 2 + q − q + ... + q
4 6
− q ) ≥ 0, adică b12 (1 − q 2 4 6
+ q − q + ... + q −q )(1 − q ) = b12 ⋅
4n

q 4n − 1 4n 2 (q
4n
− 1) 2
⋅ ⋅ (1 − q ) = b1 ⋅ ≥ 0 . 6. Din faptul că progresia geometrică este crescătoare, avem că a1 > 0 şi
− q2 −1 q2 + 1
q > 1. łinând cont că ak = a1 ⋅ qk–1, (∀) k = 2, 3, ..., 12, inegalitatea revine la a1 ⋅ q11 – a1 ≥ 11 ⋅ (a1 ⋅ q6 – a1 ⋅ q5),

203
q11 − 1
adică q11 – 1 ≥ 11 ⋅ q5(q – 1) sau încă ≥ 11q 5 . De aici găsim că 1 + q + q2 + ... + q10 ≥ 11q5, inegalitate
q −1
adevărată dacă folosim inegalitatea mediilor: 1 + q + q2 + ... + q10 ≥ 1111 1 ⋅ q ⋅ q 2 ⋅ ... ⋅ q10 = 11q 5 . 7. a) Vom aplica
metoda inductivă pentru a determina expresia lui an. Pentru n = 1, găsim a3 = 3a2 – 2a1 = 3 ⋅ 2 – 2 = 4, iar pentru
n = 2, obŃinem a4 = 3 ⋅ a3 – 2a2 = 3 ⋅ 4 – 4 = 8. Se sugerează că avem an = 2n–1, n ≥ 1. Vom arăta că şi an+1 = 2n+1.
Într-adevăr, a n +1 = 3a n − 2a n −1 = 3 ⋅ 2n −1 − 2 ⋅ 2n −1 = 2n − 2 (3 ⋅ 2 − 2) = 22 ⋅ 2 n − 2 = 2 n ; b) Din punctul anterior, bn =
= an+1 – an = 2n – 2n–1 = 2n–1. Arătăm că şirul (bn)n≥1 este o progresie geometrică. Pentru aceasta, folosim teorema
care dă condiŃiile necesare şi suficiente ca un şir cu termeni pozitivi să fie progresie geometrică: b 2k +1 = b k b k + 2 ,
(∀) k ∈ N*. În cazul nostru, b 2k +1 = (2 k ) = 2 k = 2 ( k −1)( k +1) = 2 k −1 ⋅ 2 k +1 = bk ⋅ bk+2, deci şirul este o progresie
2

a n +1 2 n +1 ⋅ 3 − n 2
geometrică. 8. a) Arătăm că reportul a doi termeni consecutivi este constant. Într-adevăr, = n 1− n = ,
an 2 ⋅3 3
n
2
1−  
(∀) n ∈ N*. Constanta q = va fi raŃia progresiei geometrice (an)n≥1. b) ObŃinem a1 ⋅   =
2 3 130
, de unde
3 2 27
1−
3
n 3
 2 2
  =   , deci n = 3. 9. Notăm cu a lungimea primei vergele şi cu q > 1 raŃia progresiei. Presupunem că
 3 3
4 3
există n + 1 vergele, n ≥ 3. Fie acum k, k ∈ 2, n astfel încât a ⋅ q k = a şi a ⋅ qm = a . Dar a ⋅ qn = 2a, de unde
3 2
n
3
qn = 2. Astfel, (q mn ) =   , iar pe de altă parte, qmn = 2m, de unde 2m+n = 3n , fals. A
2
S S S S
10. a) Se obŃine S1 = , S2 = 1 = 2 . b) Arătăm că raportul n +1 este constant. C1 A2 B1
4 4 4 Sn
1 S 1 B2 C2
Cum, din a), în general Sn = n , atunci n +1 = , ceea ce arată că şirurul (Sn)n≥1
4 Sn 4 B A1 C
este progresie geometrică.
1
−1
S S S n S 1  S
c) Aplicând formula sumei unei progresii geometrice, obŃinem + 2 + ... + n ; 4 = 1 −  < .
4 4 4 1 3  4n  3
−1
4
x 2n + a
− a 2
x − a 2x n x − a 
11. a) Observăm că n +1 = = n  , (∀) n ∈ N*. De aici, folosind inducŃia, suntem conduşi
x n +1 + a 2
xn + a x + a 
 n 
2x n
2n −1
xn − a x − a  x1 − a
la = 1  , (∀) n ∈ N*, de unde λ = , bn = 2n–1, (∀) n ∈ N*. b) Din punctul a), avem
x n + a  x1 + a  x1 + a

204
a (1 + λb n )
că x n = , (∀) n ∈ N*. 12. Cum bn = b1 ⋅ qn–1, (∀) n ∈ N*, an = bn, (∀) n ∈ N*, ceea ce arată că
1 − λb n
10 10
q 20 − 1
(an)n≥1 este progresie geometrică. De aici, S = ∑
k =1
a k bk = ∑
k =1
a 2k = a12 (1 + q 2 + ... + q16 + q18 ) = a12 ⋅
q2 − 1
.

13. Folosim metoda reducerii la absurd. Presupunem că există asemenea progresii. Fie progresia (bn)n≥1, b1, q ∈
1 − qn
∈ N*. De aici obligatoriu b1 = x2, x ∈ N*. Pe de altă parte, b1 + b 2 + ... + b n = b1 ⋅ este pătrat perfect, de
1− q
1 − qn
unde 1 + q + q 2 + ... + q n −1 = este pătrat perfect, (∀) n ≥ 2. Pentru n = 2, y2 = 1 + q, pentru n = 4 avem 1 +
1− q
+ q + q2 + + q3 = (1 + q)(1 + q2) = z2. OobŃinem că y2(1 + q2) = z2, de unde obligatoriu 1 + q2 = l2, l ∈ N. Acest
lucru este imposibil căci q2 < 1 + q2 < (q + 1)2 şi deci q < l < q + 1, fals, căci între două numere naturale
consecutive nu avem un număr natural.

3.4. NoŃiunea de funcŃie. EcuaŃii funcŃionale


1. Vom folosi metoda inductivă în ambele situaŃii. Observăm că funcŃia f se poate transcrie recurent astfel: f(k) =
= 2 + f (k − 1) , unde k = numărul de radicali; a) Evident f(n) ≥ 0, (∀) n ∈ N*. Vom arăta că f(n) < 2, (∀) n ∈

∈ N*. Aici intervine inducŃia. Avem P(1): f(1) = 2 < 2 . Presupunem P(n): f(n) < 2 şi avem atunci f(n + 1) =
= 2 + f (n ) < 2 + 2 = 2 , ceea ce încheie demonstraŃia; b) Arătăm că f este monoton crescătoare. Tot inductiv

P(1): f(1) < f(2). Într-adevăr 2 < 2 + 2 evident. Analog f (n ) < 2 + f (n ) , de unde f(n) < f(n + 1).
2. Pentru y = 0 ⇒ f(x) ≤ f(0), ∀ x ∈ R. Pentru y = –x ⇒ f(0) ≤ f(–x2), ∀ x ∈ R. Rezultă f(0) ≤ f(–x2) ≤ f(0), deci
f(t) = f(0), ∀ t ≤ 0. Înlocuind pe x cu –x şi y = –x, avem f(–2x) ≤ f(x2), ∀ x ∈ R. Astfel pentru x ≥ 0 avem f(0) ≤
≤ f(x2) ≤ f(0), deci f(t) = f(0), ∀ t ≥ 0. În concluzie f(x) = a, a constantă reală, pentru orice x ∈ R. 3. a) Avem
f(x + 2) = f(x) – f(x – 2), de unde f(x + 4) = f(x + 2) – f(x) = –f(x – 2), ∀ x ∈ R, deci f(x + 6) = –f(x), ∀ x ∈ R.
x x
Rezultă f(x + 12) = –f(x + 6) = f(x), ∀ x ∈ R. Prin urmare T = 12 este perioadă a funcŃiei; b) f ( x ) = − =
12 12 
x
=   este periodică de perioada T = 12, dar nu verifică cerinŃa. De exemplu pentru x = 2 ar trebui să avem
12 
1  1 
f(4) + f(0) = f(2), deci   =   , fals. 4. Dacă în relaŃie luăm x = y = 1, găsim f2(1) + f2(a) = 2f(1). Cum f(a) =
3 6 
a a
= 1, obŃinem f(1) = 1. Fie y = 1. Găsim f(x) ⋅ f(1) + f   ⋅ f (a ) = 2f ( x ) , de unde f ( x ) = f   , (∀) x ∈ (0, ∞).
x x
a a
Dacă y = , găsim f ( x ) ⋅ f   = 1 , de unde f2(x) = 1, adică f(x) = 1 sau f(x) = –1. Arătăm că f(x) = 1 este
x x
 a 
funcŃia căutată. Luând x = x , y = x , avem 0 ≤ f 2 ( x ) + f 2   = 2f ( x ) , de unde f(x) ≥ 0, adică funcŃia
 x
este f(x) = 1. 5. Vom arăta că funcŃia căutată este f(x) = [x]. Presupunem că există o altă funcŃie diferită de f(x) =

205
= [x]. Fie k ∈ Z. Evident f(k) ∈ [k, k + 1], din strict crescătoare. Evident f(k) = k, f(k + 1) = k + 1. Presupunem
că (∃) a ∈ (k, k + 1) astfel încât f(a) = k + 1. Cum f(na) ≥ nf(a) = n(k + 1), cum f monoton crescătoare, urmează
că na > n(k + 1) – 1 adică na > nk + n – 1. Pe de altă parte, avem nk < na < nk + n, de unde na + 1 = nk + n sau
na = nk + n – 1, adică 0 > 0, fals. Deci singura funcŃie va fi f(x) = [x].
6. a) În relaŃia de recurenŃă f(k) = 2f(k – 1) + 2k, dăm lui k valorile 1, 2, 3, ..., n şi înmulŃim relaŃiile obŃinute cu
2, 22, 23, ..., respectiv 2n–1, începând cu penultima spre prima. Adunând membru cu membru, obŃinem că f(n) =
= 2nf(0) + 2 ⋅ [1 ⋅ 2n–1 + 2 ⋅ 2n–2 + 3 ⋅ 2n–3 + ... + (n – 1) ⋅ 2 + n] = 2n + 2 ⋅ 2n–1 + + 3 ⋅ 2n–2 + ... + 22(n – 1) + 2n, (1).
Pentru calculul sumei din (1) procedăm astfel: notăm S = 2n + 2 ⋅ 2n–1 + 3 ⋅ 2n–2 + ... + 23(n – 2) + 22(n – 1) + 2n,
(2). ÎnmulŃim (2) cu 2, obŃinând 2S = 2n+1 + 2 ⋅ 2n + 3 ⋅ 2n–1 + ... + 24(n – 2) + 23(n – 1) + 22n, (3). Scăzând
membru cu membru (2) din (3), găsim că S = 2n+1 + 2n + 2n–1 + ... + 23 + 22 – 2n = 22(2n – 1) – 2n = 2n+2 – 4 – 2n =
= 2n+2 – 2(n + 2). În final, obŃinem că f(n) = 2n+2 – 2(n + 2). Din ultima relaŃie rezultă evident că f(n) < 2n+2,
(∀) n ∈ N; b) Presupunem că (∃) n ∈ N astfel încât 2n+2 – 2(n + 2) = 2k, (4) pentru orice n ∈ N. Pentru n = 1,
8 – 6 = 2k, de unde k = 1. Pentru n = 2, 16 – 8 = 2k, de unde k = 3. Arătăm că pentru n ≥ 3, egalitatea (4) nu este
posibilă. Prin inducŃie avem 2n > n + 2, n ≥ 3. Dacă k ≤ n, atunci 2n+2 – 2k ≥ 2n+2 – 2n = 2n ⋅ 3 > 3(n + 2) > 2(n + 2).
Rezultă k ≥ n + 1 şi cum k < n + 2, obŃinem k = n + 1. ObŃinem 2n = n + 2, imposibil pentru n ≥ 3.
1 1 2
În concluzie, avem: n = 1 sau n = 2. 7. Înlocuim x cu . ObŃinem sistemul f (x) + 2f   = x 2 + 3 + 2 ,
x x x
1 1 2007
1 2007
1
2f (x) + f   = 2 + 3 + 2x 2 care, rezolvat, conduce la f(x) = x2 + 1. De aici imediat
x x
∑ f (x) − 2 = ∑ k 2 − 1 =
k =2 k =2

1 2007
 1 1   1 1 1  1 6041069
=
2
∑  k − 1 − k + 1  = 1 + 2 − 2007 − 2008  ⋅ 2 = 8060112
. 8. Înlocuind x cu 1 – x, obŃinem sistemul de
k=2

2f ( x ) + 3f (1 − x ) = 4x − 1 11
ecuaŃii  , (∀) x ∈ R. Prin rezolvarea sistemului, obŃinem că f ( x ) = −4x + , (∀) x ∈ R,
3f ( x ) + 2f (1 − x ) = 3 − 4x 5
soluŃie care verifică relaŃia iniŃială. 9. Folosind a) şi fixând m, obŃinem: pentru m = 0, n = 0, obŃinem f(0) = 2 ⋅
⋅ f(0) – 1, de unde f(0) = 1. Pentru m = 1, n = 0, obŃinem f(1) = f(1) + f(0) + 2 ⋅ 1 ⋅ 0 – 1; pentru m = 1, n = 1,
obŃinem f(2) = f(1) + f(1) + 2 ⋅ 1 ⋅ 1 – 1; pentru m = 1, n = 2, obŃinem f(3) = f(1) + f(2) + 2 ⋅ 2 – 1; pentru m = 1,
n = 3, obŃinem f(4) = f(1) + f(3) + 2 ⋅ 3 – 1; ...... ; pentru m = 1, n = k – 1, obŃinem f(k) = f(1) + f(k – 1) + 2 ⋅ (k –
–1) – 1, (1). Adunând în (1) membru cu membru, obŃinem: f(k) = k ⋅ f(1) + 2(1 + 2 + ... + k – 1) = k ⋅ f(1) + k(k –
–1) – k = k ⋅ f(1) + (k – 1)2, (2). Până acum am folosit doar condiŃia a). Pentru a determina f(1) folosim acum şi
2f (1) + 1 = f (2) 3
b) Avem  , care conduce la 2f(1)2 – 3f(1) – 9 = 0, cu soluŃiile f(1) = 3 sau f(1) = − , care
 2 f ( 2) + 7 = f (1) ⋅ f ( 2 ) 2
nu convine, deoarece f: N → N. În final, obŃinem f(n) = n ⋅ 3 + (n – 1)2, de unde f(n) = n2 + n + 1. 10. a) Avem 0 ≠
≠ 1, iar f(0) = f(1) = 0; b) Vom arăta că g nu este surjectivă. Pentru aceasta vom arăta că (∀) y ∈ (0, 2) ∩ Q, nu
au imagine niciun n ∈ N, sau cu alte cuvinte, (∀) n ∈ N, g(n) ∉ Q – {0}, iar g(0) = 0 ∈ Q. Putem rescrie funcŃia
astfel: g(n ) = n + n + 1 − [ n ] − [ n + 1] . Arătăm că n + n + 1 ∈ R \ Q. Într-adevăr, fie t = n + n + 1 ∈ Q,
atunci t2 = 2n + 1 + 2 n (n + 1) . Deducem n (n + 1) ∈Q, n ∈ N*, fals (deoarece n < n (n + 1) < n + 1, ∀ n ∈ N*).
Concluzia se impune. 11. Pentru n = 1, avem i) f(1) ≤ 3 şi ii) f(1) pătrat perfect, de unde în mod imperios f(1) =
= 1, (1). Pentru n = 2, i) f(2) ≤ 7 şi ii) f(1) + f(2) = pătrat perfect. Din i) f(2) ∈ {1, 2, ..., 7} şi ii) f(1) + f(2) ∈ {1,
4}, rezultă că f(2) = 3, (2). Analog pentru n = 3, găsim f(3) ≤ 11 şi f(1) + f(2) + f(3) = pătrat perfect. De aici

206
f(1) + f(2) + f(3) ∈ {1, 4, 9}, de unde f(3) = 5, (3). Intuim că f(n) = 2n – 1, n ∈ N*, pe care o presupunem
adevărată. Arătăm f(n + 1) = 2n + 1. Pentru aceasta avem f(n + 1) ≤ 4n + 3 şi în plus f(1) + f(2) + ... + f(n) +
+ f(n + 1) pătrat perfect. Avem f(1) + f(2) + ... + f(n) + f(n + 1) = 1 + 2 + ... + 2n – 1 = n2. De aici n2 + f(n + 1) ≤ n2 +
+ 4n + 3 = n2 + 2n + 4 + 2n + 2; n2 + f(n + 1) < (n + 1)2 + 2n + 2, de unde n2 + f(n + 1) = (n + 1)2, de unde f(n +
+ 1) = 2n + 1, concluzia dorită. 12. Folosim proprietatea că [a + 1] = [a] + 1, (∀) a ∈ R; a) Avem
 1  1  2  1  2  1
f  x +  =  x +  +  x +  + [ x + 1] − [3x + 1] =  x +  +  x +  + [ x ] − [3x ] = f ( x ) ; b) Pentru x ∈ 0,  ,
 3   3   3   3   3   3
1
avem f(x) = 0, (1). Cum f este periodică de perioadă , folosind (1), deducem că f(x) = 0, (∀) x ∈ R. 13. Avem
3
f (x + a) + 3 f (x ) − 3 f (x + a) − 3 f (x ) + 3
f(x + 2a) = = , iar f ( x + 3a ) = = = f ( x ) . Aşadar, f este
1 − f (x + a) 3 1 + f (x) 3 1 + f (x + a ) 3 1 − 3f ( x )
funcŃie periodică, cu perioada T = 3a. 14. a) Pentru a arăta că f este inversabilă, arătăm că f este bijectivă.
Injectivitatea. i) x1, x2 ∈ Q, f(x1) = f(x2) implică –x1 + 2008 = –x2 + 2008, de unde x1 = x2; ii) x1, x2 ∈ R – Q,
f(x1) = f(x2) implică 2009 – x1 = 2009 – x2, de unde x1 = x2; iii) x1 ∈ Q, x2 ∈ R – Q (sau cazul simetric), cum
x1 ≠ x2, avem f(x1) = –x1 + 2008 ∈ Q şi f(x2) = 2009 – x2 ∈ R – Q, de unde f(x1) ≠ f(x2). Surjectivitatea. Fie y ∈ R
şi ecuaŃia f(x) = y, x ∈ R. Dacă x ∈ Q, atunci avem de rezolvat ecuaŃia –x + 2008 = y şi are soluŃie unică x =
= 2008 – y numai dacă y ∈ Q. Dacă x ∈ R – Q, atunci rezolvăm ecuaŃia 2009 – x = y, cu soluŃia x = 2009 – y,
2008 − x , x ∈ Q
care este soluŃie numai dacă y ∈ R – Q. Cum f  f = 1, f–1 = f, deci f–1: R → R, f–1(x) =  .
2009 − x, x ∈ R − Q
b) Să observăm că 2009 nu este pătrat perfect. Rezultă că 2009 − 2009 ∈ R – Q şi cum 2009 ∈ Q, obŃinem
f(2009) + f (2009 − 2009 ) = −1 + 2009 ; c) Cum –1 ∈ Q şi x = f–1(–1) = –(–1) + 2008 = 2009; d) EcuaŃia
2008− a, a ∈Q (1)
f(x) = a are soluŃia x = f–1(a) =  , (∀) a ∈ R. 15. (1). Fie f(x + 1) = {x + 1} + {2x + 2} + {3x + 3} =
2009− a, a ∈ R − Q
(1)
= f ( x ) , deci 1 este o perioadă. Arătăm că este perioadă principală. Pentru aceasta folosim metoda reducerii la
absurd. Presupunem că (∃) t ∈ (0, 1) astfel încât f(x + t) = f(x), (2). Din (2) avem f(0) = 0 şi f(0 + t) = f(t) ≠ 0. Ar
trebui ca 0 ≠ 0, absurd. Urmează că 1 este perioadă principală; b) Este suficient să rezolvăm inecuaŃia pe
 1  1   1   2 
intervalul [0, 1). Explicitând părŃile fracŃionare, obŃinem S = 0,  ∪   ∪   ∪   , pe care o extindem
 6  3  2   3 
prin periodicitate.

3.5. FuncŃia de gradul I şi funcŃia de gradul II


1. Fie f: R → R, f(x) = a2 + bx + c. Considerăm mulŃimile B = {x ∈ R | f(x) = x} şi C = A – B, adică x ∈ C dacă
f(x) ≠ x, dar f(f(x)) =x, x ∈ R. Evident A = B ∪ C. Avem card A = card B + card C – card (B ∩ C). Dacă
card B ≤ 1, ecuaŃia f(x) = x are o singură soluŃie, adică f(x) – x ≥ 0, (∀) x ∈ R sau f(x) – x ≤ 0, (∀) x ∈ R.
EcuaŃia f(f(x)) = x este echivalentă cu f(f(x)) – f(x) = x – f(x), (∀) x ∈ R, sau f(f(x)) – f(x) = –(f(x) – x), (∀) x ∈
∈ R. De aici C = ∅. Prin urmare card A = card B ≤ 1. Presupunem acum că card B = 2. Dacă C ≠ ∅, (∃) x0 ∈ R
astfel încât f(x0) ≠ x0 şi f(f(x0)) = x0. De aici (f  f)(f(x0)) = f(x0), adică f(x0) ∈ A. Dacă f(x0) ∈ B, avem f(f(x0)) =

207
= f(x0), adică f(x0) = x0, contradicŃie. Cum card A ≤ 4 iar card C ≥ 2, de unde card A ≤ 2. Prin urmare card A ≠
b b
≠ B. 2. a) Fie x1, x2 ∈ R – Q, x1 ≠ x2, x1 = 2 − , x2 = − 2 − ; f ( x1 ) − f ( x 2 ) = ax12 + bx1 + c − ax 22 − bx2 –
2a 2a
– c = a(x1 – x2)(x1 + x2) + b(x1 – x2) = (x1 – x2)[a(x1 + x2) + b] = (x1 – x2)(–b + b) = 0, de unde f(x1) = f(x2), ceea
ce arată că f nu este injectivă; b) Fie x1, x2 ∈ Q. Din g(x1)= g(x2) ajungem la (x1 – x2) ⋅ [a(x1 + x2) + b] = 0. De
b b
aici x1 = x2 dacă şi numai dacă ∈ R – Q (x1 + x2 ≠ − în acest caz, x1, x2 ∈ Q). 3. Fie f(x) = ax + b, a ≠ 0.
a a
y − y1
A(x1, y1), B(x2, y2) ∈ Gf, unde x1, x2, y1, y2 ∈ Z, x1 ≠ x2. Deci f(x1) = y1 şi f(x2) = y2. De aici a = 2 şi b =
x 2 − x1
= y1 − ax1 , deci a, b ∈ Q. Dar f ( 3 ) = 3( 3 + 1) = a 3 + b . De aici 3 3 + 3 = a 3 + b . De aici (a – 3) 3 = b –
b−3
– 3. Dacă a ≠ 3, atunci 3=
∈ Q absurd. Obligatoriu a = 3, de unde b = 3. Prin urmare f(x) = 3x + 3.
a −3
4. Răspunsul este NU. Într-adevăr, dacă ar exista funcŃii obŃinem: (∀) y ∈ R*, f(y2) = ay + b, f(–y)2) = –ay + b,
de unde a = 0, absurd.
5. a) Pentru existenŃa rădăcinilor reale trebuie să fie îndeplinită condiŃia ∆ ≥ 0, (∀) m ∈ (0, ∞). Însă ∆ =
= 4(m2 + 3m2 + 2)2 + 8m(m + 2)2 > 0, (∀) m ∈ (0, ∞), ceea ce face ca cerinŃa problemei să fie îndeplinită.
 1   1
b) Pentru ca x1 ∈  − , 0  trebuie ca f  − f (0) < 0 , iar pentru ca x2 ∈ (0, 1), trebuie ca f(0) ⋅ f(1) < 0. Avem
 2   2
 1 − m(3m 2 + 8m + 8)
f  −  ⋅ f (0) = < 0 , (1), (∀) m ∈ (0, ∞); f(0) ⋅ f(1) = –m(m + 4) < 0, (2), (∀) m ∈ (0, ∞).
 2 2
1
RelaŃiile (1) şi (2) arată că într-adevăr avem − < x1 < x 2 < 1 . 6. a) Folosim relaŃiile lui Viete: S = x1 + x2 = 6;
2
P = x1x2 = 3; S2 = x12 + x 22 = ( x1 + x 2 ) 2 − 2x1x 2 = 36 − 6 = 30 ; S3 = x13 + x 32 = ( x1 + x 2 )[( x1 + x 2 ) 2 − 3x1x 2 ] =
= 6[36 − 9] = 6 ⋅ 27 = 162 ; S4 = ( x12 + x 22 ) 2 − 2x12 x 22 = 900 − 18 = 882 ; b) Vom demonstra acest lucru inductiv.
i) Etapa de verificare: n = 1, S1 = 6, de unde S ⋮ 6; ii) Etapa de demonstraŃie: presupunem că x1k + x k2 ⋮ 3
şi încercăm să demonstrăm că x1k +1 + x k2 +1 ⋮ 3 . Într-adevăr, x1k +1 + x k2 +1 = ( x 1k + x k2 )(x 1 + x 2 ) − x 1 x k2 − x 2 x 1k =
= 6( x 1k + x k2 ) − x 1 x 2 ( x k2 −1 + x 1k −1 ) = 6( x 1k + x k2 ) − 3( x k2 −1 + x 1k −1 ) = 3[ 2 ( x 1k + x k2 ) − ( x k2 −1 + x 1k −1 )] , de unde x1k+1 +
+ x k2 +1 ⋮ 3. 7. Procedăm prin reducere la absurd şi presupunem că niciuna dintre ecuaŃii nu are soluŃii reale.
Atunci ∆i = b i2 − 4 aici < 0, i = 1, 3 şi cum ai > 0, i = 1, 3 , deducem că aix2 + bix + ci > 0, i = 1, 3 . Imediat avem
că 102 a1x 2 + 102 b1x + 102 c1 > 0; 10a 2 x 2 + 10b2 x + 10c 2 > 0; a 3 x 2 + b3 x + c3 > 0 , (1). Adunând în (1) membru cu
membru, găsim Ax2 + Bx + C > 0, (∀) x ∈ R, ceea ce contrazice faptul că ecuaŃia Ax2 + Bx + C = 0 are soluŃii.
8. Fie ∆1 şi ∆2 discriminanŃii celor două ecuaŃii, ∆1 = 4 ( b − ac ) , (1) , ∆2 = 4 ( a + c − b ) , ( 2 ) . Cum ∆1 > 0, avem că ac
< b, (3) şi cum a, c ∈ N* avem (a – 1)(c – 1) ≥ 0, de unde a + c ≤ 1 + ac, (4). De aici 4(a + c – ac) ≤ 4, (5). Dar din
(2), –ac > –b, a + c – ac > a + c – b ≥ 4(a + c – ac) > 4(a + c – b) = ∆2, de unde 0 < ∆2 ≤ 4 . Cum
a, c ∈ N* şi 0 < a + c – ac ≤ 1, avem doar situaŃia a + c – ac = 1, relaŃie echivalentă cu (a – 1)(1 – c) = 0; i) Dacă
a = 1, atunci ∆2 = 4(1 + c – b). Din (2), b > c, de unde b, c ∈ N*, b – c ≥ 1, (6) şi cum ∆2 = 4(1 + c – b), rezultă
că ∆2 ≤ 0, din (6); ii) Dacă c = 1, atunci b > a şi analog ∆2 = 4(1 + a – b) ≤ 0, adică ∆2 < 0. Astfel, în ambele

208
situaŃii, am găsit ∆2 ≤ 0, ceea ce arată că ecuaŃia a doua nu poate avea două soluŃii reale distincte. 9. Din relaŃiile
lui Viete, avem că x1 + x 2 = −a; x1 x 2 = b + n, prin urmare a, b ∈ Z, adică na2 + b2 ∈ N. Pe de altă
parte, na 2 + b2 = n(x1 + x 2 ) 2 + (x1 x 2 − n) 2 = (x12 + n)(x 22 + n) , ceea ce arată că na2 + b2 este compus. 10. Folosim
metoda reducerii la absurd: presupunem că în (x1, x2) există două pătrate perfecte, adică (∃) k ∈ N* astfel încât
x1 < k2 < (k + 1)2 < x2, (1). Ştim că ∆ = 4m + 1 > 0, adică rădăcinile sunt reale. Din Viete avem că
x1 + x2 = 2n + 1 > 0, x1x2 = m2 > 0, de unde x1, x2 ∈ (0, ∞). Revenind în (1), găsim că x1 < k < k + 1 < x 2 ,

de unde x2 − x 1 > 1 . Dar ( x 2 − x 1 ) 2 = x 1 + x 2 − 2 x 1 x 2 = 2 m + 1 − 2 m 2 = 1 , absurd. 11. EcuaŃia se


transcrie sub forma (x – a)2 + (x – b)2 + (x – c)2 = 0. De aici, x = a, x = b, x = c soluŃii, de unde în mod necesar a
= b = c. Astfel, (a – 1)2 + (a – 2)2 + (a – 3)2 = 3(a – 2)2 + 2 > 0. De aici, valoarea minimă a lui S va fi 2, realizată
pentru a = b = c = 2. 12. a) Avem că a + b + c = 1, 4a + 2b + c = 3, 9a + 3b + c = 2 , (1). SoluŃiile lui (1) vor fi
3 13 3 13
a = − , b = , c = −4 , deci funcŃia căutată va fi f(x) = − x 2 + x − 4 . ObservaŃie. Sistemul (1) a fost obŃinut
2 2 2 2
căutând o permutare σ: {1, 2, 3} → {1, 2, 3} pentru care σ  σ = 1, unde 1 este permutarea identică. FuncŃia
căutată va fi f(1) = 1, f(2) = 3, f(3) = 1; b) Metoda va fi reducerea la absurd şi ne bazăm pe ipoteza că a ≠ 0. Pentru
aceasta presupunem că există a, b, c ∈ Z. Cum x – y | f(x) – f(y) şi f(1) ≠ f(2) ≠ f(3) ≠ f(1), obŃinem că f(1) – f(2)
| f(f(1)) – f(f(2)), deci f(1) – f(2) ∈ {–1, 1}. Analog, f(2) – f(3) ∈ {–1, 1}, f(3) – f(1) ∈ {±1, ±2}.
Fie α = f(1) – f(2), β = f(2) – f(3), γ = f(3) – f(1). Cu α + β + γ = 0, avem posibilităŃile (α, β, γ) ∈ {(1, 1, –2),
(–1, –1, 2)}, în fiecare caz ajungând la concluzia a = 0, absurd! 13. Folosind relaŃiile lui Viete, găsim
că x1 + x 2 = a 2 + b 2 + c 2 + d 2 + 1 ∈ Z. Cum x1 ∈ Z, atunci şi x2 ∈ Z. Urmează că:
∆ = (a 2 + b 2 + c 2 + d 2 + 1) 2 − 4(ab + bc + cd + da ) < (a 2 + b 2 + c 2 + d 2 + 1) 2 . În plus ∆ şi (a2 + b2 + c2 + d2 + 1)2
au aceeaşi paritate. De aici urmează că ∆ ≤ (a2 + b2 + c2 + d2 + 1)2. Ultima inegalitate se scrie sub forma a2 + b2 +
+ c2 + d2 ≤ ab + bc + cd + da sau (a – b)2 + (b – c)2 + (c – d)2 + (d – a)2 ≤ 0, de unde a = b = c = d. EcuaŃia devine
x2 – (4a2 + 1)x + 4a2 = 0 şi are rădăcinile întregi x1 = 1, x2 = 4a2. 14. a) Pentru ca ecuaŃia dată să aibă o soluŃie
întreagă, trebuie ca numărul ∆x = 1 – 4α + 4y4 să fie pătrat perfect, adică (∃) k ∈ N astfel încât 1 – 4α + 4y4 = k2,
k − 2 y 2 = 1, (1)
adică 1 – 4α = k2 – 4y4 = (k – 2y2)(k + 2y2). Cum 1 – 4α este prim, avem  . Scăzând (1)
k + 2 y 2 = 1 − 4α, (2)
din (2), 4y2 = –4α sau –α = y2, ceea ce arată că –α este pătrat perfect. b) EcuaŃia se rescrie sub forma x2 + x – 4 –
– y4 = 0. De aici numărul ∆x = 4y4 + 17 trebuie să fie pătrat perfect, adică (∃) k ∈ N, astfel încât 4y4 + 17 = k2, de
unde (k – 2y2)(k + 2y2) = 17. De aici k − 2y2 = 1, k + 2y2 = 17 , prin urmare k = 9, y ∈ {–2, 2}, iar x ∈ {–5, 4}.
SoluŃiile vor fi deci (x, y) ∈ {(–5, –2), (–5, 2), (4, –2), (4, 2)}.

IV. Alte tipuri de probleme


1. Numărul numerelor naturale cuprinse între n2 – 5n + 6 şi n2 + n este (n2 + n) – (n2 – 5n + 6) + 1 = 6n – 5,
număr impar. Pe de altă parte n2 – 5n + 6 = n2 – n – 4n + 6 = n(n – 1_) – 4n + 6 ∈ 2N. Analog n2 + n = n(n +
+ 1) ∈ 2N. Primul număr impar este n2 – 5n + 7 iar ultimul n2 + n – 1. Numărul numerelor impare este 3n – 3, de

209
(n 2 − 5n + 7 + n 2 + n − 1) ⋅ 3 ⋅ (n − 1) (2n 2 − 4n + 6) ⋅ 3 ⋅ (n − 1)
unde suma lor va fi S = = = 3(n2 – 2n + 3)(n – 1) =
2 2
= 3(n – 1)(n2 – 2n + 3). 2. a) Dacă cele două numere sunt simultan pătrate perfecte, atunci cum x2 + 2y > x2, de
unde x2 + 2y ≥ (x + 1)2, de unde 2y ≥ 2x + 1, deci y > x. Analog obŃinem x > y, absurd; b) Din ipoteză x3 + 3y2 +
+ 3x + 1 > x3 deci x3 + 3y2 + 3x + 1 ≥ (x + 1)3, de unde x3 + 3y2 + 3x + 1 ≥ x3 + 3x2 + 3x + 1, de unde y ≥ x.
Analog x ≤ y, de unde x = y. Prin urmare nu există nici în acest caz numere naturale distincte cu proprietatea
cerută. 3. Dacă A este mulŃimea soluŃiilor şi n1 ∈ A şi îl fixăm, atunci pentru alte elemente n2, n3 ∈ A, avem
10 | n1 + n2, 10 | n1 + n3, de unde 10 | n2 + n3. Urmează că n1, n2, n3 au aceeaşi ultimă cifră. Numărul elemen-
 2009   2009   2009 
telor din A care au cifra 0 este   = 200 , iar cele care au ultima cifră 5 va fi  5  −  10  =
 10     
= 401 – 200 = 201. Prin urmare mulŃimea A va fi A = {5, 15, 25, ..., 2005} cu |A| = 201. 4. Presupunem n < m <
< k numere naturale astfel încât an = 9, am = 25, ak = 49. Fie raŃia r > 0. ObŃinem: a1 + (n – 1)r = 9, a1 + (m –
– 1)r = 25, a1 + (k – 1)r = 49. Imediat (m – n)r = 16, (k – m)r = 24, (k – n)r = 40. Mai departe (k – 2m + n)r = 8.
Notăm p = k – 2m + n > 0. Avem ak+l = ak + lr. Vrem să determinăm un indice s multiplu de p, adică ak+ps =
= 2009. Cum ak+ps = ak + psr, de unde ak+ps – ak = 2009 – 49. Imediat 8s = 1960, de unde s = 245. 5. a) Avem A =
= 27 ⋅ 1015 + 1 = 33 ⋅ (105)3 + 1 = (3 ⋅ 105)3 + 13 = (3 ⋅ 105 + 1) ⋅ [(3 ⋅ 105)2 – 3 ⋅ 105 + 1] = (3 ⋅ 105 + 1)(9 ⋅ 1010 –
– 3 ⋅ 105 + 1); b) Fie C = 125 ⋅ 1015 + 125 ⋅ 103 + 1 = 53 ⋅ 1015 + 53 ⋅ 103 + 1; D = 75 ⋅ 106 = 3 ⋅ 52 ⋅ 106. Atunci,
C = (5 ⋅ 105)3 + (5 ⋅ 10)3 + 1; D = 3 ⋅ (5 ⋅ 105)(5 ⋅ 10) ⋅ (1). Notăm a = 5 ⋅ 105, b = 5 ⋅ 103, c = 1. Astfel, B = C –
–D = a3 + b3 + c3 – 3abc = (a + b + c)(a2 + b2 + c2 – ab – ac – bc), ceea ce arată că B nu este prim.
6. a) Vom demonstra inductiv proprietatea. Etapa de verificare: n = 1, obŃinem 1 + a ≥ 1 + a, cazul de egalitate.
Etapa de demonstraŃie: Presupunem că (1 + a)k ≥ 1 + ka, (1) şi arătăm că (1 + a)k+1 ≥ 1 + (k + 1)a, (2).
(1 )
Într-adevăr, (1 + a)k+1 = (1 + a)k(1 + a) ≥ (1 + ka)(1 + a) = 1 + ka2 + ka ≥ 1 + a + ka = 1 +(k + 1)a, adică (2);
b) Dacă r şi q sunt raŃiile progresiei aritmetice, respectiv progresiei geometrice, atunci, cum an = a1 + (n – 1)r şi
bn = b1 ⋅ qn–1, (∀) n ∈ N*; i) Dacă r = 0, atunci an = a1 = b1 şi a2 = b2, de unde q = 1, astfel încât an = bn =
= a1, (∀) n ∈ N*; ii) Dacă r ≠ 0, arătăm că r > 0. Presupunem prin absurd că r ≤ 0. Cum an > 0, (∀) n ∈ N,
a −r
obŃinem a1 + (n – 1)r > 0. De aici (n – 1)r > –a1, de unde (n – 1)(–r) < a1, adică n < 1 , absurd, căci ar urma
−r
că mulŃimea numerelor naturale ar fi mărginită. De aici, cum an > 0, găsim a2 > a1 şi deci b2 > b1, de unde, cum
Bernoulli
b1 > 0, q > 1, găsim că r = a1(q – 1), (1). Pe de altă parte avem: [1 + (q – 1)]n–1 ≥ 1 + (q – 1)(n – 1), de unde
q ≥ 1 + (q – 1)(n – 1), (2). ÎnmulŃind în (2) cu a1, găsim a1 ⋅ q ≥ a1 + a1(q – 1)(n – 1), de unde Ńinând cont de
n–1 n–1

(2) şi de ipoteza a1 = b1, obŃinem b1 ⋅ qn–1 ≥ a1 + (n – 1)r, sau cu alte cuvinte cerinŃa bn ≥ an, (∀) n ∈ N.
7. Observăm că numărul pătratelor eliminate prima dată este 1, a doua oară, 8, a treia oară, 64, ş.a.m.d. Pe de altă
1 1 1
parte aria pătratelor eliminate este: prima dată , a doua oară 8 ⋅ , a treia oară 82 ⋅ , ..., a n-a oară
9 81 729
2008 2 2008 k 2008
 1  8 8
∑ ∑
1 1
8n −1 ⋅ . Suma ariilor pătratelor eliminate este dată de suma 8 k −1 ⋅  k  =   =1−   .
9n k =1 3  8 k =1  
9 9
(Am folosit suma unei progresii geometrice). 8. Fie 2k + 1 numere prime mai mari ca 2. Considerăm resturile
împărŃirii la 2k+1 a numerelor prime date. Cum numerele prime sunt impare, toate cele 2k + 1 resturi aparŃin
mulŃimii {1, 3, 5, ..., 2k+1 – 1}. Dar această mulŃime conŃine 2k numere, ceea ce înseamnă că printre cele 2k + 1
resturi, care obligatoriu există, sunt două egale, ceea ce se traduce că printre numerele prime date există două

210
care prin împărŃirea la 2k+1 dau acelaşi rest. Urmează că diferenŃa lor se divide cu 2k+1. 9. a) Primul element din
linia n este n2 – n + 1, iar primul element de pe linia următoare este (n + 1)2 – (n + 1) + 1 = n2 + n + 1. Deducem
că ultimul termen de pe linia n este n2 + n – 1. În consecinŃă, termenul din mijlocul unei linii impare este media
aritmetică a primului şi ultimului (am Ńinut cont că fiecare linie este o progresie aritmetică de raŃie 2), de unde
n2 − n +1+ n2 + n −1
avem = n 2 , ceea ce arată că termenul căutat este pătrat perfect; b) Suma elementelor de pe
2
n[(n 2 − n + 1) + (n 2 + n − 1)]
o linie o putem calcula folosind suma progresiei aritmetice: S = = n 3 . 10. Vom
2
1 1 1 1 1  1 1
demonstra prin inducŃie matematică. Pentru n = 2, = = + = + , prin
2009 7 ⋅ 287 294  7 287  7 ⋅ 294 287 ⋅ 294
1 1 1
urmare există numerele distincte x1 = 7 ⋅ 294, x2 = 287 ⋅ 294, astfel încât = + . Presupunem
2009 x1 x 2
proprietatea adevărată pentru un k şi arătăm că are loc şi pentru k + 1. Deci presupunem că există x1, x2, ... , xk ,
1 1 1 1 1 1 1 1
x1 distincte astfel încât = + ... + . Avem = + + + ... + . Evident xk ≠ 2009.
2009 x1 xk 2009 4018 2x1 2 x 2 2x k
1 1 1 1
Cum x1, x2, ..., xk distincte, 2x1, 2x2, ..., 2xk distincte, avem că = + + ... + . 11. a) Un exemplu
2009 x1 x 2 x k +1
ar fi funcŃia f(1) = 2, f(2) = 3, f(3) = 1; b) Vom arăta că doar pentru mulŃimile care au card M = 3 putem construi
funcŃii cu proprietatea (P). Pentru aceasta folosim metoda reducerii la absurd. Presupunem card M ≥ 4. Fie
M = {a, b, c, d} distincte două câte două. Pentru A = {a}, B = {b} avem f(a) = b sau f(b) = a. Dacă f(a) = b,
atunci f(a) ≠ c, de unde f(c) = a, deci f(c) ≠ b, de unde f(c) = b. Apoi, din f(a) ≠ d rezultă f(d) = a; f(d) ≠ b implică
f(b) = d. Dar f(b) = c, de unde d = c, fals. În concluzie, M are cel mult trei elemente şi anume exact trei elemente,
folosind ipoteza. Dacă M = {a, b, c}, observăm că funcŃiile cu proprietatea că au proprietatea P nu au puncte
fixe. Singurele funcŃii care sunt bijecŃii şi nu sunt monotone sunt f: M → M, f(a) = b, f(b) = c, f(c) = a şi g: M → M,
g(a) = c, g(b) = a, g(c) = b. 12. a) MulŃimea A este pătratul care are vârfurile de coordonate (5, 0); ((0, 5);
2
(–5, 0) şi (0, –5). b) Aria pătratului de 50 cm2 o împărŃim în 100 pătrate de latură . Fie un pătrat de acest tip
2
2
MNPQ de latură . Existând 100 pătrate de acest tip şi având 101 puncte, conform principiului lui Dirichlet,
2
într-un pătrat există cel puŃin 2 puncte. Cum într-un pătrat distanŃa maximă dintre două puncte se realizează
pentru cele diagonal opuse şi cum în pătrat diagonala are lungimea 1, urmează concluzia: în pătrat întotdeauna
există două puncte din cele 101 situate la o distanŃă mai mică decât 1. 13. Avem inegalitatea între media
m+ n + p
 ma + nb + cp 
aritmetică ponderată şi media geometrică: a m ⋅ bn ⋅ cp ≤   , egalitatea realizându-se pentru a =
 m+n+p 
m+n +p
k k
= b = c. Cum a + b + c = k, obŃinem a = b = c = . Valoarea maximă a produsului este   . În cazul nostru
3 3
9
m = 2, n = 3, p = 4, iar cum a + b + c = 18, k = 18. ObŃinem că maximul căutat este 6 .

211
GEOMETRIE

V. Vectori în plan

5.1. OperaŃii cu vectori


1. a) Presupunem ABCD paralelogram şi P un punct din planul său. Atunci PA + PC = 2PO = PB + PD , unde
{O} = AC ∩ BD. Reciproc: Fie O1 mijlocul lui AC . Atunci PA + PC = 2PO1 . Analog, O2 mijlocul lui BD
conduce la PB + PD = 2PO 2 . łinând cont de ipoteză, PO1 = PO 2 , de unde O1 ≡ O2, ceea ce arată că
diagonalele patrulaterului ABCD se înjumătăŃesc, adică ABCD paralelogram; b) Folosim a). În locul punctului P
considerăm, de exemplu, punctul A. Dacă arătăm că AS + AR = AP + AQ , urmează că PSRQ paralelogram. Dar
AF + AD AF + AC 2AF + AD + AC AB AN AB
AS + AR = + = = + AN = AP + AN + AB = AP + + = AP +
2 2 2 2 2 2
AN + AB C'
+ = AP + AQ , ceea ce încheie demonstraŃia.
2
2. Avem imediat, construind CC' || BD, că AC + BD = AC + CC' = AC' ; E D
AC + BD = AC' = 3l .
F O C
3. Avem XY = XM + MD + DY , (1) şi XY = XB + BN + NY , (2). Adunând membru
1 1 1
cu membru (1) cu (2), găsim 2XY = MD + BN , de unde XY = MD + BN = − ⋅
2 2 8 A B
1 1 1 1
⋅ AB + AD , (3). Dar PM = PA + AD + DM = PA + AD + AB = AD + AB + PB + BA = AD + AB − AB +
3 4 4 4
BA + BN 1 AB BN 1 BN 1 1 2 1
+ = AD + AB − AB + − = AD − AB − = AD − AB − AD = AD − AB = 2 ⋅
2 4 2 2 4 2 4 3 3 4
 1 1  1
⋅  − AB + AD  = 2XY . Avem deci XY = PM .
 8 3  2
4. Fie O un punct al planului. łinând cont că a + b = 1 şi AD = a BA + bBC + CD devine OD − OA = a (OA −
− OB) + (1 − a )(OC − OB) + OD + OC , de unde AB = a CA , ceea ce arată că punctele A, B, C sunt coliniare.
5. CondiŃia ca să existe un triunghi (chiar degenerat) este să existe relaŃia AA' + BB' + CC' = 0 . Într-adevăr,
AA' + BB' + CC' = AG + GA' + BG + GB' + CG + GC' = (AG + BG + CG ) + (GA' + GB' + GC') = 0 + 0 = 0 .
6. Ideea de demonstraŃie este de a arăta că T este centrul de greutate al triunghiului
A
ABC. Pentru aceasta: fie N şi M mijloacele segmentelor DC respectiv BC. Avem,
Ńinând cont de ipoteză, că DA + DB + EA + EC = 0 ⇔ 2 AD + DA + DB + EA +
D N E
+ EC = 2(AD + AE) ⇔ AB + AC = 2(AD + AE) , (1). Dar, pe de altă parte, alegerea
   T
lui M şi N ca mijloace implică relaŃiile 2AM = AB + AC , (2) şi 2AN = AD + AE ,
B M C

212
(3) łinând cont de (1), (2) şi (3) ajungem la 2AM = 4AN sau AM = 2AN , (4), relaŃie ce arată că punctele A,
M, N sunt coliniare.
În plus avem că N este mijlocul lui AM. Cum N era şi mijlocul lui DE, urmează că ADME este paralelogram. De
aici ME || AB şi cum M era ales mijlocul lui BC, urmează că ME linie mijlocie în ∆ABC. Analog: DM linie
mijlocie. De aici avem BE şi CD mediane în ∆ABC, de unde T este centrul de greutate al triunghiului ABC. De
11
aici TB + TC = 2TM . Dar TM = AT , de unde TB + TC = AT ⇔ TB + TC = −TA . Ultima relaŃie produce
2
soluŃia: α = –1.
7. a) Avem 2IE = IA + IB , (1); 2IF = IC + ID , (2). Adunând relaŃiile (1) D
şi (2) şi Ńinând cont că IE + IF = 0 , obŃinem relaŃia dorită IA + IB + IC + F
I
+ ID = 0 ; b) În ∆CFE avem medianele EJ şi CI, de unde urmează că M A M J
2 1
este centrul de greutate al ∆CEF. De aici avem că AM = AI + AC ⇔ E C
3 3 B
AF + AE
⇔ 3AM = 2AI + AC ⇔ 3AM = 2 ⋅ + AC ⇔ 3AM = AF + AE + AC .
2
8. a) Cum AD = AB + BD , avem AD + CB = AB + BD + CB = AB + CD ;
MB + MD A B
b) În ∆BDM, MN mediană, de unde MN = , (1). Pe de altă parte
2
MB = MA + AB , (2), iar MD = MC + CD , (3). Din (1), (2) şi (3) obŃinem că N M

MA + AB + MC + CD
MN = . Cum M este mijlocul lui AC, avem MC + MA = D C
2
= 0 , de unde 2MN = AB + CD , (4); c) Analog ca la b), în triunghiul ANC, NM mediană conduce la MN =
AN + CN
= , (5). Pe de altă parte AN = AD + DN , (6), iar CN = CB + BN , (7), de unde folosind (5), (7), (7),
2
AD + DN + CB + BN
obŃinem că MN = . Cum N este mijlocul lui BD, avem DN + BN = 0 , de unde obŃinem
2
2MN = AD + CB , (8). Adunând (4) cu (8) membru cu membru, ajungem la: MN = AB + CD + AD + CB .
9. a) Triunghiul MPQ este echilateral căci, din paralelism, 'MPQ ≡ 'MQP = 60°. Cum MM1 ⊥ BC, atunci MM1
înălŃime în ∆MPQ echilateral, deci şi mediană, de unde 2MM1 = MP + MQ , (1);
b) Prelungim MP şi notăm {P'} = MP ∩ AC, QM şi notăm {Q'} = MQ ∩ AB. A
Construim prin M, TT' || BC, cu T ∈ AB, T' ∈ AC. Avem în ∆MTQ' echilateral,
Q'
MM3 înălŃime adică 2MM 3 = MQ' + MT , (2) şi analog în ∆MP'T' echilateral cu P'
M3
MM2 înălŃime, adică 2MM 2 = MP' + MT' , (3). Adunând (1) cu (2) şi (3), obŃinem M2
T T'
2(MM1 + MM 2 + MM 3 ) = MP + MQ + MT + MQ' + MP' + MT ' , (4). Pe de altă M

parte în paralelogramul Q'MP'A avem MA diagonală, de unde MA = MP' + MQ' , B P M1 Q C


(5). Absolut analog din paralelogramele MTBP şi MT'CQ avem că MB = MT +
+ MP , (6), respectiv MC = MQ + MT' , (7). Folosind (5), (6) şi (7) avem că: 2(MM1 + MM 2 + MM 3 ) = MP ' +
+ MQ' + MQ + MT' + MP + MT = MA + MC + MB .

213
10. a) ÎmpărŃim segmentul AD în 12 părŃi congruente, iar N îl luăm la 5 unităŃi de A. C
D
Absolut analog împărŃim BC în 11 segmente congruente, iar Q îl luăm la patru unităŃi
N
de B; b) Considerăm P mijlocul lui NQ. Fie M un punct oarecare în plan. Aplicând P
2 5 Q
formula vectorului de poziŃie, obŃinem că MN = MA + MD , adică 7MN = A
7 7
B
= 2MA + 5MD , (1). Absolut analog 3MB + 4MC = 7 MQ , (2). De aici 2MA +
M
(1) + ( 2)
+ 3MB + 4MC + 5MD = 7(MN + MQ) = 14MP , (3). Fie k valoarea constantă a sumei. Din (3) avem 14MP = k ,
k k
de unde | MP |= , (4). Prin urmare, conform cu (4) avem că locul geometric este cercul de centru N şi rază .
14 14
11. a) Fie O un punct oarecare al planului. Atunci: O1O 2 = OO 2 − OO1 = D C
OB + OD OA + OC OD − OA OC − OB AD − BC O1
= − = ⋅ = , (1);
2 2 2 2 2
O2 O
b) RelaŃia AD + CB = 3O1O 2 se scrie 3O1O 2 = AB − BC , de unde, Ńinând cont
A B
de (1) din a), obŃinem 3O1O 2 = 2O1O 2 , de unde O1O 2 = 0 , adică O1 = O2, ceea ce, conform ipotezei, arată că
ABCD este paralelogram.
12. Construim QO ⊥ CD, Q ∈ (PD). Fie M mijlocul coardei AB. Avem PA + OP + C
+ PB + OP = OA + OB = 2OM , (1). Cum perpendiculara din centru cercului pe o M P
A B
coardă împarte coarda în două părŃi egale, obŃinem QC = QD, de unde PQ = PD −
O Q
CD CD
− QD = PD − , (2). Pe de altă parte PQ = CQ − CP = − CP , (3). Adunăm
2 2
D
membru cu membru (2) cu (3) şi obŃinem 2PQ = PD − CP = PD + PC , (4). Cum
MOQP este dreptunghi, QP = OM , (5), adunând membru cu membru (1) cu (4), găsim că 2PQ + 2OM = PA +
+ PB + 2OP + PD + PC , de unde Ńinând cont de (5), obŃinem în final relaŃia dorită: PA + PB + PC + PD = 2PO .

13. a) Avem: MP = MB + BC + CP , (1); MP = MA + AD + DP , (2), de unde adunând C


P
(1) cu (2) obŃinem: 2MP = (MA + MB) + (BC + AD) + (CD + DP) = BC + AD , de
1 D N
unde MP = (BC + AD) . Absolut analog QN = QD + DC + CN , (3); QN = QA + T
2 Q
+ AB + BN , (4). Adunând membru cu membru (3) cu (4) suntem conduşi la 2QN = B
A M
1
= (QD + QA) + (AB + DC) + (CN + BN) = CD + AB , de unde QN = (AB + CD) ;
2
b) Fie T mijlocul diagonalei BD. Atunci 2AT = AD + AB , (5) iar 2CT = CD + CB , (6). Scăzând membru cu
membru (6) din (5), găsim: 2(AT + TC) = AD + AB + DC + BC = 2AC , (7). Pe de altă parte 2(MP + QN) =
= AB + CD + BC + AD , (8). În final, relaŃiile (7) şi (8) conduc la MP + QN = AC . D V
14. Fie U şi V mijloacele segmentelor AB respectiv CD, iar I mijlocul segmentului C
I
UV. ObŃinem: MA + MB + MC + MD = 2MU + 2MV = 2(MU + MV) = 4MI , ceea ce A U
arată că vectorul legat trece prin punctul fix I.
M B
214
b K
15. a) Folosim relaŃia lui Chasles: AM = AB + BM = a + , (1); BN = BC +
2
2a a b
+ CN = b − , (2); AN = AD + DN = b + , (3); DM = DC + CM = a − , D N b C L
3 3 2
2 a  (a ) M
(4); b) NK = NC + CK = a + 2b = 2 + b  = AN , de unde cerinŃa; P
3  
3  (3) A a B
c) Notăm {L} = AM ∩ DC. Pentru a afla raportul folosim asemănarea.
BP AB AB AB 3 AP 3
În primul rând: ∆BPA ~ ∆NPL ⇒ = = = = , (1), de unde = , (2). Din (2)
PN NL NC + CL AB PL 5
2 + AB 5
3
AP 3
găsim = , (3). Pe de altă parte ∆BMA ≡ ∆CML, de unde LM ≡ AM, (4). Din (3) obŃinem relaŃia
LM + MP 5
AP 3 AP 3 AP 3
echivalentă = , Ńinând cont de (4). De aici rezultă că = , de unde = ,
AM + MP 5 AP + PM + AM 8 2AM 8
AP 3 AP 3 AP
adică în final = . Derivând şi ultima relaŃie, găsim = , de unde = 3 , raportul căutat.
AM 4 AM − AP 4 − 3 PM

5.2. Descompunerea unui vector


BQ 1 k
1. Fie = k . Atunci AQ = AB + AC . Punctele A, P şi Q coliniare, înseamnă că există α ∈ R*,
QC k +1 k +1
MP 2 7 2 AM 1
astfel încât AQ = 2AP . Pe de altă parte, cum = , urmează că AP = AM + AN . Dar din = ,
PN 7 9 9 MB 6
1 AN 3 3 7 1 2 3
urmează că AM = AB . Analog din = , găsim AN = AC . Revenind, găsim AP = ⋅ AB + ⋅ ⋅
6 NC 4 7 9 6 9 7
7 2 1 k 7α 2α
⋅ AC , de unde AP = AB + AC . De aici AB + AC = AB + AC , de unde, cum AB ∩
54 21 k +1 k +1 54 21
 1 7α
=
 k + 1 54 7αk 2α 36
∩ AC = {A}, avem  , de unde = , adică k = .
 k = 2α 54 21 49
 k + 1 21
2. Fie k, p ∈ R – {1}, de unde DA = k DB şi EA = pEC ; (k + 1)DB + (p + 1)EC = DB + DA + EA + EC = 0 .
Cum DB, EC necoliniari, urmează că k + 1 = p + 1 = 0, de unde k = p = –1, adică D, E mijloacele laturilor AB
respectiv AC. De aici, T centrul de greutate al triunghiului ABC, de unde TA + TB + TC = 0 , de unde TA +
+ TB = −TC , de unde α = –1.
3. Fie u un vector coliniar cu a + b − c, a − b + c, − a + b + c . Urmează că (∃) m, n, p ∈ R* astfel încât
m+n m+p n+p
m u = (a + b − c) , n u = a − b + c, pu = −a + b + c . Avem a = u, b = u, c= u , deci vectorii
2 2 2
a , b, c sunt coliniari între ei.

215
4. Ideea este de a descompune în funcŃie de AB = u, AD = v vectorii EF, MP, MN . Să vedem: deoarece
AB + DC λ + 1
DC || AB, (∃) λ ∈ R* astfel încât DC = λ AB = λ u . De aici EF = = u , de unde FE || AB. Pe de
2 2
1 1 1 1 1 1 AB + DC
altă parte, MN = MD + DE + EN = CD − AD + EF = CD − AD + ⋅ =
2 2 2 2 2 2 2
2CD − 2AD + AB − CD AB + CD − 2AD u − λ u − 2v (1 − λ )u − 2 v (1 − λ)u − 2v
= = = = , (1). Analog MP = ,
4 4 4 4 2
(2). Deci MP = 2MN , de unde rezultă că M, N, P sunt coliniare. A
DB
5. a) Din D ∈ BC avem că (∃) α ∈ R* astfel ca CD = αCB . Imediat =
CD
CD
= 1 – α, de unde AD = α AB + (1 − α )AC ;b) Notăm = α ∈ (0, 1). B D C
CB
DC BD
Imediat AD = α AB + (1 − α )AC ; c) Din b) avem AD = ⋅ AB + ⋅ AC , (1). Aplicând modulul în (1) găsim
BC BC
DC BD DC BD DC ⋅ AB BD ⋅ AC
| AD |= ⋅ AB + ⋅ AC ≤ ⋅ AB + ⋅ AC , adică AD ≤ + , de unde cerinŃa: AD ⋅
BC BC BC BC BC BC
⋅ BC ≤ DC ⋅ AB + BD ⋅ AC.
(1)
1 2 1 C
6. a) Avem BM = BC , (1); CN = CD , (2); AM = AB + BM = AB + BC ; BN = N
4 3 4 D
( 2) P M
2
= BC + CN = BC + CD ;
3 B
TM MP A
b) Construim MT || NC. Avem că ∆TPM ~ ∆APB, de unde: = , (1). Dar
AB PA D N C
BM TM BM 1 TM 1
∆BMT ~ ∆BNC, de unde = . Cum = , obŃinem = , (2). Pe de T
BC NC BC 4 NC 4 R Q
2 TM 1 M
altă parte NC = CD şi cum AB ≡ CD găsim = , (3). Revenind în (1) găsim A B
3 AB 6
MP 1 AP
= . De aici, imediat =6.
PA 6 PM
Analog construim NU || BC, PQ || NU. Fie {S} = NU ∩ AM. Avem ∆BPQ ~ ∆BNU, D N C
BP BQ AP AQ
de unde = , (4). Pe de altă parte ∆APQ ~ ∆ABM, de unde = .
PN QU PM QB P
S M
AQ
Folosind rezultatul anterior, găsim = 6 , de unde AQ = 6QB. Pe de altă parte A U Q B
QB
1 7 7 11
AB = AQ + QB = 7QB. Dar AU = AB = QB , iar QU = AQ – AU = 6QB – QB = QB . Revenind în (4)
3 3 3 3
BP 3
găsim că = .
PN 11

216
AB 3 A
7. Din ipoteză = , (1). Aplicând formula vectorului de poziŃie faŃă de D găsim
AC 2
3
AB + AC
2 2 3
AD = = AB + AC , (1). Pe de altă parte obŃinem că IG = IA + AG = C'
1 3 5 5 G
+
3 2 I
b ⋅ BA + c ⋅ CA 2  AB + AC  AB + AC b⋅ AB + c⋅ AC AB + AC B D C
= + = − = −
a+b+c 3  2 
 3 a+b+c 3

 3 
b AB + AC 
− 
2  = AB + AC − 2AB + 3AC = 3AB + 2AC . În final obŃinem IG = 1 AB + 2 AC , (3).
b(10 + b + c) 3 15 15 5 15
q
q 1 1− q
8. Din CN = qCB , avem că NB = CN , de unde AN = AC + AB = (1 − q)AC + qAB , (1).
1− q 1+
q
1+
q
1− q 1− q
1 p 1 1
Din (1) şi AM = AC + pAB , (2), punctele A, M, N coliniare dacă şi numai dacă = , adică − = 1 .
1− q q q p
3 | AP | 3 A
9. a) Din AP = AC , avem A, P, C sunt coliniare, P ∈ (AC) şi = . Atunci
5 | AC | 5 Q P
| AQ | 2 R
din 3AQ + 2BQ = 0 avem că A, B, Q coloniare, Q ∈ (AB) şi = . Atunci
| AB | 5
2 3
PQ = AQ − AP = AB − AC , iar din ipoteză 31RA + 18RB + 6RC = 0 avem că B M D C
5 5
31(RP + PA ) + 18(RP + PB) + 6(RP + PC) = 0 . Înlocuind, PA în funcŃie de AC, PC în funcŃie de AC , iar
BP folosind formule vectorului de poziŃie, ajungem la 55RP + 31PA + 18PB + 6PC = 0 , de unde 55RP = 31AP +
2 3
+ 18BP + 6CP , (1). Dar, de exemplu BP = BA + AC, PA = CA şi efectuăm calculele în (1) suntem conduşi
7 5
18 27 92 3  9 2 3 
la 55RP = 18AB − 27 AC , de unde RP = AB − AC , sau RP =  AB − AC =  AB − AC =
55 55 5 11 11  11  5 5 
9
= PQ , ceea de arată că R, P, Q sunt coliniare; b) Folosind egalitatea 55 RP = 18 AB − 27 AC , deci
11
55RP = 12AB + 6(AB + AC) = 12AB + 12AD = 24DE , unde D este mijlocul lui BC, E este mijlocul lui BD, de
BM 1
aici rezultă că A, R şi E sunt coliniare, deci E = M, în final obŃinând că = . A
CM 3
10. Din ipoteză, cum BM = m ⋅ MA , obŃinem BI + IM = m(MI + IA) , adică b
c B'
(m + 1)MI = −m IA − IB , (1). Absolut analog, CN = n NA este echivalentă cu M I N
CI + IN = n ( NI + IA) , iar după calcule ajungem la (n + 1) NI = −n IA − IC ,
(2). Din (1) şi (2) cum punctele M, I, N sunt coliniare din (1) şi (2) şi această B A a C

217
ipoteză, atunci m IA + IB şi n IA + IC sunt coliniari. Prin urmare axistă α ∈ R astfel încât:
m IA + IB = α (n IA + IC) , (3). Pe de altă parte, folosind Sylvester,
c
Avem a IA + bIB + cIC = 0 , de unde, folosind (3), ajungem la m IA − a IA − IC = αn IA + α IC . Cum vectorii
b
 a a
m − = nα m−
 b b = n , adică mb + nc = a.
IA, IC sunt necoliniari, deducem că  , de unde obŃinem
c
− = α c

 b b
11. Avem BC = 3BP = 3u , (1); AB = −4BQ = −4 v , (2); AC = AB + BC = 3u − 4 v , (3);
1 1 3
NM = NB + BM = − BC − AB = − u + 2 v , (4).
2 2 2
12. Construim R astfel încât ABCR paralelogram. Evident R este simetricul lui B
A R
faŃă de D. De aici PA = PR + RA . Dar RA = CB = CP + PB = − v + u = u − v , (1).
c b
Pe de altă parte PR = PD + DR = PD + BP + PD = 2PD + BP = 2PD − u , (2). Din E D
u P v
(1) şi (2) găsim PA = u − v + 2PD − u = 2PD − v , (3). Acum lucrurile sunt clare!
Trebuie să exprimăm PD în funcŃie de PB . B a C

Folosim automat ce n-am folosit din ipoteză: CE bisectoare. Aplicând teorema bisectoarei în ∆CBD, găsim
CD PD b b
= , de unde PD = − u , (4). Am ajuns la relaŃia dorită, înlocuind (4) în (3), anume PA = − u − v .
CB PB 2a a
13. Pentru a exprima vectorii AD şi respectiv GD , este clar că vom folosi A
BD u b
exprimarea cu ajutorul vectorului de poziŃie. Notând = k , (1), obŃinem G
DC c v
1 k 1 k
AD = AB + + AC = u+ v , (2). Povestea este să determinăm B D a C
1+ k 1+ k 1+ k 1+ k
valoarea lui k. Folosind teorema catetei, găsim AB2 = BD ⋅ BC, AC2 = DC ⋅ BC, de
AB2 DB b2 b2 c2
unde = = k = . De aici imediat, folosind (1), obŃinem: AD = u + v , (3). Pentru calculul lui
AC 2 DC c a2 a2
1
GD folosim următorul rezultat: dacă M este un punct oarecare din plan, MG = (MA + MB + MC) , de unde
3
1
dacă M ≡ A, obŃinem AG = (AB + AC) , (4). Pe de altă parte:
3
(3)
1 b2 c2  b2 1   c2 1 
GD = GA + AD = − (u + v) + 2 u + 2 v =  2 − u + +  2 −  v , (5). A
( 4) 3 a a 3 3
a a
u 5
bAB + cAC b AB + cAC
4. Avem, folosind vectorii de poziŃie, că AA' = , (1); AI = , 6 v
b+c a+b+c
5u + 6 v 5u + 6v B 8 C
(2). Înlocuind, găsim AA' = , (3), respectiv AI = , (4). A'
11 19

218
15. Vom folosi următorul rezultat: dacă O, A, B şi M ∈ (AB) sunt trei puncte A
AM
necoliniare şi = k , atunci are loc egalitatea: OM = (1 − k )OA + k OC , (1). Vrem să E
AB F
aplicăm acest rezultat pentru configuraŃia A, D, E, F ∈ (DE). Pentru aceasta trebuie să D C'
M'
DF
cunoaştem valoarea raportului . Pentru aceasta construim prin D dreapta DC', C' ∈
DE
B M C
∈ AC astfel încât DC' || BC. Acum în ∆DEC' aplicăm teorema lui Menelaus faŃă de
M ' D AC' EF DE AC' AC'
transversala A – F – M'. ObŃinem ⋅ ⋅ = 1 , (2). De aici = . Dar, folosind Thales, =
M ' E ' AE FD EF AE AC
AC
AD 1 AC DE n
= = şi cum AE = , deducem că = m = , de unde derivând ultima proporŃie, găsim valoarea
AB m n EF AC m
m
DF n
căutată = , (3). Folosind (1) găsim: AF = k ⋅ AD + (1 − k )AE , (4). Mai făcând observaŃiile că AD =
DE n + m
1 1 n m 1
= AB , (5), AE = AC , (6). łinând cont că k = , (7) relaŃia (5) se transcrie AF = ⋅ u+
m n n+m m+n m
n 1
+ v , adică AF = ( u + v) .
m+n m+n

5.3. Coordonatele unui vector. Repere


1. Construim CD ⊥ Ox, D ∈ Ox. În ∆DBC, 'CDB =
A(0,3) C(4 + 3 , 3)
CD BD
= 90°, sin('B) = , cos('B) = , de unde CD =
BC BC Q
= 3, BD = 3 . De aici OD = 4 + 3 . În concluzie P(1,1) M(4 + 3 , 2)
punctul C are coordonatele (4 + 3 , 3) . Patrulaterul F(0,1)
D ( 4 + 3 , 0)
ODCA este un dreptunghi. avem IC = IO + OC şi
O E(1,0) B(4,0)
OC = OA + AC = OA + OD . Cum vectorii OD şi OB
OD 4 + 3 4+ 3
sunt coliniari, (∃) α ∈ R* astfel încât OD = α ⋅ OB . De aici α = = , adică OD = OB , de
OB 4 4
4+ 3 4+ 3
unde OC = OB + OA . Cum IC = OC − OI = OB + OA − OI , (1). Determinăm coordonatele lui.
4 4
Construim IE ⊥ Ox, E ∈ Ox, IQ ⊥ AB. Notăm IE = r, raza cercului înscris. De aici, cum S = AB = AQ + QB =
= AF + BE = 3 – r + 4 – r, de unde r = 1. Atunci coordonatele lui I vor fi I(1, 1). Pe de altă parte OI = OE +
1 1 ( 2)
4+ 3 1 1 
+ EI = OE + OF = OB + OA , (2). Revenind în (1) găsim IC = OB + OA −  OB − OA  =
4 3 4 4 3 
4+ 3 OB OA 2 3+ 3 2 3+ 3
= OB + OA − − = OA + OB , de unde α = , β = .
4 4 3 3 4 3 4

219
2. Fie G(x, y). Vectorii 3GA, GB şi − 2GC au coordonatele (–3 – 3x, 3 – 3y), (1 – x, 2 – y), (–4 + 2x, 2 + 2y).
 7
Vectorul 3GA + GB − 2GC are coordonatele (–2x – 6, –2y + 7), de unde G − 3,  . Imediat |M| = 1.
 2

3. Alegem sistemul de coordonate (0, i, j) astfel încât O ≡ A, i = AB, j = AD . Avem A(0, 0), B(1, 0), C(1, 1) şi
AM 7 7  CN
D(0, 1). Din = 7 , M ∈ (AB) avem AM = AB şi M , 0  . Din = 2 şi N ∈ (BC), obŃinem BN =
MB 8 8  NB
1  1 BR BN 1 1
= BC şi N1,  . Notăm cu R intersecŃia dreptelor DN şi AB. Atunci = = , deci BR = AB .
3  3 RA DA 3 2
3  5 MP CN
ObŃinem B , 0  şi MR = . Aplicând Menelaus în ∆MBC pentru transversala P-N-R obŃinem ⋅ ⋅
 2  8 PC NB
BR MP 5 5 7 5 7
⋅ = 1 . De aici rezultă = şi deci MP = MC . Avem AP = AM + MP = AB + MC = AB +
RM PC 13 13 8 13 8
5 7 5 1  12 5
+ (MB + BC) = AB +  AB + AD  = AB + AD . De aici rezultatul dorit: 13AP = 12AB + 5AD .
13 8 13  8  13 13
D C
1 1 1 1
4. a) AF = AE + EF = − AD + BA = − (AD + AB) = − AC =
2 2 2 2
1
= CA . În acest caz C, A, F coliniare; b) Luând reperul (A, AB ,
2 A B
F E
AD) avem versor AB, AD , deci B(1, 0), A(0, 0), D(0, 1), C(1, 1),

 1   1 1  1 1 1
E − , 0 , F − , −  . Atunci AF − , −  , CA(−1, − 1) şi deci AF = CA , adică C, A, F coliniare.
 2   2 2  2 2 2

BQ 7 2 7 2 1 k
5. Fie = k , atunci AP = AM + AN = AB + AC şi AQ = AB + AC . Cum AP şi AQ
QC 9 9 54 21 k +1 k +1
7(k + 1) 2(k + 1) 36
sunt coliniari, rezultă = , de unde k = .
54 21k 49 y
6. Evident punctele D se găsesc pe reuniunea a două drepte după cum AD || BC,
(1) sau AB || CD, (2). Este clar că fixăm cele două drepte determinate o dată de
A şi B şi o dată de B şi C. Deci condiŃiile (1) şi (2) sunt naturale. Cum BC || Ox, * B(0,2) * C(4,2)
o dată D ∈ Ox, deci D ∈ d1, unde ecuaŃia lui d1 este y = 0. Pentru (2) este evident A(–2,0)
că trebuie să construim dreapta care trece prin C şi este paralelă cu AB. Cum *
O x
panta lui AB, mAB = 1, ecuaŃia dreptei căutate este imediată y – 4 = x – 2, sau y =
= x – 2 este dreapta d2 cu D ∈ d2. În final {D ∈ (ABC) | ABCD trapez} = y
= d1 ∪ d2, unde d1 = {(x, y) | y = 0, x, y ∈ R}, iar d2 = {(x, y) | y = x – 2, x, y ∈ R}.
7. Folosind a) Fie sistemul (0, i, j) . Atunci, folosind formula vectorului * B(6,3)
de poziŃie al unui punct M ∈ (AB) astfel încât AM = k ⋅ MB , găsim OM = *A(2,1)
O x

220
1 k
= OA + OB , (1). În cazul nostru AM = 3MB , k = 3, de unde:
1+ k 1+ k
1 3 1 3 20i 10 5  5
OM = OA + OB = (2i + j) + (6i + 3 j) = + j = 5i + j , de unde coordonatele căutate: M 5,  ;
4 4 4 4 4 4 2  2
b) În cazul când M este mijlocul lui AB avem k = 1 şi în calcul imediat conduce la:
1 1
OM = (OA + OB) = (2i + j + 6i + 3 j) = 4i + 2 j . În acest caz coordonatele vor fi M(4, 2).
2 2
8. Alegem reperul xOy astfel AC = x'x, y'y = BQ, unde {Q} = prACB. Fie atunci y
A(a, 0) , B(0, b), C(c, 0), Q(0, 0) şi N mijlocul lui AB, M mijlocul lui AC. Atunci
a b a+c  B
N ,  , M  , 0  . Fie O centrul cercului circumscris şi H ortocentrul
2 2  2  H* N
triunghiului ABC. Avem de demonstrat că HB = 2OM. Fie O(x0, y0). Evident X0 = O*
a +c C Q M A x
= XM = , (1). Pentru a afla Y0 scriem ecuaŃia dreptei ce trece prin N şi este
2
Perpendiculară pe AB. Ştim că mON ⋅ mAB = –1, din condiŃia de perpendicularitate şi unde cXY se notează panta
b a
dreptei determinată de dreapta ce trece prin X şi Y. mAB = –tg A = − , de unde m ON = . EcuaŃia va fi
a b
b a a (1)
b a  a + c a  b + ac
2
y − =  x −  , (2). De aici x0 şi y0 verifică (2) de unde y 0 = +  − = , (3). De aici
2 b 2 2 b 2 2 2b
b 2 + ac
OM = , (3). Trebuie să aflăm |HB|. Pentru aceasta fie H(xH, yH). Evident xh = 0. Pentru calculul lui yH
2b
scriem ecuaŃia dreptei ce trece prin C şi este perpendiculară pe AB. De data aceasta mCH ⋅ mAB = –1, de unde
a a ac
folosind rezultatele anterioare, m CH = , iar ecuaŃia va fi Y = (X − c) , (4). Din (4) ⇒ y H = − . Dar |HB| =
b b b
ac b + ac
2
= |QB| – |HQ| = b + = , (5). Din (3) şi (5) obŃinem rezultatul dorit 2|OM| = |HB|. y
b b
9. Fie xOy un reper ortonormat şi i, j vectorii corespunzători. Atunci există u1, u2,
v1, v2 ∈ R astfel încât u = u1 i + u 2 j , (1); v = v1 i + v 2 j , (2) şi din a u + b v =
j
2 2 2
= bu + a v ⇔ (au1 + bv1 ) + (au 2 + bv 2 ) = (bu1 + av1 ) + (bv 2 + av 2 ) ⇔ a 2 2
(u12 + O x
i
+ u 22 ) + b 2 ( v12 + v 22 ) + 2ab(u1v1 + u 2 v 2 ) = b 2 (u12 + u 22 ) + a 2 ( v12 + v 22 ) + 2ab(u1v1 +
2 2 2 2 2 2 a 2 ≠b2 2 2
+ u 2 v 2 ) ⇔ a 2 u + b 2 v = a 2 v + b 2 u ⇔ (a 2 − b 2 )( u − v ) = 0 ⇔ u = v ⇔ u = v.
x A + ax B 4 y + ay B 3a
10. Avem x M = = , (1); y M = A = y, (2), de y C(1,4)
1+ a 1+ a 1+ a 1+ a
N
 4 3a  B(0,3) P
unde M ,  , (3). Printr-un calcul absolut analog găsim:
1+ a 1+ a 
 b 4b + 3   ab + 4 4ab  M
N ,  , P ,  . Scriem ecuaŃiile dreptelor AN: (4b + O A(4,0) x
 1 + b b + 1   1 + ab 1 + ab 

221
+ 3)x + (3b + 4)y – 16b – 12 = 0, (3); CM: (a + b)x + (3 – a)y + 3a – 16 = 0, (4); BP: (3 – b)x + (4 + ab)y – 3ab –
ab + 4 4ab + 3a  ab + 4 4ab + 3a 
– 12 = 0, (5). Rezolvând sistemul (3) cu (4) obŃinem x = ; y= , deci H , 
ab + a + 1 ab + a + 1  ab + a + 1 ab + a + 1 
are proprietatea că {H} = AN ∩ CM. Cum H verifică şi (5) urmează că H este ortocentru;
MA AB − 5 2
b) În acest caz H trebuie în plus să fie intersecŃia bisectoarelor interioare, deci = −a = − = , deci
MB AC 2
5 2
a= şi analog b = 1.
2
11. Evaluăm mai întâi EF în funcŃie de a , b, c, d . Cum F este mijlocul lui BD, avem D c
d C
1
EF = (ED + EB) , (1). Pe de altă parte, cum E este mijlocul lui AC , avem FE = E
2 A F b
FA + FC AF + AC
= sau EF = , (2). Adunând (1) cu (2) găsim că 4EF = 2(EB + a B
2 2
+ ED) = −2(BE + DE) = −(BA + BC + DA + DC) = −2(−a + b − c + d) = (a + c) −(b + d) , (1).
a+c b+d
Cum EF = şi = −EF , (3), deoarece a + b + c + d = 0 . De aici:
2 2
s = α(a + c) + + β(b + d) = 2α EF − 2βEF = 2(α − β)EF , (4) (Am Ńinut cont de (2) şi (3)); b) Din punctul
precedent a), folosind (4), găsim s = 0 dacă i) α = β sau ii) EF = 0 , ceea ce arată că ABCD este paralelogram.
12. a) Fie {O} = d ∩ d'. Avem: A ' B' − AB = A' O + OD − (AO + OB) = d'
M' A
= A ' O + OB' − AO − OB = OB' − OB − (OA' − OA ) = BB' − AA' ; B
O
b) Avem MM ' = MO + OM ' = OM ' − MO = (A ' M ' − A' O) − (AM − AO) =
d M
= x ⋅ A' B' − x ⋅ AB + AO − A ' O = x '⋅A' B' + AA' − x ⋅ AB = x (A' B' − AB) +
B'
+ ( x '− x )A ' B' , (1). Absolut analog MM' = x ⋅ BB' + (1 − x)AA' + (x'− x)A' B' , b A'
a
(2); c) Evident că MM' || a ⇔ ∃ k ∈ R* astfel încât MM ' = k ⋅ AA' şi, folosind b), (1) şi (2), găsim x = x'.
13. Presupunem că ∃ λ ∈ R astfel încât d(A, dλ) = 3. Folosind formula distanŃei de la un punct la o dreaptă,
1
găsim că d(A, d λ ) = , unde A(3, –1), iar d λ = (2 + λ) x − (1 + λ) y − 6 − 4λ = 0 . Dar cum d(A, dλ) =
2
2λ + 6λ + 5
= 3 rezultă că 9(2λ2 + 6λ + 5) = 1, ajungând la cuaŃia 18λ2 + 54λ + 44 = 0, (1). Cum discriminantul ecuaŃiei din
(1) este negativ, concluzia este imediată: nu există λ ∈ R astfel încât d(A, dλ) = 3.
14. Luăm originea reperului în O şi axa absciselor A6A3. Fie M1 mijlocul lui OA 3 şi M2 A5 M2 A4
R 3
mijlocul lui A 5 A 4 . Atunci OA1 = OM1 + OM 2 = i+ R j , (1), unde i, j sunt
2 2 A6 O M1 A3
1 3 1 3
versorii celor două direcŃii. Analog OA1 = − R i − R j , (2); OA 2 = R i − Rj,
2 2 2 2 A1 A2
1 3
(3); OA 3 = R i , (4); OA 5 = − R i + R j , (5); OA 6 = −R i , (6).
2 2

222
 x x x x  3
Înlocuind în relaŃia din enunŃ obŃinem: i − 1 + 2 + x 3 + 4 − 5 − x 6  + j(− x1 − x 2 + x 4 + x 5 ) . Cum
 2 2 2 2  2
− x1 x 2 x x
i, j sunt necoliniari, găsim + + x 3 + 4 − 5 − x 6 = (− x1 − x 2 + x 4 + x 5 = , de unde:
2 2 2 2
 x 2 + 2 x 3 + x 4 = x1 + x 5 + 2 x 6
 . De aici 2 x 2 + 2x 3 + x 4 = x 4 + 2x 5 + 2x 6 , adică x 2 + x 3 = x 5 + x 6 . În final
 x1 + x 2 = x 4 + x 5
vom avea: x1 + 2x 2 + x 3 = x 4 + 2x 5 + x 6 .
15. Folosim dreapta lui Euler şi exprimarea în vectori de poziŃie, originea sistemului E D
de vectori alegând-o în centrul cercului. Fie G1, G2 centrele de greutate ale ∆ABC şi
1 1 1 A C
∆ADE. Cum OG = OH , avem OG 3 = OH 3 şi am în plus OG1 = OH1 , r H1 =
3 3 3 O
= r A + r B + r C şi r H 3 = r A + r D + r E . Pe de altă parte H1H 3 = r H 3 − r H1 = r D + r E −
− (r B + r C ) = BD + CE . De aici (x – 2009)2 + 1 = 1, de unde x = 2009.
B

VI. Coliniaritate. Con


ConcurenŃă.
oncurenŃă. Paralelism

6.1. Vectorul de poziŃie al unui punct în plan

1 4r B + 3r C + 2r D
1. Avem r G = (r A + r B + r D ) . Pe de altă parte, dacă I este centrul cercului înscris, r I = . Cum
3 4+3+2
BM r B + 2r C
= 2 , rezultă r M = . Deoarece ABCD este paralelogram, avem r A + r C = r B + r D . De aici r G =
MC 3
1 1 1 1
= (r B + r D − r C + r B + r D ) = (2r B − r C + 2r D ) . Avem MG = r G − r M = (2r B − r C + 2r D ) − (r B + r C ) =
3 3 3 3
1 1 1 1
= (r B − 3r C + 2r D ) , (1) iar MI = r I − r M = (4r B + 3r C + 2r D ) − (r B + 2r C ) = (r B − 3r C + 2r D ) , (2). Din
3 9 3 9
(1) şi (2) găsim MG = 3MI , ceea ce arată că G, I, M sunt coliniare.
2. RelaŃia TB + TC − α TA = 0 are loc imediat pentru α = –1, dacă T este centrul de greutate al triunghiului. De
aici şi ideea de a „încerca” să arătăm că E şi D sunt mijloacele laturilor AB respectiv AC. Într-adevăr din A, B, D
coliniare, (∃) x1, y1 ∈ R astfel încât DA = x1 AB, DB = y1 AB . Analog, (∃) x2, y2 ∈ R astfel încât EA = x 2 AC ,
EC = y 2 AC . RelaŃia de condiŃie conduce la (y2 + x2) AC + ( x1 + y1 )AB = 0 . AB şi AC sunt necoliniari, rezultă
x1 + y1 = 0. Dar cum DA şi DB nu sunt simultan nuli, urmează că D este mijlocul lui AB. Analog E este
mijlocul lui AC. Cu aceasta am încheiat soluŃia, căci din cele spuse α = –1.

223
1 1 1 1 1 
3. Avem r M = (r A + r B ) şi analoagele. Avem r R = (r M + r P ) =  (r A + r B ) + (r C + r D ) =
2 2 2 2 2 
1 1 1 1 1  1
= (r A + r B + r C + r D ) ; r D = (r N + r Q ) =  (r B + r C ) + (r D + r E ) = (r B + r C + r D + r E ) . Dar RS = rS −
4 2 2 2 2  4
1 1 1
− r R = (r E − r A ) = AE , prin urmare RS şi AE sunt paralele şi RS = AE . A
4 4 4
OA ⋅ OD ⋅ sin Ô1 OA ⋅ OD 3 α
S
4. Avem AOD = = = , (1). Pe de altă parte, aplicând α
D
SBOF OB ⋅ OF ⋅ sin Ô 2 OB ⋅ OF 8
1
OD AD AC O
teorema bisectoarei în ∆ABD, avem = = , (2) (BD mediană, deci 2
OB AB 2AB
1 S 3 OA AC AO BO B F C
AD = AC ). De aici AOB = = ⋅ . Notăm = k, = l . De aici
2 SBOF 8 2OF AB OF OD
AO k BF AB c
= . Analog, AF bisectoare în ∆ABC, găsim = = , unde AB = c, AC = b, BC = a şi că
AF k + 1 FC AC b
b c
AF = AB + AC , (2). A originea planului şi vom transcrie, folosind vectorul de poziŃie, relaŃia (1):
b+c b+c
b c k k  b c 
rF = rB + r C . Dar r O = r F , de unde r O =  rB + r C  . Pe de altă parte, r O =
b+c b+c k +1 k +1  b + c b+c 
1 l 1 l
= rB + rD = rB + r C , (4). Din (3) şi (4) punem condiŃia necesară să aibă loc: AB ∩ AC =
l +1 l +1 l +1 2(l + 1)
1 1 bk kc kb 1
= {A}. rB + rC = rB + r C , adică = , respectiv
1+ l 2(1 + l) (k + 1)(b + c) (k + 1)(b + c) (1 + k )( b + c) l + 1
kc l 2c b+c b b+c 3
= . De aici l = , iar k = . Revenind în (1) obŃinem ⋅ = , de unde 8b2 +
(1 + k )( b + c) 2(1 + l) b c 2c c 8
2
b b b
+ 8bc = 6c2 sau 4b2 + 4bc – 3c2 = 0. ÎmpărŃind cu c ≠ 0, obŃinem 4  + 4 ⋅ − 3 = 0 . Notând x = găsim x1 =
c c c
−4 + 8 1 3 b 1 AC 1
= = , iar x 2 = − nu convine. Urmează = , adică = , concluzia problemei.
8 2 4 c 2 AB 2
BN MA PC BN α
5. a) Notând = α, = β şi = γ , relaŃia de demonstrat se scrie 2α = β + γ. Avem = ,
BC AB AC NC 1 − α
MA β PC γ
= şi = . Trecem la vectorii de poziŃie: r P = (1 − γ )r C + γ r A . Notăm G' centrul de greutate al
MB 1 − β PA 1 − γ
1 1
∆MNP, atunci r G ' = (r M + r N + r P ) = [(1 − β + γ )r A + (1 − α + β)r B + (1 − γ + α)r C ] . Luăm originea în A şi
3 3
1 1
atunci r A = 0 , de unde r G ' = [(1 − α + β)r B + (1 − γ + α)r C ] . Fie A' mijlocul lui BC, rezultă r A ' = (r B + r C ) .
3 2
1 t
Avem G' ∈ AA' ⇔ (∃) t ∈ R* astfel încât AG ' = t AA' ⇔ [(1 − α + β)r B + (1 − γ + α)r C ] = (r B + r C ) , de unde
3 2
3t = 2(1 − α + β)
cum, pe rând,  , ajungem la 2α = β + γ; b) Cu notaŃiile de la punctul a) egalitatea de demonstrat
3t = 2(1 − γ + α)

224
β
devine = α + γ . Dacă centrele de greutate ale celor două tringhiuri coincid, atunci avem şi relaŃiile 2β =
1− β
= α + γ, 2γ = α + β, deoarece G' aparŃine şi medianelor din B şi C ale triunghiului ABC. Aceste relaŃii ne conduc
1
la α = β = γ = , deci M, N, P sunt mijloacele laturilor triunghiului ABC, ceea ce rezolvă problema.
2
AM k AM k k
6. Derivând proporŃia = , obŃinem = , de unde AM = AB , (1). P
MB 1 AB k + 1 k +1
k k k
Pe de altă parte, PM = PA + AM = PA + (AP + PB) = PA + AP + PB =
k +1 k +1 k +1
 k  k 1 k
= 1 − PA + PB = PA + PB .
 k +1 k +1 k +1 k +1 A M B
A
7. Folosim exprimarea vectorilor de poziŃie ai centrelor de greutate G1, G2,
r A + r B1 + r C1 r B + r C1 + r A1 G1 B1
G3, G, G', G''. Găsim r G1 = , (1); r G 2 = , (2);
3 3
G''
r C + r A1 + r B1 rA + r B + rC *
rG3 = , (3); r G = , (4); * G'
3 3 C1 *G
G3
r G1 + r G 2 + r G 3 r A + r B1 + r C1 G2
rG' = = , (5); r G '' = 1 , (6). Pe de altă parte: B A1 C
3 3
( 5)
r G + r G 2 + r G 3 r A + r B + r C (1), ( 2) r A + r B + r C + 2(r A1 + r B1 + r C1 ) r A + r B + r C
GG ' = r G ' − r G = 1 − = − =
( 4) 3 3 (3) 9 3
2  r A + r B1 + r C1 r A + r B + r C  ( 4) 2 2 2
=  1 −  = (r G ' ' − r G ) = GG ' ' . În final, am obŃinut că GG ' = GG ' ' , (7), relaŃie
3  3 3  ( 6 ) 3 3 3

r A + r B + r C r A1 + r B1 + r C1
ce arată că punctele G, G', G'' sunt coliniare. Avem că G ' ' G = r G − r G ' ' = − , (8).
3 3
r G1 + r G 2 + r G 3 r A + r B1 + r C1 r A + r B + r C + 2 ( r A 1 + r B1 + r C 1 )
Pe de altă parte G ' ' G ' = r G ' − r G ' ' = − 1 = −
3 3 9
r A + r B1 + r C1 1  r A + r B + r C r A1 + r B1 + r C1 
− 1 =  −  , (9). Din (8) şi (9) obŃinem relaŃia căutată G ' ' G = 3G ' ' G ' .
3 3  3 3 
A
8. Din relaŃiile din enunŃ rezultă că: y ⋅ BA1 = z ⋅ A1C , de unde (y + z) BA1 = P
z
= z(BA1 + A1C) , de unde BA1 = BC , (1). Aplicând formula vectorului de B1
y+z C1 S
y z x
poziŃie, găsim AA1 = AB + AC , (2) şi analog BB1 = BA +
y+z y+z x+z B A1 C
z x y
+ BC , (3); CC 1 = CA + CB , (4). Fie S punctul din plan determinat unic de relaŃia
x+z x+y x+y
AS = y ⋅ AB + z ⋅ AC . Atunci avem că AS = ( y + z)AA1 , astfel că S ∈ AA1. De asemenea, BS = BA + AS =

225
= BA − y ⋅ BA + z ⋅ (BC − BA) = x ⋅ BA + z ⋅ BC = ( x + z)BB1 , astfel că S ∈ BB1 şi analog CS = ( x + y)CC1 , de
unde rezultă că S ∈ CC1. De aici concluzia: dreptele AA1, BB1, CC1 sunt concurente în S;
b) Pentru orice punct P din plan avem că PS = PA + AS = PA + y(PB − PA ) + z(PC − PA ) = x PA + yPB +
+ z PC .
9. Un rezultat care extinde teorema lui Pappus la poligoane. E Q D
Mai întâi menŃionăm că centrul de greutate al unui poligon convex A1A2A3...An este
acel punct G din planul (A1A2...An) unic pentru care GA1 + GA 2 + ... + GA n = 0 , R
P
(1). Rezolvarea va fi şi pentru noi pe acelaşi calapod. Arătăm că GA + GB + GC +
A C
+ GD + GE = GM + GN + GP + GQ + GR = 0 , ceea ce arată că G este centrul de M
greutate al celor două poligoane. Deci, fie G centrul de greutate al pentagonului N
B
ABCDE. Avem că GA + GB + GC + GD + GE = 0 , (2). Pe de altă parte folosind
k GA + GB k GB + GC k GC + GD
formula vectorului de poziŃie, găsim că GM + GN + GP + GQ + GR = + + +
k +1 k +1 k +1
k GD + GE k GE + GA k +1 (1)
+ + == [GA + GB + GC + GD + GE] = GA + GB + GC + GD + GE = 0 , (2).
k +1 k +1 k +1
i −1 O
10. a) Avem că M1M i = M1M n , i = 1, n . Din relaŃia care defineşte vectorul de
n −1
poziŃie a punctului care împarte un segment într-un raport dat, rezultă:
 i −1  i −1 n −i i −1
OM i = 1 − OM1 + OM n = OM1 + OM n , i = 1, n , (1);
 n − 1  n − 1 n − 1 n −1
M1 M2 Mi Mn
n −1 n−2 Mn–1
b) Folosind (1), avem că: OM1 + OM 2 + ... + OM n = OM1 + OM1 + ... +
n −1 n −1
1 1 2 n −1 (n − 1) + (n − 2) + ... + 2 + 1 1 + 2 + ... + (n − 1)
+ OM1 + OM n + OM n + ... + OM n = OM1 + ⋅
n −1 n −1 n −1 n −1 n −1 n −1
n
⋅ OM n = (OM1 + OM n ) .
2
11. În rezolvare folosim faptul că expresia centrului de greutate G al triunghiului A
rA + rB + rC
ABC şi respectiv I centrul cercului înscris în ∆ABC sunt rG = , (1)
3 rA
a ⋅ rA + b ⋅ rB + c ⋅ rC B
respectiv rI = , (2). Expresia din enunŃ, după efectuarea rB
C
a+b+c
calculelor, este echivalentă cu: (a + b + c)(rA + rB + rC ) = 3(a rA + brB + crC ) , de rC
O
rA + rB + rC a ⋅ rA + b ⋅ rB + c ⋅ rC
unde forma echivalentă = , care, Ńinând cont de (1) şi (2), arată că rI = rG , adică
3 a+b+c
I ≡ G, lucru posibil dacă şi numai dacă ∆ABC este echilateral. A
MA + MC
12. a) Cum D este mijlocul lui (AC) avem MD = , (1). Cum a = b = 1, din
2 D
ipoteză MA + MB = MD , de unde MA = MD − MB , (2). De aici, folosind (1) găsim că:
M
B C

226
2MD = MA + MC = MD − MB + MC , de unde MD = BM + MC = BC , adică rezultatul dorit; b) Din
( 2)

1
MB = MC + CB = MC − BC = MC − MD . Din MD = ( MA + MC) găsim MC = 2MD − MA , (3). Din
2
(2) şi (3). De aici MD = MA + MB . Cum MA + a MB = b MD , de unde MA + a MB = bMA + b MB , de unde
(1 − b)MA + (a − b)MB = 0 , (4). Cum MA şi MB sunt necoliniari, relaŃia (4) are loc doar dacă a – b = 0 şi
1 – b = 0, de unde a = b = 1.
AD + CD
13. Avem, folosind regula lui Chasles, că EF = ED + DC + CF =
+ DC + D C
2
CD + CB AD − CD + CD + CB AD + CB k AD − BC
+ = = ; EF = EF = . Folosind
2 2 2 2 2 E F
ipoteza, de aici (α – 2); EF = 0 . Cum α ∈ R – {2}, avem că EF = 0 , de unde E ≡ F, A B
adică ABCD paralelogram.
14. Notăm AB = DC = a şi DA = CB = b . Evident a şi b sunt vectori necoliniari. D P C
Avem AQ = AD + DQ = µ DB + AD , (1). Dar DB = DC + CB = a + b , (2).
Înlocuind în (1) găsim AQ = µa + µ b − b = µa + (µ − 1)b , (3). Absolut analog, AP = Q
ip.
A B
= AD + DP = λ DC − b = λ a − b , (4). Din (3) şi (4) urmează că AQ şi AP coliniari
µ µ −1
dacă şi numai dacă = , de unde relaŃia dorită, după efectuarea calculelor: λ − µ = λ ⋅ µ .
λ −1
15. O problemă de dacă şi numai dacă. Presupunem că ABCD este paralelogram. D C
Cum diagonalele într-un paralelogram se înjumătăŃesc, folosind vectorul de poziŃie
al mijlocului, obŃinem: 2MO = MA + MC , (1); 2MO = MD + MB , (2). Adunând O

membru cu membru relaŃiile (1) şi (2), obŃinem relaŃia dorită: 4MO = MA + MB +


A B
+ MC + MD . Reciproc: Presupunem că 4MO = MA + MB + MC + MD . Notăm T
M
mijlocul lui AC şi S mijlocul lui BD, conform ipotezei {O} = AC ∩ BD. Pe de altă
parte, cu T mijlocul lui AC, avem 2MT = MA + MC , (3) şi 2MS = MB + MD , (4). Adunând (3) şi (4) şi Ńinând
cont de ipoteză, 2(MT + MS) = 4MO , de unde 2MO = MT + MS , (5). RelaŃia (5) arată că O este mijlocul lui
TS. Cum T, O, S coliniare şi cum {O} = AC ∩ BD, avem evident că T ≡ O ≡ S, adică avem că în ABCD
diagonalele se înjumătăŃesc, ceea ce conduce la concluzia că ABCD este paralelogram.

6.2. Centre de greutate. Baricentre


1. Folosind vectorii de poziŃie faŃă de un punct arbitrar O, avem: r M − r B = k (r C − r M ) , deci r M (1 + k ) = r B +
+ k r C . Analog r N (1 + k ) = r C + k r A şi r P (1 + k ) = r A + k r B . Sumând aceste trei relaŃii: (1 + k) (r M + r N + r P ) =
= (1 + k )(r A + r B + r C ) . Cum k ≠ –1 (altfel B coincide cu C), rezultă că triunghiurile MNP şi ABC au acelaşi
centru de greutate. Analog triunghiurile QRS şi ABC au acelaşi centru de greutate.

227
2. Notăm r X vectorul de poziŃie al punctului X în raport cu un punct oarecare O. Din MA + 2MB + 3MC = r A −
− r M + 2(r B − r M ) + 3(r C − r M ) = 0 , rezultă 6r M = r A + 2r B + 3r C . Analog două relaŃii pentru r N şi r P . Rezultă
r M + r N + r P = r A + r B + r C şi concluzia se impune.
AL AP BK
3. Din asemănarea ∆ALP ~ ∆ADC, respectiv ∆CPQ ~ ∆CAB, de unde = = = x . Analog, din
AD AC BC
AQ AM DN
asemănarea ∆AQM ~ ∆ACB şi ∆CNQ ~ ∆CAD, avem = = = y . Fie G1 şi G2 centrele de greutate
AC AB DC
AP + AM + AN x AC + yAB + AD + DN x AC + y AB + AD + yDC
ale ∆PMN şi ∆QLK. Avem: AG1 = = = =
3 3 3
x (AD + DC) + 2 y(AB + AD) ( x + 1)AD + ( x + 2 y)AB ( y + 1)AB + (2x + y)AD
= = . Analog, AG 2 = . Cum G1 ≡
3 3 3
≡ G2, găsim x + y = 1.
4. a) Avem AA' = AG + GG ' + G ' A ' , (1). Cum AG + BG + CG = 0 , respectiv A ' G ' + B' G ' + C' G ' = 0 şi scriind
relaŃii de tipul (1) pentru BB', CC' , adunând membru cu membru aceste relaŃii, găsim în final că AA' + BB' +
+ CC' = 3GG ' ; b) Avem A ' B = A' A + AB . Scriind analoagele pentru B' C şi C' A , sumând membru cu
membru, găsim A ' B + B' C + C' A = 3G ' G , (1). Procedând cu k ⋅ A ' C = k ⋅ A' A + k ⋅ AC şi k ⋅ B' A , respectiv
k ⋅ C' B şi adunând aceste relaŃii membru cu membru, suntem conduşi la k ⋅ A ' C + k ⋅ B' A + k ⋅ C' B şi de (1) şi (2)
găsim G ' G = k ⋅ G ' G , egalitatea adevărată, cum k ≠ 1, doar dacă G ≡ G'.
5. Fie G centrul de greutate comun celor două triunghiuri, atunci AA' + BB' + CC' = (AG + GA') + (BG + GB') +
+ (CG + GC') = AG + BG + CG + GA' + GB' + GC' = 0 , concluzia se impune.
MT MS 1 TS 1 Q
6. Deoarece = = , rezultă că ST || NP, iar = . Conform celei de-a doua
MP MN 2 NP 2
reciproce a teoremei liniei mijlocii, rezultă că S şi T sunt mijloacele laturilor PQ, T S
respectiv NQ, în triunghiul QNP. Deducem că M este centrul de greutate al acestui
triunghi şi atunci punctul {M ′} = QM ∩ NP este mijlocul lui NP. Avem că: M
    N P
QM = 2MM ′ = MN + MP, ceea ce încheie rezolvarea.
B' C A' C
7. a) Notăm = k1 , (1); = k 2 , (2). În ∆AA'C şi transversala B – P – B' A
B' A A' B
BC A ' P AB' BC B'
aplicăm teorema lui Menelaus: ⋅ ⋅ = 1 , (3). Din (2) găsim = 1 + k2 ,
BA' PA B' C A' B P
1 A' P PA 1 + k 2
(4). Revenind la (3) găsim (1 + k 2 ) ⋅ ⋅ = 1 , de unde = , (5). M
k1 PA PA ' k1
B A' C
Folosind formula vectorului de poziŃie a unui punct M faŃă de P, P punctul care
1+ k2
MA + MA '
1+ k2 k1 k MA + (k 2 + 1)MA '
împarte segmentul AA' în raportul , obŃinem MP = = 1 , (6). Pe de
k1 k + 1 1 + k1 + k 2
1+ 2
k1
altă parte, aplicând formula vectorului de poziŃie a unui punct M faŃă de A', A' punctul care împarte segmentul

228
MC + k 2 MB k1
BC în raportul k2, obŃinem: MA ' = , (7). Revenind la (6) obŃinem: MP = MA +
1+ k2 1 + k1 + k 2
k2 1 k1 k2 1
+ MB + MC , (8). Notând α = ,β = ,γ = , cu α, β, γ ∈
1 + k1 + k 2 1 + k1 + k 2 1 + k1 + k 2 1 + k1 + k 2 1 + k1 + k 2
∈ (0, 1), am obŃinut relaŃia cerută: MP = α MA + βMB + γ MC ;
k2 1
b) Folosind punctul a), anume relaŃia (8) în care luăm M ≡ A, găsim AP = AB + AC , (9).
1 + k1 + k 2 1 + k1 + k 2
B' C AC k2
Din = k1 , obŃinem = 1 + k1 şi prin urmare AC = (1 + k1 )AB' . Revenind la (9) găsim AP = ⋅
B' A B' A 1 + k1 + k 2
1 + k1 1 1
⋅ AB + AB' , (1). Cum prin ipoteză k1 + 1 = k2, revenind la (1), găsim AP = AB + AB' , (11),
1 + k1 + k 2 2 2
relaŃie ce arată, conform definiŃiei vectorului de poziŃie al unui punct P care împarte un segment dat într-un
raport, găsim că P este mijlocul segmentului BB'.
8. Mai întâi, aplicând teorema lui Pitagora, găsim că AC = 4. Din proprietatea punctului
C
AB + AC
G şi formula medianei AG = . Dacă A' este piciorul bisectoarei unghiului 'A,
3
atunci, utilizând teorema bisectoarei în triunghiul ABC şi apoi în triunghiul AA'B, obŃinem:
4 AB + 3 AC 4 AB + 3AC AB + AC G A'
AA ' = , (1) iar AI = , găsim că GI = GA + AI = +
7 12 3
I'
4AB + 3AC 1
+ = − AC , de unde evident GI || AC. A B
12 12
9. a) Dacă (AA' şi (BB' sunt bisectoare în ∆ABC, atunci {I} = AA' ∩ BB'. Conform A
A' B AB c
teoremei bisectoarei în triunghiul ABC avem = = , (1). ObŃinem B'
A' C AC b
I
ac
lungimea lui A ' B = , (2). Aplicând teorema bisectoarei în ∆ABA', găsim
b+c G
IA AB b + c B A' C
= = , (3). Fie G centrul de greutate al ∆ABC. Folosind definiŃia
IA' A ' B a
c
GB + GC
b bGB + cGC
vectorului de poziŃie faŃă de punctul A', obŃinem: GA ' = = , (4). Analog, folosind
1+
c b+c
b
b+c
GA + ⋅ GA' ( 4)
a 1
definiŃia vectorului de poziŃie faŃă de I ∈ (AA') şi folosind (3) şi (4), găsim GI = = ⋅
b+c a + b + c
1+
a
1  5AB + 5AC
⋅ (a ⋅ GA + b ⋅ GB + c ⋅ GC) , (5); b) Înlocuind în (5) pe a, b şi c cu valorile date, găsim IG =  +
2  3

4 BC + 4 BA 3CB + 3CA  1 3 1 1
+ + = [ AB + 2 AC + BC ] = AC = AC . Din ultima relaŃie IG = AC
3 3  36 36 12 12

229
deducem că avem IG || AG. C
10. Fie A, B, C puncte coliniare şi M un punct arbitrar. Deoarece A, B, C coliniare, B
A
∃ k ∈ R – {0, 1} astfel încât AB = k AC , (1). Dar AB = MB − MA, AC = MC − MA ,
de unde, Ńinând cont de (1) găsim MB − MA = k MC − k MA sau (1 − k )MA − MB +
+ k MC = 0 . Considerând α = 1 – k, β = –1, γ = k, obŃinem relaŃia din enunŃ αMA +
M
+ βMB + γ MC = 0 .
ip.
A
11. a) Avem AH = AO + OH = AO + OB + OC + OA = OB + OC = 2OA ' , (1). Din
AH = 2OA' avem că AH şi OA ' sunt coliniari; b) Cum o dreaptă ce pleacă din A''
centrul cercului şi trece prin mijlocul unei coarde este perpendiculară pe coardă, H
G
deci OA' ⊥ BC. Cum OA ' şi AH coliniari, urmează cu evidenŃă că AH ⊥ BC, deci O
AH înălŃime; c) Arătăm că BH coliniar cu OA ' ' , unde A'' este mijlocul lui AC.
B A'' A' C
ip.
Într-adevăr, BH = BO + OH = BO + OA + OB + OC = OA + OC = 2OA' ' . M
Din BH = 2OA' ' , urmează că BH şi OA ' ' sunt coliniari. Cum, din aceleaşi considerente ca la b) OA'' ⊥ AC şi
BH ⊥ AC, de unde, vezi a), H este ortocentrul ∆ABC, căci BH este înălŃime în ∆ABC; d) Aplicând formula
1
MC' + MC
2 1 2
vectorului de poziŃie faŃă de G ∈ AA', obŃinem MG = = MC + MC' , unde C' este mijlocul lui
1 3 3
1+
2
2 1 2 MA + MB 1 MA + MB + MC
AB. De aici avem că MG = MC' + MC = ⋅ + MC = , de unde relaŃia dorită:
3 3 3 2 3 3
3MG = MA + MB + MC ; e) Din ipoteză avem că OH = OA + OB + OC şi înlocuind în relaŃia de la d) M cu O,
găsim 3OG = OA + OB + OC , de unde OH = 3OG , ceea ce arată că vectorii OH şi OG sunt coliniari, adică
punctele O, H, G sunt coliniare.
12. Când vorbim de triunghi median ne referim de fapt la teorema lui Pappus: se A
consideră un ∆ABC şi punctele M, N, R pe AB, BC, AC care împart laturile în M
acelaşi raport. În acest caz cele două triunghiuri ABC şi MNR au acelaşi centru de B'
C' P R
greutate. Imediat, deoarece în orice triunghi O, G, H sunt coliniare şi OG = 3OH *
*
(dreapta lui Euler), urmează că cele două cercuri circumscrise sunt concentrice, cu O O
AM BN CR B C
centrul lor. Din ipoteza teoremei lui Pappus avem că = = = x, x > 0. N A'
MB NC RA
OA + x OB OB + x OC
Avem, folosind formula vectorului de poziŃie, că OM = , (1) şi analoagele, ON = , (2);
1+ x 1+ x
OC + x OA OA + a OA ' OB + bOB' OC + cOC'
OR = , (3), iar OP = , (4); OP = , (5); OP = . Din | OA |=
1+ x 1+ a 1+ b 1+ c
=| OB |=| OC | , din (4), (5), (6) avem imediat că OA ' = OB' = OC' , de unde | OA' |=| OB' |=| OC' | , deci O este şi
BA '
centrul cercului circumscris ∆A'B'C'. De aici în plus avem că a = b = c. Notând = m , avem de asemenea că
A' C
BA ' CB' AC'
= = = m, adică ∆A'B'C' este triunghi median pentru triunghiul ABC. Aplicând teorema lui
A' C B' A C' A

230
BC A ' P AB' 1
Menelaus în ∆AA'C faŃă de transversala B – P – B', găsim ⋅ ⋅ = 1 . De aici găsim că a = 1 + , (1).
BA' PA B' C m
Aplicând din nou teorema lui Menelaus în ∆ABB' cu transversala C – P – P', găsim a = m2 + m, (2). Din (1) şi
m +1 AP
(2) obŃinem m(m + 1) = , de unde m2 = 1, de unde m = 1, a = 2. Cum P este astfel încât = 2 , evident
m PA '
P ≡ G, centrul de greutate al ∆ABC, căci AP mediană, cum m = 1, BA' = A'C.
GB + GC A
13. a) Avem GA = , (1). Pe de altă parte cum 2GA' = AG , (2), din
2
proprietatea centrului de greutate avem AG = GB + GC , de unde imediat GA + C' B'
G
+ GB + GC = 0 ; b) Avem, din proprietatea centrului de greutate şi folosind formula O
OA + 2OA'
vectorului de poziŃie, că OG = , de unde 3OG = OA + 2OA' . B A' C
3
OB + OC
Dar, pe de altă parte, avem OA' = , de unde revenind, 3OG = OA + OB + OC ; c) Din b) avem că
2
OA + OB + OC = 3 | OG | . Atunci OA + OB + OC minim când | OG | minm. Cum G este pe BC, iar G este
centrul de greutate, evident OG este minim când OG ⊥ BC, adică O este piciorul perpendicularei dusă din G
pe BC.
14. Avem MA + 2MB + 3MC = 0 , de unde MO + OA + 2(MO + OB) + 3(MO + A

+ OC) = 0 , relaŃie echivalentă cu OA + 2OB + 3OC = 6OM , (1). Analog OB + M D


G
+ 2OC + 3OA = 6ON , (2) şi OC + 2OA + 3OB = 6OP , (3). Adunând membru cu
membru (1), (2) şi (3), obŃinem OA + OB + OC = OM + ON + OP , (4). Dar
B C
OA + OB + OC = 3OG , (5), unde G este centrul de greutate al triunghiului ABC.
Absolut analog avem că OM + ON + OP = 3OG ' , (6), unde G' este centrul de N

greutate al triunghiului MNP. Din (4), (5), (6) ajungem la OG = OG ' , de unde concluzia G = G', adică cele două
triunghiuri au acelaşi centru de greutate.
AG1 2
15. a) Fie P mijlocul lui BC. Cum G1 ∈ AP, avem = , (1). Avem A E
AP 3 G2
1 1 2 (2) 1 1 M
OP = (OB + OC) , (2), iar OG1 = OA + OP = OA + (OB + OC) = O D G
2 3 3 3 3
1 N
= (OA + OB + OC) , de unde relaŃia dorită 3OG1 = OA + OB + OC , B G1
3 P
(3); b) Din a) alegând O ≡ M, obŃinem pentru fiecare centru de greutate C
G1 respectiv G2, că 3MG1 = MA + MB + MC, 3MG 2 = MA + MB + MC . De aici 3(MG1 − MG 2 ) = ME − MC ,
1 1
de unde G1G 2 = CE , (4). Observăm că MN este linie mijlocie în ∆CDE, de unde NM = CE , (5). Din (4) şi
3 2
G 1G G 2G
(5) rezultă că G1G2 || MN, unde ∆GMN ~ ∆GG1G2. Din asemănare obŃinem relaŃia dorită: = .
GN GM

231
6.3. Teorema bisectoarei. Teorema lui Sylvester
1. Presupunem G centrul cercului circumscris ∆AGD. Din Sylvester avem GH = GA + GC + GD , (1). Cum G
este centrul de greutate al ∆BCD, avem că: GB + GC + GD = 0 , (2). Din (1) şi (2) avem imediat GH = BA ,
adică ABGH este paralelogram (GH || AB şi |GH| = |BA|). Reciproc: presupunem că ABGH paralelogram.
Evident GH = BA . Cum GB + GC + GD = 0 , GB + BA + GC + GD = BA = GH , adică GH = GA + GC + GD ,
(3). Fie O centrul cercului circumscris ∆ACD. Din Sylvester, OH = OA + OC + OD , (4). Scăzând (4) din (3),
găsim: GH − OH = GA − OA + GC − OC + GD − OD , adică GH = GO + GO + GO , adică 2GO = 0 , posibil
dacă G ≡ O.
2. Presupunem că ∆ABC este echilateral. Atunci bisectoarele sunt şi mediane. Deci 2AD = AB + AC, 2BF =
= BA + BC, 2CE = CB + CA . De aici rezultă 2(AD + BF + CE ) = AB + AC + BA + BC + CB + CA = 0 , adică
relaŃia cerută. Notăm BC = a, AC = b, AB = c. Presupunem AD + BF + CE = 0 . Aplicând teorema bisectoarei
b c c a a b
găsim: AD = AB + AC , BF = BC + BA , CE = CA + CB . De aici (b2 – a2) ⋅ c ⋅
b+c b+c a+c a+c b+a b+a
⋅ AB + (c 2 − b 2 ) ⋅ a ⋅ BC + (a 2 − c 2 ) ⋅ b ⋅ CA = 0 . Dar CA = (AB + BC) , de unde (b2c – a2c – ba2 + bc2) AB +
+ (ac2 – ab2 – ba2 + bc2) BC = 0 , (1). Cum AB ∩ BC ≠ ∅, relaŃia (1) are loc doar dacă ac2 – ab2 – ba2 + bc2 =
= b2c – a2c – ba2 + bc2. De aici ac2 + a2c = b2c + ab2, adică ac(a + c) = b2(a + c), adică ac = b2, sau abc = b3.
Analog pentru celelalte vârfuri. Ajungem la a3 = b3 = c3, care conduce la a = b = c, adică ∆ABC echilateral.
BE c
3. Construim bisectoarea AE, E ∈ BC şi arătăm că AE, AD coliniari. Din teorema bisectoarei, = , de unde
CE b
c c 1 λ 1
EB = − EC . Notăm λ = − . Pe de altă parte, AE = AB − AC . Găsim AE = [AB − λ AC] =
b b 1− λ 1− λ 1− λ
b  c  1 1 1
=  AB + AC = [bAB + cAC] = AD , Ńinând cont de condiŃie. Din AE = AD conduce la
b+c b  b+c b+c b+c
AE, AD coliniari, adică A, E, D coliniare, de unde AD bisectoare.
4. Ideea ar fi ca din relaŃia de condiŃie să obŃinem că OM + ON + OP = OA + OB + OC , de unde, folosind
Sylvester, avem relaŃia. Într-adevăr: 0 = MA + MB + NA + NC + PB + PA = OA − OM + OB − OM + OA − ON +
+ OC − ON + OB − OP + OC − OP = 2(OA + OB + OC) − 2(OM + ON + OP) , de unde relaŃia dorită: OA + OB +
+ OC = OM + ON + OP .
5. A se vedea orice manual. De exemplu: M. Burtea, G. Burteu, Matematică, clasa a X-a, Ed. Carminis, 2004,
pag. 228-229; a) EnunŃul teoremei lui Sylvester; b) OH = 3OG , consecinŃe ale teoremei lui Sylvester.
DE BD A
6. a) Avem din asemănarea triunghiurilor ADC şi BDE: = , (1). Notând cu
AD DC
BD c
a = BC, b = AC, c = AB şi folosind teorema bisectoarei, găsim că = , (2). Din B P
DC b D
DE c c+b
(1) şi (2) găsim = , (3), care prin derivare, conduce la AE = AD sau Q C
AD b b
c+b 1
AE = AD , (4). Pe de altă parte, folosind teorema bisectoarei, AD = ⋅ E
b b+c *
F

232
c
⋅ (b ⋅ AB + c ⋅ AC) , (5). Din (4) şi (5) obŃinem AE = AB + AC , (6). Repetând
b
b
raŃionamentul anterior, găsim că AF =
AB + AC , (7);
c
b) Din 'BAD = 'DAC = 'BED urmează că ∆ABE isoscel. Cum BP ⊥ AE, urmează că BP mediană, de unde
(8 )
1 1 1 1 1 1
AP = AE , (8). Analog AQ = AF , (9). Dar PQ = AQ − AP = AF − AE = (AF − AE) = EF .
2 2 (9) 2 2 2 2

7. a) Fie M mijlocul segmentului BC. Aplicând teorema bisectoarei şi formula A


vectorului de poziŃie pentru un punct ce împarte un segment într-un raport dat,
c
AB + AC
BA' c b bAB + c AC
găsim = , de unde AA' = = , (1). Dacă M este
A' C b
1+
c b+c
B A' M A'' C
b
AB + AC AA' + AA' '
mijlocul lui BC, AM = , (2) şi AM = , (3), de unde AA' + AA' ' = AB + AC , (4). Folosind
2 2
bAB + c AC cAB + bAC
(1) şi (4) găsim că + AA' ' = AB + AC , de unde, în final AA' ' = , (5);
b+c b+c
a BC + c BA bCA + aCB
b) Printr-un calcul analog pentru B'' şi C'' ajungem, folosind (5) că: BB' ' = , (6); CC' ' = ,
a+c b+a
cAB + bAC a BC + c BA
(7). Atunci (b + c)AA' ' + (c + a )BB' ' + (a + b)CC' ' = (b + c) ⋅ + (c + a ) ⋅ + (a + b) ⋅
b+c c+a
bCA + a CB
⋅ = c(AB + BA ) + b(AC + CA ) + a (BC + CB) = 0 .
b+a
8. Notăm cu O centrul cercului circumscris patrulaterului. Dacă Ha este ortocentrul D C
triunghiului BCD, atunci conform relaŃiei lui Sylvester, avem OH a = OB + OC + OD , (1). Ha
1 (1)
1 O
Pe de altă parte, în ∆AOHa, OMa mediană, de unde OMa = (OA + OHa ) = (OA + OB + Ma
2 2 A
B
+ OC + OD) . Absolut analog obŃinem OM b = OM c = OM d = OM e , de unde Ma ≡ Mb ≡
≡ M c ≡ M d.

9. a) În ∆AA'B se consideră A'D ⊥ AB şi Q mijlocul A A


B'
segmentului AB şi H1 ∈ A'D astfel încât A ' H1 = H*1
A' O
= 2OQ , (1). În ∆OAB, OQ mediană, de unde OQ = H1 Q
O
OA + OB C A' D
= , (2). Din ∆OA'H1 avem OH1 = OA ' +
2
B
+ A ' H1 , (3). Din (1) şi (2) înlocuite în (3), obŃinem: C'
B
OH1 = OA + OB + OA ' ;

233
b) Folosind relaŃia lui Sylvester pentru fiecare din ortocentrele H2, H3, H4, H5, H6, găsim OH2 = OA + OC + OA' ,
(4). Pe de altă parte H1H 2 = H1O + OH 2 = −OA − OB − OA' + OA + OC + OA' = OC − OB , (5). Absolut analog
găsim că H 3H 4 = OA − OC , (6), respectiv H 5 H 6 = OB − OA , (7). Adunând membru cu membru, obŃinem că
H1H 2 + H 3H 4 + H 5 H 6 = 0 . A
H
10. a) Avem HB = HO + OB , (1); HC = HO + OC , (2); HA = HO + OA , (3). Adunând *
*O
(1) cu (2) găsim HB + HC = 2HO + OB + OC , (4). RelaŃia din enunŃ se transcrie astfel: B C
*Oa
2HO + OB + OC = HO + OA + 2AO , de unde OA + OB + OC = OH , (5), relaŃie ce M
reprezintă relaŃia lui Sylvester;
b) Evident A este ortocentrul triunghiului HBC, Oa centrul cercului său circumscris, de unde, conform teoremei
lui Sylvester: O a A = O a H + O a B + O a C , (6). Pe de altă parte MA = MO a + O a A , (7). Analog: MB = MO a +
+ O a B , (8); MC = MO a + O a C , (9); MH = MO a + O a H , (10). RelaŃia de demonstrat se rescrie: MOa + Oa A =
= 3MO a + O a B + O a C + O a H − 2MO a , care revine la relaŃia (6), adevărată O a A = O a B + O a C + O a H ;
c) Avem OOa ⊥ BC, ObOc ⊥ AH, AH ⊥ BC, deci ObOc ⊥ BC şi OOa ⊥ ObOc. Absolut analog arătăm că ObO ⊥
⊥ OaOc, de unde concluzia.
11. Folosind relaŃia lui Sylvester pentru O centrul cercului circumscris şi H1, H2, H3, E D
H4 ortocentre, găsim: OH1 = OA + OB + OC , (1); OH 2 = OB + OC + OD , (2); *H 3

OH 3 = OC + OD + OE , (3); OH 4 = OA + OC + OE , (4). Pentru a arăta că *


H4 *O H2
*
H1H2H3H4 paralelogram, arătăm, de exemplu, că H1H 2 = H 3 H 4 . Folosind (1), (2), A
*H1 C
(1)
(3), (4), găsim H1H 2 = OH 2 − OH1 = OD − OA = AO + OD = AD , (5); H 3H 4 =
( 2) B
(3)
= OH 3 − OH 4 = OD − OA = AO + OD = AD , (6). Din (5) şi (6) obŃinem relaŃia de
( 4)
condiŃie ca H1H2H3H4 să fie paralelogram.
BA' c A
12. Folosind teorema bisectoarei pentru AA', bisectoare interioară, găsim = ,
A' C b
b C' B'
(1), de unde, folosind formula vectorului de poziŃie, găsim că AA' = AB + I
b+c
c b
c ac
+ AC , (2). Din (1) avem imediat că BA' = , (3), de unde, cu teorema
b+c b+c B A' a C
AI (3) c b+c
bisectoarei = = , de unde imediat:
A' I ac a
b+c
b+c 1 1
AI = AA' = (bAB + cAC) , (4). Analog cu (4) obŃinem imediat BI = (a BA + cBC) ,
a+b+c a+b+c a+b+c
1 c
(5) şi respectiv CI = (bCB + a CA ) , (6). Folosind (4), (5) şi (6) împreună cu ipoteza, găsim (bAB +
a+b+c 2a
c
+ cAC) + (−(a + c)AB + cAC) + b AB − (a + b)AC = 0 sau, în final, după efectuarea calculelor, găsim:
2b

234
 bc c(a + c)   c2 c2 
 − + b AB +  + − a − b AC = 0 , (7). Cum AB şi AC sunt vectori necoliniari, relaŃia (7) are
 2a 2b   2a 2b 
 bc c(a + c)
 2a − 2b + b = 0
loc dacă şi numai dacă  2 2
, de unde rezultă concluzia problemei, a = b, c2 = 2a2, adică ∆ABC
 +c c
−a −b =0
 2a 2b
este dreptunghic în C.
13. Este de fapt enunŃul reciprocei celebrei teoreme a lui Pappus. Aproape sigur se A
sugerează şi rezultatul m = n = p. Într-adevăr, fie O un punct în plan, G centrul de
greutate al ceor două triunghiuri. De aici OA + OB + OC = OG = OM + ON + OP , de P N
unde OM − OA + ON − OB + OP − OC = 0 ce conduce la AM + BN + CP = 0 , (1).
1
Dar, folosind formula vectorului de poziŃie şi ipoteza, avem că: AM = AB + B M C
m +1
m 1 n 1 p
+ AC , (2); BN = BC + BA , (3); CP = CB + CA , (4). łinând cont că BC = BA +
m +1 n +1 n +1 p +1 p +1
 1 n 
+ AC , (5) şi folosind relaŃiile (2), (3), (4) în (1), după efectuarea calculelor ajungem la AB ⋅ 1 − −  +
 p + 1 n + 1
 m 1 
+ BC + − 1 = 0 , (6). RelaŃia (6) ne permite să ajungem la concluzie, căci cum AB şi BC sunt
 m + 1 n + 1 
1 n m 1
necoliniari, suntem conduşi la 1 − − = 0 , respectiv + − 1 = 0 , de unde m = n = p.
1+ p n +1 m +1 n +1
H
14. a) Arătăm pe rând că H1H2DA este paralelogram. Pentru aceasta aplicăm * 3
condiŃia necesară şi suficientă ca un patrulater să fie paralelogram: H1H 2 = H2
D *
= DA , adică 2 vectori opuşi să fie coliniari. Într-adevăr: H1H 2 = H1O + OH 2 ,
C
(1), de unde O este centrul cercului circumscris patrulaterului ABCD. Conform H4
*
relaŃiei lui Sylvester, avem: OH1 = OA + OB + OC , (2) şi analoagele: OH 2 =
*
= OB + OC + OD , (3); OH3 = OC + OD + OA , (4); OH 4 = OD + OA + OB , (5). O H1
A
Revenind în (1) obŃinem, folosind (2) şi (3): H 1 H 2 = − OA − OB − OC + B
+ OB + OC + OD = OD − OA = AO + OD = AD , ceea ce demonstrează că
H1H2DA este paralelogram. Absolut analog se procedează cu restul patrulaterelor H2H3AB, H3H4BC, H4H1CD;
b) Cum H1H 2 = AD, H 2 H 3 = BA, H 3H 4 = CB şi H 4 H1 = DC , adică cele două paatrulatere sunt congruente.
Cum ABCD inscriptibil, urmează că H1H2H3H4 este înscris într-un cerc.
A
15. Vectorii AA1 , BB1 , CC1 pot forma un triunghi dacă şi numai dacă AA1 + BB +
b c C1 B1
+ CC1 = 0 , (1). Avem, folosind teorema bisectoarei, AA1 = AB + AC , (2); I
b+c b+c
c a a b
BB1 = BC + BA , (3); CC1 = CA + CB , (4), unde a, b, c reprezintă
a+c a+c a+b a+b
B A1 C
lungimile laturilor triunghiului. Înlocuind (2), (3), (4) în (1) şi efectuând calculele,

235
 b a   c a   c b 
obŃinem:  − AB +  − AC +  − BC  = 0 . De aici, rescriind, găsim: c ⋅ (b2 –
b+c a+c b+c a+b a+c a+b 
– a2) AB + b(c 2 − b 2 )AC + a (c 2 − b 2 )BC = 0 , (5). Folosind faptul că AC = AB + BC şi înlocuind în (5), găsim
(cb2 – ca2 + bc2 – a2b) AB + (ac 2 − ab 2 + bc 2 − ba 2 )BC = 0 , (6). Deoarece vectorii AB şi BC nu sunt coliniari,
cb 2 − ca 2 + bc 2 − a 2 b = 0 (b + c)( bc − a 2 ) = 0
relaŃia (6) are loc doar dacă  , relaŃie echivalentă cu  , de unde
ac 2 − ab 2 + b 2 c − ba 2 = 0 (a + b)(c 2 − ab) = 0
a = b = c, adică ∆ABC este echilateral.

6.4. Teorema lui Menelaus şi Ceva


1
1. a) Se ştie că dacă AM = k MB , k ≠ –1, k ∈ R, atunci pentru orice punct O al planului avem OM =OA +
k +1
k 1 α 1 α
+ OB , (1). Dacă α = β = γ, folosind (1) găsim: OM = OB + OC , OP = OC + OA ,
k +1 α +1 α +1 α +1 α +1
1 α
OQ = OA + OB . De aici OM + OP + OQ = OA + OB + OC . Dacă G este centrul de greutate al
α +1 α +1
1 1
∆ABC, avem OG = (OB + OA + OC) , iar dacă G' este centrul de greutate al ∆MPQ, avem OG' = (OA + OB +
3 3
+ OC) , de unde OG = OG ' , adică G ≡ G'; b) Aplicăm teorema lui Ceva în ∆ABC dreptelor AM, BP, CQ:
BM PC QA
dreptele sunt concurente ⇔ ⋅ ⋅ = 1 , de unde αβγ = 1; c) Din ipoteză m > 0. Rezultă că N ∉ BC şi
MC PA QB
AQ
este necesar ca = γ ∈ (0, 1) pentru că NQ ∩ AC = {S}. Aplicând Menelaus în ∆ABC pentru punctele N, S,
QB
AS NC QB AS NB QA
Q cu ⋅ ⋅ = 1 , de unde = ⋅ = m ⋅ γ. Raportul căutat este mγ, γ ∈ (0, 1).
SC NB QA SC NC QB
NB 1
2. a) Fie N ∈ (BM), NM = BM − BN = 4BN − BN = 3BN . De aici = − . Pentru a arăta că A, N, P
NM 3
1 NB 1 k
coliniare, arătăm că (∃) λ ∈ R*, astfel încât AN = λ AP . Din k = − = , găsim AN = AB − ⋅
3 NM 1− k 1− k
3 1 3 1 PB 1 6 1 86 1  8
⋅ AM = AB + AM = AB + AC . Dacă = − , avem AP = AB + AC =  AB + AC  = ⋅
4 4 4 8 PC 6 7 7 7  8 8  7
3 1  8 7 PB 1 QB
⋅  AB + AC  = AN . Ajungem la AN = AP , ceea ce arată că A, N, P coliniare; b) Din =− =
4 8  7 8 PC 6 QA
CM PB CM QA
(Thales). Deoarece M este mijlocul lui (AC) avem = −1 , deci ⋅ ⋅ = −1 şi, conform reciprocei
AM PC AM QB
teoremei lui Ceva, dreptele AP, BM şi CQ sunt concurente.

236
3. Trebuie să demonstrăm că dreptele AD, BE, CF sunt concurente. Pentru A
aceasta, conform reciprocei teoremei lui Ceva, ar fi suficient să arătăm că
AF BD CE AD AF
⋅ ⋅ = 1 . Teorema bisectoarei conduce la = (pentru bisec-
FB DC EA DB FB E
DC CE F P
toarea DF în ∆ADB), respectiv = (pentru bisectoarea DE în ∆ADC).
AD EA
AF BD CE AD DC BD
De aici avem că ⋅ ⋅ = ⋅ ⋅ = 1 , de unde concluzia
FB DC EA DB AD DC B D C
problemei.
4. a) Construim CC1 || AD, C1 ∈ AB, CC2 || AB, C2 ∈ AD. Punctele F
AC1
C1, C2 sunt unic determinate din construcŃie. Notăm x = , y =
AB D P
AC 2 AC1 AC 2
= . De aici AC = AC1 + AC2 = ⋅ AB + ⋅ AD . Pe de C2 N C
DA AB AD
BC1 BC AC1 FC FC
altă parte = , de unde = , deci x = . Analog M
AB BF AB FB FB A C1 B E
CE FC EC
y= . Prin urmare AC = ⋅ AB + ⋅ AD ;
DE FB ED
AE AE AF
b) Din A ∈ (BE), avem AE = − ⋅ AB , deci x1 = − şi y1 = 0. Din A ∈ (DF), AF = − ⋅ AD , prin
AB AB AD
AF 1 1  FC EC  1 1
urmare y2 = − , x 2 = 0 ; c) Cum AM = AC =  AB + AD  ; AN = (AB + AD) ; AP = (AE +
AD 2 2  BF ED  2 2
1  AE AF   
  BC 
 DC 
+ AF) = −  ⋅ AB + ⋅ AD  . Cum 2MN = 2(AM − AN) = ⋅ AB + ⋅ AD şi 2PN = 2(AN − AP) =
2  AB AD  BF DF
BE DE BE BC DE DC
= ⋅ AB + ⋅ AC . MN şi PN sunt coliniari, adică M, P, N coliniare.Dacă . De aici : = : .
AB AD AB BF AD DF
BC BE DC DF BA FD CE BC AF ED
De aici ar trebui ca : = : , lucru ce rezultă din faptul că ⋅ ⋅ = ⋅ ⋅ ⋅ (am
BF AB DE AD BE FA CD BF AD EC
aplicat pentru membrul stâng Menelaus în ∆AED şi transversala F-B-C, iar pentru membrul drept, Menelaus în
∆FCD şi transversala B-A-E). A
BD CE AF
5. Notăm = k1 , = k 2 şi = k 3 , (1). Conform teoremei lui Ceva, avem
DC EA FB
BD k CE k2 AF B'
k1k2k3 = 1. Derivând relaŃiile din (1) găsim: = 1 , = , = C' M
BC 1 + k1 CA 1 + k 2 AB
k3 AF BD CE k k k 1
= , (2). Atunci ⋅ ⋅ = 1 ⋅ 2 ⋅ 3 = , B A' C
1 + k3 AB BC CA 1 + k1 1 + k 2 1 + k 3 (1 + k1 )(1 + k 2 )(1 + k 3 )
(3). Pe de altă parte, utilizând inegalitatea mediilor, obŃinem: 1 + k1 ≥ 2 k 2 , 1 + k 2 ≥ 2 k 2 , 1 + k 3 ≥ 2 k 3 , (4),
1 1 1 1 1
de unde ⋅ ⋅ ≤ = , (5). Din (3) şi (5) obŃinem inegalitatea cerută:
1 + k1 1 + k 2 1 + k 3 8 k1k 2 k 3 8
AE BD CE 1
⋅ ⋅ ≤ .
AB BC CA 8

237
c2 b2 BM C
6. a) Aplicând teorema catetei, găsim că BM = , (1), CM = , (2), de unde =
a a CM
N
c2
= , (3). Folosind formula vectorului de poziŃie al punctului M, găsim că b P
b2 M
b2 c2 b
AM = 2 AB + 2 AC , (4). Folosind teorema bisectoarei, avem AP = AB +
b + c2 b + c2 b+c A c B
c 1 1  b
+ AC , (5), iar cum AN este mediană, găsim AN = AB + AC , (6). De aici, MP = MA + AP =  −
b+c 2 2 b+c
b 2   c
 c 2  bc(c − b) bc(b − c) bc(b − c) bc(b − c)
−  AB +  − 2 
AC , adică MP = 2 AB + 2 AC = 2 (AC − AB) = ⋅
a2  b+c a  a ( b + c) a ( b + c) a ( b + c) a 2 ( b + c)
bc(b − c) 1 b  1 c  c−b
⋅ (AC + BA) = 2 BC , (7). Calculăm acum PN = PA + AN =  − AB +  − AC = ⋅
a ( b + c) 2 b+c 2 b+c 2(b + c)
b−c b−c b−c b−c
⋅ AB + AC = (AC − AB) = (AC + BA) = BC , (8). Din (7) şi (8) obŃinem
2( b + c ) 2( b + c ) 2(b + c) 2 ( b + c)
2bc 2bc
imediat rezultatul dorit: MP = 2
PN , (9); b) Aplicând modulele în a) găsim | MP |= 2 PN , de unde | MP |=
a a
2bc 2bc
= 2 2
| PN | , (10). Cum, folosind inegalitatea mediilor, găsim b + c ≥ 2bc, adică 2
2 2
≤ 1 , (11). Din
b +c b + c2
2bc
(10) şi (11) găsim că | MP |= 2 | PN |≤| PN | , de unde concluzia MP ≤ PN.
b + c2 A
7. i) În ∆ABA' cu transversala C' – M – C aplicăm teorema lui Menelaus,
C ' A BC MA ' C ' A A ' C MA
obŃinând: ⋅ ⋅ = 1 , de unde = ⋅ , (1). Analog în B'
C ' B A ' C MA C' B BC MA ' C' M
∆ACA' cu transversala B' – M – B aplicând teorema lui Menelaus găsim că:
B' A BC MA ' B' A BA' MA
⋅ ⋅ = 1 , adică = ⋅ , (2). Adunând (1) cu (2) suntem B A' C
B' C BA' MA B' C BC MA'
C' A B' A MA  A ' C BA'  MA
conduşi la: + =  + = , adică relaŃia cerută;
C' B B' C MA '  BC BC  MA '
A' M B' C C′A
ii) Notăm = m, = n, = p . Cum AA' ∩ BB' ∩ CC' = {M}, aplicând teorema lui Ceva, avem că
A' C B' A C′B
MA 1 MB 1 MC 1
mnp = 1, (3). Pe de altă parte, folosind (i) obŃinem = p + , (4); = m + , (5); = n + , (6).
MA ' n MB' p MC' m
MA MB MC  1 1 1
Din (4), (5) şi (6) găsim: + + + 2 = (m + n + p) +  + +  + 2 , (7). Pe de altă parte,
MA' MB' MC' m n p
MA MB MC  1  1  1 1 1 1 1 1 1
⋅ ⋅ =  p +  m +  n +  = mnp + + n + p + m + + + = ( m + n + p) +  + +
MA' MB' MC'  n  p  m m n mnp p m n
1 MA MB MC MA MB MC
+  + 2 , (8). Din (7) şi (8) avem relaŃia dorită + + +2= ⋅ ⋅ ;
p MA' MB' MC' MA ' MB' MC'

238
MA MB MC  1 1 1 1 1 1
iii) Folosind (7) găsim: + + = (m + n + p) +  + +  = m + + n + + p + ≥ 2 + 2 + 2 =
MA' MB' MC' m n p m n p
1
= 6. (Am folosit faptul că ∀ x > 0, x + ≥ 2 ).
x
18
8. a) Din AD = AB , deducem 18 MB + 17 MA = 35 MD (am aplicat A
35
formula vectorului de poziŃie). De aici 36MB + 34MA = 70MD , (2). łinând
1 D
cont de ipoteză găsim − 5MC = 70MD , de unde MD = − MC , (3), relaŃie
14
Q
ce arată că punctele M, C, D sunt coliniare; b) Aplicând teorema lui B C
Menelaus în triunghiul BDC cu transversala A – M – Q, găsim:
AD QB MC M
⋅ ⋅ =1.
AB QC MD
QB 5
Din a) anume (3) şi ipoteza, găsim că =− , (4). Atunci, folosind formula vectorului de poziŃie, găsim:
QC 36
36 5 36MB + 51MC 34 MA 41
MQ = MB + MC = = − MA , de unde = , raportul cerut.
41 41 41 41 MQ 34 A
9. Fie D mijlocul laturii BC, MN ∩ AD = {G} şi MN ∩ BC = {P}. Notând
DG M N
= x , vom avea aplicând Menelaus în ∆ABD cu transversala M – G – P: G
AG
B D C P
PB DG AM
⋅ ⋅ = 1 , (1). De asemenea aplicând teorema lui Menelaus în ∆ADC faŃă de transversala G – N – P:
PD AG MB
PD CN AG MB PB CN PC MB
⋅ ⋅ = 1 , (2). Din (1) = ⋅ x , (3) iar din (2) = ⋅ x , (4). Din (3) şi (4) avem +
PC NA GD MA PD NA PD MA
CN (PC + PB) 1 PC + PC + CD + BD
+ = x⋅ . Dacă G este centru de greutate al ∆ABC, x = , de unde =
NA PD 2 2(PC + CD)
2PC + 2DC 2(PC + CD) MB CN MB CN
= = = 1 , de unde + = 1 . Reciproc: dacă + = 1 , găsim că 1 =
2(PC + CD) 2(PC + CD) MA NA MA NA
x (PC + PB) 1 DG 1
= , de unde PD = x(PC + PB), de unde găsim x = , care arată că = şi cum G ∈ AD
PD 2 AG 2
mediană, rezultă că G este centru de greutate.
10. Dacă D ∈ (AC), conform ipotezei 'ABD, 'BAD sunt unghiuri ascuŃite. Ideea de B
demonstraŃie este sugerată de ipoteză: există 3 ceviene speciale. De aici ne aşteptăm
ca determinarea lui D să se facă în funcŃie de laturile triunghiului iniŃial. Fie (AE,
E ∈ (BD) bisectoarea din A, (BF), F ∈ (AD) mediana din B şi (DG), G ∈ AB, G I E
înălŃime, respectiv în ∆ABD. Din DG ⊥ AB şi m('ABC) = 90°, avem că DG || BC, α1
AG AD α2
(1). De aici, aplicând teorema fundamentală a asemănării, găsim = , (1), iar A F D C
GB DC

239
AB EB
conform teoremei bisectoarei aplicată în ∆ABD, = , (2). Cum {I} = DG ∩ AE ∩ BF, aplicând Ceva,
AD ED
BG AF DE DC AF AD
găsim ⋅ ⋅ = 1 , (3) şi conform cu (1) şi (2) relaŃia (3) se transcrie: ⋅ ⋅ = 1 , de unde CD =
GA FD EB AD FD AB
= AB, relaŃie ce demonstrează că punctul D este unic pe (AC).
11. Evident, problema pare un caz mai general al problemei piesei de 5 lei a lui
łiŃeica. Ca şi acolo, ni se sugerează folosirea reciprocei lui Ceva, mai ales că T1 ∈ O1 T 1 O2
OT O T
∈ (O1O2), T2 ∈ (O2O3) şi T3 ∈ (O1O3), (1). Ar trebui să avem că 1 1 ⋅ 2 2 ⋅
T1O 2 T2 O 3 I
O 3T3 T T2
⋅ = 1 , (2). Acest lucru este evident dacă Ńinem cont de (1). Într-adevăr, 3
T3O1
a b c O3
înlocuind în (2) în membrul stâng valorile corespunzătoare, găsim ⋅ ⋅ = 1 ,
b c a
adică 1 = 1.
12. Aproape automat ne gândim la cele două teoreme: Ceva şi Menelaus, teoreme A
cerute de ipotezele problemei. Sigur va fi o combinaŃie de aceste teoreme. Deci,
BA' CB' AC'
aplicând teorema lui Ceva în ∆ABC găsim ⋅ ⋅ = 1 , (1). Aplicăm apoi D B'
A' C B' A C' B C'
teorema lui Menelaus în triunghiurile BB'C' şi BB'C cu transversalele A – D – O,
AB C' D B' O AC B' O A' B O
respectiv A – O – A'. ObŃinem ⋅ ⋅= = 1 , (2); ⋅ ⋅ = 1 , (3). B A' C
AC DB' OB AB' OB A' C
AB C' D AB' A ' C
Dacă împărŃim (2) cu (3) obŃinem ⋅ ⋅ ⋅ = 1 , (4). ÎnmulŃind, în final, (1) cu (4), găsim că
AC' DB' AC A ' B
CB' AB C' D | CB' | | C' D | AC
⋅ ⋅ = 1 , (5), de unde avem rezultatul dorit ⋅ = = constant.
C' B AC B' D | C' B | | B' D | AB
A' B B' C C' A
13. Notăm = m, = n şi = p . Conform teoremei lui Ceva, mnp = 1.
A' C B' A C' B A
PA C' A B' A 1 PB
Aplicând teorema lui Van Aubel, obŃinem = + = p + , (1); =
A' P C' B B' C n PB'
A ' B C' B 1 PC A' C B' C 1 B'
= + = m + , (2); = + = n + , (3). ÎnmulŃind membru cu C' P
A ' C C' A p PC' A' B B' A m
PA PB PC  1  1  1  1
membru (1), (2), (3) găsim ⋅ ⋅ =  p +  m +  n +  = 2 +  p +  + B A' C
PA' PB' PC'  n  n  m  n
 1  1 PA PB PC
+  m +  +  n +  = 2 + + + = 2 + 2007 = 2009 .
 p  m PA ' PB' PC'
14. a) Fie AD mediană în ∆ABC, {G} = MN ∩ AD, G centrul de greutate al A
∆ABC. Aplicăm teorema lui Menelaus pentru ∆BAD şi transversala M – B – P.
PB DG AM M
ObŃinem ⋅ ⋅ = 1 , (1). Aplicând din nou teorema lui Menelaus în
PD GA MB G N
PD EN AG
∆DAC şi transversala G – N – P, de unde ⋅ ⋅ = 1 , (2). Din (1) obŃinem
PE NA GD
B D C P

240
| MB | 1 | PB | | NC | 1 | PC |
= ⋅ , (3) iar din (2) găsim că = ⋅ , (4). Adunând (3) cu
| MA | 2 | PD | | NA | 2 | PD |
| MB | | NC | 1 | PB | + | PC | 1 | PC | +2 | DC | + | PC | 1 2(| PC | + | CD |)
(4) rezultă + = ⋅ = ⋅ = ⋅ = 1 (Am Ńinut cont
| MA | | NA | 2 | PC | 2 |D| 2 | PD |
că BD ≡ DC);
b) Fie AA' bisectoarea interioară a unghiului 'A, A' ∈ BC şi AA' ∩ A
∩ MN = {I}. Din teorema bisectoarei aplicată în ∆BAA' faŃă de BI,
A' B A ' I C' B'
bisectoare {B'} = BI ∩ AC, obŃinem: = , (5). Analog, aplicând I
AB IA M
N
teorema bisectoarei în ∆AA'C faŃă de bisectoarea CI, unde {C'} = IC ∩
CA ' A ' I
∩ AB, obŃinem că = , (6), de unde folosind (5) şi (6) găsim că
CA IA B A' C P
A' I A' B CA ' A' B + A' C BC
= = = = , (7). Aplicând teorema lui Menelaus în ∆BAD' cu transversala M –
IA AB AC AB + AC AB + AC
PB A ' I AM
– I – P, găsim ⋅ ⋅ = 1 , (8) şi repetând procedeul în ∆AA'C cu transversala I – N – P obŃinem că
PA ' IA MB
PA ' CN AI MB PB A ' I NC A' I
⋅ ⋅ = 1 , (9). Ca la punctul anterior, din (8) găsim = ⋅ , (10) iar din (9) = ⋅
PC NA A' I MA PA ' IA NA IA
PC MB PB BC NC PC BC
⋅ , (11). Din (7), (10), (11) ajungem la = ⋅ , (12) respectiv = ⋅ , (13).
A' P MA PA ' AB + AC NA A' P AB + AC
| AC | | AB | | AC | | MB | | AB | | NC |
ÎnmulŃind (12) cu iar (13) cu şi adunând cele două relaŃii, obŃinem: ⋅ + ⋅ =
| BC | | BC | | BC | | MA | | BC | | NA |
AC BC PB AB BC PC 1  AC ⋅ PB + AB ⋅ PC  1
= ⋅ ⋅ + ⋅ ⋅ = ⋅  = ⋅
BC AB + AC PA ' BC AC + AB PA ' AB + AC  PA '  AB + AC
AC(PA '+ A' B) + AB(PA '−A ' C) 1  A ' P (AC + AB) (AC ⋅ A' B − AB ⋅ A ' C) 
⋅ = ⋅ + =1+
PA ' AB + AC  A' P A' B 
AC ⋅ A' B − AB ⋅ A ' C AB BA'
+ = 1 , deoarece aplicând teorema bisectoarei faŃă de bisectoarea AA' găsim = ,
A' B(AB + AC) AC A ' C
relaŃie echivalentă cu AC ⋅ BA' – AB ⋅ A'C = 0.

VII. Perpendicularitate.
erpendicularitate. RelaŃii metrice. Produs scalar
1 D N C
1. Pentru demonstraŃie vom arăta că produsul scalar AM ⋅ BN = 0 ; AM ⋅ BN = (AB +
4
1 1 M
+ AC)(BC + BD) = (AB ⋅ BC + AB ⋅ BD + AC ⋅ BC + AC ⋅ BD) = (AB ⋅ BD + AC ⋅ BC) ,
4 4
2
(1). Dar AB ⋅ BD = −BA ⋅ BD = − | BA | ⋅ | BD | ⋅ cos ('ABD) = − ⋅ | BA | ⋅ | BD | , (2). A B
2
2
Analog AC ⋅ BC = CA ⋅ CB =| CA | ⋅ | CB | ⋅ cos('ACB) = | CA | ⋅ | CB | , (3). Înlocuind (2) şi (3) în (1),
2
obŃinem relaŃia cerută.

241
CB A
2. Din AD ⊥ BE, urmează că AD ⋅ BE = 0 , (1). Dar AD = AC + ; BE = BC +
2
2
CA AC ⋅ CA BC
+ CE = BC + . Revenind în (1) găsim AD ⋅ BE = AC ⋅ BC + ⋅ +
3 3 2 E
CA ⋅ CB
+ = 0 , (2). De aici, folosind teorema cosinusului şi notaŃiile consacrate,
6 B D C
b2 a 2
BC = a, CA = b, AB = c, relaŃia (1) se scrie b ⋅ c ⋅ cos('C) – − + ab cos Ĉ = 0,
3 2
a 2 + b2 − c2 7a 2 + 7 b 2 − 7c 2
de unde 7ab ⋅ cos('C) – 2b2 – 3a2 = 0. Înlocuind ab ⋅ cos('C) = , avem − 2b 2 −
2 2
– 3a2 = 0, adică a2 + 3b2 – 7c2 = 0. Dar b 2 = a 2 + c 2 − 2ac cos B̂ = a2 + c2 – ac, de unde a2 + 3a2 + 3c2 – 3ac –
a a 3 + 73
– 7c2 = 0, de unde ecuaŃia omogenă 4a2 – 3ac – 4c2 = 0. ÎmpărŃind cu c2, şi notând x = , găsim = .
c c 8
3. Raportăm triunghiul BCA la reperul xOy din figură. Formând produsul
y
B̂ a b  B(0,b)
scalar DC' ⋅ DA ' şi notăm ϕ = . Avem DC' ⋅ DA' =  i + j  ⋅
2 2 6 
ϕ
a b  a 2 b2
⋅  i + j = − + , (1). Pe de altă parte, DC' ⋅ DA' =| DC' | ⋅ | DA' | ⋅  a b a b
2 6  4 36 A'  − ,  60° C'  , 
 2 2 2 2
1  a 2 b 2  a 3
⋅ cos 60° =  +  , (2). Din (1) şi (2) obŃinem = = tgϕ . De D x
2  4 36  b 9
C(–a,0) O A(a,0)
13 13
aici cos 2ϕ = , de unde 'B = arccos .
14 14 A
4. ∆ABC, O centrul cercului, O este şi centru de greutate. De aici OA + OB + OC = 0 .
2 2
De aici MA = MO + OA , (1) şi analoagele. Avem MA ⋅ MA = MA = MO + 2MO ⋅
2 O C
⋅ OA + OA , (2). Cum OA = OB = OC = OM = R, adunând relaŃiile de tip (2), găsim:
2 2 2 2 2 2 2
MA + MB + MC = 3MO + 2MO(OA + OB + OC) + OA + OB + OC = 6R 2 . B M

5. Notăm BC = a , AC = b, AB = c . Dublă implicaŃie. (⇔) Presupunem că


 c + b  − c + a  2 A
0 = AA' ⊥ BB' ⇔    = 0 ⇔ −c + c ⋅ a − b ⋅ c + a ⋅ b = 0 ; − c 2 − ac ⋅
 2  2 
  
b2 + c2 − a 2 B'
⋅ cos('B) – bc ⋅ cos('A) + ab ⋅ cos('C) = 0. Cum bc ⋅ cos('A) = şi
2
a 2 + c2 − b2 b2 + c2 − a 2 a 2 + b 2 − c2 G
analoagele, suntem conduşi la − c 2 − − + =
2 2 2 B A' C
= 0, de unde –2c2 – a2 – c2 + b2 – b2 – c2 + a2 + a2 + b2 – c2 = 0, de unde 5c2 = a2 + b2.

242
6. a) Fie l1 , l 2 , l3 versorii vectorilor OA, OB, OC . Notăm s = l1 + l 2 + l3 . Evident C
2     2  2  2
s = (l1 + l2 + l3 )2 ≥ 0 , din proprietăŃile produsului scalar. ObŃinem l1 + l 2 + l3
+ 2(l1 ⋅ l 2 + l1 ⋅ l3 + l 2 ⋅ l3 ) ≥ 0 ; 3 + 2(cos 2A + cos 2B + cos 2C) ≥ 0, de unde 2B
O 2A
3
inegalitatea cerută: cos 2A + cos 2B + cos 2C ≥ − , (1); b) Cum 1 – cos 2A = 2C
2
A B
3
= 2sin2 A şi analoagele, retranscriem (1): 3 – cos 2A – cos 2B – cos 2C ≤ 3 + .
2
9 9
Imediat 2(sin2A + sin2B + sin2C) ≤ , de unde relaŃia cerută: sin2A + sin2B + sin2C ≤ .
2 4
7. Considerăm l1 , l 2 , l3 versorii vectorilor BC, CA, AB . Notăm s = l1 + l2 + l3 . A
2    l3
Aplicând produsul dintre s şi el însuşi, găsim s = (l1 + l2 + l3 )2 ≥ 0 (am
 2  2  2
folosit proprietăŃile produsului scalar). s2 = l1 + l2 + l3 + 2(l1 ⋅ l2 + l3 ⋅ l1 + l2 ⋅
l2
⋅ l3 ) ≥ 0 . Imediat 3 + 2[cos(π – C) + cos(π – A) + cos(π – B)] ≥ 0, de unde π–C l2
3 B l C
cos A + cos B + cos C ≤ . 1
2
8. Avem MA = MO + OA , de unde MA2 = R2 + OA2 + 2MO ⋅ OA = 4R2 + 2MO ⋅ OA . Analog MB2 = (MO +
+ OB) 2 = 2R 2 + 2MO ⋅ OB şi analog MC 2 = 2R 2 + 2MO ⋅ OC . Atunci: MB2 + MC2 = 4R2 + 2(OB + OC)MO =
= 4R 2 + 2MO ⋅ OA = MA 2 , de unde, folosind reciproca teoremei lui Pitagora, avem că triunghiul format din
MA, MB, MC este dreptunghic.
9. a) Fie BC = BA + AC , de unde | BC |=| BA + AC |≤| BA | + | AC | , de unde a ≤ b + c, (1). Pe de altă parte,

| BC |=| BA − CA |≥ | BA | − | CA | , adică a ≥ |b – c|, (2). Cum A ∉ (BC), inegalităŃile din (1) şi (2) vor fi stricte;

b2 + c2 − a 2
b) Din |b – c| < a < b + c rezultă că b2 – 2bc + c2 < a2 < b2 + c2 + 2bc, adică –1 < < 1 . Considerăm
2bc
două semidrepte Ax şi Ay pe care luăm punctele B, respectiv C, astfel încât AB = c, AC = b. Atunci BC2 = b2 +
+ c2 – 2bc ⋅ cos(' (AB, AC) ) = a2, de unde BC = a.
10. a) Din triunghiurile ABM şi ACM avem AC – BM < AM < AC + BM, AC – CM < AM < AC + CM, adică
a a a a
b − < m a < b + , c − < m a < c + . Prin însumare obŃinem b + c – a < 2ma < a + b + c, (1);
2 2 2 2
b) Folosind a) prin scrierea relaŃiilor de tip (1) pentru mb, respectiv mc, însumarea membru cu membru, se ajunge
la inegalitatea dorită: p < ma + mb + mc < 3p.

243
VIII. Transformări geometrice
8.1. TranslaŃia
C
1. Folosim translaŃia. Fie M = t AB (D) . Patrulaterul ABMD este paralelogram şi deci

'BMD = 60°, iar 'MDA = 120° = 'MBA. Tot din translaŃie, MD ≡ AB = 3 . În M B


∆MDC avem m('MDC) = 60°. Aplicăm teorema cosinusului, MC2 = MD2 + DC2 –
– 2MD ⋅ cos('MDC), de unde MC = 3. Aplicând reciproca teoremei lui Pitagora,
acum în ∆MDC, găsim 'CMD = 90°, de unde 'CMB = 30°. De asemenea ∆MCB D A
isoscel cu 'CMB = 30° = 'CBM. Imediat m('CBA) = 120° + 30° = 150°. Imediat
m('BCD) = 360° – (60° + 60° + 150°) = 90°. A
2. Folosim translaŃia! Fie M = t BC (A) . Patrulaterul ABCM este paralelogram şi deci
6 3
'BCM = 30°, 'BCD = 360° – (90° + 60° + 150°) = 60°. În ∆MDC aplicăm teorema
cosinusului şi găsim: MD2 = DC2 + MC2 – 2DC ⋅ MC ⋅ cos 30°; MD2 = 36, de unde B
MD = 6. Aplicând reciproca lui Pitagora în acest triunghi MDC, găsim 'DMC = 90°. 150°
Cum 'MDC = 60°, urmează că 'MDA = 'MAD = 30°. Deci AM ≡ AD şi 'AMD = 30°
90 30°
1
= 120°. Aplicând teorema cosinusului în ∆AMD găsim că AD2 = 36 + 36 + 2 ⋅ 36 ⋅ . D° C
2 P
ObŃinem AD = 6 3 . Cum BC = AM = 6, am obŃinut laturile patrulaterului ABCD.
3. Fie patrulaterul ABCD în care AD ≡ BC, iar M şi N mijloacele lui DC, respectiv
AB. Aplicăm segmentului |DA| o translaŃie Tv de vector v1 = DM şi segmentului
C
M
|BC| translaŃia Tv de vector v 2 = CM . Se formează paralelogramele DAA'M, res-
2 D
pectiv BCMB'. De aici MA' ≡ MB', deci ∆MA'B' isoscel. Cum AA' ≡ BB', urmează că A' G B
AA'BB' este paralelogram, deci N ∈ A'B'. Urmează că MN mediană în ∆MA'B'
N
isoscel, deci şi bisectoare. Cum 'A'MB' ≡ '(AD, BC), unghiuri cu laturi paralele, deci A B'
bisectoarele lor vor fi paralele.

4. Construim DP ⊥ BC. Fie E ortocentrul triunghiului BKH ({E} = B K' P C


= KK' ∩ HH' ∩ KK'). Aplicăm triunghiului BKE translaŃia Tv
E
de vector v = KD . Vom avea D = Tv K, P = Tv B, H = Tv E . Cum H' H
b
translaŃia este o izometrie ∆BEK ≡ ∆PHD, iar laturile omoloage
paralele. Deci BE ≡ PH şi BE || PH, de unde, cum BE ⊥ KH, PH ⊥ a
B'
⊥ KD, deci m('PHK) = 90°. De aici KH2 = PH2 + PK2. ObŃinem
A K D
PH = KH 2 − PK 2 . Cum BD ≡ PK (diagonale în dreptunghi),
D C
găsim PH = b2 − a 2 .
5. Considerăm două translaŃii ale figurii F, amândouă de vector v , cu | v |= 0,001 şi direcŃii M
care fac un unghi de 60°. Fie F1 şi F2 figurile obŃinute prin aplicarea translaŃiei figurii F. O 60°
Evident F ∩ F1 = F ∩ F2 = ∅ = F1 ∩ F2 (!!! Dacă F1 ∩ F2 ≠ ∅, atunci distanŃa dintre ele ar N
A B

244
fi 0,001. Absurd. La fel F ∩ F1 = ∅. Fie A' ∈ F şi A ∈ F punctul corespunzător din A care trece în A'. Atunci
|AA'| = 1, absurd). Construim în exteriorul pătratului ABCD pătratul A'B'C'D' cu laturile la diustanŃa de 0,001 de
laturile pătratului. Evident orice punct din F1, F, F2 se găseşte în interiorul acestui pătrat A'B'C'D'. ObŃinem
SF + SF1 + SF2 ≤ SA'B'C'D' = 1,004004. Dar SF = SF1 = SF2 (figurile obŃinute prin translaŃie pentru care aria este
1,004004
invariant!). De aici SF < < 0,34 .
3

6. Notăm AC = b, BD = c. Fie translaŃia de vector v = AB , E = t v C . D C E


Cum AC || BE, avem 'CBE = π – θ. Aplicăm teorema cosinusului în θ
∆BDE. ObŃinem DE2 = BD2 + BE2 – 2BD ⋅ BE ⋅ cos(π – θ) = b2 + c2 + O
b c
+ 2bc ⋅ cos θ. Dar DE = CD + CE, de unde (CD + a)2 = b2 + c2 + 2bc ⋅
A a B
⋅ cos θ. Notând x = CD găsim ecuaŃia x2 + 2ax + a2 – b2 – c2 – 2ac ⋅
⋅ cos θ = 0. SoluŃia care convine va fi CD = b 2 + c 2 + 2bc cos θ − a . A v A'

7. a) d separă punctele A şi B. Fie v un vector de aceeaşi direcŃie cu d şi


N d
| v |= a . Considerăm v = AA' , adică t v A = A' . Fie A'B ∩ d = {N},
M' N' M
t − v N = M ∈ d. Linia poligonală AMNB verifică proprietatea. Într-adevăr dacă
B
M', N' ∈ d, M'N' = a, avem AM + MN + NB = A'N + MN + NB = A'B + MN =
= A'B + M'N' < A'N' + N'B + M'N' = AM' + M'N' + N'B;
b) d nu separă punctele A şi B. Fie v un vector de direcŃie d şi | v |= a . Fie
B
t v A = A' . Evident AA' =| v | . Fie A'' simetricul lui A' faŃă de d, A''B ∩ d =
A A'
= {N}, iar t − v ( N) = M ∈ d, atunci linia poligonală AMNB are proprietatea
din enunŃ. Într-adevăr, oricare ar fi M', N' ∈ d, cu M'N' = a, avem AM +
+ MN + NB = A''B + M'N' ≤ A''N' + N'B + M'N' = A'N' + N'B + M'N' =
= AM' + M'N' + N'B. M N M' N'

A
A''
8. Fie ACDEFB un drum oarecare. Construim A' = C
= t CD A şi t fe B' = B . Avem AC + CD + DE + EF + A' M

+ FB = AA' + A'D + DE + EB' + B'B. Drumul cel D


mai scurt se obŃine când punctele A', D, E, B', B sunt N
coliniare.
P E

Q F
B'

245
8.2. Omotetia
A
 1  1  1
1. Pentru prima omotetie H A,  ; a) H A,  (A) = A; H A,  (B) = B', cu
 2  2  2
1  1  1 B' C'
AB' = AB ; H A,  (C) = C', cu AC' = AC . Triunghiul A'B'C' este: (A'B'C') =
2  2 2
= H  1  (ABC) (vezi Fig.1).
 A,  B C
 2
Fig.1
Pentru cealaltă omotetie H(G,2) obŃinem: H(G,2)(A) = A', GA' = 2GA ; H(G,2)(B) = A'
= B', GB' = 2GB ; H(G,2)(C) = C', GC' = 2GC . Triunghiul A'B'C' este (A'B'C') =
= H(G,2)(ABC) (vezi Fig.2); b) Fie triunghiul ABC cu AB = AC. Atunci H(0, k) = A
= A', OA' = k OA ; H(0, k) = = B', OB' = k OB . Imediat A' B' = A' O + OB' = k ⋅
⋅ AO + k OB = k (AO + OB) = k AB , (1). Analog găsim A' C' = k AC , (2), de
unde, aplicând modulele în (1) şi (2) găsim A'B' = A'C', adică omoteticul unui B C
triunghi isoscel este un triunghi isoscel. Absolut analog în cazul triunghiului B' C'
dreptunghic şi cel echilateral. Fig.2
1
2. a) Dacă există acest punct O, avem f(0) = O, de unde OO = (3AO + BO + CO + DO) , de unde 3AO + BA +
5
1
+ AO + CA + AO + DA + AO = 0 , adică AO = (AB + AC + AD) , deci O este unic determinat;
6
b) Dacă f = H(0, k), atunci OM' = k OM , (∀) M ∈ (ABC), unde M' = f (0) . Cum OM' = OM + MM' = OM +
1 1 11 11
+ (3AM + BM + CM + DM) = OM + (6OM + 3AO + BO + CO + DO) = OM , de unde k = , de unde
5 5 5 5
 11 
f = H 0,  .
 5
3. Pentru fiecare punct M există un unic punct O astfel încât Of (M) = k OM , relaŃie echivalentă cu MB =
1
= Mf (M) . Punctul O nu depinde de alegerea lui M. Într-adevăr, dacă se consideră un alt punct N ≠ M şi
1− k
1 1 1
punctul O', pentru care O' f ( N) = k O' N . Atunci OO' = OM + MN + NO' = Of (M) + f (M)f ( N) + f ( N)O' =
k k k
1 1
= [Of (M) + f (M)f ( N) + f ( N)O'] = OO' , adică (k − 1)OO' = O sau OO' = O , de unde O ≡ O'. Prin urmare
k k
Of (M) = k ⋅ OM , (∀) M ∈ E. O este un punct fix al lui f, adică f(0) = O, căci Of (O) = OO = O . În concluzie f =
= H(O,k).
4. Din H  3  (G) = M , atunci BM ≡ MC. Analog, H  3  (G ) = N , P A
 A,   B, 
 2  2
P
atunci AN ≡ NC. Fie O mijlocul lui AB şi {R} = NO ∩ AM. Atunci R N
AP 2 O
R este mijlocul lui AM. Cum H  2  ( M ) = P , obŃinem = şi cum
 1,  AM 5 G
 5
B M C

246
AR 1 AP 1 PR 1 NP 1
= , suntem conduşi la = , de unde = , (1). Cum ND = 5NP avem = , (2). Din (1) şi
AM 2 AR 5 AP 4 PD 4
(2) obŃinem ∆APD ~ ∆RPN, adică RN || AD. Cum RN || BC obŃinem AD || || BC, (3). Dar AD = 4RN = 4 ⋅
1 1 1
⋅ ON = 4 ⋅ ⋅ BC = BC , (4). Din (3) şi (4) ABCD paralelogram. A
2 2 2
5. Notăm A1 mijlocul lui BC şi B1 mijlocul laturii AC. Avem CN = P
3 3 BN BG 2 B1
= CM, BP = BN ; AQ = 2AP. Avem = = , adică GN ||
4 2 BP BB1 3 G N* * Q
2 AP AB1 1 M
|| B1B şi GN = B1P . Analog = = , de unde B1P || CQ şi
3 AQ AC 2
1 1 B A1 C
B1P = CQ . ObŃinem GN = CQ şi GN || CQ. Rezultă că ∆GMN ~
2 3
~ ∆QMC, adică G, M, Q coliniare şi Q = H(G,4)(M).
4 A
6. Avem H  4  (M) = B conduce la BC = CM . Analog, H  3  (B) = N
 C, −  3  A, 
 3  4
R S T
3 2
conduce la AN = AB ; H  2  (M) = P conduce la AP = AM .
4  A,  3 C' Q
 3

3 AC + 3AB 2 N P
Dar AM = CM − CA = AC − (AC − AB) = ; AP = AM =
4 4 3
AC + 3AB 1 1 2AC − 3AB B M C
= ; NP = AP − AN = AC − AB = . Fie C' ∈ (AB)
6 6 4 12
2 1
astfel încât C'A = C'B. Imediat AC' = AN . Avem ∆AC'P ~ ∆ANM, C'P || NM, rezultând că C' P = AC . Fie
3 6
1 2 1
R ∈ AB, AR ≡ RC'. Construim RS || C'P. Imediat AS = AP şi C' P = NM = AC . Ducem ST || PQ ⇒ T
2 3 12
1 1 1 1 1
mijlocul lui AQ, deci AT = AQ . Dar AR = AB , de unde AT = AC ; RT = BC = (AC − AB) . Cum
2 4 4 4 4
3AC − 3AB − AC 2AC − 3AB 1 2AC − 3AB 2AC − 3AB
ST = RT − RS = = ; ST = PQ , de unde PQ = 2 ⋅ = .
12 12 2 12 6
1 1
Urmează că PQ = 2 NP, NQ = 3NP şi cum NC' = 3NA . Urmează că C' P = AQ şi cum 3C' P = AC , conduce
3 2
1 1 1 A
la 3 ⋅ AQ = AC , adică AQ = AC . Urmează că H  1  (C) = Q .
3 2 2  A, 
 2
1 B1
7. Fie M, N, P mijloacele laturilor BC, CA, AB. Avem AA2 = AC1 + A2
k +1 P N
k 2k 2k 2k G
+ AB1 = AM . Analog BB2 = BN , CC2 = ⋅ C1
k +1 (k + 1) 2 (k + 1) 2 (k + 1)2 C2
B2
⋅ CB . Evident, atunci AA2, BB2, CC2 sunt concurente în G, centrul de
B M A2 C

247
2k 2
greutate al triunghiului ABC. Avem AA2 = AM, AG = AM .
(k + 1) 2
3
2(k 2 − k + 1) k2 − k +1
De aici GA2 = AG – AA2 = ⋅ AM = GA . De aici:
3(k + 1) 2 (k + 1) 2
GA 2 k 2 − k + 1 GB2 GG 2
= = = . Prin urmare, triunghiul A2B2C2 se obŃine din triunghiul ABC printr-o
GA (k + 1) 2 GB GC
k2 − k +1
omotetie de centru G şi raport .
(k + 1) 2
8. Evident dacă A, B', C', D sunt punctele de intersecŃie cu cele două cercuri ale M T
coardei AB, A, B ∈ C1 iar B', C' ∈ C2. Fie A ∈ C1, B punctul diametral opus lui A. A N
* * C
1
Fie B', punctul determinat de condiŃia AB' = AB . B' O
3
B
Cazul 1. Dacă r1 = 3r2. În acest caz, B' ∈ C2 şi diametrul AB este soluŃia problemei,
C' fiind diametral opus lui B' în C2.
Cazul 2. r1 > 3r2, atunci B' ∈ ext C2(O, r2) şi problema nu are soluŃie. Într-adevăr fie AC care trece prin A şi
intersectează C2(O, r2) în M şi N. Dacă T este mijlocul lui AC, deci şi a lui MN, atunci MT2 = OM2 – OT2 =
1 1 1 1 1 1 1 1
= r22 − BC 2 < r12 − BC2 = AB2 − BC 2 < (AB2 − BC 2 ) = AC2 . Imediat MT < AC , de unde
4 9 4 36 4 36 36 6
1
MN < AC .
3
1
Cazul 3: r1 < 3r2. Fie omotetia de centru A şi de raport . Atunci H  1  (B) = B' . Din proprietăŃile omotetiei,
3  A, 
 3

 AB' 
cercul C1(O, r1) se transformă în cercul C3  A,  . Fie {M1, M2} = C2 ∩ C3. Ducem prin M1 respectiv M2 cele
 2 
două coarde care rezolvă problema.

IX.1. AplicaŃiile calculului vectorial în geometrie.


Demonstrarea unor teoreme celebre

C' A A' B B' C A


1. Fie ∆ABC şi A'–B'–C' transversală. Fie = α, = β şi = γ.
C' B A' C B' A
C'
Punem condiŃia ca vectorii C' B' şi C' A' să fie coliniari. Avem, pe rând, B'

1 BC + γ BA  γ 1  1
C' B' = C' B + BB' = − BA + =  − BA +
 ⋅
α +1 1+ γ 1+ γ α +1 γ +1 B C A'
1 β
⋅ BC , (1); C' A = C' B + BA' = − BA + BC , (2). Din (1) şi (2) cei doi vectori sunt coliniari dacă şi
α +1 β −1

248
γ 1 1

1+ γ α +1 1+ γ
numai dacă = , relaŃie echivalentă cu αβγ = 1.
1 β

α +1 β −1
ObservaŃie. Evident, α, β, γ ∉ {–1, 1}, căci în caz contrar transversala C'–B'–A' se transformă în paralelă cu BC
şi similarele.
C' A A' B B' C A
2. Notăm = α, = β, = γ . Fie {P} = AA' ∩ BB' ∩ CC'. Atunci
C' B A' C B' A
r P = x ⋅ r A ' + (1 − x )r A ; r P = y ⋅ r B' + (1 − y)r B ; r P = z ⋅ r C' + (1 − z)r C , unde C' B'
r B + βr C rC + γrA r A + αr B
r A' = , r B' = , r C' = . Dacă considerăm originea P
1+ β 1+ γ 1+ α
x βx y B A' C
vectorilor de poziŃe A obŃinem: r P = + r C ; r P = (1 − y)r B + rC ;
1 + βr B 1 + β 1+ γ
αz x αz βx y
rP = r B + (1 − z)r C . De aici avem: = 1− y = şi = = 1 − z . Eliminând în aceste relaŃii
1+ α 1+ β 1+ α 1+ β 1+ γ
(1 + α)(1 − y) 1+ β β(1 + γ )
x, y, z obŃinem αβγ = 1. Deoarece x = (1 – y)(1 + β), z = , de unde x = ,y= ,
α 1 + γβ + β 1 + γβ + β
1+ α 1
z= ⋅ .
α 1 + γβ + β
B' C'
Rezultatul se păstrează dacă două dintre puncte sunt prelungirile laturilor, iar al
treilea este pe o latură, dar nu în vârfurile triunghiului. EsenŃial este ca două dintre A
dreptele AA', BB' şi CC' să fie concurente, căci în caz contrar putem avea de
1
exemplu, situaŃia din figura alăturată pentru α = , β = 1, γ = 2.
2 B A' C

BM
3. Cum M este un punct pe latura BC, considerăm raportul = k , de unde folosind vectorul de poziŃie
MC
1 k
obŃinem: AM = AB + AC . Ridicând la pătrat (practic aplicând produsul scalar AM ⋅ AM ), găsim:
1+ k 1+ k
(k + 1) 2 AM 2 = AB2 + k 2 AC2 + 2k AB ⋅ AC = AB2 + k 2 AC2 + 2k ⋅ (AB ⋅ AC ⋅ cos('BAC) = AB2 + k2AC2 + 2k ⋅
BA 2 + AC 2 − BC 2 BM
⋅ = (1 + k )AB2 + k (k + 1)AC2 − kBC 2 . Înlocuind k = , obŃinem relaŃia cerută.
2 MC

4. În demonstraŃie vom folosi relaŃia lui Euler: OM ⋅ NP + ON ⋅ PM + A


+ OP ⋅ MN = 0 , (1). Pentru început aplicăm (1) punctelor ω, A', B', C'. d ω
B'
ObŃinem ωA ' ⋅ B' C' + ωB' ⋅ C' A ' + ωC' ⋅ A ' B' = 0 . Vrem să demonstrăm că C'
A'
ωA ' ⊥ BC , ceea ce încheie demonstraŃia. Pentru aceasta observăm
că BC = BB' + B' C' + C' C , CA = CC' + C' A ' + A ' A , AB = AA' + A' B' +
 B C
+ B 'B . A arăta că ωA ' ⊥ BC este echivalent cu a arăta că ωA ' ⋅ BC = 0 .

249
Avem ωA ' ⋅ BC = ωA ' ⋅ BC + ω
 ' ⋅ CA
B +ω
C ⋅ AB
'  = ωC'(AA' + A' B' + B' B) = ωA' ⋅ B' C' + ωB' ⋅ C' A' + ωC' ⋅ A' B' +
0 0

+ A' A(ωB' − ωC') + B' B(ωC' − ωA') + C' C(ωA' − ωB') = A' A ⋅ C' B' + B' B ⋅ A' C' + C' C ⋅ B' A' = 0 , căci A'A ⊥ C'B',
B'B ⊥ A'C', C'C ⊥ B'A'. În final am obŃinut relaŃia dorită: ωA ' ⋅ BC = 0 .
5. Avem G centrul de greutate al ∆ABC, A', B', C' mijloacele laturilor BC, CA, A
2 2 2
AB. Folosim produsul scalar MA2 + MB2 + MC2 = MA + MB + MC = b
c
= (MG + GA) 2 + (MG + GB) 2 + (MG + GC) 2 = 3MG 2 + 2MG (GA + GB + GC) +
 C' B' M
0 G
2
2 2 2 2  4
+ GA + GB + GC = 3MG 2 + GA 2 + GB2 + GC 2 . Dar GA2 =  A' A  = ⋅
3  9
2 B a A' C
 AB + AC  1 1 2
⋅   2 2
= (AB + AC + 2AB ⋅ AC ⋅ cos Â) = (c + b + b + c – a ) =
2 2 2 2
 2  9 9
 
1 1
= (2c2 + 2b2 – a2). Analog, calculând GB2, GC2, obŃinem în final: MA2 + MB2 + MC2 = 3MG2 + (2c2 + 2b2 –
9 9
2 2 2
1 a +b +c
– a2 + 2a2 + 2c2 – b2 + 2b2 + 2a2 – c2) = 3MG2 + (3c2 + 3b2 + 3a2) = 3MG2 + .
9 3
2
6. Presupunem că cele trei perpendiculare sunt concurente în O. Atunci A1B – A
2 2
– A1C2 = A1B − A1C = (A1O + OB) 2 (A1O + OC) 2 = A1O2 + 2A1O ⋅ OB+ OB2 −
B1
− A1O 2 − 2A1O ⋅ OC − OC 2 = BO 2 − OC 2 + 2A1O(OB − OC) = OB2 − OC 2 + C1
+ 2A1O(CO + OB) = OB2 − OC 2 + 2A1O ⋅ CB = OB2 – OC2. (Am Ńinut cont că O
O1A ⊥ BC. Analog B1C – B1A = OC – OA , C1A – C1B = OA – OB .
2 2 2 2 2 2 2 2

Adunând cele 3 relaŃii anterioare membru cu membru, obŃinem rezultatul dorit. B A1 C


Reciproc: presupunem că are loc relaŃia noastră, OA1 ⊥ BC, OB1 ⊥ AC. Fie {O} = A1O ∩ OB1, iar OC2 ⊥ AB,
C2 ≠ C1. Atunci are loc, conform cu necesitatea. AC 22 + BA12 + CB12 = BC 22 + CA12 + AB12 , (1) şi în plus, conform
ipotezei, AC12 + BA12 + CB12 = BC12 + CA12 + AB12 , (2). Scăzând membru cu membru (1) şi (2), obŃinem AC 22 −
2 2 2
− AC12 = BC 22 − BC12 . Notăm AB = c, AG = x, C 2 C1 = y, C1B = z . Atunci AC 2 − AC1 = BC 2 − BC1 , de
unde x2 – ( x + y) 2 = (a − x ) 2 − (−a − x − y) 2 . După calcule găsim 2a ⋅ y = 0 , posibil doar dacă a ⊥ y , absurd,
a = 0 , absurd, y = 0 convine, de unde C1 ≡ C2.
7. Presupunem că A, B, C, D sunt vârfuri ale unui paralelogram. Atunci AC = D C
2 2 2 2
= AB + AD ; AC = (AB + AD) = AB + AD + 2AB ⋅ AD . Analog BD = BC + O
2 2 2 2
+ CD , de unde BD = (BC + CD) = BC + CD + 2BC ⋅ CD . Adunând membru
cu membru, găsim AC2 + BD2 = AB2 + BC2 + CD2 + AB2 + 2AB ⋅ AD − 2BC ⋅ DC . A B
2 2
Cum ABCD paralelogram, obŃinem AC2 + BD2 = AB2 + BC2 + CD + AB .
Reciproc: Presupunem relaŃia satisfăcută. Fie E mijlocul lui AC şi F mijlocul lui BD. Vom arăta că E ≡ F,
ceea ce arată că patrulaterul ABCD paralelogram. Notăm AB = a , BC = b, CD = c, AC = m, BD = n . Avem

250
EF = EA + AB + BF ; EF = EC + CD + DF şi adunând membru cu membru, găsim 2EF = AB + CD + EA + EC +
2
+ BF + + DF = a + c . Cum a + b + c + d = 0; 2EF = −b − d . Imediat 4EF = a 2 + c 2 + 2a c = b 2 + d 2 + 2bd . Pe de
altă parte 4EF = a + c − b − d , de unde imediat: 16EF = a2 + b2 + c2 + d2 + 2a c + 2bd − 2(a b + a d + bc + cd ) sau
8EF2 = 2(a b + a d + bc + cd) , (1). Dar AC = a + b = −c − d , de unde a2 + b2 + 2a b = m 2 = c 2 + d 2 + 2cd , (2).
Analog BD = b + c = −a − d . łinând cont de (2) şi înlocuind în (1), găsim: –8EF2 = m2 – a2 – b2 + m2 – c2 – d2 +
+ n2 – b2 – c2 + n2 – a2 – d2 sau –8EF2 = 2(m2 + n2 – a2 – b2 – c2 – d2) sau –4EF2 = m2 + n2 – a2 – b2 – c2 – d2.
2
Cum m2 + n2 – a2 – b2 – c2 – d2 = 0, găsim − 4EF = 0 , de unde EF = 0 , adică E ≡ F.
ObservaŃie: Teorema lui Euler mai este cunoscută şi sub numele de teorema lui Apollonius.
8. Fie k raportul în care sunt împărŃite laturile. Atunci AP = k ⋅ PB, BM = k ⋅ A
1
⋅ MB, CN = k ⋅ NA . Fie G centrul de greutate al ∆ABC; GP = GA +
1+ k P
k 1 k 1 k N
+ GB ; GM = GB + GC ; GN = GC + GA . G
1+ k 1+ k k +1 1+ k k +1
Adunând membru cu membru, obŃinem GP + GM + GN = GA + GB + GC =
= 0 , ceea ce demonstrează teorema. B M C

9. Prelungim AC şi AB cu AM = BE, AN = CD şi construim NAMP


M
paralelogram. ObŃinem că AP este bisectoare şi diagonală în para-
N O1
lelogramul ANPM. Urmează că ANPM este romb. De aici
2 1
| AN |=| AM | . Cum | CE |=| BD | şi BE = BC + CE ; CE = CB + BE . A
Imediat ∆BEC ≡ ∆DCB (LLL), căci | BE | = | BD | iar | BE | =| AM |= 12
E D
=| AN | =| CD | . Urmează că 'EBC ≡ 'DCB, adică ∆ABC isoscel.
O
B C

10. Notăm cu M, N, P mijloacele diagonalelor AC, BD, EF. Fie u = AB , E


u+v 1
v = AD , AF = pu, AE = q v . Avem AN = , AP = (pu + q v); AM =
2 2
D
1 FC BC 1 C
= AC . Atunci notăm = x iar = y . De aici AC = pu + P
2 CD EC 1+ x
v M* *
x 1 yq v p 1 x yq N
+ v , iar AC = u+ . ObŃinem = ; = , de
1+ x 1+ y 1+ y 1+ x 1+ y 1+ x 1+ y A B F
u
q(p − 1) (p − 1) 1  p(q − 1) q (p − 1) 
unde x = ,y = . De aici AM =  u+ .
q −1 p(q − 1) 2  pq − 1 pq − 1 
1 AC pu + q v
Deducem NP = ND + DE + EP = [(p − 1)u + (q − 1) v] ; MP = AP − AM = AP − = −
2 2 2

251
p(q − 1)u + q(p − 1) v pq pq
− = ⋅ [(p − 1)u + (q − 1) v] = NP , ceea ce arată că punctele M, N, P sunt
2 2(pq − 1) 2(pq − 1)
coliniare.

IX.2.
IX.2. Probleme de algebră rezolvate
cu ajutorul calcululu
calculului
lui vectorial
 
1. Are loc inegalitatea lui C.B.S. u ⋅ v ≤ u ⋅ v , (1). Considerăm u(1,1) şi v( 1 − a 2 , 1 − b 2 ) . Aplicând

(1) obŃinem 1 − a 2 + 1 − b 2 ≤ 2 ⋅ ( 1 − a 2 ) 2 + ( 1 − b 2 ) 2 sau 1− a2 + + 1 − b 2 ≤ 2( 2 − a 2 − b 2 ) .

(a + b) 2
Arătăm că 2( 2 − a 2 − b 2 ) < 2 1 − , (2). Se ridică în (2) la pătrat, se fac calculele şi se ajunge la
4
(a – b)2 ≥ 0.
|u⋅v|
2. Aplicăm inegalitatea ≤ 1; a (n 2 , n 3 , n 4 ) , b(n 3 , n 4 , n 2 ) , respectiv x (n +1 2 , n +1 3 , n +1 4 ) ,
|| u || ⋅ || v ||
  n n
y(n +1 3 , n +1 4 , n +1 2 ) ; a ⋅ b = 2 ⋅ 3 + n 3 ⋅ n 4 + n 4 ⋅ n 2 = n 6 + n 8 + n 12 ; || a || ⋅ || b || = ( n 2 ) 2 + (n 3 ) 2 + ( n 4 ) 2 ⋅

⋅ (n 3 ) 2 + (n 4 ) 2 + (n 2 ) 2 = (n 4 + n 9 + n 16 )(n 9 + n 16 + n 4 ) = n
4 + n 9 + n 16 . De aici notând cu S mem-
|| a || ⋅ || b ||
brul stâng, avem: S = > 1 , (1). Procedând la fel cu perechea de vectori x şi y , obŃinem, notând cu D
| a ⋅b |
|x⋅y|
membrul drept: D = < 1 , (2). Din (1) şi (2) obŃinem relaŃia dorită S > D.
|| x || ⋅ || y ||
3. Folosim inegalitatea || v1 + v 2 || ≤ || v1 || + || v 2 || . Fie v1 = ( a , b , c , d ) , v 2 (0, b , c , d ) , v3 (0, 0, c ,
d ), v 4 (0, 0, 0, d ) . De aici || v1 + v 2 + v 3 + v 4 || ≤ || v1 || + || v 2 || + || v 3 || + || v 4 || , adică:

( a ) 2 + ( 2 b ) 2 + (3 c ) 2 + ( 4 d ) 2 ≤ ( a ) 2 + ( b ) 2 + ( c ) 2 + ( d ) 2 + ( b )2 + ( c )2 + ( d )2 +

+ ( c ) 2 + ( d ) 2 + ( d ) 2 , adică inegalitatea cerută: a + 4b + 9c + 16d ≤ a + b + c + d + b + c + d +

+ c+d + d .
4. Fie vectorii u ( x, y, z) şi v(1, 2, 3) . Folosind a ⋅ b ≤ || a || ⋅ || b || , obŃinem (x + 2y + 3z)2 ≤ (x2 + y2 + z2)(12 + 22 +
x y z
+ 32). Observăm că 14 ≤ 14, deci în inegalitatea C.B.S. de mai sus avem cazul de egalitate. Fie = = =m.
1 2 3
Suntem conduşi la m2 = 1, adică m ∈ {–1, 1}. Convine m = 1, de unde S = (1, 2, 3).

252
5. Avem, adunând toate ecuaŃiile, că: (2x + 2)2 + (3y – 2)2 + (z + 3)2 + (t – v A(2x,3y,z,t)
– 1) 2
= 32, (1). Avem OA + OB ≥ AB , (2). Fie OA = a (2x , 3y, z, t );
OB = b(−2, 2, − 3,1) . Din (1) şi (2) avem:
O u
4 2 = (2x + 2) 2 + (3y − 2) 2 + (z + 3) 2 + ( t − 1) 2 ≤ 4x 2 + 9 y 2 + z 2 + t 2 +

+ 4 + 4 + 9 + 1 = 4x 2 + 9 y 2 + z 2 + t 2 + 3 2 . Pe de altă parte, adunând


ultimele 3 relaŃii, găsim: 4x2 + 9y2 + z2 + t2 = 2, de unde obŃinem 4 2 ≤ 4 2 . t
B(–2,2,–3,1)
2 x 3y z t
Urmează că în (2) avem cazul de egalitate, adică: = = = = λ.
2 −2 3 −2
1 1 2 1 
ObŃinem pe rând λ = , de unde S =  , − , − ,1 .
3 3 9 3 
27
6. Scriind relaŃiile lui Viète, găsim: x1 + x2 + x3 + x4 = 6; x1x2 + x1x3 + x1x4 + x2x3 + x2x4 + x3x4 = . Găsim că
2
x12 + x 22 + x 32 + x 24 = ( x1 + x 2 + x 3 + x 4 ) 2 − 2 ( x1x2 + x1x3 + x1x4 + x2x3 + x2x4 + x3x4) = 36 – 27 = 9. Considerăm

vectorii u ( x1 , x 2 , x 3 , x 4 ) , v( y1 , y 2 , y 3 , y 4 ) . Cum u ⋅ v ≤ || u || ⋅ || v || , pentru y1 = y2 = y3 = y4 = 1,


( x12 + x 22 + x 32 + x 24 )(12 + 12 + 12 + 12 ) ≥ (x1 ⋅ 1 + x2 ⋅ 1 + x3 ⋅ 1 + x4 ⋅ 1)2, de unde 4 ⋅ 9 ≥ 36, adică relaŃia de mai
3 81 81
sus are loc cu egalitate şi anume x1 = x2 = x3 = x4 = . Imediat găsim a = − şi b = − .
2 20 16
7. Considerăm vectorii u ( x, y, z) şi v(1, 2, 3) . Avem (| u ⋅ v |) 2 ≤|| u ||2 ⋅ || v ||2 , adică (x + 2y + 3z)2 ≤ (12 + 22 +
a2 x 2 y2 z 2 x 2 + y2 + z 2 a2
+ 32)(x2 + y2 + z2). Deducem x2 + y2 + z2 ≥ . De aici k 2 = 2 = 2 = 2 = ≥ 2 . Deducem că
14 1 2 3 14 14
a a 3a a2
x= ,y = ,z = . Dar min(x2 + y2 + z2) = , de unde a ∈ {–14, 14}.
14 7 14 14
8. Din cea de-a doua ecuaŃie găsim a2 + b2 + c2 + d2 + e2 + f2 = 20. Considerăm vectorii x (1,1,1,1,1) şi y(a , b, c ,
d, e). Avem | x ⋅ y |2 = (a ⋅ 1 + b ⋅ 1 + c ⋅ 1 + d ⋅ 1 + e ⋅ 1)2. De aici (| x ⋅ y |) 2 ≤|| x ||2 ⋅ || y ||2 , adică (a + b + c + d +

+ e)2 ≤ ( 12 + 12 + 12 + 12 + 12 ) 2 ( a 2 + b 2 + c 2 + d 2 + e 2 ) 2 , adică (10 – f)2 ≤ 5(20 – f)2, de unde găsim 0 ≤ f ≤


10 10 4
≤ . Aşadar max f = , care se realizează pentru a = b = c = d = e = .
3 3 3
IX.3.
IX.3. Rezolvarea unor probleme de trigonometrie
trigonometrie
cu metoda vectorială
y
1. i) Considerăm vectorul v = OA , de modul 1, în reperul xOy. Atunci OA = OA' + OB .
2
Dar OA ' = cos x ⋅ i şi analog OB = sin x ⋅ j ; OA = (cos x ⋅ i + sin x ⋅ j) 2 = cos 2 x ⋅ i 2 + B A
2
+ sin 2 x ⋅ j2 + 2 sin x cos x ⋅ i ⋅ j = cos 2 x + sin 2 x , (1). Pe de altă parte OA =| OA | ⋅ x
⋅ | OA | cos 0 = 1 , (2), de unde, folosind (1) şi (2) avem sin x + cos x = 1.
 2 2
O A'

253
ii) Notăm 'AOC = x, 'COD = y. Construim DP ⊥ Ox, DE ⊥ OC. Avem: OP + y D
y
+ PD = OD; OE + ED = OD . Rezultă OP + PD = OE + ED . Aplicăm teorema
C
proiecŃiilor: prOx OP + prOx PD = prOx OE + prOx ED . Cum | OD | = | OC | = | OA | = 1 , x x
O P A
avem cos(x + y) + 0 = cos y ⋅ cos x – sin y ⋅ sin x, de unde egalitatea dorită.

2. i) Considerăm xOy, în care O ≡ C, B ∈ Ox, OA = b, AB = c, CB = a . A A'

Evident a = b + c . Pe de altă parte, a = a ⋅ i = (b + c)i = b ⋅ i + c ⋅ i = bcos C + b c


+ ccos B; E
D
ii) Fie E ∈ (CA). Ridicăm în E perpendiculara pe AC, EP, unde {P} = EP ∩
∩ Ox , {D} = EP ∩ AB. Notăm v = PD, astfel încât | v | = 1 . Avem în acest O a B P
π 
caz: BC = BA + AC . De aici v ⋅ BC = v ⋅ BA + v ⋅ AC , adică a cos − Ĉ  =
 2 
π  a c a b
= c ⋅ cos − Â  , de unde a ⋅ sin('C) = c ⋅ sin('A), adică = . Analog, obŃinem = , de
2  sin A sin C sin A sin B
unde relaŃia cerută.
y
3. i) Avem OM a = i cos a + j sin a ; OM b = i cos b + j sin b . Fie Mb(cosb,sinb)
Ma(cosa,sina) ϕ=a=b
ϕ = a – b. Avem OM a ⋅ OM b = 1 ⋅ 1 ⋅ cos ϕ = cos(a – b), (1). Pe x
O
de altă parte OM a ⋅ OM b = (i cos a + j sin a )(i cos b + j sin b) =
= cos a cos b + sin a sin b , (2). Din (1) şi (2) avem i);

ii) Analog OM a = i cos a + j sin a ; OM b = i cos(2π − b) + j ⋅ y


⋅ sin(2π – b). OM a ⋅ OM b = 1 ⋅ 1 ⋅ cos(2π – (a + b) = cos(a + Mb(cosb,sinb) Ma(cosa,sina)
2π − (a + b)
+ b). Pe de altă parte OM a ⋅ OM b = (i cos a + jsin a )(i cos b − b a x
O
− j sin b) = cos a cos b − sin a sin b , adică ii).

iii) Considerăm vectorii OA = a , OB = b , astfel încât | a | =| b | , 'DOB = 'DOA = x.


A D B
Avem OA ⋅ OB =| OA | ⋅ | OB | cos 2 x = a 2 cos 2 x , (1). Pe de altă parte OA = OD +
+ DA; OB = OD + DB , de unde OA ⋅ OB = (OD + DA)(OD + DB) + DA ⋅ DB . Dar |a| x x |b|
2 2 2 2 2
DA ⋅ DB =| DA | ⋅ | DB | cos 0 − a sin x , (2), iar pe de altă parte, OD = a cos x ,

O
(3). De aici OA ⋅ OB = a 2 (cos 2 x − sin 2 x ) , (4). Din (1) şi (4) avem cos 2x = cos2 x –
– sin2 x.

254
4. i) Notăm a versorul lui OA , b versorul lui OB, c versorul lui OC , unde O y
A
este centrul cercului circumscris ∆ABC. Fie s = a + b + c , de unde s2 ≥ 0, adică
2 2 2 a c C
a + b + c + 2(a ⋅ b + b ⋅ c + c ⋅ a ) ≥ 0 . Avem 1 + 1 + 1 + 2(cos 2C + cos 2A + x
O
3 b
+ cos 2B) ≥ 0, de unde cos 2A + cos 2B + cos 2C ≥ − ;
2
B
2 2 2
ii) Considerăm i, j, k versorii lui BC, CA, AB şi în acest caz (i + j + k ) 2 ≥ 0 , de unde i + j + k + 2(i ⋅ j + i ⋅ k +
3
+ j ⋅ k ) ≥ 0 , adică 3 – 2(cos A + cos B + cos C) ≥ 0, adică cos A + cos B + cos C ≤ .
2
5. Considerăm ∆ABC şi M un punct din planul ∆ABC, astfel încât MA = cos A, A
M
MA + MB
MB = tg B, MC = sin C. Fie E ∈ AB astfel încât AE = EB. Avem ME = .
2 E
 2  2  
 2 MA + MB + 2MA ⋅ MB 2
Avem ME = . De aici 4ME = cos 2 A + tg 2 B + 2MA ⋅
4 B C
2 cos 2 A + tg 2 B
⋅ MB . De aici MA ⋅ MB = 2ME − . Dar ME mediană în ∆AMB, de unde ME2 =
2
2(cos 2 A + tg 2 B) − c 2   c 2 cos 2 A + tg 2 B cos 2 A + tg 2 B − c 2
= , de unde MA ⋅ MB = cos 2 A + tg 2 B − − = , (1).
4 2 2 2
tg 2 B + sin 2 C − a 2 sin 2 C + cos 2 A − b 2
Analog MB ⋅ MC = , (2); MC ⋅ MA = , (3). Adunând membru cu
2 2
a 2 + b2 + c2
membru (1), (2), (3), obŃinem MA ⋅ MB + MB ⋅ MC + MC ⋅ MA = cos 2 A + tg 2 B + sin 2 C − . Pe de
2
2 2 2
altă parte avem: (MA + MB + MC) 2 ≥ 0, adică MA + MB + MC + 2(MA ⋅ MB + MA ⋅ MC + MB ⋅ MC) ≥ 0.
Adică cos2 A + tg2 B + sin2 C + 2(cos2 A + tg2 B + sin2 C) – (a2 + b2 + c2) ≥ 0. Imediat obŃinem inegalitatea
dorită.
6. Considerăm poligonul A1A2...An, cu n laturi, regulat. Fie m('(Ox, A1A2)) = α. y A
3
2π A1
Vorbind de unghiuri orientate, vom avea că m('(Ox, A2A3)) = α + (vezi şi A2
n 2π
4π 2π x
definiŃia rotaŃiei), iar m('(Ox, A3A4)) = α + ş.a.m.d. Pe de altă parte avem: O n A 1
n
A1A 2 + A 2 A 3 + ... + A n A1 = 0 , (1). Considerăm poligonul regulat raportat la An
reperul xOy, de versori i şi j , ai celor două axe. ÎnmulŃind în (1) cu i , obŃinem:
 2π   2(n − 1)π 
i ⋅ A1A 2 + i ⋅ A 2 A 3 + ... + i ⋅ A n A1 = 0 ;| A1A 2 | cos α + | A 2 A 3 | cos α +  + ...+ | A n A1 | cos α + ,
 n   n 
 2π 
(2). Cum | A1A 2 | = | A 2 A 3 | = ... | A n A1 | , găsim în (2) că obligatoriu cos α + cos α +  + ... +
 n 
 2(n − 1)π 
+ cos α + = 0.
 n 

255
y
T
7. Fie cercul de rază OA = 1 = OB, AT = tg a. Considerăm omotetia de centru O şi B T'
OA M
raport k = , adică OA = k ⋅ OP . Evident H(P) = A. Arătăm că H(M) = T, (1). Q
OP
a x
OT OA AT AT
Cum = = n, OT = k ⋅ OM , de unde (1). Pe de altă parte, = = O P A
OM OP MP OQ
OA tga 1 sin a
= , adică = , de unde tg a = . Fie ctg a = BT' şi fie H omo-
OP sin a cos a cos a
OB cos a
tetia de centru O şi raport k = . Absolut analog ctg a = .
OQ sin a

IX.4.
IX.4. AplicaŃiile trigonometriei în
rezolvarea problemelor de geometrie
A
cos
1. Aplicând formula l a =
2bc A
cos şi egalitatea la = lb, rezultă că 2 = a (b + c) , (1). Folosim metoda
b+c 2 B b (a + c)
cos
2
reducerii la absurd. Presupunem A ≠ B.
a ( b + c) A B
Cazul I. A > B. Imediat a > b, de unde > 1 şi folosind (1), obŃinem cos > cos , de unde A < B.
b ( a + c) 2 2
ContradicŃie! Analog A < B. Rămâne singura variantă 'A ≡ 'B. B
2x 2y
2. Considerăm m(AC) = 2x + 2y ≤ 180°. Fie B ∈ (AC). Notăm m(AB) = 2x, m(BC) =
= 2y. Aplicăm în ∆ABC teorema sinusului şi găsim AC = 2R ⋅ sin(x + y), (1). Analog, A C
M
π  *O
scriind teorema sinusurilor în ∆BCD, găsim: BD = sin  + x − y  , (2). Din (1) şi (2)
2 
 π    π π 
obŃinem: AC + BD = 2R sin( x + y) + sin  + x − y  = 4R sin  x +  cos − y  . D
  2    4   4  B1
π π 3π  π π
Cum < +x< , de aici sin  x +  > sin , (3) sau 45° – y < x + 45° < 135° – y,
4 4 4  4 4
π D E
de unde cos(45° – y) > > cos , (4). Din (3) şi (4) AC + BD > 2R. A1 C1
4
C A
3. Aplicăm pe rând teorema sinusurilor în triunghiurile A1DC, B1DE, C1EA,
A C sin D B1D sin E C1E sin A B
D1AB şi E1BC, obŃinem relaŃiile: 1 = ; = ; = ;
A1D sin C B1E sin D C1A sin E E1 D1
D1A sin B E1B sin C
= ; = , care prin înmulŃire membru cu membru, conduc la relaŃia dorită.
D1B sin A E1C sin A
4. Unghiurile triunghiului sunt în progresie aritmetică dacă sunt de forma α – β, α, α + β, cu α + β > 0 şi β ≥ 0.
Deoarece α – β + α + α + β = 180°, suntem conduşi la α = 60°. Din teorema sinusului, laturile triunghiului vor fi
2Rsin(α – β), 2Rsin α, 2Rsin(α + β) şi sin(α – β) ≤ sin α ≤ sin(α + β); i) Dacă laturile sunt în progresie
1
aritmetică, obŃinem sin α = [sin(α – β) + sin(α + β)], de unde α = 60°, β = 0°, adică doar triunghiurile
2

256
echilaterale au proprietatea cerută; ii) Două laturi sunt în progresie geometrică, avem sin2 α = sin(α + β)sin(α –
– β), care conduce la α = 60°, adică numai triunghiul echilateral este singurul care respectă condiŃiile şi în acest
caz.
1a c 1 π
5. Din 2b2 = a2 + c2 şi b2 = a2 + c2 – 2accos B, găsim cos B =  +  ≥ = cos . Cum funcŃia cosinus este
4c a 2 3
π
descrescătoare, avem: 'B ≤ . Am considerat că a2, b2, c2 sunt în progresie aritmetică în această ordine. Analog
3
pentru celelalte două ordonări.
6. Fie a, x, y laturile unui triunghi, a, u, v lungimile laturilor celuilalt triunghi. Din condiŃie avem x + y = u + v.
Scriind aria sub forma lui Heron, rezultă (a – x + y)(a + x – y) = (a – u + v)(a + u – v) sau a2 – (x – y)2 – (u – v)2,
de unde |x – y| = |u – v|. De aici, dacă x – y = u – v, avem x = u, y = v, iar dacă x – y = v – u, avem x = v, y = u.
7. Notăm x = m('OMC), y = m('ONC), OC = a. Aplicând teorema sinusurilor în
a R a 3
∆MOC, obŃinem: = , de unde sin x = . Analog în ∆NOC avem
sin mx sin 2 π 2R M
*
3 C O
A * B
a 3  π *
sin y = . Cum ∆MOC şi ∆NOC au câte un unghi obtuz, rezultă că y ∈  0,  şi
2R  2 N
din sin x = sin y rezultă x = y. Imediat ∆MOC ≡ ∆NOC, de unde MC = NC.
A B C B C A B C
8. a) Trebuie arătat că cos < cos + cos (sau omoloagele). Avem cos + cos − cos = cos + cos −
2 2 2 2 2 2 2 2
B+C B+C B−C B+C B+C π−B π−C
− sin = 2 cos  cos − sin  = 2 cos ⋅ sin ⋅ sin > 0 ; b) Trebuie arătat că
2 4  4 4  4 4 4
sin 2A < sin 2B + sin 2C (sau analoagele). Avem imediat sin 2B + sin 2C – sin 2A = 4sin A ⋅ cos B ⋅ cos C > 0.
9. Notăm MA = x, MB = y, MC = z iar cu α = m('MBC), β = m('CMA), γ = A
3
= m('AMB). Vom arăta că cos α + cos β + cos γ ≥ − , (1). Într-adevăr,
2
x
3 α+β α −β 3 γ
cos α + cos β + cos γ + = 2 cos cos + cos γ + = 2 cos 2 − 1 −
2 2 2 2 2 γ β
γ α −β 3 1  γ γ α −β α −β α − β y αM z
− 2 cos cos + = 4 cos2 − 4 cos cos + cos2 + sin2 =
2 2 2 2 2 2 2 2 2  B C
1  
2 2 2
γ α −β 2 α − β y + z − 1
=  2 cos − cos  + sin  ≥ 0 . Transcriem (1) folosind teorema cosinusului: +
2  2 2  2  2 yz
 
x 2 + z 2 − 1 x 2 + y2 − 1 3
+ + ≥ − , de unde x(y2 + z2 – 1) + y(y2 + z2 – 1) + z(x2 + y2 – 1) + 3xyz ≥ 0. În final
2xz 2 xy 2
găsim: (x + y + z)(xy + xz + yz) – (x + y + z) ≥ 0 sau (x + y + z)(xy + yz + zx – 1) ≥ 0, inegalitate ce ne conduce
la concluzia xy + yz + xz ≥ 1.
10. Parcurgem o serie de egalităŃi cunoscute! În primul rând notăm α = m('GAB), β = m('GBC), γ = m('GCA).
c a 2 sin C − sin A cos B 2 sin(A + B) − sin A cos B
Din teorema sinusurilor: = , de unde: ctg α = = =
sin(β + α) 2 sin α sin B sin A sin B sin A
= ctg A + 2tg A. Notăm x = ctg A, y = ctg B, z = ctg C, de unde ctg α = y + 2x şi analoagele: ctg β = x + 2z,
ctg γ = z + 2y. Din α + β + γ = π, găsim identitatea ctg α + ctg β + ctg γ = ctg α ⋅ ctg β ⋅ ctg γ sau y + 2x + x +
+ 2z + 2y = (y + 2x)(x + 2z)(z + 2y), (1). Din A + B + C = π deducem că xy + yz + xz = 1, (2), relaŃia (1) se

257
transformă în: 3(x + y + z) = 2(x + y + z)(xy + yz + zx) + 2(xz2 + zy2 + yx2) + 3xyz. łinând cont de (2) obŃinem
x + y + z = 2(xz2 + zy2 + yx2) + 3xyz, (3). ÎnmulŃind (2) cu x + y + z, suntem conduşi la x + y + z = xy + xyz +
+ x2z + xy2 + y2z + xyz + xyz + yz2 + xz2, (4). Din (3) şi (4) x2y + y2z + z2x = x2z + z2y + y2z; x2(y – z) + zy(y –
– z) – z(y – z)(y + z) = 0 sau (y – z)(x – z)(z – x) = 0, de unde concluzia.

IX.5.
IX.5. AplicaŃiile trigonometriei în algebră
1. Evident x = y = z = 1 conduce la rezultat. Reciproc: considerăm relaŃia adevărată C
şi arătăm x = y = z = 1. Pentru aceasta considerăm în cercul C(O, R) înscris în
2π R
∆ABC, în care AC = 1, 'CAB = , AB = x. Imediat, cu teorema cosinusului,
3 A
120°
x +1 *
BC = x 2 + x + 1 . Pe de altă parte, cum BC < AB + AC, deducem 1 < ≤ O
2
x + x +1 α
R
2R sin α + 2R sin(60 − α ) 2 ⋅ 2 sin 30 ⋅ cos(α − 30 ) 2 2 3
≤ = ≤ = (am aplicat B
2R sin 120 
3 3 3
x +1 2 3
torema sinusurilor şi Ńinem cont că cos(α – 30°) ≤ 1, egalitate pentru α = 30°. De aici ≤ , (1),
x + x +1 2 3
egalitatea realizându-se pentru x = 1. Analog pentru y, z, obŃinem inegalităŃile de tip (1), de unde concluzia.
2. Notăm a = sin α + sin β, b = cos α + cos β. NotaŃia are sens, căci din a2 + b2 ≤ 4, găsim cos(α – b) ≤ 1, deci
4 − a 2 − b2 1 − cos(α − β) α −β b 4 − a 2 − b2 cos α + cos β α −β
există α, β ∈ R. ObŃinem = = tg ⋅ = ⋅ tg =
2
a +b 2
1 + cos(α − β) 2 2 2
a +b 2
2 2
2
sin α − sin β a 4 − a 2 − b2 sin α + sin β sin α − sin β
= . În sfârşit: ± = ± = sin α sau sin β. De aici:
2 2 a 2 + b2 2 2
1 − sin 2 α + 1 − sin 2 β = cos α + cos β = b .
f (n + 1) − f (n ) 1
3. RelaŃia o transcriem sub forma: = 2 , (1), ceea ce sugerează notaŃia f(n) = tg xn. Din (1)
1 + f (n + 1)f (n ) 2n
1 1 1
găsim tg(xn+1 – xn) = 2
, de unde xn+1 – xn = arctg 2 + knπ, kn ∈Z, (2). Pe de altă parte: arctg 2 =
2n 2n 2n
(2a + 1) − (2n − 1)
= arctg = arctg(2n + 1) − arctg(2n − 1) , (3). Din (2) şi (3) găsim xn+1 – xn = arctg(2n + 1) –
1 + (2n + 1)(2n − 1)
– arctg(2n – 1) + knπ, kn ∈ Z, (4). În final, dând valori lui n şi adunând în (4) membru cu membru, xn = arct(2n –
 π
– 1) + x1 – arct 1 + pnπ, pn ∈ Z, de unde tg xn = tg[arctg(2n – 1) + x1 – arct 1 + pnπ] = tg arctg(2n − 1) − +
 4
n −1
+ f (1)
 n −1   n −1 
+ arctgf (1) + p n π] = tg arctg + arctgf (1) + p n π = tg arctg + arctgf (1) = n , unde f(1) ≠
 n   n  1 − n − 1 f (1)
n

258
n −1
f (1) +
n n , (∀) n ∈ N – {0, 1}.
≠ , n ∈ N – {1}. Rezultatul căutat va fi f (n ) =
n −1 n −1
1 − f (1)
n
4. Identitatea ctg α ⋅ ctg β + ctg β ⋅ ctg γ + ctg γ ⋅ ctg α = 1, (1) dacă α + β + γ = π, sugerează substituŃiile x =
= ctg α, y = ctg β, z = ctg γ. Identitatea (1) este echivalentă cu tg α + tg β + tg γ(1 – tg α ⋅ tg β) = 0. Dacă tg α +
1 1 − tg α ⋅ tg β
+ tg β ≠ 0, tg γ ≠ 0, obŃinem = sau ctg γ + ctg(α + β) = 0, de unde α + β + γ = π. Avem f(α) =
tgγ tg α + tg β
α α β β γ γ
= 1 + ctg 2 α − ctg α = tg , de unde relaŃia dorită: f(x)f(y) + f(y)f(z) + f(z)f(x) = tg tg + tg tg + tg ⋅
2 2 2 2 2 2
α
⋅ tg =1.
2
5. Fie y = f(x). Evident y > 0. De aici putem studia imaginea funcŃiei f2(x), deducând Im f(x). Substituim x =
3 + 2 | sin ϕ | 1 4t 2 + 12 | t | +9
= cos ϕ , ϕ ∈ [0, 2π], y = , de unde y2 = ⋅ , unde t = sin ϕ, (1). Fie t ≥ 0,
ϕ 2 | t | +1
2 cos + 2 | sin ϕ |
2
1 1
sin ϕ > 0, t ∈ [0, 1], ϕ ∈ [0, π]. Atunci (1) devine y2 = 2(t + 1) + + 2 ≥ 4 , cu egalitate pentru t = − ∉
2( t + 1) 2
13 33  13 33 
∉ [0, 1]. Deci, dacă t ∈ [0, 1], ≤ y2 ≤ , de unde y ∈  ,  , (2). Analog, pentru ϕ ∈ [π, 2π], t ≤ 0,
2 4  2 2 
 26 33   26 33 
t ∈ [–1, 0] găsim y ∈  ,  , (3), de unde Im f =  , .
 2 2   2 2 
 π
6. Evident x = 0 şi x = 1 nu sunt soluŃii. Fie x ∈ (0, 1). Facem substituŃia x = cos ϕ, ϕ ∈  0,  . EcuaŃia se
 2
1
transcrie 8cos α(1 – 2cos2 α)(8cos4 α – 8cos2 α + 1) = , (1). Cum 1 – 2cos2 α = –cos 2α, de unde cos 4α =
4

= 8cos4 α – 8cos2 α + 1 – 8sin α cos αcos 2α cos 4α = sin α care conduce la –sin 8α = sin α sau 2 sin ⋅
2
7α 2kπ π 4 kπ  π
⋅ cos = 0 . SoluŃiile vor fi α1 = , k ∈ Z, α2 = ± + , k ∈ Z. Cum α1, α2 ∈  0,  , găsim α1 ∈
2 9 7 7  2
 2 π 4π   π 3π 
∈  , , α 2 ∈  ,  . În final, card E = 4.
9 9  7 7 
 π
7. Deoarece x, y, z > 0, există α, β, γ ∈  0,  astfel încât x = tg α, y = tg β, z = tg γ. Vom considera inegalitatea
 2
3 3 π
sin α + sin β + sin γ ≤ , egalitatea fiind valabilă pentru α = β = β = , α + β + γ = π. Sistemul se transcrie
2 3
tg α + tg β + tg γ = tgα ⋅ tg β ⋅ tg γ
  π
 3 3 . Cum tg α + tg β + tg γ = tg α ⋅ tg β ⋅ tg γ, (∀) α, β, γ ∈  0,  , de unde α =
sin α + sin β + sin γ =  2
 2

259
π
=β=γ= ,x=y=z= 3.
3
x y z x y y z z x
8. Fie x, y, z ∈ (0, π) şi notăm a = tg , b = tg , c = tg . Evident că are loc tg tg + tg tg + tg tg = 1 .
2 2 2 2 2 2 2 2 2
 a2 b2 c 2  3 3 a2 b2 c2
Transcriem inegalitatea sub forma 3 – 2 + + ≤ sau ≤ + + . Practic
1+ a
2
1 + b 2 1 + c 2  2 4 1 + a 2 1 + b2 1 + c2
x y z 3 x y z 9
suntem conduşi la sin 2 + sin 2 + sin 2 ≥ , echivalentă cu cos 2 + cos 2 + cos 2 ≤ , în final găsind
2 2 2 4 2 2 2 4
3
binecunoscuta cos x + cos y + cos z ≤ .
2
 π π  π π
9. Fie numerele a1, a2, a3, ..., a13 şi notăm ai = tg αi, αi ∈  − ,  . ÎmpărŃim intervalul − ,  în 12 părŃi
 2 2  2 2
congruente. Cum a1, a2, a3, ..., a13 sunt în număr de 13, există două, conform principiului lui Dirichlet, în acelaşi
π
interval: (di, di+1). Fie αi+1 < αi ∈ (di, di+1). Evident 0 ≤ αi+1 – αi ≤ . Pentru αi, αi+1, există ai = tg αi, ai+1 =
12
π 2− 3
= tg αi+1. Notăm x = tg αi, y = tg αi+1. ObŃinem 0 ≤ tg(x – y) ≤ tg = .
12 2+ 3
π
2tg
π π π 12
ObservaŃie: Pentru calculul tg se pleacă de la tg şi se foloseşte tg = .
12 6 6 1 − tg 2 π
12
a+c  π π
10. Din ipoteză b(1 – ac) = a + c şi în ipoteza ac ≠ 1, obŃinem b = . Există x, y ∈  − ,  astfel încât a =
1 − ac  2 2
2 2
= tg x, c = tg y şi evident b = tg(x + y). Expresia se transcrie E(a, b, c) = E(x, y) = 2 − 2 −
tg x + 1 tg ( x + y) + 1
3
− 2
= sin y ⋅ sin |2x + y| + 3cos2 y. Notând t = |sin y|, găsim |E(x, y)| ≤ 2|sin x| ⋅ |sin(2x + y)| + |3cos2 | ≤
tg y + 1

≤ 2|sin x| + 3(1 – sin2 x) = t + 3(1 – t2) = –3t2 + 2t + 3. FuncŃia f(t) = –3t2 + 2t + 3 are maxim; maxim f = − =
4a
10 1
= , egalitatea realizându-se pentru t = , sin c ⋅ sin(2x + y) ⋅ cos2 y > 0, sin(2x + y) = 1. În final: E(x, y) ≤
3 3
10 1 π   π π   1  π π 
≤ , x = ±  − − y  ∈  − ,  , iar y ∈ ± arcsin  − ,  .
3 2 2   4 4  3  6 6 

260
X.1.
X.1. Probleme care folosesc inducŃia matematică
în geometrie
1. Fie P(n) propoziŃia OP1 + OP2 + ... + OPn ≥ 1 . Vom demonstra inductiv. P(1): | OP1 |≥ 1 . Fie n = 2k + 1 şi
presupunem P(2k – 1) adevărată şi că cel mai mare unghi 'PiOPj ca fiind 'P1OP2n+1 (eventual renumerotăm
vectorii ca să avem satisfăcută condiŃia). Avem: OP2 + OP3 + ... + OP2 k ≥ 1 , (1). Dacă:

i) OP1 + OP2 k +1 = 0 , relaŃia devine OP1 + OP2 + ... + OP2 k +1 ≥ 1 ; R


S P2
ii) Dacă OP1 + OP2 k +1 = OR , atunci OR diagonală în rombul OP1RP, unghiurile
P2k+1 P1
'ROP1, 'ROP2k+1 sunt ascuŃite. Fie OS = OP2 + OP3 + ... + OP2 k . Cum S este în
interiorul unghiului 'P1OP2k–1, atunci S va fi în unul dintre unghiurile 'ROP1 sau O
π
'ROP2k+1 deci m('ROS) < . Fie T vârful paralelogramului ORTS. Avem OT > OS, de unde OT ≥ 1. Dar
2
OT = OP2 + ... + OP2 k +1 , de unde OP1 + OP2 + ... + OP2 k +1 ≥ 1 .
2. Pentru n = 1, a1 = 3 există un triunghi şi toate laturile le colorăm într-o singură culoare. Să presupunem
afirmaŃia adevărată pentru n ≤ k – 1 şi să o demonstrăm pentru n = k, care este legat de celelalte ak – 1 din pasul
k (a k −1 − 1) + 1
anterior, astfel: ak – 1 = k(ak–1 – 1) + 1 cu segmente de k culori. Deoarece > a k −1 − 1 , rezultă că
k
cel puŃin ak–1 dintre aceste segmente are aceeaşi culoare c1. Dacă unul din segmentele ce unesc aceste ak–1 puncte
are culoarea c1, atunci s-a format un triunghi monocolor de culoare c1, dacă nu, atunci cele ak–1 puncte sunt unite
prin segmente de celelalte k – 1 culori. Conform ipotezei inductive se formează un triunghi monocolor.
3. Pentru n = 2 propoziŃia este evident adevărată. Presupunem propoziŃia adevărată pentru k ≥ 2, prin urmare
segmentele a1, a2, ..., an au un punct comun P. Demonstrăm că există un punct omun segmentelor a1, a2, ..., ak,
ak+1, unde ak+1 este un segment care are un punct comun cu fiecare din primele k segmente. Dacă M şi N sunt
capetele segmentului ak+1, atunci avem situaŃiile lui P faŃă de M şi N:
i) Dacă P ∈ [MN] atunci P este comun tuturor segmentelor a1, a2, ..., ak+1;
ii) Dacă ordinea este P, M, N, atunci punctul comun tuturor segmentelor a1, a2, ..., ak, ak+1 este M. Într-adevăr
segmentele a1, a2, ..., ak conŃin pe P şi conform ipotezei inductive, a1, a2, ..., ak–1, ak+1 au un punct comun Q ∈
∈ [MN]. Analog a1, a2, ..., ak–2, ak, ak+1 au un punct comun R ∈ [MN]. Deci toate segmentele a1, a2, ..., ak+1 au în
comun punctul M. Cazul N între P şi M se tratează analog.
4. Demonstrăm inductiv. Pentru n = 3, evident. Fie n + 1 puncte necoliniare. Putem arăta că există o dreaptă care
să treacă numai prin două puncte ale mulŃimii. Le notăm acele puncte cu Pn, respectiv Pn+1. Conform pasului
inductiv există n drepte determinate de punctele P1, P2, ..., Pn. La aceasta mai adăugăm dreapta determinată de Pn
şi Pn+1, care nu coincide cu niciuna din cele n drepte, din felul cum au fost alese punctele Pn şi Pn+1.
5. Pentru n = 1, dreapta determinată de cele 2 puncte realizează împărŃirea cerută a
poligonului în 2 poligoane convexe. Presupunem proprietatea adevărată pentru n – 1 şi
fie 2n puncte în interiorul poligonului, n ≥ 2. Fixăm o dreaptă d care nu intersectează
poligonul. Fie A punctul cel mai apropiat de dreapta d şi B punctul ales din cele 2n – 1 d α
puncte, pentru care unghiul format de dreapta AB cu dreapta d măsurat în sens A B
trigonometric este minim. Dreapta AB împarte poligonul în două poligoane conveze
P1 şi P2. Poligonul P1 nu conŃine niciun punct din cele 2 puncte, iar P2 conŃine
în interior cel mult 2n – 2 puncte. Aplicând ipoteza de inducŃie, deducem că poligonul

261
iniŃial se împarte în n + 1 poligoane conveze, pe ale căror laturi se află toate cele 2n puncte.
6. Pentru prima etapă a inducŃiei luăm n = 1, deci a1 = 3. Avem 3 puncte şi o singură culoare, n = 1, cu care
colorăm laturile triunghiului obŃinând triunghiul monocolor. Pentru pasul inductiv presupunem proprietatea
adevărată pentru an–1 puncte. Deducem o relaŃie de recurenŃă între an şi an–1 pentru a demonstra proprietatea
 1 1 1  1 1 
pentru an puncte. ObŃinem an = 1 + n(n – 1)! 1 + + ... + +  = 1 + n (n − 1)!1 + + ... + =1 +
 1! ( n − 1)! n!   1! ( n − 1)! 
+ n[an–1 – 1] + 1 = 1 + [n(an–1 – 1) + 1]. Luăm punctul care se găseşte în afara parantezei drepte şi-l unim cu
celelalte din paranteze cu segmente de n culori. Dacă împărŃim numărul din paranteza dreaptă cu n, obŃinem un
număr mai mare decât an–1 – 1. Prin urmare, cu cel puŃin an–1 puncte, punctul considerat este unit cu segmente de
aceeaşi culoare. Dacă în aceste an–1 puncte ar fi un segment de culoare i, atunci problema este rezolvată. Dacă nu
toate, cele an–1 puncte sunt colorate în n – 1 culori, în acest caz folosim ipoteza inductivă.
7. Fie n = 1. F1 = 2, care ne spune că o dreaptă a împărŃit planul în două semiplane. Presupunem că n drepte,
n2 + n + 2
n ≥ 1, două câte două secante şi trei câte trei neconcurente, împart planul în Fn = părŃi. Considerăm în
2
plan n + 1 drepte în poziŃia generală. Primele n din cele n + 1 împart planul în Fn părŃi. A n + 1-a dreaptă pe care
o notăm d, intersectează o dată cele n drepte, conform ipotezei. Cele n puncte distincte de pe dreapta d determină
pe aceasta, n – 1 segmente şi 2 semidrepte. Astfel dreapta d intersectează n + 1 părŃi din Fn părŃi câte erau, deci
n 2 + 3n + 4 (n + 1) 2 + (n + 1) + 2
numărul părŃilor creşte cu n + 1. Urmează că Fn +1 = Fn + n + 1 = = .
2 2

X.2.
X.2. Probleme de geometrie combinatorică
1. DistanŃa între oricare două vârfuri este evident mai mare sau egală cu 1, deci dacă două vârfuri sunt de aceeaşi
culoare, am terminat. Dacă cele 6 vârfuri sunt de culori diferite, atunci oricum am colora centrul cercului,
distanŃa de la el la vârful de aceeaşi culoare este 1. A
2. Presupunem prin absurd că pentru colorare, orice două puncte de aceeaşi culoare se c1 c3
află la distanŃă mai mică sau egală cu 1,7 avem o colorare ca în figura alăturată. Pentru
orice distanŃă ρ ∈ (0, 1,7) pe arcul MN există două puncte de culoare între care distanŃa
ρ = d = 1,7 < 3 . Absurd! Cu aceasta demonstraŃia este încheiată. B C
c2
3. Considerăm reŃeaua laticeală. În această dreaptă, pe dreapta y = 0 avem o infinitate de noduri, dintre care o
infinitate în aceeaşi mulŃime a partiŃiei C1. Considerăm nodurile de deasupra lor situate pe dreapta y = 1. Dacă
două dintre ele ar fi în C1, s-a format un dreptunghi în C1, dacă nu, rămânem cu o infinitate de puncte pe dreapta
y = 1, aflată în C2, deasupra unor puncte de pe dreapta y = 0 din C1. Considerăm nodurile de deasupra acestora
pe dreapta y = 2, cu o infinitate în C3. RaŃionând inductiv, ajungem pe dreapta y = n cu o infinitate de puncte în
una din mulŃimile C1, C2, ..., Cn. Dacă aceasta este Ck, se formează dreptunghi cu vârfurile în Ck (două noduri pe
dreapta y = n şi două noduri pe dreapta y = k – 1).
4. Ne imaginăm că am construit două cercuri în care pe circumferinŃă am construit 4n puncte: 2n puncte roşii, 2n
puncte negre. Punem un cerc peste celălalt, unul rămânând fix. Numărăm când se suprapune un punct roşu peste
unul negru. Este suficient să arătăm că există o rotaŃie la care se produc n + 1 asemenea „suprapuneri”. Avem
4n – 1 rotaŃii şi fiecare punct negru ajunge peste fiecare punct roşu, în total avem 4n2 = 2n × 2n suprapuneri.
4n 2
Deoarece > n , există o rotaŃie la care se produc n + 1 suprapuneri.
4n − 1

262
5. Alegem trei puncte A, B, C de culorile a, b, c. Dacă ele sunt coliniare, s-a terminat. Dacă nu, ele sunt vârfurile
unui triunghi şi presupunem că toate punctele dreptei AB sunt de culori a şi b, ale dreptei BC de culorile b şi c,
ale dreptei AC, de culorile a şi c. Fie D un punct din plan, de culoarea d (necoliniar cu AB, BC sau CA). Una din
dreptele DA, DB sau DC taie dreapta BC, AC sau respectiv AB şi atunci pe ea se află puncte de trei culori (de
exemplu dacă AD ∩ BC = {E}, atunci A are culoarea a, D are culoarea d şi C are culoarea b sau c).
6. Fie cele şase culori 1, 2, 3, 4, 5, 6. Aşezăm cubul pe o masă astfel încât faŃa 1 să se afle jos. Considerăm faŃa 2.
Dacă aceasta se află sus, atunci putem roti cubul după o axă verticală până când faŃa 3 se află în faŃă. Acum
cubul este fixat. Rămân 3! = 6 moduri de a completa colorarea. Acum să presupunem că faŃa 2 este vecină cu
faŃa 1. Atunci rotim cubul până 2 vine în faŃă. Din nou am fixat cubul, iar colorarea poate fi completată în 4! =
= 24 feluri. În total avem 6 + 24 = 30 colorări distincte ale cubului cu şase culori.
2n (2n − 3)
7. Un poligon convex cu 2n vârfuri are = n (2n − 3) diagonale. Numărul diagonalelor paralele cu o
2
anumită latură este cel mult n – 2. De aici numărul diagonalelor paralele cu vreo latură este 2n(n – 2). Deoarece
2n(n – 2) < n(2n – 3), una din diagonale nu este paralelă cu niciuna din laturi.

X.3. Probleme de extrem


BM CN AP A
1. Notăm BC = a, AC = b, AB = c, de asemenea = k1 , = k2 , = k3.
MC NA PB
k1 1 k2 1 R P N Q
Imediat BM = ⋅ a , MC = ⋅a ; CN = ⋅ b, NA = ⋅ b , AP = * *
k1 + 1 k1 + 1 1+ k2 k2 +1
k3 1 *
= ⋅ c, PB = ⋅ b . Aplicând relaŃia lui Stewart în ∆ABC, obŃinem: B C
1 + k3 1 + k3 M
T
b 2 k1 (k1 + 1) + c 2 (k1 + 1) − a 2 k1 c 2 k 2 (k 2 + 1) + a 2 (k 2 + 1) − b 2 k 2
AM 2 = ; BN 2
= ;
(k1 + 1) 2 (k 2 + 1) 2
a 2 k 3 (k 3 + 1) + b 2 (k 3 + 1) − c 2 k 3
AP 2 = . Calculăm apoi puterea punctelor M, N şi P faŃă de cerc şi obŃinem:
(k 3 + 1) 2
BM ⋅ MC AN ⋅ NC
AM ⋅ MT = BM ⋅ MC, BN ⋅ NQ = AN ⋅ NC, CP ⋅ PR = AP ⋅ PB, de unde MT = , NQ = ,
AM BN
AP ⋅ PB AM AM AM 2 b 2 k1 (k1 + 1) + c 2 (k1 + 1) BN
PR = . De aici = = = − 1 . Analog: =
CP MT BM ⋅ MC BM ⋅ MC a 2 k1 NQ
AM
c 2 k 2 (k 2 + 1) + a 2 (k 2 + 1) CP a 2 k 3 (k 3 + 1) + b 2 (k 3 + 1) AM BN CP b 2
= − 1 ; = − 1 . În final, + + = (k1 + 1) +
b2k 2 PR c2k 3 MT NQ PR a 2
c2 a2 c 2 k1 + 1 a 2 k 2 + 1 b 2 k 3 + 1
+ ( k 2 + 1) + ( k 3 + 1) + ⋅ + 2⋅ + 2⋅ − 3 . Aplicând inegalitatea mediilor, obŃinem:
b2 c2 a 2 k1 b k2 c k3

AM BN CP b2 c2 a2 c2 k + 1 a 2 k 2 + 1 b2 k3 + 1
+ + ≥ −3 + 66 2 (k1 + 1) ⋅ 2 (k 2 + 1) ⋅ 2 (k 3 + 1) ⋅ 2 ⋅ 1 ⋅ ⋅ ⋅ 2⋅ = –3 +
MT NQ PR a b c a k1 b 2 k 2 c k3
(k1 + 1) 2 (k 2 + 1) 2 (k 3 + 1) 2 2
+ 66 ⋅ ⋅ ≥ −3 + 66 4 ⋅ 4 ⋅ 4 = 9 . (S-a aplicat inegalitatea pentru x > 0, ( x + 1) ≥ 4).
k1 k2 k3 x

263
~ ~ ~
b) Fie T0, Q0, R0 mijloacele arcelor BC, AC, respectiv AB. Notăm S1 = σ(BT0 C), S2 = σ(CQ 0 A ), S3 = σ(AR 0 B) .
BT ⋅ TC sin(BT̂C) BT ⋅ TC sin  2S
Plecăm de la observaŃia că S = σ(BTC) = = . De aici BT ⋅ TC = =
2 2 sin A
= constant. Valoarea maximă a produsului constant este când factorii sunt egali. Ori acest lucru conduce la BT0 =
= T0C, unde T0 este mijlocul arcului BC. În concluzie acest lucru se realizează când AM este bisectoarea
1 1 1 1 1 1
unghiului A. Analog pentru celelalte triunghiuri. Avem deci concluzia: + + ≥~ +~ +~ .
S1 S2 S3 S1 S2 S3
C
2. Fie p semiperimetrul patrulaterului, S aria sa şi α măsura
d d sin α d1d 2
unghiului dintre diagonale. Avem S = 1 2 ≤ şi S = pr.
2 2
dd d2
Cum p ≥ 4r, avem pr ≥ 4r2 şi din S ≤ 1 2 avem că d1d2 ≥ 8r2. D B
2 d1
Urmează că valoarea minimă a produsului d1d2 este 8r2.

3. Avem a ⋅ MA + b MB + cMC = (a + b + c)MI , implică, aplicând A


modulul: 2p ⋅ MI < a ⋅ MA + b ⋅ MB + c ⋅ MC. Dacă M este pe
cercul circumscris triunghiului dar nu aparŃine mulŃimii {A, B, C}, C
utilizând teorema lui Ptolemeu, obŃinem pMI < aMA sau pMI <
< bMB sau pMI < cMC, deci pMI < max{aMA, bMB, cMC}.
D
4. Avem MA + MC ≥ AC, iar MB + MD ≥ BD, (1). Atunci MA +
+ MB + MC + MD ≥ ≥ AC + BD = constant. Urmează că minimul M
sumei MA + MB + MC + MD se realizează dacă M este punctul de
intersecŃie al diagonalelor pentru care suma este minimă.
A
B

5. Construim mijloacele laturilor patrulaterului. Fie deci S mijlocul


lui AD, Q mijlocul lui BC. Aplicând teorema medianei în ∆AMD şi A
∆MBC, obŃinem: AM2 + MD2 + MB2 + MC2 = 2(SM2 + SQ2) + P
2 2 B
AD + BC
+ , (1). Membrul drept din (1) este minim dacă SM2 +
2 S
+ SQ2 este minimă. Fie T mijlocul lui SQ. Avem 4TM2 = 2(SM2 + T
*
+ MQ2) – SQ2. Evident că TM este minimă când TM = 0, adică Q
T ≡ M. Analog obŃinem o relaŃie similară luând P mijlocul lui AB, R *K
mijlocul lui CD şi M ≡ K, unde K este mijlocul segmentului PR.
M
Cum PQRS este paralelogram, urmează că K ≡ T ≡ M când M este D
intersecŃia diagonalelor paralelogramului determinat de mijloacele R
C
laturilor sale.

264
6. Construim în exteriorul triunghiului ABC triunghiurile echilaterale ABC1, B1
ACB1, BCA1. Cel puŃin unul din patrulaterele CMBA1, CMAB1, BMAC1 este C1 A
convex. Fie, de exemplu, acesta CMBA1. Aplicând inegalitatea lui Ptolemeu,
rezultă MA1 ≤ MB + MC, de unde AA1 ≤ MA + MA1 ≤ MA + MB + MC. Cum M
AA1 = AT + BT + CT, unde T este punctul lui Torricelli, urmează că M
realizează minimul sumei MA + MB + MC dacă M este punctul lui Torricelli B A
(punctul de intersecŃie al cercurilor circumscrise triunghiurilor C1AB, B1AC,
A1BC.

A1
7. Fie A1 mijlocul lui BC. Scriem relaŃia lui Stewart pentru ∆MAA1 şi punctul
G ∈ AA1: AA1 ⋅ MG2 = MA2 ⋅ GA1 + MA12 ⋅ GA – AG ⋅ GA1 ⋅ AA1. Folosind
2 1 AA1
faptul că AG = AA1 , A1G = AA1 , obŃinem: MG2 = MA2 ⋅ + MA12 ⋅
3 3 3
2 2  MA 2 2 2 
⋅ AA1 − AA13 , adică AA1 ⋅ MG2 = AA1  + MA12 − AA12  , de
3 3  3 3 9  A
M
MA 2 2 2
unde MG 2 = + MA12 − AA12 , (1). Deoarece MA1 este mediană în G
3 3 9
2(MB2 + MC 2 ) − BC 2
∆MBC, găsim MA12 = = , (2). Înlocuind (2) în (1) şi
4 B A1 C
efectuând calculele, ajungem la 3MG = MA + MB2 + MC2 – GA2 – GB2 –
2 2

2 1
–GC2, (3). (Am folosit faptul că AA1 = AG, A1G = AG ). Evident că MG
3 2
minimă dacă M ≡ G. (Am Ńinut cont că GA2 + GB2 + GC2 este constantă).

X.4. Locuri geometrice


1. a) Presupunem AB < AC. Notăm u versorul vectorului AB şi v versorul A
vectorului AC . Construim bisectoarea AD şi mediana AM cu D, M ∈ (BC), N
RB + RC RA + AB + RA + AC 2RA + AB + AC
PB = λ. Avem RM = = = =
2 2 2 P R
Q
PA + QA + AB + AC PB + QC λ u + λ v λ
= = = = (u + v) , (1). Avem deci
2 2 2 2 B D M C
2
că RM = u + v , (2). Aplicând teorema bisectoarei, expresia ei vectorială,
λ
bc b+c
ObŃinem AD = (u + v) , (3), de unde AB = c, AC = b. De aici AD = u + v , (4). Din (2) şi (4) găsim
b+c bc
b+c 2 2bc
AD = RM sau AD = RM , (5), ceea ce arată că AD || RM;
bc λ λ ( b + c)
b) Locul geometric este imediat: este format din paralela prin M la bisectoarea AD, anume segmentul de dreaptă
cuprins pe această paralelă între punctele în care această paralelă taie laturile BC, respectiv AC.

265
2. Fie M' proiecŃia vectorului OM pe direcŃia vectorului OA . De aici OA ⋅ OM = M
= OA ⋅ OM ⋅ cos α = OA ⋅ OM' şi cum, din ipoteză OA ⋅ OM = k , urmează că OA ⋅
⋅ OM = k. Dar, O, A puncte fixe. De aici M' fixat. Atunci M se poate proiecta în M'
α
dacă şi numai dacă M se deplasează pe o perpendiculară pe OA, acesta fiind locul
O M' A
k
geometric căutat. Practic, pe OA construim punctul M' de abscisă (faŃă de 0) .
OM
În această abscisă a punctului M' ducem perpendiculara pe OA. Locul geometric al punctelor M este această
perpendiculară.
3. Fie O mijlocul lui AB. Atunci produsul MA ⋅ MB este puterea punctului M faŃă M
AB2 2
de cercul de diametru AB, adică MA ⋅ MB = ρ(M) = OM – 2
= OM −
4
d 2 (A, B)
– = constant. Cum O este fix, locul geometric al punctului M este cercul
4
A O B
AB2
concentric cu cercul de diametru AB şi cu raza 9+ .
4
4. a) Avem m ⋅ G m A − G m B + G m C = 0 , (1), (∀) m ∈ R*. Deci, pentru m = 1 avem G1C = −BA = AB . Cum A
1
este dreptunghi urmează că G1 ≡ D. Din (1) obŃinem mG1A − G1B + G1C = 0 , de unde AG1 = BC . Cum G1 ≡
m
≡ D, mulŃimea cerută este dreapta AD; b) Avem: MA − MB + MC = MG1 , de unde | MG1 | = a şi din a), cum
G1 ≡ D, avem MD = a. MulŃimea cerută va fi deci cercul C(D, BD). (Am Ńinut cont că BD = a).
5. Fie O centrul pătratului ABCD. Atunci avem: f(M) = 3(MO + MA) + (MO + OB) − 2(MO + OC) − 2(MO +
+ OD) = 3OA + OB – 2OC − 2OD . Cum OC = −OA, OD = −OB , rezultă că f(M) = 5OA + 3OB . Fie E şi F
punctele fixate astfel încât OE = 5OA, OF = 3OB . Fie G al patrulea vârf al dreptunghiului cu laturile (OE) şi
(OF). Atunci f(M) = OG = constant. Avem MA + MB + MC + MD = 4MO . Rezultă deci că 4MO = OG. Cum
a 2 5a 2 3a 2
AO = OB = , OE = , OF = , OG = a 17 , de unde rezultă că M descrie cercul de centru O şi
2 2 2
a 17
rază .
4
6. Fie punctele P, N ∈ AB astfel încât PA = k PB respectiv NA = −k NB . Atunci P şi N aparŃin locului
geometric căutat. Fie acum un punct M1 oarecare în plan. M1A + k M1B = M1N + NA + k (M1 N + NB) = (1 + k) ⋅
⋅ M1 N + NA + k NB = (1 + k )M1N , (1). Pe de altă parte M1A − k M1B = M1P + PA − k (M1P + PB) = (1 – k) M1P ,
(2). Efectuăm produsul relaŃiilor (1) şi (2). Urmează că (M1A + k M1B)(M1A − k M1B) = (1 + k )(1 − k )M1N ⋅ M1P ,
(3). Pentru k ≠ 1, din (3) M1A 2 − k 2 M1B 2 = (1 − k 2 )M1 N ⋅ M1P şi deci M1 este punct al locului geometric căutat
dacă şi numai dacă M1 N ⋅ M1P = 0 , (4), relaŃie ce implică că dreptele M1N şi M1P sunt perpendiculare în M1.
Din această relaŃie avem că ∆PM1N dreptunghic într-un cerc de diametru NP. Prin urmare locul geometric căutat
este cercul de diametru NP.

266
7. Fie E şi F mijloacele laturilor AB respectiv CD şi O mijlocul segmentului EF. C
Atunci MA ⋅ MB = ρ1 (M ) este relaŃie ce reprezintă puterea punctului M faŃă de F
cercul de diametru (AB) şi MC ⋅ MD = ρ 2 (M ) reprezintă puterea punctului M faŃă de D
O M
2
2 AB
cercul de diametru CD. Atunci ρ1 (M) = MA ⋅ MB = ME − , ρ 2 (M ) = MC ⋅
4
CD 2 1 A E B
⋅ MD = MF 2 − . Urmează că MA ⋅ MB + MC ⋅ MD = k = ME 2 + MF 2 − ⋅
4 4
1
⋅ (AB2 + CD 2 ) şi deoarece (AB2 + CD 2 ) = constant, rezultă că ME2 + MF2 = constant. Prin urmare locul
4
geometric al punctului M este un cerc cu centrul în O, mijlocul segmentului EF.

X.5. Probleme de construcŃii geometrice


1. Un cerc C1(O, r) taie Ox, Oy în A, respectiv B. Un cerc y y
C2(U, r) taie Uz în C. Fie cercul C3(A, AB) şi cercul
D B
C4(C, AB). C2(U, r) ∩ C4(C, AB) = {D, D'}. Unghiurile
'DUZ şi 'D'UZ sunt unghiurile congruente cu 'xOy.
Într-adevăr, ∆AOB ≡ ∆CUD, de unde concluzia. Folosind z x
rezultatul anterior vom construi paralela dusă printr-un U C O A
punct exterior unei drepte date. Fie dreapta (d) dată şi
M ∉ d. Luăm N, P ∈ d. Construim ca mai sus MQ astfel
încât 'MNP ≡ 'NMQ şi MQ să fie în semiplanul M Q
delimitat de MN, ce nu conŃine N. d
P N

x
ab x b
2. i) Punem egalitatea x = sub forma = . Consi-
c a c C
derăm unghiul 'xOy, pe Ox luăm punctele B şi C astfel b
încât OB = c, BC = b. Fie A ∈ Oy, astfel încât OA = a. B
Construim prin C, exterior dreptei AB paralela la AB, c
y
CM || AB, unde {M}= CM ∩ Oy. Segmentul AM = x,
valoarea căutată. Într-adevăr, aplicând Thales în ∆OMC O a A x M
faŃă de paralela AB, găsim AM = x;
ii) Fie o dreaptă d pe care luăm punctul A. De o parte şi
alta a punctului A luăm punctele B şi C astfel încât E
AB = a, AC = b. Construim cercul de diametru BC.
Construim în A perpendiculara pe d. Notăm cu E
intersecŃia cercului cu perpendiculara. Segmentul AE va fi d
acel y căutat. Acest lucru este evident dacă aplicăm B a A b C
teorema înălŃimii în ∆BEC dreptunghic în E. Construim
cercul de centru O, rază r, şi d o secantă.

267
3. Se consideră două puncte A şi B pe dreapta d astfel încât AB = a. Dacă a > 2r,
atunci nu există un astfel de cerc. Dacă a = 2r, atunci cercul are diametrul AB, iar
dacă a < 2r, atunci există două cercuri de rază r care conŃin punctele A şi B. Fie P O'
exterior dreptei d. Paralela prin P la dreapta d intersectează unul din aceste cercuri în d
A B
A' şi B'. Transformatele acestuia prin translaŃiile de vectori A ' P , respectiv B' P sunt O
cercuri care satisfac condiŃiile din enunŃ.
P A' B'
4. Se consideră translaŃia t v a planului care transformă cercul C1(O1, r1)
M N
în C'1(O'1, r1), unde O1' = t v (O1 ) . Dacă C'1(O'1, r1) ∩ C2(O2, r2) =
= {N, N'}, atunci punctele M = t v ( N ) ∈ C1(O1, r1) şi M' = − t v ( N ' ) ∈ * * *
O1 O'1 O2
∈ C1(O1, r1) satisfac relaŃiile MN = v, M ' N' = v . Dacă C'1(O'1, r1) ∩ M'
N'
∩ C2(O2, r2) = {N}, atunci t − v ( N ) = M ∈ C1(O1, r1) satisface MN = v .
În fine, dacă C'1(O'1, r1) ∩ C2(O2, r2) = ∅, atunci problema nu are
v
soluŃie.
5. Fie A, B proiecŃiile lui O1 respectiv O2 pe (d), iar t AB translaŃia planului de vector O2
*
M M'
AB . Dacă t AB (O1 ) = O1' , atunci imaginea lui C1(O1, r1) prin t AB este cercul N
* * * N'
C'1(O'1, r1). Dacă C'1(O'1, r1) ∩ C2(O2, r2) = {M', N'}, atunci t BA (M' ) = M ∈ C1(O1, O1* * O'1
r1), t BA ( N ' ) = N ∈ C1(O1, r1), MN || d şi MN = M'N'. Dacă C'1(O'1, r1) ∩ C2(O2, r2) =
= {M}, atunci t BA (M' ) = M ∈ C1(O1, r1), MM' || d şi MM' este tangentă comună A B
d
cercurilor date. Dacă C'1(O'1, r1) ∩ C2(O2, r2) = ∅, atunci problema nu are soluŃie.

6. Presupunem problema rezolvată şi se notează cu E şi F


punctele în care perpendiculara pe (d) din centrul cercului dat, F
intersectează cercul C şi cu M punctul de tangenŃă dintre cercul
O A
căutat şi dreapta (d). Se observă că omotetia cu centrul în A,
care transformă cercul C în cercul căutat, transformă unul din
punctele E sau F în M. Deci M este intersecŃia dintre dreptele F
EA, FA cu (d).
M d M
A
7. Presupunem problema rezolvată. Fie C(O1, r1) o soluŃie, M
punctul de tangenŃă şi H omotetia cu centrul în M care
transformă cercul C(O1, r1) în cercul C(O, r). Se notează B' =
= H(B), B'A' = H(BA), B'C' = H(CB). Atunci B'A' || BA, B'C' || C'
|| BC; cercul C(O, r) este tangent dreptelor B'A' şi B'C', iar B'
{M} = BB' ∩ C(O, r). Punctul B' este intersecŃia tangentelor la O1
cercul C(O, r), paralele cu BA şi BC, deci este determinat. M O
Punctul M de tangenŃă a cercurilor este punctul de intersecŃie al
dreptei BB' cu C(O, r). Prin urmare sunt două soluŃii. B C

A'
268
X.6. Alte tipuri de probleme

1. Avem PA = PM + MA, PB = PM + MB . De aici n ⋅ PA + m ⋅ PB + p ⋅ PC = n ⋅ PM + n ⋅ MA + m ⋅ PM + m ⋅


⋅ MB + p ⋅ PC = (m + n )PM + pPC (Am Ńinut cont că n ⋅ MA = m ⋅ BM) . Aplicând Menelaus în ∆MBC şi
AB MP CN PM p
transversala A–P–N, găsim ⋅ ⋅ = 1 , de unde = . Cum PM şi PC au sensuri opuse, găsim
AM PC NB PC m + n
relaŃia cerută.
2. Dacă ABC echilateral, atunci AP, BQ, CR medianele ∆ABC şi evident AP + BQ + CR = 0 . Reciproc: notăm
cAB + bAC a BC + cBA a CB + bCA
AR = AQ = a, BR = BP = b, CP = CQ = c. De aici, AP = , BQ = , CP =
b+c a+c a+b
2
(a − c)(ac − b ) (a + b)(ab − c 2 )
łinând cont de relaŃia din enunŃ şi că CB = AB − AC , obŃinem: AB + ⋅
(a + b)(b + c)(a + c) (a + b)(b + c)(a + c)
⋅ AC = 0 . Cum AB, AC sunt necoliniari, atunci (a + c)(ac – b2) = 0, respectiv (a + b)(ab – c2) = 0. Din ac = b2,
ab = c2 găsim (b – c)(a + b + c) = 0, de unde b = c şi imediat a = b = c, adică ∆ABC este echilateral.
1 m 1
3. Imediat, AB = m ⋅ AD, CQ = m ⋅ QD, BE = m ⋅ BC şi FQ = FC + FD = FD + (FC − FD) =
m +1 m +1 m +1
1 1 1
= FD + (DF + FC) = FD + DC = FD + FE (punctele C, D, F, E sunt vârfurile unui para-
m +1 m +1 m +1
1 p −1
lelogram). Notăm p ⋅ DM = DE , ME = (p − 1)DM , de unde FM = FE + FD . Cum punctele F, M, Q
p p
1 l l(p − 1)  l(p − 1) 
coliniare, avem FQ = l ⋅ FM , de unde FD + FE = FE + FD . De aici avem că 1 − FD =
m +1 p p  p 
l 1  l(p − 1) l 1 l l 1
=  − FE . Cum D, F, E necoliniare, avem 1 – = − , adică 1 – l + = − , de
 p m +1 p p m +1 p p m +1
1 m+2
unde = l − 1 , adică l = , de unde m + 2 = p, ce conduce la relaŃia cerută.
m +1 m +1

4. Fie D simetricul lui A faŃă de BM, {S} = AD ∩ RM. Evident ∆AMD este isoscel. 'AMS = 180° – 'AMB =
= 180° – (180° – 'BAM – 'MBA) = 'BAM + 'ABM = 30°, de unde 'AMD = 60°, adică ∆AMD echilateral.
Fie {E} = BD ∩ MC. Avem 'MED = 180° – 'CED = 180° – (90° – 'ACE) = 120°. Imediat 'MED + 'DAM =
= 180°, adică patrulaterul AMED inscriptibil. De aici 'DEA = 'DMA = 60°, 'DEC = 'DEA = 60°, deci DE este
bisectoare pentru 'AEC. Dar cum AC ⊥ DE obŃinem AB = BC.
5. Fie M' astfel încât M'A || BC, M'A = AF, 'M'AF = 'ABC. Vom arăta că M' ≡ M. Cum triunghiul M'AF este
AB̂C
isoscel, obŃinem 'FM'A = 90° – . Avem astfel că ∆M'AF ~ ∆FBD, dar cum B, F, A coliniare, atunci şi M',
2
BÂC + AB̂C AĈB
F, D coliniare. Mai mult avem AM' = AF = AE, deci 'AEM = 90° – = . Deci 'KEA =
2 2
= 'M'EA, deci M', K, E coliniare. În concluzie cum M' se află la intersecŃia dintre DF şi CE, avem M ≡ M'.

269
MD A
6. În ∆AMD, 'AMD = 90° avem sin α = , de unde MD = AD ⋅ sin α, (1). În
AD
α α
ND
∆MND, 'NMD = α, 'MND = 90°, avem sin α = , de unde ND = MD ⋅
MD
(1)
⋅ sin α = AD ⋅ sin2 α, (2). Cum 3MD = AD + 2ND, folosind (1) şi (2), găsim M α N
3AD ⋅ sin α = AD + 2AD ⋅ sin α. Notând sin α = x, găsim ecuaŃia 2x – 3x + 1 =
2 2

1 B D C
= 0 cu soluŃiile x1 = 1, x 2 = . De aici 'MAD = 90° sau 'MAD = 30°. Dacă
2
'MAD = 90°, A, B, C coliniare absurd. Urmează că 'BAC = 60°.
BM BM
7. Dacă = 1 ⇔ DM || AE ⇔ AE, DM, FN nu sunt concurente. De aici ≠ 1⇔ A
ME ME
⇔ DM ∩ AE = {T} ≠ ∅. Notăm {P} = TF ∩ EC. Aplicăm teorema lui Menelaus în
triunghiurile ABE respectiv AEC şi transversalele D – M – T, respectiv F – P – T. D F
AT EM BD TA EP CF EM PE
ObŃinem: ⋅ ⋅ = 1 , (1), respectiv ⋅ ⋅ = 1 , (2). De aici = ≠
TE BM AD TE PC FA BM CP E
CP CN B M P C
≠ 1. De aici obligatoriu = , adică AE, DM, FN sunt concurente, pentru P = N ⇔
PE NE
CP CN
⇔ = ≠ 1. T
PE NE
8. a) Fie M şi N proiecŃiile pe AD ale punctelor I2, I2. Cum I1I2 || BC urmează că r1 = r2,
unde r1, r2 reprezintă razele celor două cercuri înscrise în cele două triunghiuri: ABD şi
respectiv ACD. De aici M ≡ N, de unde AD ⊥ I1I2 şi urmează cu ∆ABC isoscel de unde
AB ≡ AC;
A
AB + AC
b) Cum AD mediană, avem AD = , (1). Pe de altă parte, din
2
∆ABD dreptunghic găsim AB = AC = 10. Pe de altă parte, AI1 =
8AB + 10AD
= , de unde, Ńinând cont de (1), obŃinem AI1 =
6 + 8 + 10 M * I2
8AB + 5AB + 5AC 13AB + 5AC 5AB + 13AC I1 * N
= = , (2). Analog AI 2 = , (3).
24 24 24 B D C
De aici, din (2) şi (3) găsim, rezolvând sistemul
13AB + 5AC = 24AI1 13u − 5v − 5u + 13v
 că AB = , respectiv AC = .
5AB + 13AC = 24AI 2 6 6
9. Fie I centrul cercului înscris în ∆ABC. Fie a, b, c, lungimile laturilor ∆ABC, a = A
= BC, b = AC, c = AB iar P ∈ BC, Q ∈ AC, R ∈ AB punctele de contact ale
cercului înscris în ∆ABC. Avem de arătat că PA ∩ CR ∩ BQ = {Γ}. Pentru R Q
Γ I
PB CQ RA
aceasta vom folosi teorema lui Ceva. Avem de arătat că ⋅ ⋅ = −1 (*).
PC AQ RB
Pentru aceasta folosim faptul că tangentele duse dintr-un punct exterior la cerc B P C
sunt congruente, avem că AR = AQ = p – a, BR = BP = p – b, CP = CQ = p – c.

270
p−b p−a p−a
De aici PB = − PC , (1) şi reciprocele: QA = − QC , (2) respectiv RA = − RB , care înlocuite în
p−c p−c p−b
(*) fac relaŃia adevărată, ceea ce folosind reciproca lui Ceva, {Γ} = AP ∩ CR ∩ BQ.
sin 53 − 2 cos 40 sin 13 sin(40 + 13 ) − 2 cos 40  sin 13
10. Avem E = = =
cos 63 cos 63
sin 40 cos 13 + sin 13 cos 40 − 2 cos 40 sin 13 sin 40 cos 13 − cos 40 sin 13 sin( 40 − 13 )
= = = =
cos 63 cos 63 cos 63
sin 27  sin(90 − 63 ) cos 63
= = = = 1.
cos 63 cos 63 cos 63
11. a) Fie BE = k ⋅ BD , k ∈ (0, 1). Dar k ⋅ BD = k ⋅ (BA + BC) = k ⋅ BA + k ⋅ BE ; D C
CE = k ⋅ CD + (k − 1)CB = k ⋅ BA + (k − 1)CB sau cum CE = EC' ; EC' = k ⋅ O
⋅ BA + (1 − k )CB = k ⋅ BA + (k − 1)BC , (1). Dar FC' || AD || BC, urmează că există
A F E
un l ∈ R astfel încât C'F = l ⋅ BC , (2). ObŃinem că BF = BC' + C' F = BC + CC' + B
(1)
G C'
+ C' F = BC + 2EC + C' F = BC + 2[k BA + (k − 1)BC] + l ⋅ BC = 2k BA + (l + 2k − 1) ⋅
( 2)

⋅ BC , (3). Cum BF şi BA coliniari în (3) obligatoriu l + 2k – 1 = 0, de unde l = 1 – 2k, (4). Pe de altă parte,
(1) ( 4)
avem că FE = FC' + C' E = l ⋅ BC + k ⋅ BA + (k − 1)BC = (1 − 2k )BC + k ⋅ BA + (k − 1)BC = k ⋅ BA + k ⋅ CB = k⋅
( 2)

⋅ (CB + BA) = k ⋅ CA . RelaŃia FE = k ⋅ CA , (5) arată că vectorii FE şi CA , coliniari, deci FE || CA;


(1) (3) ( 4)
b) Avem FG = FC' + C' G = l ⋅ CB + FA = l ⋅ CB + (BA − BF) = l ⋅ CB + BA − 2k ⋅ BA = (1 − 2k )CB + (1 − 2k ) ⋅
⋅ BA = (1 − 2k )(CB + BA) = (1 − 2k )CA , adică în final FG = (2k − 1)CA , (6). Din (5) şi (6) rezultă imediat că
punctele E, F, G sunt coliniare.
12. Este o inegalitate, de fapt, algebrică. Notând tg 'B = x, x > 0, obŃinem transcrierea inegalităŃii sub forma
1 1 1
+ ≥ , (1). Inegalitatea (1), după efectuarea calculelor, este echivalentă cu (x – 1)2 ≥ 0, adevărată.
1 + 3x 1 + 3 2
x
Egalitatea se obŃine în cazul triunghiului dreptunghic isoscel cu 'B = 45°, adică tg B = x = 1.
13. Vom folosi următoarea definiŃie: Fie O un punct în planul P şi k ∈ R*. Se numeşte omotetie de centru O şi
raport (putere) k transformarea care face ca fiecărui punct M ∈ P să-i corespundă punctul M' ∈ P astfel încât
punctele O, M, M' să fie coliniare şi OM ' = k OM . A'
2
Rezolvarea este o simplă aplicare a acestei definiŃii. Avem: OA ' = OA şi respectiv
3
2 2 2 A
OB' = OB , de unde A ' B' = OB' − OA' = (OB − OA) = AB . De aici:
3 3 3
2 2
| A ' B' |= | AB |= a .
3 3
O B B'

271
14. Vom demonstra următoarea inegalitate: dacă a, b, c, d ∈ R, atunci:
a 2 + b 2 + c 2 + d 2 ≥ (a + c) 2 + (b + d) 2 , (1). Se face ridicând la pătrat în ambii membri, în final ajungându-se
la inegalitatea adevărată: (ad – bc)2 ≥ 0. Să arătăm cum folosim acest rezultat în rezolvarea problemei noastre.
De fapt rezolvând problema ajungem la interpretarrea geometrică a unui celebru rezultat algebric în limbaj
geometric. y
Considerând în plan un reper ortogonal xOy, fie A(x1, y1), B(x2, y2), A(x1,y1)
 x + x 2 y1 + y 2   x + x 4 y3 + y 4 
C(x3, y3), D(x4, y4). Atunci P 1 ,  ; Q 3 ,  B(x2,y2)
 2 2   2 2  Q
* x
şi inegalitatea de demonstrat |AC| + |BD| ≥ 2|PQ| este echivalentă cu:
D(x4,y4) O *P
( x 3 − x1 ) 2 + ( y 3 − y1 ) 2 + ( x 4 − x 2 ) 2 + ( y 4 − y 2 ) 2 ≥

≥ ( x 3 + x 4 − x1 − x 2 ) 2 + ( y 3 + y 4 − y 2 − y1 ) 2 . Notăm x4 – x2 = a; x3 –
C(x3,y3)
– x1 = c; y4 – y2 = b; y3 – y1 = d, de unde ultima inegalitate este de fapt
transcrierea inegalităŃii (1).
3 sin x − sin 3x
15. a) Din sin 3x = 3sin x – 4sin3 x, cos 3x = 4cos3 x – 3cos x rezultă că sin 3 x = , cos 3 x =
4
3 cos x + cos 3x cos 3x (3 sin x − sin 3x ) sin 3x (3 cos x + cos 3x )
= . De aici, sin 3 x ⋅ cos 3x + cos3 x ⋅ sin 3x = + =
4 4 4
3(sin x ⋅ cos 3x + sin 3x ⋅ cos x ) 3
= = sin 4x ; b) Cum E(x) este independentă, alegând de exemplu x = 0,
4 4
π π π nπ π nπ n
x = , găsim că E (0) = E  , de unde sin n ⋅ cos + cos n ⋅ sin = , (1). Pe de altă parte, avem
8 8 8 8 8 8 4
 nπ nπ π nπ  n π nπ π nπ π π π 3π
 sin cos + cos n sin  ≤ sin ⋅ cos + cos n ⋅ sin ≤ sin + cos = sin + sin =
 8 8 8 8  8 8 8 8 8 8 8 8
 π 3π   3π π 
 +   − 
π π π
= 2 sin  8 8  cos 8 8  = 2 sin cos = sin = 1 (Am folosit că dacă x ∈ (0, 1), xn < x, iar |sin x|,
 2   2  4 4 2
   
   
n
|cos x| ≤ 1). Din (1) avem deci ≤ 1 , adică n ∈ {0, 1, 2, 3, 4}. Deja rezolvarea este o simplă verificare. Se
4
obŃine n ∈ {0, 4}, (2).
π
ObservaŃie. Sigur că putem continua fără a calcula efectiv, observând că E(0) = E  , ce se reduce la
4
π nπ nπ 
sin n  sin + cos  = 0 . De aici imediat se obŃin soluŃiile din (2).
4 4 4 
16. Considerăm sistemul de axe perpendiculare xOy. FaŃă de sistemul ales considerăm mulŃimea
dreptunghiurilor care „acoperă” (conŃin) pe D. Dintre aceste dreptunghiuri îl considerăm pe cel de arie minimă.
Fie MNPQ acel dreptunghi. Notăm cu S aria lui D, iar SMNPQ este aria dreptunghiului MNPQ. Cum, din ipoteză,

272
2
 MN + NP  1
MN + NP = 1, avem următorul şir de inegalităŃi: S < SMNPQ = MN ⋅ NP ≤   = (relaŃie dată de
 2  4
1
inegalitatea dintre media aritmetică şi cea geometrică). Avem deci până acum arătat că S < , (1).
4
Pe de altă parte, pentru fiecare disc Di se proiectează pe axele Ox, respectiv Oy în M2 M3
segmentele MiMi+1, respectiv NiNi+1. Urmează că putem forma două linii M1
poligonale în jurul lui MN. Folosind proprietatea liniei poligonale avem |MM1| +
M2
+ |M1M2| + ... + |M29N| > MN, (1); |MN1| + |N1N2| + ... + |N29N| > NP, (2). Dar
M N
proiecŃiile |MiMi+1| sau |NiNi+1| nu sunt altceva decât diametrele discurilor D1, D2,
y
..., D30. Fie ri, i = 1, 30 raza fiecărui disc. Atunci (1) şi (2) se retranscriu:
2r1 + 2r2 + ... + 2r30 > MN, (3), respectiv după reordonare 2r1 + 2r2 + ... + 2r30 > Mi+1
> NP, (4). Adunând membru cu membru (2) cu (3), obŃinem: r1 + r2 + ... + r30 > *D
i

MN + NP 1
30 30 30 Mi
>
4
= , (5). Pe de altă arte, S >
4 ∑ D = ∑ πr
k =1
k
k =1
2
k =π ∑r
k =1
2
k = π(r12 +
O x
+ r22 + ... + 2
r30 ). Însă, folosind inegalitatea dintre media pătratică şi media
2
r12 + r22 + ... + r30
2
 r + r + ... + r30  2 2
aritmetică, găsim ≥ 1 2  , de unde r1 + r2 + ... +
30  30  M2 M3
( 5)
2 1 2 1
+ r30 ≥ (r1 + r2 + ... + r30 ) > M1
30 480 , (6). Pe de altă parte, avem în final că S > π ⋅
M29
π 1 (6)
1 *
⋅ r12
+ πr22
+ ... + > ⋅ 2
πr30 > . M
3 160 160 N
ObservaŃie. EsenŃial în rezolvarea problemei este faptul că dispunerea în plan a N1 N29
discurilor este astfel făcută încât ele sunt disjuncte oricare două. N2 N3

273
CLASA A X - A

ALGEBRĂ

I. MulŃimi de numere

I.1. MulŃimea numerelor reale

I.1.1. PUTERI ŞI RADICALI

I.1.1.1. OperaŃii. Numere raŃionale şi iraŃionale. Puteri. IdentităŃi.

1. Prin ridicarea la cub găsim 7 − a 2 + 7 + a 2 + 33 49 − 2a 2 ⋅ 2 = 8 , de unde 63 49 − 2a 2 = −6 , care conduce


la 49 – 2a2 = –1 sau a ∈ {–5, 5}; 2. Avem a = 3 4 , b = 4 3 sau a = 12 256 , b = 12 27 . Imediat m = b şi M = a.

Imediat 12 27 < 12 x 2 < 12 256 . Atunci mulŃimea H va fi H = {6, 7, 8, ..., 15}. Imediat S = 105; 3. Avem 2a =
= 5 ⋅ 7, 5b = 2 ⋅ 7, 7c = 2 ⋅ 5 de unde rezultă că 2abc = (5 ⋅ 7)bc = (5b)c ⋅ (7c)b = 2c ⋅ 7c ⋅ 2b ⋅ 5b = 2c+b ⋅ 7c ⋅ 5b = 2b+c ⋅
⋅ 2 ⋅ 5 ⋅ 2 ⋅ 7 = 2b+c+2 ⋅ 35 = 2b+c+2 ⋅ 2a = 2a+b+c+2. De aici abc = a + b + c + 2; 4. Vom arăta că [A] = n. În primul
2k + 1 2 k +1 2
rând: 2 k +1 = 1+ > 1 , (∀) k = 1, n , de unde, prin însumare după k, obŃinem A > n. Conform
2k − 1 2k − 1
2k + 1 2 k +1 2  2 
inegalităŃii mediilor, pentru fiecare k = 1, n , obŃinem 2 k +1 = 1+ = 2 k +1 1
⋅ 1
 ⋅ ...
 ⋅ 11 + <
2k − 1 2k − 1 2k  2k − 1 
2
1 ⋅ 2k + 1 +
2k − 1 = 1 + 2 1 1
< =1+ − . Prin însumare după k, a celor n inegalităŃi
2k + 1 (2k − 1)(2k + 1) 2k − 1 2k + 1
1
obŃinem A < n + 1 – < n + 1 . Avem deci: n < A < n + 1, de unde rezultă că [A] = n; 5. Din 3 a − b =
2n + 1
= 3 b − a avem 3 a + a = 3 b + b , (1). Să considerăm funcŃia f: (0, ∞) → R, f ( x ) = x + 3 x . FuncŃia f este strict
crescătoare pe (0, ∞), deci f este injectivă. Urmează că relaŃia (1) se transcrie sub forma f(a) = f(b), de unde a =
= b. CondiŃia din ipoteză conduce la a + 1 = 33 a 2 (1 − a ) . Folosind relaŃia dintre unegalitatea mediilor găsim
a + a +1− a a +1 1
3
a 2 (1 − a ) = 3 a ⋅ a ⋅ (1 − a ) ≤
= . Egalitatea are loc dacă a = 1 – a, de unde a = b = .
3 3 2
n n (n + 1) n n (2n + 1)
6. a) Imediat S = 1 + 2 + ... + n – = − = ; b) Pentru fiecare radical în parte găsim:
4 2 4 4
4n + 1 1 1 n n (2n + 1) n (2n + 3) n (n + 1) n n (2n + 3)
− n = n + − n = , de unde + = , (1) şi respectiv + = ,
4 4 2 2 4 4 2 4 4

274
(2). Din (1) şi (2) relaŃia. 7. Egalitatea x + y + z + u + v = x + y + 2 z − 2 + u + v se rescrie sub forma
2 2 2 2
 1  1  1  1 2 1
 x −  +  y −  +  u −  +  v −  + ( z − 2 − 1) = 0 , de unde x = y = u = v = şi z = 3.
 2  2  2  2 4
Imediat S = 4.

I.1.1.2. Ordonare. InegalităŃi


ln sin x ln cos x
1. Avem (m + 1) ln cos x = ln sin x, (n + 1) ln sin x = ln cos x , de unde m = −1, n = − 1 . De aici
ln cos x ln sin x
sin 2 x
ln tgx ⋅ ln
ln 2 sin x − ln 2 cos x 2 . Cum sin x ∈ (0, 1), cos x ∈ (0, 1), sin 2x ∈ (0, 1), de unde
m–n= =
ln sin x ln cos x ln cos x ⋅ ln sin x 2
sin 2x  π
π
ln sinx < 0, ln cos x < 0, ln < 0 . Urmează că: a) ln tg x > 0 conduce la x ∈  ,  , de unde m < n;
2 4 2
 π π
b) ln tg x < 0, conduce la x ∈  0,  , de unde m > n; c) ln tg x = 0, conduce la x = , de unde m = n.
 4  4
n
1
1 1 1 1
+ + + ... + n
1−   n
1
= 3S . Calculăm S = + + + ... + n = ⋅   = 1 −   <
2 4 8 2 1 1 1 1 1 2
2. Membrul stâng se scrie E = 3
2 4 8 2 2 1− 1 2
2
< 1. De aici 3S < 3, de unde m = 3. 3. Simplă aplicare a inegalităŃii mediilor:
a+
a + ...
  + a + b+
 b + ...
  + b + c+
 c + ...
 +c
≥ a + b + c a b ⋅ b c ⋅ c a . 4. Avem xk – yk = ( 2k + 2k − 1)( 2k + 1 −
b ori c ori a ori
a+b+c
− 2k ) − ( 2k − 2k − 1)( 2k + 1 + 2k ) = −4k − 2k (2k − 1) < 0 . În plus xk, yk > 0. Cum xk < yk, atunci şi
n n


k =1
xk < ∏y
k =1
k , adică an < bn. 5. Inegalitatea este echivalentă cu ( 3 ) n − n 3 > ( 2 ) n − n 2 . Fie funcŃia

2
f: (1, ∞) → R, f(x) = ( x ) n − n x şi făcând substituŃia 2 n x = y , obŃinem g: (1, ∞) → R, g(y) = y n − y 2 n , care
este strict crescătoare pentru n ≥ 6. Urmează că şi f este strict crescătoare. Aşadar f(3) > f(2) echivalent cu
3n − n
3 > 2 n − n 2 . 6. Avem că x 2 − 2x sin z − 4 cos z + 5 + y 2 − 2 y sin z − 6 cos z + 10 =

= (x − sin z)2 + (2 − cos z)2 + (y − sin z) 2 + (3 − cos z)2 ≥ (x − y) 2 + (5 − 2 cos z) 2 după aplicarea inegalităŃii

lui Minkowski. Pe de altă parte (x − y)2 + (5 − 2cos z)2 ≥ (5 − 2cos z)2 ≥ 5 − 2 cos z ≥ 3 , (∀) z ∈ R. 7. Vom
a −b
a a
demonstra că (∀) a ≥ 0, b > 0, atunci   ≥ 1 . Într-adevăr: i) dacă a ≥ b, rezultă ≥ 1 , de unde evident
b b
a −b a −b
a a a
  ≥ 1 ; ii) dacă a < b urmează că < 1 şi   > 1 . Pe de altă parte, din m2 + n2 ≥ 2mn rezultă
b b b

275
n −m
n
m2 + n2 ≥ 2m + n m m + n ⋅ n m + n = 2m + n m 2 n ⋅ n 2 m ⋅   ≥ 2m + n m 2 n ⋅ n 2 m .
m

I.1.2. LOGARITMI

I.1.2.1. OperaŃii. IdentităŃi cu logaritmi


1. Folosim identitatea a3 + b3 + c3 = (a + b + c)(a2 + b2 + c2 – ab – ac – bc) + 3abc, în care, folosind substituŃia
x y z π
a = lg , b = lg , c = lg , obŃinem identitatea cerută. 2. Se ştie că dacă a + b + c = 0, a, b, c ∈ R –  + kπ ,
y z x 2
x y z
k ∈ Z}, atunci tg a + tg b + tg c = tg a ⋅ tg b ⋅ tg c. Notând a = lg , b = lg , c = lg , obŃinem că a + b + c =
y z x
x y z xyz x lg y
= lg + lg + lg = lg = 0 , ceea ce demonstrează relaŃia. 3. a) Evident E > 0, de unde lg E = lg lg x =
y z x xyz y
a lg b b lg c c lg a
= lg y ⋅ lg x – lg x ⋅ lg y = 0, de unde E = 1; b) Putem scrie P = a lg b − lg c ⋅ b lg c − lg a ⋅ c lg a − lg b =
⋅ ⋅ = 1,
b lg a c lg b a lg c
deoarece fiecare raport este 1, conform a). 4. łinând seama de egalitatea log 1 b = − log a b , obŃinem: E =
a
= log sin x cos x + log cos x sin x − 1 − log tgx cos x − log ctgx sin x − log cos x sin x − log sin x cos x = −1 − log tgx cos x +
1
+ log tgx sin x = log tgx tgx − 1 = 0 , deci E nu depinde de x. 5. RelaŃia din enunŃ este echivalentă cu +
log a (c + b)
1 2
+ = , de unde log a (c − b) + log a (c + b) = 2 . Urmează că log a (c 2 − b 2 ) = 2,
log a (c − b) log a (c − b) ⋅ log a (c + b)
care exprimată astfel: c2 – b2 = a2 este adevărată, conform ipotezei că ∆ABC este dreptunghic în C. 6. Vom arăta
k log k −1 a
k log a b b
că termenii de rang k, k = 2, n din fiecare membru sunt egali, adică a =b , (1). Într-adevăr,

logaritmând în baza a în (1), se obŃine: k log a b = log a b ⋅ k log kb −1 a . Dar log a b ⋅ k log kb −1 a = k log ak b ⋅ log kb −1 a =

= k log a b ⋅ (log a b ⋅ log b a ) k −1 = k log a b . 8. Din egalitate avem n + 1 = a1 ⋅ a2 ⋅ a3 ⋅ ... ⋅ an, de unde luând ak =
k +1 2 3 4 n +1
= , (1), k = 1, n găsim a1 ⋅ a 2 ⋅ a 3 ⋅ ... ⋅ a n = ⋅ ⋅ ⋅ ... ⋅ = n + 1. Prin urmare cum ak ∈ Q ∩ (1, 2],
k 1 2 3 n
alegerea lui ak de forma (1) rezolvă problema. 7. Fie a = log3 30 = log 3 2 + log 3 5 + 1 = x + y + 1 , unde x = log3 2,
log 33 5 y log 3 8
y = log3 5. Analog b = log 30 5 = = , de unde y = ab, iar x = a – ab – 1. În final, log 30 8 = =
log 3 30 a log 3 30
3x 3(a − ab − 1) 3(log 3 30 − log 3 30 ⋅ log 30 3 ⋅ 5 − 1)
= = , de unde log 30 8 = . 9. Folosim următoarea relaŃie:
a a log 3 30
1 1 1  n   n   n 
= = − 1 . De aici  = − n =  .
logk (k + 1) − 1 1
−1 1 − logk (k + 1)  logk (k + 1) − 1 1 − logk +1 k  1 − logk +1 k 
logk +1 k

276
10. Vom nota an = logn(n + 1) + logn+1n şi vom arăta că an ∈ (2, 3), de unde, evident an ∉ Z. Într-adevăr, cum
logn(n + 1) şi logn+1n > 0, de unde aplicând inegalitatea mediilor (aritmetică şi cea geometrică) găsim: an =
= log n (n + 1) + log n +1 n ≥ 2 log n (n + 1) ⋅ log n +1 n = 2 , deci an > 2. Pe de altă parte logn(n + 1) + logn+1n =
1 1
= log n (n + 1) + . Notând logn(n + 1) = t, avem că t > 1. Deci avem de arătat că t + < 3 , (∀) t > 1. Pe
log n (n + 1) t
3− 5 3+ 5 
de altă parte, din t2 – 3t + 1 < 0 găsim t ∈  ,  , (1). Arătăm că (1) este adevărată, (∀) n ≥ 2, n ∈ N.
 2 2 

3− 5
Cum < 1 < t , ceea ce arată că an > 2. Arătăm că an < 3. Asta înseamnă să arătăm pentru început că
2
log n (n + 1) < 2 , adică n + 1 < n2, (∀) n ≥ 2. Dar ultima inegalitate conduce la n2 – n – 1 > 0, care este adevărată
3+ 5
1+ 5 
căci n ∈  
, ∞ ⊂ [2, ∞), adică n ∈ [2, ∞). Cum n < n 2 , am arătat că an < 3.
2
 2 
 

I.1.2.2. Logaritmi. Ordonare. InegalităŃi


1 1
1. Notăm logab = x, logbc = y, logca = z, cu x, y, z > 0, x ⋅ y ⋅ z = 1. Inegalitatea se transcrie: + +
k+x k+y
1 2 k3 − 2
+ ≤ şi efectuând calculele, obŃinem: k 3 ≤ k ( xy + yz + zx ) + 2 , de unde ≤ xy + yz + zx. Cum xy +
k+z k k
k3 − 2
+ yz + zx ≥ 33 ( xyz) 2 = 3 , şi ≤ 3 . Pentru k ∈ (0, 2], rezultă concluzia. 2. Logaritmând în x12010 >
k
> x2x3...x2011, obŃinem lg x12010 > lg(x 2 x 3 ...x 2011 ) = lg x 2 + lg x 3 + ... + lg x 2011 , (1). Cum x1, x2, ..., x2011 ∈ (1, ∞),
lg x1 , lg x 2 , ..., lg x 2011 ∈ (0, ∞), de unde lg x 2 + lg x 3 + ... + lg x 2011 ≥ 20102010 (lg x 2 )(lg x 3 )...(lg x 2011 ) , de unde
lg 2010 x1 > (lg x 2 )(lg x 3 )...(lg x 2011 ) . 3. Avem (5 + 35 ) 2 = 10(6 + 35 ) . Logaritmând relaŃia precedentă,
obŃinem 2 lg(5 + 35 ) = 1 + lg(6 + 35 ) . Cum (x – 1)2 ≥ 0, scris sub forma x2 ≥ 2x – 1, înlocuind x =
= lg(5 + 35 ) obŃinem lg 2 (5 + 35 ) ≥ 2 lg(5 + 35 ) − 1 = lg(6 + 35 ) . 4. Aplicăm inegalitatea C-B-S..
log a b log b c log c a ( log a b + log b c + log c a ) 2
În acest caz găsim: + + ≥ , (2). În (2) aplicând inegalitatea
a+b b+c c+a 2( a + b + c )
dintre media aritmetică şi media geometrică, găsim log a b + log b c + log c a ≥ 36 log a b ⋅ log b c ⋅ log c a = 3.
log a b log b c log c a 9
(Am folosit că loga b ⋅ logb c ⋅ logc a = 1). Revenind în (2) găsim + + ≥ , inega-
a+b b+c c+a 2(a + b + c)
litatea dorită. 5. Pentru orice xn ∈ (1, 2] avem (xn – 1)(xn – 2) ≤ 0, de unde x 2n ≤ 3x n − 2 . ObŃinem atunci imediat
log x 1 (3x 2 − 2) + log x 2 (3x 3 − 2) + ... + log x n (3 x 1 − 2 ) ≥ log x 1 x 22 + log x 2 x 32 + ... + log x n x 12 =
= 2(log x 1 x 2 + log x 2 x 3 + ... + log x n x1 ) ≥ 2n n log x 1 x 2 ⋅ log x 2 x 3 ⋅ ... ⋅ log x n x1 = 2n. 6. Folosind relaŃia din ine-
galitatea dintre media geometrică şi cea aritmetică, găsim:

277
3
 log (1+ a + b) + log3 (1+ b + c) + log3 (1+ c + a) 
log3 (1 + a + b) ⋅ log3 (1 + b + c) ⋅ log3 (1 + c + a ) ≤  3  =
 3 
3
  1+ a + b +1+ b + c +1+ c + a  
3
3  log 3   
 log (1 + a + b)(1 + b + c)(1 + c + a)    3   = 1 . 7. Avem, folosind inega-
= 3  ≤
 3   3 
 
 
2 log k ( k +1)
litatea dintre media aritmetică şi cea geometrică, pentru fiecare termen, că: k < k log k ( k +1) +1 =
n n n n
= k logk k (k +1) < k(k + 1), (1). Atunci, prin sumare în (1) obŃinem ∑k
k =2
2 log k ( k +1)
< ∑ k(k + 1) = ∑ k + ∑ k =
k =2 k =2
2

k =2
2
n (n + 1)(2n + 1) n (n + 1) n (n + 1)(2n + 1) + 3n (n + 1) − 12 (n − 1)( n + 4n + 6)
= + −2= = , (∀) n ∈ N, n ≥ 2.
6 2 6 3
8. Vom folosi inegalitatea (x + 1)2 ≥ 1 + 2x; lg2 5 + lg2 7 = (lg 10 + lg 0,5)2 + (lg 10 + lg 0,7)2 ≥ 1 + 2lg 0,5 + 1 +
+ 2lg 0,7 = 2 + lg 0,25 + lg 0,49 = lg(100 ⋅ 0,25 ⋅ 0,49) = lg 12,25 > lg 12. 9. Vom folosi inegalitatea: (∀) x > 0,
1 1  1 
(∀) n ∈ N*, x n + n > x n −1 + n −1 , (1). Într-adevăr inegalitatea (1) o scriem sub forma (x − 1)  x n −1 − n  ≥ 0,
x x  x 
inegalitate adevărată, (∀) x > 0. Înlocuind în (1) cu loga b, obŃinem inegalitatea de demonstrat. 10. Plecând din
ab + bc + ca a + b + c b c a
inegalitatea mediilor, obŃinem ≥ a b c , (1). Aplicând monotonia funcŃiei logaritmice pentru
a+b+c
ab + bc + ca
a, b, c ≥ 2 şi inegalitatea din (1), se obŃine după însumare membru cu membru că: log a +
a+b+c
ab + bc + ca ab + bc + ca 1
+ log b + log c ≥ [b log a a + c log a b + a log a c + b log b a + b log b b + a log b c +
a+b+c a+b+c a+b+c
1
+a logc c + c logc b + b logc b] = [a + b + c + (a loga c + c logc b + b logb a) + (c log a b + a log b c + b log c a )] , (2).
a +b+c
a log a c + c log c b + b log b a
Aplicând din nou inegalitatea dintre media aritmetică şi media geometrică, avem: ≥
a +b+c
≥ c ⋅ log c b ⋅ log b a = 1 , de unde a log a c + c log c b + b log b a ≥ a + b + c, (3). Revenind în (2) şi
a + b + c log
a
folosind (3) şi la cealaltă expresie din (2), găsim:
ab + bc + ca ab + bc + ca ab + bc + ca 1
log a + log b + log c ≥ [(a + b + c) + (a + b + c) + (a + b + c)] = 3 .
a+b+c a+b+c a+b+c a+b+c

I.1.3. METODE NESTANDARD DE REZOLVARE A PROBLEMELOR

I.1.3.1. Metoda trigonometrică


1. ObŃinem 6 + 1 + cos 2A ≥ 4 + 2cos 2B + 2cos 2C, de unde 2(cos 2B + cos 2C) – cos 2A ≤ 3. În final,
suntem conduşi la cos2 A + 2cos A ⋅ cos (B – C) + 1 ≥ 0, (1). Privind (1) ca un trinom de grad II în variabila cos A,
 2π  3A
găsim că (1) este adevărată dacă ∆ ≤ 0, care este evidentă: cos2(B – C) – 1 ≤ 0. 2. Cum A ∈  0,  , ∈
 3  4

278
 π 3B 3C  π   π
∈  0,  şi analoagele, , ∈  0,  . Cum funcŃia tangentă este convexă pe  0,  funcŃionează Jensen:
 2 4 4  2  2
3A 3B 3C 3A 3B 3C
+ + tg + tg + tg
tg 4 4 4 ≺ 4 4 4 , de unde imediat tg 3A + tg 3B + tg 3C ≥ 3 . 3. Folosind monotonia
3 3 4 4 4
 π
funcŃiei tg pe intervalul  0,  avem (a – b)(tg A – tg B) ≥ 0, inegalitate ce conduce la:
 2
a2 b2  1 1  b2 c2  1 1 
+ ≥ ab +  , (1). Folosind analoagele + ≥ bc +  , (2);
cos A cos B  cos A cos B  cos B cos C  cos B cos C 
c2 a2  1 1 
+ ≥ ac +  , (3). Adunând membru cu membru (1), (2), (3) găsim:
cos C cos A  cos C cos A 
a2 b2 c2 a ( b + c ) b ( a + c ) c( b + a )
+ + ≥ + + .
cos A cos B cos C cos A cos B cos C
S abc a+b+c 27
4. Folosind teorema sinusurilor şi r = , R = , inegalitatea se reduce la ≥ 2 , ce este echivalentă
P 4S abc 4P
a+b+c 3
cu (a + b + c)3 ≥ 27abc, care este de fapt binecunoscuta inegalitate a mediilor: ≥ abc . 5. Folosind
3
a b c 1 a b a 1 2
teorema sinusurilor, inegalitatea revine la 3 + 3 + 3 ≥ 2 , (*). Dar + ≥ 2 , de unde 3 + ≥ 2 , (1).
b c a R b a b ab b
a b c 1 1 1
Analog obŃinem similarele inegalităŃi pe care le adunăm membru cu membru: 3 + 3 + 3 + + + ≥
b c a ab bc ac
 1 1 1  1 1 1 1 1 1
≥ 2 2 + 2 + 2  , (2). Pe de altă parte 2 + 2 + 2 ≥ + + , (3). De aici revenind în (2) şi Ńinând
b c a  a b c ab bc ac
a b c 1 1 1 1 1 1 a+b+c 2p p 1 1
cont de (3) găsim 3 + 3 + 3 ≥ + + . Dar + + = = = = ≥ , din
b c a ab bc ac ab bc ac abc 4RS 2Rpr 2Rr R 2
relaŃia lui Euler. Aceasta încheie soluŃia obŃinând ceea ce am dorit, adică (*). 6. Din 2A + 3B = 180° şi 'A +
a + b 2R sin  + 2R sin B̂
+ 'B = 180° – 'C, deducem că 'B = 2'C – 180° şi 'A = 360° – 3'C. Avem = =
c 2R sin Ĉ
sin  + sin B̂ sin 3Ĉ − sin 2Ĉ 4 + 16 5 a+b 5
= = = –4cos2 C – 2cos C + 1, (1) ≤ = , adică ≤ , sau 4(a + b) ≤ 5c
sin Ĉ sin Ĉ 16 4 c 4

(am folosit că în (1), privită ca funcŃie de gradul 2, avem că maximul ei este − ). 7. Din a > b avem sin A >
4a
> sin B şi cum ha ⋅ 2R = bc, de unde ha = 2Rsin B ⋅ sin C. Deci a demonstra an + h an > b n + h nb revine la sinn A +
+ sinn B ⋅ sinn C ≥ sinn B + sinn C ⋅ sinn A sau (sinn A – sinn B)(1 – sinn C) ≥ 0, inegalitate adevărată. Semnul egal
se realizează pentru sinn C = 1, adică 'C = 90°.

I.1.3.2. Metoda vectorială

1. Considerăm vectorii a ( x 3 , y 3 , z 3 ) , b( x 4 , y 4 , z 4 ) . Atunci (a ⋅ b) ≤| a |2 ⋅ | b |2 , (1), de unde (x7 + y7 + z7)2 ≤

279
x 4 y4
≤ (x6 + y6 + z6)(x8 + y8 + z8), adică 9 ≤ 9. Sistemul este compatibil pentru a şi b coliniari. Rezultă: = =
x 3 y3
z4
= = k. ObŃinem x = 1, y = 1, z = 1. y
z3
a−b
2. E(t) = t 2 − 24 t + 169 − t 2 + 6t + 25 = ( t − 12) 2 + 25 − (t + 3)2 + 16 . a
Considerăm vectorii a = ( t − 12, 5), b = ( t + 3, 4) . În inegalitatea | a | − | b | ≤| a − b |
b
O x
obŃinem ( t − 12) 2 + 25 − ( t + 3) 2 + 16 ≤ (t − 12 − t − 3) 2 + (5 − 4)2 = 226 .

 x ( x + y) + y( y + z ) = 0

3. Sistemul se rescrie sub forma: x ( x + 1) + y(2z + 1) = 0 . Consisderăm vectorii a ( x, y) ,
 2 2 2 2
4( x + y) + 4( y + z) = ( x + 1) + (2z + 1)
2 2
b( x + y, y + z), c = ( x + 1, 2z + 1) . Sistemul se rescrie sub forma a − b = 0; a − c = 0; 4b = c . Se analizează
1
a = 0 sau a ≠ 0 . Se obŃine singura soluŃie acceptabilă x – y = 0, z = − . 4. Folosim C kn = C kn −1 + C kn −−11 . Alegem
2
vectorii a ( C kn −1 , C kn ) , b( C kn , C kn +1 ) şi că | a ⋅ b |≤| a | ⋅ | b | , (1). De aici | a ⋅ b |= C kn −1 ⋅ C kn + C kn ⋅ C kn +1 ,

(2), iar | a | ⋅ | b |= C kn −1 + C kn ⋅ C kn +1 + C kn ++11 < C kn ++11 , (3). Din (1), (2) şi (3) se obŃine inegalitatea dorită.
5. Considerăm vectorii x = (a1 , a 2 ); y = (a 3 , a 4 ); z = (a 5 , a 6 ); t = (a 7 , a 8 ) . Cel puŃin unul din
π y
unghiurile ' ( x, y) , ' ( y, t ) , ' ( y, z) , ' (z, t ) este mai mic sau egal cu . Exprimând produsul z
2
scalar în două moduri, se obŃine condiŃia dorită. (Exemplu: x ⋅ y = a1a 3 + a 2 a 4 , x ⋅ y =| x | ⋅ O x
⋅ | y | ⋅ cos'(x, y) şi analoagele). t

6. Alegem vectorii v1 = (1,1), v 2 = ( 4a1 + 1, 4a 2 + 1) . Cum v1 ⋅ v 2 ≤| v1 | ⋅ | v 2 | obŃinem: 4a1 + 1 + 4a 2 + 1 ≤

≤ 12 + 12 ⋅ ( 4a1 + 1) 2 + ( 4a 2 + 1) 2 = 2 4a1 + 4a 2 + 2 = 2 ⋅ 4(a1 + a 2 ) + 2 = 6 . 7. Prima ecuaŃie este


2
 1  1 
echivalentă cu (x – 1)2 +  y −  + (z − 2) 2 − 1 = 0 şi reprezintă ecuaŃia unei sfere de centru 1, , 2  şi rază r = 1.
 2  2 
 1
 x − 1 y − 2 z − 2
EcuaŃia (2) reprezintă ecuaŃia unui plan de vector (3, –4, 12). ObŃinem:  = = , (1). Deoarece
 3 −4 12
3x − 4 y − 12z = 12
1
3 ⋅1 − 4 − + 12 − 2 − 12
2
distanŃa de la centrul sferei la planul 3x – 4y – 12z = 12 este d = = 1 = r şi soluŃia sistemului
32 + 4 2 + 12 2
10 21 14
(1) va fi reprezentată de punctul de contact (planul tangent). Rezolvând (1) obŃinem x = ,y = ,z = .
3 26 13

280
I.1.3.3. Alte metode
1. a) Arătăm că (∀) n ∈ N, n3 = 9k + r, (1), r ∈ {0, 1, 8}. Din teorema împărŃirii cu rest, r ∈ {0, 1, 2, 3, 4, 5, 6, 7,
8}. Pe de altă parte orice număr natural este de forma n = 9l + x, x ∈ {0, 1, 2, ..., 8}. Din (1), (9l + x)3 = 9k + r,
de unde r – x3 ∈ M9. Cum x ∈ {0, 1, 8}, analizăm pe rând fiecare caz parcurgând pe rând x = 0, x = 1, ..., x =
= 8. ObŃinem corespunzător, că r ∈ {0, 1, ..., 8}; b) Fie [3 9n + 1] = p ∈ Z, de unde 9n + 1 ∈ [p3, (p + 1)3], de
unde 9n + 7 < (p + 1)3 + 6. De aici 9n + 7 ∉ {(p + 1)3, (p + 1)3 + 1, ... , (p + 1)3 + 5}. ( Din a) restul împărŃirii
unui cub la 9 este {0, 1, 8}). łinând cont de acest lucru găsim că 9n + 7 ≥ p3 şi 9n + 7 < (p + 1)3, de unde
[3 9n + 7 ] = p , ceea ce indică concluzia [3 9n + 1] = [3 9n + 7 ] . 2. Avem (∀) x ∈ R, n[x] ≤ [nx] ≤ n[x] + n – 1.
În consecinŃă avem [x] + [3x] + [6x] + + [9x] + [18x] + [36x] ≤ [x] + 3[x] + 2 + 6[x] + 5 + 9[x] + 8 + 18[x] + 17 +
+ 36[x] + 35 = 73[x] + 67. Pe de altă parte [x] + [3x] + [6x] + [9x] + [18x] + [36x] ≥ 73[x]. Deci 73[x] ≤ 14305 ≤
≤ 73[x] + 67. Prin urmare 73[x] ≤ 73 ⋅ 195 + 70 ≤ 73[x] + 67, imposibil. Urmează că ecuaŃia nu are soluŃii.
3. Avem a = 8n + 2, b = 9n + 1. Pentru n impar observăm u(b) = 0, deci b se divide cu 10. Pe de altă parte, u(8n) = 8,
dacă n = 4k + 1, care vor fi numerele căutate (Am notat u(x) = ultima cifră a lui x). 4. Notăm [x] = k şi ε = {x}.
ObŃinem x = [x] + {x}, (∀) x ∈ R, adică x = k + ε. De aici 2011x = 2011k + 2011ε. Deci pentru m = [2011ε],
2011ε ∈ [0, 2011). Cum m ∈ {0, 1, 2, ..., 2010}, cu [2011x] = 2011{x} + m. 5. InducŃia este metoda folosită în
demonstrarea proprietăŃii. Pentru k = 1, este adevărată din ipoteză. Presupunem că xk | 2 k − 2 şi vom
x

demonstra că xk+1 | 2 k +1 − 2 . Pentru aceasta fie xk+1 – 1 = 2 xk − 2 = m ⋅ xk, unde m ∈ N, m ≥ 2. Atunci 2 xk +1 − 2 =


x

(m −1)x k
xk
= 2(2 − 1)[2 + 2(m − 2)x k + ... + 1] = M ⋅ xk+1, unde M este un număr natural, ceea ce trebuia demonstrat.
6. Cum numărul din stânga este divizibil cu 9, deci şi numărul din dreapta trebuie să fie divizibil cu 9. Aşadar
suma a + 2 + 9 = 11 + a trebuie să fie multiplu de 9, de unde a = 7. Imediat t = 0.

I.1.4. IDENTITĂłI TRIGONOMETRICE. SUME ŞI PRODUSE. INEGALITĂłI


1 1

1 1 1 1
1. Avem 2 = k k + 1 , de unde, imediat arătăm că arctg 2 = arctg − arctg , de unde
k + k +1 1+ ⋅
1 1 k + k +1 k k +1
k k +1
n

∑ arctg k
1 1 n
= arctg1 − arctg = arctg . 2. Rescriem inegalitatea de demonstrat astfel:
k =1
2
+ k +1 n + 1 n +2

sin 4 x + cos 4 x sin x cos x 3 2 − t2


3 − − < 0 , (∀) x ∈ R, (1). Notând sin 2x = t, atunci (1) se transcrie astfel: −
4 2 8 8
t 3
− − ≤ 0 care este echivalentă cu (t + 1)2 ≥ 0, ceea ce încheie problema. 3. Avem următoarea inegalitate:
4 8
(sin x1 – cos x2)2 + (sin x2 – cos x3)2 + (sin x3 – cos x4)2 + ... + (sin xn – cos x1)2 ≥ 0. De aici obŃinem:
2(sin x1 cos x2 + sin x2cos x3+ ...+ sin xncos xn) ≤ sin2 x1 + cos2 x1 + sin2 x2 +cos2 x2 + ... + sin2 xn + cos2 xn,
n π n n
de unde E ≤ . Cum, pentru x1 =x2 = …=xn = , E = , rezultă că max E = .
2 4 2 2
n n
sin( k + 1) x cos kx − sin kx cos(k + 1) x
∑ ∑
1 1 1
4. Avem Sn = = = ⋅
k =1
2 sin kx ⋅ sin( k + 1) x 2 sin x k =1
sin kx ⋅ sin( k + 1) x 2 sin x
n
sin4 x

1 sin nx
⋅ [ctgkx − ctg(k + 1) x ] = [ctgx − ctg(n + 1)x ] = . 5. Avem echivalenŃele 0 = +
k =1
2 sin x 2 sin 2 x ⋅ sin( n + 1) x a

281
cos 4 x 1 (1 − cos 2 x ) 2 cos 4 x b2 b
+ − = + = (a + b) cos 4 x − 2b cos 2 x + . De aici cos 2 x = şi
b a+b a b a+b a+b
a sin 2010 x cos2010 x a1005 1 a b1005 b
sin2 x = = . Aşadar, ⋅ sin x + ⋅ cos x = ⋅ 1005 + ⋅ =
a+b a 1005
b1005
(a + b)1005
a a + b (a + b)1005
a+b

1 a+ b a+ b  π  π  π
= ⋅ = . 6. Presupunem că sin  x −  + sin  y −  + sin  z −  < 0 . De aici
(a + b)1005 a+b (a + b) 2011  6  6  6

1 3 3 3 3 3
avem că (cos x + cos y + cos z) > (sin x + sin y + sin z) > . Imediat cos x + cos y + cos z > , de
2 2 4 2
27
unde cos2 x + cos2 y + cos2 z + 2cos xcos y + 2cos ycos z + 2cos x cos z > , (1). Cum sin x + sin y + sin z ≥
4
3 9
≥ , găsim sin2 x + sin2 y + sin2 z + 2sin ysin x + 2sin xsin z + 2sin zsin y ≥ , (2). Adunând (1) cu (2) găsim
2 4
3 + 2cos(x – y) + 2cos(x – z) + 2cos(y – z) > 9, de unde cos(x – y) + cos(x – z) + cos(y – z) > 3, imposibil.
1 sin(( x + 1)  − x  ) 1 1
7. Avem = = [ tg ( x + 1) − tgx  ] . Însumând, găsim +

cos x cos( x + 1) 
sin 1 cos x  cos( x + 1)  sin 1 cos 0 cos1
1 1 tg89 ctg1 cos1
+  
+ ... +  
= 
= = .
cos 1 cos 2 cos 88 cos 89 sin1 sin1 sin 2 1


I.2. MulŃimea numerelor complexe


I.2.1. PROPRIETĂłI. OPERAłII
 (1 + u1 )(1 + u 2 )(1 + u1u 2 ) 
1. Fie z1 = ru1, z2 = ru2, cu |u1| = |u2| = 1. Atunci v = r 2 1 +  ∈ R+ dacă numărul u = 1 +
 u1u 2 
(1 + u1 )(1 + u 2 )(1 + u1u 2 )
+ ∈ R+. Folosind forma trigonometrică avem u1 = cos t1 + isin t1, u2 = cos t2 + isin t2,
u1u 2
t1  t t  t  t t 
 cos 1 + i sin 1  ; 1 + u 2 = 2 cos 2  cos 2 + i sin 2  , 1 +
u1u2 = cos(t1 + t2) + isin(t1 + t2), 1 + u1 = 2 cos
2 2 2 2 2 2
t +t  t +t t +t  t t t +t t +t
+ u1u 2 = 2 cos 1 2  cos 1 2 + i sin 1 2  . ObŃinem u = 1 + 8 cos 1 cos 2 cos 1 2 = 1 + 4 cos 1 2 ⋅
2  2 2  2 2 2 2
2
 t +t t −t  t +t t +t t −t t −t  t +t t −t 
⋅  cos 1 2 + cos 1 2  = 1 + 4 cos2 1 2 + 4 cos 1 2 cos 1 2 = sin 2 1 2 +  2cos 1 2 + cos 1 2  . În
 2 2  2 2 2 2  2 2 
concluzie v ∈ R+. 2. a) Avem (z1 + z 2 + z 3 )(z12 + z 22 + z 32 − z1z 2 − z1z 3 − z 2 z 3 ) + 3z1z 2 z 3 = z13 + z1z 22 + z1z 32 −
− z12 z 2 − z1z 2 z 3 + z12 z 2 + z 32 + z 2 z 32 − z1z 22 − z1z 2 z 3 − z 22 z 3 + z12 z 3 + z 22 z 3 + z 33 − z1z 2 z 3 − z1z 32 − z 2 z 32 = z13 + z 32 +
+ z 33 ; b) Din ipoteză, z13 + z 32 + z 33 = −z1z 2 z 3 , de unde –4z1z2z3 = (z1 + z2 + z3) ⋅ [(z1 + z2 + z3)2 – 3(z1z2 + z1z3 +
z1 + z 2 + z 3 1
+ z2z3)] sau [(z1 + z 2 + z 3 ) 2 − 3(z1z 2 + z1z 3 + z 2 z 3 )] = −4 . Din |zi| = 1, găsim zi = ,i = 1,3 . De
z1z 2 z 3 zi

282
aici (z1 + z2 + z3)3 = z1z2z3 ⋅ [3(z1 + z2 + z3) (z1 + z 2 + z 3 ) − 4] . Notând z = z1 + z2 + z3, obŃinem z3 = z1z2z3 ⋅
⋅ (3z ⋅ z − 4) , (1). Aplicând modulul în (1) găsim S3 = |3S2 – 4|, cu soluŃiile S = 1 şi S = 2 admisibile.
a2
3. Din |z1| = a obŃinem |z1|2 = a2, z1 ⋅ z1 = a 2 , de unde z1 = . Folosind şi analoagele şi din condiŃia z1 + z2 +
z1
a 2 b 2 c2
+ z3 = 0 suntem conduşi la z1 + z 2 + z 3 = 0 , de unde o putem transcrie sub forma + + = 0 , de unde
z1 z 2 z 3
a 2 b2 c2
+ = . După calcule a2z2(z1 + z2) + b2z1(z1 + z2) = (a2 + b2)z1z2, de unde concluzia b 2 z12 + a 2 z 22 = 0.
z1 z 2 z1 + z 2
4. Pentru a rezolva problema plecăm de la ideea că trebuia să arătăm că şirul (zn)n≥1 este periodic, ceea ce arată că
mulŃimea termenilor şirului este finită. În primul rând condiŃia z 2k = z k −1 ⋅ z k +1 , (∀) k ≥ 2, scrisă sub forma
zk z zk 1
= k +1 , zk ∈ C*, k ≥ 2 conduce la faptul că notând = , q ∈ C* (ca la şirul de rapoarte) arată că
z k −1 zk z k −1 q
zk+1 = q ⋅ zk, k ≥ 2 sau zk = z1 ⋅ qn–1. Ultima relaŃie arată că şirul (zn)n≥1 este o progresie geometrică. Acum
exploatăm faptul suplimentar că şirul are trei termeni reali, iar modulul tuturor termenilor este acelaşi. Fie zm, zn,
zp termenii reali ai şirului. Presupunem că m < n < p. Din faptul că am, an, ap ∈ R şi |zm| = |zn| = |zp| urmează
firesc că zm = zn = zp. Fiind termeni ai progresiei geometrice, avem zm = a1 ⋅ qm–1, zn = a1 ⋅ qn–1, zp = a1 ⋅ qp–1, de
z zp
unde avem că m = q m − n = 1 , iar = q p − n = 1 . Notăm l = min(m – n, p – n). Evident că ql = 1. Numărul l
zn zn
astfel determinat va număra cei l + 1 termeni ai progresiei, l fiind de fapt perioada acestui şir. De ce acest lucru:
dacă l ∈ N, atunci (∃) termenul al+1 al progresiei astfel încât al+1 = a1 ⋅ ql = a1, ceea ce arată că l este perioada
 a  b
şirului, de unde şi concluzia: mulŃimea termenilor progresiei este „l + 1”, finită. 5. Avem  z1 +  z 2 +  =
 z 2  z1 
ab ab
= z1z 2 + + a + b, (1). Notând z = z1z2. Din (1) şi ipoteză găsim z + ∈ R. Aceasta conduce la egalitatea
z1z 2 z
a ab  ab  ab
z+ =z+ , de unde (z − z)1 −  = 0 , (2). Cum ab < 0, 1 − > 0 , căci z ⋅ z =| z |2 > 0 , de unde
z z  z⋅z  z⋅z
obligatoriu în (2), z = z , ceea ce arată că z ∈ R. Notând z = l ∈ R*. De aici z1 ⋅ z2 = l. De aici
l lz1 l ⋅ z1 l
z2 = = = , unde z1 ⋅ z1 =| z1 |2 = p ∈ R*+. În final, alegând k = , găsim concluzia: z 2 = k ⋅ z1 .
z1 z1 ⋅ z1 p p
6. a) a = 1, b = i, c = –i; b) Dacă unul din fraŃi este nul, atunci evident că condiŃia conduce la faptul că ceilalŃi doi
au imaginile simetrice faŃă de origine; c) Metoda: reducerii la absurd! Presupunem că |a| ≥ |b| ≥ |c| ≥ 2. Atunci
|abc| = |a + b + c| ≤ |a| + |b| + |c| ≤ 3|a|. De aici |b| ⋅ |c| ≤ 3. Dar cum |b| ⋅ |c| ≥ |c|2 suntem conduşi la |c|2 ≤ 3, de
a2 a2
unde |c| ≤ 3 < 2 , absurd! 7. a) Din |z1| = |z2| = a, avem |z1|2 = |z2|2 = a2, de unde z1 = , z2 = . Analog din
z1 z2
| z12 + z1z 2 + z 22 |= a 2 avem | z12 + z1z 2 + z 22 |2 = a 4 , de unde (z12 + z1z 2 + z 22 )(z12 + z1z 2 + z 22 ) = a 4 , de unde,
efectuând calculele, găsim că (z1 + z 2 ) 2 (z12 + z 22 ) = 0 . De aici z1 = –z2, (1) sau z12 = −z 22 , (2). Din (1) şi ipoteză
z1
∈ C – R găsim z1 = ±iz2, de unde z12010 + z 2010 2 = (±iz 2 ) 2010 + z 2010
2 = i 2010 ⋅ z 2010
2 + z 2010
2 = z 2010
2 (1 + i 2010 ) =
z2

283
z1
= 0 ∈ R; b) Din (1) şi (2) şi concluzia că ∈ R găsim obligatoriu că z1 = –z2 şi z12 = z 22 ,de unde evident
z2
z1
z12010 + z 2010
2 ∈ R; c) Din ∈ C – R, atunci z1 = –iz2. ObŃinem z12009 + z 2009
2 = z 2009
2 + (−iz 2 ) 2009 = z 2009
2 (1 − i).
z2
Imediat | z12009 + z 2009
2 |=| z 2 |2009 2 = a 2009 2 . 8. Din z1 + z2 ∈ R avem că z1 + z 2 = z1 + z 2 , de unde
z1 − z 2 = z1 − z 2 , (1). Din z1 ⋅ z 3 + z 3 ⋅ z 2 ∈ R, urmează că: z1 ⋅ z3 + z 3 ⋅ z 2 = z1 ⋅ z 3 + z 3 ⋅ z 2 , de unde z 3 (z1 −
− z 2 ) = z 3 (z1 − z 2 ) , (2). Din (1) şi (2) găsim z 3 (z1 − z 2 ) = z 3 (z1 − z 2 ) , adică (z1 − z 2 )(z 3 − z 3 ) = 0 ; (3).
Deoarece z3 ∈ C – R găsim că z 3 ≠ z 3 , de unde obligatoriu z1 = z 2 sau z12010 = z 2010 2 , rezultând că
z + z 2 + z 3 + z1z 2 + z1z 3 + z 2 z 3 + z1z 2 z 3
z12010 + z 2010
2 = z12010 + z12010 ∈ R. 9. a) Notăm z = 1 în condiŃia |zi| = 1,
1 + z1z 2 z 3
1 1 1 1 1 1 1
+ + + + + +
1 z z 2 z3 z1 ⋅ z 2 z1 ⋅ z3 z 2 ⋅ z3 z1 ⋅ z 2 ⋅ z3
i = 1, 3 , avem |zi|2 = 1, de unde z i = , i = 1, 3 . ObŃinem z = 1 =
zi 1
1+
z1 ⋅ z 2 ⋅ z3
1 + z1 + z 2 + z 3 + z1 ⋅ z 2 + z1 ⋅ z 3 + z 2 ⋅ z 3
= = z , de unde z ∈ R; b) Din z1 + z2 + z3 = 1, obŃinem z12 + z 22 + z 32 +
1 + z1 ⋅ z 2 ⋅ z 3
 1 1 1
+2(z1z 2 + z 2 z 3 + z1z 3 ) = 1 , obŃinând z12 + z 22 + z 32 + 2 + +  = 1 . Avem imediat z12 + z 22 + z 32 + 2 ⋅
 z 3 z 2 z1 
 z z z 
⋅  3 2 + 2 2 + 1 2  = 1 . Cum |z1| = |z2| = |z3| = 1 şi din z1 + z2 + z3 = 1 găsim z1 + z 2 + z 3 = +1 . De aici
| z | | z2 | | z1 | 
 3
z
z12 + z 22 + z 32 + 2 = 1 , de unde z12 + z 22 + z 32 = −1 . 10. Fie z = |z| (cos α + isin α). Atunci = cos α + i sin α , iar
|z|
|z| 1 z |z|
= = cos α – isin α. De aici + = 2 cos α ∈ R. De aici rezultă că z2 + |z|2 = 2z ⋅ |z| ⋅
z cos α + i sin α |z| z
⋅ cos α, de unde z2 = 2|z| ⋅ cos α ⋅ z – |z|2, de unde a = 2|z|cos α, iar b = –|z|2.

I.2.1. PROPRIETĂłI ALE NUMERELOR COMPLEXE. OPERAłII


1. i) A = (a + bω + cω2)(a + bω2 + cω) = a2 + b2 + c2 – ab – ac – bc ∈ R; ii) 2(aω2 + bω + c)(aω + bω2 + c) =
= 2(a2 + b2 + c2 – ab – ac – bc) = (a – b)2 + (a – c)2 + (b – c)2. 2. Efectuând calculele şi aducând la acelaşi
numitor, găsim numărătorul (a – b)(b – c)(c – a)(1 + i), de unde E = 1 + i. 3. a) Notăm x = a + bω + cω2 şi cu y =
= a + bω2 + cω. Notăm S = x + y = 2a – b – c şi P = a2 + b2 + c2 – ab – ac – bc. E = S3 – 2PS = S(S2 – 2P) = (2a –
– b – c)(2b – c – a)(2c – a – b). 4. P1(z) = (z – ω)(z + ω)2 iar P2(z) = (z – ω2)2(z + ω)2; P1(z2) = (z2 – ω)(z2 + ω)2 =
= (z2 – ω)(z2 – ω4)2 = (z2 – ω)(z – ω2)2(z + ω2)2 = (z2 – ω)(z + ω2)(z – ω2)2(z + ω2) = (z2 – ω)(z + ω2)P2(z), de
unde A = (z2 – ω)(z + ω2). Analog P2(z) = (z – ω)(z2 + ω2)P1(z), de unde B = (z – ω)(z2 + ω2). 5. Se scrie (a +
+ ib)2 = a4 + b4 – 6a2b2 + 4ab(a2 – b2)i şi analoagele. Se Ńine cont că b + c + a = 0 şi se obŃine: i[4abc(a – b) +
+ 4abc(b – c) + 4abc(c – a)] = 4abc(a – b + b – c + c – a) = 0, de ubde E ∈ R. Reciproca este falsă. Producem un
contraexemplu: a = 0, b = c = 1. Atunci E = i4 + (1 + i)4 + 14 = 1 – 4 + 1 = –2 ∈ R iar a + b + c = 2 ≠ 0.

284
I.2.2. INTERPRETĂRI GEOMETRICE. FORMA TRIGONOMETRICĂ
1. a) z = q, q ∈ Q, este soluŃie a problemei; b) Fie z ∈ C – Q astfel încât Re z ∈ Q, Im z ∈ Q*, |z| ∈ Q. Scriem z
sub formă trigonometrică, z = r(cos α + isin α). Fie r ∈ Q*+, rcos α ∈ Q, rsin α ∈ Q*, α ∈ (0, π) ∪
1− t2 2t
∪ (π, 2π) sau r ∈ Q*+, cos α ∈ Q, sin α ∈ Q*, α ∈ (0, π) ∪ (π, 2π) sau r ∈ Q*+, ∈ Q, ∈ Q*, unde
1+ t2 1+ t2
α 1− t2
t = tg . Dacă t ∈ Q*, avem z = r(cos α + isin α), z ∈ C – Q. Arătăm că t ∈ Q*. Într-adevăr, fie =a,
2 1+ t2
2t 1− a b
= b . Din prima egalitate găsim t 2 = şi înlocuind în a doua găsim t = ∈ Q. În concluzie
1+ t 2
1 + a 1 + a
1− t2 2 t  1
numerele căutate sunt de forma z = r 2
+ i , r ∈ Q*+, t ∈ Q* sau de forma z = q, q ∈ Q. 2. z + +
2 
 1 + t 1 + t  z
1 1 1 1 1 2 3
+ 2 ∈ R dacă şi numai dacă z + + 2 = z + + 2 , adică (z3 + z + 1) z = (z + z + 1)z 2 . Notând t = z ⋅ z ,
z z z z z
1
găsm (z − z)( t 2 − t − 2 Re z) = 0 , de unde z = z , adică z ∈ R. 3. Avem z k + z k ≥ 0 şi în plus z k = , k = 1,3 .
zk
z1 z 2 z1 z3 z 2 z3
Fie z = z1 + z2 + z3. Evident z + z ≥ 0 . De aici |z|2 = z ⋅ z = 3 + + + + + + ≥ 1 , de unde |z| ≥ 1.
z 2 z1 z3 z1 z 3 z 2
 π π
4. Fie z = ρ(cos θ + isin θ), z = ρ(cos θ − i sin θ) . ObŃinem Z = ρ(cos θ + isin θ) – ρ cos + i sin (cos θ −
 3 3
 π  π   π  π 
– isin θ) = ρ(cos θ + isin θ) – ρ cos − θ i + sin  − θ   = ρ cos θ − cos − θ  + i(sin θ − sin  − θ   =
 3  3   3  3 
 π  π π  π   π  π π  π   π π 
= ρ − 2 sin sin  θ −  + 2i cos sin  θ −  = 2ρ sin  θ −  − sin + i cos  = 2ρ sin θ −  cos +  +
 6  6  6  6   6  6 6   6   2 6 
n n
 π π   π  2π 2π 
+ i sin  +  = 2ρ sin  θ −  cos
 2 6   6  3
+ i sin  . 5. Fie zi = cos ϕi + isin ϕi. Atunci cos
3  ∑ ϕ + i sin ∑ ϕ
i =1
i
i =1
i =

n
ϕi  ϕ ϕ 
= 1 ⇒ ∑ϕ
i =1
i = 2kπ , (1). Dar 1 + zi = 1 + cos ϕi + isin ϕi = 2 cos
2 
 cos i + i sin i  . Pe de altă parte
2 2
n n
ϕi  ϕ ϕ 
n
ϕi  n
ϕi
n
ϕi 
∏ (1 + z i ) = ∏ 2 cos
2 
 cos i + i sin i  = 2
2 2 
n
∏ cos
2
 cos
 ∑ 2
+ i sin ∑ 2  . De aici şi (1) găsim
i =1 i =1 i =1  i =1 i =1 
n
ϕi
i sin ∑2
i =1
= i sin kπ = 0.

I.2.3. MODULUL NUMĂRULUI COMPLEX. INEGALITĂłI

1. a) Fie |z1| = |z2| = r. Avem |αz1 + z2|2 = (αz1 + z2) (α z1 + z 2 ) = (1 + α 2 )r 2 + α (z1 z 2 + z1z 2 ) = (z1 + αz 2 )(z1 +

285
+ α z 2 ) =| z1 + αz 2 |2 ; b) n|z1 + z2| = |(n – 1)z1 + z1 + z2 + (n – 1)z2| ≤ |(n – 1)z1 + z2| + |z1 + (n – 1)z2| = 2|(n – 1) ⋅
⋅ z1 + z2|. 2. a) Avem | z1 + z 2 |2 + | z1 − z 2 |2 = (z1 + z 2 )(z1 + z 2 ) + (z1 − z 2 )(z1 − z 2 ) =| z1 |2 + z1 z 2 + z 2 z1 + | z 2 |2 +
+ | z1 |2 −z1z 2 − z 2 z1 + | z 2 |2 = 2(| z1 |2 + | z 2 |2 ) ; b) Folosind a) găsim |z1 + z2|2 + 2 = 4, de unde | z1 + z 2 |= 2 .
3. Folosim egalitatea |z + w|2 + |z – w|2 = 2|z|2 + 2|w|2, (1). Ridicând la pătrat ambii membri găsim: |c – a|2 + |c +
+a|2 + 2|c – a| ⋅ |c + a| = |c – b|2 + |c + b|2 + 2|c – b| ⋅ |c + b|, de unde, aplicând (1) şi condiŃia, găsim: 2|c|2 + 2|a|2 +
3
1  1 1  1
+2|b|2 = 2|c|2 + 2|b|2 + 2|a|2. 4. Notăm r = z + , (1). ObŃinem şirul de implicaŃii:  z +  = z 3 + 3 + 3 z +  .
z  z z  z
3
 1 1  1 1 1 1
Aplicând modulul, găsim:  z +  = z 3 + 3 + 3 z +  ≤ ≤ z 3 + 3 + 3 z + ≤ 2 + 3 z + . Folosind (1),
 z  z  z  z z z
obŃinem r3 ≤ 2 + 3r, r ≥ 0. ObŃinem r3 – 3r – 2 ≤ 0, de unde (r + + 1)2(r – 2) ≤ 0, ce conduce la r ≤ 2, adică
1 1
z + ≤ 2 . 5. Metoda de rezolvare folosită va fi metoda reducerii la absurd. Presupunem că z − < 2 . Atunci
z z

1 1 2 1 1  1
2
1  2 
z3 − = z − ⋅ z + + 1 = z − ⋅  z −  + 3 < z − ⋅ ⋅  z − 1 + 3  = 14 , absurd, căci z 3 − 1 ≥ 14 ,
z 3
z z 2
z  z z  z  z3
 
1
conform ipotezei. Firesc deci, z − ≥ 2 . 6. Folosim inegalitatea modulului. ObŃinem şirul de inegalităŃi:
z
|z1 + 1| + |z2 + 1| + |z1z2 + 1| ≥ |z1 + 1| + |1 + z1z2 – z2 – 1| = |z1 + 1| + |z2| ⋅ |z1 – 1| = |z1 + 1| + |z1 – 1| ≥
≥ |1 + z1 – z1 + 1| = 2. 7. Avem |(1 – α)(a – b) – β(a – c)| + |(1 – β)(a – – c) – α(a – b)| ≤ |1 – α| ⋅ |a – b| + |β| ⋅
⋅ |a – c| + |1 – β| ⋅ |a – c| + |α| ⋅ |a – b| = |a – b|(1 – β + β) + |a – c|(γ + 1 – γ) = |a – b| + |a – c| (Am Ńinut
1  1
cont că α, β > 0, α + β = conduce la α, β ∈  0,  ). 8. Cu notaŃiile a = |z1 – z2|, b = |z2 – z3|, c = |z1 – z3|,
2  2
1 1 1 1
inegalitatea de demonstrat se rescrie sub forma + + ≥ . Dar, conform inegalităŃii C-B-S, găsim
ab bc ac r 2
1 1 1 (1 + 1 + 1) 2 9
+ + ≥ ≥ . Pe de altă parte a2 + b2 + c2 = |z1 – z2|2 + +|z2 – z3|2 + |z3 – z2|2 =
ab bc ca ab + bc + ca a 2 + b 2 + c 2
= 3(|z1|2 + |z2|2 + |z3|2 – |z1 + z2 + z3|2) ≤ 3(|z1|2 + |z2|2 + |z3|2) = 9r2, de unde concluzia problemei.
9. Din inegalitatea modulului găsim |z – z1| + |z – z2| + |z – z3| ≥ |3z – (z1 + z2 + z3)| = 3|z|, (1). Analizăm cazurile:
i) dacă |z| ≥ 1, înlocuind în (1) problema este rezolvată; ii) dacă z = 0, avem egalitate în inegalitatea
propusă; iii) dacă |z| < 1, atunci z −1 − z i−1 =| z − z i | ⋅ | z −1 | , (2), (∀) i = 1, 2, 3. Deoarece |z| < 1, atunci

|z–1| > 1. Din |zi| = 1 găsim |zi|2 = 1 şi imediat relaŃia echivalentă z i ⋅ z i = 1 ce conduce la z i−1 = z i ,
(∀) i = 1, 2, 3. ObŃinem z1−1 + z −2 1 + z 3−1 = z1 + z 2 + z 3 = z1 + z 2 + z 3 = 0 . RelaŃia (1) se poate rescrie:
z −1 − z1−1 + + z −1 − z −2 1 + z −1 − z 3−1 ≥ 3 | z −1 | sau folosind (2) obŃinem |z – z1| ⋅ |z–1| + |z – z2| ⋅ |z–1| + |z – z3| ⋅

 2 
⋅ |z–1| ≥ 3|z–1|, de unde inegalitatea propusă. 10. Fie z = r(cos t + isin t), r ∈  0, , t ∈ [0, 2π). Inegalitatea din
 2 
 

286
n

 n k
2

n− ∑ cos 2kt
enunŃ devine 
∑
 k =1
r sin kt  <


k =1
2
. Utilizând inegalitatea lui Cauchy-Buniakovski-Schwarz,

 n k 
2
 n 2 k  n  1 − r 2n
n
1 − cos 2kt 1 − r 2n
n− ∑ cos 2kt
obŃinem: 
 ∑
r sin kt  ≤ 
  ∑r 
 ∑
sin 2 t  = r 2 ⋅
 2∑⋅
1 − r k =1 2
= r2 ⋅
1− r2
⋅ k =1
2
<
 k =1   k =1  k =1 
n
n− ∑ cos 2kt  2  r2 1 − r 2n
< k =1
deoarece din r ∈  0, , 1 – r2n < 1, iar < 1 , de unde r2 ⋅ <1.
2  2  1− r 2
1− r2
 

II. FuncŃii
II.1. ProprietăŃi generale ale funcŃiilor. OperaŃii
II.1.1. FUNCłII INJECTIVE
1. Metoda folosită: reducerea la absurd! Presupunem că există o funcŃie injectivă cu proprietatea că f(3x) +
+ f(5x) = 4, (∀) x ∈ (0, ∞). Pentru x = 1 găsim f(3) + f(5) = 4, (1) iar pentru x = log35 găsim f(5) + f (5log 3 5 ) = 4,
(2). Din (1) şi (2) găsim f(3) = f (5log 3 5 ) şi cum f injectivă, 3 = 5log35, de unde log35 = 1, adică 5 = 3. Absurd! În
concluzie: nu există funcŃii injective ce verifică ipotezele problemei. 2. Metoda: reducerea la absurd. Pentru x =
= 2, avem f(2) + f ( 2 ) + f (4) = 2009 , (1) iar pentru x = 4, f(4) + f(2) + f(16) = 2009, (2). Din (1) şi (2) şi
ipoteza, f injectivă, găsim f ( 2 ) = f (16) , de unde 2 = 16 , absurd! Concluzia: f nu este injectivă. 3. Metoda:
reducerea la absurd! Avem f(1) = 2 = f(2) şi cum f ar fi injectivă, am obŃine 1 = 2, absurd! În concluzie nu există
funcŃii injective cu proprietatea dată. 4. a) Pentru x1 = x2 = 1, găsim f(1) = 2f(1), de unde f(1) = 0; b) Pentru
 1   1 
x 2 = ∈ (0, ∞ ) rezultă f(1) = f(x1) + f   , deci f   = – f(x1), ∀x1 ∈ (0, ∞ ). Fie x1, x2 ∈ (0, ∞ ) cu f(x1) =
1
x1 x
 1  x1 
x   1 
= f(x2), atunci f  1  = f ( x1 ) + f   = f(x1) – f(x2) = 0, şi cum 1 este unica soluŃie a ecuaŃiei f(x) = 0, rezultă
 x2   x2 
că x1 = x2, deci f este injectivă. 5. Metoda folosită este reducerea la absurd! Presupunem f injectivă. Pentru x = 0
obŃinem: 3f3(0) – f2(0) – f4(0) – 4 ≥ 0, (1), de unde, notând f(0) = y, obŃinem y4 – 3y3 + y2 + 4 ≤ 0, de unde
y2(y – 2)2 + y(y – 2)2 + (y – 2)2 ≤ 0 sau (y – 2)2(y2 – y + 1) ≤ 0, (2). Din (2) cum y2 – y + 1 > 0, obligatoriu y = 2,
de unde f(0) = 2, (3). Dacă x = 1, după înlocuire, ajungem la 3f3(1) – f2(1) ≥ f4(1) + 1. Notând t = f(1), după
înlocuire şi factor comun, absolut analog ca până acum, ajungem la f(1) = 2, (4). Din (3) şi (4) şi ipoteza că f este
injectivă ajungem la f(0) = f(1), adică 0 = 1. 6. Folosim metoda reducerii la absurd. Presupunem f injectivă.
Pentru x = 1, obŃinem relaŃia f(3) + f(0) = 1, (1) iar pentru x = 27, f(327) + f(3) = 1, (2). Din (1) şi (2) găsim
f(327) = f(0) şi cum am presupus că f injectivă, obŃinem 327 = 0, absurd! Urmează firesc că f nu este injectivă.
7. Presupunem că f injectivă. Atunci din f(x) = f(y) implică f(f(x)) = f(f(y)) de unde [x] = [y], absurd! De
1 2
exemplu, alegând x = ≠ y = , deci f nu poate fi injectivă!
2 3

287
II.1.2. FUNCłII SURJECTIVE

1. Dacă A = {x1, x2, ..., xa}, B = {y1, y2, ..., yb} cu a ≥ b, definim f: A → B, f(xi) = yi, i = 1, b , f(xi) = yk, (∀) i =
= b, a , iar k fixat, k = 1, b . Evident f este surjectivă. Reciproc: presupunem că există o surjecŃie f: A → B. Cum
f: A → B, f(A) ⊂ B avem card B = card f(A) ≤ card A, cu alte cuvinte, b ≤ a. 2. Fie a > 0 şi să presupunem prin
absurd că există o funcŃie f cu proprietatea din enunŃ. Cum f este surjectivă, pentru fiecare k ∈ N, există mk ∈ N
astfel încât f(mk) = k. Evident avem mi ≠ mj, (∀) i, j ∈ N, i ≠ j. Folosind relaŃia din enunŃ, vom avea k = f(mk) ≥
k   k  
≥ a ⋅ m 2k , de unde m k ≤ , adică {m0, m1, m2, ..., mk} ` 0,1, 2, ...,    , (∀) k ∈ N, (1). Fie funcŃia
a   a  
n n
g: N → R, g(n ) = (n − 1) 2 − , (∀) n ∈ N. Urmează că (∃) n0, astfel încât g(n0) ≥ 0, adică (n0 – 1)2 ≥ 0 , de
a a
n0  n0 
unde n0 – 1 ≥ ≥  , (2). În (1) luăm k = n0, de unde rezultă {m 0 , m1 , m 2 , ..., m n 0 } ⊆ {0, 1, 2, ...,
a  a 
 n 0  
   ⊂ {0, 1, 2, ..., n0 – 1}, absurd deoarece m0, m1, ..., m n 0 sunt distincte două câte două. Urmează că nu
 a  
există funcŃii cu proprietatea din enunŃ. 3. Pentru x = 0 găsim f(f(y)) = f(0) + f2(y). Cum f este surjectivă, penru
orice t din [ 1, ∞ ), există yt ∈ R astfel încât f(yt)= t. Rezultă f(t)= f(f(yt)) = f(0)+f2 (yt)= f(0) +t2 = t2 +a.
Rezultă Imf = [ a, ∞ ) şi cum f este surjectivă deducem a = 1. 4. Fie g: R+ → R, surjectivă. Notăm
g( x ), x > 0 f ( x ) − g( x ), x > 0
 
f1 ( x ) = f (0), x = 0 , f2(x) = 0, x = 0 . Dar g(R+) = R = f1(R+) ⊂ f1(R), de unde f1(R) = R,
f ( x ) − g (− x ), x < 0 g (− x ), x<0
 
(1). Analog R = g(R+) = f2(R–) ⊂ f2(R) de unde f2(R) = R, (2). łinând cont de (1) şi (2) avem: f = f1 + f2. 5. i) Fie
M = maxA, m = minA. Atunci M – m = max | x − y | , (1). Presupunem că f ar fi surjecŃie şi atunci rezultă că
x , y∈A

(∃) x, y ∈ A astfel încât f(x) = M, m = f(y), de unde M – m = |M – m| = |f(x) – f(y)| < |x – y| ≤ M – m, din (1). Ar
rezulta că M – m < M – m, ceea ce este absurd, deci f nu este surjecŃie; ii) Fie f: A → A cu proprietatea din
enunŃ. Cum A este finită, ea se poate ordona: A = {x1 < x2 < x3 < ... < xn–2 < xn–1 < xn}. Să presupunem că f(x) ≠ x,
(∀) x ∈ A. Vom partiŃiona mulŃimea A în două submulŃimi astfel: B = {x ∈ A | f(x) > x} şi C = {x ∈ A | f(x) < x}.
Dacă x ∈ B şi y ∈ C, evident x < y. (în caz contrar, dacă x > y, atunci f(x) > x > y > f(y) şi deci |f(x) – f(y)| > |x – y|,
absurd). Dar A = B ∪ C şi B ∩ C = ∅. Fie xp ultimul element al lui B şi xp+1 primul element al lui C şi deci
x ≤ xp < xp+1 ≤ y, (∀) x ∈ B, y ∈ C. Avem f(xp) > xp şi deci f(xp) ≤ xp+1, iar f(xp+1) < xp+1, de unde f(xp+1) ≤ xp. În
consecinŃă |f(xp) – f(xp+1)| ≥ |xp+1 – xp|, contradicŃie. Deci, (∃) x0 ∈ A astfel încât f(x0) = x0. 6. a) f(x) =
 π
sin , x ≠ 0 π
= x . EcuaŃia sin = λ, λ ∈ [–1, 1] are o infinitate de soluŃii în [–1, 1];
0, x
x=0

288
0, x ∈ {0, 1} ∪ {k ∈ N m k ∉ N, (∀) m ∈ N}

1, x ∈ {2, 3, 5, ..., n , ...}, n nr. prim

 2 2 2 2
b) f ( x ) = 2, x ∈ {2 , 3 , 5 , ..., n , ...}, n nr. prim .
..............................................................
 s s s s
s, x ∈ {2 , 3 , 5 , ..., n , ...}, n nr. prim
...............................................................

7. Vom arăta că soluŃia căutată va fi f: N → N, f(n) = n + + (–1)n, (∀) n ∈ N. Evident funcŃia g: N → N, g(n) =
= n + (–1)n este surjectivă, căci ecuaŃia g(n) = k are întotdeauna soluŃie în N. Vom arăta că f(n) = g(n), (∀) n ∈ N.
Pentru k = 0, f(n) = 0 are soluŃia n = 1, de unde f(1) = g(1) = 0. Pentru n ≥ 2 f(n) ≥ n + (–1)n ≥ 2, de unde n = 0,
deci f(0) = 1. Ca atare f(n) = g(n), (∀) n ∈ {0, 1}. Presupunem prin absurd că f ≠ g. Fie a cel mai mic număr
natural astfel încât f(n) = g(n), (∀) n < a şi f(a) > g(a). Cazuri: i) a = 2p + 1, p ∈ N*. Cum f(n) = g(n), (∀) n < a,
avem pentru n = a, f(a) > g(a) = a – 1. De aici n > a, f(a) ≥ g(n) ≥ a + 1, de unde a – 1 ∉ f(N), contradicŃie;
ii) a = 2p, p ∈ N*. Cum f(n) = g(n), (∀) n < a, pentru n = a, f(a) > g(a) = a + 1 şi pentru n ≥ a + 1, f(n) ≥ g(n) ≥
≥ a + 2, de unde a – 1 ∉ f(N), contradicŃie.

II.1.3. FUNCłII BIJECTIVE

1. g este bijectivă ca sumă a două funcŃii bijective; h1(x) = x, h2(x) = 3


x , (∀) x ∈ R. Vom arăta că f = g–1 şi cum
g bijecŃie, se ştie că şi inversa sa va fi bijecŃie. Deci arătăm că f = g–1. RelaŃia f ( x ) + 3 x ≤ x ≤ f ( x ) + 3 f ( x ) este
echivalentă cu (f  g)(x) ≤ x ≤ (g  f)(x). Dacă x = g–1(t), în (f  g)(x) ≤ x, se obŃine f(t) ≤ g–1(t), (∀) t ∈ R, (1).
(Am Ńinut cont că g bijecŃie admite inversă!). Pe de altă parte, x ≤ (g  f)(x) conduce la f(x) ≥ g–1(x), (∀) x ∈ R,
(2). Din (1) şi (2) concluzia, f = g–1. 2. Avem f bijectivă este echivalent cu (∀) w ∈ C, există un unic z ∈ C cu
proprietatea w = f(z). Într-adevăr, w = 2z + z , (1) conduce la w = 2z + z , de unde, folosind (1) şi (2), găsim că
2 1 2 1
z= w − w . Evident că inversa sa va fi f −1 (z) = z − z , (∀) z ∈ C (s-a folosit faptul că orice bijecŃie este
3 3 3 3
inversabilă, f–1 reprezentând inversa lui f). 3. Este suficient să producem un exemplu; f: R → R,
x +1 x+3
f(x) = , bijectivă, astfel încât f (f ( x )) = . 4. a) Presupunem că f nu este injectivă. Atunci (∃) n1, n2 ∈
2 4
∈ Z, n1 ≠ n2, (1) şi f(n1) = f(n2). Dacă în ultima egalitate aplicăm f, în ambii membri, găsim f(f(n1)) = f(f(n2)),
adică n1 = n2, absurd! Înseamnă că presupunerea este falsă. Arătăm că f este surjectivă. Notăm y = f(n), (∀) n ∈
∈ Z. Atunci (∀) y ∈ Z vom arăta că există x ∈ Z astfel încât f(x) = y. Alegem x = y. Atunci, (∀) n ∈ Z, avem că
(∀) y ∈ Z, y = f(n), există x ∈ Z, x = y astfel încây f(x) = f(y) = f(f(n)) = n, ceea ce arată că f surjectivă. b. Dacă
f este bijectivă este şi injectivă, deci, din ipoteza i) şi ii) suntem conduşi la f(f(n – 2) – 2) = f(f(n)), de unde f(n) =
= f(n – 2) – 2, (∀) n ∈ Z. Pentru n = 2k + 1 (impar), găsim f(2k + 1) = –2k = –(2k + 1) + 1, (∀) k ∈ Z iar pentru
n = 2k (par), obŃinem f(2k) = –2k + 1, (∀) k ∈ Z, de unde f(n) = –n + 1, (∀) n ∈ Z. Imediat, cum f bijecŃie, deci
inversabilă, deducem că f = f–1, adică f(n) = f–1(n) = –n + 1, (∀) n ∈ Z. 5. a) FuncŃia f1: (–∞, 1] → R, f1(t) = t2 –
– 2t + 2 este strict descrescătoare iar funcŃia f2: (1, ∞) → R, f2(t) = 1 − t este de asemenea strict
x − 2 x + 2, x ≤ 1
2
descrescătoare. De aici, f: R → R va fi f ( x ) =  . Urmează că f(R) = (–∞, 0) ∪ [1, ∞), de unde,
1 − x , x >1
evident, f nu este surjectivă. FuncŃia f fiind strict descrescătoare pe ramuri şi f((–∞, 1]) = [1, ∞) iar f((1, ∞)) =

289
= (–∞, 0), deci f este injectivă. Cum f(R) = (–∞, 0) ∪ [1, ∞), atunci (∃) y ∈ [0, 1) astfel încât (∀) x ∈ R, f(x) ≠ y,
ceea ce arată că f nu este surjectivă; b) Vom analiza cazurile: i) k > –1; g(R) = (–∞, 0) ∪ [k + 1, ∞) şi 1 + k > 0,
deci g nu este surjectivă; ii) k < –1; g(R) = (–∞, 0) ∪ [k + 1, ∞) şi 1 + k < 0, deci g nu este injectivă; iii) k = –1,
1 + k = 0; g(R) = R, deci g surjectivă şi g((–∞, 1]) = [0, ∞), g((1, ∞)) = (–∞, 0), deci g injectivă. De aici g
f ( x ) − 1, x ≤ 1
bijectivă; c) Din b) avem g bijecŃie, deci g inversabilă. Atunci g–1: R → R. Cum g( x ) =  . Cum
f ( x ), x >1
y = g(x), găsim y = = (x – 1)2, de unde x = 1 − y sau y = 1 − x , de unde x = (1 – y)2. Urmează că
1 − x , x ≥ 0
g −1 ( x ) =  .
(1 − x ) 2 , x < 0
x2 + x
6. Cum f(0) = a, f(1) = a + 1, găsim a ≥ 0 şi a ≤ 0, de unde a = 0. Deci f ( x ) = . f este bijectivă dacă pentru
x2 +1
orice y∈ [0,1] există un unic x∈[0,1] astfel încât f(x) = y. Pentru y =1 găsim x = 1, iar pentru y ∈ [0,1) găsim
1 + 4y − 4y 2 − 1
x = . Fie a ∈ R, a ≠ 0. Presupunem că există funcŃii cu proprietatea din enunŃ. Dacă în
2(1 − y)
condiŃie înlocuim x cu a2, găsim a2f2(a4 + a2) – 2af(a4 + a2) + 1 ≤ 0 sau [af(a4 + a2) – 1] ≤ 0, (1). Înlocuind în
condiŃie x = 1 găsim a2f2(a2 + 1) – 2af(a2 + 1) + 1 ≤ 0 sau [af(a2 + 1) – 1]2 ≤ 0, (2). Din (1) şi (2) găsim pe rând
1 1
f(a4 + a2) = = , respectiv f (a 2 + 1) = , de unde f(a4 + a2) = f(a2 + 1). Cum f bijecŃie, urmează că f injecŃie, de unde
a a
x
a + a = a + 1, de unde a ∈ {–1, 1}. Reciproc: Dacă a = 1, producem ca exemplu funcŃia f: R → R, f ( x ) = ,
4 2 2
2
x
bijectivă, care verifică condiŃia. Imediat dacă a = –1, f ( x ) = − , bijectivă, verifică.
2

II.1.4. FUNCłII MONOTONE

1. Membrul stâng al egalităŃii g(x) = f(f(x)) este o funcŃie crescătoare, (∀) x ∈ [0, 1], iar membrul stâng
1 1 1
h: [0, 1] → R, h(x) = {x} nu este o funcŃie strict crescătoare. Într-adevăr, fie < 1 , h   = > h(1) = 0.
2 2 2
(g  f )( x1 ) − (g  f )( x 2 ) g(f ( x1 )) − g (f ( x 2 )) g (f ( x1 )) − g (f ( x 2 )) f ( x1 ) − f ( x 2 )
2. Avem = = ⋅ < 0 , ceea ce arată
x1 − x 2 x1 − x 2 f ( x1 ) − f ( x 2 ) x1 − x 2
că g  f este strict descrescătoare. 3. Fie x ∈ [–a, 0]. De aici fie x1, x2 ∈ [–a, 0] cu x1 < x2, de unde –x1 > –x2, deci

a – x1 > a – x2; (a – x1)2 > (a – x2)2, de unde b2 + (a – x1)2 > b2 + (a – x2)2, adică b2 + (a − x1)2 > b2 + (a − x 2 )2 .

Analog b 2 + (a + x1 ) 2 > b 2 + (a + x 2 ) 2 , de unde f(x1) > f(x2), ceea ce arată că f este descrescătoare pe [–a, 0].
(1 − x1 ) 1− x2
Procedând la fel, arătăm că f este crescătoare pe [0, a]. 4. Din x1 < x2 avem f(x1) – f(x2) = 2
− =
(x1 + 2) (x 2 + 2)2
x 22 + 4x 2 + 4 − x1x 22 − 4x1x 2 − 4x1 − x12 − 4x1 − 4 + x 2 x12 + 4x1x 2 + 4 x 2
= =
( x1 + 2) 2 ( x 2 + 2) 2

290
x 22 − x12 + 8( x 2 − x1 ) − x1x 2 ( x 2 − x1 ) ( x 2 − x1 )[ x1 + x 2 − x1x 2 + 8]
= = > 0 , (∀) x1, x2 ∈ [–1, 1]. 5. Vom arăta că
( x1 + 2) 2 ( x 2 + 2) 2 ( x 1 + 2) 2 ( x 2 + 2) 2
 a + b a + b  a+b
f: a ,  → R, f este crescătoare, iar pentru f:  , b  → R, f este descrescătoare. Fie a ≤ x1 < x2 < .
 2   2  2
De aici f(x1) < f(x2). Într-adevăr, x1 − a + b − x1 ≤ x 2 − a + b − x 2 revine la ( x1 − a )(b − x1 ) ≤
 a + b
≤ ( x 2 − a )(b − x 2 ) , de unde (x2 – x1)(x1 + x2 – a – b) ≤ 0, dacă x1 < x2, x1, x2 ∈ a , . Analog cazul
 2 
a + b  a + b 
f:  , b  → R, atunci f(x1) > f(x2) dacă x1 < x2, x1, x2 ∈  , b  , căci (x2 – x1)(x1 + x2 – a – b) ≥ 0, dacă
 2   2 
a + b 
x1 < x2, x1, x2 ∈  , b  . 6. Distingem cazurile: a) a = 0, f constantă; b) a = 1 sau b = 1, f constantă; c) 0 < a <
 2 
< 1, 0 < b < 1, f crescătoare; d) a > 1, b > 1, f crescătoare; e) 0 < a < 1, b > 1, f crescătoare; f) a > 1, 0 < b < 1, f
descrescătoare. 7. Demonstrăm prin inducŃie că f(n) = n – 1 + f(1), (∀) n ≥ 1. Este o inducŃie de tip Cauchy. Deci
f(2) = f(1) + 1, iar f(4) = f(2) + 2 = f(1) + 3. Deoarece f(1) + 1 = f(2) < f(3) < f(4) = f(1) + 3. De unde rezultă că
f(3) = f(1) + 2. Presupunem f(k) = f(1) + k – 1, (1). Arătăm că f(k + 1) = f(1) + k. Din ipoteză f(2k) = f(k) + k.
Cum f(1) + k – 1 = f(k) < f(k + 1) < f(k + 2) < ... < f(2k – 1) < f(2k) = f(1) + 2k – 1. Cum între f(1) + k – 1 şi
f(1) + 2k – 1 sunt exact k – 1 numere naturale, f(k + 1), f(k + 2), ..., f(2k – 1), urmează că f(k + 1) = f(1) + k. În
concluzie f(n) = f(1) + n – 1.

II.1.5. FUNCłII INVERSABILE

1. Folosim următorul rezultat: Orice funcŃie bijectivă este inversabilă. Prin urmare vom arăta că f este bijectivă.
avem f injectivă pe (–∞, 0) şi în plus f((–∞, 0)) = (∞, 1), (1). Analog f injectivă pe [0, π] şi în plus f([0, π]) =
= [1, –1], (2). În sfârşit f injectivă pe (π, ∞) şi în plus f((π, ∞)) = (–1, –∞), (3). Urmează că f injectivă pe R şi din
(1), (2), (3) avem că f(R) = R, deci f surjectivă. Concluzia este imediată: f este bijectivă. Avem f–1: R → R,
− log 2 x , x ∈ (1, ∞)
−1 
f ( x ) = arccos x, x ∈ [−1,1] . 2. Presupunem x = y, de unde f2(x) – f(x) ⋅ [x + (–1)xp – 2] = 2[x + (–1)xp], de
− πx, x ∈ (−∞, − 1)

unde f(x)(f(x) + 2) – (x + (–1)xp)(f(x) + 2) = 0. De aici [f(x) + 2] ⋅ [f(x) – (x + (–1)xp)] = 0. Urmează că f(x) = –2,
f(x) = x + (–1)xp, (∀) x ∈ Z. Se vede că ultima funcŃie este bijectivă, deci inversabilă. Evident f(x) = x – (–1)xp,
1 x 2 + 4x − x
(∀) x ∈ Z. 3. Din relaŃia dată rezultă g(x) = f(x) + 1 = , de aici f ( x ) = , (1) şi g(x) =
xf ( x ) 2x
x 2 + 4x + x 1
= , (2). Se verifică uşor că f şi g bijecŃii, de unde f–1 şi g–1 există. Se calculează f −1 ( t ) = şi
2x t ( t + 1)
1
g −1 ( t ) = , de unde g −1 ( t + 1) = f −1 ( t ) . 4. EcuaŃia f(x) = y se reduce la x – [x] = y, (1). Pentru x ∈ (k, k +
t ( t − 1)
+ 1), ecuaŃia (1) revine la x – k = y, x = y + k, care este unică pentru (∀) k ∈ Z, k fixat, y ∈ [0, 1). De aici f este
bijectivă, deci inversabilă şi f–1: [0, 1) → [k, k + 1), f–1(y) = y + k. 5. Cum f: R → R este impară, rezultă că
f(–x) = –f(x), (∀) x ∈ R, iar din faptul că f este inversabilă, rezultă că din f(x) = y, x = f–1(y). Fie f–1: R → R.
Avem f–1(–y) = f–1(–f(x)) = f–1(f(–x)) = (f–1  f)(–x) = –x = –f–1(y), ceea ce arată că f–1 este impară. 6. Fie f: R →

291
→ R strict crescătoare. Evident f este injectivă şi cum f este şi surjectivă, urmează că f este bijectivă, deci
inversabilă. Fie y1 < y2, y1, y2 ∈ R. Cum f surjectivă, (∃) x1, x2 ∈ R astfel încât f(x1) = y1, f(x2) = y2. Avem deci
f(x1) < f(x2) şi cum f strict crescătoare, urmează că x1 < x2, adică f–1(y1) < f–1(y2). 7. Dacă în prima relaŃie
înlocuim x cu x – 1, obŃinem: 2f(1 – 2x) + g(–8x – 7) = 2(–6x + 1). Rezolvăm acum sistemul:
2f (1 − 2x ) + g(−8x − 7) = 2(−6 x + 1)
 . Găsim f(–2x + 1) = –4x + 3, respectiv g(–8x – 7) = –4x – 4, de unde
f (1 − 2 x ) ⋅ g(−8x − 7) = 7
x −1
f(x) = 2x + 1, g( x ) = . Cum f  g = 1R şi g  f = 1R, rezultă că f şi g sunt inverse una alteia.
2

II.1.6. FUNCłII PERIODICE


π π
1. a) RelaŃia a1 cos b1x + a 2 cos b 2 x = 0 , având loc (∀) x ∈ R, ea este adevărată şi pentru x = sau x =
b1 b2
b1 2π
obŃinem a1 = 0. b) Presupunem ∈ Q, de unde b1, b2 ∈ N*, (b1, b2) = 1. Alegem T = , unde d = (b1, b2) =
b2 d
 2π   2π 
c.m.m.d.c. al lui b1 şi b2. Atunci f(x + T) = a1 cos b1  x +  + a 2 cos b 2  x +  = a1 cos b1x + a 2 cos b 2 x =
 d   d 
= f(x), T > 0. Reciproc: presupunem că f este periodică de perioadă T > 0. Atunci f(x + T) = f(x), de unde
a1cos(b1x + b1T) + a2cos(b2x + b2T) = a1cosb1x + a2cosb2x, de unde a1[cos(b1x + b1T) – cosb1x] + a2[cos(b2x +
bT  b T b T  b T
+ b2T) – cosb2x] = 0, sau 2a1 sin 1 sin  b1x + 1  + 2a 2 sin 2 sin  b 2 x + 2  = 0 . Folosind a), avem
2  2  2  2 
b1T b 2T 2πk1 2πk 2 b1 k1
sin = sin = 0 , de unde b1 = iar b 2 = , cu k1, k2 ∈ Z. De aici = ∈ Q. 2. Luând părŃile
2 2 T T b2 k 2
întregi ale celor doi membri ai egalităŃii, avem [f(x + 2)] = [[f(x)] + {f(x + 1)}] = [[f(x)]] = [f(x)], (1), (∀) x ∈ R.
Luând părŃile fracŃionare ale celor doi membri ai egalităŃii, avem: {f(x + 2)} = {[f(x)] + {f(x + 1)}} = {{f(x +
+ 1)}} = {f(x + 1)}, (∀) x ∈ R, (2). În (2) înlocuind x cu x – 1, obŃinem: {f(x + 1)} = {f(x)}, (∀) x ∈ R, de unde
{f(x + 2)} = {f(x)}, (∀) x ∈ R, (3). Din (1) şi (3), adunând membru cu membru, obŃinem: [f(x + 2)] + {f(x +
+ 2)} = [f(x)] + {f(x)}, de unde f(x + 2) = f(x), (∀) x ∈ R, adică f periodică de perioadă T = 2. 3. Arătăm că orice
număr raŃional T ≠ 0 este perioadă a funcŃiei f, adică f(x + T) = f(x), (∀) x ∈ R, T ∈ Q*; i) Dacă x ∈ Q, x + T ∈
∈ Q şi deci f(x + T) = f(x) = a; ii) Dacă x ∈ R – Q, x + T ∈ R – Q şi deci f(x + T) = f(x) = b. Arătăm că dacă T
este iraŃional, atunci egalitatea f(x + T) = f(x), (1) nu are loc, (∀) x ∈ R. Într-adevăr, presupunând (1) adevărată
şi pentru T ∈ R – Q, (∀) x ∈ R, alegând x = –T, obŃinem f(0) = f(–T), adică a = b, fals. 4. Din relaŃia dată
1 1 1 1 1
obŃinem f ( x + 2) = = =1− , iar f(x + 3) = = = f ( x ) , (∀)
1 − f ( x + 1) 1 − 1 f (x) 1 − f ( x + 2) 1 − 1 + 1
1 − f (x) f (x)
x ∈ R. Rezultă că f(x + 3) = f(x), (∀) x ∈ R, adică f este o funcŃie periodică de perioadă 3. 5. Efectuând în relaŃia
dată substituŃia x → x + 1, obŃinem: f ( x + 2) + f ( x ) = 2f ( x + 1) = 2f ( x ) − 2f ( x − 1) , deci f(x + 2) = f(x) –
− 2f ( x − 1) , (1). De aici f(x + 4) = f(x + 2) − 2f ( x + 1) , (2). Imediat f(x + 4) = f ( x ) − 2[f ( x + 1) + f ( x −
−1)] = −f ( x ) , deci f(x + 8) = –f(x + 4) = f(x), adică f este periodică cu T = 8. 6. RelaŃia dată se mai scrie f2(x –
b2 b2 b2
– a) – + f 2 (x + a ) − = 0 , (1). Notând g( x ) = f 2 ( x − a ) − , relaŃia (1) devine g(x) + g(x + 2a) = 0, de
2 2 2
unde g(x + 2a) = –g(x). Imediat g(x + 4a) = –g(x + 2a) = g(x), de unde rezultă că funcŃia g este periodică cu

292
b2 b2
perioada T = 4a. De aici g(x + 4) = f2(x + 3a) − . Cum g(x) = g(x + 4a), obŃinem f 2 (x − a) − = f 2 (x + 3a) −
2 2
b2
− , de unde f(x – a) = f(x + 3a), de unde f(x) = f(x + 4a), (∀) x ∈ R, de unde rezultă că f este periodică cu
2
f (x ) − 1 f ( x + 1) − 1 1
perioada T = 4a. 7. Din relaŃia din enunŃ avem f ( x + 1) = , de unde f(x + 2) = =− .
f (x ) + 1 f ( x + 1) + 1 f (x)
1
Imediat f ( x + 4) = − = f ( x ) , de unde rezultă că funcŃia f este periodică, de perioadă 4. Mai trebuie să
f ( x + 2)
arătăm că f(x) ∈ R – {–1, 0}, (∀) x ∈ R. Metoda folosită va fi reducerea la absurd. Presupunem că (∃) x0 ∈ R,
astfel încât f ( x0 ) = −1 . ObŃinem f(x0 + 1) ⋅ f(x0) + f(x0 + 1) + 1 = f(x0), de unde f(x0) = 1, absurd. Analog dacă
(∃) x1 ∈ R astfel încât f(x1) = 0, atunci f(x1 + 1) ⋅ f(x1) + f(x1 + 1) + 1 = f(x1), de unde f(x1 + 1) = –1, absurd (vezi
anterior).

II.1.7. FUNCłII CONVEXE


 x + y  f ( x ) + f ( y) y+z
1. Presupunem f convexă în sens Jensen. Atunci, (∀) x, y, z ∈ A avem: f  ≤ , (1); f  ≤
 2  2  2 
f ( y) + f ( z )  z + x  f (z) + f ( x )
≤ , (2); f  ≤ , (3). Adunând membru cu membru (1), (2), (3), obŃinem că
2  2  2
x+y y+z z+x
f  + f  + f  ≤ f ( x ) + f ( y) + f (z) , (1). Presupunem (1) acum adevărată. Făcând z = y,
 2   2   2 
x+y  x + y  f ( x ) + f ( y)
obŃinem 2f   + f ( y) ≤ 2f ( y) + f ( x ) , de unde f  ≤ , ceea ce arată f semiconvexă. 2. Fie
 2   2  2
f, g ∈ M şi p, q ≥ 0, p + q = 1. Trebuie să arătăm că h = pf + qg ∈ M. Fie x1, x2 ∈ I şi λ ∈ [0, 1]. Atunci h(λx1 +
+ (1 – λ)x2) = pf(λx1 + (1 – λ)x2) + qg(λx1 + (1 – λ)x2) ≤ p[λf(x1) + (1 – λ)f(x2)] + q[λg(x1) + (1 – λ)g(x2)] =
= λ[pf(x1) + qg(x1)] + (1 – λ)[pf(x2) + qg(x2)] = λh(x1) + (1 – λ)h(x2), adică ceea ce trebuia demonstrat.

3. Presupunem că f nu este convexă, deşi este semiconvexă mărginită superior. Atunci există a, b ∈ I, α0 ∈
1 
∈ (0, 1) –   cu f((1 – α0)a + α0b) > (1 – α0)f(a) + α0f(b), (1). Putem presupune că f(a) = f(b) = 0, în caz
2
( t − a )f ( b ) − ( t − b ) f ( a )
contrar se consideră funcŃia g: I → R, g(t) = f(t) – . Din (1) obŃinem că f((1 – α0)a +
b−a
 1
+ α0b) > 0; i) Dacă α0 ∈  0,  , luăm α1 = 2α0 ∈ (0, 1). Notăm cu t0 = (1 – α0)a + α0b, t1 = (1 – α1)a + α1b.
 2
t + a  1 − 2α 0 1  2α 0 b  t + a  f ( t 1 ) + f (a ) f ( t 1 )
Avem: 1 = + a + = (1 – α0)a + α0b = t0 şi deci f ( t 0 ) = f  1 ≤ = , de
2  2 2 2  2  2 2
1  t +b 1
unde f(t1) ≥ 2f(t0), (2); ii) Dacă α0 ∈  ,1 , atunci notând cu α1 = 2α0 – 1 şi 1 = [(1 − 2α 0 + 1)a + (2α0 –
 2  2 2
 t + b  f ( t1 ) + f ( b ) f ( t1 )
– 1 + 1)b] = (1 – α0)a + α0b = t0 şi deci f ( t 0 ) = f  1 ≤ = , de unde f(t1) ≥ 2f(t0), (3). Din
 2  2 2

293
1 
(2) şi (3) am arătat că există α1 ∈ (0, 1) –   cu f(1 – α1)a + α1b) ≥ 2f(t0). Inductiv, arătăm că există αn ∈
2
1 
∈ (0, 1) –   , cu f(1 – αn)a + αnb) ≥ 2nf(t0), ceea ce contrazice faptul că f este mărginită superior.
2
f ( t ) − f (a )
4. Considerăm funcŃia ra: I – {a} → R, ra ( t ) = , (1). Alegem t1, t2, t3 ∈ I, cu t1 < t2 < t3. Cum f nu este
t−a
monotonă, presupunem că nu este crescătoare. Atunci f(t1) > f(t2) < f(t3). Urmează că există un t0 ∈ [t1, t3] astfel
încât t0 este punct de minim local pentru f. Vom arăta că t0 este punct de minim global, adică f(t) ≥ f(t0), (∀) t ∈
f ( t ) − f ( t 0 ) f ( t1 ) − f ( t 0 )
∈ I; i) Dacă t < t1, atunci rt 0 ( t ) ≤ rt 0 ( t1 ) , de unde ≤ ≤ 0 şi deci f(t) ≥ f(t0); ii) Dacă t >
t − t0 t1 − t 0
f (t) − f (t 0 ) f ( t 3 ) − f (t 0 )
> t3, atunci rt 0 ( t ) ≥ rt 0 ( t 3 ) rezultă ≥ ≥ 0 şi deci f(t) ≥ f(t0). Dacă a < b < t0, atunci din
t − t0 t3 − t0
f ( b) − f (a ) f ( t 0 ) − f (a )
ra(b) ≤ ra(t0) rezultă ≤ ≤ 0 şi deci f(b) ≤ f(a), adică f este descrescătoare pe I ∩ (–∞, t0).
b−a t0 − a
f ( t 0 ) − f ( b) f ( b) − f (a )
Dacă t0 < a < b, atunci rb(t0) ≤ rb(a), avem că: 0 ≤ ≤ şi deci f(b) ≥ f(a), adică f este
t0 − b b−a
 f ( x ) + f ( y)    x + y 
crescătoare pe I ∩ [t0, ∞). 5. i) Avem h(x) + h(y) = g(f(x)) + g(f(y)) ≥ 2g  ≥ 2g f    =
 2    2 
x+y  f ( x ) + f ( y)    x + y  x+y
= 2h   ; ii) h(x) + h(y) = g(f(x)) + g(f(y)) ≤ 2g  ≤ 2g f    = 2h   . 6. i) h(x) +
 2   2    2   2 
 f ( x ) + f ( y)    x + y  x+y
+ h(y) = g(f(x)) + g(f(y)) ≤ g  ≤ g f    = h  ; ii) h(x) + h(y) = g(f(x)) + g(f(y)) ≥
 2    2   2 
 f ( x ) + f ( y)    x + y  x+ y −1 −1
≥ 2g   ≥ 2g f    = 2h   . 7. Fie f(x1) = y1, f(x2) = y2. Atunci x1 = f ( y1 ), x 2 = f ( y 2 ) .
 2    2   2 
Cum f convexă, f(λx1 + (1 – λ)x2) ≤ λf(x1) + (1 – λ)x2, λ ∈ [0, 1]. De aici f–1(f(λx1 + (1 – λ)x2) ≤ f–1(λf(x1) + (1 –
– λ)x2); λx1 + (1 – λ)x2 ≤ f–1(λf–1(y1) + (1 – λ)f–1(y2)) sau λf–1(y1) + (1 – λ)f–1(y2) ≤ f–1(λf–1(y1) + (1 – λ)f–1(y2)),
de unde f–1 concavă, f–1 crescătoare evident!

II.1.8. ALTE TIPURI DE PROBLEME CU FUNCłII


z 3 − z1
1. Dacă z1, z2, z3 distincte două câte două, atunci A(z1), B(z2), C(z3) sunt coliniare dacă şi numai dacă ∈
z 2 − z1
f (z 3 ) − f (z1 ) az 3 + b − az1 − b z 3 − z1
∈ R*. Notând M(f(z1)), N(f(z2)), P(f(z3)), avem: = = ∈ R*, deci punctele
f (z 2 ) − f (z1 ) az 2 + b − az1 − b z 2 − z1
1  1
  z + , z ≠ 0 1
M, N, P sunt coliniare. 2. i) Fie f: C → C, f (z ) =  2  z . Din f(z) = z deducem, fie z = 0, fie z = ,
0, z
 z=0
deci z ∈ {–1, 0, 1}. Prin urmare f(M) = M echivalent cu M ∈ {O, A(1), B(–1)}; ii) Fie M(z) ∈ C(O, 1).
Deducem |z| = 1, iar z are forma trigonometrică z = cos t + isin t, t ∈ [0, 2π]. Rezultă f(z) = cos t, deci
f(C(O, 1)) = [AB]; iii) AplicaŃia f nu este injectivă, de exemplu f(z) = f (z) dacă |z| = 1. 3. a) Calcul direct;

294
x+y
b) Vom parcurge în etape rezolvarea acestei probleme. Mai întâi câteva notaŃii. Din a), = a (1 − a ) x + a ⋅
2
x+ y  x+y x+y x+y
⋅  + (1 − a ) ay + (1 − a )  , (1). Notând X = (1 – a)x + a ⋅ , Y = ay + (1 – a) ⋅ , atunci (1) se
2   2  2 2
x+y x+y
transcrie = aX + (1 − a )Y , de unde f   = f (aX + (1 − a )Y ) , (2).
2  2 
 x + y  f ( x ) + f ( y)
Etapa 1. Demonstrăm că f  = se poate generaliza sub forma:
 2  2
 x + x 2 + x 3 + ... + x n −1 + x n  f ( x1 ) + f ( x 2 ) + ... + f ( x n −1 ) + f ( x n )
f 1 = , (3). DemonstraŃie se face inductiv. O
 n  n
lăsăm ca exerciŃiu!
Etapa 2. Rezolvarea propriu-zisă. Vom arăta că A = B reducând problema la dubla incluziune; i) B ⊂ A şi
ii) A ⊂ B; i) B ⊂ A. Fie f ∈ B. Atunci f(ax + (1 – a)y) = af(x) + (1 – a)f(y), (∀) a ∈ (0, 1), (∀) x, y ∈ R. Dacă
1  x + y  f ( x ) + f ( y)
luăm a = , găsim f  = , ceea ce arată că f ∈ A; ii) A ⊂ B. Fie f ∈ A. Atunci, folosind (3)
2  2  2
 x + x 2 + x 3 + ... + x n −1 + x n  f ( x1 ) + f ( x 2 ) + ... + f ( x n −1 ) + f ( x n )
găsim că: f  1 = , de unde notând x1 = x2 = x3 =
 n  n
 mX + (n − m)Y 
= ... = xm = X şi xm+1 = xm+2 = ... = xn = Y, ultima inegalitate se transcrie f  =
 n 
mf (X) + (n − m)f (Y) m
= . Dacă notăm = a , obŃinem a ∈ (0, 1) ∩ Q. De aici se transcrie f(aX + (1 – a)Y) =
n n
= af(X) + (1 – a)f(Y), ceea ce arată că f ∈ A. 4. Din h: R → R bijecŃie, atunci h este şi surjecŃie. De aici: există
x0 ∈ R astfel încât h(x0) = 0, de unde f(x0) = x0. Pe de altă parte, cum (f  g)(x0) = (g  f)(x0), obŃinem f(g(x0)) =
= g(f(x0)) = g(x0). Pe de altă parte h(g(x0)) = g(x0) – f(g(x0)) = 0 şi cum h şi injectivă, avem h(g(x0)) = h(x0) = 0
găsim g(x0) = x0. 5. Avem f(x) = 0 este echivalent cu [x2 + x + 1] = 0, (1). Cum x2 + x + 1 > 0, (∀) x ∈ R,
urmează că 0 ≤ x2 + x + 1 < 1 este condiŃie echivalentă cu (1). De aici x ∈ (–1, 0). Pe de altă parte f(x) =
[ x 2 + x + 1] x 2 + x + 1 x 1 3 x 1
= ≤ = 1+ 2 ≤ 1 + = . (Am folosit 1 + x2 – 2x ≥ 0, de unde 2 ≤ ). 6. Scriem
x2 +1 x2 +1 x +1 2 2 x +1 2
relaŃia din enunŃ pentru x, εx, ε2x şi adunăm membru cu membru cele trei egalităŃi, rezultă că 2[f(x) + f(εx) +
+ f(ε2x)] = (1 + ε + ε2)x = 0. Cum f(εx) + f(ε2x) = εx, (∀) x ∈ C, găsim că f(x) = –εx, (∀) x ∈ C. 7. Înlocuind z
cu –z în relaŃia de condiŃie, găsim: f(–z) + f(z) = z2 – f(–iz), (∀) z ∈ C, (1). Din (1) şi relaŃia de condiŃie,
obŃinem f(iz) = f(–iz), (2), (∀) z ∈ C, de unde, înlocuind z cu –iz în (2) găsim f(z) = f(–z), (3), (∀) z ∈ C.
Ipoteza se rescrie astfel: 2f(z) = z2 – f(iz), (4), (∀) z ∈ C. Înlocuind în (4) z cu iz, găsim 2f(iz) = –z2 – f(–z),
(∀) z ∈ C. Imediat 2f(iz) = –z2 – f(z), (∀) z ∈ C, (5). Din (4) f(iz) = z2 – 2f(z), (6). Din (5) şi (6) 2z2 – 4f(z) =
z
= –z2 – f(z), duce, în final, la f(z) = z2. 8. Evident f(z) ∈ Im z. Într-adevăr, fie z ∈ C, atunci f (z ) = , iar
1+ | z |
|z|
| f (z) |= < 1 , (∀) z ∈ C, ceea ce arată că f(z) ∈ Im f. Arătăm şi reciproc: (∀) y ∈ Imf, (∃) z ∈ C astfel
1+ | z |
z
încât y = f(z). Din ultima relaŃie găsim = y , (1), de unde z = y(1 + |z|), (2). Aplicând în (1) modulul,
1+ | z |

295
|y|  | y| 
găsim |z| = |y| + |y| ⋅ |z|, de unde |z| = , (3). Folosim (3) în (2) şi găsim că z = (1 + |z|)y = 1 +  ⋅ y =
1− | y |  1− | y | 
y
= . Cum |y| < 1, 1 – |y| ≠ 0, deci z ∈ C există. Aceasta încheie demonstraŃia faptului că Imf = {z ∈ C |
1− | y |
| |z| < 1}. 9. Avem: f(x) = (x2 + 5x)2 + 10(x2 + 5x) + 24. Fie funcŃiile g(x) = x2 + 5x, x ∈ R şi h(x) = x2 + 10x +
25  25 
+ 24, x ∈ R. Atunci f(x) = (h  g)(x), (∀) x ∈ R. Observăm că min (g( x )) = − , de unde g(R) = − , + ∞  .
x∈R 4  4 
  25    25 
Analog min (h ( x )) = −1 , rezultând că h  − , ∞   = [−1, ∞) . FuncŃiile g: R → − 4 , + ∞  şi
x∈R  4   
 25 
h: − , + ∞  → [–1, ∞) sunt surjective, de unde rezultă că şi funcŃia h  g: R → [–1, ∞) este surjectivă, de
 4 
unde (h  g)(R) = [–1, ∞). Rezultă că f(R) = [–1, ∞). 10. i) Fie z1, z2 ∈ C şi s ∈ [0, 1]. Atunci, folosind
convexitatea funcŃiei pătratice f: R → R, f(x) = x2, obŃinem: |a(sz1 + (1 – s)z2)2| = |a| ⋅ |sz1 + (1 – s)z2|2 ≤ |a|(s|z1| +
+ (1 – s)|z2|)2 ≤ |a|(s|z1|2 + (1 – a)|z2|2), (1). Analog, prin calcul şi folosind proprietăŃile modulului găsim:
|b(sz1 + (1 – s)z2 + c)| ≤ s|bz1 + c| + (1 – s)|bz2 + c|, (2). Din (1) şi (2) obŃinem f(sz1 + (1 – s)z2) ≤ sf(z1) + (1 –
– s)f(z2); ii) Dacă z1 ∈ C* cu |z1| < 1, atunci z1 = ρ1(cos α1 + isin α1), α1 ∈ (0, 2π] şi 0 < ρ1 < 1. Atunci există r1,
r2 ∈ (0, 1) astfel încât r1 < ρ1 < r2. Fie w1 = r1(cos α1 + isin α1) şi w2 = r2(cos α2 + isin α2). Atunci există un s ∈
∈ (0, 1) astfel încât z1 = sw1 + (1 – s)w2 şi din (i) f(z1) < sf(w1) + (1 – s)f(w2) ≤ max{f(w1), f(w2)}. Luând z2 = w1
sau w2 găsim f(z1) ≤ f(z2), ceea ce rezolvă problema.

II.2. FuncŃii particulare

II.2.1. FUNCłIA EXPONENłIALĂ


1. Dacă y = x, rezultă f(2x) ≥ f2(x) ≥ a2x > 0, (∀) x ∈ R, adică f(x) > 0, (∀) x ∈ R. Dacă y = 0 găsim f(x) ≥
≥ f(x)f(0), de unde f(0) ≤ 1, (1). Dacă y = –x, găsim f(0) ≥ f(x)f(–x) ≥ a0 = 1, adică f(0) ≥ 1, (2). Din (1) şi (2)
avem f(0) = 1, (3). Pe de altă parte pentru y = 0, obŃinem: f(x)f(0) ≥ ax, de unde Ńinând cont de (3), găsim f(x) ≥
1 1
≥ ax, (4). Cum 1 = f(0) ≥ f(x) ⋅ f(–x), găsim că f(–x) ≤ ≤ x = a − x , (5). Din (4) şi (5), f(x) = ax.
f (x ) a
(m2 − 1)a + m
2. Punem condiŃia 2
> 0 , (∀) a ∈ R, urmează că ecuaŃiile (m2 – 1)a + m = 0 şi ma2 + a + m + 1 = 0
ma + a + m − 1
x
 1 
nu trebuie să aibă soluŃie. Din aceste condiŃii găsim m = –1. FuncŃia va fi deci: f(x) =  2  . 3. Pentru
a −a +2
x = 0, din a) găsim f(0) ≤ 1, (1). Dacă y = 0, din b) găsim f(x) ≤ f(x)f(0) şi cum din definiŃia lui f, f(x) > 0, (∀)
x ∈ R, găsim f(0) ≥ 1, (2). Din (1) şi (2) imediat, f(0) = 1. Luând y = –x găsim f(0) = 1 = f(x – x) ≤ f(x)f(–x), de
1 1
unde f(–x) ≥ ≥ = 2010 − x , (3). Tot din a) f(–x) ≤ 2010–x, (4). Urmează că f(x) = 2010x. 4. Din
f (x ) 2010 x
f(f(f(x))) = f(2x–1) şi f(f(f(x))) = 2f(x)–1 urmează că f(2x–1) = 2f(x)–1 sau 2f(2x–1) = 2f(x), (1). Pentru x = 1, din (1)
găsim 2f(1) = 2f(1), (2), iar pentru x = 2, tot din (1) 2f(2) = 2f(2), (3). Din (2) şi (3) analizând ecuaŃia 2x = 2x care
are numai soluŃiile {1, 2}, găsim {1, 2} = {f(1), f(2)}. 5. Dacă y = 1, obŃinem f(x + 1) = af(x) + f(1), (1), iar dacă

296
x = 1 obŃinem f(y + 1) = ayf(1) + f(y), (2), (∀) y ∈ R. Dacă în (2) luăm x = y găsim f(x + 1) = axf(1) + f(x), (3).
ax −1
Din (1) şi (3) găsim af(x) + f(1) = axf(1) + f(x), de unde f ( x ) = f (1) . 6. Făcând x = y = 0, găsim 0 ≤ 2f(0) ≤
a −1
≤ f(0), de unde f(0) = = 0. Pentru y = 0 rezultă x ≤ 2xf(x) ≤ 2xf(x), de unde f(x) ≥ x2–x, (1). Pentru y = –x rezultă
0 ≤ 2xf(x) + 2–xf(–x) ≤ ≤ 0. ObŃinem imediat 2xf(x) + 2–xf(–x) = 0. Urmează că 2xf(x) = –2–xf(–x). De aici f(x) =
= –2–2xf(–x), (2). Din (2) revenind în (1), obŃinem: –2–2xf(–x) ≥ x2–x sau f(–x) ≤ –x ⋅ 2x, (2). În (2) înlocuind x cu
–x, găsim f(x) ≤ x ⋅ 2–x, (3). Din (1) şi (3) găsim f(x) = x ⋅ 2–x. 7. Luăm x = 0 obŃinem în i) că 1 = f2(0) = f(x) ⋅ f(–y),
f (ky) f (( k − 1) y)
de unde f(–y) = [f(y)]–1, (∀) y ∈ Q, (*). Luând x = ky condiŃia ii) se transcrie = , (1). Luăm în
f ((k + 1) y) f (ky)
f (ny) 1
(1) pe rând k = 1, 2, 3, ..., n şi înmulŃind egalităŃile obŃinem = , (2). Luând în (2), n = 1, 2, 3, ..., m şi
f ((n + 1) y) f ( y)
f ( y) 1
înmulŃind egalităŃile, obŃinem: = , de unde f(my) = [(f(y))]m, (3). RelaŃia (3) pentru y = 1 ne dă:
f ((m + 1) y) [f ( y)]m
f(m) = [(f(1))]m, (∀) m ∈ N, au dacă notăm a = f(1) găsim f(m) = am, (∀) m ∈ N, (4). Tot relaŃia (3) ne dă pentru
m m n
n   n    n  n
y = , m, n ∈ N, m ≠ 0, f (n ) = f   , (5). Din (4) şi (5) obŃinem a n = f   , de unde a m = = f   . În
m   
m   
m m
concluzie, (∀) x ∈ Q+, f(x) = a , (∀) x ∈ Q+. Folosind (*) şi ultima egalitate, găsim f(x) = a , (∀) x ∈ Q.
x x

II.2.2. FUNCłIA LOGARITMICĂ


x
2x −1 1
1. CondiŃiile de existenŃă: x ∈ N, 2 ≤ x ≤ 2008, x
> 3− x , (1). Se consideră funcŃia f: R → R, f(x) =   +
2 +1 3
x x
1 1
+   +   , strict descrescătoare pe [1, ∞). CondiŃia (1) revine la f(x) < 1, îndeplinită pentru x ≥ 2. În
2 6
concluzie logaritmul are sens pentru x ∈ {2, 3, 4, ..., 2008}. 2. Din b) găsim că f(x) ≥ log3x – 1, (∀) x ∈ (0, ∞),
(1). În a) înlocuind x cu 3x, găsim f(x) + 2 ≤ log33x, care conduce la f(x) ≤ log3x – 1, (2). Din (1) şi (2), f(x) =
= log3x – 1. Analog, din a) g(x) ≥ log3x + 1, (∀) x ∈ (0, ∞), (3). Din b), înlocuind x cu 3x, găsim g(x) ≤ log3x +
+ 1, (∀) x ∈ (0, ∞), (4). Din (3) şi (4) găsim g(x) = log3x + 1. 3. Pentru ca funcŃia f să fie definită (∀) x ∈ R,
trebuie ca mx2 – 2(m + 2)x + 9 > 0, (∀) x ∈ R, (1). Dacă m = 0, atunci (1) nu are loc. Urmează că m ∈ R – {0}.
Cazuri: i) m < 0; (1) nu are loc, (∀) x ∈ R; ii) m > 0 şi ∆ = 4(m + 2)2 – 36m < 0, de unde m ∈ (1, 4). 4. Înlocuind
x
x cu găsim f(x) ≤ log 2 x − 1 , (1). Pe de altă parte, f(x) ≥ log2x – 1, (2). Din (1) şi (2) găsim f(x) = log2x – 1.
2
5. Pentru y = 1 găsim: f(x) ≥ f(x) + f(1) ≥ ln x, de unde f(x) ≥ ln x, (∀) x > 0, iar f(1) ≤ 0, (2). Pentru x = 1, din
1 1
(1), avem f(1) ≥ 0, (3). Din (2) şi (3) obŃinem f(1) = 0. Pentru y = obŃinem 0 = f(1) ≥ f(x) + f   ≥ 0 , de
x x
1 1 1
unde f(x) = − f   , (4). Revenind în (1), găsim f   ≥ ln , de unde –f(x) ≥ –ln x, sau f(x) ≤ ln x, (5). Din (1)
 
x  
x x
şi (5) imediat f(x) = ln x, (∀) x > 0. 6. Pentru x = y = 1 obŃinem 0 ≤ 2f(1) ≤ f(1), de unde f(1) = 0. Dacă x = y,
1 1 1
2x2ln x ≤ 2xf(x), de unde f(x) ≥ xln x, (1), (∀) x > 0. Pentru y = , găsim 0 ≤ f ( x ) + xf   ≤ f(1) = 0, de
x x x

297
1 1 1
unde f ( x ) = − x 2 f   , (2). Folosind (2), avem: − x 2 f   ≥ x ln x , adică − xf   ≥ ln x , (3). Înlocuind în (3) x
x x x
1 1
cu , rezultă − f ( x ) ≥ ln x , de unde f(x) ≤ xln x, (6). Din (1) şi (4) concluzia f(x) = xln x. 7. Pentru x = y = 1,
x x
1 1 1 1 1
0 ≤ 2f(1) ≤ f(1), de unde f(1) = 0. Pentru y = obŃinem 0 ≤ xf(x) + f   ≤ 0 , de unde f   = − xf ( x ) ,
x x x x x
1 1 1
(1). Dacă y = 1, găsim ln x ≤ xf(x), (2). Din (1) şi (2), imediat ln x ≤ − f   , (3). În (3) înlocuim x cu , de
x x x
ln x
unde –ln x ≤ –xf(x) sau xf(x) ≤ ln x, (4). Din (2) şi (4), xf(x) = ln x, adică concluzia f(x) = , (∀) x > 0.
x

II.2.3. FUNCłII TRIGONOMETRICE


1. Imediat 2 sin 2 x(2 + cos x ) = 4 sin 2 x + sin 3x + sin x . RelaŃia se transcrie: (f(3x) – sin 3x) + 4(f(2x) –
– sin 2x) + (f(x) – sin x) = 0, (∀) x ∈ R, (1). Notăm g: R → [–2, 2], g(x) = f(x) – sin x, de unde |g(x)| ≤ |f(x)| +
+ |sin x| ≤ 2, (2). RelaŃia (1) se transcrie astfel g(3x) + 4g(2x) + g(x) = 0, (∀) x ∈ R, (3). Arătăm că relaŃia (3)
este adevărată doar dacă g ≡ 0, (4). Pentru a demonstra (4) presupunem că g ≠ 0. Pentru acest lucru presupunem
1
că (∃) x0 ∈ R astfel încât g(x0) ≠ 0, de unde evident |g(x0)| > 0. Din (3), aplicând modulul găsim |g(2x)| ≤ , (∀)
2
1
x ∈ R. Înlocuind 2x cu x, obŃinem din aproape în aproape |g(x)| ≤ k , (∀) k ∈ N*, (∀) x ∈ R. Cum |g(x0)| > 0,
2
1 1 1 1
(∃) n ∈ N* astfel încât 2 n −1 ≤ < 2 n , rezultând că n −1 ≥| g ( x 0 ) |≥ n , contradicŃie cu |g(x)| ≤ k , (∀)
| g( x 0 ) | 2 2 2
sin x + cos x
k ∈ N*. 2. Imediat! Luând x = y în relaŃia din enunŃ, găsim 2f(x) = sin x + cos x, de unde f(x) = = ,
2
sin x − cos x sin y − cos y
(1). łinând cont de (1) relaŃia dată se transcrie sub forma: g( x ) − = g ( y) − − , (∀) x,
2 2
sin x − cosx π 
y ∈ R. Dacă luăm y = 0, găsim g(x) = + g(0) . 3. Notăm sin t = x, t ∈  , π . Rezultă că x ∈ [0, 1] şi
2 2 
t = π – arcsin x. De aici f(x) = (π – arcsin x) = π(π – arcsin x) = (arcsin x) – πarcsin x. 4. i) Calcule şi se arată
2 2

 2π   4π   2π 
că f(x) + f  x +  + fx +  = 0 ; ii) Din i) urmează că cel puŃin unul din numerele f(x), f  x + ,
 3   3   3 
 4π 
fx +  este negativ şi deci există o infinitate de numere reale x pentru care f(x) ≤ 0. 5. i) Se arată că f(x) +
 3 
 2π   2π 
+ fx +  + fx −  = 3; ii) Din (1) şi faptul că f(x) > 0, evident f(x) < 3, (∀) x ∈ R. 6. Fie xn = cos nπx,
 3   3 
yn = sin nπx, an = xn + yn. Avem (xn + yn)2 + (xn – yn)2 = 2( x 2n + y 2n ) = 2 , de unde ( x n − y n ) 2 = 2 − ( x n − y n ) 2 =
= 2 – a 2n , (1). Deoarece {an | n ∈ N} este finită şi {bn | n ∈ N} este finită, din (1) unde bn = xn – yn. Dar xn =
1 1
= (a n + b n ) , y n = (a n − b n ) , adică mulŃimile {xn | n ∈ N} şi {yn | n ∈ N} sunt finite. Există deci p ≠ q astfel
2 2
încât xp = xq şi există r ≠ s aşa încât yr = ys. De aici cos pπx = cos qπx şi sin rπx = sin sπx, de unde rπy ± sπy ∈

298
∈ 2πZ, ceea ce arată că x, y ∈ Q. 7. a) 0 ∈ M implică sin 0 + cos 0 ∈ M, deci 1 = sin 2π + cos 2π ∈ M, de unde
3π 3π 3π
π ∈ M. Imediat –1 = sin π + cos π ∈ M şi cum –1 = sin 2 ⋅ + cos 2 ⋅ ∈ M, de unde ∈ M; b) Deoarece
4 4 4
x x x
sin x + cos x = sin 2 ⋅ + cos 2 ⋅ , combinând ii) şi iii) avem că dacă x ∈ M, atunci ∈ M şi, prin inducŃie,
2 2 2
x π 
∈ M, (∀) n ∈ N*. Cum π ∈ M, avem deci  n n ∈ N, n ≥ 2 care este o mulŃime infinită de numere iraŃionale
2n 2 
subunitare.

III.
II. EcuaŃii

III.1. EcuaŃii în numere întregi

1. Avem x2000 + y2000 + 1998 = 2000xy; rezultă că avem două posibilităŃi: i) x > 0, y > 0; ii) x < 0, y < 0. Avem
2000 2000 2000
+
x2000 + y2000 + 1 + ...
1 + 1 ≥ 2000 x y , adică x2000 = y2000 = 1, deci soluŃiile vor fi elementele mulŃimii
1998 ori

{(–1, –1), (1, 1)}. 2. Notăm f(x) = x2000 + 2000x. Fie x < 0, atunci evident f(x) ∉ Z. Dacă x = 0, f(0) = 1, absurd,
căci 1 < 2001. Cum f(1) = 2001, 2001 > 2001, fals. Dacă x = 2k, f(x) = 4(21998 ⋅ k2000 + 10002 ⋅ 2000k), număr
compus. Dacă x = 2k + 1, f(x) este compus. Urmează că dacă p > 2001 prim, ecuaŃia nu are soluŃii. 3. Dacă x1 +
+ x2 + ... + xn > 0, ecuaŃia devine x1(x1 – 1) + ... + xn(xn – 1) = 0, de unde x1, x2 , ... , xn ∈{0,1}, cel puŃin un xi
nenul. Dacă x1 + x2 + ... + xn < 0, obŃinem x1, x2 , ... , xn ∈{0, –1}, cel puŃin un xi nenul. 4. Dacă
x1 x 2 x x x
= = ... = n , am obŃine 1 + + ... + n = n , în contradicŃie n ≥ 2011. Apoi, folosind mediile, se arată că
x2 x3 x1 x2 x1
x1 + x 2 = a x1' + x '2 = b
se ajunge la situaŃia anterioară. 5. Cum x1, x2, x1' , x '2 ∈ N şi folosind Viète, , (1); , (2)
x1 x 2 = b x '2 x1' = a
rezultă a, b ∈ N. Adunând (1) şi (2) membru cu membru, găsim x1 + x2 + x1' + x '2 = x1x 2 + x1' x '2 sau (x1 – 1)(x2 – 1) +
+ ( x1' − 1)( x '2 − 1) = 2 . Cazuri: i) x1 = 0 conduce la a = b = 0 absurd; ii) x1 = 1 coduce la a = 6, b = 5; iii) toate
rădăcinile mai mari ca 1, imposibil.

III.2. EcuaŃii iraŃionale


1. Din condiŃiile de existenŃă obŃinem x ∈ [1, ∞). De aici avem x + [x] ∈ [2, ∞). Dar x − x ≤ 1, x + [x ] ≤ 1
 −1+ 5 
şi x + [x] ≤ 1, absurd. Urmează că ecuaŃia nu are soluŃie. 2. a) Din condiŃii găsim x ∈  , 1 . Avem şirul
 2 

de implicaŃii: x − 1 − x = 1 − x , de unde 2 x = 1 + 1 − x , adică 5x – 2 = 2 1 − x . În final obŃinem


16
ecuaŃia 25x2 – 16x = 0, cu soluŃia acceptabilă x = ; b) CondiŃii: 2x + 1 ≥ 2, 11 – 4x ≥ 2, 3x – 2 ≥ 2 şi 2x + 1,
25
11 – 4x, 3x – 1 ∈ N. Analiza condiŃiilor conduce la posibilele soluŃii x ∈ {1, 2}. Prin verificare x = 2 este

299
singura admisă. 3. EcuaŃia este echivalentă cu x = x + 1(1 − 2 x ) , x ≥ -1. Dacă x > 0, 1 − 2 x < 0 şi în

acest caz nu avem soluŃii. Dacă x < 0, 1 − 2 x > 0 şi în acest caz nu avem soluŃii. Urmează că singura soluŃie
care convine este x = 0. 4. Notăm a = 3 7x + 18 şi b = 3 x 2 − 6 x + 1 . Transcriem ecuaŃia sub forma
3
7x + 18 + 3 x 2 − 6x + 1 − 3 = 3 x 2 + x − 8 şi revenind la notaŃii, ecuaŃia se transcrie sub forma a + b – 3 =
= 3 a 3 + b 3 − 27 , de unde (a + b)(3ab – 9(a + b) + 27) = 0 sau 3(a + b)(a – 3)(b – 3) = 0 (am ridicat la cub şi
calcule!). Avem cazurile a + b = 0 sau a = 3 sau b = 3. După efectuarea calculelor în fiecare caz găsim
9
x = , x = 3 ± 35 . 5. Evident că soluŃiile ecuaŃiei, dacă există, ele sunt pozitive. Aplicând inegalitatea mediilor
7
x + k − 1 x + 1 + 1 + ... + 1 x + k −1 k2
obŃinem: = ≥ k
x ⋅ 1 ⋅ 1 ⋅ ... ⋅ 1 , de unde k ≥ x , (1). Din sumarea în inegali-
k k k
n n
x + k −1
tatea (1) membru cu membru, obŃinem: ∑
k =2
k
k
≥ ∑
k=2
k2
x , (2). Deoarece trebuie să avem egalitate în

inegalitatea (2), găsim x = 1.

III.3. EcuaŃii în mulŃimea numerelor complexe


b c
1. Fie z1, z2 rădăcinile ecuaŃiei az2 + bz + c = 0. Avem z1 + z2 = − , z1z 2 = . Din condiŃia |a + bε + cε2| ≤ |a|
a a
b c c b
obŃinem 1 + ε + ε 2 ≤ 1 , (1). Pe de altă parte expresia ε 2 + ε + 1 este egală cu z1z2ε2 - (z1 + z2) ε + 1 =
a a a a
= z1ε(z2ε – 1) – (z2ε – 1) = (z1ε – 1)(z2ε – 1). Revenind în (1), obŃinem |z1ε – 1| ⋅ |z2ε – 1| < 1, de unde |z1ε – 1| <
< 1, (2) sau |z2ε – 1| < 1, (3). Pe de altă parte, |z1| = |z1ε| = |z1ε – 1 + 1| ≤ |z1ε – 1| + 1 ≤ 1 + 1 = 2. Analog pentru
z2. 2. Fie z = x + iy, x, y ∈ R. RelaŃia devine (x2 – y2 – 3x + 2) + i(2xy – 3y) ≤ 0. De aici obligatoriu
 3
2xy − 3y = 0  y = 0 x = 2
 2 . De aici avem cazurile:  2 sau  . ObŃinem în final că z ∈
x − y − 3x + 2 ≤ 0 x − 3x + 2 ≤ 0
2
 9 − y2 − 9 + y ≤ 0
 4 2
3  z 2 − z1 z −z
∈ [1, 2] ∪  + iy y ∈ R  . 3. Fie punctele A(z1), B(z2), C(z3). RelaŃia Re = 0 conduce la arg 2 1 ∈
2  z3 − z 2 z3 − z2
 π 3π 
∈  ,  , adică faptul că triunghiul ABC este dreptunghic în A. Prin urmare relaŃia devine AB2 + AC2 = BC2
2 2 
x 2
z1 − z 2 z1 − z 2
sau |z1 – z2|2 + |z1 – z3|2 = |z3 – z2|2. Rezultă că x = 2 este soluŃie a inecuaŃiei. Dacă x > 2, <
z3 − z 2 z3 − z 2
x 2
z1 − z 3 z1 − z 3
respectiv < , de unde |z1 – z2|x + |z1 – z3|x < |z3 – z2|x. SoluŃia inecuaŃiei va fi [2, ∞). 4. Notăm
z3 − z 2 z3 − z 2
α+γ
α = arg a, β = arg b, γ = arg c. Atunci a = |a|(cos α + isin α), c = |c|(cos γ + isin γ). Cum β = , atunci b =
2

300
 α+γ α+γ  α+γ α+γ
= |b| ⋅  cos + i sin  = (| a | + | c |) cos + i sin  . Deci ∆ = b – 4ac = (|a| + |c|) ⋅ [cos(α +
2 2

 2 2   2 2 
2
 α+γ α+γ
+ γ) + isin(α + γ)] – 4|a| ⋅ |c| ⋅ [cos(α + γ) + isin(α + γ)] = (|a| – |c|)2 ⋅  cos + i sin  . De aici imediat
 2 2 
c γ−α γ−α  γ−α γ−α
z1 =  cos + i sin  iar z 2 = − cos + i sin  , de unde |z2| = 1. 5. Dacă a = 0, ecuaŃia devine
a 2 2   2 2 
bz2 + bz = 0 . Dacă b ≠ 0, evident că are o soluŃie de modul 1. Dacă b = 0, cu atât mai mult. Fie a ≠ 0. Se observă
1 1 1 1 
că dacă admite soluŃia z0, admite şi rădăcina . Cu alte cuvinte {z1, z2, z3} =  , ,  . Analizăm
z0  z1 z 2 z 3 
1 1
următoarele cazuri: i) există i = 1, 3 astfel încât z i = , de unde |zi| = 1; ii) nu există i = 1, 3 astfel încât z i = ,
zi zi
1 1 1 1
adică z i ≠ , (∀) i = 1, 3 . Deci z1 = , z 2 = , z3 = , de unde z1 ⋅ z 2 ⋅ z 2 ⋅ z 3 = 1 , de unde |z2| = 1.
zi z2 z3 z1

III.4. EcuaŃii exponenŃiale şi logaritmice


1. EcuaŃia se transcrie: (7 3 ) x + (83 ) x + (93 ) x = 3 ⋅ (7 ⋅ 8 ⋅ 9)x. Notând a = (73)x, b = (83)x, c = (93)x şi folosind
a+b+c 3
inegalitatea mediilor, găsim ≥ abc , inegalitate cu egalitate în cazul nostru. De aici, a = b = c, adică
3
343x = 512x = 729x, de unde rezultă x = 0 soluŃie unică. 2. Fie a = [log2x] = [log3x], a ∈ Z. ObŃinem imediat a ≤
≤ log2x < a + 1, a ≤ log3x < a + 1, deci 2a ≤ x < 2a+1 şi x ∈ [3a, 3a+1). Notăm Ia = [3a, 2a+1) şi Ja = [2a, 3a+1). Avem
x ∈ Ia, dacă a ≥ 0, x ∈ Ja, dacă a < 0. Pentru a ≥ 2, 3a > 2a+1, deci Ia = ∅, iar pentru a ≤ –3, 2a > 3a+1, Ja = ∅. În
concluzie soluŃia este x ∈ J–2 ∪ J–1 ∪ I0 ∪ I1. 3. Evident x, y ∈ (0, ∞). Vom arăta că sistemul are soluŃia unică
x = 3 log 3 x < 1
 . ObŃinem, presupunând că avem şi alte soluŃii: i) x ∈ (0, 3). Atunci  . ObŃinem
y = 2 log 3 x + log 2 y = 2
3x < 27
log2y > 1, de unde y > 2, (1). Pe de altă parte  x , 2y < 4, de unde y < 2, (2). RelaŃiile (1) şi (2) sunt
3 − 2 = 23
y

contradictorii. Deci nu există soluŃii x ∈ (0, 3); ii) Presupunem soluŃii x ∈ (3, ∞). Analog, înlocuind în cele două
x = 3 x 
ecuaŃii, ajungem la contradicŃie. Prin urmare  soluŃie unică. 4. a) EcuaŃia se transcrie log n  + n − 1 =
y = 2 n 
1 x  1
= n log n x . Avem, folosind inegalitatea mediilor, ⋅ (n − 1 + log n x ) ≥ n log n x , adică log n  + n − 1 ≤ ⋅
n  n  n
x  x
⋅(n − 1) log n x sau log n  + n − 1 ≤ log n n ⋅ n , de unde singura soluŃie, pe cazul de egalitate, x = n;
 n  n
1 1 2
b) Transcriem ecuaŃia sub forma: ( x −1)( x − 2) + ( x −1)( x − 3) = ( x −1)( x − 4) . Notând 2x–1 = a, a > 0, ecuaŃia se
2 2 2
1 1 2 1 1
transcrie x − 2 + x − 3 = x − 4 care conduce la 2 + = 2 sau 2a2 – a – 1 = 0, cu soluŃia a = 1, de unde x = 1.
a a a a a

301
z z
3  4
5. Considerăm funcŃia f: R → R, f (z) =   +   , strict descrescătoare. Urmează că este injectivă şi f(2) =
5 5
x2 x2
1 3 4
= , (1). Transcriind ecuaŃia sub forma   +  = 1 şi notând x2 = y şi Ńinând cont de (1), găsim y = 2, de
2 5 5
unde imediat soluŃii x ∈ {− 2 , 2 } . 6. Evident x, y ∈ R – {0}. Vom lua pe rând cazurile: i) x, y > 0. ObŃinem
1 1
4x +
16x + 2 x ≥ 2 2 x ≥ 2 2 4 = 8 . Analog pentru y. De aici, nu există soluŃii x, y > 0; ii) x, y < 0. Dacă x, y < 0,
1 1
x
16 + 16 + y
2x + 2 < 1 + 1 + 1 + 1 = 4 , deci nici în acest caz nu avem soluŃie; iii) x > 0, y < 0. Şi în acest caz,
y

analizând i) şi ii), obŃinem că ecuaŃia nu are soluŃii. 7. a) Fie t = log5x, x > 0. Atunci x = 5t. Cum x log 5 4 =
= 4log 5 x = 4 t , ecuaŃia se transcrie 4t + 5t = 7t + 2. FuncŃia f: R → R, f(t) = 4t + 5t este convexă pe R, deci ecuaŃia
f(t) = 7t + 2 are cel mult două soluŃii, anume t = 0, t = 1, de unde x = 1, x = 5; b) Transcriem ecuaŃia sub forma
t
b
b log a x + c log a x = a log a x . Din a2 = b2 + c2, b, c ∈ R*+ – {1}, obŃinem b, c < a. Considerăm funcŃia f(t) =   +
a
t loga x loga x
c b c
+   , t = logax. Evident f strict descrescătoare, de unde injectivă. Cum f(2) = 1, ecuaŃia   +  =
 
a a a
= 1 are singura soluŃie logax = 2, de unde x = a2. 8. a) Se observă că ( 4 − 15 )( 4 + 15 ) = 1 , (1). Notând
1
(4 + 15 ) x = z > 0 , găsim, Ńinând cont de (1) că z + = 62 , cu soluŃiile z ∈ {31 + 8 15 , 31 − 8 14 } . Găsim
z
(4 + 15 ) x = (4 + 15 ) 2 sau (4 + 15 ) x = (4 − 15 ) 2 cu soluŃiile x ∈ {–2, 2}; b) Din condiŃii x ∈ (8, ∞). Avem
log 2 (log 2 (log8 x ))
log 2 (log 4 (log8 x )) = sau [log 4 (log 8 x )]2 = log 2 (log8 x ) 2 . Schimbând din nou baza, găsim
2
egalitatea [log 2 (log 8 x )]2 = 4 log 2 (log 8 x ) , de unde log 2 (log8 x ) = 0 sau log 2 (log8 x ) = 4 . Găsim soluŃiile x ∈
∈ {8, 816}, singura care convine fiind x = 248. 9. Folosim relaŃia dintre inegalitatea mediei geometrice şi cea
2
 log 3 (1 + sin 2 x ) + log 3 (1 + cos 2 x ) 
 
aritmetică. ObŃinem 1 = log 3 (1 + sin 2 x ) ⋅ log 3 (1 + cos 2 x ) ≤  2 2
 =
2 2  2 
 
2
 2  2 2  
2
 log 3 (1 + sin x )(1 + cos x )   log 3  1 + sin x + 1 + cos x  
2 2
    2  
= 2
 ≤  2 
 = 1 . Avem deci cazul de egalitate în inega-
 2   2 
   
 
π   3π 
litatea binecunoscută, de unde log 3 (1 + sin 2 x ) = log 3 (1 + cos 2 x ) , cu soluŃia x ∈  + kπ ∪  + kπ  .
2 2 4 k∈Z 4  k∈Z
10. Punând condiŃia 3x – 1 > 0, găsim x > 0. Fie funcŃia f: R → (0, ∞), f(x) = log3(2x + 1), bijectivă, având
inversa f–1: (0, ∞) → R, f–1(x) = log2(3x – 1). Urmează că avem de rezolvat ecuaŃia f(x) = f–1(x). Cum f şi f–1 sunt
simetrice în raport cu prima bisectoare, deci valorile comune vor coincide pe dreapta y = x, când f(x) = f–1(x) = x.

302
x x
2 1
Rezolvăm log3(2x + 1) = x, ecuaŃie echivalentă cu   +   = 1 . Considerăm funcŃia g: R → (0, ∞),
 3 3
x x
2 1
g( x ) =   +   strict descrescătoare, deci injectivă. Cum 1 = g(1) = g(x), găsim unica soluŃie x = 1.
 3 3

III.5. EcuaŃii trigonometrice


π
1. EcuaŃia se transcrie 2cos2x + 2(sin y + cos y)sin x ⋅ cos x + sin2x = 0, sau, pentru x ≠ 2kπ ± , k ∈ Z, tg2x +
2
π
+ 2(sin y + cos y)tg x + 2 = 0, (1). Pentru ca (1) să aibă soluŃii reale, ∆ ≥ 0, de unde sin 2y = 1, adică y ∈  +
4
+kπ , k ∈ Z}. Atunci tg x = ± 2 , de unde x ∈ {±arctg 2 + lπ , l ∈ Z}. 2. Notăm x + 10° = α. EcuaŃia se
rescrie: tg 3α ⋅ tg(3α – 10°) ⋅ tg(3α + 10°) ⋅ tg(α – 5°) ⋅ tg(α + 5°) = − 3 sau tg3(x + 10°) ⋅ tg(3x + 20°) ⋅
⋅ tg(3x + 40°) = − 3 . Cum tg(3x + 30°) = tg 3α, tg(3α – 10°) ⋅ tg 3α ⋅ tg(3α + 10°) = − 3 , de unde tg 9α =
2π 2π π 3π π π π
= − 3 , adică 9α = + kπ , adică α = + k , cu x = + k , x = + k , k ∈ Z. 3. Întrucât [sin x] ∈
3 9 9 18 9 6 9
sin x < 0 0 ≤ sin x < 1 sin x = 1 . SoluŃia este:
∈ {–1, 0, 1}, avem de rezolvat sistemele: i)  ; ii)  ; iii) 
− 1 < tgx < 0 0 ≤ tgx < 1 1 ≤ tgx < 2
 π π 
∪  - 4 + 2kπ, 4 + 2kπ  . 4. Presupunând prin absurd că ecuaŃia admite soluŃii, obŃinem sin x = 1 şi sin(x – 1) = 0.
k∈Z
π 2
Se obŃine x = + 2kπ , k ∈ Z şi x −1 = lπ, l ∈ Z, de unde π = ∈ Q, absurd. 5. i) sin x sin 2 x ⋅
2 4 k − 2l + 1
1 1 1 1 3
⋅ sin 3x = sin 4x − cos 4x sin 2 x ≤ + = . De aici sin 4x = 1 şi sin 2xcos 4x = –1, (1) absurd, căci sin 4x = 1
4 2 4 2 4
x 1
duce la cos 4x = 0, absurd din (1); ii) Avem 2|sin x|⋅ sin 2 < 2 şi x 2 + 2 ≥ 2 ⇒ nu vom avea niciodată egalitate.
5 x

III.6. EcuaŃii funcŃionale


2010 x
1. Cum 2010 x = 2010 ≤ 2010 = 1005 , notând t = 2 , (1) găsim că domeniul său este [–1005, 1005]. Din
x2 +1 x+
1 2 x +1
x
2  2010  2
2000 2 20002 x 4 + 1  2010    − 2, | x | ≤ 1005 ,
(1) t = , de unde t = , de unde 2
=  − 2 . Imediat f(x) =  x 
1 1 x  t 
x+ 2
x + 2 +2 
x x C( x ), | x | > 1005
1
unde C este o funcŃie arbitrară. 2. Se obŃine f ( x ) = f ( x + 1) − , (1). ObŃinem f(0) = f(1)–
( x + 1)( x + 2)
1 1 1
− , f (1) = f (2) − , ..., f(x – 1) = f(x) – , (2). Adunând membru cu membru în (2), găsim f(x) =
1⋅ 2 2⋅3 x ( x + 1)

303
x x
= f(0) + . Făcând x = 2010, de unde f(0) = 0, în final f(x) = . 3. Pentru x = 0 obŃinem f(–y) ≤ 1, (∀)
x +1 x +1
1 f ( x − y)
y ∈ R, deci f(t) ≤ 1, (1). Conform cu enunŃul, f ( y) ≥ 1 − + , (∀) x > 0, (∀) y ∈ R. De aici deducem
x x
1 f ( y) 1 f ( y)
f(x – y) ≥ 1 − + , (∀) x > y. Folosind ipoteza, 1 − + − xf ( y) ≤ 1 − x şi presupunând f(t) < 1, găsim
x x x x
x2 < 1, absurd. Urmează că funcŃia căutată va fi f(t) = 1. 4. Pentru x = y găsim [f(x) – x – 1] ⋅ [f(x) + x – 1] = 0.
x + 1, x ∈ A
Avem deci f(x) =  , unde A şi B partiŃii ale lui R. Urmează că A = {0} sau B = {0}. ObŃinem
− x + 1, x ∈ B
1, x = 0 x + 1, x ∈ R *
f1(x) =  sau f2(x) =  , deci f1(x) = -x+1, x ∈ R sau f2(x) = x + 1, f2(x) = x + 1,
− x + 1, x ∈ R - {0} 1, x=0
x ∈ R. 5. Dacă f este o funcŃie cu proprietatea dată, atunci f(y + x) = max{f(y), x} + min{f(x), y}. Adunând cu
relaŃia de condiŃie, găsim 2f(x + y) = f(y) + x + f(x) + y, (∀) x, y ∈ R. Pentru y = 0 obŃinem f(x) = x ≠ f(0), (1).
Arătăm f(0) = 0. Notăm f(0) = k. Găsim x + y + k = max{y + k, x} + min{x + k, y}. Făcând x = 0, y = k, găsim
2k = k + max{0, 2k}. De aici ori 2k = k, ori 2k = 3k, de unde k = 0. FuncŃia căutată va fi deci f(x) = x.

III.7. EcuaŃii nestandard


1. Fie a soluŃia ecuaŃiei x ⋅ 2x = 1 (este unică, căci f: R*+ → R este strict crescătoare). Deoarece 2a > 0 şi a ⋅ 2a = 1
1
rezultă a > 0. Cum 2a > 1 urmează a ∈ (0, 1). De aici 2a < 2, (∀) a < 1 şi cum a ⋅ 2a = 1, obligatoriu a > . În
2
1  1
concluzie a ∈  , 1 . Pentru x = a, înlocuit în relaŃia de condiŃie, găsim f(a) – a ⋅ f2(a) ≥ − f (a ) , inegalitate
2  a
1  1 
echivalentă cu (a ⋅ f(a) – 1)2 ≤ 0, de unde f (a ) = = 2 a . În concluzie, a ∈  , 1 este soluŃie a ecuaŃiei f(x) = 2x.
a 2 
2. FuncŃia f: R → R, f(x) = 117–2x este descrescătoare, iar g: R → R, g(x) = 3x + 2 este crescătoare pe R. De aici
rezultă că graficele celor 2 funcŃii au cel mult un punct comun. Observăm că f(3) = g(3) = 11, de unde x = 3 este
2
unica soluŃie a ecuaŃiei. 3. Avem sin2x(1 + ctg2x) = 1, de unde log 4 2 = − y 2 + 6 y − 5 , (1). Din condiŃia
sin 2x
 π 
sin 2x > 0, x ∈ ∪
k =0
2 
2
 2kπ, + 2kπ  . Egalitatea (1) se transcrie sub forma − log 4 2 sin 2x = −( y − 3) . Cum 0 <

< sin 2x ≤ 1, − log 4 2 sin 2 x ≥ 0 , iar –(y – 3)2 ≤ 0. De aici obligatoriu (y – 3)2 = 0 şi log 4 2 sin 2 x = 0 , de unde y =
π π t
= 3, x = (−1) k + k , k ∈ Z. 4. Notăm t = x2 – 2011x. EcuaŃia se transcrie astfel: 2010t + = 1 , (1), de
4 2 2010 x
unde 2010t+x – 2010x = t. Avem 2010x(1 – 2010t) = t, (1). În (1) analizăm cazurile: i) t > 0, de unde 2010x(1 –
– 2010t) < 0, deci nu avem soluŃie; ii) t < 0, de unde 2010x(1 – 2010t) > 0, deci nici în acest caz nu avem soluŃie.
x
Urmează că singura soluŃie t = 0, de unde x ∈ {0, 2011}. 5. Cum 3x + 3–x + 1 ≥ 2 + 1 = 3 şi 3 cos ≤ 3 , (∀) x ∈
3
∈ R, rezultă că avem egalitate, adică x = 0.

304
GEOMETRIE
IV.1. AplicaŃiile numerelor complexe în geometrie
2
1. a) Avem (z – 1)2 + (z – ε)2 + (z − ε) 2 = 3z 2 + (1 + ε 2 + ε ) − 2(1 + ε + ε)z = 3z2 + (1 + ε2 + ε4) – 2(1 + ε + ε2)z =
= 3z2. (Am Ńinut cont că ε = ε 2 ) ; b) Alegem un sistem de axe cu originea în O, Ox = OA şi unitatea de măsură
OA = OB = OC. Punctele A, B, C, M au afixele 1, ε, ε respectiv z şi avem MA2 + MB2 + MC2 = |z – 1|2 + |z –
– ε|2 + | z − ε |2 ≥| (z − 1) 2 + (z − ε) 2 + (z − ε) 2 = |3z2| = 3|z|2 = 3OM2. 2. a) Din |z – zk| = |zk|, pentru k = 1 obŃinem
|z – z1|2 = |z1|2, de unde (z – z1) (z − z1 ) = z1 ⋅ z1 conduce la z ⋅ z = z1 ⋅ z + z1 ⋅ z , (1). Analog pentru k ∈ {2, 3, 4}
obŃinem relaŃii analoage lui (1) şi adunând aceste relaŃii membru cu membru, găsim 4z z = z(z1 + z 2 + z 3 + z 4 ) +
+ (z1 + z 2 + z 3 + z 4 )z , de unde ajungem la 2z z = 0 sau |z|2 = 0, adică |z2| = 0, de unde z2 = 0, adică z1 + z2 + z3 +
+ z4 = 0, cu alte cuvinte z = 0; b) Din a), cum z1 + z2 + z3 + z4 = 0, găsim z1 + z2 = –(z3 + z4), de unde (z1 +
+ z2)2 = (z3 + z4)2, (1) respectiv |z1 + z2|2 = |z3 + z4|2. Din ultima inegalitate găsim (z1 + z 2 )(z1 + z 2 ) = (z 3 + z 4 ) ⋅
z12 + z 22 z 32 + z 24
⋅ (z 3 + z 4 ) , iar după efectuarea calculelor, găsim z1 z 2 + z1z 2 = z 3 z 4 + z 3z 4 , echivalentă cu = .
z1z 2 z3z 4
Folosind (1) găsim z1z2 = z3z4, (2). Plecând de la z1 + z2 + z3 + z4 = 0 scrisă sub forma egalităŃii, z1 + z4 = –(z2 + z3) şi
urmând aceiaşi paşi, ajungem la z1z4 = z2z3, (3). Din (2) şi (3) găsim z1(z1 – z4) = z3(z4 – z2) sau (z1 – z3)(z2 – z4) = 0,
de unde z1 = z3 sau z2 = z4, ceea ce arată că punctele de afixe z1, z2, z3, z4 sunt vârfurile unui dreptunghi.
3. a) |z – z1|2 + |z – z2|2 + |z – z3|2 = (z − z1 )(z − z1 ) + (z − z 2 )( z − z 2 ) + y
+ (z − z 3 )(z − z 3 ) = |z|2 − z ⋅ z1 − z1 ⋅ z + | z1 |2 + | z |2 − z ⋅ z 2 − z 2 ⋅ z + A
2 2 2
+ | z 2 | + | z | −z ⋅ z 3 − z 3 ⋅ z + | z 3 | = 3|z| + 3 – z(z1 + z 2 + z 3 ) − z ⋅
2

⋅ (z1 + z 2 + z 3 ) = 3(| z |2 +1) (am Ńinut cont că z1 + z2 + z3 = 0 şi z1 + P N x


O
+ z 2 + z 3 = 0) ; b) Fie A(z1), B(z2), C(z3). Cum z1 + z2 + z3 = 0 şi |z1| = –1 1
= |z2| = |z3| = 1, atunci punctele A, B, C sunt vârfurile unui triunghi B M C
echilateral înscris în cercul de centru O(0, 0) şi rază 1. În ∆ABC
echilateral înscriem cercul C(O, r). Avem că AB = 3 , iar raza cercului
1  1 1
înscris va fi atunci r = . Cum M ∈ C  0,  şi dacă notăm M(z) afixul său, găsim |z| = . Pe de altă parte, MA =
2  2  2
 1  15
= |z – z1| şi analoagele, de unde MA2 + MB2 + MC2 = |z – z1|2 + |z – z2|2 + |z – z3|2 = 3 + 1 = (am Ńinut cont de a)).
4  2
4. Notăm cu literele mici afixele punctelor A, B, ... . Fie E mijlocul lui AC, F D
C
mijlocul lui BD, P mijlocul lui MN. Pe de altă parte, fie k1 > 0, astfel încât
M E
AM BN
= k1 , (1) şi k2 > 0 astfel încât = k 2 , (2). A arăta că punctele F, P, E P
MD NC *
A
coliniare, înseamnă a găsi α, β, γ, nu toate nule, astfel încât α + β + γ = 0 şi N
F
b+d m+n a +c
α ⋅ f + β ⋅ p + γ ⋅ e = 0, (3). Revenind la (1) găsim f = ,p = ,e = ,
2 2 2

B
305
BN k2
de unde (3) se transcrie α(b + d) + β(m + n) + γ(a + c) = 0, (4). Din (1) şi (2) găsim = şi
BC 1 + k 2
AM k1 AM BN 1
= şi înlocuind în + = 1 , găsim k1k2 = 1. Notând k1 = k ∈ R*+, atunci k 2 = . Atunci
AD k1 + 1 AD BC k
1
b+ c
a + dk k = c + kb , de unde m + n = b + d + + a + c . Revenind în (4) găsim că (α + kβ)(b + d)
m= , n=
1+ k 1 k +1 1+ k
1+
k
 β  β
+  + γ (a + c) = 0 . Alegând α = –kβ, γ = , β ≠ 0, β ∈ R, găsim cele trei valori pentru α, β, γ, care ne
1+ k  1+ k
arată că punctele P, E, F coliniare. 5. Ideea este de a găsi o condiŃie pentru care afixele punctelor M, N, P sunt
vârfurile unui triunghi echilateral. Fie deci a, b, c afixele punctelor A, B, C şi a', b', c' afixele punctelor A', B', C'.
2π 2π
Atunci a' = aε, b' = bε, c' = cε, unde ε = cos + i sin . Dacă notăm cu m, n, p afixele punctelor M, N, P,
3 3
aε + b bε + c cε + a
avem m = ,n = ,p = . Prin calcul, arătăm că m2 + n2 + p2 = mn + np + mp, care reprezintă
2 2 2
condiŃia echivalentă cu ∆MNP echilateral. 6. Fie A1, B1, C1 de afixe coliniare f(z1), f(z2), f(z3), iar A, B, C de
f (z1 ) − f (z 2 )
afixe z1, z2, z3. CondiŃia de coliniaritate a lui A1, B1, C1 se traduce prin relaŃia ∈ R. Atunci
f (z 2 ) − f (z 3 )
az1 + b − az 2 − b a (z1 − z 2 ) z − z2
= = = 1 , relaŃie ce traduce că A, B, C coliniare. 7. Se asociază fiecărui punct
az 2 + b − az 3 − b a (z 2 − z 3 ) z 2 − z3
a +c b+d
 . RelaŃia de demonstrat revine la |b – a| + |c – b| + |d – c| +
2 2 2
afixul său A(a), B(b), C(c), D(d), E , F
 2   2 
2
a+c b+d
+|a – d|2 = |c – a|2 + |d – b|2 + 4 ⋅ ⋅ − . Se foloseşte apoi proprietatea că z ⋅ z =| z |2 şi după efectuarea
2 2
calculelor, se obŃine rezultatul aşteptat.

IV.2. Alte probleme


1. Orice tetraedru regulat ABCD satisface condiŃiile din enunŃ. Fie E un punct egal depărtat de A, B, C, D.
Urmează că E este centrul sferei circumscrise tetraedrului ABCD. Dar EA ≠ AB, deci mulŃimea nu poate avea
mai mult de 4 puncte. 2. Evident acest lucru căci se foloseşte faptul că intersecŃia oricăror 4 sfere este nevidă.
n n
3. Notăm cu Sk aria cercului mare al sferei Sk şi avem 32 = ∑ 4S
k =1
k , deci ∑S
k =1
k = 8 . Raza oricărui cerc mare

n
π π
∑S
1 32
este cel mult , deci Sk ≤ , de unde 8 = n ≤n , de unde n ≥ > 10 , deci există 11 sfere. Proiectăm
2 4 k =1
4 π
diametrele sferelor pe o muchie. Există un punct comun tuturor acestor proiecŃii. Un plan perpendicular pe
1
această muchie, în acest punct va intersecta toate sferele. 4. Fie 6 sfere de rază cu centrele în A1, A2, A3, A4,
9

306
A5, A6 astfel încât M să fie un punct comun tuturor acestor sfere. Atunci distanŃa de la M la A1, A2, A3, A4, A5
1 1
este cel mult deci cele 5 puncte pot fi înglobate în sfere de centru M şi rază . Volumul celor 1331 sfere va
9 9
2
45  1  11 V
fi V = 1331 ⋅ ⋅   . Toate aceste sfere se află în interiorul cubului de latură . Cum > 4 , V' volumul
3 9 9 V'
cubului, deci există puncte acoperite de cel puŃin 5 sfere. 5. Separăm mulŃimea nodurilor laticeale în 8 clase, în
funcŃie de paritatea coordonatelor: M1(p, p, p), M2(p, p, i), M3(p, i, p), M4(i, p, p), M5(p, i, i), M6(i, p, i),
M7(i, i, p), M8(i, i, i), unde p = coordonată pară, i = coordonată impară. Din cele 9 puncte, cel puŃin două sunt de
acelaşi tip (clasă), deci mijlocul segmentului AB are şi el coordonate întregi.

307
CLASA A X I - A

ALGEBRĂ

1. a) Dacă X(1) = 1 ⇒ X(1) = X(X(1)) = σ(1) = 3, fals. Dacă X(1) = 2 ⇒ X(2) = X(X(1)) = σ(1) = 3. Atunci
 1 2 3
X(3) = X(X(2)) = σ(2) = 1 ⇒ X =   . Dacă X(1) = 3 ⇒ X(3) = X(X(1)) = σ(1) = 3, fals; b) Deoarece în
 2 3 1
S3 trei permutări sunt pare şi trei impare, dacă σ1, σ2, ..., σ6 sunt cele 6 permutări într-o ordine oarecare, avem
ε(σ1 ⋅ σ2 ⋅ ... ⋅ σ6) = (−1) m ( σ1 ) + m ( σ 2 ) + ...+ m ( σ 6 ) = −1 , deci σ1 ⋅ σ2 ⋅ ... ⋅ σ6 este permutare impară; 2. Dacă σ este
permutarea căutată, vom avea că Aσ ≥ Aσ⋅(k,1), oricare ar fi transpoziŃia (k, 1) ∈ Sn. Inegalitatea revine la: (ak –
– aσ(k))2 + (al – aσ(l))2 ≥ (ak – aσ(l))2 + (al – aσ(k))2, de unde obŃinem că (ak – al)(aσ(k) – aσ(l)) ≥ 0, (∀) l ∈ 1, n . Rezultă
că σ nu are inversiuni, prin urmare este permutarea identică; 3. Fie X ∈ S7 soluŃie. Atunci X(X(1)) = 4;
X(X(2)) = 7; X(X(3)) = 6; X(X(4)) = 5; X(X(5)) = 1; X(X(6)) = 3; X(X(7)) = 2. Fie a) X(1) = 1, atunci X(1) = 4,
fals; b) X(1) = 2, atunci X(2) = 4, X(4) = 7, X(7) = 5, X(5) = 2, deci X(1) = X(5), fals; c) Analog, X(1) = 3, 4, 6
sau 7 nu conduc la soluŃie; d) X(1) = 5, atunci X(5) = 4, X(4) = 1. Analizând X(2) = 2, 3, 6 sau 7, deducem
1 2 3 4 5 6 7
X(2) = 3, X(3) = 7, X(7) = 6, X(6) = 2 şi soluŃia X =   sau X(2) = 6, X(6) = 7, X(7) = 3,
5 3 7 1 4 2 6
1 2 3 4 5 6 7
X(3) = 2 şi soluŃia X =   ; 4. a) Fie k ∈ {2, 3, ..., n – 1} şi σ = (k, k + 1). Cum X(k) =
5 6 2 1 4 7 3
= X(σ(k + 1)) = σ(X(k + 1)), X(k + 1) = X(σ(k)) = σ(X(k)) şi X(k) ≠ X(k + 1), rezultă X(k), X(k + 1) ∈ {k, k +
+ 1}. Analog, considerând transpoziŃia (k – 1, k), obŃinem X(k) ∈ {k – 1, k}, deci X(k) = k şi X(k + 1) = k + 1.
Rezultă X(k) = k, k = 2, 3, ..., n, deci X(1) = 1 şi X = e; b) Fie ciclul σ = (1 2 ... n). cum σ(1 2) σ–1 = (2 3),
σ(2 3) σ–1 = (3 4), ..., σ(n – 2 n – 1) σ–1 = (n – 1, n), iar y comută cu (1 2) şi σ, rezultă că y comută cu
transpoziŃiile (1 2), (2 3), ..., (n – 1 n), deci y = e; 5. a) Fie x1, x2, ..., xn o permutare a elementelor mulŃimii A.
Dorim ca m = max{x1 + x2, x2 + x3, ..., xn–1 + xn} să fie minim. (x1 + x2) + (x2 + x3) + ... + (xn–1 + xn) = 2(x1 + x2 + ...
+ xn) – (x1 + xn) = 2(1 + 2 + ... + n) – (x1 + xn) = n(n + 1) – (x1 + xn). Cum x1 + xn ≤ n + n – 1 ⇒ (x1 + x2) + (x2 +
( x + x 2 ) + ( x 2 + x 3 ) + ... + ( x n −1 − x n ) 1
+x3) + ... + (xn–1 + xn) ≥ n(n – 1) + 1 ⇒ 1 ≥n+ . Dar cum m ≥ xi +
n −1 n −1
m + m + ... + m ( x1 + x 2 ) + ( x 2 + x 3 ) + ... + ( x n −1 − x n ) 1
+ xi+1, (∀) i ∈ 1, n − 1 ⇒ ≥ ≥n+ ⇒ m ≥ n + 1.
n −1 n −1 n −1
Putem arăta că acest minim este atins de următoarea ordonare: n, 1, n – 1, 2, n – 2, 3, ... . Se observă că suma
oricăror doi termeni consecutivi este, alternativ, n sau n + 1, deci maximul acestor numere este n + 1; b) Punctele
Mi au coordonatele Mi(2k – i + 1, i) şi atunci MiMi+1 = 2 şi toate punctele se află pe dreapta x + y – 2k – 1 = 0;
n (n + 1)  n (n + 2) n (n + 1) 
6. a) X(1) + X(2) + ... + X(n) = nI2 + A =  n (n + 1) n (1 − n )  . Observăm că X(a) ⋅ X(b) = X[(a +
2 − 
 2 2 
+1)(b + 1) – 1] şi atunci X(1) ⋅ X(2) ⋅ ... ⋅ X(n) = X[(1 + 1)(2 + 1)...(n + 1) – 1] = X[(n + 1)! – 1]; b) Deoarece
 n +1  n +1 n +1
X(1) + X(2) + ... + X(n) = nX  , ecuaŃia devine X  = X[(n + 1)!−1] ⇔ (n + 1)!−1 = ⇔
 2   2  2

308
a b 
(n + 1)(2n! – 1) = 2, cu soluŃia unică n = 1; 7. b) Matricele de forma   , a, b, c ∈ C, a2 + bc = 1 sunt din
c − a
a b 
M; c) Matricele de forma   , a, b ∈ C, verifică cerinŃa problemei; d) Folosind b) şi c) căutăm A, B, C
b − a
care să verifice A2 = B2 = C2 = I2 şi AB + BA = O2; AC + CA = O2; BC + CB = O2. Putem considera
0 1 0 − i 1 0 
A =  ; B =   ; C =   ; 8. a) Calcul direct; b) CerinŃa problemei este echivalentă cu P(n): Bn ⋅
 1 0   i 0   0 −1 
⋅ U = λn ⋅ U, (∀) n ≥ 1 (inducŃie matematică); c) Folosind punctul b), suma elementelor de pe fiecare linie a
1 0 
matricei M2 este egală cu 25, deci suma elementelor lui M2 este 50; 9. a) B = C =   ; b) Fie g(x) = g(y).
 0 −1
Atunci f(g(x)) = f(g(y)) ⇒ ⇒ Ag(x) + f(x)B = Ag(y) + f(y)B ⇒ f(x)B = f(y)B şi înmulŃind la dreapta cu
matricea C, obŃinem f(x)BC = f(y)BC sau f(x) = f(y) şi cum f este injectivă, rezultă că x = y, ceea ce arată că g
 2n − 1 2 n 0 
  det X n 
2n
 2n + 2 − 1 
2n
lim
2n − 22 n + 2

este injectivă; 10. Xn = =  2 0  ; lim  2n + 2  = lim 1 − 2n + 2  = en →∞ 2 = e −1 ;


n n 2 n +2
2 −1
  n →∞
 2  n →∞
 2 
0 0 2 n +1 − 1
 
11. Observăm că ε3 = 1. Deoarece (A + ε2In)(B + εIn) = AB + εA + ε2B + In = In, deducem că matricele A + ε2In
şi B + εIn sunt inversabile şi sunt una inversa celeilalte. Atunci (B + εIn) (A + ε2In) = In, de unde BA + ε2B + εA
+ In = In sau BA + εA + ε2B = On, deci AB = BA; 12. a) InducŃie după n: Pentru n = 1, A = α1A + β1I3 cu α1 = 1
şi β1 = 0. Presupunem că An = αnA + βnI3, αn, βn ∈ Z şi să arătăm că An+1 = αn+1A + βn+1I3 cu αn+1, βn+1 ∈ Z.
Cum A2 = A + 2I3, avem An+1 = αnA2 + βnA = (αn + βn)A + 2αnI3, deci αn+1 = αn + βn ∈ Z şi βn+1 = 2αn ∈ Z;
1
b) De la punctul a) deducem βn = 2αn–1 şi de aici αn+1 = αn + 2αn–1, de unde αn = a ⋅ (–1)n + b ⋅ 2n cu a = − şi
3
n n n n n n
1 2 − (−1) 2 + 2 ⋅ (−1) 2 (−1)
b = . ObŃinem A n = A+ I3 = (A + I 3 ) + ⋅ (2I3 − A ) ; 13. A = I3 + B, unde
3 3 3 3 3
 1 1 1
 
B = 1 1 1 cu Bn = 3n–1B, (∀) n ≥ 1. Rezultă că An = I3 + C1n B + C 2n B2 + ... + C nn B n = I 3 + C1n B + C 2n 3B + ... +
 1 1 1
 
0 a − b
1 4n − 1  
+ C nn ⋅ 3n −1 B = I3 + B(C1n ⋅ 3 + C 2n ⋅ 32 + ... + C nn ⋅ 3n ) = I 3 + ⋅ B ; 14. A = I3 + B, unde B =  b 0 0  .
3 3 a 0 0 
 
Se obŃine An = I3 + C1n B + C 2n B2 ; 15. Polinomul caracteristic fA = det(A – λI3) asociat matricei A are rădăcinile
1 1 3
 
λ1 = –2, λ2 = 5, λ3 = 0. Atunci An = A1 ⋅ (–2)n + A2 ⋅ 5n, (∀) n ≥ 1. Avem –2A1 + 5A2 =  2 1 2  şi 4A1 +
3 1 1
 
12 5 8   1 0 − 1  2 1 2
  1  1  (−2) n
+ 25A2 = 10 5 10  , de unde găsim A1 =  0 0 0  şi A 2 =  2 1 2  , deci A n = ⋅
 8 5 12  2  5  2
  −1 0 1   2 1 2

309
 1 0 − 1  2 1 2  a n 0 bn 
  5n    
⋅ 0 0 0  + ⋅  2 1 2  ; 16. b) det(A + B ) = det(a I2) = a ; 17. Observăm că A =  0 3n 0  ;
n n n 2n n

 − 1 0 1  2  2 1 2 b 
     n 0 an 
 π π 
 cos − sin 0
 3 3 
5n + (−1) n 5n − (−1) n π π
a n + b n = 5n şi a n − b n = (−1) n . Rezultă a n = ; bn = ; 18. B =  sin cos 0  şi
2 2  3 3 
 0 0 1
 
 
 nπ nπ 
 cos − sin 0
 3 3 
 nπ nπ
n
B = sin cos 0  , n ∈ N*; 19. Folosim faptul că dacă X ∈ M2(C) şi Xn = O2, atunci X2 = O2.
 3 3 
 0 0 1
 
 
Presupunem că A + B nilpotentă ⇒ (A + B)2 = O2 ⇒ A2 + AB + BA + B2 = O2 ⇒ AB + BA = O2 ⇒ tr(AB) =
= –tr(BA) = –tr(AB) ⇒ tr(AB) = tr(BA) = 0 şi det(AB) = det(BA) = 0. Astfel, (AB)2 – tr(AB) ⋅ AB + (det(AB)) ⋅
⋅ I2 = O2, deci (AB)2 = O2. Analog, (BA)2 = O2, deci A ⋅ B şi B ⋅ A sunt nilpotente. Să presupunem că A ⋅ B =
= B ⋅ A sunt nilpotente. Atunci (A + B)2 = A2 + AB + BA + B2 = AB + BA; (A + B)4 = (AB)2 + AB2A + BA2B +
+ (BA)2 = O2, deci A + B este nilpotentă; 20. b) Din Y2011 = AY şi Y2011 = YA, conform cu punctul a), deducem
a b  a 2010 2010a 2009 b 
că Y =   şi Y 2010 =   . Găsim a = b = 1 sau a = b = –1; 21. a) Deoarece detA = 0 ⇒
 0 2010 
0 a   a 
 x y
⇒ A2 = (a + b)A şi de aici An = (a + b)n–1A; b) Avem detX5 = detA = 0 ⇒ detX = 0 şi dacă X =   , rezultă
z t 
x ( x + t ) 4 = a

 y( x + t ) 4 = a
că X = (x + t)X. EcuaŃia (x + t) X = A ⇔ 
2 4
. Cum (x + t)5 = a + b ⇒ x + t = 5 a + b ⇒ x =
4
 z ( x + t ) = b
 4
t ( x + t ) = b
a b −1 0  −1 0
= = y şi z = = t ; 22. Avem X 4 − 2X 2 =  X = X  şi de aici X are forma X =
5
(a + b ) 4 5 4
(a + b )  10 4   10 4 

 a 0  a3 0 
=   iar X 3 =  3 . Găsim a3 – 2a = –1 şi b3 – 2b = 4, cu soluŃiile în Z, a = 1 şi b = 2,
 2 b − 2a 3 b 3 
 2 b − 2a b  
1 0 an na n −1b 
deci X =   ; 23. a) A n =  , (∀) n ≥ 1; b) Avem X6 + X2 = AX şi X6 + X2 = XA, de aici AX =
0 n 
 2 2  a 
a b a5 5a 4 b   a b   2 6 
= XA. Rezultă că X are forma X =   . EcuaŃia este echivalentă cu  + =  ⇔
0 a 
0
 a 5   0 a   0 2 

310
a 5 + a = 2  1 1 a b
⇔ cu a = 1 şi b = 1, deci X =   ; 24. Dacă X =   , cum detX = 0, deducem Xn = (a +
5a b + b = 6
4
 0 1 c d
+ d)n–1X. ObŃinem (a + d)2009 ⋅ a = 4, (a + d)2009 ⋅ b = 6, (a + d)2009 ⋅ c = 8 şi (a + d)2009 ⋅ d = 12. Cum (a + d)2009 ⋅
1 2009
4 6 8 12
⋅ (a + d) = 16, deducem a + d = 16 2010 sau (a + d) 2009 = 16 2010 = p . ObŃinem: a = , b = , c = şi d = ,
p p p p
a b c 0 1 0
1 4 6     
deci soluŃia ecuaŃiei este: X =   ; 25. Fie X =  d e f  şi A =  0 0 1  ; X6 = X ⋅ A = AX ⇒ X =
p  8 12  g h i  1 0 0
   
a b c
  1
=  c a b  . Dar detX = (a + b + c) ⋅ [(a − b) 2 + (b − c) 2 + (c − a ) 2 ] şi detX = 1 ⇒ (a + b + c) ⋅ [(a – b)2 + (b –
b c a 2
 
a + b + c = 1 a + b + c = 2
– c)2 + (c – a)2] = 2 ⇒  sau  . ObŃinem
(a − b) 2 + (b − c) 2 + (c − a ) 2 = 2 (a − b) 2 + (b − c) 2 + (c − a ) 2 = 1
a b
(a, b, c) ∈ {(1, 0, 0); (0, 1, 0); (0, 0, 1)}; 26. Fie X =   . Cum detX = 0 ⇒ X2 = (a + d)X şi Xn = (a + d)n–1 ⋅ X
c d
3 1
= =   ⇒ a ⋅ (a + d)n–1 = 3, b ⋅ (a + d)n–1 = 1, c ⋅ (a + d)n–1 = 6 şi d ⋅ (a + d)n–1 = 2. Deducem (a + d)n = 5 ⇒
 6 2 
n 5 , n impar
⇒ a+d= . ObŃinem apoi valorile lui a, b, c şi d; 27. Fie X soluŃie, atunci Xn+1 = AX = XA. Dacă
± n 5 , n par
x y z  x 0 0  xn 0 0 
    
X =  t u v  , atunci y = z = t = v = b = 0 şi a = c – x. Rezultă X =  0 u 0  şi X =  0
n
un 0  .
a b c c − x 0 c 
 cn − x n 0 c n 
     
Rezultă xn = 2, un = –1, cn = 3, care are soluŃie în R numai pentru n impar, x = n 2 , u = −1, c = n 3 ;
3 x y
28. b) Folosind a) pentru a = 3, b = 4, ε = ; c) Fie X soluŃie, atunci Xn–1 = XB = BX. Dacă X =  ∈
2 z t 
 x 6x 
∈ M2(C), atunci X =  8x  . Folosind a) pentru a = x, b = 8x , ε = 3 , rezultă:
 x 3 2
 3 
 x n (5n x n (5n + (−3) n ) 3 
+ (−3) n )
 ⋅ 6 12 5n − (−3) n
Xn =  n n 2 2 2  . Rezultă x n = , xn = n , deci n =2
 x (5 − (−3) n ) x (5 + (−3) ) 
n n n n
5 + (−3) n
5 − (−3) n
5 + (−3) n
 
 3 2 
 0 −1
care nu are soluŃii în N. EcuaŃia dată nu are soluŃie; 29. a) Avem, de exemplu, soluŃiile X 2 =   , a, b ∈ Z;
b − a

311
 X 2 ... O 2 
 
b) Dacă n este par şi X2 este soluŃia de la punctul a), avem soluŃiile:  ... ... ...  . Dacă n este impar, ecuaŃia
 O ... X 
 2 2

 a 
2
 a2 
este echivalentă cu  X + I n  =  − b I n , care nu are soluŃii în cazul în care a2 – 4b < 0, deoarece
 2   4 
determinanŃii matricelor din cei doi membri au semne diferite. Rezultă că ecuaŃia are soluŃii pentru n par;
30. Căutăm pe B sub forma B = I4 + xA + yA2 + zA3. Atunci B2 = I4 + 2xA + (x2 + 2y)A + (2z + 2xy)A3.
3 3 1
ObŃinem 2x = 3, x2 + 2y = 3, 2z + 2xy = 1, deci B = I 4 + A + A 2 − A 3 ; 31. det(Xp) = 0 ⇒ detX = 0 şi
2 8 16
a b
 , rezultă X2 = (a + + d)X ⇒ Xn = (a + d)n–1X. Găsim a + d = 2 şi X n = ( 2 ) n −1 X şi atunci
p p
dacă X = 
c d
p
1 − ( 2 )n
Y(n) se va scrie sub forma: Y (n ) = [1 + 2 + ( 2 ) 2 + ... + ( 2 ) n −1 ]X şi de aici Y (n ) =
p p p
p
X , deci an =
1− 2
p n
1 − ( 2 )n
=
1− 2
p
; 32. a) InducŃie matematică; b) Avem det(Ak) = (detA)k = cosk 2x. Deci ∑ det A
k =1
k
= cos 2 x +

 n + Sn n − Sn 
n    n k
+ cos 2 2 x + ... + cos n 2x = Sn . Dar ∑ Ak =  2

2  şi se obŃine det
n − S n n + Sn    ∑
A  = nSn . Deci
k =1    k =1 
 2 2 
1 1
n
x −x

k =1
det A k
1 1
2
1
2
3 1
= → 0, n → ∞; 33. Fie D = − x x = 3x 2 − x + , care are valoarea minimă
 n  n 2 2 2 4
det 
 ∑
 k =1
Ak 


x
1
−x
1
2 2
a1 a2 1 1
1 b1 b 2 1 1
; 34. Considerăm determinantul ∆ = = = 0 . Dezvoltând ∆ după coloana a patra, avem ∆ = D1 –
16 c1 c2 1 1
d1 d 2 1 1
– D2 + D3 – D4 = 0 şi concluzia se impune; 35. Scădem ultima linie din primele 2010 linii şi astfel, de pe fiecare
din primele 2010 linii se scoate factorul 2, deci detA se divide cu 22010; 36. Se adună prima linie la a doua şi a
treia şi se scoate factorul comun pe cele două linii, 2. Generalizare: Dacă A ∈ Mn(R), atunci 2n–1 divide detA;
37. Avem ∆n = 2cos α ∆n–1 – ∆n–2, n ≥ 3. Cum ∆1 = cos α; ∆2 = cos 2α, obŃinem ∆n = cos nα; 38. Avem
 a1a n  0  a1 
     
 a 2a n  0  a2 
pentru ultima coloană  ...  = p ...  + a n  ...  . Notând det(An) = Dn, rezultă Dn = pDn–1 + a n ⋅
     
 a n −1a n  0  a n −1 
 2     
 an + p  1  an 

312
a12 + p a1a 2 ... a1
a a a 22 + p ... a 2
⋅ 2 1 . Dacă în ultimul determinant scădem din prima coloană ultima coloană înmulŃită cu a1,
... ... ... ...
a n a1 a na 2 ... a n
p 0 ... a1
0 p ... a 2
din a doua coloană ultima coloană înmulŃită cu a2, etc., obŃinem: Dn = pDn–1 + + a n ⋅ , deci Dn =
... ... ... ...
0 0 ... a n
D n D n −1 1 2 D n −1 D n − 2 1 2 D D 1 D 1 2
= pDn–1 + p n −1a 2n . Rezultă= n −1 + a n , n −1 = n − 2 + a n −1 , ..., 22 = 1 + a 22 ; 1 = 1 + a1 .
pn p p p p p p p p p p
D 1
Adunând, rezultă nn = 1 + (a12 + a 22 + ... + a 2n ) , de unde: D n = p n + p n −1 ⋅ (a12 + a 22 + ... + a 2n ) , deci Dn ⋮ pn–1;
p p
a b
39. Fie A =   , a, b, c, d ∈ R, atunci det(A2 + I2) = 0 ⇒ (a2 + bc + 1)(d2 + bc + 1) – bc(a + d)2 = 0 ⇒
 c d 
(ad – bc – 1)2 + (a + d)2 = 0, deci ad – bc = 1 şi a + d = 0. Rezultă detA = 1; 40. a) A2 – B2 = AB – BA ⇔
(A – B)(A + B) = On ⇒ det(A – B) ⋅ det(A + B) = 0 ⇒ det(A – B) = 0 sau det(A + B) = 0; b) A2 + B2 = AB + BA
⇔ (A – B)2 = On ⇒ [det(A – B)]2 = 0 ⇒ det(A – B) = 0; 41. Fie C = AB – BtA, atunci Ct = (AB – BtA)t = BtAt –
– AtB = BtA – AB = –C, deci det(Ct) = det(–C). Rezultă detC = 0. Prin urmare 8 – 8detB = 0, de unde detB = 1;
42. Dacă adunăm a treia linie la prima şi a doua, pe primele două linii apare factorul comun 2, deci un
determinant posibil se divide cu 4. Valorile posibile ale determinanŃilor sunt –4, 0 sau 4, deci valoarea maximă
 1 1 1
 
este 4. O astfel de matrice este (de exemplu)  − 1 1 1 . Interpretare geometrică: |detA| = 2SMNP, unde M, N,
 1 − 1 1
 
P sunt vârfuri (nu neapărat distincte) ale pătratului ABCD, A(1, 1); B(–1, 1); C(–1, –1), D(1, –1) (rezultă detA ∈
n (n + 3)
∈{–4, 0, 4}); 43. a) A ∆OA n A n +1 = ; b) Deoarece f(n) ≥ f(1), (∀) n ≥ 1, rezultă că f(1) este valoarea
2
minimă, deci n = 1; 44. Dacă AB = BA, atunci A2B = A(AB) = A(BA) = (AB)A = (BA)A = BA2. Reciproc, fie
A2B = BA2. Cum A2 – (a + d)A + (ad – bc)I2 = 0, rezultă A2 = tA – ∆ ⋅ I2, t = a + d ≠ 0, ∆ = detA. Atunci
(tA – ∆I2) ⋅ B = B ⋅ (tA – ∆I2), de unde AB = BA; 45. Notăm A = X – Y, B = Y – Z. Rezultă A + B = X – Z şi
AB = On şi cum matricele X, Y, Z comută două câte două, obŃinem: BA = YX – ZX – Y2 + ZY = AB = On, adică
A şi B comută; A2B2 = A(AB)B = A(BA)B = (AB)(AB) = (AB)2 = (BA)2 = B(BA)A = B2A2. Atunci
det[(X – Y)4 + + (Y – Z)4 + (Z – X)4] = det[A4 + B4 + (A + B)4] = det[(A2 + B2)2 – 2A2B2 + ((A + B)2)2] =
=det[(A2 + B2)2 + (A2 + B2)2] = 2n ⋅ [det(A2 + B2)]2 ≥ 0; 46. Fie λ1, λ2 ∈ C valorile proprii ale lui A. Deoarece
det(A2 + A + I2) = det(A2 + 2A + I2), rezultă că (λ21 + λ1 + 1)(λ22 + λ 2 + 1) = (λ21 + 2λ1 + 1)(λ22 + λ 2 + 1) , de unde
λ21λ 2 + λ1λ22 + +3λ1λ 2 + λ1 + λ 2 = 0 , deci td + 3d + t = 0, unde t = trA = λ1 + λ2 ∈ Z şi d = detA = λ1λ2 ∈ Z.
ObŃinem (t, d) ∈ {(–2, 2); (0, 0); (–4, –4); (–6, –2)}, de unde t + d ∈ {0, –8}; 47. Fie d = detA şi cum trA = 1,
A2 = A – dI2 şi atunci A3 = (1 – d)A – dI2, deci A3 + A + I2 = (2 – d)A + (1 – d)I2 ⇒ det(A3 + A + I2) = d3 – 2d2 –
– d + 3. Inegalitatea cerută ⇔ 2d4 – 4d3 + d2 + d + 1 ≥ d4 – 2d3 – d2 + 3d ⇔ (d2 – d)2 + (d – 1)2 ≥ 0, adevărată;
−1− i 3
48. Fie P = x2 – x + d, d = detA, ε = . Avem: det(A2 + A + I2) = det(A – εI2) ⋅ (A − εI 2 ) = det(A − εI 2 ) ⋅
2

313
2 2 2
⋅ det(A − εI 2 ) = det(A − εI 2 ) = P(ε) = ε2 − ε + d = d2 +3 ≥ 3; 49. Din det(A ⋅ At) = detI2 rezultă detA ∈ {–1, 1}.
a b  d − b
Prin urmare A este inversabilă şi A–1 = At. Fie A =   . Dacă detA = –1, obŃinem −   =
c d − c a 
a c  d − b a c
=   , ad – bc = –1; a + d = 2, imposibil. Dacă detA = 1, atunci   =   , ad – bc = 1, a + d =
b d − c a  b d
 0 0
= 2. Rezultă A = I2; 50. a) A =   ; b) Fie A ∈ M2(Z) şi notăm t = trA, d = detA. Avem: det(A + I2) = t +
1 1
+ d + 1. Dar A2 = tA – dI2, de unde det(A2 + I2) = t2 + d2 – 2d + 1, ceea ce conduce la relaŃia (d – 2)2 + (t – 1)2 =
= 6, care nu este posibilă în numere întregi; 51. Deoarece det(A – εkI) = detA – εk ⋅ trA + ε2k = ε2 – εk+1 + ε2k,
n n n n
avem ∑ε
k =1
k
det( A − ε k I) = ∑ε
k =1
k +2
− ∑ε
k =1
2 k +1
+ ∑ε
k =1
3k
= 0 ; 52. a) Din det(A2 – 2I2) = det(A – 2I 2 ) ⋅

a b
⋅ det(A + 2 I 2 ) = 0 rezultă det( A − 2I 2 ) = 0 sau det( A + 2 I 2 ) = 0 . Fie A =   ∈ M2(Q), atunci (ad –
c d
– bc) ± 2 (a + d) + 2 = 0 , deci ad – bc = –2 şi a + d = 0; b) A2 = (a + d)A – (ad – bc)I2 = 2I2; 53. Fie t = trA şi
d = detA. Avem: det(A – xI2) = x2 – xt + d, de unde det(A + I2) = 1 + t + d. Dacă ε este rădăcina cubică a unităŃii,
2
avem det(A2 + A + I2) = det(A – εI2) ⋅ det(A − εI 2 ) = (ε2 – tε + d) (ε − t ε + d ) = 1 + t2 + d2 + t – d + dt. Atunci
t2 + d2 + 1 + t – d + dt = (t + d + 1)2, deci (t + 3)(d + 1) = 3. ObŃinem (t, d) ∈ {(–2, 2); (0, 0); (–4, –4); (–6, –2)},
deci t + d ∈ {–8, 0}; 54. b) Fie funcŃia f: R → R; f(t) = det[tA + (1 – t)B] + det[(1 – t)A + tB]. Avem f(0) =
1 1
= f(1) = detA + detB şi f   = det(A + B) = detA + detB. Deoarece f este o funcŃie polinomială de grad cel
2 2
mult doi, deducem că f(t) = f(0), (∀) t ∈ R, ceea ce arată cerinŃa problemei; 55. a) Fie B = A – At ⇒ Bt = At –
 a b c
 
– A = –B. Cum detB = detBt ⇒ detB = (–1)3detB ⇒ detB = 0; b) Fie A =  m n p  ⇒ A – At =
 q r s
 
 0 b − m c − q
 
= m − b 0 p − r  . Cum det(A – At) = 0 ⇒ rang(A – At) ≤ 2. Dacă rang(A – At) ≤ 1, atunci (b – m)2 =
 q−c r−p 0 

= (c – q)2 = (r – p)2 = 0 ⇒ m = b, p = r şi q = c ⇒ A = At, fals; 56. a) det(AB) = detA ⋅ detB = 1 ⇒ detB ≠ 0 ⇒ B
este inversabilă; AB = In ⇔ A = B–1 ⇔ BA = In, deci AB = BA; b) pA + qB = A ⋅ B ⇔ (A – qIn)(B – pIn) =
1  1  1  1  1  1 
= pqIn ⇔  A − I n  B − I n  = I n şi conform punctului a)  A − I n  B − I n  =  B − I n  A − I n  , iar
q  p  q  p  q  p 
de aici rezultă AB = BA, deci ApBq = BqAp; 57. Din A + B = In rezultă A2011 + A2010B = A2010, de unde A2010B =
= On. Atunci In + A2009B = (A + B)(In + A2009B) = A + A2010B + B + BA2009B = In + BA2009B, de unde A2009B =
= BA2009B. ObŃinem (In + A2009B)(In – A2009B) = In – A2009BA2009B = In – A2009(A2009B) = In – A2008A2010B = In.
Deducem că matricea In – A2009B este inversabilă şi (In – A2009B)–1 = In + A2009B; 58. Avem AB = BA şi A2007B =
= On, deci A2007B2007 = (AB)2007 = On. Cum (In – AB)(In + AB + (AB)2 + ... + (AB)2006) = In – (AB)2007 = In,
rezultă cerinŃa problemei; 59. a) Evident AB = BA şi AkB = On. Atunci (AB)k = AkBk = On. Avem (In – AB) ⋅
⋅ [In + AB + (AB)2 + ... + (AB)k–1] = In – (AB)k = In şi In + AB) ⋅ [In – AB + (AB)2 – ... + (AB)k–1] = In + (AB)k =
= In, deci In – AB şi In + AB sunt inversabile; b) (In – AB)–1 + (In + AB)–1 = (In + AB + (AB)2 + ... + (AB)k–1) +

314
+ (In – AB + (AB)2 – ... + (AB)k–1) = 2(In + (AB)2 + ... + (AB)k–1); 60. RelaŃia dată se scrie (A3 + In)(In – B) = In
şi de aici rezultă că A3 + In şi In – B sunt inversabile, una fiind inversa celeilalte. Scriind relaŃia dată sub forma (A3
+ In)B = = A3, deducem B = (A3 + In)–1A3. Cum (A3 + In)A = A(A3 + In), rezultă că A(A3 + In)–1 = (A3 + In)–1A.
 
 
 1 0 0 
1
Deducem că AB = BA = (A3 + In)–1A4; 61. a) detA = 6 ≠ 0, deci există A1, A −1 =  0 0  ; b) Fie X soluŃie,
 2 
 1
0 0 
 3
a 0 0
 
atunci X2011 = XA = AX, deducem că X =  0 b 0  , a, b, c ∈ C. Se verifică uşor că
0 0 c
 
 a 2010 0 0 

X 2010
= 0 b 2010
0  . Rezultă a2010 = 1, b2010 = 2, c2010 = 3. Fiecare ecuaŃie are, în C, 2010 soluŃii. Sunt
 
0 0 c 2010 
 
20103 matrici X; 62. Prin calcul M = {A, A2, –I3, –A, –A2, I3}, deci m = 6. Rezultă A6k+1 + A6k+2 + ... + A6k+6 =
mn 6n
=A6k(A + A2 + ... + A6) = O3 şi ∑i =1
Ai = ∑A
i =1
i
= (A + A 2 + ... + A 6 ) + (A 7 + ... + A12 ) + ... + (A6n–5 + ... + A6n) =

=On. EcuaŃia devine –XA–1 = A2010, de unde X = –A2011 = –(A6)335 ⋅ A = –A; 63. ÎnmulŃind relaŃia iniŃială cu A la
stânga şi cu B la dreapta, obŃinem A2 + + B2 = AB sau (A – B)2 = –BA. Analog, înmulŃind la stânga cu A–1 şi la
dreapta cu B–1, obŃinem (A–1)2 + (B–1)2 = A–1 ⋅ B–1 = (BA)–1. Cum (A – B)2 ⋅ [(A–1)2 ⋅ (B–1)2] = –(BA) ⋅ (BA)–1 =
=I2n, prin trecere la determinanŃi, obŃinem cerinŃa problemei; 64. Fie d = detA ≠ 0. Avem d ⋅ det(A – A–1) =
det(A + I3 ) ⋅ det(A − I 3 )
= detA ⋅ det(A – A–1) = det(A2 – I3) = det(A + I3) ⋅ det(A – I3) ⇒ det(A – A–1) = . Fie
d
p(x) = det(A – xI3) şi λ1, λ2, λ3 valorile proprii ale lui A; p(x) = –(x – λ1)(x – λ2)(x – λ3), unde λ1 + λ2 + λ3 = 0,
λ21 + λ22 + λ23 = trA 2 = 0 , deci λ1λ2 + λ2λ3 + λ3λ1 = 0. Rezultă că det(A + I3) = p(–1) = (λ1 + 1)( λ2 + 1)( λ3 + 1)
şi det(A – I3) = p(1) = (λ1 – 1)( λ2 – 1)( λ3 – 1) şi de aici det(A + I3) ⋅ det(A – I3) = d2 – 1. Deci det(A – A–1) =
1
= d − = det A − det A −1 ; 65. Pentru n ≥ 9, printre numerele n + 1, n + 2, ..., n + 9 găsim cel mult un pătrat
d
perfect (n + 1 ≤ p2, p ≥ 4 ⇒ n + 9 ≤ p2 + 8 < (p + 1)2). 1) Dacă n + 1 = p2, rezultă că numerele [ n + 1], [ n + 2] ,
... , [ n + 9 ] sunt p, p + 1, ..., p + 1 şi det(An) = 0 (are două linii egale); 2) Dacă n + 2 = p2, numerele sunt: p, p,
p + 1, ..., p + 1; 3) Dacă n + 3 = p2, numerele sunt p, p, p, p + 1, ..., p + 1. În cazurile 2 şi 3, det(An) = 0, având
două linii egale; 4) Dacă n + 4 = p2 sau n + 5 = p2, găsim două coloane egale; 5) Dacă n + 6 = p2 sau n + 7 = p2
sau n + 8 = p2 sau n + 9 = p2, găsim două linii egale; b) Dacă printre numerele n + 1, n + 2, ..., n + 9 nu este
niciun pătrat perfect, toate elementele sunt egale. În concluzie det(An) = 0 pentru n ≥ 9.

315
ANALIZĂ MATEMATICĂ

5a 2 + 2b 2 + 2
1. CondiŃia dată impune = 1 , de unde (a – 2)2 + (3a – 2b)2 = 0 şi a = 2, b = 3; 2. a = 4; b = 12; c =
2a + 6ab
1
= 8049; 3. Prin calcul găsim x n = 1 − şi lim x n = 1 iar lim x n n = e −1 ; 4. a) ExistenŃa celor două şiruri
n +1 n →∞ n →∞

(an) şi (bn) se arată dacă dezvoltăm (2 + 5 ) n cu binomul lui Newton şi în plus se arată că (2 − 5 ) n = a n −
(2 + 5 ) n + (2 − 5 ) n (2 + 5 ) n − (2 − 5 ) n an
– b n 5 ; b) ObŃinem a n = şi b n = şi apoi lim = 5;
2 2 5 n →∞ bn

 π
5. Notăm OP1 = a şi m(XOY) = α ∈  0,  . Avem OP2 = acos α, OP3 = a(cos α)2 şi prin inducŃie găsim OPn =
 2
n −1
= a(cos α) . Deoarece lim (cos α )
n–1
= 0 , deducem că lim OPn = 0 ; 6. a) Dacă r > 1 este raŃia, atunci Sn =
n →∞ n →∞
p p
r  1  ' r  1  Sn r p − 1 rp
= ⋅  1 −  ; S n = ⋅  1 −  şi atunci = ; b) lim S n = ; lim S'n =
a1p (r 2 p − 1)  r pn  a1p (r p − 1) 2  r pn  S'n r p + 1 n →∞ a1p (r 2 p − 1) n → ∞
rp
= şi lim R p = e0 = 1 ; 7. Fie P un polinom de grad n ≥ 2, P(x) = xn + a1xn–1 + ... + an. Calculăm l =
a1p (r p − 1) 2 p →∞

P( x ) − x n
= lim ( n P( x ) − x ) . Avem l = lim =
x →∞ x →∞ n
P ( x ) n −1 + x n P( x ) n − 2 + ... + x n −1
a1x n −1 + ... + a n a
= lim = 1 . Rezultă L = lim [8 x 8 + x 7 + 1 − x − (9 x 9 − x 8 + 1 − x)] =
x →∞  n −1 n −2  n x →∞
  P( x )   P(x)  
x n −1  n   +n   + ... + 1
  x   x  
 
1 1 17
= + = ; 8. Avem x n 2 = {n} + { n 2 + 1} + { n 2 + 2} = n 2 + 1 − n + n 2 + 2 − n → 0 şi x n 2 −1 = { n 2 − 1} +
8 9 72
x 2n y n 1
+ {n} + { n 2 + 1} = n 2 − 1 − (n − 1) + n 2 + 1 − n → 1 ; 9. z n = 2 2
= yn ⋅ 2
.
3x n − 2x n y n + y n  yn  yn
  − 2 ⋅ +3
 xn  xn
2 1
y  y 1 1 1
Dar  n  − 2 ⋅ n + 3 ≥ 2, (∀) n ≥ 1 ⇒ 0 < 2
≤ , deci lim z n = 0 ; 10. a) ; b) 1; c) e 2 ;
2 n →∞ 2
 xn  xn  yn  y
  − 2 ⋅ n + 3
 xn  xn
11. Arătăm, prin inducŃie matematică, că n ≤ an ≤ n + 1, (∀) n ≥ 2. Pentru n = 2 avem 2 ≤ a2 ≤ 3, adevărat, căci
a2 = 3. Dacă presupunem adevărat n ≤ an ≤ n + 1, demonstrăm că n + 1 ≤ an+1 ≤ n + 2. Într-adevăr, an+1 = n +
a n a n +1 1 n an n +1
+ n ≥ n + = n + 1 şi a n + 2 = n + n ≤ n + = n + 1 + < n + 2 . Atunci ≤ ≤ , (∀) n ≥ 2, deci
n n n n n n n n
a 1
lim n = 1 ; 12. a) Observăm că fn este strict crescătoare, deci şi f n−1 este strict crescătoare. Deoarece f n   =
n →∞ n n

316
1 1 1
= 3
+ n > n ⇒ > f n−1 (n ) , deci a n < , (∀) n ≥ 1; b) Din fn(a) = 0 ⇒ a = f n−1 (a ) < f n−1 (n ) = a n , de aici
n n n
1  1  1 n2 1
rezultă 0 < an < , (∀) n ≥ 1, deci lim a n = 0 . Deoarece f n  = + < n , (∀) n ≥ 1 ⇒ <
 n + 1  (n + 1) n +1 n +1
n →∞ 3
n
1 1 n
< f n−1 (n ) = a n , de unde găsim că < an < ⇒ < na n < 1 , (∀) n ≥ 1, deci lim (na n ) = 1 ; c) Avem bn =
n +1 n n +1 n →∞
1 1 1 1 1 1 
= a1 + a2 + ... + an > + + ... + şi cum lim  + + ... +  = ∞ ⇒ lim b n = ∞ ; 13. |xn+p – xn| ≤ |xn+p –
2 3 n +1 n → ∞ 2 3 n +1 n →∞

1
– xn+p–1| + |xn+p–1 – xn+p–2| + ... + |xn+1 – xn| ≤ , (∀) n ≥ 1 şi (∀) p ≥ 1. Deoarece
2009 ⋅ 2010 n +1
1
lim = 0 ⇒ (xn)n este convergent; 14. Folosim două rezultate auxiliare. Primul este: Dacă
n →∞ 2009 ⋅ 2010 n +1

x + x 2 + ... + x n
lim x n = x, atunci lim 1 = x . Al doilea este: Dacă avem şirurile (xn)n≥1, (yn)n≥1, (zn)n≥1, cu |xn –
n →∞ n →∞ n
n

∑a b
1 a
– yn| ≤ zn, (∀) n ≥ 1 şi lim z n = 0 , iar lim y n = a ∈ R , atunci lim x n = a . Fie x n = k σ( k ) şi yn = ⋅
n →∞ n →∞ n →∞ n n
k =1
n n n n n
a k − a b σ( k ) (a k − a ) 2
∑ ∑ ∑ ∑ ∑
1 a
⋅ b j . Aplicând inegalitatea Cauchy, avem: a k b σ(k ) − bk = ⋅ ≤ ⋅
j =1
n k =1
n k =1 k =1 n n k =1
n
n
b 2k
⋅ ∑n
k =1
= z n ; 15. FuncŃia f trebuie să aibă proprietatea că mulŃimea Fm = {n ∈ N* | f(n) = m} = ∅ este finită,

(∀) m ≥ 1. Într-adevăr, dacă presupunem că există ma ∈ N*, pentru care Fm a este infinită, atunci fixăm a < ε <
< | a − a m a | . Fie n0 ≥ 1 astfel încât |bn – a| < ε, (∀) n ≥ n0. MulŃimea {1, 2, ..., n0} este finită, deci există n1 > n0
cu n1 ∈ Fm a . Ajungem la o contradicŃie: ε > | a − b n 1 | = | a − a f ( n 1 ) | =| a − a m a | > ε ; 16. ObŃinem xn+1 – yn+1 = 3 ⋅
⋅ (xn – yn) şi cum x1 – y1 = –3, obŃinem xn – yn = (–3) ⋅ 3n–1 = –3n. Avem xn+1 = 3xn + 2 ⋅ 3n şi de aici:
 x 1 = 3x 0 + 2 ⋅ 1

 x 2 = 3x1 + 2 ⋅ 3
 ⇒ x n = 3n −1 (2n + 3); y n = 3n −1 (2n + 6) ; 17. Se arată prin inducŃie matematică că an =
.......... .......... ..
x = 3x + 2 ⋅ 3n −1
 n n −1
an 2n + 1
= (2n – 1)(2n + 3), (∀) n ≥ 1. Deci lim = 4 ; 18. RelaŃia se mai scrie sub forma ⋅ a n +1 −
n →∞ n 2
( n + 1)( n + 2)
a
2n − 1 n (n + 1)(n 2 − n + 1)  1  n
− ⋅ an = n , iar prin sumare se obŃine: a n = şi lim a n = ∞ , iar lim 1 + 3  =
n (n + 1) 2(2n − 1) n →∞ n → ∞ n 
5 n xn 2 1 2 3
= 4 e ; 19. x n = n + 2 ⋅ 3 , iar lim n +1 = ; 20. x n = n şi yn = x0 + 2x1 + 3x2 + ... + nxn = 1 + + 2 +
2 n →∞ 3 3 2 2 2
n 1 1 2 3 n 1 1 1 1 n
+ ... + n −1 . Din ⋅ y n = + 2 + 3 + ... + n , deducem y n − y n = 1 + + 2 + ... + n −1 − n . Rezultă
2 2 2 2 2 2 2 2 2 2 2

317
n+2 1 1 1 1
yn = 4 – n −1
şi lim y n = 4 ; 21. Avem =1+ 2⋅ . Rezultă = 2 n − 1 , n ≥ 1, de unde a n = n şi
2 n → ∞ a n +1 a n a n 2 −1
x −2 x −2 1
lim a n = 0 ; 22. a) Avem y n +1 = n +1 = n = y n , de unde rezultă că (yn) este o progresie
n →∞ x n +1 + 1 4( x n + 1) 4
n −1
1 1 yn + 2
geometrică cu raŃia . Urmează că y n = y1 ⋅   şi x n = . Cum lim y n = 0 , rezultă că lim x n = 2;
4 4 1 − yn n →∞ n →∞

zn − α a −c a −α
b) Fie y n = , unde α2 + (c – a)α – b = 0, α > a, α > . Atunci y n +1 = ⋅ y n ⇒ (yn) este o
z n + ( −a + α ) 2 c+α
n −1
a −α a−α
progresie geometrică cu raŃia şi deci y n = y1 ⋅   , cu lim y n = 0 . Rezultă că lim z n = α ;
c+α c+α n →∞ n →∞

1 8x 3n− 1 2 x n +1 − 1 2x − 1
23. Inductiv se arată că x n = . De asemenea, y n = = , de aici y1 ⋅ y 2 ⋅ ... ⋅ y n = 2 ⋅
3 n −1 +1 2x n − 1 2x n − 1 2 x1 − 1
2
2x 3 − 1 2 x n +1 − 1 2 x n +1 − 1 −1
⋅ ⋅ ... ⋅ = . Rezultă că lim ( y1y 2 ...y n ) = = 2 ; 24. Găsim a2 = 1, a3 = log13 17 > 1.
2x 2 − 1 2x n − 1 2 x1 − 1 n →∞ 2 x1 − 1
Avem a0 < a1 ≤ a2 < a3 şi prin inducŃie se arată că (an)n≥0 este crescător şi că an < 2, (∀) n ≥ 1. Urmează că (an)n
este convergent. Dacă lim a n = a ⇒ 13a = 12 a + 5a , ecuaŃie care are soluŃia unică a = 2; 25. a) InducŃie după n;
n →∞

b) a n +1 − a n = −a n a n < 0 ⇒ (a n ) n ≥1 este descrescător şi fiind şi mărginit, este convergent, cu lim a n = 0;


n →∞

c) Din faptul că 0 < an < 1, deducem a n < a n sau a 2n < a n a n , iar de aici a n +1 − a n = −a n a n <
< −a 2n . ObŃinem a 2n ≤ a n − a n +1 şi b n ≤ a1 − a 2 + a 2 − a 3 + ... + a n − a n +1 = a1 − a n +1 ≤ a1 ; 26. a) (xn) este strict
descrescător, mărginit inferior de zero, deci convergent. Fie l = lim x n , rezultă l2 ≤ k(l – l) şi de aici
n →∞
n n
xn 1  2 1  k
∑ ∑p
1 k 1 1 k
lim x n = 0 ; b) Avem ≤  x n + 2  ≤ ( x n − x n +1 ) + 2 , deci y n ≤ ( x p − x p +1 ) + 2
= ⋅
n →∞ n 2 n  2 2n 2 p =1
2 p =1
2
n n

∑p ∑p
1 1 1
⋅ ( x 1 − x n +1 ) + 2
. Dar xn → 0, iar 2
este convergent. Rezultă că (yn) este mărginit. cum yn+1 – yn =
2 p =1 p =1
x n +1
= > 0 , (yn) este monoton. Rezultă că (yn) este convergent; 27. RelaŃia x n +1 ≤ x 2n − λx n se scrie 0 < xn+1 ≤
n +1
≤ x n ( x n − λ) . Deducem că xn > λ, (∀) n ≥ 1. Dar xn+1 – xn ≤ xn(xn – 1 – λ) ≤ 0, de unde xn+1 ≤ xn, (∀) n ∈ N*.
Atunci şirul (xn)n≥1 este descrescător şi cum este mărginit, este convergent. Fie L = lim x n ∈ [λ,1 + λ] . Trecând
n →∞
la limită în relaŃia din ipoteză, obŃinem L ≤ L – λL, de unde L ≥ 1 + λ. Dar L ≤ 1 + λ, deci L = 1 + λ. Cum şirul
2

(xn)n≥1 este descrescător, obŃinem xn ≥ inf{xn | n ∈ N*} = lim x n = L = 1 + λ ≥ x n , (∀) n ∈ N. În concluzie xn =


n →∞

= 1 + λ, (∀) n ∈ N*; 28. Avem a 2n (a 2n +1 − 2a n +1 ) = 27 şi cum an+1 > 0, rezultă an+1 > 2, deci an > 2, n ≥ 1.
27 27
Deducem a n +1 = 1 + + 1 şi cum an > 2 ⇒ a n +1 < 1 + 1 + < 4 . Deci an ∈ (2, 4), n ≥ 1. De asemenea,
a 2n 4

318
27 27
2
− 2
a n +1 a n −1
an+2 – an = = x n (a n −1 − a n +1 ) , xn > 0. Rezultă an+2 – an = xn ⋅ xn–1(an – an–2), n ≥ 2. De aici
27 27
2
+1 + +1
a n +1 a 2n −1
subşirurile (a2n) şi (a2n+1) sunt monotone şi cum sunt mărginite, sunt convergente. Fie lim a 2 n = a şi
n →∞
lim a 2 n +1 = b , a, b ∈ [0, 4]. Rezultă a2b2 = 27 + 2ab2 şi a2b2 = 27 + 2ba2, de unde a = b = 3, deci lim a n = 3 ;
n →∞ n →∞
n n 1 2 3
29. Se arată, prin inducŃie, că a n < , (∀) n ≥ 1 ⇒ a n +1 + a n = , n ∈ N*. Deducem a 2 n = − + −
n +1 n +1 2 3 4
4 2n − 1  1   1   1   1   1  1 1 1 1 1 1
− + ... + = 1 −  − 1 −  + 1 −  − 1 −  + ... + 1 −  = 1 − + − + − ... − = +
5 2n  2   3   4   5   2 n  2 3 4 5 2 n n +1
1 1  2n 
+ + ... + . Rezultă lim a 2 n = ln 2 şi apoi lim a 2 n +1 = lim  − a 2 n  = 1 − ln 2 . Rezultă că (an) este
n+2 2n n → ∞ n → ∞ n → ∞  2n + 1 
divergent; 30. a) (1) ⇔ x 3n ( x n − x n +1 ) ≥ (xn – 3)2, (∀) n ≥ 1. Cum xn – xn+1 ≥ 0, găsim că xn > 0, (∀) n ≥ 1, adică
0 < xn ≤ x1, (∀) n ≥ 1. Deducem că (xn)n≥1 este mărginit, deci convergent. Dacă l = lim x n , deducem (l – 3)2 ≤ 0,
n →∞
de unde l = 3; b) Dacă xn < 0, (∀) n ≥ 1, din (1) rezultă că xn – xn+1 < 0, (∀) n ≥ 1, deci (xn)n≥1 este convergent.
Deducem că l = lim x n = 3 , ceea ce contrazice faptul că l ∈ [x1, 0]; 31. a) Cum x n +1 − x n = ax 2n + (b − 1) x n + c
n →∞

şi ax 2n + (b − 1) x n + c > 0 , (∀) n ≥ 1, deoarece are ∆ < 0, deducem că şirul (xn)n≥1 este strict crescător, deci există
lim x n = l ∈ (0, ∞). Dacă l ∈ (0, ∞), trecem la limită în relaŃia de recurenŃă şi obŃinem al2 + (b – 1)l + c = 0,
n →∞
xn+2
x x ...x x x ax 2 + bx + c
ecuaŃie care nu are soluŃii. Deci l = ∞; b) Putem scrie: lim 1 2 n n +1 = lim n2+ 2 = lim n +1 2 n +1 =
n →∞ x n +1 n →∞ x
n +1
n →∞ x n +1
x1x 2 ...x n
= a, de unde concluzia punctului b); 32. a) Se arată prin inducŃie: n − 1 ≤ a n ≤ n , (∀) n ≥ 1; b) Deducem
2
( n − 1) a 2n n a 2n
≤ ≤ , deci lim = 1 ; 33. Presupunem că există un astfel de şir, atunci xn > 0, (∀) n ≥ 1; xn+1 =
n n n n →∞ n
ln(1 + x n − 1) x −1 x
= ≤ n = x n − 1 ≤ n < x n , (∀) n ≥ 1. Rezultă că (xn)n este descrescător şi xn > 1, deci (xn)n
xn −1 xn −1 2
l
este convergent. Dacă l = lim x n , obŃinem l ≤ , deci l ≤ 0, fals; 34. Din ipoteză rezultă: x 3n ( x n +1 − x n ) ≤
n →∞ 2
≤ –(1 + xn)2 ≤ 0, (∀) n ≥ 1 şi cum (xn)n este strict crescător, rezultă că xn ≤ 0, (∀) n ≥ 1, deci (xn)n este
convergent. Fie l = lim x n . Trecând la limită, obŃinem l3(l – l) ≤ –(1 + l)2 ≤ 0 ⇒ l = –1, deci xn < –1, (∀) n ≥ 1;
n →∞

 π π  π π 
x n = cos 12 x n −1 + sin 12 y n −1 x  x   cos sin 
35. Avem  sau  n  = A ⋅  n −1  , A =  12 12  . Se obŃine  x n  =
 − sin π π y 
 y = − sin π x π  yn   y n −1   cos   n
 n n −1 + cos y n −1
12 12  12 12 

319
x  nπ nπ nπ nπ
= A n ⋅  0  , iar de aici x n = cos x 0 + sin y 0 , y n = − sin x 0 + cos y 0 , cu xn+24 = xn, yn+24 = yn,
y
 0 12 12 12 12
(∀) n ≥ 1, deci perioada căutată este T = 24; 36. Prin inducŃie se arată că xn ∈ (1, 2), pentru orice n ≥ 2. Avem
xn+1 – 1 = 1 − ( x n − 1) 2 , n ≥ 1. Notăm an = xn – 1 ∈ (0, 1) şi obŃinem a 2n +1 + a 2n = 1 , n ∈ N*. Din a 2n + 2 + a 2n +1 =
= 1 = a n2 +1 + a n , n ≥ 1, deducem an+2 = an, n ∈ N. Prin urmare a2 = a4 = a6 = ... şi a1 = a3 = a5 = ... . Şirul (an) este
2
convergent dacă şi numai dacă a2 = a1. Rezultă a1 = a2 = . Prin urmare (xn) este convergent dacă şi numai
2
2
dacă x1 = 1 + ; 37. Se constată că x0 < x1 < y0 şi x0 < x1 < y1 < y0. Prin inducŃie se arată că xn < xn+1 < yn+1 <
2
< yn. ObŃinem că şirurile (xn)n, (yn)n sunt mărginite şi monotone, deci convergente. ObŃinem şi că lim x n =
n →∞

 π θ θ
= lim y n = l ∈ R. Considerăm θ ∈  0,  astfel încât a = bcos θ. Găsim x1 = b cos 2 şi y1 = b cos şi prin
n →∞  2 2 2
n
θ θ b sin θ
inducŃie se arată că y n = ∏ cos 2 k
şi x n = y n cos
2 n
. De aici deducem y n =
θ
, deci lim x n =
n →∞
k =1 2 n ⋅ sin n
2
sin θ 12 + 2 2 + ... + (n + 1) 2 1
= lim y n = b ⋅ ; 38. Folosind Lema Stolz, limita este egală cu lim = ; 39. Folosim
n →∞ θ n → ∞ (2n + 1) 3
24
[( n + 1) p x ] [( n + 1) p x ] np x
Lema lui Stolz, limita cerută este egală cu lim = lim ⋅ = ;
n → ∞ ( n + 1) p +1 − n p +1 n →∞ n p p +1
(n + 1) − n p +1
p +1
a a2
40. a) an = a1 + (n – 1)r, cu a1 > 0, r > 0, deci lim a n = ∞ ; b) Notăm x n = 1 + + ... +
n →∞ a1 a1 + a 2
an xn 1 a n +1 1 r
+ . Cu Stolz avem lim = lim = lim =0;
a1 + a 2 + ... + a n n →∞ a n r n → ∞ a1 + a 2 + ... + a n +1 r n →∞
a n+2
1p + 2 p + ... + n p (n + 1) p 1
41. Se aplică lema Cesaro-Stolz. a) lim +
= lim p +1 p +1
= ; b) lim b n =
n →∞ n p 1 n → ∞ (n + 1) − n p +1 n →∞

(p + 1)(1p + 2 p + ... + n p ) − n p +1 (p + 1)(n + 1) p − (n + 1) p +1 + n p +1 1


= lim = lim = ; 42. a) Se aplică inegalitatea
n →∞ (p + 1)n p n →∞ (p + 1)(n + 1) p − (p + 1)n p 2
n
 ln a  ln a
lui Bernoulli; b) Trecem la limită în inegalitatea de la punctul a); lim 1 +  ≥ lim (1 + ln a ) ⇔ e ≥ 1 +
n → ∞ n  n →∞
n
+ ln a ⇔ a – 1 ≥ ln a, (∀) a ≥ 1; c) Conform punctului b) ⇒ n
k − 1 ≥ ln n k , (∀) k ≥ 1 ⇒ ∑(
k =1
n
k − 1) ≥

n n
ln 1 + ln 2 + ... + ln n ln 1 + ln 2 + ... + ln n ln(n + 1)
∑ ∑
ln k
≥ ⇒ n
k −n≥ şi cum lim = lim = ∞ , rezultă
n n n →∞ n n→∞ 1
k =1 k =1
a n +1 a a a
cerinŃa problemei; 43. Avem x1 + x2 + ... + xn = ln + ln n + 2 + ... + ln 0 = ln 0 şi de aici
an a n +1 a1 an

320
x 1 + x 2 + ... + x n

a0  1  n  n
x + x 2 + ... + x n
= e x 1 + x 2 +...+ x n sau a n = a 0 ⋅    . Deoarece lim x n = l ⇒ lim 1 =l≠0,
an   
e n → ∞ n → ∞ n
rezultă că lim a n = a 0 ⋅ 0l = 0 (căci l ≠ 0), deci (an)n este convergent. CondiŃia i) nu este necesară; 44. Folosind
n →∞

an   a
lema lui Stolz, lim = a . Rezultă lim (a n +1b n +1 − a n b n ) = lim  (a n +1 − a n )b n +1 + n (b n +1 − b n ) ⋅ n
 = ab + ca;
n →∞ n n →∞ n → ∞  n
45. Prin inducŃie matematică se arată că an ∈ (0, 1), (∀) n ≥ 1. Atunci an+1 – an = − a 2n < 0 , (∀) n ≥ 1 ⇒ (an) este
n n +1− n
descrescător şi mărginit, deci convergent. ObŃinem lim a n = 0 ; lim na n = lim = lim =
n →∞ n →∞ n →∞ 1 n →∞ 1 1

an a n +1 a n
= lim (1 − a n ) = 1 ; 46. 1) (an) este strict crescător. Dacă ar fi mărginit, ar fi convergent; l = lim a n ⇒ l = l +
n →∞ n →∞

1 a 3n  3a 2n
+ , fals. Deci lim a = ∞ ; 2) Folosind lema Stolz, lim = lim ( a 3
− a 3
) = lim  +
n → ∞ a 2 + a + 1
n n +1 n
l2 + l + 1 n →∞ n →∞ n n →∞
 n n

3a n 1 
+ + 2  = 3 . Deci limita cerută este egală cu 3
3 ; 47. a) Se arată prin inducŃie
3
(a 2n + a n + 1) 2
(a n + a n + 1) 
3( x n − x n −1 ) 1 − x 02
matematică că xn > 0, (∀) a ≥ 1. Deoarece x n +1 − x n = şi x1 − x 0 = < 0 , avem
( x n + 2)(x n −1 + 2) x0 + 2
sgn(xn+1 – xn) = sgn(xn – xn–1) = ... = sgn(x1 – x0) = –1, deducem (xn)n este strict descrescător şi cum xn > 0,
x −1
(∀) n, rezultă că (xn)n este convergent; b) Avem lim x n = 1 şi x n − 1 = n −1 , de unde xn > 1, adică
n →∞ x n −1 + 2
 1 
şirul   este nemărginit superior;
 x n −1 
xn −1 1 n 2 LemaS− C 1 (n + 1) 2 − n 2
lim n 2 ( x n − 1) = lim n 2 ⋅ = lim = lim =
n →∞ n →∞ xn +1 2 n →∞ 1 2 n →∞ 1

1
xn −1 x n +1 − 1 x n − 1
1 (2n + 1)(x n − 1) 1 1 n S− C 1 1 1
= lim = lim [2n ( x n − 1) + x n − 1] = lim = lim = ⋅
2 n →∞ xn +1 4 n →∞ 2 n →∞ 1 2 n →∞ 1

1 2
xn −1 x n +1 − 1 x n − 1
xn −1
⋅ lim = 0; 48. a) Se arată prin inducŃie că 0 < xn < 1, (∀) n ≥ 1. Se observă că xn+1 – xn = − x 4n < 0 , (∀) n ≥
n →∞ xn +1
≥ 1, adică şirul este monoton şi fiind şi mărginit, este convergent. Se obŃine că lim x n = 0 ; b) lim ( x14 + x 42 +
n →∞ n →∞

+ ... + x 4n ) = lim [( x1 − x 2 ) + ( x 2 − x 3 ) + ... + ( x n − x n −1 )] = lim ( x1 − x n −1 ) = x1 ; c) Avem că lim 3


n ⋅ xn =
n →∞ n →∞ n →∞

n  1  n +1− n
= lim . Deoarece   este şir nemărginit, deducem: lim 3
n ⋅ x n = lim =
n →∞ 3 1  x3  n →∞ n →∞ 3 1 1
 n  n ≥1 −
x 3n x 3n +1 x 3n

321
x 3n +1 ⋅ x 3n (1 − x 3n ) 3 1 1 1 1
= lim 3
3 3
= lim 3
3 3 2
= ; 49. Avem + xn = , deci −
n→∞ x n − x n +1 n → ∞ 1 + (1 − x n ) + (1 − x n ) 3
3 xn x n +1 x n +1
1
− = x n > 0 , rezultă că (xn) este strict descrescător şi cum (xn) este mărginit, rezultă că (xn) este convergent.
xn
n n +1− n
Găsim lim x n = 0 . Folosind Stolz, avem lim nx n x n = lim . Cum lim =
n →∞ n →∞ n →∞ 1 n →∞ 1 1

xn xn x n +1 x n +1 xn xn
1 1 1 1
= lim = , rezultă lim nx n x n = , deci lim n 2 x 3n = ; 50. a) Cum xn > 0,
n →∞ (1 + x n x n ) 2 + (1 + x n x n ) + 1 3 n →∞ 3 n →∞ 9

1 1 1 1 1
(∀) n ≥ 1, avem x 2n = + + ... + ⇒ x n +1 = x 2n + . Dar (xn+1 – xn)(xn+1 + xn) = > 0 ⇒ xn+1 > xn,
x1 x 2 x n −1 xn xn
1
(∀) n ≥ 1, deci (xn)n este strict crescător. Dacă (xn)n ar fi mărginit şi lim x n = l ⇒ l 2 = l 2 + , fals,
n →∞ l
x 3 3
x − xn 3  1 
deci lim x n = ∞ ; b) lim n = lim n +1 = lim (x n +1 − x n )(x 2n +1 + x n +1 ⋅ x n + x 2n ) = lim  x 2n + − xn  ⋅
n →∞ n →∞ n n →∞ n + 1 − n n →∞ n → ∞ xn 
 
1 1
xn + + x 2n + + xn
 1 1  x 2n xn 3 x 3
⋅  x 2n + + x n x 2n + + x 2n  = lim = , deci lim 3 n = 3 ; 51. lim (a x + 1 +
 xn xn  n →∞ 1 2 n →∞ n 2 x →∞
  x 2n + + xn
xn
− ∞, a + 2b + 3c < 0

+ b 4x + 1 + c 9x + 1) = + ∞, a + 2b + 3c > 0 ; 52. Dacă a ≠ 2, limita propusă nu este finită, deci a = 2. Găsim
0, a + 2b + 3c = 0

5
apoi b = − ; 53. i) Din lim (a x 2 + ax + b x 2 + bx − 2 x ) = ∞ ⋅ (a + b − 2) se impune condiŃia a + b – 2 = 0.
4 x→∞

 1 1 
 1 + a  2 − 1  b 2 
1 + 
 a    
Atunci avem: 1 = lim  a 1 +  − a +  b 1 + − b  = lim a ⋅ 
x
⋅a + b⋅ 
b x
⋅ b , de unde
x → ∞  x   a b
  x  x → ∞  
 x x 
 
a + b = 2. Găsim a = b = 1; ii) Avem (un)n, (vn)n, un = 1 + 2 + ... + n , vn = n . Deoarece vn > 0, (∀) n ≥ 1,
2 2 2 n n

u − un u
lim v n = ∞ şi lim n +1 = 1 . Aplicând lema lui Stolz-Cesaro, avem lim n = 1 , deci c = 1; iii) Avem
n →∞ n → ∞ v n +1 − v n n →∞ vn

n + 3 n +1 − n − 5 n +1 −2
m = lim = lim = −1 . ObŃinem funcŃia f: R → R,
n →∞ n →∞ n 3 n 5
n + 3 n +1 + n + 5 n +1 + + +
n +1 n +1 n + 1 n +1
ln(1 + ax + x 2 )
f ( x ) = 3 x 3 + 3x 2 + 2x + 1 − 3 x 3 − x + 1 ; 54. Dacă b ≠ 1, atunci l = lim = 0 , fals. Rezultă
x →0 x + b −1

322
ln(1 + ax + x 2 ) (ax + x 2 )( x + 1 + 1)
b = 1 şi l = lim ⋅ = 2a . Din 2a = 2 rezultă a = 1;
x →0 ax + x 2 x
 x 
ln1 + 2  
x x
ln(e 2 + 2x ) − ln(e 2 + x )  e +x 1  x + 1 − x  1∞ 1
55. a) lim = lim = 2 ; b) lim 1 +  = 2 |a −1|
,
x → ∞
x→0 x x→0 x
⋅ ( e 2
+ x ) e
 (a − 1) 2 x 2 + 1  e
2
e +x
1 3 5
de unde deducem că |a – 1 | = ⇔ a ∈  ,  ; 56. a) Se arată prin inducŃie matematică:
4 4 4
1

1 − cos x ⋅ cos 2x ⋅ ... ⋅ cos nx 1 2  sin x  x −1 1 ctg1
lim 2
= (1 + 2 2 + ... + n 2 ) , (∀) n ≥ 1; b) lim   =e ;
x→0 x 2 x →1 sin 1 
1 1
1+ x − m 1− x
n
(1 + x ) n − 1 − (1 − x ) m 1 1 m + n (x + a x ) x − 1
c) lim = lim = + = ; 57. a) lim =
x →0 x x→0 x n m mn x →0 (x + bx )x − 1
x x
e x ln( x + a )
−1 e x ln( x + a ) x ln( x + b x ) ln(1 + x + a x − 1) x + bx −1 x + ax −1
= lim x
= = lim ⋅ x
⋅ ⋅ ⋅ =
x →0 e x ln( x + b )
−1 x → 0 x ln( x + a x ) e x ln( x + b ) − 1 x + ax −1 ln(1 + x + b x − 1) x + b x − 1
x
 x + ax 
  −1 a
2
 bx  1 + ln 1
1 + ln a (x + a x ) x − b x
= ; b) lim = lim   = b ; 58. a) e a ln a ln b ; b) Este suficient să arătăm
1 + ln b
2 x
x →0 (x + b x ) x − a x x →0
 x + bx  b
  − 1 1 + ln a
 ax 
 
că dacă (xn)n≥1 are lim x n = ∞ , atunci lim f −1 ( x n ) = ∞ . Fie (xn)n≥1 strict crescător şi xn = f(yn). Atunci yn =
n →∞ n →∞
= f–1(xn). Cum (xn)n≥1 este strict crescător şi f este strict crescătoare, rezultă că (yn)n≥1 este strict crescător. Dar f
este monotonă şi (xn)n nemărginit, rezultă că (yn)n≥1 este nemărginit, deci lim f −1 ( x n ) = ∞ ;
n →∞
1 n n
ctgx x
 n   n  tgx 1∞ lim ∑ k 2 ⋅ tgx ∑k2 n ( n +1)( 2 n +1)

∏ ∑
x →0
59. lim 1 + ln (1 + k 2 x ) = lim 1 + ln(1 + k 2 x ) = e k =1
= e k =1 =e 6 ;
x →0  x →0
x >0  k =1 x >0  k =1 
  f (x)      f ( x)    f ( x)   g( x ) 
ln1 + ln1 +    ln1 + ln1 +   ln1 +  ln1 + 
ln(ln(e + f (x))  e      e 
= lim 
e
= lim   ⋅
e
60. lim ⋅ ⋅
x →a ln(ln(e + g( x)) x →a   g ( x )   x →a   f ( x) 
+
f ( x)   g( x )  
ln1 + ln1 +   ln 1  ln1 + ln1 + 
  e     e  e   e  

g( x ) 
f ( x )  1 
n k n  1  1 e − 1 1 n (n + 1) 
∑ ∑
f (x ) 1
⋅ e ⋅ = lim ; 61. Cum an = ⋅ en − k = en ⋅ 1 − ⋅  şi
 g( x )  g( x )  x → a g( x ) n  k =1 n  n n 2 
ln1 +    k =1 
 e n − 1 
 e  
 1  1 3
lim n a n − 1 = ln a , a > 0, rezultă că lim a n = e − 1 − = e − ; 62. Suntem în cazul 1∞. Cum
n →∞   n →∞ 2 2
 

323
n
 k  1 
  a n − 1 
 1   ∑  i 
1
n  1 
lim ∑ n  a in −1 
lim n a n − 1 = ln a, a > 0, limita cerută este egală cu lim 1 +
i =1    = e k n →∞ k =1  
 =
n →∞   n → ∞ k 
   
 
 
1
(ln a1 + ln a 2 + ...+ ln a k ) ln k a 1 a 2 ...a k ex − x − 1
= ek = e = k a1a 2 ...a k ; 63. Notăm f ( x ) =
şi lim f ( x ) = l ∈ R. Avem l =
x2 x →0

1  1   1   1 
= lim f   = lim  n 2 e n − n − n 2  şi l = lim f   = lim  4n 2 e 2 n − 2n − 4n 2  şi de aici l – 2l = lim (4n 2 ⋅
n → ∞  n  n → ∞  n → ∞  2n  n → ∞  n →∞
   
2
1 1   1  1
2
⋅ e 2n − 2n 2 e n − 2n ⇔ l = lim 2n 2  e 2 n − 1 ⇔ l = ; 64. Fie (pn), (qn) şiruri de numere naturale cu limita ∞,
 n →∞   2
  
astfel încât pn – q n 3 → 0 (de exemplu pn, qn care verifică egalitatea p n − q n 3 = (2 − 3 ) n ) . Rezultă f (p n −
− q n 3 ) = p n − q n 3 + q n ( 3 − 2 ) . Pentru n → ∞, deoarece f are limită în x = 0, rezultă lim f ( x ) =
x →0
x sin x
2 −1 3 −1
= lim f (p n − q n 3 ) = ∞ ; 65. Pentru x = 0 rezultă b = –2 şi |2x + 3sinx + ax – 2| ≤ |x3| ⇒ + +a ≤
n →∞ x x
≤ x2, (∀) x ∈ (–1, 0) ∪ (0, 1). Trecând la limită pentru x → 0, rezultă a = –ln 6; 66. Dacă există x0 ∈ R cu f(x0) <
< 1 ⇒ e x 0 < 0 , fals, deci f(x) > 1, (∀) x ∈ R. Deoarece ln x < x, (∀) x > 0 ⇒ ln f(x) < f(x) ⇒ f(x)ln f(x) < f2(x)
x
⇒ f(x) > e 2 , (∀) x ∈ R ⇒ lim f ( x ) = ∞ . Prin logaritmare, ipoteza devine ln x = ln[ln f(x) + ln (ln f(x))] ⇒
x →∞
f (x )
ln x ln[ln f ( x ) + ln(ln f ( x ))]  ln f ( x ) ln(ln f ( x )) ln f ( x )   ln x  x
⇒ lim = lim ⋅ + ⋅  = 0 . Rezultă lim 1 +  =
x →∞ f (x ) x → ∞ ln f ( x ) + ln(ln f ( x ))  f (x ) ln f ( x ) f (x )  x → ∞ f ( x ) 
f ( x ) ln x

 ln x  ln x x
= lim 1 +  = 1 ; 67. a) f este strict crescătoare pe R, deci injectivă. Cum f este continuă pe R şi
x → ∞ f ( x ) 
lim f ( x ) = –∞, lim f ( x ) = ∞ ⇒ Im f = R ⇒ f surjectivă, deci bijectivă; b) Cum f este strict crescătoare pe
x → −∞ x →∞

f −1 ( x ) y
R ⇒ f–1 este strict crescătoare pe R. Notăm f–1(x) = y ⇔ x = y3 + y. Avem lim n
= lim =
x →∞ y →∞ n
x y3 + y
0, n = 2

= 1, n = 3 ; 68. Fie ls = lim f ( x ) şi l d = lim f ( x ) . Avem echivalenŃele: ls = lim f ( x ) ⇔ (∀) (xn)n, xn <
x→x0 x→x0 x→x0
∞, n > 3 x<x0 x >x0 x<x0

< x0, lim x n = x 0 ⇒ lim f ( x n ) = ls şi l d = lim f ( x ) ⇔ (∀) (xn)n, xn > x0, lim x n = x 0 ⇒ lim f ( x n ) = l d .
n →∞ n →∞ x→x0 n →∞ n →∞
x >x0

1 1  1  1  1
Deoarece x 0 − < x 0 < x 0 + , (∀) n ≥ 1 şi lim  x 0 −  = lim  x 0 +  = x 0 , rezultă lim f  x 0 −  = ls ,
n n n → ∞ n  n → ∞ n n →∞  n

324
 1  1  1
lim f  x 0 +  = l d , deci ls = ld. Cum f este crescătoare, avem f  x 0 −  < f ( x 0 ) < f  x 0 +  , de unde ls ≤
n →∞  n  n  n
≤ f(x0) ≤ ld, adică lim f ( x ) = f ( x 0 ) ; 69. 1) Avem f(x) ≥ 0, (∀) x ∈ [0, 1]. Dacă a = b = 0 ⇒ f(0) ≤ 0, deci f(0) =
x→x 0
= 0 = min f ( x ) . Dar (∀) x ∈ [0, 1] ⇒ 1 – x ∈ [0, 1] şi x + (1 – x) = 1. Aşadar, f(x) ≤ f(x) + f(1 – x) ≤ f(1) = 1
x∈[ 0 ,1]

⇒ max f ( x ) = f (1) = 1 ; 2) Fie x, y ∈ [0, 1], x ≤ y ≤ 1. Cum x + (y – x) = y ⇒ f(x) ≤ f(x) + f(y – x) ≤ f(y), deci f
x∈[ 0,1]
este monotonă. Fie x1, x2, x3 ∈ (0, 1), x1 < x2 < x3. Dacă x1 < x < x2 ⇒ f(x1) ≤ f(x) ≤ f(x2) ⇒ f ( x1 ) ≤ lim f ( x ) ≤
x↑x 2
≤ f(x2) şi analog f ( x 2 ) ≤ lim f ( x ) ≤ f ( x 3 ) . Aşadar f(x2 – 0) şi f(x2 + 0) sunt finite, deci f are limite laterale finite
x↓ x 2

  1
0, x ∈ 0, 2 
  
în orice punct x0 ∈ (0, 1); 3) f: [0, 1] → [0, ∞), f(x) =  . Se arată că f verifică proprietăŃile a) şi b);
1, x ∈  , 1
1
 
2 
1 1 1 1
70. Fie f(0) = a. Dacă a ≠ 0, atunci pentru x = , y = 0 avem f   − a = f   ⋅ a ⇒ a = 0, fals, deci f(0) = 0.
a  
a a a
Pentru y = 0 obŃinem f(x) = xf(x) ⋅ f(0) + f(0) ⇒ f(x) = 0, (∀) x ∈ R. CondiŃia de continuitate este inutilă.

325
CLASA A X II - A

ALGEBRĂ

I. Legi de compoziŃie
1. a) e = 4 element neutru; b) Se arată că x  3 = 3  x = 3 şi se obŃine (–2010)  (–2009)  ...  2010 = 3; 2. a) x  x ≥
≥ 2008 ⇔ (x – 2008)2 ≥ 0, (∀) x ∈ R; b) Deoarece x  2008 = 2008  x = 2008, (∀) x ∈ R ⇒ c = 2008;
c) Fie H o parte stabilă finită a lui R în raport cu legea . Dacă x ∈ H, atunci x  x = (x – 2008)2 + 2008 ∈ H,
x  x  x = (x – 2)3 + 2008 ∈ H şi inductiv x

x  ...
  x = ( x − 2008) n + 2008 ∈ H. Cum H este finită, există m,
n de x

n ∈ N*, m > n, astfel încât (x – 2008) + 2008 = (x – 2008)n + 2008. Rezultă (x – 2008)n((x – 2008)m–n – 1) = 0,
m

deci x ∈ {2007, 2008, 2009}. Se verifică că H este una din mulŃimile {2008}, {2009}, {2007, 2009}, {2008,
2009}, {2007, 2008, 2009}; 3. Pentru (∀) x > –5, (∀) y > –5, trebuie ca să avem x & y > –5. Alegem şirul (yn)n,
yn ∈ Q, yn > –5 şi lim y n = −5 . Pentru x > –5, avem x & yn > –5 ⇒ lim ( x &yn) ≥ –5 ⇒ a ≥ 45. Dacă a ≥ 45,
n →∞ n →∞
(∀) x > –5, (∀) y > –5 ⇒ (x + 5)(y + 5) > 0 şi a – 50 ≥ –5 ⇒ x & y > –5; 4. a) Presupunem că x, y sunt două
ax = b x = ab
soluŃii. Atunci  ⇒ , deci x = y; b) Fie a ∈ A şi A = {a1, a2, ..., an}; σa(x) = ax funcŃie bijectivă,
ay = b  y = ab
σa(a) = a. Dacă σa(x) = ax şi ax ≠ x, (∀) x ≠ a ⇒ σa(x) ≠ x, (∀) x ≠ a. MulŃimea A – {a} poate fi împărŃită în
două submulŃimi disjuncte deoarece: [a ⋅ ai = aj ⇒ a ⋅ aj = ai] ⇔ [σ(ai) = aj ⇒ σ(aj) = ai]. Rezultă că A – {a} are
un număr par de elemente, deci A are un număr impar de elemente; 5. a) Dacă în i) considerăm y = z = e, atunci
x  x = e, (∀) x ∈ M; b) Observăm că e ∈ A, deci mulŃimea A este nevidă. Dacă x, y ∈ A, atunci x  y = (e  x) 
 (e  y) = y  x, adică elementele din A comută. Pentru x, y ∈ A, avem: e  (x  y) = (x  x)  (x  y) = y  x =
= x  y, unde am folosit rezultatul de la punctul a) şi faptul că elementele din A comută. Cu aceasta problema
este rezolvată.

II. Grupuri
6. a) Se arată că (∀) x, y > 1 ⇒ x  y > 1. Se verifică uşor că „” este asociativă, comutativă, are element neutru e =
x
= 2 şi orice element x ∈ G este simetrizabil, simetricul lui x fiind x ' = ∈ G; b) Din condiŃiile
x2 −1
f (xy) = f (x)  f (y),(∀) x, y > 0
 ⇒ m + n = 2 şi xy + n = m2 + y + (mn – m)x + (mn – m)y + n2 – 2n + 2, (∀) x, y > 0,
f (1) = 2
deducem m = n = 1. Se verifică apoi că funcŃia f ( x ) = x + 1 este izomorfism; c) Se ştie că f–1 este izomorfism
de grupuri şi f–1(x) = x2 – 1, (∀) x > 1. Notăm x  x  ...  x = a. Avem f–1(a) = f–1(x  x  ...  x) =

326
= f–1(x) ⋅ ... ⋅ f–1(x) = (f–1(x))2010, de unde a2 – 1 = (x2 – 1)2010, deci a = 1 + ( x 2 − 1) 2010 . Găsim x = 2 ;
7. a) (∀) x, y ∈ G ⇒ x & y ∈ G. Într-adevăr, cum x, y ≠ –a ⇒ b(x + a)(y + a) ≠ 0 ⇒ x & y ≠ –a. Se arată că legea
1
„&” este asociativă, comutativă, are element neutru e = − a şi orice element x este simetrizabil, x' =
b
2 2 2
1 − a b − ab x
= ∈ G, deci (G, &) este grup; b) Din comutativitate deducem d & x = d, (∀) x ∈ R ⇒ bdx +
b 2 (x + a)
+ abd + abx + a2b – a = d sau d(bx + ab – 1) + a(bx + ab – 1) = 0 ⇒ (bx + ab – 1)(d + a) = 0, (∀) x ∈ R ⇒ d +
+ a = 0 ⇒ d = –a; c) (–2010a) & (–2009a) & ... & 0 & a & ... & (2010a) = –a < 0; 8. b) FuncŃia f: G → R*+, f(x) =
= x + 5 este izomorfism de grupuri; c) PărŃile stabile cerute sunt: {–4}, {–5}, {–5, –4}, {–6, –4}, {–6, –5, –4};
9. Observăm că (∀) x, y ∈ G ⇒ (x – 3)(y – 3) > 0 ⇒ x & y > 3, deci x & y ∈ G. Legea „&” este asociativă,
comutativă, are element neutru e = 4 ∈ G şi orice element x ∈ G este simetrizabil, simetricul lui x este x' = 3 +
1
+ ∈ G, deci (G, &) este grup abelian. Se arată că funcŃia f: G → R, f(x) = ln(x – 3) este izomorfism de la (G, &)
x −3
la (R, +); 10. Din (x & y) & z = x & (y & z), (∀) x, y, z ∈ M, deducem a2 = –a, de unde rezultă că a ∈ {0, –1}. Dacă a = 0
⇒ M = R*+ şi x & y = x ⋅ y, deci (M, &) este grup. Dacă a = –1 ⇒ M = (–1, ∞) şi x & y = xy +
+ x + y şi se arată că (M, &) este grup. În concluzie a ∈ {0, –1}; 11. a) Se arată că (∀) x, y ∈ G ⇒ x & y ∈ G;
( x + y + z)(1 + a ) 2 + xyz
(∀) x, y, z ∈ G, (x & y) & z = x & (y & z) = . Elementul neutru este 0 şi orice element
xy + yz + zx + (1 + a ) 2
x ∈ G este simetrizabil, simetricul lui x fiind (–x) ∈ G. Cum legea este şi comutativă, urmează că (G, &) este
3n − 1 x+y
grup abelian; b) ⋅ (a + 1) , n ∈ N*; 12. G = (–1, 1) împreună cu operaŃia x & y = , (∀) x, y ∈ G
n
3 +1 1 + xy
formează o structură de grup abelian. Fie f: G → G1, f(x) = ax + b bijectivă şi f(x & y) = f(x) z f(y) şi g: G → G2,
g(x) = x2n+1 o funcŃie bijectivă şi g(x & y) = g(x) S g(y). Rezultă că (G1, z) şi (G2, S) sunt grupuri abeliene
izomorfe;
13. a) Deoarece {x} = x + [x], (∀) x ∈ R şi [x + k] = [x] + k, (∀) x ∈ R, k ∈ Z, deducem: (x  y)  z = x  (y  z) =
={x + y + z}, deci operaŃia „” este asociativă. OperaŃia este comutativă, elementul neutru este e = 0 şi orice
element x ∈ G este simetric şi anume x' = {1 – x}, deci (G, ) este grup; b) Fie orice x, y ∈ Hn ⇒ x =
'
d1 d d  d  d   d 
= , y = 2 . Dacă y' este simetricul lui y şi considerăm x > y, avem: x  y' =  1    2  =  1   1 − 2  =
n n n   n  n   n 
 1 x f (x ) 
 d1 d 2   d1 − d 2  d1 − d 2  
=  +1−  =  = ∈ Hn, deci H este subgrup al lui G; 14. a) Din  0 1 x ⋅
n n   n  n 0 0
 1 
 1 y f ( y)   1 x + y f ( y) + xy + f ( x ) 
   
⋅ 0 1 y  = 0 1 x+y  ∈ G. ObŃinem f(x + y) = f(x) + f(y) + xy, (∀) x, y ∈ R. Căutăm o
0 0 1  0 0 1 
   
1
soluŃie particulară sub forma f0 = ax + bx + c, a, b, c ∈ R. ObŃinem f0: R → R, f 0 ( x ) = x 2 + bx , b ∈ R. Dacă
2
2
f este o altă soluŃie a ecuaŃiei funcŃionale, avem f(x + y) = f(x) + f(y) + xy, f0(x + y) = f0(x) + f0(y) + xy ⇒ (f –
– f0)(x + y) = (f – f0)(x) + (f – f0)(y), (∀) x, y ∈ R şi f – f0 = g. Cum f, f0 sunt continue ⇒ f – f0 este continuă şi

327
  1 2  
 1 x x + αx  
1 2   2  
atunci (f – f0)(x) = dx ⇒ f ( x ) = x + αx , α ∈ R; b) Urmează că G = A x =  0 1 x  α ∈ R .
2  0 0  
1
   
   
Din relaŃiile Ax ⋅ Ay = Ax+y, A0 = I3 şi A −x1 = A − x , rezultă că G este grup; 15. Axiomele se verifică uşor iar
izomorfismul este dat de funcŃia F: G → Z, F(fn) = n; 16. Deoarece Xa ⋅ Xb = Xa+b+ab şi cum a + b + ab = (a +
+ 1)(b + 1) – 1 > –1, (∀) a, b > –1, rezultă că G este parte stabilă în raport cu înmulŃirea matricelor. Se verifică
imediat axiomele grupului. FuncŃia f: G → R, f(Xa) = ln(a + 1) este izomorfism; 17. b) (∀) a, b ≠ –1 ⇒ a + b +
+ ab = (a + 1)(b + 1) – 1 ≠ –1, deci G este parte stabilă în raport cu înmulŃirea. ÎnmulŃirea matricelor este
asociativă, elementul neutru este I2 = X(0) ∈ G şi orice element X(a) ∈ G este simetrizabil, simetricul X'(a) =
 −a 
= X  ∈ G; c) X(1) ⋅ X(2) ⋅ ... ⋅ X(2010) = X(2011! – 1); 18. a) Axiomele se verifică uşor iar izomorfismul
 a +1
 a 0 bi 
 
este dat de funcŃia f: C* → G, f(a + ib) =  0 0 0  ; b) Observăm că f(1 + i) = A şi folosind proprietăŃile
 bi 0 a 
 
izomorfismului rezultă: f[(1 + i) ] = A . Cum (1 + i)2009 = 21004(1 + i), rezultă A2009 = f(21004 + 21004i) =
2009 2009

 21004 0 21004 i 
 
= 0 0 0  ; 19. b) Dacă A, B ∈ G ⇒ A + I2, B + I2 sunt inversabile ⇒ A  B + I2 = (A + I2)(B + I2)
 1004 
2 i 0 21004 
 
inversabilă, deci A  B ∈ G. Legea „” este asociativă (calcule), are element neutru (O2) şi orice element este
simetrizabil (simetricul lui A ∈ G este A' = (A + I2)–1 – I2), deci (G, ) este grup; 20. ObŃinem:
 0 0 0 xy   0 xy 0 0 
   
 0 0 x 0  3  0 0 0 xy 
A =
2
;A =  ; A4 = xyI4. Cum detA ≠ 0, G conŃine numai matrice inversabile.
0 y 0 0 y 0 0 0
   
1 0 0 0  0 0 x 0 
   
Dacă E este elementul neutru al grupului (G, ⋅), atunci AE = A pentru orice A din G. Rezultă E = I4 ∈ G. Prin
urmare, există p ∈ N* astfel încât Ap = I4, rezultă xy = 1 şi cum x, y ∈ [–1, 0), avem x = y = –1. Astfel A4 = I4 şi
G = {I4, A, A2, A3}. Se verifică uşor că (G, ⋅) este grup; 21. Fie e elementul neutru. Dacă y = e ⇒ f(x) =
= x  f(f(e)) = x  α, unde α = f(f(e)). Dacă f(x) = f(y) ⇒ x  α = y  α ⇒ x = y ⇒ f este injectivă. Avem f(x) =
= f(e  x) = e  f(f(x)) = f(f(x)), adică f(f(x)) = f(x) şi cum f este injectivă ⇒ f(x) = x, (∀) x ∈ G. Deci f = 1G; dar
1G(x  y) = x  y = x  1G(1G(y)) ⇒ 1G ∈ A. Rezultă |A| = 1; 22. Pentru x = e avem f(f(f(y))) =
f injectiva
= f(f(y)) ⇒ f(f(y)) = f(y) ⇒ f(y) = y. RelaŃia dată devine x2x3y = xyx2 ⇒ x4y = yx2. Pentru y = e ⇒ x2 = e ⇒
grupul (G, ⋅) este abelian; 23. b) f: Ma → Z, f([(a + i)4] = n este izomorfism de grupuri; c) Dacă
z ∈ M1 ∩ M2, rezultă că există n, k ∈ Z, astfel încât (1 + i)n = (2 + i)k ⇒ |(1 + i)n| = |(2 + i)k| ⇒ 2n = 5k, egalitate
posibilă numai dacă n = k = 0. Rezultă că M1 ∩ M2 = {1}; 24. a) Se arată că (∀) a, b ∈ G ⇒ a  b ∈ G. Deoarece
k 2 (a + b + c) + abc
(a  b)  c = a  (b  c) = = , (∀) a, b, c ∈ G, deci operaŃia „” este asociativă. Elementul
k 2 + ab + bc + ac
neutru este e = 0 ∈ G şi simetricul lui a ∈ G este a' = –a ∈ G, deci (G, ) este grup abelian; b) Deoarece

328
1 k+t 1 (k + a )(k + b)
f (t) = ln , se arată că f este fijectivă. Cum f (a  b) = ln = f (a ) + f (b) , (∀) a, b ∈ G,
2k k − t 2k (k − a )(k − b)
rezultă că f este izomorfism; 25. b) g(x) = x este automorfism ⇒ axa–1 = x, (∀) x ∈ G şi (∀) a ∈ G ⇒
ax = xa ⇒ G abelian; h(x) = x–1 este automorfism ⇒ h(x) = g(x), (∀) x ∈ G ⇒ x = x–1 ⇒ x2 = e, (∀) x ∈ G;
26. b) Pentru orice fa, fb ∈ Int(G) ⇒ (fa  fb)(x) = fa(fb(x)) = abxb–1a–1 = (ab)x(ab)–1 = fab(x), (∀) x ∈ G,
de unde rezultă că fa  fb = fab ∈ Int(G), (∀) fa, fb ∈ Int(G). Deoarece fa ∈ Int(G) este inversabilă şi
−1
f a ( x ) = a −1x (a −1 ) −1 = f a −1 ( x ) ⇒ f a−1 = f a −1 ∈ Int(G), deci Int(G) este subgrup al grupului (Aut(G), ); c)
Deoarece 1G: G → G, 1G(x) = x, (∀) x ∈ G este automorfism, rezultă că 1G = fa, de unde axa–1 = x, (∀) x ∈ G şi
(∀) a ∈ G ⇒ ax = xa, (∀) x, a ∈ G ⇒ G este comutativ; 27. Dacă e ∈ G – H şi luăm x ∈ H ⇒ x = x ⋅ e ∈ G – H,
fals, deci e ∈ H. Fie x ∈ H şi dacă x–1 ∈ G – H, obŃinem e = x ⋅ x–1 ∈ G – H, fals, deci x–1 ∈ H, (∀) x ∈ H.
Pentru (∀) x, y ∈ H, dacă x ⋅ y ∈ G – H, avem x–1 ∈ H şi y = x–1 ⋅ (xy) ∈ G – H, fals. Urmează că H este
x ∈ A ⇒ x p ∈ H
subgrup; 28. a) Evident e ∈ A şi e ∈ B, deci A ≠ ∅, B ≠ ∅; b) Fie x ∈ (A – B) ∩ H ⇒  . Dar
x ∈ H
H este subgrup ⇒ xp+1 = xp ⋅ x ∈ H ⇒ x ∈ B, fals; 29. f(x ⋅ y) = f(x) ⋅ f(y) ⇔ (xy)2009 = x2009 ⋅ y2009; g(xy) = g(x) ⋅ g(y)
⇔ (xy)2010 = x2010 ⋅ y2010. Dar (xy)2010 = (xy)2009 ⋅ xy = x2009 ⋅ x ⋅ y2009 ⋅ y ⇒ y2009 ⋅ x = x ⋅ y2009. Cum f este
surjecŃie ⇒ (∀) z ∈ G, (∃) y ∈ G, încât f(y) = z ⇔ y2009 = z. Atunci z ⋅ x = x ⋅ z, (∀) x, z ∈ G ⇒ G este abelian;
30. a) InducŃie după m; b) Avem succesiv: y4 = (y2)2 = (xyx–1) ⋅ (xyx–1) = xy2x–1 = x2yx–2; y8 = (y4)2 = (x2yx–2) ⋅
⋅(x2yx–2) = x2y2x–2 = x3yx–3; y16 = (y8)2 = (x3yx–3) ⋅ (x3yx–3) = x3y2x–3 = x4yx–4; y32 = (y16)2 = (x4yx–4) ⋅ (x4yx–4) =
= x4y2x–4 = x5yx–5 = y, deci y31 = e (s-a folosit x5 = x–5 = e); 31. a) (xyx−1)2009= (xyx−1) ⋅ (xyx−1) ⋅ ...⋅ (xyx−1) = xy2009x−1 ;
  
2009
b) (yxy–1)4 = yx4y–1 = e ⇒ yx4 = y ⇒ x4 = e. Din xy = yx2 ⇒ x = yx2y–1 ⇒ x2 = yx4y–1 = e ⇒ xy = y ⇒ x = e;
32. RelaŃia ababa = babab se scrie astfel: ababa(ba)–1 = babab(ba)–1 ⇔ aba = babab(ba)–1 ⇔ (ba)–1aba =
= (ba)–1babab(ba)–1 ⇔ (ba)–1aba = bab(ba)–1 ⇔ x–1ax = = xbx–1, unde x = ba. Folosind formulele: (x–1ax)n =
= x–1anx, (xbx–1)n = xbnx–1, deducem că (x–1ax)n = (xbx–1)n ⇒ x–1anx = xbnx–1. Dacă an = e, atunci xbnx–1 = e ⇒
bn = e, iar dacă bn = e, atunci x–1anx = e ⇒ an = e; 33. a) Trebuie arătat că: 1) (∀) x, y ∈ Ha ⇒ x ⋅ y ∈ Ha; 2) (∀)
x ∈ Ha ⇒ x–1 ∈ Ha. Fie (∀) x, y ∈ Ha ⇒ x = ak, y = ap ⇒ x ⋅ y = ak+p ∈ Ha. Fie (∀) x ∈ Ha ⇒ x = ak ⇒ x–1 =
= a–k ∈ Ha, deci H este subgrup; b) ( ⇐ ) Dacă există h, k ∈ Z, h ≠ k astfel încât ah = ak ⇒ ah–k = e (în ipoteza că
h > k) ⇒ Ha = {e, a, a2, ..., ah-k-1}, deci Ha este finit; (⇒) Dacă Ha este finit, cum a, a2, a3, ... ∈ Ha, există k,
h ∈ Z, k ≠ h astfel încât ah = ak (în caz contrar, Ha ar fi infinită); 34. a) Dacă G = G1 ∪ G2, G1, G2 sunt
subgrupuri proprii, atunci există x ∈ G1 – G2 şi y ∈ G2 – G1 şi avem: dacă z = xy ∈ G1 ⇒ y = x–1z ∈ G1, fals;
dacă z = xy ∈ G2 ⇒ x = zy–1 ∈ G2, fals; b) Grupul lui Klein K = {e, a, b, c} se scrie K = K1 ∪ K2 ∪ K3, K1 =
= {e, a}, K2 = {e, b}, K3 = {e, c}.

III. Grupuri finite


35. a) Avem fa  fb = fa+b, ga  gb = fa–b, fa  gb = ga+b şi ga  fb = ga–b şi deci G este parte stabilă. Legea „” este
asociativă, elementul neutru este f0 ∈ G şi orice element este simetrizabil (fa' = f–a şi ga' = g–a). Cum ga  gb ≠ gb 
 ga ⇒ (G, ) este grup necomutativ; b) f ak = f ka ≠ f 0 , (∀) a ≠ 0, k ∈ N*; ga =f0 ⇒ ord(ga) = 2 ⇒ A =
2

x = a −1b
= {1, 2}; 36. Avem  ⇒ a–1b = b–1a ⇒ ba–1b = a ⇒ a–1ba–1b = e ⇒ (a–1b)2 = e. Dar a–1b nu poate avea
−1
x = b a
ordinul 2 deoarece G are ordin impar, deci ord(a–1b) = 1 ⇒ a–1b = e ⇒ b = a. Deci, dacă a = b ⇒ x = e, dacă

329
a ≠ b ⇒ sistemul nu are soluŃie; 37. a) Deoarece σ = (1 2 3) (4 5 6 7), iar ciclii disjuncŃi (1 2 3) şi (4 5 6 7)
comută şi au ordinele 3, respectiv 4 în grupul S7, rezultă că σ are ordinul 12. Atunci H = {e, σ, σ2, ..., σ11}
reprezintă subgrupul ciclic generat de σ. Cum H este ciclic şi ord(H) = 12, rezultă că H este izomorfism (Z12, +),
1 2 3 4 5 6 7 a b c d e f g
deci m = 12; b) Fie θ =   . Atunci π = θ ⋅ σ ⋅ θ–1 =   = (a b c)(d e f g).
a b c d e f g b c a e f g d
Cum π = (2 3 7)(1 4 5 6), iar descompunerea în produs de cicli disjuncŃi este unică, rezultă că (a b c) = (2 3 7);
1 2 3 4 5 6 7
(d e f g) = (1 4 5 6). Putem alege θ =   ; c) Fie A mulŃimea permutărilor din S7 care se
 2 3 7 1 4 5 6
scrie ca produs de doi cicli disjuncŃi de lungime 3 şi respectiv 4. Din b) rezultă că xσx–1 ∈ A, (∀) x ∈ S7, deci
1 2 3 4 5 6 7
M ⊂ A. Reciproc: dacă π = (a b c)(d e f g) ∈ A şi θ =   , rezultă θ ⋅ σ ⋅ θ–1 = σ ∈ M,
a b c d e f g
astfel A = M. Deoarece cu orice submulŃime cu k elemente a mulŃimii {1, 2, ..., 8} putem forma (k – 1)! Cicli
disjuncŃi de lungime k, rezultă că M are C37 ⋅ 2!⋅3! = 420 elemente; 38. Fie G = {e, a, x1, x2, ..., xn}. Dacă a2 ≠ e,
atunci a ≠ a–1, a–1 ∈ G, deci (a-1)2 = e, deci a2 = e, contradicŃie. Rezultă că a2 = e; 39. Cum ord(e) = 1, rezultă că
mulŃimea formată din ordinul elementelor este {1, 2, 3, ..., n}. Dacă n = 2, atunci x2 = e, (∀) x ∈ G, deci x = x–1,
(∀) x ∈ G. Rezultă xy = (xy)–1 = y–1x–1 = yx, (∀) x, y ∈ G. Reciproc: Fie x, y ∈ G, ord x = n – 1, ord y = n. Cum
G este abelian, ord(xy) = = (n – 1)n. Din (n – 1)n ≤ n, n ≥ 2, rezultă n = 2; 40. Pentru x = e ⇒ f(f(y)) = yf(e),
(∀) y ∈ G ⇒ f injectivă. Pentru y = e ⇒ f(xf(x)) = f(x2), (∀) x ∈ G şi cum f injectivă ⇒ xf(x) = x2 sau f(x) = x,
(∀) x ∈ G ⇒ f bijectivă. RelaŃia din enunŃ devine x2y = yx2, G are 2n + 1 elemente, deci x2n+1 = e, (∀) x ∈ G.
Avem xy = x2n+2y = (xn+1)2y = y(xn+1)2 = yx2n+2 = yx; 41. a) ⇒ b). Fie x ∈ G ⇒ H = {x, x2, ..., xk, ...} este parte
stabilă ⇒ H este subgrup ⇒ e ∈ H. Deci există n ≥ 1, xn = e ⇒ ord(x) finit; b) ⇒ a). Fie H ⊂ G o parte stabilă.
Dacă arătăm că (∀) x ∈ H ⇒ x–1 ∈ H, atunci H este subgrup. Fie x ∈ H şi ord(x) = n ⇒ xn = e ⇒ xn–1 ⋅ x = x ⋅
⋅ xn–1 = e ⇒ x–1 = xn–1. Dar xn–1 ∈ H ⇒ x–1 ∈ H; 42. Dacă G – {a, b} este subgrup ⇒ n – 2 | n ⇒ n ∈ {3, 4}.
Dacă n = 3 ⇒ G = {e, x1, x2} ⇒ G – {x1, x2} = {e} este subgrup. Dacă n = 4 ⇒ G = {e, x1, x2, x3}. Cum G j K
sau G j Z4 ⇒ există un element de ordin 2, fie x1, x12 = e . Atunci G – {x2, x3} este subgrup; 43. a) ord(a) = 3,
ord(b) = 2 şi elementele e, a, a2 sunt distincte şi b ∉ {e, a, a2}. Elementele b, ba, ba2 sunt distincte şi diferite de e,
a, a2 şi cum ordG= 6 ⇒ G = {e, a, a2, b, ba, ba2}. Cum c ∉ {e, a, a2, b} ⇒ c = ba sau c = ba2; b) c = ba ⇒ c2 =
= baba = bbaa = b2a2 = a2 ≠ e; c3 = b ≠ e şi cum ord(c) divide ordG = 6 ⇒ ord(c) = 6. Dacă c = ba2 ⇒ c2 =
= ba2ba2 = baaba2 = baba2a2 = baba = bba3 = e ⇒ ord(c) = 2; 44. Conform teoremei lui Lagrange, p | 625, de
unde p ∈ {5, 25, 125, 625}. Dacă p = m ⋅ n cu m, n ∈ N* – {1}, atunci pentru un element a ∈ G, avem ap = e ⇔
⇔ amn = e ⇔ (am)n = e, deci elementul am ∈ G ar avea ordinul ≤ m < mn = p, fals. Urmează că p = prim, deci p =
 a b  
= 5. Fie G = M2(Z5) =   a , b, c, d ∈ Z 5  . Avem ord(G) = 625 şi toate elementele au ordinul 5, căci
 c d  
a b a b  a b   0̂ 0̂ 
  +   + ... +   =   ; 45. a) Fie H ⊂ G cu cel puŃin  n  + 1 elemente. Fie a ∈ G un element
 0̂ 0̂  2
 c d   c d   c d     
oarecare. Considerăm funcŃia f: H → G, f(x) = ax . Observăm că f este injectivă, deci card(Im f) = card(H)≥
–1

n 
≥   + 1 , deci Im f ∩ H are cel puŃin un element y ⇒ există x ∈ H astfel ca f(x) = y ⇒ ax–1 = y ⇒ a = yx, ceea
2
ce trebuia demonstrat; b) Fie K = {e, a, b, c} grupul lui Klein. MulŃimea H = {e, a} este subgrup, şi nu este
e , x ∈ H
generator pentru K; 46. Fie L2(G1) = {y ∈ G1 | y2 = e1} şi a ∈ L2(G1). Definim fa: G → G1, fa(x) =  1 .
a, x ∈ G − H

330
Vom arăta că fa este morfism de grupuri: fa(x & y) = e1 = e1 ⋅ e2 = fa(x) ⋅ fa(y), (∀) x, y ∈ H; fa(x & y) = e1 = a2 =
= a ⋅ a = fa(x) ⋅ fa(y), (∀) x, y ∈ G – H; fa(x & y) = a = e1 ⋅ a = fa(x) ⋅ fa(y), (∀) x ∈ H şi y ∈ G – H. Fie N(H) =
= {f: G → G1, f morfism şi f(H) = {e1}}. Definim F: L2(G1) → N(H), F(a) = fa. Se arată că F(b) bijectivă. Dacă
F(a) = F(b) ⇒ fa = fb şi dacă c ∈ G – H avem a = fa(c) = fb(c) = b ⇒ F injectivă. Fie f ∈ N(H), c ∈ G – H şi
a = f(c). Cum c & c ∈ H şi f(H) = {e1} ⇒ f(c & c) = e1, deci f2(c) = e1. Deducem a2 = e1, deci a ∈ L2(G1). Dacă
x ∈ G – H, cum x & c ∈ H ⇒ e1 = f(x & c) = f(x) ⋅ f(c) = f(x) ⋅ a, deci f(x) = a–1 = a. Rezultă f(x) =
e , x ∈ H
= 1 = fa(x), adică f = fa = F(a) ⇒ F este surjectivă. Cum G1 este finit, rezultă că L2(G1) este finită şi
a , x ∈ G − H
deci |L2(G1)| = |N(H)|; 47. Fie (G1, ⋅) un grup abelian finit şi e1 elementul neutru. Dacă p = ∏ x , atunci p
x∈G 1
2
=

= ∏ (x ⋅ x
x∈G 1
−1
) = e1 , (1). Notăm cu q = ∏ x şi fie H un subgrup al lui G cu proprietatea (A) şi α = ∏ x , iar
x∈G 1 x∈H

β= ∏ x . Cum α ⋅ β = q şi α = β rezultă α
x∈G − H
2
= q. Însă din (1) obŃinem α = e, deci q = e. Fie H1 ≠ G un subgrup
2

oarecare al lui G şi α1 = ∏x , β
x∈H1
1 = ∏ x . Cum α
x∈G − H1
1 ⋅ β 1 = q = e ⇒ β1 = α1−1 . Dar α12 = e (din (1)), rezultă

α1−1 = α1, deci α1 = β 1.

IV. Inele. Corpuri


48. a) Pentru x = –1, (–1)2 = –1 ⇒ 1 + 1 = 0; b) Fie x, y ∈ A, atunci (x + y)2 = x + y ⇒ x2 + xy + yx + y2 = x + y,
deci xy + yx = 0, cum 1 = –1, rezultă xy = –yx = yx; c) Fie x ∈ A, x ≠ 0, cum A este corp din x2 = x, rezultă x = 1.
Prin urmare A = {0, 1}; 49. Înlocuim y cu y + x ⇒ x ⋅ (y2 + yx + xy + x2 – y – x) = (y2 + yx + xy + x2 – y –
– x) ⋅ x, de unde rezultă că x ⋅ (y2 – y) + x2y = (y2 – y) ⋅ x + yx2 şi cum x ⋅ (y2 – y) = (y2 – y) ⋅ x ⇒ x2y = yx2,
(∀) x, y ∈ A. Dar xy2 – xy = y2x – yx, de unde rezultă xy = yx, deci A este inel comutativ; 50. a) Deoarece
f(x, y) = (xy)2 – x2y2 = xyxy – xxyy = x(yx – xy)y, se poate scrie: f(1 + x, y) = (1 + x)(yx – xy)y, f(x, 1 + y) =
= x(yx – xy)(1 + y), f(1 + x, 1 + y) = (1 + x)(yx – xy)(1 + y) ⇒ E(x, y) = yx – xy; b) Scriind relaŃia (xy)2 –
– (yx)2 = x2y2 – y2x2 pentru x + 1 şi y rezultă prin scădere xy2 = y2x. La fel, scriind pentru x şi y + 1, se obŃine
(xy – yx) + (xy – yx) = 0. Din ipoteză rezultă xy = yx; 51. Vom arăta că a2 = a, b2 = b şi c2 = c. Într-adevăr, a2 =
= a ⋅ a = aba = a ⋅ bca = abc = ab = a; b2 = b ⋅ b = bcb = bcab = bca = bc = b; c2 = c ⋅ c = cac = cabc = cab = ca =
= c. Avem (a – b)2 = (a – b)(a – b) = a2 – ab – ba + b2 = a2 – a – bca + b2 = a – a – b + b = 0, deci a – b = 0 ⇒ a =
 2̂ 3̂ 0̂ 1̂   0̂ 2̂ 0̂ 2̂   2̂ 1̂ 0̂ 3̂ 
  
 3̂ 0̂ 1̂ 2̂   2̂ 0̂ 2̂ 0̂   1̂ 0̂ 3̂ 2̂ 
= b. Analog b = c; 52. a) A =   , 2A =   , 3A =   = C şi 4A = O4, deci
 0̂ 1̂ 2̂ 3̂   0̂ 2̂ 0̂ 2̂   0̂ 3̂ 2̂ 1̂ 
 1̂ 2̂ 3̂ 0̂   2̂ 0̂ 2̂ 0̂   3̂ 2̂ 1̂ 0̂ 
  
a b
H = {A, B, C, O4}; b) Alcătuim tabla adunării şi constatăm că (H, +) este grup; 53. Fie A =   ∈ M2(Z3).
c d
Avem relaŃia A2 – (a + d)A + (ad – bc)I2 = O2. Cum G este grup finit cu trei elemente, rezultă A3 = I2, deci det(A) =
= 1̂ . ObŃinem A2 – (a + d)A + I2 = O2, (1) ⇒ A3 – (a + d)A2 + A = O2 ⇒ –(a + d)A2 + A + I2 = O2, (2). Din

331
(1) şi (2) obŃinem (1̂ + a + d)[(1̂ − a − d)A + I 2 ] = O 2 . Dacă 1̂ + a + d ≠ 0̂ , obŃinem (a + d − 1̂)A = I 2 , adică
(a + d − 1̂) ⋅ a = 1̂

(a + d − 1̂) ⋅ b = 0̂
 . Din prima găsim că a + d − 1̂ este element inversabil în Z3, deci a = d = ( a + d − 1̂ )–1 şi b =
(a + d − 1̂) ⋅ c = 0̂

(a + d − 1̂) ⋅ d = 1̂
= c = 0̂ , apoi înlocuind în prima ecuaŃie, obŃinem a = d = 1̂ , contradicŃie cu 1̂ + a + d ≠ 0̂ . Rămâne astfel că
1̂ + a + d = 0̂ , adică a + d = 2̂ şi ad − bc = 1̂ . Rezultă (a, d) ∈ {(0̂, 2̂); (2̂, 0̂); (1̂, 1̂)} , adică A ∈ {A1, A2, A3, A4, A5,
 0̂ 1̂           
A6, A7, A8}, unde A1 =   ; A 2 =  0̂ 2̂  ; A 3 =  2̂ 1̂  ; A 4 =  2̂ 2̂  ; A 5 =  1̂ 1̂ ; A 6 =  1̂ 2̂  ;
 2̂ 2̂   1̂ 2̂   2̂ 0̂   1̂ 0̂   0̂ 1̂  0̂ 1̂ 
           
1̂ 0̂   
A7 =   ; A 8 =  1̂ 0̂  . łinând cont că A12 = A 4 , A 22 = A 3 , A 52 = A 6 , A 72 = A8 , găsim grupurile G1 = {I2, A1,
1̂ 1̂   2̂ 1̂ 
   
A4}; G2 = {I2, A2, A3}; G3 = {I2, A5, A6}; G4 = {I2, A7, A8}; 54. Fie n cu proprietatea cerută, atunci n ≥ 4. Din x + y =
= 2z şi xy = z2 rezultă (x – y)2 = 0̂ . Dacă n = 4, x – y = 2̂ , deci x = 3, y = 1, deci 2z = 0̂ , z 2 = 3̂ , fals. Nu
convine n = 5 (deoarece a 2 ≠ 0̂ , a ∈ Z*5). Analog în cazurile n = 6 şi n = 7. În cazul n = 8 rezultă x – y = 4̂ ,
deci (x, y) ∈ ∈ {(5̂, 1̂); (6̂, 2̂); (7̂, 3̂)} , care nu convin. Cum în Z9, 2̂ + 8̂ = 2 ⋅ 5̂ şi 2̂ ⋅ 8̂ = 5̂2 , rezultă că n minim
căutat este egal cu 9; 55. Fie p = detA. Deoarece A ⋅ A* = pI3 ⇔ det(A ⋅ A*) = p3 ⇔ det(A) ⋅ det(A*) = p3. Cum
det(A*) = 4̂ şi p ≠ 0̂ , avem că p 2 = 4̂ , deci p ∈ {2̂, 5̂} ; A ⋅ A* = pI3 ⇔ A = p ⋅ (A*)–1. Dar (A*)–1 = 4̂ −1 ⋅
 2̂ 4̂ 0̂   2̂ 4̂ 0̂   4̂ 1̂ 0̂   1̂ 2̂ 0̂ 
       
* −1
⋅  0̂ 2̂ 4̂  = 2̂ 0̂ 2̂ 4̂  =  0̂ 4̂ 1̂  . Dacă p = 2̂ , atunci A = 2̂ ⋅ (A ) =  0̂ 1̂ 2̂  . Dacă p = 5̂ , atunci
       
 4̂ 0̂ 2̂   4̂ 0̂ 2̂   2̂ 0̂ 4̂   2̂ 0̂ 1̂ 
       
 6̂ 5̂ 0̂ 
   â b̂ 
* −1
A = 5̂ ⋅ (A ) =  0̂ 6̂ 5̂  ; 56. Avem detX6 = detA = 0̂ ⇒ detX = 0̂ . Dacă X =   ⇒ detX = â 2 − b̂ 2 =
   b̂ â 
 5̂ 0̂ 6̂   
 
= 0̂ ⇔ â 2 = b̂ 2 . Dar â 2 , b̂ 2 ∈ {0̂, 1̂, 2̂, 4̂} ; â 2 = b̂ 2 = 0̂ ⇒ â = b̂ = 0̂ nu convine; â 2 = b̂ 2 = 1̂ ⇒ (â , b̂) ∈ {(1̂,1̂);
⌢⌢
ˆ 2);(2,5);(5,
(1̂, 6̂); (6̂,1̂); (6̂, 6̂) ; â 2 = b̂2 = 2̂ ⇒ (â, b̂) ∈ {(3̂, 3̂); ( 4̂,4̂); (3̂, 4̂); (4̂, 3̂)} ; â 2 = b̂2 = 4̂ ⇒ (â, b̂) ∈ {(2, ˆ ˆ ˆ ˆ 2),(5,5)}
ˆ .
⌢ ⌢ ⌢ ⌢ ⌢ ⌢ ⌢ ⌢ ⌢ ⌢ ⌢ ⌢ ⌢

{
Prin verificare găsim (a, b) ∈ (1, 6), (6,1),(3, 4), (4, 3), (2,5), (5, 2) }.

332
ANALIZĂ MATEMATICĂ

I. Primitive
'
x2 −1  1 
 x + + 1 2
x2  x   1   1 
1. a) I1 = ∫  x2 + x +1
2
dx = ∫  1 
2
dx , deci I1 = ln  x + + 1 +  x + + 1 − 2 + C;
 x   x 
2
  −2  x + + 1 − ( 2 )
 x   x 
 
(3x + m)(3x 2 + 2mx + 5) + (3x + m) 1 (3x 2 + 2mx + 5)' 1 (3x 2 + 2mx + 5)'
b) Avem: I 2 = ∫ (3x 2 + 2mx + 5) n
dx = ∫
2 (3x 2 + 2mx + 5) n −1
dx + ∫
2 (3x 2 + 2mx + 5) n
dx =

1 1 a a
=− n −2
− n −1
+ C ; 2. Se găseştew A(x) = 2 şi B(x) = − x −
2
2(n − 2)(3x + 2mx + 5) 2
2(n − 1)(3x + 2mx + 5) λ λ
'
b 3x 4 + 2 x 3 + x 2 − 2010  −x  x

λ
; 3. ∫ 4 3 2
( x + x + x + 2010)
dx =  4 ∫3 2  dx = − 4
 x + x + x + 2010 
3 2
x + x + x + 2010
+ C ; 4. a) I1 = x2 +

1  2x + 3 2x + 3 
+ 3x + ln(x2 + 3x + 3) + C; I2 = ln(x2 + 3x + 3) – 2
x + 3x + 3 ∫
+ C ; b) I n =  2
 (x + 3x + 3)
n −1
+ 2 n
(x + 3x + 3) 
dx

1 1 1 1
⇒ In = − ⋅ − ⋅ + C ; 5. Deoarece f(x) =F' (x), obŃinem (e2xF(x))' =
n − 2 ( x 2 + 3x + 3) n − 2 n − 1 ( x 2 + 3x + 3) n −1
1 2 x 1 2x 1
∫e
2x
= e2xsin2 3x, deci e2xF(x) = sin 2 3xdx ⇒ e 2 x F( x ) =
e − e (3 sin 6x + cos 6x ) + C , deci F(x) = −
4 40 4
1 1
− (3 sin 6x + cos 6x ) + ce − 2 x . Cum F este mărginită pe R şi lim F( x ) = c ⋅ ∞ , rezultă c = 0 şi F( x ) = −
40 x → −∞ 4
1 π π 1 π 2− 2
− (3 sin 6x + cos 6x ) . F are perioada T = ; 6. a) f   = − ∈ Q şi f   = ∈ R – Q; b) Deoarece
40 3  
4 2 8 4
cos 2x cos 4x 1 π 3 π
F(x) = − + C şi F(0) = C + ∈ Q ⇒ C ∈ Q, cum F  = − + C , urmează concluzia F  ∈ Q;
4 8 8 2 8 2
12 5 12 5
7. a) Se obŃine A =
13
şi B = ; b)
13 ∫ f (x)dx = 13 x + 13 ln(3sin x + 2 cos x) + C ; 8. Presupunem că f admite
 1 1 2
xarctg x + 2 ln( x + 1) + C1 , x < 0

primitive, atunci acestea au forma F: R → R, F( x ) = k , x = 0 . Cum F este continuă,
 1 1
xarctg + ( x 2 + 1) + C 2 , x > 0
 x 2
 π π
rezultă C1 = C2 = k. FuncŃia obŃinută este derivabilă pe R*  Fs' (0) = − , Fd' (0) =  .
 2 2

333
SoluŃie alternativă.
 1 π
arctg x , x < 0 2 , x<0  π
  − 2 , x < 0
 π π 
Avem f ( x ) = − , x=0 +  , x=0 + 0, x = 0 = f1 (x) + f 2 (x) + f3 (x) . Cum f1 şi f2
 2 2 π
 π  1  , x>0
− 2 , x>0 arctg x , x>0 2
 
sunt continue, au primitive. Dacă f are primitive, rezultă că f3 are primitive, fals, deoarece f3 nu are proprietatea
sin x cos x
Darboux; 9. a) Se notează I1 = ∫
sin x + cos x
dx , I 2 = ∫ sin x + cos x
dx . Avem I1 + I2 = x, I2 – I1 = ln|sin x +
x 1 1
+ cos x|, deci I1 = − ln | sin x + cos x | + C ; b) Se integrează prin părŃi, apoi se face substituŃia =t.
2 2 x
arcsin x 1 1 1+ 1− x2
ObŃinem ∫ x2
dx = −
x
arcsin
x
− ln
x
+ C; 10. Fie J = ∫ cos x ⋅ ln(2 + sin 2x)dx . Atunci:
I + J = ∫ (sin x + cos x ) ln(2 + sin 2x )dx = ∫ (sin x + cos x ) ln(3 − (sin x − cos x ) )dx = ∫ ln(3 − t )dt = tln(3 – t ) –
2 2 2

3−t
– 2t – 3 ln
3+t
+ C, t = sin x – cos x şi J – I = ∫ (cos x − sin x) ln(2 + sin 2x)dx = ∫ (cos x − sin x) ln(1 +
∫ ln(1 + u
2
+ (sin x + cos x ) 2 )dx = )du = u ln(1 + u 2 ) − 2u + 2arctgu + C, u = sin x + cos x. Se deduce I; 11. Pentru
x x  π
x ∈ (0, π), cu substituŃia t = tg (unde ∈  0,  , deci are sens tangenta), obŃinem integrala asociată
2 2  2
1 2 2 2 t 1 2
∫ ⋅
1− t 1+ t2
2
dt = ∫ 5+ t 2
dt =
5
arctg
5
+ C, deci, pe acest interval,
3 + 2 cos x ∫
dx =
5

3+ 2⋅
1+ t2
x
tg
⋅ arctg 2 + C. Procedăm analog pentru x ∈ (π, 2π) şi atunci primitivele funcŃiei date (care există, f fiind
5
 x
 2 tg
 arctg 2 + C1 , x ∈ (0, π)
 5 5

continuă), sunt F(x) = a , x=π , unde a, C1 şi C2 sunt astfel încât F să fie derivabilă.
 x
 tg
 2
arctg 2 + C 2 , x ∈ (π, 2π)
 5 5
π π
Impunem întâi ca F să fie continuă şi obŃinem că C1 = a − , C2 = a + , iar în continuare se arată uşor că,
5 5
2
pentru orice valoare a lui a, funcŃia F astfel determinată este derivabilă, iar derivata sa este f; 12. a) F( x ) = ⋅
7

334
x
2tg +1
⋅ arctg 2 +C este primitivă a lui f pe [ 0, π ) ; b) Fie H: [0, 2π) → R, H(x) =
7
 x
 2 2 tg + 1
2 , x ∈ [0, π)
 arctg
 7 7
 2π π
= k ' , x=π . Din continuitatea lui H deducem k = , k' = şi astfel H este
 7 7
x
 2 tg + 1
 2 arctg 2 + k, x ∈ (π, 2π)
 7 7
x
1 + 2tg
1 2
primitiva lui f pe [0, 2π); 13. Pentru x ∈ [0, π) şi x ∈ (π, 2π] avem ∫ 2 + sin x
dx =
3
arctg
3
2 + C. F o

primitivă
 x
 2 1 + 2tg
2 + C , x ∈ [0, π)
 arctg 1
 3 3

a lui f este de forma: F( x ) = C 2 , x=π . Din continuitate găsim C1 = C 2 −
 x
 1 + 2tg
 2 arctg 2 + C , x ∈ (π, 2π]
 3 3
3

π π sin x + 1 sin x + 1 3 sin x − 1 1 1 t6 +1


− , C3 = C 2 + ; 14. Notăm 3 =t⇒ = t . Atunci 3 =− , = şi
3 3 cos x cos x cos x t cos 2 x 2t 3
6t 2 3 3  t4 t2 1 1  3 sin x + 1
dx = 6
t +1
dt . ObŃinem I =
2 ∫
( t 3 − t + t − 3 − t − 5 )dt =  − − 2 + 4  , deci I =
2  4 2 2t 4t 

8 cos x

2 2
sin x + 1 3 3  sin x + 1  3  cos x  3 cos x 3 cos x
⋅3 − ⋅   − ⋅3   + ⋅ ⋅ + C; 15. Avem:
cos x 4  cos x  4  sin x + 1  8 sin x + 1 sin x + 1
ln x + 2009 x ln x + 2009x ( x + 1)(ln x + 2010) dx
∫ ( x + ln x + 2010) 2
dx = ∫
x ( x + ln x + 2010) 2
dx = ∫
x ( x + ln x + 2010) 2
dx −
x ( x + ln x + 2010)
=∫
'
 1  dx ln x + 2010 dx
= − ∫
 x + ln x + 2010 
 ⋅ (ln x + 2010)dx − ∫
x ( x + ln x + 2010)
=−
x + ln x + 2010
+
x ( x + ln x + 2010)
− ∫
dx ln x + 2010 x sin x + cos x sin x − x cos x
− ∫
x ( x + ln x + 2010)
=−
x + ln x + 2010
+ C; 16. a) I1 = ln
x cos x − sin x
+ C; b) I2 =
x sin x + cos x
+ C;

sin x e x + cos x
17. a) Considerăm I1 = ∫
e x + sin x + cos x
dx şi I 2 = ∫
e x + sin x + cos x
dx şi avem I1 + I2 = x, I1 – I2 = ln|ex +

1 1 1 1  4 1 
+ sin x + cos x|, deci I1 = x − ln | e x + sin x + cos x | + C; b) I + J = arctg  x − 4  + C; I – J =
2 2 4 2 2 x 

335
1
− 2x4 +
1 x4 x2 − x (e x + x 2 + x + 1) − (e x + 2x + 1)
= ln
8 2 x4 + 1 + 2
+ C şi apoi se deduce I; 18. a) ∫ ex + x 2 + x + 1
dx = ∫ ex + x 2 + x + 1
dx =

x4
 (e x + x 2 + x + 1)'  2x + 1
= ∫ 1 − x  dx = x − ln( e + x + x + 1) + C; b) Avem I = ∫
x 2
2 2
dx =
 e + x + x + 1  ( x + x − 2)(x 2 + x ) + a
dy dy dy
= ∫ 2
y − 2y + a
= ∫ 2
( y − 1) + a − 1
(am făcut schimbarea x2 + x = y); 1) Dacă a < 1 ⇒ I = ∫
( y − 1) − ( 1 − a ) 2
2

1  x2 + x −1− 1− a  1 1
⇒I= ⋅ ln 2  + C; 2) Dacă a = 1 ⇒ I = + C; 3) Dacă a > 1 ⇒ I = ⋅
2 1− a  x + x −1+ 1− a  1− x2 − x a −1
 
x2 + x −1 sin 2 x + sin 2x + 1 e x (sin 2 x + 1) + e x ⋅ sin 2x
⋅ arctg
a −1
+ C; 19. Avem ∫ e − x + sin 2 x + 1
dx = ∫ 1 + e x (sin 2 x + 1)
dx =

[1 + e x (sin 2 x + 1)]' 1
= ∫ x 2
1 + e (sin x + 1)
dx = ln[1 + e x (sin 2 x + 1)] + C; 20. Fie t = arctg x. Atunci x = tg t, dt =
1+ x2
dx , 1 +

1 tg t − t sin t cos t − t cos 2 t


+ x2 = 1 + tg 2 t =
cos 2 x
. Avem de calculat: 2 ∫ 1
⋅ t 3
dt = 2 ∫ t3
dt . Folosim integrarea prin

cos 2 t
 1  t cos2 t − sin t cos t 2t sin t cos t − sin2 t t cos2 t − sin t cos t sin2 t

părŃi: 2 (sin t cos t − t cos2 t ) − 2 dt =
 2t  t2
+ ∫
t2
dt =
t2
+
t
+

 x 
arctg − 2 
t − sin t cos t  1 + x  + C; 21. Avem e x ⋅ 1 dx =
+C=
t2
+ C. Rezultă că primitivele căutate sunt:
(arctgx ) 2 cos 2 x ∫
∫ ∫
= e x ( tgx )' dx = e x tgx − e x ⋅ tgxdx şi ∫e ln cos xdx = ∫ (e x ) 'ln cos xdx = exlncos x + ∫ e tgxdx , deci avem
x x

'
 1   2 2 2
∫ ex  2
 cos x
+ ln cos x dx = e x ( tg x + ln cos x ) + C; 22. Avem  x 2 cos  = 2 x cos + 2 sin , (∀) x ≠ 0.
  x  x x
 2
 x cos , x ≠ 0
FuncŃia g: R → R, g(x) =  x este continuă pe R, deci are primitive; fie G o primitivă. Se arată că
 0, x=0

 x2 2
 cos − G ( x ) + C, x ≠ 0
funcŃia F: R → R, F(x) =  2 x este primitivă pentru f; 23. FuncŃia G: R → R, G(x) =
− G (0) + C, x=0

 3 1  1 2 1
x cos 2 , x ≠ 0 2 sin 2 + 3x cos 2 , x ≠ 0
= x este derivabilă şi G'(x) = g(x) =  x x . FuncŃia h: R → R, h(x) =
0, x = 0 0, x = 0
 

336
 2 1
3x cos 2 , x ≠ 0
= x este continuă, deci are primitive şi deoarece 2f = h + g ⇒ f are primitive; 24. Fie funcŃia
0, x = 0

 2  1
x ⋅  cos  ln x, x > 0 F( x ) − F(0)
F: [0, ∞) → R, F(x) =   x . Deoarece − 0 ≤ x ln x , (∀) x > 0 şi lim x ln x = 0
x x→0
0, x=0 x >0

rezultă că F'(0) = 0, iar cum F este derivabilă pe (0, ∞) ⇒ F este derivabilă pe [0, ∞); F'(x) =
  1   1
2x ⋅  cos  ln x, x > 0 x  cos , x > 0
  x   x + f (x )
= + 0, şi cum funcŃiile g şi h sunt continue ⇒ f are
0, x=0  x=0
(= g) (= h )
'
 1 1 1 1 1 1
primitive pe [0, ∞); 25.  x 2 ln x sin  = 2x ln x sin + x sin − ln x cos , (∀) x > 0 ⇒ ln x cos = x sin ⋅
 x x x x x x
'
 1 1
⋅ (2 ln x + 1) −  x 2 ln x sin  . Fie funcŃiile g: (0, ∞) → R, g( x ) = x sin ⋅ (2 ln x + 1) . Cum lim g( x ) = 0 ⇒
 x x x ↓0

 1
x sin ⋅ (2 ln x + 1), x > 0
⇒ g(x) =  x este continuă, deci are primitive (fie G o primitivă). Dacă f are primitive,
0, x=0
 2 1
G ( x ) − x ln x sin + C, x > 0
atunci o primitivă a ei are forma F(x) =  x ; lim F( x ) = G (0) + C, F'(0) = G'(0), de
x→0
0, x=0 x >0

f ' (x )
unde găsim că m = 0; 26. Din ipoteză ⇒ f'(x) = 2010f(x) ⇒ = 2010 ⇒ lnf(x) = 2010x + a ⇒ f(x) =
f (x)
f (x) 1
= e 2010 x + a şi cum f(0) = 1 ⇒ a = 0, deci f(x) = e2010x. ObŃinem ∫ f 2 ( x ) + 2010
dx =
2010 2010
arctge 2010x + C;

x 1− x2 x f ' (x ) 2x
27. Avem F(x) = f ( x ) , ( ∀ ) x > 0 ⇒ F' ( x ) = f (x ) + 2 f ' (x ) ⇒ =x+ 2 , ( ∀) x >
x2 +1 ( x 2 + 1) 2 x +1 f (x) x +1
2
x2
x2 +a  x 2 + 1  x 2−1
> 0 ⇒ ln f(x) = + ln( x 2 + 1) + a ⇒ f ( x ) = ( x 2 + 1)e şi cum f(1) = 1 ⇒ f ( x ) = 
2 e
 ;
2  2 
x x 2 2x
28. a) Derivând, obŃinem: f ( x ) = f ( x ) − − 2 + (2f ' ( x ) − 1) + 2
, (∀) x > 0 ⇒ f'(x) = 1 + 2 , ( ∀) x >
2 2 1+ x x +1
1 x 4019
> 0 ⇒ f(x) = x + ln(x2 + 1) + C; b) ∫ x(x 4020
+ 1)
dx = ∫ x 4020 ( x 4020 + 1)
dx . Cu substituŃia x4020 = t, integrala

1 dt 1 t dx 1 x 4020
devine
4020 ∫ = ln
t ( t + 1) 4020 t + 1
+ C. Rezultă: ∫ x(x 4020
+ 1)
= ln 4020
4020 x +1
+ C; 29. a) Observăm că

f ' (x )
f'(x) ≠ 0, (∀) x ∈ R ⇒ f' are semn constant pe R, deci f este strict monotonă; b) Avem = 1 , ( ∀) x ∈
2
f (x) + 4

337
∈ R ⇒ există a ∈ R astfel încât ln(f ( x) + f 2 (x ) + 4 ) = x + a , (∀) x ∈ R, de unde deducem f(x) + f 2 (x ) + 4 =
1 x +a 1 2 3 1
= ex+a şi apoi f ( x ) = e − 2e − x − a . Dar f (0) = e a − a = − , de unde deducem a = 0, deci f ( x ) = e x −
2 2 e 2 2
f ' ( x )
– 2e–x; 30. a) F(x) = f(x)e–x, (∀) x ≥ 0 ⇒ f(x) = f'(x) ⋅ e–x – f(x) ⋅ e–x ⇒ = e x + 1 , (∀) x ≥ 0 ⇒ lnf(x) = ex +
f (x)
x
+ x + a ⇒ f(x) = e e + x + a şi cum f(0) = e ⇒ a = 0, deci f ( x ) = e e + x , (∀) x ≥ 0; b) Deoarece F'(x) = f(x) ≠ 0,
x

(∀) x ∈ R ⇒ F' are semn constant pe R ⇒ F este strict monotonă, deci F este injectivă. Dacă înlocuim pe x cu 0
1
şi apoi cu 1, obŃinem F(0) = F(0) ⋅ F(1) = F(1), de unde F(0) = F(1), fals; 31. h(x – a) = (x – a)f(x); f(x) = ⋅
2a
1
⋅ (g(x) – h(x – a)) şi F( x ) = (G ( x ) − H( x − a )) + C este o primitivă a lui f; 32. a) Avem F(F–1(x)) = x, (∀) x ∈
2a
'
1 1 
∈ J ⇒ F'(F–1(x)) ⋅ (F–1)'(x) = 1, (∀) x ∈ J şi astfel (F −1 )' ( x ) = −1
, x ∈ J;  [F −1 ( x )]2  = F −1 ( x ) ⋅
f (F ( x ))  2 
1 F −1 ( x ) 1 e x + e−x
⋅ (F −1 ( x )) ' = F −1 ( x ) ⋅ −1
f (F ( x ))
, x ∈ J, deci ∫ −1
f (F ( x ))
dx = [F −1 ( x )]2 + C, x ∈ J; b) Fie f(x) =
2 2
,x∈

e x − e−x
∈ R şi F: R → R, F(x) = . Avem F' = f, F bijectivă şi cum F −1 ( x ) = ln( x + x 2 + 1) , x ∈ R ⇒
2
ln( x + x 2 + 1) F −1 ( x ) 1
⇒ f( F −1 ( x ) ) = x 2 + 1 , deci ∫ x2 +1
dx = ∫ f (F−1
( x ))
dx = ln 2 ( x + x 2 + 1) + C; 33. Fie
2
F2 (x ) sin 2 x
G, H: R → R, G(x) = ln(1 + sin2 x) şi H(x) = G(x) – , (∀) x ∈ R. H este derivabilă şi H'(x) = –
2 1 + sin 2 x
F2 (x ) l 2
– F(x) ⋅ f(x) ≥ 0, (∀) x ∈ R, deci există lim F( x ) = l ∈ R ∪ {±∞}, rezultă că există lim = ∈ R ∪ {∞}.
x→∞ x →∞ 2 2
π 
Atunci funcŃia G are limită la ∞, fals (deoarece G(nπ) → 0 şi G  + 2nπ  → ln 2 ); 34. Considerăm funcŃia
2 
x
f1: [0, ∞) → R, f1(x) = F(x) – x + ln(x + 1). Avem f1' ( x ) = f ( x ) − ≤ 0 , deci f1 este descrescătoare. Cum x ≥
x +1
≥ 0, rezultă f1(x) ≤ f1(0), de aici F(x) – x ≤ F(0) – ln(x + 1). Deoarece lim (F(0) − ln( x + 1)) = −∞ , rezultă că
x→∞
există lim (F( x ) − x ) = −∞ . La fel considerând funcŃia f2: (–∞, 0] → R, f2(x) = F(x) – x – ln(1 – x), deducem
x→∞
lim (F( x ) − x ) = ∞ . Prin urmare ecuaŃia F(x) – x = 0, x ∈ R, are cel puŃin o soluŃie. Unicitatea soluŃiei rezultă
x → −∞
din stricta monotonie a funcŃiei g: R → R, g(x) = F(x) – x; 35. a) Putem presupune că f este crescătoare. La fel se
va demonstra dacă funcŃia este descrescătoare. Demonstrăm că funcŃia f2 este funcŃie continuă. Avem de analizat
mai multe cazuri.
Dacă f(x) ≥ 0, (∀) x ∈ R, din condiŃia că f este crescătoare, rezultă că f2 este crescătoare şi cum are
proprietatea lui Darboux, rezultă că f2 este continuă şi deci f4 este continuă, de unde rezultă că f4 admite pri-
mitive pe R.
Dacă f(x) ≤ 0, (∀) x ∈ R, se procedează analog, folosind funcŃia g:= –f.

338
În cazul general, să notăm I:= {x ∈ R, f(x) ≥ 0} şi J:= {x ∈ R, f(x) < 0}. Din monotonia funcŃiei f rezultă că
există x0 ∈ R, astfel încât I = [x0, ∞) şi J = (–∞, x0) sau I = (x0, ∞) şi J = (–∞, x0]. În mod analog se arată că f2
este continuă pe I şi pe J şi cum are proprietatea lui Darboux, ea este continuă pe I ∪ J = R. Deci f4 este continuă
x + 1, x ≥ 0
pe R, de unde rezultă că admite primitive pe R; b) Fie f: R → R, f ( x ) =  . FuncŃia f2 este continuă pe
 x − 1, x < 0
R şi deci admite primitive pe R, iar funcŃia f3 nu are proprietatea lui Darboux, deci nu admite primitive pe R;
36. a) (∀) c ∈ (a, b), definim gc: [a, b] → R, gc(x) = F(x) – xf(c). FuncŃia gc este derivabilă şi g 'c ( x ) = f ( x ) − f(c).
Deducem că g 'c ( x ) < 0 , (∀) x ∈ [a, c) şi g 'c ( x ) ≥ 0 , (∀) x ∈ [c, b]. Rezultă că gc nu este injectivă, deci există

∫x
2
x1 ∈ [a, c) şi x2 ∈ [c, b] astfel ca gc(x1) = gc(x2), de unde rezultă concluzia problemei; b) (1 − x 3 ) 2010 dx =

( x 3 − 1) 2011
=
3 ⋅ 2011
+ C; 37. Fie f0 astfel de funcŃie şi F elementul neutru al grupului. Cum f ( x )dx = F( x ) + C, ∫
rezultă F(F(x) + c) = F(x) + c, (∀) x, c ∈ R. Pentru x = 0, F(F(0) + c) = F(0) + c, de unde F(c) = c, (∀) c ∈ R.
Rezultă că grupul căutat este x + c = {fc: R → R, fc(x) = x + c, c ∈ R}; 38. Compunerea funcŃiilor este lege de
compoziŃie internă pe F deoarece (f  g)'(x) = f'(g(x)) ⋅ g'(x) ∈ G. Compunerea funcŃiilor este asociativă, iar
funcŃia 1R: R → R, 1R(x) = x, (∀) x ∈ R este din F şi este element neutru în raport cu compunerea funcŃiilor.
Cum f': R → R are proprietatea lui Darboux, atunci f'(x) > 0, (∀) x ∈ R sau f'(x) < 0, (∀) x ∈ R ⇒ f este strict
monotonă, deci injectivă. FuncŃia f este şi surjectivă, rezultă că f este bijectivă, deci inversabilă. Cum f'(x) ≠ 0,
1
(∀) x ∈ R ⇒ f–1 este derivabilă şi (f −1 )' ( y) = −1
∈ G, deci F este grup.
f ' (f ( y))

II. Integrale
39. FuncŃia este continuă pe R*, iar limitele laterale în x = 0 sunt egale cu 0, prin urmare f este continuă pe R,
1 0 1 π − 2 ln 2
deci integrabilă pe orice compact. Avem
−1 −1 ∫
f ( x )dx = f ( x )dx + f ( x )dx =
0 ∫ 4 ∫
; 40. Facem schimbarea
2
a ln x 2 a a ln a 4 a a ln u
x = a ⋅ u . ObŃinem ∫1 x (x + a)
dx =
a ∫ 1
u2 +1
du +
a ∫ 1
u2 +1
du . În a doua integrală facem
a a

1 a ln u a2 ln x 2 a  1 
schimbarea u = şi obŃinem
t ∫ 1
a
u2 +1
du = 0 . Deci ∫1 x (x + a )
dx =
a
ln a ⋅  arctg a − arctg

 ;
a
1
41. a) Dacă notăm f ( x ) = arctgx + arctg
, cum f'(x) = 0, (∀) x > 0 ⇒ f este constantă pe (0, ∞), deci f(x) =
x
π a arctgx 1
= f(1) =
2
, (∀) x > 0; b) Dacă notăm I = 1
a
x ∫
dx , cu schimbarea de variabilă
x
= t , rezultă I =

1 1
arctg arctgx + arctg
a a a π π ln a
= ∫1
a
t
t dt , de unde I + I =
∫1
a
x
x dx =
∫ 1
a
2x
dx = πln a, deci I =
2
; 42. Deoarece

339
π 2 π
2 cos nx − cos 2 (n − 1) x
cos nx – cos (n – 1)x = –sin x ⋅ sin(2n – 1)x, obŃinem ∫ dx = ∫ π [ − sin( 2n − 1) x ]dx =
2 2 2
π
sin x
3 3

π π
cos(2n − 1) 1
cos(2n − 1) x

2 3 ; 43. Fie I =
= =− 2010 x 2010 arccos 2010 xdx , cu substituŃia 2010x = t. ObŃinem
2n − 1 π 2n − 1 −
1
3 2010

1 1 1 −1
∫t ∫ u 2010 arccos(−u )du =
2010
I= arccos tdt = ⋅ A . Cu substituŃia t = –u, obŃinem A = −
2010 2011 −1 2010 2011 1

1 2π π π 1 3x
=
−1 ∫ u 2010 (π − arccos u )du =
2011
− A , deci A =
2011
şi I =
2011⋅ 2010 2011
; 44. a)
0 32 x − 3 x +1 + 3
dx = ∫
π 1 x 1 1− t
=
3 ln 3
; b) Facem schimbarea de variabilă 1 – x = t ⇒ I =
0 x 1
3 +3 −3− x
dx =
0 3 + 31− t − 3
t
dt = ∫ ∫
1 1 1 3t π 1
∫ ∫ ∫e ∫
x
= dt − I ⇒ 2I = dt ⇒ I = ; 45. ⋅ dx = −e x ctgx + e x ctgxdx
0 3t + 31− t − 3 0 32 t − 3 ⋅ 3 t + 3 2 3 ⋅ ln 3 sin 2 x
π π π
şi ∫ ∫
e x ln(sin x )dx = e x ln(sin x ) − e x ctgxdx . Rezultă că I = e x (ln(sin x ) − ctgx ) π4 = e 6 ( 3 + ln 2) − e 4 (1 +
6
π 2
π π π 1
+ ln 2 ) ; 46. a) ∫ arcsin(sin x)dx = ∫
0 0
2 xdx + ∫π (π −
2
x )dx =
4
; b) Facem schimbarea de variabilă t = şi
x
2 +1 ln x 2 +1
ln t 2 +1 ln x
avem ∫ 2 −1 1 + x 2
dx = − ∫ 2 −1 1 + t 2
dt , deci ∫ 2 −1 1 + x2
dx = 0 ; 47. Cu substituŃia 16 – 4x = 4t – 16, deci x =

3 4
11− 2t 5 4
11 − 2x ⋅ e16− 4x 5 4
2x − 5 
= 8 – t, I = ∫ 5 4 11− 2t + 4 2t − 5 ⋅ e 4t −16
(−dt) = ∫
3 4 2x − 5 + 4 11 − 2x ⋅ e16− 4x
dx = ∫ 1−
3

4
dx =
2x − 5 + 4 11− 2x ⋅ e16−4x 
b b 2π (2π − x) 2 ⋅ sin(2π − x )
= 2 – I. Rezultă I = 1; 48. Vom folosi identitatea: ∫ a
f (x)dx = ∫ a
f (a + b − x)dx ş I = ∫
0 1 + cos2 (2π − x)
dx =

2π (4π 2 − 4πx + x 2 ) ⋅ (− sin x ) 2π − sin x 2π


x sin x 2 π x 2 ⋅ sin x
= ∫
0 2
1 + cos x
dx = 4π 2 ∫0 1 + cos 2 x
dx + 4π ∫
0 1 + cos 2 x
dx −
0 1 + cos 2 x ∫ dx = 4π 2 ⋅

2π 2 π x sin x
⋅ arctg(cos x ) 0 + 4πJ − I = 4πJ − I ⇒ I = 2πJ, unde J = ∫ 0 1 + cos 2 x
dx . Scriem J = K + L, unde K =

π x sin x 2π x sin x
= ∫
0 1 + cos 2 x
dx , L = ∫0 1 + cos 2 x
dx . În integrala L facem schimbarea de variabilă x = t + π şi obŃinem: L =

π ( t + π) sin( t + π) π ( t + π) ⋅ (− sin t ) π π2 π2
= ∫
0 1 + cos 2 ( t + π)
dt = ∫
0 1 + cos 2 t
dt = − K + arctg (cos t ) 0
= − K −
2
. Găsim J = K + J =
2
, de

 x 
1  tg 
unde I = –π3; 49. a) Primitiva pe [0, π) este I = arctg 2  + C; b) Primitiva pe [0, 2π] este F(x) =
2  2 
 
 

340
  x 
 1  tg 
 arctg 2  + C, x ∈ [0, π)
 2  2 
  
 

 π 2π π
=
2 2
+ C, x=π ; c) ∫ f (x)dx = F(2π) − F(0) =
0 2
; 50. Fie F o primitivă a funcŃiei

  x 
 1  tg  π
 arctg 2 + + C, x ∈ (π, 2π]
 2  2  2
  
 
π 1 ln[1 + tg 2 (arctgx)] ln(1 + x 2 )
din integrală. Atunci f(x) = F(arctgx) − F  şi f'(x) = F '(arctgx) ⋅ = = .
4 1+ x 2
1+ x2 1 + x2
x ln(1 + x 2 ) 1 1 x ln(1 + x2 )
∫ ∫
1
Avem (xf(x))' = f(x) + xf'(x) ⇒ f(x) = (xf(x))' – ; f ( x )dx = xf ( x ) 0 − dx = f(1) –
1+ x2 0 0 1+ x 2

π
1 1 1 x2
− ln 2 2 . Dar f(1) =
4 ∫ π ln(1 +
4
4 tg 2 t )dt = 0 , deci ∫ 0
f ( x )dx = − ln 2 2 ; 51. a)
4 ∫ xg ' ( x )dx =
2
+ x sin x + cos x +

+ C; b) Cum g'(x) = 1 + cos x ≥ 0, (∀) x ∈ R ⇒ g este strict crescătoare pe R ⇒ g este injectrivă. Deoarece g
este continuă pe R şi lim g ( x ) = ±∞ ⇒ Img = R, de unde rezultă că g este surjectivă. Deducem că g este
x → ±∞

bijectivă, deci inversabilă; c) Deducem că f ( x ) = g −1 ( x 3 ) , (∀) x ∈ R ⇒ f este continuă, deci are primitive;
1
d) Facem schimbarea de variabilă g–1(x3) = t ⇔ x3 = g(t). Avem x 2 dx = g' ( t )dt . Dacă x = 0 ⇒ t = g–1(0) = 0.
3
π
π π 1  t2  π2 − 4
3
1
∫ ∫
−1
Dacă x = π ⇒ t = g (π) = π . ObŃinem
3 2
x f ( x )dx = tg ' ( t )dt =  + t sin t + cos t )  = ;
0 3 0 3 2  6
0
2
1 t (2t + 1) 1 1 23 3 22
52. a) ∫0 f (t)
dt = ln 3 ; b)
2 ∫ f (x)dx = 15 . Făcând schimbarea g(x) = y ⇒ ∫ g(x)dx = 15 ; c) Utilizând
0 1

3 α 3
b), inegalitatea devine: 3 – ∫ g(x)dx + ∫ g(x)dx ≥ α ⇔ ∫ g(x)dx ≤ 3 − α . Cum x ≤ 3 şi g este crescătoare ⇒
1 1 α
3 3
⇒ g(x) ≤ g(3) = 1 ⇒ ∫ g(x)dx ≤ ∫ dx = 3 − α ; 53. Considerăm funcŃia F: [–2, 2] → R, F(x) = (x – 16) ⋅
4
α α
x x
⋅ ∫
−2
f ( t )dt . Deoarece f este continuă ⇒ F este derivabilă şi F'(x) = 4 x 3 ∫
−2
f ( t )dt + ( x 4 − 16)f ( x ) , (∀) x ∈

∈ [–2, 2]. Cum F(–2) = F(2) = 0, aplicând teorema lui Rolle, rezultă că există c ∈ (–2, 2) astfel încât F'(c) = 0,
c c  4 c
−2 ∫
adică 4c 3 f ( t )dt = (16 − c 4 )f (c) , de unde
−2 c 4 ∫
f ( t )dt =  3 − f (c) ; 54. Din teorema de medie, există c1 ∈

∈ (a, b) astfel încât f(c1) = f(a). Aplicăm teorema lui Rolle funcŃiei F: [a, b] → R, F(x) = (f(x) – f(a))ex pe
intervalul [a, c1]. Există c ∈ (a, c1) ⊂ (a, b) astfel încât F'(c) = 0. Din F'(c) = 0 rezultă concluzia; 55. a) Vom arăta

341
0, x ∈ [0, b]
x − b

că pentru orice a ∈ (0, 1) există f ∈ F astfel încât T(f) = a. Fie 0 < b < c < 1 şi funcŃia f(x) =  , x ∈ ( b , c) ,
c −b
1, x ∈ [c,1]
2−b−c
care aparŃine mulŃimii F. ObŃinem T(f) = . Punând condiŃia T(f) = a, găsim că b + c = 2 – 2a. Luăm b =
2
= 1 – a – ε, c = 1 – a + ε cu 0 < ε < min(a, 1 – a) şi obŃinem un exemplu de funcŃie pentru care T(f) = a, ceea ce
a
 b x, x ∈[0, b]

demonstrează surjectivitatea. Un alt exemplu de funcŃie pentru care T(f) = a este f(x) = a, x ∈ (b, 2b) ,
 1− a a −c
 x+ , x ∈[2b,1]
 1 − 2b 1 −c
1− a  1 
unde b = ∈  0,  ; b) Deoarece pentru orice ε ∈ (0, m), unde m = min(a, 1 – a) se obŃine T(f) = a, rezultă
2−a  2
că T nu este injectivă. Se pot da însă două funcŃii particulare g, h ∈ F astfel încât T(g) = T(h). Spre exemplu,
  1
2 x , x ∈ 0, 
  4
 1 1 3 1
pentru g(x) = x, (∀) x ∈ [0, 1] şi h(x) =  , x ∈  ,  , avem T(g) = T (h ) = ; 56. a) I5 =
 2  4 4  2
 3 
2x − 1, x ∈  , 1
 4 
5 x ( x − [ x ]) 2 3 x (x− 2) 4 x ( x − 3) 5 x (x − 4) 97 x{x}
= ∫
1 [x]
dx = ∫ x(x − 1)dx + ∫
1 2 2
dx + ∫
3 3
dx + ∫
4 4
dx =
36
; b) Deoarece
[x ]
=

k +1
x 2 − x[ x ]
n k +1 x 2 − kx
n
 x3 x 2  n −1 1  1 1 
=
[x ]
, avem: In = ∑∫
k =1
k k
dx = ∑ 
k =1 

 3k − 2 
k
=
2
+ 1 + + ... +
3 2
 , de unde rezultă
n −1
1 1 1
1+ + ... +
I n −1 1 n − 1 = 1 + 1 ⋅ lim 1 2n [x ]
lim n = lim
n→∞ n n → ∞ 2n
+ lim
3 n→∞
2
n
n = ; 57.
2 3 n→∞ n + 1 − n 2 ∫
0 [ x ]2 + 5[ x ] + 6
dx =

2 n −1 k +1 2 n −1 2 n −1 2 n −1 2 n −1 2n + 2
 3 2   1 1 
∑∫ ∑ ∑ ∑ ∑ ∑k −
k k 1 1
= dx = =  − = +2  − =
k =0
k 2
k + 5k + 6 k =0
(k + 2)(k + 3) k =0 
k+3 k +2 k =0
k+3 k =0 
k +3 k +2 k =1

 1 1  2n + 2 2n + 2 
∑ k − ln(2n + 2) + n + 1 − 2 + ln
1 1 1 5 5 2
= −1 − + 2 −  , deci L = lim  = C − + ln , C fiind
2  2n + 2 2  n → ∞ an   2 a
 k =1

 n 1  n 10 2 3

[x] 1 2
constanta lui Euler, C = lim 
n → ∞
 k =1 k
− ln n  ; 58.


∫ 0 2
[ x ] + 3[ x ] + 2
dx = ∫2
0
dx + ∫
1 1+ 2 + 3
dx + ∫ 4 + 6 + 2 dx +
2

n n −1 1 1 1 2 1  1 1   2
+ ... + ∫n −1 n ( n + 1)
dx = + + ... + +
3 4
− . Astfel, L =
n n +1 2
lim 1 + + ... + − ln n  + lim 
n → ∞ 2 n  n → ∞ n + 1
−2+

342
1

+ ln n − ln(n + 1)) = C – 2, unde C este constanta lui Euler; 59. a) Notăm t =


x
⇒ an =
∫0
n f ( t )dt
. Deoarece
n 1
n
x x

lim
∫ 0
f ( t )dt
= f (0) ⇒ lim a n = f (0) ; b) Deoarece lim
∫ f (t )dt − xf (0) = 1 f ' (0) , obŃinem
0
lim n[a n − f (0)] =
x →0 x n →∞ x →0 x2 2 n →∞
1
1
∫ n f ( t )dt − f (0)
1 1 1 x
n
∫ (x − x + x 3 − ... + (−1) n −1 x ) ln(x + 1)dx ⇒ bn = ∫ x + 1 ⋅ ln(x +
2
= lim 0
= f ' (0) ; 60. b n =
n →∞ 1 2 0 0

n2
1 ( − x ) n +1 1 x 1 1 1
+ 1)dx + ∫ 0 x +1
⋅ ln(x + 1)dx ; I1 = ∫ x + 1 ⋅ ln(x + 1)dx = ∫ ln(x + 1)dx − ∫ x + 1 ⋅ ln(x + 1)dx = 2 ln 2 − 1 −
0 0 0

ln 22 1 ( − x ) n +1 1 ln 2 (− x)n +1
1

2
; ∫ 0 x +1 ∫
⋅ ln(x + 1)dx ≤ ln 2 x n +1dx =
0 n+2
, de unde rezultă că lim ∫
n →∞
0
x +1
⋅ ln(x + 1)dx = 0 .

ln 2 2a + x
Deci lim b n = 2 ln 2 − 1 − ; 61. a) FuncŃia f: [–a, a] → R, f ( x ) = sin 2 nx ⋅ ln este impară, deci In =
n →∞ 2 2a − x
a 1 a 2a + x 1 a 2a + x 1 a
= ∫−a
f ( x )dx = 0 ⇒ lim I n = 0 ; b) K n = ⋅ (1 − cos 2 nx ) ⋅ ln
n →∞ 2 0 2a − x ∫
dx = ln
2 0 2a − x
dx −
2 0
cos 2 nx ⋅ ∫ ∫
2a − x b
⋅ ln
2a + x n → ∞ a ∫
dx . Cum lim cos nxg( x )dx = 0 , unde g este derivabilă cu derivata continuă, rezultă că lim K n =
n →∞

1 a 2a + x 1 2
= ∫ln
2 0 2a − x
dx = a (3 ln 3 − 4 ln 2) ; 62. b) De la punctul a) avem e ≤ f(x) ≤
2 ln 2
, (∀) x ∈ [2, 3] şi

2 2
după integrare obŃinem e ≤ I ≤ . Deoarece e > 2, iar < 3 , rezultă că [I] = 2; 63. Notăm I =
ln 2 ln 2
π π
sin 2009 x + cos 2 x cos 2009 x + sin 2 x π π
= ∫
0
2
sin 2009 x + cos 2009 x + 1
dx şi J = ∫0
2
sin 2009
x + cos 2009
x +1
dx . Atunci I + J =
2
; x = − t conduce la I =
2
1 1
π  1 1 sin x sin x
= J, deci I =
4
. Dacă x ∈  ,  , avem: n2sin x ≤
n +1 n  x2
≤ (n − 1) 2 sin x ; n 2 ∫ n sin xdx
1
n +1
≤ ∫ n
1
n +1
x2
dx ≤

1 1
1 2n + 1 sin x 1 2n − 1
≤ (n + 1) 2 ∫ n sin xdx
1
n +1
; 2n 3 sin
2n (n + 1)
⋅ sin
2n (n + 1)
≤n ∫ n
1
n +1
x 2
dx ≤ 2(n + 1) 2 ⋅ n ⋅ sin
2 n ( n + 1)
⋅ sin
2 n (n + 1)
1
sin x π
ObŃinem lim n
n →∞ ∫ n
1
n +1
x 2
dx = 1 . Avem
4
< 1 ; 64. Folosim inegalitatea Cebîşev pentru funcŃii de monotonii

b b b 1 ln(1 + 2x ) 1 1 1
diferite: ∫ a
f ( x )dx ⋅ ∫ a ∫
g( x )dx ≥ (b − a ) (fg)( x )dx .
a
Astfel, ∫0 1 + 3x 2
dx ≤ ∫
0
ln(1 + 2x )dx ⋅ ∫ 1 + 3x
0 2
dx şi

1 3 1 1 π
deoarece ∫ ln(1 + 2x)dx = 2 ln 3 − 1 , iar ∫ 1 + 3x
0 0 2
dx =
3 3
, rezultă inegalitatea cerută; 65. Arătăm întâi că

343
2 2
 b   b 
pentru o funcŃie integrabilă f: [a, b] → R există inegalitatea: 
 ∫
a   a ∫
f ( x ) ⋅ sin xdx  +  f ( x ) ⋅ cos xdx  ≤ (b − a ) ⋅

b
⋅ ∫a
f 2 ( x )dx , (*). Aplicăm forma integrală a inegalităŃii Cauchy-Buniakovski, pentru funcŃiile integrabile

(∫ ) (∫ ) (∫ )
2
2 2
 b 

b b b
f, g: [a, b] → R, f (x)g(x)dx ≤ f 2 (x)dx ⋅ g 2 (x)dx ; g(x) = sin x ⇒  f ( x ) sin xdx  ≤
a a a
 a 
2
b b  b  b b
∫ ∫ ∫
≤ f ( x )dx ⋅ sin xdx ; g(x) = cos x ⇒  f ( x ) cos xdx  ≤ f 2 ( x )dx ⋅ cos 2 xdx . Prin adunarea membru ∫ ∫
2 2
a a  a  a a

cu membru a ultimelor inegalităŃi, obŃinem (*). Pentru a = 1, b = 2 şi f: [1, 2] → R, f(x) =


2
x+2  2 
=
( 4 x + 2 x + 2) r + 2 r 2 x
2
, integrabilă, unde r =
 1  ∫
x 2 + x + 1 . Înlocuind în (*) ⇒  f ( x ) sin xdx  +

2
 2  2 1 2

 1 ∫
+  g ( x ) cos xdx  ≤
 ∫
1
f 2 ( x )dx . Arătăm că
6 ∫ f 2 ( x )dx <
şi atunci rezultă inegalitatea din enunŃ.
1
2 2 x+2 2 2 x 2 + 2x + 2 − 2x 2 − x 2 2r 2 − 2 x 2 − x
∫ 1
f 2 ( x )dx = 2∫ 2
1 ( 4 x + 2 x + 2) r + 2 r x
dx = 2 2
1 ( 2 x + x + 1) ⋅ 2r + 2r x ∫
dx = 2
1 ( 2 x + x + 1 + 2rx ) ⋅ 2r
dx = ∫
2rx + 2r 2 − 2rx − 2x 2 − x 2 x + 2r + 1
2 2 x +r−x⋅ '
2 2r − 2 x − x 2 2 2 x 
= ∫ 1 ( x 2 + r 2 + 2rx ) ⋅ 2r
dx = ∫1
2r
(x + r) 2
dx = ∫ 1 ( x + r) 2
2r dx = ∫
1

 +r
x
 dx =
' 2
2 x  x 2 1 1 1 1
= ∫ 
1  2
 x + x + x +1 
 dx =

=
x + x + x +1 1 2 + 7 1+ 3 2 3 6
2
− < − = ; 66. FuncŃia g: (0, ∞) → R,

g(x) = log2 x este concavă. Dacă ∆ = (x0, x1, ..., xn) este o diviziune a intervalului [0, 1] şi ξi ∈ [xi, xi+1], atunci
n −1  n −1  1  1  1
∑ g(f (ξi ))(x i +1 − x i ) ≤ g
 ∑
f (ξi )(x i +1 − x i )  , de unde
 0
g(f ( x ))dx ≤ g f ( x )dx  , deci
 0  ∫
0
log 2 f ( x )dx ≤ ∫ ∫
i=0  i =0 
 1  1 1

≤ log 2  f ( x )dx  . Rezultă 2 ∫0 2 ∫ f (x)dx .
log f ( x ) dx

 0  0

344
Concursul de matematică „LaurenŃiu Panaitopol”, Bucureşti, 7 noiembrie 2009

CLASELE A IX-
IX-A – A X-
X-A

1. a) Putem, de exemplu, să luăm triunghiurile PQR şi PQS cu m('PQR) = m('PQS) = 90° şi m('PRQ) = x°,
PQ PQ
m('PSQ) = y°, iar R şi S de părŃi opuse ale lui PQ. Din x < y reiese PS < PR, de unde sin x° = < =
PR PS
ab sin C
= sin y°; b) Din ipoteză şi S = reiese că sin A ≤ sin A', sin B ≤ sin B', sin C ≤ sin C', deci, conform a),
2
A ≤ A', B ≤ B', C ≤ C'. Deoarece A + B + C = A' + B' + C', deducem A = A', B = B', C = C', adică triunghiurile
sunt asemenea. Pentru raportul k de asemănare avem k ≤ 1 şi k2 ≥ 1, deci k = 1; 2. Fie p < q numerele prime.
Observăm că p + q = 2s, s ≥ 4. Dacă s are cel puŃin doi factori primi, atunci concluzia se verifică. În caz contrar,
p+q r−x
s= este un număr prim situat între p şi q – contradicŃie cu faptul că p, q sunt consecutive; 3. =
2 x
 r
1− , x ≥ r
 x r r r r 2ab
= . Rezultă că M(r) = max  − 1,1 −  . Deoarece − 1 ≥ 1 − ⇔ r ≥ , deducem că M(r) =
 r − 1, x ≤ r a b a b a+b
 x
r 2b b−a 2ab r 2a b−a 2ab
= −1 ≥ −1 = dacă r ≥ şi M(r) = 1 − ≥ 1 − = dacă r ≤ . Astfel, minM(r)
a a+b a+b a+b b a+b a+b a+b
b−a 2ab
este şi se atinge pentru r = ; 4. Fie b1 ≤ b2 ≤ ... ≤ b100 şi f1 ≤ f2 ≤ ... ≤ f100 vârstele băieŃilor B1, B2, ...,
a+b a+b
B100, respectiv ale fetelor F1, F2, ..., F100 (măsurate în zile). Avem de arătat că |bi – fi| ≤ 7, pentru orice i. Fie i
arbitrar. Analizăm doar cazul bi ≥ fi, celălalt caz fiind analog. În cazul în care, în situaŃia iniŃială, băiatul Bi a dansat
cu fata Fj, j ≤ i, atunci bi – fi ≤ bi – fj ≤ 7. În caz contrar, Bi a dansat iniŃial cu Fk, k > i, iar bi – fi ≤ bk – fk ≤ 7.

CLASELE A XI-
XI-A – A XII-
XII-A

1. Dacă un termen al şirului este 2, atunci, începând cu acel termen, şirul este 2, 3, 2, 3, 2, 3, ..., deci este
a +1
periodic. În caz contrar, pentru n ≥ 1, a n +1 ≤ n < a n (deoarece an este impar şi mai mare ca 1), adică am avea
2
un şir strict descrescător de numere naturale, fals; 2. y = 2 + mx este ecuaŃia unei drepte care trece prin punctul
2 sin 2 x
1 − cos x 2 x 1 − cos x x 1 − cos x
(0, 2). = = tg 2 pentru x ≠ π + 2nπ, n ∈ Z, deci = tg şi arctg =
1 + cos x 2 cos x 2
2 1 + cos x 2 1 + cos x
2
x 1 − cos x x
= + kπ , k ∈ Z. Pentru x ∈ [2nπ, π + 2nπ), n ∈ N*, avem arctg = − nπ , iar pentru x ∈ (π + 2nπ,
2 1 + cos x 2

345
1 − cos x x
2π + 2nπ], n ∈ N*, avem arctg = − − (n + 1)π . Graficul funcŃiei definite de expresia
1 + cos x 2
1 − cos x
arctg , considerată pe domeniul maximal, este cel din figură (funcŃia este pară, deci graficul este
1 + cos x
simetric faŃă de Ox). Punctele marcate nu fac parte din grafic.

(0,2)
*
* * * *

–π π 3π

Dreapta y = 2 + mx taie graficul precedent în toate situaŃiile, cu excepŃia cazului când este orizontală. Dacă trece
prin unul dintre punctele marcate, atunci ea taie porŃiunea de grafic care „începe” (pentru m < 0) sau „se
termină” (pentru m > 0) cu punctul marcat;
3. Avem relaŃia z 3i = (z1 + z 2 + z 3 )z i2 − (z1z 2 + z1z 3 + z 2 z 3 ) + z1z 2 z 3 . Dacă m > 1, reiese z1m + 2 + z m
2
+2
+ z 3m + 2 =
= (z1 + z 2 + z 3 )(z1m +1 + z m
2
+1
+ z 3m +1 ) − (z1z 2 + z1z 3 + z 2 z 3 )(z1m + z m m m −1
2 + z 3 ) + z1z 2 z 3 ( z1 + zm
2
−1
+ z 3m −1 ) .
În sfârşit, din z13 + z 32 + z 33 = (z1 + z 2 + z 3 )(z12 + z 22 + z 32 − z1z 2 − z1z 3 − z 2 z 3 ) + 3z1z 2 z 3 , rezultă z1z2z3 = 0, ceea
ce arată că presupunerea făcută mai sus este falsă. Astfel, cel puŃin unul dintre numere este nul. Reluând
raŃionamentul pentru cele două numere rămase şi folosind z i2 = (z1 + z 2 )z i − z1z 2 , deducem, ca mai sus, că unul
dintre ele este nul, de unde concluzia; 4. Dacă dreapta are ecuaŃia mx = ny, m, n ∈ Z, m2 + n2 ≠ 0, atunci ea taie
cercurile care au centrele în punctele (nk, mk), k ∈ Z. În caz contrar, ea are o ecuaŃie de forma y = mx, m ∈ R –
| mp − q |
– Q, iar distanŃa de la ea la punctul (p, q) este <| mp − q | . Considerăm numerele m – [m], 2m – [2m],
m2 + 1
..., 100m – [100m]. Aceste 100 de numere sunt distincte (altfel am avea m ∈ Q) şi sunt situate în intervalul
(0, 1), deci există două – fie acestea am – [am], bm – [bm], a > b – la distanŃă mai mică decât 0,01. Luând p1 =
= a – b, q1 = [am] – [bm], găsim un punct (p1, q1), cu p1 > 0, situat la distanŃă < 0,01 de dreaptă. Luând apoi
numerele 2p1m – [2p1m], 4p1m – [4p1m], ... , 200p1m – [200p1m] şi raŃionând ca mai sus, găsim un punct (p2, q2)
cu p2 > p1, situat la distanŃă < 0,01 de dreaptă. Cele de mai sus se pot repeta şi construim astfel inductiv un şir de
puncte (pn, qn), cu 0 < p1 < p2 < ... , situate la distanŃă < 0,01 de dreaptă.

Concursul interjudeŃean de matematică „Cristian S. Calude”, EdiŃia a X-a,


GalaŃi, 31 octombrie 2009
CLASA A IX-
IX-A
25
k (3k + 1)

x x x
1. b) Membrul stâng este S = , dar = ≥ . Deducem S ≥
k =1
2000 − k (3k + 1) 2000 − x x (2000 − x ) 1000
25
25 ⋅ 26 2

1
≥ ⋅ k (3k + 1) = > 16,8 ; 2. a) Deoarece x ∈ R – Z, rezultă {x} ∈ (0, 1). Cum [x] = x – {x} şi
1000 k =1 1000

346
{x + k} = {x}, k ∈ Z, ecuaŃia este echivalentă cu: [x] ⋅ (1 – k2{x}) = 0. Pentru k = 0 sau k = 1 rezultă soluŃia x ∈
 1 
∈ (0, 1) iar pentru k ≥ 2, x ∈ (0, 1) ∪ m + , m ∈ Z  ; b) Notăm a + b = x, b + c = y, c + a = z, x, y, z ∈ (0, ∞).
 k 
ObŃinem x + y + z = 1 iar inegalitatea echivalentă cu 1 ≥ 3(xy + yz + zx) sau încă (x + y + z)2 ≥ 3(xy + yz + zx),
adevărată deoarece x2 + y2 + z2 ≥ xy + yz + zx, pentru orice x, y, z; 3. a) În orice triunghi ha ≤ la ≤ ma. Rezultă
a+b a+b
∑ la + h b
≥ ∑
ma + mb
. Dacă a ≤ b ≤ c, atunci ma ≥ mb ≥ mc, deci a + b ≤ a + c ≤ b + c şi ma + mb ≥ ma +

a+b
∑m ∑ (a + b) ⋅ ∑ m ∑ (a + b) =
1 1
+ mc ≥ mb + mc. Conform inegalităŃii şi Cebîşev, ≥ . Cum
a + m b 3 a + mb
(1 + 1 + 1) 2 9(a 2 + b2 + c 2 )

1
= 2(a + b + c) şi ≥ , iar (ma + m b + mc ) 2 ≤ 3(ma2 + m b2 + mc2 ) = ,
m a + m b 2(m a + m b + m c ) 4
obŃinem inegalitatea cerută; b) Din congruenŃa triunghiurilor ADE şi AHB rezultă HB ⊥ DE. La fel BG ⊥ DF şi
(BG) ≡ (DF). Dacă notăm cu O mijlocul lui (BD), rezultă (OM) ≡ (ON), OM ⊥ ON şi (OP) ≡ (OQ), OP ⊥ OQ.
Din congruenŃa ∆NOQ ≡ ∆MOP rezultă MP ⊥ NQ, deci MNPQ este ortodiagonal şi prin urmare MN2 + PQ2 =
= MQ2 + NP2.

CLASA A X-
X-A

sin 2B + sin 2C sin 2B sin 2C


1. a) Notăm AO ∩ BC = {M}, atunci OA = − ⋅ OM şi OM = ⋅ OB + ⋅
sin 2A sin 2B + sin 2C sin 2B + sin 2C
2
⋅ OC , din care rezultă relaŃia cerută; b) Avem: MA2 = MA = (MO + OA) 2 = MO 2 + R 2 + 2MO ⋅ OA . Rezultă
că ∑ MA 2
sin 2A = (MO 2 + R 2 ) ∑ sin 2A + 2MO ⋅ (∑ OA sin 2A ) şi de aici k = 4(MO 2
+ R2)sin A sin B sin C,
k − 2S 2
deci MO 2 = ⋅ R . Prin urmare dacă k < 2S, M ∈ ∅; dacă k = 2S, atunci M = 0, iar dacă k > 2S, M
2S
k − 2S k +1  1
aparŃine cercului de centru O şi rază R ⋅ ; 2. a) Evident A > n. Cum 2n + 2 = 2 n + 2 1⋅ 1⋅ ...
 ⋅ 1 1 +  ≤
( 2 k +1) de 1
2S k k
n
1  1 1 1 
∑ k(k + 1) , deci A < n +
1 1
≤  2k + 2 +  (inegalitatea mediilor), rezultă A < n + 1 − <
2k + 2  k 2 k =1
2  n +1
1 1
< n + 1. Astfel [A] = n; b) Observăm că log2k+1 (2k) ⋅ log2k +1 (2k + 2) ≤ (log2k +1 (2k) + log2k +1 (2k + 2))2 = ⋅
2 2
1
⋅ log 2 2k +1 (4k 2 + 4k) < log 2 2k +1 (2k + 1) 2 = 1 . Prin urmare log 2 k +1 (2k ) < log 2 k + 2 (2k + 1) . Rezultă că p =
2
n n n
lg(2k + 1)
∏ ∏ ∏ lg(2k + 2) = lg(2n + 2) ⋅
lg 2 1 lg 2
= log 2 k +1 (2k ) < log 2 k + 2 (2k + 1) = . Prin urmare p < . Sunt
k =1 k =1 k =1
p lg(2n + 2)

lg 2 1 2
soluŃii ale inecuaŃiei toate numerele naturale n pentru care < , deci n > 2 2010 −1 şi concluzia
lg(2n + 2) 2010
problemei se impune; 3. Fie M = maxA, m = minA şi atunci M – m = max | x − y | . Presupunem că f ar fi
x , y∈A
surjectivă, atunci există x, y ∈ A astfel încât M = f(x), m = f(y). Deci M – m = |f(x) – f(y)| < |x – y| ≤ |M – m| =

347
= M – m, fals. Deoarece f nu este surjectivă, rezultă că f(A) ⊂ A; f2(A) = f(f(A)) ⊂ f(A), dacă f(A) are cel puŃin
două elemente. ObŃinem şirul de incluziuni: A⊂ f( A) ⊂ f(f(A)) ⊂ ... care de la un anumit rang nu mai poate fi
constituit din incluziuni stricte (deoarece A este finită). Fie n ∈ N* cu proprietatea că fn(A) = fn+1(A). Evident
fn(A) are un singur element. Deci fn(A) = x0. Rezultă f(x0) = x0.

CLASA A XI-
XI-A

1. a) FuncŃia sin este periodică, cu perioada 2π şi funcŃia tg cu perioada π; sin x > 0; tg x > 0 ⇒ căutarea soluŃiei
 π sin 2 x sin 2 x
x în intervalul  0,  . Notăm cu t = log2 (sin x). Atunci sin x = 2t şi tg2 x = 3t. Dar tg 2 x = = ⇒
 2 cos 2 x 1 − sin 2 x
t
4t 4t 3
⇒ tg 2 x = , de unde se obŃine ecuaŃia 3 t
= ⇔ 3t − 12 t = 4 t ⇔   = 1 + 3t , (1). FuncŃia f1: R → R,
1 − 4t 1 − 4t 4
t
3
f1 ( t ) =   este strict descrescătoare, iar f2: R → R, f2(t) = 1 + 3t este strict crescătoare. Rezultă că ecuaŃia (1)
4
1 π
are cel mult o soluŃie. Se verifică t = –1 este soluŃie, de unde sin x = , deci x = + 2kπ ne dă mulŃimea
2 6
a + b = c + d
2 2
soluŃiilor căutate; b) Cu notaŃiile a = 2x, b = 16t, c = 4z, d = 4y, obŃinem:  . Apoi:
c 2 + d 2 = a + b
 2 2 (a + b ) 2
a + b = c + d ≥
 2
 . Prin înmulŃire, obŃinem: (a + b)(c + d) ≤ 4 şi apoi 2 ab ⋅ 2 cd ≤ (a + b)(c + d) ≤ 4,
c 2 + d 2 = a + b ≥ ( c + d) 2
 2
deci abcd ≤ 1. În fine: abcd = 2x ⋅ 16t ⋅ 4z ⋅ 4y = 2x+2y+2z+4t ≤ 20, de unde x + 2y + 2z + 4t ≤ 0, ceea ce trebuia
demonstrat; 2. a) xn+1 = 2(xn – 3)2 + 3 ⇔ 2(xn+1 – 3) = 4(xn – 3)2. Vom nota: yn = 2(xn – 3); y0 = 2; yn+1 = y 2n .
n n n
Prin inducŃie se demonstrează că y n = 2 2 , de unde 2(xn – 3) = 2 2 , deci x n = 2 2 −1 + 3 . În final, L =
1 n 1
= lim n (2 2 −1 + 3) = ; b) Dacă m = 1, este evidentă concluzia. Pentru m = 2, A1 = Q, A2 = R – Q.
n →∞ 22 2
MulŃimile A1 şi A2 sunt dense în R. Atunci, rezultatul este evident. Fie m ≥ 3, m – 1 = k ≥ 2 şi definim A1 = Q,
A2 = { a k 2 | a ∈ Q*}, A3 = { a k 2 2 | a ∈ Q*}, ..., Ak = { a k 2 k −1 | a ∈ Q*}, Am = R – A1 ∪ A2 ∪ ... ∪ Ak. Pentru
α ∈ R, avem: α ∈ A1 ∪ A2 ∪ ... ∪ Ak sau α ∈ Am. Dacă Ai ∩ Aj ≠ ∅, unde i, j ∈ {2, 3, ..., m – 1}, i < j, atunci
a
(∃) a, b ∈ Q* astfel încât a ⋅ k 2i −1 = b ⋅ k 2 j−1 ⇒ = k 2i − j , care este o contradicŃie. În concluzie, {A1, A2, ...,
b
Am} determină o partiŃie pentru R. Se ştie propoziŃia:
„Dacă A ⊂ R este densă în R şi r ∈ R*, atunci mulŃimea B = {r ⋅ a | a ∈ A} este densă în R”.
Atunci obŃinem o partiŃie a mulŃimii R cu ajutorul mulŃimilor dense A1, A2, ..., Am, de unde (∀) α ∈ R, (∀) i ∈
∈ {1, 2, 3, ..., m}, (∃) (xn)n≥1 ⊂ Ai şir neconstant astfel încât lim x n = α . Cu aceasta, construcŃia este terminată;
n →∞
3. a) Se observă că h(x, y) = f(y) – f(x) este strict crescătoare faŃă de y şi strict descrescătoare faŃă de x, de unde
h ( x , x + y)
(∀) y > x ⇒ h(x, y) > 0, g(x, y) se poate scrie: g( x, y) = . Dacă x – y ≥ 0 sau x + y ≤ 0, atunci
h ( x − y, x )

348
conform ipotezei, avem: h(x, x + y) < 2 ⋅ h(x – y, x), (1') şi în afară de aceasta, pentru toŃi y > 0 avem: h(0, y) <
< 2 ⋅ h(–y, 0). Pentru toŃi x ≥ 0, y > 0 avem: h(x + y, x + 2y) ≤ 2 ⋅ h(x, x + y); h(x + 2y, x + 3y) < 2 ⋅ h(x + y, x +
+ 2y) < 4 ⋅ h(x, x + y), de unde h(x + y, x + 3y) = h(x + y, x + 2y) + h(x + 2y, x + 3y) < 6 ⋅ h(x, x + y), (1);
b) Trebuie să mai analizăm cazurile: 1) x – y < 0 < x şi 2) x < 0 < x + y.
y  y y 3y   y 
Cazul 1) Dacă x – y < 0 ≤ x − , după (1) avem: h(x, x + y) = h  x − , x − +  < 6 ⋅ h x − , x  . Cum h
2  2 2 2   2 
 y  y
descreşte faŃă de primul argument, avem: 6 ⋅ h  x − , x  < 6 ⋅ h ( x − y, x ) < 14 ⋅ h ( x − y, x ) . Dacă x − < 0 < x,
 2  2
atunci x < y – x < x + y < 3(y – x), după (1), h(y – x, x + y) < h(y – x, 3(y – x)) < 6 ⋅ h(0, y – x). Deoarece
h(x, y – x) < h(0, y – x) obŃinem: h(x, x + y) = h(x, y – x) + h(y – x, x + y) < 7 ⋅ h(0, y – x) < 14 ⋅ h(x – y, 0) <
< 14 ⋅ h(x – y, x).
Cazul 2) h(x, 0) < 2 ⋅ h(2x, x) < 2 ⋅ h(x – y, x) şi h(0, x + y) < 2 ⋅ h(–x – y, 0) < 2 ⋅ h(x – y, 0) = 2 ⋅ h(x – y, x) +
+ 2 ⋅ h(x, 0) < 6 ⋅ h(x – y, x), de unde: h(x, x + y) = h(x, 0) + h(0, x + y) < 14 ⋅ h(x – y, x). Rezultă că g(x, y) <
< 14.

CLASA A XII-
XII-A

1. a) Fie g(x) = f(x) – 5, (∀) x ∈ R, atunci g(x + 3) + g(x – 1) = g(x + 1) + x – 5, (∀) x ∈ R. Pentru x = 4 avem
g(7) + g(3) = g(5) – 1. Pentru x = 6, avem g(9) + g(5) = g(7) + 1, de unde g(9) + g(3) = 0. Dacă g(3) = 0, avem
f(3) = 5 deci 3 este soluŃie. Dacă g(3) ⋅ g(9) ≠ 0, avem g(3) < 0 şi g(9) > 0 sau g(3) > 0 şi g(9) < 0. FuncŃia g are
proprietata lui Darboux ⇒ (∃) x0 ∈ (3, 9), astfel încât g(x0) = 0 ⇒ f(x0) = 5; b) Prin înmulŃire la dreapta cu a2 a
relaŃiei a3 ⋅ b = b ⋅ a2, obŃinem: (a3 ⋅ b) ⋅ a2 = (b ⋅ a2) ⋅ a2 ⇔ a3 ⋅ (b ⋅ a2) = b ⋅ a4 ⇔ a3(a3 ⋅ b) = b ⋅ a4 ⇔ a6 ⋅ b =
= b ⋅ a4. ÎnmulŃind relaŃia obŃinută la dreapta cu a4, se obŃine: (a6 ⋅ b) ⋅ a4 = (b ⋅ a4) ⋅ a4 ⇔ a6 ⋅ (b ⋅ a4) = b ⋅ a8 ⇔
⇔ a6 ⋅ (a6 ⋅ b) = b ⋅ a8 ⇔ a12 ⋅ b = b ⋅ a8 ⇔ e ⋅ b = b ⋅ a8 ⇔ b = b ⋅ a8 ⇔ a8 = e. Avem: a12 ⋅ b = b ⋅ a8 ⇔ a8 ⋅ (a4 ⋅
⋅ b) = b ⋅ a8 ⇔ e ⋅ (a4 ⋅ b) = b ⋅ e ⇔ a4 ⋅ b = b ⇔ a4 = e; a6 ⋅ b = b ⋅ a4 ⇔ a4 ⋅ (a2 ⋅ b) = b ⋅ a4 ⇔ e ⋅ (a2 ⋅ b) = b ⋅ e
⇔ a2 ⋅ b = b ⇔ a2 = e; a3 ⋅ b = b ⋅ a2 ⇔ a2 ⋅ (a ⋅ b) = b ⋅ a2 ⇔ e ⋅ (a ⋅ b) = b ⋅ e ⇔ a ⋅ b = b ⇔ a = e; c) 1) Pentru
a = b obŃinem a & (a & c) = a & (c & a) ⇒ a & c = c & a; 2) Avem a & (b & c) = b & (c & a) = c & (a & b) = (a & b) & c
(din comutativitate). 2. a) 1) Deoarece (an) este strict crescător, rezultă că an ≥ 1, (∀) n ≥ 1. Atunci
1 1
x n ≤ 1 + + ... + < 3 şi cum xn+1 – xn > 0, rezultă că (xn) este şir convergent;
2! n!
1  1 1  1  1 1  1
xn+1 − xn = ⋅ 1+ +... + ≤ ⋅ 1 + + ... + k −1  < , (∀)
a1a2 ⋅...⋅ an  an+2 an+2 ⋅...⋅ an+k  a1a 2 ⋅ ... ⋅ a n +1  a n +1 a n +1  a1a 2 ...a n (a n +1 − 1)
p
k ≥ 2. Fie l = lim x n şi presupunem că l = , p, q ∈ N, (p, q) = 1. Trecând la limită în inegalităŃile: 0 < xn+k – xn <
n →∞ q
1 p 1
< , k → ∞, rezultă că: 0 < − x n ≤ , am Ńinut cont că xn < l, (∀) n ≥ 1.
a1a 2 ...a n (a n +1 − 1) q a1a 2 ...a n (a n +1 − 1)
p 1 p  q
Pentru n = q + 1, avem 0 < − x q +1 ≤ ≤ ⇒ 0 <  − x q +1  ⋅ qa1a 2 ...a q +1 ≤ <1.
q a1a 2 ...a q +1 (a q + 2 − 1)  q  a q+2 − 1

p  P( n )
Dar  − x q +1  ⋅ qa1a 2 ...a q +1 ∈ N, fals; 2. Presupunem că există două funcŃii polinomiale P şi Q cu x n = ,
 q  Q (n )
P(n + 1) P(n) 1 H(n)
(∀) n ≥ 1. Atunci x n +1 − x n = − ⇒ = , unde s-a notat H(x) = P(x + 1) ⋅
Q(n + 1) Q(n) a1a 2 ...a n +1 Q(n) ⋅ Q(n + 1)

349
H(n )
⋅Q(x) – P(x) ⋅ Q(x + 1), rezultă că H este o funcŃie polinomială. Urmează că 1 = ⋅ a1a 2 ...a n +1 , (∀)
Q(n ) ⋅ Q(n + 1)
H(n ) Q(n ) ⋅ Q(n + 1) f (n + 1)
n ≥ 1, deducem că: ⋅ ⋅ a n + 2 = 1 . Însă lim = 1 , oricare ar fi f funcŃie
Q(n + 1) ⋅ Q(n + 2) H(n ) n → ∞ f (n )

H (n ) Q(n + 1) Q(n + 2)
polinomială nenulă. Deci lim a n + 2 = lim ⋅ ⋅ = 1 , fals; b) Arătăm că M = {1}. Să
n →∞ n → ∞ H ( n + 1) Q(n ) Q(n + 1)
presupunem că există m ∈ M, m > 1. Atunci există p prim, p | m şi cum (an, m) = 1, (∀) n ∈ N*, rezultă că (an, p) =
= 1, (∀) n ∈ N*. Deci există p prim, p ∈ M. Cum a1 = 10 şi a2 = 48, este evident că p ≠ 2, 3, 5. Deci p ≥ 7. Din
Mica Teoremă a lui Fermat, avem: 2p–1 ≡ 3p–1 ≡ 6p–1 ≡ 1 (mod p). Atunci 6 ⋅ ap–2 = 3 ⋅ 2p–1 + 2 ⋅ 3p–1 + 6p–1 – 6 ≡
≡ 3 + 2 + 1 – 6 ≡ 0 (mod p). Deci p | 6ap–2 şi cum p ≠ 2, 3 ⇒ p | ap–2. Deci M = {1}; 3. Vom folosi următoarea
lemă: Dacă f, g: R → R f admite primitive, g este derivabilă, cu g' continuă, atunci f ⋅ g admite primitive.
DemonstraŃie. Fie F' = f, avem (F ⋅ g)' = f ⋅ g + f ⋅ g' ⇒ f ⋅ g = (F ⋅ g)' – f ⋅ g', urmează că f ⋅ g admite primitive.
Fie A = {a, b}, a < b. Pe intervalele (–∞, a] şi [b, ∞) funcŃia g este strict monotonă. Deoarece g este funcŃie
surjectivă şi, folosind proprietatea de monotonie, rezultă că există limitele: lim g ( x ) şi lim g( x ) , una este –∞
x → −∞ x →∞
şi cealaltă +∞. Analizăm cazul lim g( x ) = –∞ şi lim g( x ) = +∞, celălalt caz se tratează analog. Avem că a este
x → −∞ x →∞
punct de maxim (strict), iar b este punct de minim (strict); y
fie u < a < b < v; f(u) = f(v). Notăm cu I = (–∞, u] şi J = [v, +∞);
α = f(u), I1 = (–∞, α]; J1 = [α, +∞). Notăm cu g1 restricŃia lui g la *
intervalul I; g1: I → I1. FuncŃia g1 este bijectivă, de două ori
derivabilă, g1'' continuă. FuncŃia f  g1 ⋅ g1', conform lemei, *
–1
admite primitive, fie una F1 pe I. Avem: (F1  g1 )'(t) = (F1' 
 g1–1)(t) ⋅ (g1–1)'(t) = (f  g ⋅ g1')(g1–1(t)) ⋅ (g1–1)'(t) = f(t) ⋅ u a 0 b v x
⋅ g1'(g1–1(t)) ⋅ (g1–1)'(t) = f(t). Deoarece (t)' = (g1  g1-1)'(t) = 1,
t ∈ I1. Deci f are primitive pe I1. La fel se arată că restricŃia lui f la J1 admite primitive.
Folosim următoarea lemă: Fie f: R → R, α ∈ R. Dacă restricŃiile lui f la intervalele (–∞, α] şi [α, ∞) admit
primitive, atunci f admite primitive.
DemonstraŃie. Fie F1 şi F2, astfel încât F1'(x) = f(x), (∀) x ≤ α şi F2'(x) = f(x), (∀) x ≥ α. Considerăm F(x) =
F ( x ), x ≤ α F( x ) − F(α ) F ( x ) − F1 (α )
= 1 , cu c = F1(α) – F2(α). Avem lim = lim 1 = f (α) ⇒ F'(α – 0) = f(α).
F
 2 ( x ) + c, x > α x ↑ α x − α x ↑ α x−α
La fel F'(α + 0) = f(α). Deci F'(α) = f(α).

Concursul de matematică „UNIREA” EdiŃia a X-a, Focşani, 29 ianuarie 2010


CLASA A IX-
IX-A

 2 
1. Dacă n = ak, cu k natural, atunci  n 2  = k 2 . Reciproc, să presupunem că n nu se divide cu a. Fie n = aq + r,
 a 
n2 2 2aqr + r 2 2aqr + r 2
cu 0 < r < a. Atunci = q + . Dar 2aqr + r 2
< 2a 2
q + a 2
, deci < 2q + 1 . Pe de altă parte,
a2 a2 a2
2aqr + r 2
dacă 2aqr + r2 < a2, atunci 2aqr + r2 < 2n – 1 = 2aq + 2r – 1, contradicŃie, deci ≥ 1 . Deducem că q2 + 1 ≤
a2

350
n2  n2 
≤ 2
< (q + 1) 2 , deci  2  nu poate fi pătrat perfect; 2. Fie y > 0 şi funcŃiile g, h: R → R, g(x) = f(x) + f(x + y);
a  a 
1
h(x) = f(x) + f(x + 2y). Din ipoteză rezultă că g(x), h(x) ∈ Q, pentru orice x ∈ R. Dar f(x) = (g ( x ) + h ( x ) −
2
– g(x + y)), deci f(x) ∈ Q, pentru orice x ∈ R; 3. a) DemonstraŃia sintetică sau vectorială; b) Folosind a) şi
AM BM CM AM BM CM
teorema lui Ceva, avem: ⋅ ⋅ = 2+ + + ; 4. Fie D mijlocul laturii BC şi A', B', C', D'
MA' MB' MC' MA ' MB' MC'
proiecŃiile punctelor A, B, C, D pe dreapta d. Atunci avem AA' = 2DD' = BB' + CC', de unde [AMN] = [BMN] +
+ [CMN], de unde 2[AMN] + [BOC] – [MON] = [AMN] + [BMN] + [CMN] + [BOC] – [MON] = [ABC].

CLASA A X-
X-A

1. O analiză simplă ne arată că a1 = 1, a2 = 2, a3 = 4. Să considerăm cele an+2 submulŃimi ale mulŃimii {1, 2, ...,
n + 1, n + 2} care nu conŃin numere consecutive. SubmulŃimile care conŃin numărul n + 2 nu conŃin n + 1, deci
sunt submulŃimi (incluzând submulŃimea vidă) ale mulŃimii {1, 2, ..., n} care nu conŃin numere consecutive, prin
urmare numărul lor este an + 1. SubmulŃimile care nu conŃin numărul n + 2 sunt, evident, în număr de an+1. De
a a a
aici concluzia; 2. CondiŃia ac = ac se scrie: =   , deci ∈ R. Aceasta înseamnă că punctele A, C şi O
c c c
(originea sistemului de axe) sunt coliniare. Similar, B, D, O sunt coliniare, deci O este intersecŃia diagonalelor
patrulaterului ABCD. Dar atunci suma OA + OC are direcŃia diagonalei AC, iar suma OB + OD direcŃia
diagonalei BD. Nu putem avea: OA + OC + OB + OD = 0 decât dacă OA + OC = OB + OD = 0 . De aici rezultă
că O e mijlocul fiecărei diagonale, deci ABCD este paralelogram; 3. a) Pentru x = y = 0 obŃinem f(0) = 0. Pentru
x = 1, y = –1 obŃinem: f(1)(1 + f(–1)) = 0, deci f(1) = 0 sau f(–1) = –1. Să presupunem că f(1) ≠ 0. Pentru x = 2,
y = –1 rezultă f(1) + f(–2) = –1, iar pentru x = –2, y = 1, f(–2)(1 – f(1)) = 0. Din ultimele două relaŃii deducem:
(1 + f(1))(1 – f(1)) = 0, deci f(1) = 1 sau f(1) = –1; b) Pentru y = 1, relaŃia (*) devine f(x + 1) = f(1) + f(x)f(1),
pentru orice x ∈ Z.
Cazul 1. Dacă f(1) = –1, relaŃia se scrie: f(x + 1) = –1 – f(x) şi obŃinem imediat prin inducŃie că: f(x) =
0, x par
= .
− 1, x impar
Cazul 2. Dacă f(1) = 0, avem evident f(x) = 0, pentru orice x ∈ Z.
Cazul 3. Dacă f(1) = 1, obŃinem: f(x + 1) = 1 + f(x), şi deducem inductiv că f(x) = x, pentru orice x ∈ Z. Toate
cele trei soluŃii verifică condiŃia (*); 4. Fie a, b, c afixele punctelor A, B, C. Avem d = kb + (1 – k)c, e = kc +
a −b d−e
+ (1 – k)a, f = ka + (1 – k)b. CondiŃia de asemănare a triunghiurilor ABC şi DEF este: = . Efectuând
a −c d−f
1
calculele, obŃinem: (2k – 1)(a2 + b2 + c2 – ab – ac – bc) = 0. Cum k ≠ , deducem că, condiŃiile sunt echi-
2
valente.

351
CLASA A XI-
XI-A

1. a) Se verifică, folosind proprietăŃile determinanŃilor; b) Problema se reduce la faptul că determinantul


Vandermonde V(a1, a2, ..., an) este divizibil cu 1! ⋅ 2! ⋅ ... ⋅ (n – 1)!; 2. a) Se verifică prin calcul direct; b) Se ştie
că tr(AB) = tr(BA) şi că tr(AB)2 = tr(BA)2. Din a), tr(AB) = a + d şi tr(AB)2 = a2 + d2 + 2bc, rezultă concluzia;
n n n

∑ ∑ ∑
1
1 1 1
n

n k =1 k k =1 k n k k
3. a) ExerciŃiu clasic; b) Avem că ≤ ≤ k =1 . Concluzia rezultă din lema „cleştelui”;
ln n ln n ln n
4. a) Din ipoteză rezultă că |f(x)| ≤ |f(0)| + |g(x)| + |g(0)|, deci f este mărginită pe o vecinătate V a lui ∞. Fie
h(x) = sup f ( t ) , pentru x ∈ V; h este descrescătoare şi mărginită, deci lim h ( x ) = l ∈ R. Arătăm că
t≥x x→∞

lim f ( x ) = l . Într-adevăr, dacă lim h ( x ) = L şi ε > 0, atunci pentru o vecinătate U a lui ∞ avem |g(x) – L| <
x →∞ x →∞
1 2 2
< ε , deci |g(x) – g(y)| < ε , de unde |f(x) – l| ≤ ε < ε ; b) Nu; un exemplu este dat de f(x) = sin x, g(x) = x.
3 3 3

CLASA A XII-
XII-A

1. a) Avem x3y3 = xyxyxy, deci x2y2 = (xy)2, (∀) x, y ∈ G. Rezultă x4y4 = (x2)2(y2)2 = (y2x2)2 = (xy)4. ObŃinem
astfel x3y3 = (xy)3, de unde, conform ipotezei, xy = yx; b) Din cele de mai sus şi din condiŃiile de morfism, x3y3 =
=(yx)3 = y3x3. Folosind ipoteza deducem tz = zt, (∀) t, z ∈ G; c) Un exemplu este grupul diedral D4, grup în care
x4 = x, (∀) x ∈ D4; 2. Pentru X ∈ G avem XS = ∑ XA = ∑ B = S , deoarece funcŃia A → XA este injectivă.
A∈G B∈G
Prin adunare, deducem S2 = kS. Rezultă (S – kIn)2 = –kS + k2In = –k(S – kIn), de unde obŃinem inductiv,
(S – kIn)p = (–k)p–1(S – kIn). Pe de altă parte, dacă valorile proprii ale matricei S sunt λ1, ..., λn, atunci valorile
proprii ale matricei S2 sunt λ21 , ..., λ2n , de unde max|λi|2 ≤ kmax|λi|, deci max|λi| ≤ k. Cum t = nk = ∑λ i , reiese
că λi = k, (∀) i, deci polinomul caracteristic al matricei S este (X – k) . Deducem astfel că (S – kIn) = On =
n n

1
1 x
= (–k)n–1(S – kIn), de unde concluzia; 3. ∫f  dx = n
0 n ∫
0
n f ( y)dy . Limita şirului dat conduce la L =

1 x f (x) 1
= lim
x→0 x2
⋅ ∫ f ( y)dy . Folosind l'Hospital şi definiŃia derivatei,
0
L = lim
x →0 2x
= f ' (0) ; 4. |F(x)| ≤ |F(0)| +
2
x x 1 π
+ ∫ f (t )dt ≤ |F(0)| + ∫
0 0 t2 +1
dt ≤| F(0) | + . Fie g(x) = sup F( t ) ; g este descrescătoare şi mărginită, deci există
2 t≥x

şi este finită limita lim g( x ) = l . Arătăm că lim F( x ) = l . Într-adevăr, dacă ε > 0, atunci pentru o vecinătate U a
x→∞ x→∞

π 1 2 2
lui ∞ avem: arctgx − < ε , deci |F(x) – F(y)| ≤ |arctg x – arctg y| < ε , de unde |F(x) – l| ≤ ε < ε , (∀) x ∈ U.
2 3 3 3

352
Concursul interjudeŃean de matematică „Grigore Moisil”, Urziceni,
EdiŃia a VI-a, 29-31 ianuarie 2010

CLASA A IX-
IX-A

1 1 1  1  1
1. Notând Hn = 1 + + ... + , inegalitatea se rescrie:  H 2n − H n  > H n ⇔ 2nH 2 n > (2n + 1)H n ⇔
2 n n +1 2  2n
 1 1 1  1 1 2n 1
⇔ 2n(H2n – Hn) > Hn ⇔ 2n  + + ... +  > 1 + + ... + , adevărat întrucât > , k ∈ {1,
 n + 1 n + 2 2 n  2 n 2 n − k k +1
2, ..., n – 1}, n ≥ 2; 2. Termenul general al şirului este de forma an2 + bn + c. Se deduce că a = –72, b = 492 şi
c = 1020. Al optulea termen este 348; 3. a) (n + 1)2 se găseşte în mulŃime; b) Dacă k4 < n, atunci n3 + n2 + n +
1
+ 2 > (k + 1)4; 4. Notăm rX vectorul de poziŃie al punctului X în raport cu o origine O. Deducem: rM = ⋅ (rB +
4
1
+ rC + rD + rE ) şi analoagele pentru rN , rP , rQ . Prin calcul rM + rP − rN − rQ = (rB + rD − rA − rC ) . Atunci:
4
MNPQ este paralelogram dacă şi numai dacă rM + rP = rN + rQ ⇔ rB + rD = rA + rC ⇔ ABCD este paralelogram.

CLASA A X-
X-A

1. f(64) = f(16) = f(8) = 9. Dar f(20) = f(16 + 4) = f(64) = 9. Acum f(9) = f(20) = 9; 2. Un exemplu ar fi z1 = 1,
−1+ i 3 z z − w (z1 + z 2 ) z z + z 2 z 3 + z 3 z1
z2 = , z3 = 0. Dacă z3 ≠ 0, atunci z 3 = 1 2 , unde w = 1 2 . Se arată uşor
2 w − z1 − z 2 z1 + z 2 + z 3
1
că z 3 = ; 3. Observăm că 7, 19 şi 31 sunt numere prime. Există deci n, m ∈ Z astfel încât 1 = 31 ⋅ n + 7 ⋅ 19 ⋅
z3
⋅ m. Avem că h(x + 1) = h(x + 31n + 7 ⋅ 19 ⋅ m) = h(x + 7 ⋅ 19 ⋅ m) = k – f(x + 7 ⋅ 19 ⋅ m) – g(x + 7 ⋅ 19 ⋅ m) =
= k – f(x) – g(x) = h(x), k ∈ Z fixat. Deci h are perioada 1, deci este constantă. Analog pentru f şi g; 4. a) EcuaŃia
este echivalentă cu următoarea: log 0,5 (sin 2 x ) ⋅ log 0,5 (cos 2 x ) = 1 . Cu inegalitatea mediilor obŃinem succesiv: 1 =
2
 2 
 2
2
  log 2 sin 2x 
log (sin x cos x )
= log 0,5 (sin 2 x ) ⋅ log 0,5 (cos 2 x ) ≤  0,5  = 4  ≤ 1 . Egalitatea se obŃine când tg2 x =
 
 2  
2

 
π π
= 1, deci x = ± + kπ , k ∈ Z; b) Analog ca la punctul a) obŃinem: x = ± + kπ , k ∈ Z.
4 4

CLASA A XI-
XI-A

1. Şirul (xn) este mărginit şi monoton descrescător cu limita 0. Folosind lema Stolz-Césaro se poate arăta
lim y n = 1 ; 2. RelaŃia din enunŃ se scrie echivalent: (I2 – A)4 = O2. De aici rezultă că (I2 – A)2 = O2, adică A =

353
2
1 1  1 1
=  A + I 2  ; 3. a) Şirurile (an), (bn), (cn) tind la zero (rezultă de exemplu din f n   → 0, f n   → 0 ,
 2 2   
2 3
1 2
f n   → 0 ). Considerăm wn = |an| + |bn| + |cn|; b) Alegem gn(x) = xn, |xn| ≤ vn nu este satisfăcută dacă x = n ;
 
n 3
n–1
4. Ambele afirmaŃii pot fi dovedite prin inducŃie. Pentru k = 1 se obŃin egalităŃi cunoscute: detA* = d şi A* =
= dA–1. Presupunem că (a) şi (b) au loc pentru k. În relaŃia B ⋅ B* = (detB)In luăm B = ((...(A*)*...)*)* cu det B =
k k +1
= d ( n −1) (conform ipotezei inductive). Se obŃine imediat că detB* = d ( n −1) şi B* = (detB)B–1, deci, în cele din
urmă, (a) şi (b) au loc pentru k + 1.

CLASA A XII-
XII-A

1
a dx 1 t 2010 dt a 1
1. Fie I(a ) = ∫1
a
2
(1 + x )(1 + x 2010
)
, a > 0. Cu substituŃia x = rezultă I(a ) = −
t ∫a
a
(1 + t 2 )(1 + t 2010 )
= ∫ 1+ t
1
a
2

 1  1 1 a dx
∫ (1 + x
a
⋅ 1 − 2010 
dt = arctgt 1 − I(a ) . Rezultă că I(a) =  arctga − arctg  . Acum J(a) = 2
=
 1+ t  a
2 a 0 )(1 + x 2010 )
1 1 1
dx dx 1
= ∫ 0
a
(1 + x )(1 + x 2010 )
2
+ I(a ) . Cum 0 ≤ ∫0
a
(1 + x )(1 + x 2010 )
2
≤ ∫
0
a dx =
a
, rezultă că:

1
dx π
lim
a →∞ 0
a
∫ 2
(1 + x )(1 + x 2010
)
= 0 şi lim J (a ) = lim I(a ) = ; 2. a) Putem considera funcŃia f: [0, 1] → [0, 1], f(x) =
a → ∞ a → ∞ 4
0, x ∈ [0, a ]
1− c x − a

= x c , c ∈ (0, 1). Evident f ∈ F şi T(f) = c; b) Fie f: [0, 1] → [0, 1], f ( x ) =  , x ∈ (a , b) , unde 0 ≤ a < b ≤
b − a
1, x ∈ [b,1]
a+b
≤ 1. Se verifică imediat că T(f ) = 1 − . Alegând convenabil a, b, c, d (0 ≤ a < b ≤ 1, 0 ≤ c < d ≤ 1, a + b =
2
= c + d), obŃinem două funcŃii de forma lui f, u şi v, u ≠ v şi T(u) = T(v); 3. Derivând egalitatea f2(x) = 2010 +
x
+ ∫0
(f 2 ( t ) + f '2 ( t ))dt , obŃinem: 2f(x)f'(x) = f2(x) + f'2(x), de unde (f(x) – f'(x))2 = 0, pentru orice x ∈ R. Rezultă

f'(x) = f(x), x ∈ R, deci f(x) = cex, impunând ca f să verifice relaŃia iniŃială, găsim c = 2010 ; 4. Pentru orice x,
y ∈ S: x & y = (x & y) & (x & y) = (y & (x & y)) & x = ((x & y) & x) & y = ((y & x) & x) & y = ((x & x) & y) & y = (x &
& y) & y = (y & y) & x = y & x.

354
Concursul interjudeŃean de matematică „Nicolae Coculescu”, Slatina,
EdiŃia a VI-a, 27 noiembrie 2009

CLASA A IX-
IX-A

1 1 1 1 a +b 1a +b a +b a+b


1. Folosind inegalitatea ≤  +  , rezultă ≤  +  şi analoagele, obŃinem +
a +c 4a c a+c 4 a c  a+c
b+c c+a 1a+b a +b b+c b+c c+a c+a 1   b c a   a b c 
+ + ≤  + + + + +  = 3 + 2 + +  +  + +  .
b+a c+b 4 a c b a c b  4  a b c   b c a 
a b c b c a 1
Notăm = x, = y, = z . Atunci xyz = 1 şi = yz, = zx, = xy . Este suficient să arătăm că (3 + 2( xy +
b c a a b c 4
1
+ yz + zx) + x + y + z) ≤ [(xy + 1) + (yz + 1) + (zx + 1) ], inegalitate care este echivalentă cu x2y2 + x2z2 +
2 2 2
4
+ y2z2 ≥ x + y + z. Aceasta rezultă aplicând inegalitatea A2 + B2 + C2 ≥ AB + BC + CA numerelor A = xy, B =
= yz şi C = zx; 2. Pentru început să observăm că [12S1] = [12S2] = 2 şi [12S3] = 3. Având în vedere că pn+1 – pn ≥
1 2 2 2 2 2 1 p − p2
≥ 2, pentru orice n ≥ 2, obŃinem: 2Sn = + + + + ... + + ≤ + 3 +
3 p 2 p 3 p3p 4 p 4 p 5 p n −1p n p n p n +1 3 p 2p3
p 4 − p3 p5 − p 4 p − p n −1 p n +1 − p n 1  1 1   1 1   1 1   1 1 
+ + + ... + n + = +  −  +  −  +  −  + ... +  −  +
p 3p 4 p 4p5 p n −1p n p n p n +1 3  p 2 p3   p3 p 4   p 4 p5   p n −1 p n 
 1 1  1 1 1 1 1 2 1 11
+  −  = + − < + = . Deci Sn < , (∀) n ≥ 2. Pentru orice n ≥ 3, avem Sn ≥ S3 = >
p
 n p n +1  3 p 2 p n +1 3 p 2 3 3 42
1 1 1
> , deci < Sn < , pentru orice n ≥ 3. Ca urmare, 3 < 12Sn < 4, de unde [12Sn] = 3, (∀) n ≥ 3.
4 4 3
3. a) Fie N şi P punctele în care ME intersectează laturile [AB], respectiv A
[AC]. Cum AE este şi bisectoare şi înălŃime în triunghiul ANP, rezultă că R
[AN] ≡ [AP], [EN] ≡ [EP] şi 'ANP ≡ 'APN. Ca urmare, avem şi 'BNM ≡
MN BN MN
≡ 'CPM, deci ∆BNM ~ ∆CPM, de unde = . Notând = r şi
MP CP MP
CP = x, rezultă BN = rx, AN = c – x, AP = b – rx. Cum AN = AP, rezultă P
E
b−c cr − b
c – x = b – rx, de unde x = şi AN = AP = . Deoarece E şi Q N F
r −1 r −1 M
1
sunt mijloacele segmentelor [NP] şi [BC], rezultă EQ = ( NB + PC) = B Q C
2
1  NB PC  r (b − c) b−c b−c r 1 
=  ⋅ AB + ⋅ AC  = AB + AC = ⋅
 AB + AC  , (1). Întrucât [NE] ≡ [NP]
2  AB AC  2c ( r − 1) 2 b ( r − 1) 2(r − 1) c b 
FN 1 r
şi F este simetricul lui M faŃă de E, rezultă [FN] ≡ [MP] şi [FP] ≡ [MN], deci = . Atunci AF = AN +
FP r 1+ r
1 r AN 1 AP r (cr − b) cr − b cr − b  r 1
+ AP = ⋅ ⋅ AB + ⋅ ⋅ AC = AB + AC =  AB + ⋅
1+ r 1 + r AB 1 + r AC c(1 + r )(r − 1) b(1 + r )(r − 1) (1 + r )(r − 1)  c b
⋅ AC) , (2). Din relaŃiile (1) şi (2) deducem că există α ∈ R astfel încât EQ = α AF , deci EQ || AF; b) Fie R

355
proiecŃia lui M pe bisectoarea exterioară a unghiului A. Atunci RA ⊥ AE, şi cum AE ⊥ EM şi MR ⊥ RA, rezultă
că patrulaterul ARME este dreptunghi. Atunci AR || ME şi [AR] ≡ [ME] ≡ [FE], deci AREF este paralelogram.
Ca urmare, RE || AF, şi cum AF || EQ, rezultă că punctele R, E, Q sunt coliniare;
4. Notăm cu Li segmentele şi cu li lungimile lor, i = 1, n . Proiectăm fiecare segment pe două laturi adiacente ale
pătratului. Fie acestea [AB] şi [AD]. Notăm cu pi şi qi lungimea proiecŃiei segmentului Li pe [AB], respectiv
p + qi
[AD]. Avem: li = p i2 + q i2 , de unde obŃinem că i ≤ l i ≤ pi + q i , pentru orice i = 1, n . Atunci avem:
2
100 = l1 + l2 + ... + ln ≤ (p1 + p2 + ... + pn) + (q1 + q2 + ... + qn), (1); 100 2 = 2 (l1 + l 2 + ... + l n ) ≥ (p1 + p 2 + ... +
+ p n ) + (q1 + q 2 + ... + q n ) , (2). Din relaŃia (1) rezultă că cel puŃin una din sumele p1 + p2 + ... + pn şi q1 + q2 +
+ ... + qn este mai mare sau egală cu 50. Presupunând că p1 + p2 + ... + pn ≥ 50, pe latura [AB], există un punct
acoperit de cel puŃin 50 de proiecŃii ale segmentelor Li. O dreaptă paralelă la AD dusă prin acest punct
intersectează cel puŃin 50 de segmente. Din relaŃia (2) rezultă că cel mult una din sumele p1 + p2 + ... + pn şi q1 +
+ q2 + ... + qn este mai mică sau egală cu 50 2 ≅ 70,7... , deci este strict mai mică decât 71. Presupunând p1 +
+ p2 + ... + pn < 71, atunci există cel puŃin un punct pe [AD] acoperit de cel mult 70 de proiecŃii ale segmentelor
Li. O dreaptă paralelă la AB dusă prin acest punct intersectează cel mult 70 de segmente.

CLASA A X-
X-A
x
− 3) x
+ 3)
1. EcuaŃia se scrie sub forma 10 log 7 (10 = 7 lg(7 + 3 . Notăm log7 (10x – 3) = y şi lg(7x + 3) = z. Atunci
7 z + 3 + 10 y  y = lg(7 z + 3)  y = f (z)

 y x  y 
ecuaŃia devine: 10 = 7 + 3, şi obŃinem relaŃiile: 7 + 3 = 10 ⇔ x = lg(7 + 3) ⇔ x = f ( y) , unde f: R → R,
y z

 x z  x z = f ( x )
7 + 3 = 10 z = lg(7 + 3) 
f(t) = lg(7t + 3). FuncŃia f este strict crescătoare, fiind o compunere de funcŃii strict crescătoare. Din motive de
simetrie, putem presupune că x ≤ y ≤ z (orice altă ordonare se tratează analog). Atunci f(x) ≤ f(y) ≤ f(z), de unde
rezultă z ≤ x ≤ y, deci x = y = z. ObŃinem 7x + 3 = 10x, ecuaŃie care are soluŃia unică x = 1; 2. a) Fie x, y ∈ R
x−y  x+y x+ y
astfel încât f(x) = f(y). Atunci a(x – y) = b(cos y – cos x) + c(sin y – sin x) = 2 sin ⋅  b sin − c cos ,
2  2 2 
x+y x+y x−y x−y
(1). Dar b sin − c cos ≤ b 2 + c 2 ≤| a | şi sin ≤ , cu egalitate dacă x = y, deci |a(x – y)| =
2 2 2 2
x−y x+y x+y
= |a| ⋅ |x – y| ≥ 2 sin ⋅ b sin − c cos . Din relaŃia (1) deducem că în inegalitatea de mai sus avem
2 2 2
egalitate, deci x = y, adică f este injectivă; b) Fie T > 0 o perioadă a funcŃiei g. RelaŃia g(x + T) = g(x), (∀) x ∈ R
T  T  T 
conduce la f(x + T) – f(x) ∈ Z, (∀) x ∈ R, echivalent cu: aT − 2 sin  b sin  x +  − c cos x +   ∈ Z, (∀)
2  2  2 
T  T  T 
x ∈ R. Notăm h: R → R, h(x) = aT − 2 sin  b sin  x +  − c cos x +   . Atunci Im h ⊂ Z, (2). Cum
2   2   2 
 T  T T
imaginea funcŃiei x → b sin  x +  − c cos x +  este intervalul [− b 2 + c 2 , b 2 + c 2 ] , dacă sin ≠ 0 şi
 2  2 2
b 2 + c 2 ≠ 0 , atunci Im h conŃine şi numere care nu sunt întregi, contradicŃie cu (2). Ca urmare, condiŃia (2)

356
este verificată în unul din cazurile: 1) b2 + c2 = 0 şi aT ∈ Z, de unde b = c = 0, a ∈ R; dacă a ≠ 0 perioadele
k T p
funcŃiei g sunt de forma , k ∈ Z; 2) sin = 0 şi aT ∈ Z, de unde T = 2kπ, k ∈ Z şi a = , p ∈ Z, deci a =
a 2 2 kπ
q
= , q ∈ Q şi b, c ∈ R; 3. a) Notăm x0 = (1 – t0)a + t0b. Aşadar f(x0) = (1 – t0)f(a) + t0f(b). Presupunem prin
π
reducere la absurd că există t ∈ (0, 1) astfel încât f((1 – t)a + tb) < (1 – t)f(a) + tf(b). Analizăm cazul t ∈ (0, t0).
Cazul t ∈ (t0, 1) se tratează analog. Fie xt = (1 – t)a + tb. Cum 0 < t < t0 < 1, rezultă că a < xt < x0 < b, deci există
x − x t (1 − t 0 )a + t 0 b − (1 − t )a − tb t 0 − t
λ ∈ (0, 1) astfel încât x0 = (1 – λ)xt + λb. Mai precis avem: λ = 0 = = .
b − xt b − (1 − t )a − tb 1− t
1− t0
Atunci f(x0) = f((1 – λ)xt + λb) ≤ (1 – λ)f(xt) + λf(b) < (1 – λ)[(1 – t)f(a) + tf(b)] + λf(b) = ⋅ [(1 − t )f (a ) +
1− t
t −t
+ tf (b)] + 0 f (b) = (1 − t 0 )f (a ) + t 0 f (b) = f ( x 0 ) , contradicŃie.
1− t
SoluŃie alternativă. Vom folosi faptul că dacă f este convexă pe intervalul I, atunci pentru orice α ∈ I, funcŃia
f ( x ) − f (α )
pα: I – {α} → R, p α ( x ) = este crescătoare. În condiŃiile problemei, notând x0 = (1 – t0)a + t0b,
x−α
f (a ) − f ( x 0 ) f ( b ) − f ( x 0 ) f ( b ) − f (a )
rezultă = = , adică p x 0 (a ) = p x 0 (b) . Cum p x 0 este funcŃie crescătoare pe
a − x0 b − x0 b−a
R – {x0}, deducem că p x 0 este funcŃie constantă pe mulŃimea [a, b] – {x0}. Rezultă că pentru orice x ∈ [a, b] –
f ( x ) − f ( x 0 ) f (b) − f (a ) f ( b) − f (a )
– {x0} avem: p x 0 ( x ) = p x 0 (a ) ⇔ = ⇔ f (x ) = ( x − x 0 ) + f ( x 0 ) . Cum ultima
x − x0 b−a b−a
f ( b) − f (a )
egalitate este evident adevărată şi pentru x = x0, deducem că: f ( x ) = ( x − x 0 ) + f ( x 0 ) , (∀) x ∈ [a, b],
b−a
adică f este funcŃie afină pe intervalul [a, b]. Concluzia este acum imediată;
 b a  b a
b) RelaŃia din enunŃ se poate scrie sub forma f  ⋅a + ⋅ b = ⋅ f (a ) + ⋅ f (b) şi, notând
a+b a+b  a+b a+b
b
t0 = , avem f((1 – t0)a + t0b = (1 – t0)f(a) + t0f(b). Conform punctului a), rezultă că f((1 – t)a + tb) = (1 –
a+b
x−a b−x x−a
– t)f(a) + tf(b), pentru orice t ∈ [0, 1]. Notând x = (1 – t)a + tb, avem t = , deci f ( x ) = f (a ) + ⋅
b−a b−a b−a
f ( b ) − f (a ) bf (a ) − af (b)
⋅ f ( b) = x+ , pentru orice x ∈ [a, b], deoarece atunci când t parcurge intervalul [0, 1], x
b−a b−a
parcurge intervalul [a, b]. În consecinŃă, funcŃia f este afină pe intervalul [a, b], deci graficul său conŃine
segmentul [AB], unde A(a, f(a)) şi B(b, f(b)), a cărui lungime este cel puŃin egală cu b – a;
4. Pentru x = y = 0 obŃinem f(0) = 0, iar pentru y = 0 rezultă f(x3) = 3f2(x), (∀) x ∈ R, (1). Pentru x = 0 şi y =
= −3 x , obŃinem f(–x) + f(x) = 0, deci f(–x) = –f(x), (∀) x ∈ R, adică f este impară. Pentru y = x obŃinem
f(2x3) = 2xf2(x) şi, Ńinând cont de (1), avem f(2x3) = 2f(x3), (∀) x ∈ R, (2). Vom demonstra prin inducŃie
propoziŃia P(n): f(nx3) = nf(x3), pentru orice x ∈ R şi orice n ∈ N*. Egalitatea are loc pentru x = 0, aşa că în
continuare vom fixa x ≠ 0. PropoziŃiile P(1) şi P(2) sunt adevărate. Presupunem că P(k – 1) şi P(k) sunt adevărate
pentru un k ≥ 2 şi arătăm că P(k + 1) este adevărată. Pentru x → 3
k ⋅ x şi y → x, din relaŃia din enunŃ obŃinem:
f (x 3 )
f ((k + 1) x 3 ) + f ((k − 1) x 3 ) = 2x 3 x ⋅ [f 2 (3 k ⋅ x ) + f 2 ( x )] − 2 xf ( x )f (3 k ⋅ x ) . Din relaŃia (1) rezultă f2(x) = ,
x

357
f (kx 3 )
pentru orice x ∈ R*, deci f 2 (3 k ⋅ x ) = 3
şi, Ńinând cont de ipoteza de inducŃie, rezultă: f((k + 1)x3) =
kx
 kf ( x 3 ) 
= 2x 3 k ⋅  3 + f 2 ( x ) − 2xf ( x )f (3 k ⋅ x ) − (k − 1)f ( x 3 ) = (k + 1)f ( x 3 ) + 23 k ⋅ xf 2 ( x ) − 2xf ( x )f (3 k ⋅ x ) =
 k ⋅ x 
= (k + 1)f ( x 3 ) + 2xf ( x ) ⋅ [3 k ⋅ f ( x ) − f (3 k ⋅ x )] . Rămâne să arătăm că 3
k ⋅ f ( x ) = f (3 k ⋅ x ) . łinând cont de
relaŃia (2) avem: f (kx 3 ) = f ((3 k ⋅ x ) 2 ) = 3 k ⋅ x ⋅ f 2 (3 k ⋅ x ) . Dar f (kx 3 ) = kf ( x 3 ) = kxf 2 ( x ) , deci kxf2(x) =
= 3 k ⋅ x ⋅ f 2 (3 k ⋅ x ) , de unde, simplificând prin 3
k ⋅ x , obŃinem: (3 k ⋅ f ( x )) 2 = f 2 (3 k ⋅ x ) , deci f (3 k ⋅ x ) =
= 3 k ⋅ f ( x ) sau f (3 k ⋅ x ) = −3 k ⋅ f ( x ) . Pentru x → 3 x , din relaŃia (1) obŃinem că f ( x ) = 3 x ⋅ f 2 (3 x ) , deci
f (x ) 1 f (x )
3
> 0 , de unde, înmulŃind cu , rezultă > 0 , (∀) x ∈ R*. Presupunând f (3 k ⋅ x ) = −3 k ⋅ f ( x ) ,
x 3
x 2 x

f (3 k ⋅ x ) f (x )
obŃinem =− , contradicŃie, deoarece membrul stâng este pozitiv, iar membrul drept este negativ.
3
k ⋅x x
Ca urmare, f (3 k ⋅ x ) = 3 k ⋅ f ( x ) . Aşadar, f(nx3) = nf(x3), (∀) x ∈ R, (∀) n ∈ N*, de unde, pentru x → 3
x,
x x x 1
obŃinem f(nx) = nf(x), (∀) n ∈ N*. Pentru x → , obŃinem f ( x ) = nf   , deci f   = f ( x ) . Atunci
n n n n
m   x x m
f  x  = f  m ⋅  = mf   = f ( x ) , (∀) m, n ∈ N*, deci f(qx) = qf(x), (∀) q ∈ Q+, (∀) x ∈ R. Din
 n   n  n n
imparitatea funcŃiei f deducem f(qx) = qf(x), (∀) q ∈ Q, (∀) x ∈ R. Din relaŃia (1), pentru x = 1 obŃinem f2(1) =
= 1, deci f(1) ∈ {0, 1}. Dacă f(1) = 0, atunci f(q) = 0, (∀) q ∈ Q, deci mulŃimea A = {x ∈ R | f(x) = 0} este
infinită. Ca urmare, f(1) = 1, deci f(q) = q, (∀) q ∈ Q. CerinŃa problemei se reduce la a arăta că pentru orice x ∈
1
∈ R, există a, b, ∈ Q astfel încât |b – a| ≤ şi a < x < b, ceea ce rezultă astfel:
2
1 1
- dacă x ∈ Q, alegem a = x − şi b = x + ;
4 4
1  1
- dacă x ∈ R – Q, alegem a = [ x ] + şi b = [x] sau b = [x] + 1, după cum x ∈  [ x ], [ x ] +  sau x ∈
2  2
 1 
∈  [ x ] + , [ x ] + 1 .
 2 

CLASA A XI-
XI-A

1. Avem succesiv: 2A(B–1 + C) = B(A–1 + C) ⇔ 2AB–1 + 2AC – BA–1 – BC = On ⇔ 2AB–1 – BA–1 – (2A – B) ⋅
⋅ C = On ⇔ 2AB–1 – BB–1 + 2AA–1 – BA–1 + (2A – B)C = In ⇔ (2A – B)B–1 + (2A – B)A–1 + (2A – B)C = In, de
unde rezultă că (2A – B)(B–1 + A–1 + C) = In. Ca urmare, matricea 2A – B este inversabilă şi (2A – B)–1 = B–1 +
+ A–1 + C, deci avem şi (B–1 + A–1 + C)(2A – B) = In. ObŃinem: 2B–1A – B–1B + 2A–1A – A–1B + 2CA – CB = In
⇔ 2(B–1 + C) ⋅ A = (A–1 + C) ⋅ B; 2. a) Dacă λ este o valoare proprie a matricei A, atunci λ3 = 3λ – 2, deci λ ∈
∈ {–2, 1}. Ca urmare, polinomul caracteristic al matricei A are forma PA(X) = det(XIn – A) = (X – 1)α(X + 2)β,
cu α + β = n. Deoarece A2 + A + In = (εIn – A)( ε2In – A) = PA(ε)PA(ε2) = PA(ε)PA( ε) = PA (ε) ⋅ PA (ε) = | PA(ε)|2.

358
Dar PA(ε) = (ε – 1)α( ε + 2)β = (ε – 1)α (–1 – ε2 + 2)β = (ε – 1)α(1 – ε2)β = (ε – 1)α(1 – ε)β , deci |PA(ε)| =
= |(ε – 1)α(1 – ε)β =| ε − 1 |α ⋅ | 1 − ε |β =| ε − 1 |α + β =| ε − 1 |n = ( 3 ) n , de unde det(A2 + A + In) = 3n;
 d1 0 ... 0 
 
 0 d2 ... 0 
b) A =  , unde d k ∈ {−2,1}, k = 1, n (nu toate egale);
... ... ... ... 
 
0 0 ... d n 

19
3. Demonstrăm prin inducŃie matematică inegalitatea a 2 n > 2n , (∀) n ≥ 2. Într-adevăr, avem a 4 = > 2 şi,
9
1 1 1 1
presupunând a 2 k > 2k pentru un număr natural k ≥ 2, avem: a 2 k + 2 = + > = >
2k + 1 a 22 k +1 a 22k +1  1 1 
2
 
 2k + a 2 
 2k 
1
> 2
= k 2 > 2k + 2 . În concluzie, a 2 n > 2n , pentru orice n ≥ 2, deci lim a 2 n = +∞ . Întrucât
n →∞
 1 1 
 + 
 2k 2k 
1 1  1 1 
a2n+1 = + 2 , rezultă lim a 2 n +1 = lim  + 2  = 0 . Şirul (an)n≥1 conŃine două subşiruri cu limite diferite,
2n a n n →∞ n → ∞ 2n an 

1
ea n a n − ln n 1 1 1
deci nu are limită; 4. a) Pentru orice n ≥ 1 avem = e a n − ln n
=e 2 . Notăm H n = 1 + + + ... + , n ≥
n 2 3 n
1 1 1
≥ 1. Atunci a n = H 2 n − H n , de unde rezultă că a n − ln n = (H 2 n − ln 2n ) − (H n − ln n ) + ln 2 . Având în
2 2 2
 1  1
vedere că lim (H n − ln n ) = C (constanta lui Euler), obŃinem lim  a n − ln n  = lim (H 2 n − ln 2n ) − lim (H n −
n →∞ n → ∞ 2  n → ∞ 2 n →∞
1
1 1 ea n C + ln 2
− ln n ) + ln 2 = C −
C + ln 2 = C + ln 2 , de unde lim = e2 = 2 e C ; b) Arătăm că pentru orice
2 2 n →∞ n

număr natural n ≥ 1, există kn ∈ N* astfel încât n ≤ a k n < n + 1 , adică [a k n ] = n . Deoarece [a1] = 1, putem
1 1 1
considera k1 = 1. Fie n ≥ 2. Întrucât a n > 1 + + ... +  → ∞ , rezultă că an → ∞, deci există rn ∈ N*, astfel
2 2 n
încât a rn ≥ n . Notăm cu kn cel mai mic număr natural cu proprietatea a k n ≥ n . Rezultă kn ≥ 2 şi a k n −1 < n, deci
1 1
akn − < n , adică a k n < n + < n + 1 . În concluzie, n ≤ a k n < n + 1 . Deoarece (an)n≥1 este strict
2k n − 1 2k n − 1
crescător şi a k n < n + 1 ≤ a k n +1 , pentru orice n ≥ 1, rezultă că şirul (kn)n≥1 este strict crescător. Ca urmare, şirul
1 1
+
(sn)n≥1, sn = a k n , este un subşir al şirului (an)n≥1. Deoarece n ≤ sn < n + 1, (∀) n ≥ 1, obŃinem: + ... +
n+2 n+3
1 1 1 1 1 1 1 1 1 1
+ < + + ... + ≤ + + ... + . Notând u n = + + ... + , pentru
2 n + 1 s n +1 s n + 2 s 2n n + 1 n + 2 n+n n +1 n + 2 n+n

359
1 1 1 1 1
orice n ≥ 1, avem u n − + < + + ... + ≤ u n . Cum un → ln 2, rezultă lim s n = ln 2 .
n + 1 2 n + 1 s n +1 s n + 2 s 2n n →∞

CLASA A XII-
XII-A

1. a) ⇒ b) Cum Sn este izomorf cu Un şi cum Xn = A, pentru orice X ∈ Sn, rezultă că A este element neutru în Sn,
deci A2 = A; b) ⇒ a) Cum A2 = A şi A ≠ I2, rezultă că detA = 0. Dar Xn = A, deci detX = 0, pentru orice X ∈ Sn.
De aici rezultă că, dat fiind X ∈ Sn, există t ∈ C astfel încât X2 = tX. Atunci A = Xn = tn–1X, şi cum A ≠ O2,
rezultă t ≠ 0, deci X = ξA, unde ξ = t–(n–1) ∈ C. Din Xn = A rezultă ξn = 1, deci Sn ⊂ {ξA | ξn = 1}. Reciproc,
să observăm că dacă X ∈ {ξA | ξn = 1}, atunci Xn = A, deci Sn = {ξA | ξn = 1} = A ⋅ Un. Concluzia este
x 1 ( x )2 − ( 1 − x )2 + 1 1  1 
acum evidentă; 2. Întrucât = ⋅ =  x − 1 − x +  , rezultă
x + 1− x 2 x + 1− x 2  x + 1− x 
x 1 1 dx
că ∫ x + 1− x
dx =
3
( x x + (1 − x ) 1 − x ) +
2 ∫ x + 1− x
. Pentru a determina primitivele funcŃiei

1  π
f: (0, 1) → R, f ( x ) = , efectuăm schimbarea de variabilă ϕ:  0,  → (0, 1), x = ϕ(t) = sin2 t.
x + 1− x  2
 π sin 2t
Trebuie să calculăm primitivele funcŃiei h:  0,  → R, h(t) = f(ϕ(t)) ⋅ ϕ'(t) =
 2  sin t + cos t ∫
. Avem: h ( t )dt =

 t 
2 tg − 1' dt
(sin t + cos t ) 2 − 1  1   2  1
= ∫ sin t + cos t  ∫
dt =  sin t + cos t − dt = sin t − cos t +
sin t + cos t   t
∫ 
2
= sin t – cos t +
2

 tg − 1 − 2
 2 
t  t 
−1− 2
tg  1 + 2 − tg 
1 2  + C . Cum tg t = sin t = x
⋅ ln 2 + C = sin t − cos t + ln , rezultă:
t
tg − 1 + 2 2  2 − 1 + tg 
t 2 1 + cos t 1+ 1− x
 
2  2
xdx x x + (1 − x ) 1 − x
x − 1− x 1 ( 2 + 1)(1 + 1 − x ) − x
∫ x + 1− x
=
3
+
2
+ ln
2 2 ( 2 − 1)(1 + 1 − x ) + x
+C;

3. CondiŃia b) impune card(H1 ∪ H2) ≥ 2p2 – (p – 2). Într-adevăr, presupunând contrariul, mulŃimea K = G –
– (H1 ∪ H2) ar conŃine cel puŃin p – 1 elemente. Atunci, pentru orice α ∈ H1 ∪ H2 rezultă că S = K ∪ {α} are cel
puŃin p elemente, deci card(S ∩ (H1 ∪ H2)) ≥ 2, de unde obŃinem card(K ∩ (H1 ∪ H2)) ≥ 1, contradicŃie.
Presupunem prin absurd că H1 ≠ G şi H2 ≠ G. Atunci ord(H1) < ord(G), şi cum ord(H1) | ord(G) = 2p2, rezultă
ord(H1) ≤ p2. Atunci 2p2 – (p – 2) ≤ card(H1 ∪ H2) = ord(H1) + ord(H2) – card(H1 ∩ H2) = ord(H1) + ord(H2) –
– 1 ≤ p2 + ord(H2) – 1, de unde obŃinem că ord(H2) ≥ p2 – p + 3. Dar ord(H2) ∈ {1, 2, p, 2p, p2} şi cum p este
prim, rezultă p2 – p + 3 > 2p. Ca urmare, rezultă ord(H2) = p2. Analog, obŃinem ord(H1) = p2, deci card(H1 ∪
∪ H2) = 2p2 – 1. Ca urmare, mulŃimea K = G – (H1 ∪ H2) conŃine un singur element. Dacă b ∈ H1 – {e} şi c ∈
∈ H2 – {e}, atunci bc ∉ H1 ∪ H2, deci bc ∈ K. Ca urmare, card(H1 – {e}) ⋅ card(H2 – {e}) ≤ cardK = 1, de unde
rezultă că mulŃimile H1 – {e} şi H2 – {e} conŃin cel mult un element, adică ord(H1) ≤ 2 şi ord(H2) ≤ 2. ObŃinem
p2 ≤ 2, contradicŃie.
SoluŃie alternativă. Din teorema lui Cauchy, există un element a ∈ G de ordin p. MulŃimea <a> = {e, a, a2, ...,
ap–1} are cardinalul p, deci există k ∈ 1, p − 1 astfel încât ak ∈ H1 ∪ H2. Să presupunem că ak ∈ H1. Atunci {e, ak,

360
a2k, ..., a(p–1)k} ⊂ H1, şi cum (k, p) = 1, rezultă <a> ⊂ H1. Fie x ∈ G arbitrar, mulŃimea {x, ax, ..., ap–1x} are p
elemente, deci există i, j ∈ 0, p − 1 , i < j, astfel încât {aix, ajx} ⊂ H1 ∪ H2. Dacă niciunul dintre numerele aix şi
ajx nu e în H1, atunci aix ∈ H2, de unde rezultă că aj–i ∈ H2 şi cum (j – i, p) = 1, rezultă <a> ⊂ H2, deci card(H1 ∩
∩ H2) ≥ card<a> = p, contradicŃie cu H1 ∩ H2 = {e}. Ca urmare, aix ∈ H1 sau ajx ∈ H1 şi cum ai, aj ∈ H1, rezultă
x ∈ H1. Aşadar G ⊂ H1, deci G = H1; 4. (c) Dacă f(x) = 1, pentru orice x ∈ R, atunci F(x) = x + α, α ∈ R, deci
lim (F(n ) − n ) = α ; (⇒) Fie T ∈ R – Q, T > 0 o perioadă a funcŃiei f. Pentru orice x ∈ R avem: (F(x+ T) –
n →∞
T
– F(x))' = f(x + T) – f(x) = 0, deci F(x + T) – F(x) = c, unde c = F(T) – F(0) = ∫ f (t )dt . FuncŃia G: R → R,
0
c c 
G(x) = F(x) – x este derivabilă pe R şi periodică de perioadă T. Întrucât F(n) – n = G(n) +  − 1n , (∀) n ≥
T T 
≥ 1, şi G este mărginită, în ipoteza c ≠ T ar rezulta lim (F(n ) − n ) ∈ {–∞, +∞}, contradicŃie. Prin urmare, cu
n →∞
necesitate avem c = T, deci G(x) = F(x) – x. Atunci şirul (G(n))n≥1 este convergent. Fie λ = lim G (n ) . Fie x ∈
n →∞
∈ R. Deoarece mulŃimea A = {k – hT | k, h ∈ N} este densă în R, există un şir crescător (an)n≥1 ⊂ A – {x}, an =
= kn – hnT, astfel încât lim a n = x şi (kn)n≥1 este strict crescător. Cum G(kn) = G(an) şi G este continuă, rezultă că
n →∞
λ = lim G (k n ) = lim G (a n ) = G ( x ) , deci G este constantă. Rezultă f(x) = F'(x) = (G(x) + x)' = 1, pentru orice
n →∞ n →∞
x ∈ R.

Concursul interjudeŃean de matematică „Ion Ciolac”, 8 mai 2010

CLASA A IX-
IX-A

1. Aplicăm inegalitatea mediilor pentru numerele pozitive xi – 1 şi xj – 1, unde 1 ≤ i < j ≤ 2010:


xi + x j 1 2010 ⋅ 2009
( x i − 1)( x j − 1) ≤
2
− 1 . Rezultă că ∑
1≤ i < j≤ 2010
(x i − 1)(x j − 1) ≤ ∑
2 1≤ i < j≤ 2010
(x i + x j ) −
2
, adică

2009 ⋅ ( x1 + x 2 + ... + x 2010 ) 2010 ⋅ 2009


∑ ( x i − 1)( x j − 1) ≤
1≤ i < j ≤ 2010
2

2
. În condiŃiile problemei, membrul drept este

2009 ⋅ 4020 2010 ⋅ 2009


egal cu: − = 2009 ⋅ 1005 şi este egal cu membrul stâng. Având egalitate în inegalitatea
2 2
mediilor, rezultă x1 – 1 = x2 – 1 = ... = x2010 – 1, deci x1 = x2 = ... = x2010 şi din x1 + x2 + ... + x2010 = 4020,
obŃinem x1 = x2 = ... = x2010 = 2; 2. Notăm cu ma, mg şi mh respectiv media aritmetică, media geometrică şi media
armonică a numerelor a, b, c. Trebuie să demonstrăm că 3ma + mh ≥ 4mg. Aplicând inegalitatea mediilor pentru
numerele ma, ma, ma, mh, obŃinem 3m a + m h ≥ 44 m 3a m h . Cum ma ≥ mg, avem 44 m 3a m h ≥ 44 m g m a2 m h , deci

este suficient să demonstrăm că 44 m g m a2 m h ≥ m g . Această inegalitate se scrie echivalent: m a2 m h ≥ m 3g ⇔


2
a+b+c 3abc
⇔  ⋅ ≥ abc ⇔ (a – b)2 + (b – c)2 + (c – a)2 ≥ 0, ceea ce este evident; 3. Dreapta BE
 3  ab + bc + ca
intersectează AC în S. [PE] este linie mijlocie în tringhiul ABS deci BE = ES. În triunghiul ABS, AE este
bisectoare şi mediană, deci AE ⊥ BE. Analog, CF ⊥ AF. Patrulaterul ABDE este inscriptibil, deci 'DEF ≡ 'B ≡

361
A
≡ 'MNP, (1). Patrulaterul ADFC este inscriptibil, deci 'DFE ≡ 'C ≡ 'MPN, (2). EF = AF – AE = b cos −
2
A A A B−C B+ C A A
− c cos = (b − c) cos ; EF = 2R(sin B – sin C) cos = 4R sin cos cos = 4Rsin 30° sin ⋅
2 2 2 2 2 2 2
A a
⋅ cos = 3 sin A = = PN . łinând cont de relaŃiile (1) şi (2), rezultă concluzia.
2 2

CLASA A X-
X-A

1. Notăm cu O punctul de afix 0 şi cu A, B, C, D punctele având afixele z1, z2, z3, z4 în ordinea crescătoare a
argumentelor reduse, iar M şi N sunt punctele de afixe z1 + z2, respectiv z3 + z4. Din ipoteză, z1 + z2 = –(z3 + z4).
Atunci O, M, N sunt coliniare. Deoarece OA = OB = OC = OD = r, paralelogramele AOBM şi CODN sunt
romburi, de unde deducem OM ⊥ AB şi ON ⊥ CD, deci AB || CD. Analog, AD || BC, deci ABCD este
paralelogram. Fiind inscriptibil, rezultă că ABCD este dreptunghi. atunci z1 = –z3 şi z2 = –z4. Finalizare: suma
2
 n 2  n 2  n 
este zero; 2. Aplicăm inegalitatea Cauchy-Buniakowski-Schwartz, 

ak  ⋅
 
bk  ≥ 
  ∑ ∑
a k b k  , pentru p =
 ∑
 k =1   k =1   k =1 
= n + 1, ak = 1 şi b k = C k2 n−1+1 , (∀) k ∈ {1, 2, 3, .., p}. ObŃinem (n + 1) ⋅ S ≥ (C02 n +1 + C12 n +1 + ... + C n2 n +1 ) , unde
2
1 1 
S = (C02 n +1 ) 2 + (C12 n +1 ) 2 + (C 22 n +1 ) 2 + ... + (C n2 n +1 ) 2 . Rezultă S ≥ ⋅ (2C 02 n +1 + 2C12 n +1 + ... + 2C 2n n +1  .
n + 1  2 
Dar, din formula combinărilor complementare, 2C 2k n +1 = C k2 n +1 + C 22 nn ++11− k . Înlocuind în inegalitatea de mai sus,
2
 2 n +1 k  2 n +1

∑ ∑
1 1
obŃinem S ≥ ⋅ ⋅ C 2 n +1  . Deoarece C 2k n +1 = 2 2 n +1 , rezultă inegalitatea cerută; 3. Avem condiŃia
n + 1 22  
 k =0  k =0
de existenŃă a logaritmului, x + y > 0. Notăm t = log3 (x + y), de unde x + y = 3t, (1). Logaritmând în baza 3 în
(3x − 2 y) ⋅ t = 4 (2)
prima ecuaŃie, deducem (3x – 2y) ⋅ log3 (x + y) = 4, iar cu notaŃia făcută sistemul devine:  2 .
t = x − y (3)
t 2 + 3t 3t − t 2
Din (1) şi (3) obŃinem 2x = t2 + 3t, de unde x = . Înlocuind în (1), deducem y = . Înlocuind x şi y
2 2
în (2) obŃinem (3 + 5 ⋅ t ) ⋅ t = 8. FuncŃia f: R → R, f(t) = (3 + 5 ⋅ t ) ⋅ t este strict crescătoare pe (0, +∞). Cum
t 2 t 2

f(1) = 8, rezultă că pe intervalul (0, +∞) avem soluŃia unică t = 1. Cum pentru t ≤ 0 avem f(t) ≤ 0, deducem că
singura soluŃie este t = 1. Finalizare: x = 2 şi y = 1.

CLASA A XI-
XI-A

1. det(X + Y) + det(X – Y) = 2(detX + detY), (∀) X, Y ∈ M2(C), de unde rezultă că det(AB + BA) + det(AB –
– AB) = 2(det(AB) + det(BA)) = 4detA ⋅ detB. Dar (AB – BA)2 – tr(AB – BA) ⋅ (AB – BA) + det(AB – BA) ⋅
⋅ I2 = O2; tr(AB – BA) = tr(AB) – tr(BA) = 0, astfel obŃinem relaŃia de demonstrat (AB – BA)2 = (det(AB +
(1 + x n +1 ) n +1 1 2
+ BA) – 4detA ⋅ det B) ⋅ I2; 2. RelaŃia din enunŃ se scrie + n +1
≤ , (1). Cum xn >
(1 + nx n ) 2
(1 + x n +1 ) 1 + nx n

362
(1 − x n +1 ) n +1 1 (1 + x n +1 ) n +1 1 2
> 0, avem + n +1
≥ 2 ⋅ ⋅ n +1
= , (2). Din (1) şi (2)
(1 − nx n ) 2
(1 + x n +1 ) (1 + nx n ) 2
(1 + x n +1 ) 1 + nx n
(1 − x n +1 ) n +1 1 2
deducem că + = , deci având egalitate în inegalitatea mediilor,
(1 − nx n ) 2 (1 + x n +1 ) n +1 1 + nx n
n +1
(1 − x n +1 ) n +1
∑C
1
= ⇔ (1 + x n +1 ) n +1 = 1 + nx n . Atunci 1 + nxn = k k
n +1 x n +1 ≥ 1 + (n + 1) x n +1 > 1 +
(1 − nx n ) 2
(1 + x n +1 ) n +1 k =0
+ nxn+1 ⇒ xn > xn+1. Astfel şirul (xn)n∈N* este strict descrescător şi 0 < xn ≤ x1, (∀) n ∈ N*, deci este convergent;
F( x ) f ( x )( x − 1) − F( x )
3. Fie g: (1, ∞) → R, g( x ) = ; g este derivabilă şi g' ( x ) = ≥ 0 , (∀) x > 1, de unde g este
x −1 ( x − 1) 2
F( x )
monotonă pe (1, ∞), deci există lim = L ∈ R . Pentru n ∈ N*, cum f are proprietatea lui Darboux,
x →1 x − 1
x >1

 1    1 
f  1, +1  şi f  1, + 1  sunt intervale ce diferă în cel mult un punct, deci f(1) este punct interior sau capăt
 n    n 
 1   1 1
al intervalului f  1, +1  , pe care îl notăm cu J. Atunci pentru oricare n ∈ N rezultă că  f (1) − , f (1) +  ∩
 n   n n 
 1 1 
∩ J ≠ ∅, prin urmare există yn ∈  f (1) − , f (1) +  ∩ J şi cum f are proprietatea lui Darboux, există an ∈
 n n
 1
∈ 1,1 +  astfel încât f(an) = yn, an → 1, an > 1 şi yn → f(1) = 0. Din enunŃ, (x – 1)f(x) ≥ F(x) ≥ 0, (∀) x ∈
 n
F( x ) F(a n )
∈ [1, ∞) ⇒ F(1) = 0 şi 0 ≤ ≤ f ( x ) , (∀) x > 1. Din 0 ≤ ≤ f (a n ) = y n → 0 , avem L = 0 ⇔
x −1 an −1
F( x ) − F(1)
⇔ lim = F'(1) = 0 = f(1) ⇒ F' = f.
x →1 x −1
x >1

CLASA A XII-
XII-A

ˆ 2,
1. Scăzând ecuaŃiile, obŃinem (â − 3̂)( x − y) = 0̂ . Dacă aˆ ∈ {0, ˆ 4,6}
ˆ ˆ ⇔ aˆ − 3ˆ ∈ U(Z8), rezultă x = y. Înlocuind în

sistem, obŃinem (â + 3̂) x = 2̂ . Cum â + 3̂ este inversabil, obŃinem x = y = 2̂(â + 3̂) −1 = 2̂(â + 3̂) . Dacă â = 1̂ ,
x + 3̂y = 2̂
sistemul se scrie  . ÎnmulŃind prima ecuaŃie cu 3̂ , obŃinem 3̂x + y = 6̂ , deci sistemul nu are soluŃie.
3̂x + y = 2̂
Dacă â = 3̂ , sistemul se reduce la 3̂( x + y) = 2̂ ⇔ x + y = 6̂. S = {( k̂, 6̂ − k̂ ) | k̂ ∈ Z8}. Dacă â = 5̂ , sistemul se scrie
5̂x + 3̂y = 2̂
 . Adunând ecuaŃiile, obŃinem 0̂ = 4̂ , deci sistemul nu are soluŃie. Dacă â = 7̂ , sistemul se scrie
3̂x + 5̂y = 2̂
7̂ x + 3̂y = 2̂
 . Cele două ecuaŃii sunt echivalente (înmulŃind-o pe a doua cu 5̂ , se obŃine prima), deci sistemul se
3̂x + 7̂ y = 2̂

363
reduce la 7̂ x + 3̂y = 2̂ ⇔ 7̂(7̂ x + 3̂y) = 2̂ ⇔ x = 6̂ + 3̂y . Rezultă S = { (6̂ + 3̂k̂ ), | k̂ ∈ Z8}; 2. Cu schimbarea de
b a b
variabilă a + b – x = y avem ∫ f (a + b − x + t )dx = −∫ f (y + t)dy = ∫ f (x + t)dx , deci relaŃia din enunŃ devine
a b a
b b
∫ f (x + t)dx ≥ ∫ f (x)dx , (1). FuncŃia f este continuă deci are primitive. Fie F o primitivă. RelaŃia (1) devine:
a a
F(b + t) – F(a + t) ≥ F(b) – F(a), (∀) t ∈ R, (2). Fie v: R → R, v(t) = F(b + t) – F(a + t). RelaŃia (2) se scrie v(t) ≥
≥ v(0), (∀) t ∈ R, deci 0 este punct de minim pentru funcŃia v. FuncŃia v este derivabilă şi v'(t) =
= f(b + t) – f(a + t). Din teorema lui Fermat rezultă v'(0) = 0, adică f(a) = f(b); 3. Inegalitatea de demonstrat este
4 4
3 2 2 4 2 4
2 1 1 ∫
echivalentă cu: f (1) ⋅ xdx + f ( x )dx ≥ ∫ ∫1
3
(2 − x ) 3
f ( x )dx ⇔ ∫ (x ⋅ f (1) + 1 ⋅ f (x))dx ≥ ∫
1 1
3
(2 − x ) 3
f ( x )dx , (1).

x ⋅ f (1) + 1 ⋅ f ( x )  x ⋅1 + 1⋅ x 
Cum f este concavă, avem pentru orice x: ≥ f  , deci:
x +1  x +1 
2 2
 2x 
∫ ∫
2x
(xf (1) + f (x))dx ≥ (x + 1)f   dx , (2). Cu schimbarea de variabilă = t , avem:
1 1  x +1 x +1
4
2  2x  4
∫1
( x + 1)f  dx =
 x +1 ∫ 1
3
(2 − t ) 3
f ( t )dt . Înlocuind în relaŃia (2), obŃinem relaŃia (1) care este echivalentă cu

concluzia.

Concursul interjudeŃean de matematică „Argument”, Baia Mare, EdiŃia I


CLASA A IX-
IX-A

1. a) P(1) + P(3) = P(2) + P(4) = P. Deci P(1) sau P(3) este mai mic sau egal cu D C' C
S(1) AA' AD' S(3) 4 3
P. Analog pentru P(2), P(4); b) Dacă = ⋅ = x ⋅ y , atunci = D'
B'
S AB AD S M
1 2
2 2
S(1) ⋅ S(3)  x +1− x   y +1− y 
= (1 – x)(1 – y). Deci 2
= x(1 − x ) ⋅ y ⋅ (1 − y) ≤    = A A' B
S  2  2 
1 S(1) 1 S(3) 1 y
= . Deci ≤ sau ≤ . Analog pentru S(2), S(4); 2. Rezultă y2 +
16 S 4 S 4
+ y(x2 + x) + x3 – x2 – 1 = 0. Iar ∆ = = x4 – 2x3 + 5x2 + 4. Cum (x2 – x + 2)2 < D C E
* * *
< ∆ < (x2 – x + 3)2, rezultă că y este iraŃional; *
b
*B
3. Cum AB = DC ⇒ 2(1 – a) = 2(1 – b). Deci a = b. Apoi EB = b – (1 – a) = x
* O a*
= 2a – 1, iar CB = (2a – 1) 2 . Rezultă că 2(1 – a) = (2a − 1) 2 . ObŃinem A
a
* *
2 x –b x
a=b= .
2

CLASA A X-
X-A

1. Luând y = –f(x) rezultă f(f(–f(x)) – 2x) = –3x + f(0). Deci f este surjectivă. Fie x0 cu f(x0) = 0. Pentru orice t ∈
∈ R există y ∈ R cu f(y) = 2x0 + t. Rezultă f(t) = f(f(y) – 2x0) = f(f(x0) + y) – 3x0 = f(y) – 2x0 – x0 = t – x0. Prin

364
x n +1 + x n −1
urmare f: R → R, f(x) = x + a, a ∈ R; 2. a) Din x n2+1 − x n + 2 x n = x n2 − x n +1 x n −1 , n ≥ 1, rezultă: =
xn
xn + xn+2  x n +1 + x n −1  x + x n −1
= , pentru orice n ≥ 1. Prin urmare şirul   este constant; n +1 = α, α ∈ R implică
x n +1  x n  x n
x2 + x0
xn+1 = αxn – xn–1, n ≥ 1; b) Cum α = ∈ Z şi xn+1 = αxn – xn–1, n ≥ 1, prin inducŃie rezultă xn ∈ Z, pentru
x1
orice n ∈ N; 3. Folosind produsul scalar, CD2 = XD2 + XC2 – 2XD ⋅ XC şi analoagele. ObŃinem XA ⋅ XB = XB ⋅
⋅ XC = XC ⋅ XD = XD ⋅ XA . Rezultă că XA ⊥ BD, XB ⊥ CD, XC ⊥ BD, XD ⊥ AC şi AC ⊥ BD. Prin urmare,
dacă AC ⊥ BD, atunci locul geometric este AC ∩ BD, iar în caz contrar este mulŃimea vidă.

CLASA A XI-
XI-A
n +1 n +1 n n
1. Avem a n +1 + b n +1 = (a n + b n ) 2 = ... = (a 0 + b 0 ) 2 = C 2 , apoi a n +1 = a n ⋅ C 2 , b n +1 = b n ⋅ C 2 , n ∈ N.
Monotonia şi mărginirea se studiază uşor în funcŃie de C = a0 + b0; 2. a) Ridicând la pătrat relaŃia din enunŃ,
rezultă 2(z + 1)(z + 1) = (z + i)(z − i) + (z − i)(z + i) + 2 | z + i | ⋅ | z − i | , echivalent cu z + z =| z + i | ⋅ | z − i |=| z 2 + 1 | .
2
Rezultă Re z ≥ 0 şi (z + z) 2 = (z 2 + 1)(z + 1) . După calcule condiŃia dată este echivalentă cu Re z ≥ 0 şi
(z ⋅ z − 1) 2 = 0 sau încă |z| = 1, Re z ≥ 0; b) Alegem poziŃia pătratului astfel încât afixele vârfurilor A, B, C, D să
fie 1, i, –1 şi respectiv –i. Folosind a), locul geometric este cercul circumscris pătratului; 3. Demonstrăm mai
1 2 3 4 
întâi pentru n = 2p, prin inducŃie după p ≥ 2. Pentru p = 2 avem permutarea σ 2 =   . Dacă σ 2 p =
1 3 2 4 
 1 2 ... 2 p 
=  este permutarea pentru n = 2p, definim:
 σ1 σ 2 ... σ p 
 2 

 1 2 ... 2p 2 p + 1 2 p + 2 ... 2 p +1 
σ 2 p+1 =  . Acum pentru n între 2p–1 şi 2p, folosim
 2σ1 − 1 2σ 2 − 1 ... 2σ p − 1 2σ1 2 σ ... 2 σ 
 2 2 2 
p

 1 2 ... n 
permutarea σ 2 p şi formăm permutarea σ n =   în care x1, x2, ..., xn s-au reŃinut în aceeaşi ordine
 x1 x 2 ... x n 
din σ1 , σ 2 , ..., σ 2 p prin eliminarea numerelor mai mari ca n.

CLASA A XII-
XII-A
1  1  1
1. a) 2 & 2 = 4a – 2 ∈ [0, 2] ⇒ a ∈  ,1 . Reciproc, dacă a ∈  ,1 , atunci 0 ≤ xy − xy + 2 ≤ axy – x – y +
2  2  2
(b + c) x
+ 2 ≤ xy – x – y + 2 ≤ 2, oricare ar fi x, y ∈ [0, 2]; b) x & x = , pentru orice x ∈ (–1, 1). Cum, dacă b +
1− x2
( b + c) x b( x − y) x−y
+ c ≠ 0, lim = ±∞ , rezultă b + c = 0, deci x & y = . Cum − 1 < < 1 , pentru orice x, y ∈
x →1 1 − x 2
1 − xy 1 − xy
x <1

∈ (–1, 1), rezultă b ∈ [–1, 1]; c) Legea & are element neutru e = 2 pentru a = 1. Legea  nu are element neutru;

365
n +1
7  7 
2. Observăm că 4xn+1 + 5yn+1 = 4xn + 5yn = ... = 4x0 + 5y0 şi x n +1 − y n +1 = − ( x n − y n ) = ... =  −  (x 0 −
20  20 
n +1
 7  4x + 5y 0
– y0). Rezultă 9x n +1 = 4x 0 + 5 y 0 + 5 −  ( x 0 − y 0 ) , deci (xn) este convergent şi lim x n = 0 şi
 20  n → ∞ 9
4 x + 5x 0  π 
apoi lim y n = 0 ; 3. Avem: (A + iB)(A – iB) = A2 + B2 – i(AB – BA) =  ctg − i  ⋅ (AB – BA). Cum
n →∞ 9  k 
n
 π  nπ nπ
det(A + iB)(A – iB) ∈ R, rezultă  ctg − i  ∈ R şi de aici cos − i sin ∈ R, deci n ⋮ k. Spre exemplu,
 k  k k
A . . . .  B . . . .
    0 1 0 0
A 2010 =  . A . . .  , B 2010 =  . B . . .  , unde A =  , B =   şi k = 2.
 . . . . A . . .   0 0  1 0 
   . B

Concursul interjudeŃean de matematică „Petre Moroşan – TRIDENT”,


„Memorialul Mircea Ganga”, EdiŃia a VII-a, SecŃiunea A (M1), Brăila,
6-8 noiembrie 2009

CLASA A IX-
IX-A

1. Fie AD înălŃimea din A, BE bisectoarea 'B, iar CF mediana din C. Dreptele AD, BE, CF sunt concurente dacă
AE CD BF AE c CD b 2 BF
şi numai dacă ⋅ ⋅ = 1 . Cum = ; = ; = 1 , concluzia se impune; 2. Conform inegali-
EC BD FA EC a BD c 2 FA
1 1 1 9 9
tăŃii Cauchy, + + ≥ ≥ , deoarece a2 + b2 + c2 ≥ ab + ac + bc;
c + ab b + ac a + bc a + b + c + ab + ac + bc 2
3. Pentru x = zt, t ∈ R, condiŃia se scrie echivalent [zt] + [zt + yz] = [t], pentru orice t ∈ R. Pentru t = 0, obŃinem
1  1 1 1
[yz] = 0. Pentru t = , z ≠ 0 obŃinem z ∈  ,  , iar pentru t = 2, obŃinem z = . Folosind eventual identitatea
z  3 2  2
Hermite, rezultă că y = 1.

CLASA A X-
X-A

1
1. Notăm rX vectorul de poziŃie al punctului X în raport cu o origine O. Deducem: rM = ⋅ (rB +
4
1
+ rC + rD + rE ) şi analoagele pentru rN , rP , rQ . Prin calcul rM + rP − rN − rQ = (rB + rD − rA − rC ) . Atunci:
4
MNPQ este paralelogram dacă şi numai dacă rM + rP = rN + rQ ⇔ rB + rD = rA + rC ⇔ ABCD este paralelogram.
a 3 (a + b) a 2b2 a 2b 2 a 2 b2 ab a 3 (a + b)
2. Avem că 2
a + ab + b 2
= a 2
− 2
a + ab + b 2
, iar 2
a + ab + b 2

3
3 a 3 b3
=
3
. Deci ∑ a 2 + ab + b 2
≥ a2 +

366
ab + bc + ca 2
+ b 2 + c2 – ≥ (ab + bc + ca ) = 2 ; 3. Luând x = y = z = t = 0, rezultă f(0, 0) = 0. Luând z = t = 0,
3 3
rezultă f(x + y, 0) = f(x, 0) + f(y, 0), (∀) x, y ∈ Q. De aici obŃinem că f(x, 0) = f(1, 0) ⋅ x, (∀) x ∈ Q. Folosind a
doua condiŃie din ipoteză obŃinem că f(1, 0) ∈ {0, 1}. Analog rezultă că f(0, t) = f(0, 1) ⋅ t, (∀) t ∈ Q şi f2(0, 1) =
= f(1, 0) ∈ {0, 1}. Deci f(x, y) = ax + by, (∀) x, y ∈ Q, unde a ∈ {0, 1}, iar b ∈ {–1, 0, 1}. În final se obŃine că
f(x, y) = 0, f(x, y) = x + y sau f(x, y) = x – y, pentru orice x, y ∈ Q.

CLASA A XI
XI-A

1. a) Permutarea σ este impară iar x2a este pară. EcuaŃia nu are soluŃii; b) ε(σ2l+1) = [ε(τ1) ⋅ [ε(τ2) ⋅ ... ⋅ [ε(τ2l+1)]2,
absurd, pe acelaşi principiu ca mai sus; c) Numărul permutărilor impare din S4k+2, k ≥ 1 este par; 2. Dacă notăm
a 0 0
 
C = AB = BA, atunci AC = CA şi BC = CB. Prin calcul se obŃin soluŃiile: A =  0 a 0  şi B =
0 0 a 
 
 0 b 0  0 b 0 a 0 0
     
=  0 0 b  , unde ab = 2009; A =  0 0 b  şi B =  0 a 0  , unde ab = 2009 şi A = B =
 2009b 0 0   2009b 0 0  0 0 a 
     
 0 0 1
  1
=  2009 0 0  ; 3. Folosind identitatea x3 + y3 + z3 – 3xyz = (x + y + z)(x2 + y2 + z2 – xy – yz – xz) = ⋅
 0 2
 2009 0 
⋅ (x + y + z)[(x – y)2 + (y – z)2 + (z – x)2] şi a doua relaŃie din ipoteză rezultă că: (an + bn + 1)[(an – bn)2 + (an –
2 2
– 1)2 + (bn – 1)2] ≤ , n ≥ 1. Cum an + bn + 1 ≥ 1, (∀) n ≥ 1, obŃinem că (an – bn)2 + (an – 1)2 + (bn – 1)2 ≤ . De
n n
aici rezultă că lim a n = lim b n = 1 .
n →∞ n →∞

CLASA A XII-
XII-A

1. În (*) axa–1 = x3, (∀) x ∈ G, luăm x = a. Rezultă a2 = e. Rămâne de arătat că xa = ax, (∀) x ∈ G. Înlocuind pe
'
2x 1  1 
x cu ax în (*) rezultă x = axa, adică ax = xa; 2. f ( x ) = x ⋅ x 2
=− ⋅ x ⋅ x  . Se integrează prin
(2 + 1) ln 2  2 +1
1 1 x 1
părŃi: ∫ f (x)dx = − ln 2 ⋅ 2 x
+ −
+ 1 ln 2 ln 2 2
⋅ ln(2 x + 1) + C ; 3. a) Verificare imediată; b) Se obŃine că |ϕ(x) –

x − 1, x ≥ 1
– 1| = |x – 1| =  . Dacă (∃) x1, x2 ∈ (1, ∞), astfel încât ϕ(x1) = x1 > 1 şi ϕ(x2) = 1 – x2 < 1, rezultă că
1 − x , x < 1
(∃) c între x1 şi x2 astfel încât ϕ(c) = 1, absurd. Deci ϕ(x) = x, (∀) x ≥ 1 sau ϕ(x) = 2 – x, (∀) x > 1. Analog pe
x, x ≥ 1
(–∞, 1). În concluzie cele 4 funcŃii sunt ϕ1(x) = x; ϕ2(x) = 2 – x; (∀) x ∈ R; ϕ3 ( x ) =  ; ϕ4(x) =
2 − x , x < 1
2 − x , x ≥ 1
= , care sunt continue; c) În cazul f(x) = cos x obŃinem ϕ(x) = ±x + 2kπ, unde k = k(x) ∈ Z. Fixăm
x, x <1

367
un interval (nπ, (n+1)π), n ∈ Z. Fie mulŃimile A = {x ∈ (nπ, (n + 1)π) | ϕ(x) = x + 2k(x)π}; B = {x ∈ (nπ, (n +
+ 1)π) | ϕ(x) = –x + 2k(x)π}. Demonstrăm că A = ∅ sau B = ∅. Fie, prin absurd, că A ≠ ∅ şi B ≠ ∅. Există a <
< b, a ∈ A, b ∈ B. Fie α = sup{x ∈ A | x < b}. Dacă α = b, există un şir (xn): xn ∈ A, xn → α, ϕ(xn) = xn +
ϕ( x n ) − x n
+ 2k(xn)π, ϕ(α) = –α + 2k(α)π. Cum ϕ este continuă, ϕ(xn) → ϕ(α). Deoarece k ( x n ) = ∈ Z este

convergent, rezultă k(xn) = k = constant pentru n suficient de mare. Din lim ϕ( x n ) = ϕ(α) rezultă b + 2kπ =
n →∞
= –b + 2k(b)π, deci (k(b) – k)π = b ∈ (nπ, (n+1)π), fals. Dacă a > b, analog, schimbând A cu B. Dacă α < b,
analog, considerăm şirul yn → α, yn ∈ B. În concluzie ϕ(x) = x + k(x)π, pentru orice x ∈ (nπ, (n+1)π) sau ϕ(x) =
= –x + 2k(x)π, pentru orice x ∈ (nπ, (n+1)π). Cum k(x) ∈ Z este continuă pe intervalul (nπ, (n+1)π), rezultă că
este constantă, deci ϕ este derivabilă pe acest interval, rezultă derivabilă pe R – {kπ, k ∈ Z}. Spre exemplu
− x + 2nπ, x ∈ [(2n − 1)π,2nπ]
funcŃia φ(x) =  verifică cerinŃele problemei.
 x − 2nπ, x ∈ (2nπ,(2n + 1) π)

Concursul interjudeŃean de matematică „Dimitrie Pompeiu”, EdiŃia a X-a,


Botoşani, 14-16 mai 2010

CLASA A IX-
IX-A

BF 3
1. Aplicăm teorema lui Menelaus în ∆BCD şi obŃinem că = . Aplicăm teorema lui Ceva în ∆ABC şi
BC 5
AE 3 A 3 A ∆BDF 3 A ∆CEF 1 A ∆DEF 3
obŃinem că = . Determinăm rapoartele ∆ADE = , = , = şi atunci = ;
AC 8 A ∆ABC 28 A ∆ABC 7 A ∆ABC 4 A ∆ABC 14
2. Pentru m = 1, relaŃia de la b) devine f(2n + 1) = 2f(n) + 1, n ∈ N*, (1). Pentru n = 1, avem f(3) = 3. Pentru m =
= 3, relaŃia de la b) devine f(4n + 3) = f(n) + f(3n) + 3. Totodată, folosind (1), avem: f(4n + 3) = 2f(2n + 1) + 1 =
= 4f(n) + 3. Din cele două, obŃinem: f(3n) = 3f(n), n ∈ N*, (2). Folosind relaŃiile (1) şi (2) avem: f(6n + 3) =
= 3f(2n + 1) = 6f(n) + 3. Pe de altă parte, folosind (1), avem: f(6n + 3) = 2f(3n + 1) + 1. În consecinŃă, f(3n +
+ 1) = 3f(n) + 1, (3). Din (3) avem că f(6n + 1) = 3f(2n) + 1, iar din (1) şi (2) avem că f(6n + 1) = 2f(3n) + 1 =
= 6f(n) + 1, deci f(2n) = 2f(n), n ∈ N*, (4). Arătăm că f(n) = n. Presupunem afirmaŃia adevărată pentru toŃi n ≤ m
(m ≥ 1) fixat) şi o demonstrăm pentru m + 1. Dacă m + 1 = 2k, folosim (4) şi ipoteza şi obŃinem f(m + 1) =
= f(2k) = 2f(k) = 2k = m + 1. Dacă m = 2k + 1, avem, folosind (1) şi ipoteza: f(m + 1) = f(2k + 1) = 2f(k) + 1 =
2 2
 x   x 
= 2k + 1 = m + 1. Deci f(n) = n, n ∈ N*; 3. a) Direct. Rescriem ecuaŃia sub forma:  a − 0  +  b − 0  +
 2   2 
x 02 1
+ = , de unde deducem că x0 ∈ {–1, 0, 1}. Analizăm cazurile x0 = –1, x0 = 0, x0 = 1 şi obŃinem de fiecare
2 2
1 1
dată că a 2 + b 2 = . Reciproc, dacă a 2 + b 2 = , atunci x0 = 0 este rădăcină întreagă; b) Dacă x1 şi x2 sunt
2 2
2 2 1
rădăcini întregi, atunci a + b = şi ecuaŃia devine x2 – x(a + b) = 0, deci una dintre rădăcini va fi x1 = 0. Se
2
analizează situaŃiile x2 = –1, x2 = 0, x2 = 1.
Problema suplimentară
Se consideră punctul D simetricul punctului C' faŃă de punctul B' şi se renotează A = E şi A' = F.

368
CLASA A X-
X-A

1. Pentru centrul I al cercului înscris în ∆ABC are loc formula 2p ⋅ OI = a ⋅ OA + b ⋅ OB + c ⋅ OC , oricare ar fi punctul
log a +1 (a 2 + 2a )
O. 2. a) 2x > y dacă şi numai dacă loga+1(a + 2) ⋅ loga+1a < 1. Dar log a +1 (a + 2) ⋅ log a +1 a < <
2
log a +1 (a + 1) 2
< = 1 şi concluzia e imediată; b) A1 > B1 este echivalentă cu 2x + x + y > y + 3x , prin
2
ridicare la pătrat şi reducerea termenilor asemenea, 2x ( x + y) > 3xy , care este echivalentă cu 2x > y,
adevărată. Se demonstrează prin inducŃie că An > Bn, (∀) n ∈ N*; 3. Considerăm mulŃimea nevidă A = {n ∈ N |
| f(0) < f(n)}. Să notăm cu b cel mai mic număr din mulŃimea A. Deci f(0) < f(b). Fie t = 2010f(b) – f(0). Cum f
este bijectivă, există c ∈ N, astfel încât f(c) = t. Prin urmare f(0) + f(c) = 2010f(b). Cum f(c) = 2009f(b) + f(b) –
– f(0) > 2009f(b) > f(b) > f(0) rezultă că c ∈ A. Din minimalitatea lui b obŃinem că b < c. Deci numerele 0 < b <
< c satisfac cerinŃa din enunŃul problemei.
Problema suplimentară
2S 2S 2S
Presupunem cunoscută aria S (> 0) a triunghiului căutat. Va avea laturi a = , b= ,c = . Se impun
d e f
1 1 1
condiŃii şi se simplifică S. De exemplu, + > , f(d + e) > de, etc.
d e f

CLASA A XI-
XI-A

1. Utilizăm inegalitatea: rang(AB) ≤ min(rangA, rangB); a) rang(AB) ≤ rangA ≤ 3 şi cum AB ∈ M4(R), rezultă
că det(AB) = 0; b) rangA = rang(A(BB–1)) = rang(AB)B–1) ≤ rang(AB) ≤ rangA; c) (A – bIn)(B – aIn) = abIn
implică det(B – aIn) ≠ 0; A(B – aIn) = bB şi conform b), rezultă că rangA = rang(bB) şi cum b ≠ 0, rezultă că
π π
rang(bB) = rangB; 2. a) Se observă că x 0 = cos şi x1 = cos . Prin inducŃie matematică, se arată că
2 4
π  π 
x n = cos n +1 . Deoarece lim x n = 1 , rezultă că şirul (xn)n∈N este convergent; b) lim 4 n 1 − cos n +1  =
2 n →∞ n →∞  2 
2 π
2 sin n + 2 2
n 2 π 2  π  π2 −2 x
= lim 4 ⋅ 2 sin n + 2 = lim ⋅  n + 2  ⋅ 4n = ; 3. f'(x) = ≤ 0 , (∀) x ∈ R; f''(x) =
n →∞ 2 n → ∞  π  2  8 (1 + x 2 ) 2
 n+2 
2 
− 4x (1 − x 2 )
= . Constatăm că f''(x) < 0 pe [–2π, –π] iar f este de două ori derivabilă şi se obŃine că funcŃia f este
(1 + x 2 ) 2
strict concavă pe [–2π, –π]. Oricare ar fi c ∈ [–2π, –π], există f'(c) ∈ R. Urmează existenŃa a două puncte
A(a, f(a)) şi B(b, f(b)), pentru care panta dreptei AB să fie egală cu f'(c). Concluzia este că (∀) c ∈ (–2π, –π),
f ( b ) − f (a )
(∃) a, b ∈ R astfel încât = f ' (c ) .
b−a
Problema suplimentară
Luăm BA' = A'C = 1 şi AA' = 10. Atunci BB' < BC = 2 şi BB' + CC' < 4 < 10.

369
Concursul interjudeŃean de matematică „Vasile Dumitrache”

CLASA A IX-
IX-A

1. Fie k 2 − pk = a ∈ N. Atunci k2 – pk – a2 = 0. Cum k ∈ Z rezultă ∆ = p2 + 4a2 = b2. Din (b – 2a)(b + 2a) = p2,
b − 2a = 1  b − 2a = p  p +1
2
 p −1
2
p prim, obŃinem  sau  . ObŃinem k =   , k = −  , p impar; k = 0 sau k = p;
b + 2a = p  b + 2a = p
2
 2   2 
BA1 CB1 AC1 BA 2 k + 1
2. Notăm = = = k . Din teorema lui Menelaus în ∆ABB1, găsim = 2 , de unde pentru
A1C B1A C1B A 2 B1 k
k2 k +1 1 k
orice punct O din plan, OA 2 = OB + 2 OB1 , cum OB1 = OC + OA , obŃinem:
2
k + k +1 k + k +1 1+ k k +1
1
OA 2 = 2
(k 2 OB + k OA + OC) şi analoage pentru OB2 , OC 2 . Se verifică că OA 2 + OB2 + OC 2 =
k + k +1
= OA + OB + OC şi de aici concluzia problemei. Din AA 2 + BB2 + CC 2 = 0 , rezultă şi partea a doua a
problemei; 3. Fie [x] = k ∈ Z, respectiv {x} = α ∈ [0, 1). EcuaŃia devine (k + α)n = αn – kn. Pentru k ≥ 0, 1 >
> αn = (k + α)n + kn ≥ 2kn, de unde k = 0. ObŃinem soluŃia x ∈ [0, 1). Pentru k < 0, evident k + α < 0. Dacă n este
par, αn – kn < 0 < (k + α)n, nu avem soluŃii. Dacă n este impar, (k + α)n < 0 < αn – kn, nu sunt soluŃii.

CLASA A X-
X-A

∑z
2 2
+1 −6
1. EcuaŃia se scrie 2 x (2 y − 19) = 2 ⋅ 3 ⋅ 5 ⋅ 67. ObŃinem x = 0, y = 4, z = 2 şi k
− 1 = 30 ;
k =x
2. Considerăm planul complex raportat la originea O, centrul cercului circumscris. Fie a, b, c, d afixele punctelor
A, B, C, D. Prin urmare, |a| = |b| = |c| = |d| = R şi (a – b)2 + (b – c)2 + (c – d)2 + (d – a)2 = 8R2. Folosind relaŃia
z ⋅ z =| z |2 , condiŃia din enunŃ devine: ab + a b + bc + bc + cd + cd + da + da = 0 sau încă (a + c)(b + d) + (a + c) ⋅
a+c
⋅ (b + d) = 0, deci a + c = 0 (adică (AC) este diametru), b + d = 0 (adică (BD) este diametru) sau  +

b+d
a+c a+c x 3x
+ = 0, de unde ∈ iR* (deci AC ⊥ BD); 3. EcuaŃia este echivalentă cu (m − 1) sin sin = 0, x ∈
b+d b+d 2 2
2π 4π
∈ [0, 2π). Cum m ≠ 1, obŃinem soluŃiile x1 = 0, x 2 = , x3 = . Concluzia problemei se impune.
3 3

CLASA A XI-
XI-A

1. Polinomul P(X) = det(A + XB) – det(AX + B) are gradul mai mic sau egal cu k şi are k + 1 rădăcini. Rezultă
f (x )
că P este polinomul nul. Deci P(0) = 0 şi atunci detA = detB; 2. Fie g: R → R, g( x ) = 2 . Din mărginirea
x +1
funcŃiei f, lim f ( x ) = lim f ( x ) = 0 . Considerăm funcŃia h: R → R, h(x) = g(x + 1) – g(x). Dacă h nu se
x→∞ x → −∞

370
anulează, atunci h are semn constant. Fie h > 0. Atunci g(x + 1) > g(x) pentru orice x ∈ R. Rezultă g(n) > g(1) >
> g(0) > g(–n), pentru orice n ∈ N, n ≥ 2. Cum lim g( x ) = 0 , deducem 0 ≥ g(1) ≥ g(0) ≥ 0, imposibil. Analog,
x → ±∞
h < 0. Prin urmare există b ∈ R cu h(b) = 0, deci g(b + 1) = g(b). Notăm b + 1 = a şi concluzia se impune;
1 + k[a k ] 
3. Avem kak < 1 + [kak] ≤ 1 + kak ≤ 2kak, de unde [ak] ≤   ≤ [2a k ] , k ∈ N*. Sumând aceste inegalităŃi,
 k 
n n
 1  1 x − yn 1 n(2a n + 1)
∑ ∑  a
n 1
rezultă: 0 < xn – yn < a k + 2  ≤ k +  ≤ na n + = n (2a n + 1) . Rezultă 0 ≤ n ≤ ⋅
k =1   k =1
2 2 2 zn 2 zn
şi limita cerută este egală cu zero.

Concursul interjudeŃean de matematică „LaurenŃiu Panaitopol”, Tulcea,


15 mai 2010

CLASA A IX-
IX-A

1. Dacă x1, x2 ∈ Z sunt soluŃiile, din 2m = x1 + x2, 2m2 = x1x2 + 25 rezultă m ∈ Z. Cum x ∈ R, ∆ = 4(25 – m2) ≥
≥ 0. Rezultă m ∈ {–5, –4, –3, –2, –1, 0, 1, 2, 3, 4, 5}. Cum rădăcinile trebuie să fie întregi, ∆ este pătrat perfect.
Convin numai m ∈ {–5, –4, –3, 0, 3, 4, 5}; 2. f(4) = 2 + f(2) = 3 + f(1). Cum f(1) = a2, f(4) = b2, a < b rezultă
(b – a)(b + a) = 3, deci a = 1 şi b = 2 şi atunci f(1) = 1, f(4) = 4. Inductiv, f(2n) = 2n, n ∈ N. Din stricta monotonie
rezultă f(x) = x, x ∈ N; 3. Fie F, G, H centrele celor trei cercuri. Arătăm că FG = ρ' (deci triunghiul BFG are
1
laturile de lungimi BF = R, BG = ρ, FG = ρ', iar 'BFG = 'BFC = 'BAC = constant) cel puŃin unul din
2
unghiurile B sau C este ascuŃit. Fie acesta 'B. Considerăm F', G' proiecŃiile punctelor F, G pe AB. Deducem că
1 1
F' este mijlocul lui (AB) şi G' este mijlocul lui BE, FG = 
F' G ' = F' G ' şi F'G' = BF' – BG' =
cos(90 − B̂) sin B̂
1 1 1 m 1
= AB − AE = ρ'sin('ADE) = ρ'sin'B), deci FG = ρ'; 4. AP = AC + AD , AN = AC +
2 2 m +1 m +1 k +1
k 1 p 1 l
+ AB , CQ = AC + CD şi CM = CA + CB . Adunând aceste relaŃii şi Ńinând cont că
k +1 p +1 p +1 l +1 l +1
 1 1   1 1 
AD = BC , CD = BA , obŃinem AP + AN + CQ + CM =  − AB +  − BC = 0 , rezultă m =
 m +1 l +1  k +1 p +1
= l şi k = p, adică patrulaterele ANCQ, AMCP şi QMNP sunt paralelograme, deci diagonalele lor au mijlocul
comun.

CLASA A X-
X-A

 1 π
1. În mod necesar x ∈ [–1, 1]. Pentru x ∈ − 1, −  , obŃinem arcsin x = –2 < − , imposibil. Pentru x ∈
 2 2
 1  π 1  1
∈ − , 0  , obŃinem x = –sin 1, nu convine, deoarece sin 1 > sin = . Pentru x ∈ 0,  , obŃinem x = 0.
 2  6 2  2

371
1 
Pentru x ∈  ,1 , obŃinem x = sin 1, soluŃie. În sfârşit x = 1 nu este soluŃie; 2. Din n ∉ N rezultă f injectivă,
2 
iar din [a + b n ] = impar, rezultă că f nu este surjectivă; 3. Din ipoteză Re(z) > Im(z), deci z = r(cos t + isin t) cu
 3π π   3π π   3π π 
r > 0, t ∈  − ,  , apoi Re(z2) > Im(z2) şi 2t ∈  − ,  implică 2t ∈  − ,  . Inductiv rezultă că 2n ∈
 4 4   2 2   4 4
 3π π 
∈ −
 4 4
,  pentru orice n ∈ N, deci t = 0; 4. 2
1≤ i < j ≤ n

cos(a i − a j ) = 2 ∑(cos a i cos a j + sin a i sin a j ) =
1≤ i < j≤ n
2 2
 n   n  n
=
 ∑   ∑
cos a i  +  sin a i  −
 ∑ (cos 2
a i + sin 2 a i ) ≥ −n .
 i =1   i =1  i =1

CLASA A XI-
XI-A

1. Deoarece (an) este convergent, există M > 0 astfel încât |xn| ≤ M, (∀) n ∈ N. Fie acum ε > 0. Există nε ∈ N
3
astfel încât |an – 6| < ε pentru orice n ≥ nε. Deducem că pentru n ≥ nε şi A = M + 3 M + ... + 3 M ( nε radicali),
3
6 − ε + ... + 3 6 − ε − A < x n < 3 6 + ε + ... + 3 6 + ε + A , unde numărul radicalilor în fiecare membru este n – nε.
Se arată uşor că şirurile din primul şi ultimul membru al inegalităŃii precedente sunt convergente şi au limitele
soluŃiile αε, β ε ale ecuaŃiilor x3 = x + 6 – ε, respectiv x3 = x + 6 + ε. Rezultă αε > 2 – ε, βε < 2 + ε pentru ε
suficient de mic, obŃinem 2 – 2ε < xn < 2 + 2ε, pentru n suficient de mare, concluzia se impune; 2. Din prima
relaŃie rezultă AB + BA = On, deci AB = –BA. Din (A + B)4 = ((A + B)2)2 = A4 + A2B2 + B2A2 + B4 = A4 + B4,
deducem B2A2 + A2B2 = On. Dar B2A2 = BBAA = –BABA = –ABAB = –(AB)2 şi A2B2 = AABB = –ABAB =
= –(AB)2, deci –2(AB)2 = On, de unde concluzia problemei; 3. a) Dacă există cx ≠ dx astfel încât f'(cx) = f'(dx),
atunci există c între cx şi dx astfel încât f''(c) = 0, fals; b) Deoarece derivata lui f' nu se anulează restricŃia lui f' la
codomeniu este inversabilă, iar inversa g este derivabilă. Din cx = g(f(x + 1) – f(x)) rezultă că h: R → R, h(x) =
= cx este derivabilă. Prin derivare f'(x + 1) – f'(x) = f''(cx) ⋅ cx', rezultă cx' = 1, (∀) x ∈ R. Deci cx = x + k, k
constantă reală. Prin urmare f(x + 1) – f(x) = f'(x + k), (∀) x ∈ R, prin inducŃie rezultă relaŃia cerută; 4. Dacă d =
= detA > 0, cum AA* = dIn, rezultă A2 + dIn = 2In, deci A2 = (2 – d)In şi de aici, trecând la determinanŃi, d2 = (2 –
– d)n. Pentru d > 0, funcŃia f(d) = d2 + (d – 1)n (n impar) este strict crescătoare, deducem d = 1 este unica soluŃie a
ecuaŃiei obŃinute. Pentru d = 1, A2 = In, deci A = A–1 şi din ipoteză A = A–1 = A*. RelaŃia cerută se verifică
imediat.

CLASA A XII-
XII-A

1. a) Demonstrăm că dacă f are proprietatea P şi este integrabilă, atunci f este injectivă. Fie α, β ∈ [a, b] şi f(α) =
α, n impar
= f(β). Considerăm şirul x n =  . Deoarece şirul (f(xn)) este convergent (fiind constant) rezultă că (xn)
β, n par
este convergent, deci α = β. Pe de altă parte, f are proprietatea lui Darboux (având primitive) şi cum f este şi
x , x ∈ [a , b ]
injectivă, rezultă că este strict monotonă, deci integrabilă; b) FuncŃia f: [a, b] → R, f ( x ) =  este
b + 1, x = b
integrabilă, are proprietatea P şi nu are primitive; 2. Fie F o primitivă a funcŃiei f. RelaŃia dată se rescrie F(b) –
– bf(c) ≠ F(a) – af(c), pentru orice a şi b. Deci funcŃia g(x) = F(x) – xf(c) este injectivă, cum este şi continuă este

372
strict monotonă. Rezultă că g'(x) = f(x) – f(c) are semn constant. Din g'(c) = 0 rezultă că c este punct de extrem
pentru g' (deoarece g'(x) ≥ 0 = g'(c), (∀) x sau g'(x) ≤ 0 = g'(c)). Din teorema lui Fermat, g''(c) = 0, de unde
f'(c) = 0; 3. Din ipoteză există şirurile (an), (bn) de numere naturale an < bn < an+1 < bn+1 şi P(an) ≤ Q(an), P(bn) ≥
≥ Q(bn), pentru orice n. Considerând polinomul f = P – Q, din f(an) ≤ 0 ≤ f(bn), rezultă că f are o infinitate de
rădăcini, deci f = 0; 4. Dacă x ∈ A este inversabil, atunci xn = 1. Fie x arbitrar din A, din xk(xn – 1) = 0, rezultă
că x(xn – 1) este nilpotent. Rezultă că xn+1 – x + 1 este inversabil, de unde (xn+1 – x + 1)n = 1, putem considera
P(x) = (xn+1 – x + 1)n – 1.

Concursul interjudeŃean de matematică „Victor Vâlcovici”, Brăila,


15 mai 2010
CLASA A IX-
IX-A

1. Pentru x = 0, rezultă f(f(y)) = y + f(f(0)), (∀) y ∈ R, pentru y = 0 obŃinem f(2x + f(0)) = f(f(x)) + x, (∀) x ∈ R.
Din aceste relaŃii, f(2x + f(0)) = 2x + f(f(0)), (∀) x ∈ R, deci f(2x + f(0)) = 2x + f(0) + f(f(0)) – f(0), (∀) x ∈ R.
Rezultă f(x) = x + a, a ∈ R, funcŃii care verifică condiŃia dată; 2. Notăm a = tg x, b = tg y, c = tg z, a, b, c ∈
a 2 + 1 b2 + 1 c2 + 1 a2 b2 c2
∈ (0, ∞). Inegalitatea devine + + ≥ 3 , a, b, c ∈ (0, ∞) şi rezultă din + + ≥
b+c a+c b+a b+c a+c a+b
(a + b + c) 2 a + b + c 1 1 1 9 a+b+c 9 BA'
≥ = şi + + ≥ şi + ≥ 3 ; 3. Fie =α,
2(a + b + c) 2 b + c a + c a + b 2(a + b + c) 2 2( a + b + c ) A' C
CB' AC' 1 α 1
= β, = γ , αβγ = 1. Avem: AA' = AB + AC şi analoagele. Rezultă AG ' = (AA' + AB' +
B' A C' B α +1 α +1 3
1  1 γ   α 1   2 1
+ AC') =  + AB +  + AC . Dar AG = AA' = (AB + AC) . Cum G = G', rezultă
3  α + 1 γ + 1   α + 1 β + 1   3 3
1 γ α 1
+ = 1; + = 1 , de unde α = β = γ şi cum αβγ = 1 ⇒ α = β = γ = 1. Prin urmare triunghiul
α +1 γ +1 α +1 β +1
ABC este echilateral. Rezultă că raportul cerut este egal cu 4.

CLASA A X-
X-A

1. a) a s⋅sin x + t ⋅cos x ⋅ b s⋅cos x + t ⋅sin x = 10 (s sin x + t cos x ) lg a + (s cos x + t sin x ) lg b . Dar (s ⋅ sin x + t ⋅ cos x) ⋅ lg a + (s ⋅ cos x +
+ t ⋅ sin x) ⋅ lg b = sin x(s ⋅ lg a + t ⋅ lg b) + cos x(t ⋅ lg a + s ⋅ lg b) ≤ (s lg a + t lg b) 2 + ( t lg a + s lg b) 2 =

= (s 2 + t 2 ) lg 2 a + (s 2 + t 2 ) lg 2 b + 2st ⋅ 2 lg a lg b ≤ (s 2 + t 2 )(lg 2 a + lg 2 b) + 2st (lg 2 a + lg 2 b) = (s + t) ⋅

⋅ lg 2 a + lg 2 b ; b) Nu am folosit condiŃia a, b ≥ 1. Prin urmare inegalitatea se păstrează pentru a, b > 0; 2. Dacă


z ∈ A – Im f, există y ∈ Im f astfel încât f(z) = y (din f(f(z)) = f(z)). Pentru k număr natural fixat, 1 ≤ k ≤ n sunt
C kn moduri de a alege Im f (pentru care f(x) = x). Pentru ∀ z ∈ A – Im f (n – k elemente) există x ∈ Im f cu
f(z) = x. Numărul funcŃiilor definite pe A – Im f cu valori în Im f este kn–k. Avem C kn k n − k funcŃii care au
n
proprietatea că Im f are k elemente. În total ∑C k
k =1
k n −k
n ;

373
BA' CB' AC' 1 α
3. Fie =α, = β, = γ , αβγ = 1. Avem: AA' = AB + AC şi analoagele. Rezultă
A' C B' A C' B α +1 α +1
1 1  1 γ   α 1   2 1
AG ' = (AA' + AB' + AC') =  + AB +  + AC . Dar AG = AA' = (AB + AC) .
3 3  α + 1 γ + 1   α +1 β +1  3 3
1 γ α 1
Cum G = G', rezultă + = 1; + = 1 , de unde α = β = γ şi cum αβγ = 1 ⇒ α = β = γ = 1. Prin
α +1 γ +1 α +1 β +1
urmare triunghiul ABC este echilateral. Rezultă că raportul cerut este egal cu 4.

CLASA A XI-
XI-A

a2
1. Evident an > 0, iar din a n +1 − a n = > 0 , rezultă că (an) este strict crescător. Dacă ar fi mărginit, ar fi
an
a2 a2 a2 a2
convergent. Fie l = lim a n ⇒ l − l = , fals. Rezultă că lim a n = ∞ . Din n 2+1 = n2 + 2 + 2 rezultă (an+1 –
n →∞ l n →∞ a a an
4 2
a a
– an)(an+1 + an) = 2 + 2a 2 şi folosind Stolz, lim n = lim (a 2n +1 − a 2n ) = 2a 2 . Rezultă că limita cerută este
an n →∞ n + a n →∞

2 2
egală cu a 2 ; 2. Din f'(x) > 0, (∀) x ≥ 2, rezultă că există lim f ( x ) . Cum 2 4
≤ 2 , (∀) x ≥ 2, rezultă
x →∞ x + f (x) x
2 2 2
că f ' ( x ) − 2 ≤ 0 , x ≥ 2. Considerăm funcŃia g: [2, ∞) → R, g( x ) = f ( x ) + . Cum g'(x) = f'(x) − 2 ≤ 0,
x x x
2
rezultă că g este descrescătoare. Din x ≥ 2 rezultă g(x) ≤ g(2), de unde f ( x ) ≤ 1 − . Prin urmare lim f ( x ) ≤ 1 <
x x →∞
< ln 3; 3. a) Cum A este singulară, rezultă, rangA ≤ n – 1. Dacă rangA ≤ n – 2, rezultă A* = On, fals. Rezultă că
rangA = n – 1. Inductiv se arată rang(Am) ≥ n – m (din inegalitatea Sylvesete, rang(Am+1) ≥ rang(Am) + rangA –
– n). Aşadar rang(An–1) ≥ 1, deci An–1 ≠ On; b) Avem AA* = (detA) ⋅ In = 0. Din inegalitatea Sylvester,
rang(AA*) ≥ rangA + rangA* – n, rezultă rangA* ≤ 1 şi cum A* ≠ 0, rezultă rangA* = 1. Dacă p ≥ n + 1, vom
avea în A* elemente pe cel puŃin două linii şi două coloane care au cel puŃin două elemente nenule. Dar cum
rang(A*) = 1, toŃi minorii de ordin 2 din A* trebuie să fie nuli. Cum p ≥ n + 1 ≥ 4, vom avea în A* o submatrice
a 0
de forma   (sau o permutare de linii sau coloane), unde a, b sunt elemente nenule, deci rangA* ≥ 2, fals.
* b
Dacă nu ar exista o astfel de situaŃie, atunci cele p elemente sunt dispuse pe x linii şi y coloane, la intersecŃiile
dintre acestea sunt cele p elemente nenule, deci p = u ⋅ v, u, v ≥ 2, fals. În concluzie, p ≤ n.

Concursul interjudeŃean de matematică „Alexandru Myller”, 2010

CLASA A IX-
IX-A

1. Dacă x ∈ [n2, (n + 1)2], n ∈ N, n ≥ 5, avem 2x ≥ 2n2 ≥ (n + 2)2 şi intervalul [x, 2x] conŃine întregii în progresie
 25 
geometrică (n + 1)2, (n + 1)(n + 2), (n + 2)2. Dacă x ∈  ,16 , atunci [x, 2x] conŃine progresia 16, 20, 25.
2 

374
Dacă x ∈ [16, 18], atunci [x, 2x] conŃine progresia 18, 24, 32, iar dacă x ∈ [18, 25], intervalul [x, 2x] conŃine
progresia 25, 30, 36; 2. Fie F mulŃimea submulŃimilor date. Din ipoteză F conŃine câte un element al fiecăreia
din cele 2n–1 perechi de forma (X, A – X) cu X ⊂ A. Astfel, dacă X, Y∈ F atunci X ∩ Y ∈ F, deoarece
intersecŃia mulŃimilor X, Y, A – X ∩ Y este nevidă. Prin urmare dacă X ∈ F are k ≥ 2 elemente şi Y ⊂ X, Y ≠ X,
Y ≠ ∅, atunci F conŃine una dintre mulŃimile Y sau X – Y, deci F conŃine şi o mulŃime nevidă cu mai puŃin de
k elemente. Prin inducŃie F conŃine o mulŃime cu un singur element a, deci toate mulŃimile din F îl conŃin pe a;
3. Putem presupune că x1 < x2 < ... < xk < 0 ≤ xk+1 < ... < xn. Rezultă xk+j ≥ xk+1 + j – 1 ≥ 0 pentru 1 ≤ j ≤ n – k şi
n n −k k −1
xk–i ≤ xk – i < 0 pentru 0 ≤ i ≤ k – 1. Astfel S = ∑
p =1
x 2p ≥ ∑
j =1
( x k +1 + j − 1) 2 + ∑ (x
i =0
k − i) 2 . Dacă xk ≤ –1, S ≥

n −k k −1 n −k
n (n + 1)(n − 1)
≥ ∑
j =1
( j − 1) 2 + ∑
i =0
(i + 1) 2 ≥
12
, iar dacă xk ∈ (–1, 0), xk+1 ≥ xk + 1 > 0 şi S ≥ ∑ (x
j =1
k + j) 2 +

k −1
n (n + 1)(n − 1)
+ ∑ (x
i=0
k − i) 2 ≥
12
; 3. a) Fie S punctul de concurenŃă al cevienelor. Aplicând teorema lui Menelaus

CM BS NT CN AP
în ∆NBM cu transversala S–T–C şi în ∆BAN cu transversala P–S–C rezultă: ⋅ ⋅ = 1 şi ⋅ ⋅
CB SN TM CA PB
BS
⋅ = 1 . De aici, folosind teorema lui Ceva în ∆ABC, rezultă concluzia problemei; b) Dacă MN || AB,
SN
concluzia este evidentă. În caz contrar, fie {U} = EF ∩ MN, {V} = NE ∩ BC. Din asemănări de triunghiuri
NU NE PE BN AN ⋅ BC 1 BC AN
avem: = = = = ⋅ = ⋅ . Deci T şi U împart (MN) în acelaşi raport, prin
UM MF PM BM AC BM AC BM
urmare T şi U coincid şi de aici concluzia problemei.

CLASA A X-
X-A

1. Cum x ∈ (1, 2], atunci (x – 1)(2 – x) ≥ 0, de unde 3x – 2 ≥ x2. La fel 3y – 2 ≥ y2, y ∈ (1, 2]. Rezultă că:
 1 
logx (3y – 2) + logy (3x – 2) ≥ logx y2 + logy x2 = 2 log x y +  ≥ 4 ; 2. Fie t = |1 + z|, deci t ∈ [0, 2] şi t2 =
 log y
x 
2
= (1 + z) (1 + z) , de unde z + z = t 2 − 2 . Rezultă |1 – z + z2|2 = (1 – z + z2) (1 − z + z ) = ( t 2 − 3) 2 . Astfel
13
inegalitatea este echivalentă cu 3 ≤ t + | t 2 + 3 | ≤ , t ∈ [0, 2]. Se arată uşor că funcŃia f: [0, 2] → R are un
4
1 3 13 1 7 15
minim egal cu 3 pentru t = 3 , deci z =
±i şi un maxim egal cu pentru t = , deci z = − ± i ;
2 2 4 2 8 8
1 1 a1 a2 −1 1 1 a n −2
3. Evident x ∈ Q, x > 0. Cum ≤ ; ≤ = − ; ≤
a1a 2 ...a n a 2 a 3 ...a n a 2 a 3 ...a n a 2 a 3 ...a n a 3 ...a n a 2 ...a n a n −1a n
a −1 1 1 a a −1 1
≤ n −1 = − ; n −1 ≤ n = 1− , rezultă x ≤ 1. Prin urmare x ∈ (0, 1] ∩ Q. Observăm că x = 1
a n −1a n a n a n −1a n an an an
p
poate fi scris sub forma dorită, alegând n= 1, a1 = 1, iar dacă x = ∈ (0, 1) cu p < q, p, q ∈ N*, atunci x = x ⋅
q

375
p 1 1 2 p −1
⋅ 1=  + + + ... +  , deci putem considera n = p, a1 = 1, a2 = 2, ..., ap–1 =
q  1 ⋅ 2 ⋅ ... ⋅ p 2 ⋅ 3 ⋅ ... ⋅ p 3 ⋅ 4 ⋅ ... ⋅ p p 
= p – 1, ap = q. În concluzie numerele căutate sunt elementele mulŃimii Q ∩ (0, 1]; 4. Presupunem că f nu este
izometrie. Există atunci X, Y ∈ P astfel încât XY ≠ f(X)f(Y). Fie XSTY un patrulater convex. Demonstrăm că
există două vârfuri ale acestui patrulater, pe care le vom numi A şi B, astfel încât AB < f(A)f(B). În caz contrar
XS ≥ f(X)f(S), ST ≥ f(S)f(T), TY ≥ f(T)f(Y), YX > f(Y)f(X) şi prin adunarea acestor relaŃii P(XSTY) >
> P[f(X)f(S)f(T)f(Y)], contradicŃie. Fie C ⊂ P , astfel încât P[ABC] < 2α, unde α este lungimea segmentului
f(A)f(B) (de exemplu considerăm un triunghi ABC isoscel cu CA = CB şi hC < α(α − AB) ). Dacă AMNB este
un patrulater convex cu M, N interioare triunghiului ABC, atunci P[AMNB] < P[ABC]. Pe de altă parte
P[AMNB] = P[f(A)f(M)f(N)f(B)] = f(A)f(M) + f(M)f(N) + f(N)f(B) + α ≥ 2α > P[ABC], contradicŃie.

CLASA A XI-
XI-A

ax + b a b a bn 
1. Căutăm o funcŃie f: (0, ∞) → (0, ∞), f ( x ) = . Dacă A =   şi A n =  n  , n ∈ N*, atunci
cx + d  c d   cn d n 
4
a n x + bn a b  41 40 
f [n ] (x ) = . Rămâne de rezolvat ecuaŃia A 4 =   =   . O soluŃie este a = d = 2, b = c = 1,
cn x + d n c d  40 41 
2x + 1
adică f ( x ) = ; 2. Fie m cel mai mic număr natural cu proprietatea că (AB)m = On. Deoarece ABA ≠ On,
x+2
rezultă m > 1 şi (AB)m–1 ≠ On. Dacă A(BA)m–1 ≠ On, matricea C = (BA)m–1 satisface cerinŃa enunŃului, deoarece
ABAC = (AB)mA = On şi AC = A(BA)m–1 ≠ On. Dacă A(BA)m–1 = On, cum ABA ≠ On, rezultă m > 2. Atunci
ABA(BA)m–2 = On şi A(BA)m–2 ≠ On (altfel (AB)m–1 = On, fals). Considerăm C = (BA)m–2. Cum ABAC =
= A(BA)m–1 = On şi AC = A(BA)m–2 ≠ On, soluŃia se încheie; 3. Fie x ∈ R şi a > 2 astfel încât x ∈ [–a, a].
c c
Conform teoremei lui Weierstrass, funcŃia |f| are pe [–a, a] un punct de maxim c. Cum ∈ [–a, a] şi + 1 ∈
2 2
1 c  1 c 1 1 3
∈ [–a, a], rezultă că |f(c)| = f  + 1 − f   ≤ | f (c) | + | f (c) | = | f (c) | , de unde f(c) = 0, deci f este
2 2  4 2 2 4 4
identic nulă pe [–a, a], deci f este nulă pe R; 4. Presupunem că ar exista un şir (xn), xn > 0, strict crescător şi
nemărginit, pentru care şirul f(xn) nu are limită. Fie a < b două puncte limită ale şirului (f(xn)) şi ε > 0, c ∈ R,
astfel încât a + ε < c < b – ε. Fie (yn) şi (zn) două subşiruri ale lui (xn) astfel încât f(yn) < a + ε şi f(zn) > b – ε,
oricare ar fi n ≥ 1. Pentru n ≥ 1, cum f(yn) < c < f(zn), există tn între yn şi zn, astfel încât f(tn) = c. Alegem m ≥ n
astfel încât min{ym, zm} > max{yn, zn}. Analog există tm între ym şi zm, astfel încât f(tm) = c. Cum tm > tn, ecuaŃia
f(x) = c are o infinitate de soluŃii, contradicŃie. Rezultă că pentru orice şir (xn) strict crescător şi nemărginit, şirul
(f(xn)) are limită. Prin urmare f are limită la infinit.

CLASA A XII-
XII-A
x
1. Fie F: [a, b] → R, F( x ) = ∫ f (t)dt . Deoarece f este integrabilă, F este continuă. Notăm cu A ⊂ [a, b]
a
mulŃimea punctelor de maxim global ale lui F, care este nevidă, conform teoremei lui Weierstrass. Fie c ∈ A.
y
Dacă c < b, atunci din ipoteză există y ∈ (c, b) astfel încât F(y) – F(c) = ∫c
f ( t )dt > 0 , deci F(y) > F(c), fals.

376
b
Deoarece A este nevidă şi A ∩ [a, b) = ∅, rezultă că A = {b}. ObŃinem F(b) > F(a) = 0, deci ∫ a
f (t)dt > 0 ;
2. Pentru x = 0, inegalitatea este evidentă. Fie x > 0 şi t, y ∈ [0, x]. Atunci, cum f este crescătoare, rezultă (f(t) –
– f(y))(t – y) ≥ 0, deci tf(t) + yf(y) ≥ tf(y) + yf(t). Pentru y fixat, integrând inegalitatea precedentă în funcŃie de t
x x2 x
pe [0, x], obŃinem ∫ tf (t)dt + yf (y)x ≥ f (y) + y ∫ f (t)dt . Integrăm acum noua inegalitate după y pe [0, x].
0 2 0

x x x2 x x2 x x x
ObŃinem: x ∫ tf (t)dt + x ∫ yf (y)dy ≥ ∫ f (y)dy + ∫ f (t)dt . Rezultă 2x ∫ tf (t)dt ≥ x 2 ∫ f (t)dt şi de aici
0 0 2 0 2 0 0 0

inegalitatea cerută; 3. a) Dacă E este elementul neutru al lui G, atunci E ⋅ E = E, de unde tr(E) = tr(E ⋅ E) = tr2E.
Rezultă că tr(E) ∈ {0, 1}. Deci tr(E) ≠ tr(I3), atunci E ≠ I3; b) Din E ⋅ E = E şi E ≠ I3, rezultă că detE = 0. Fie A ∈
∈ G. Cum An = E, obŃinem det(An) = det(E), deci detA = 0. Fie λ1, λ2, λ3 ∈ C valorile proprii ale lui A. Cum
tr(A2) = tr2(A), rezultă λ21 + λ22 + λ23 = (λ1 + λ 2 + λ 3 ) 2 , deci λ1λ 2 + λ1λ 3 + λ 2 λ 3 = 0 . Deoarece detA = 0, rezultă
că polinomul caracteristic al lui A este X3 – tr(A) ⋅ X2, deci A3 = tr(A) ⋅ A2. Prin inmulŃire cu simetrica lui A2 din
G obŃinem A = tr(A) ⋅ E. Dacă O3 ∈ G, atunci O3 ⋅ O' = E, deci A = O3, oricare ar fi A ∈ G, fals. Prin urmare
tr(A) ≠ 0, oricare ar fi A ∈ G. Considerăm F: G → C*, F(A) = tr(A). Cum F este morfism injectiv de grupuri, G
este izomorfism cu Im(G), care este subgrup în (C*, ⋅) cu n elemente, deci egal cu Un; 4. Presupunem că a şi b
sunt inversabile. Fie c ∈ A un element inversabil. Deoarece funcŃia f: A → A, f(x) = cx este injectivă, rezultă că
f este bijectivă. obŃinem: c7a = c7 ∑
x∈A
x7 = ∑
x∈A
c7 x 7 =∑
x∈A
(cx)7 = a şi analog b = c8b. Cum a, b sunt inversabile,

rezultă c7 = c8 = 1, deci c = 1. Deoarece 1 este singurul element inversabil al lui A şi –1 este inversabil, rezultă
1 = –1, deci 2x = 0, oricare ar fi x ∈ A. Prin urmare, conform teoremei lui Cauchy, A are 2n elemente, n ≥ 2.
FuncŃia g: A → A, g(x) = x8 este injectivă (deoarece g(x) = g(y) ⇒ x8 – y8 = 0 ⇒ (x – y)8 = 0 ⇒ 1 – (x – y)8 = 1,
de unde 1 – (x – y) este inversabil, deci x – y = 0). Cum g este bijectivă, rezultă x= ∑ ∑
x∈A
x 8 = 1 . Dar suma
x∈A
elementelor într-un 2-grup elementar cu cel puŃin patru elemente este nulă, contradicŃie.

Concursul interjudeŃean de matematică „Alexandru Papiu Ilarian”,


EdiŃia a XIV-a, Târgu-Mureş, 2009

CLASA A IX-
IX-A

r
1. a) 2, 4, 5, 8, 10, 14, 16, 20, 22, 25; b) Fie x = 30k + r, 0 ≤ r < 30, soluŃie a ecuaŃiei date, atunci 12k +   +
2
r r   r   r 
+   = 10 +   +   +   . ObŃinem k = 0 şi r = 25, deci x = 25; 2. Fie a, b, c lungimile laturilor
5  6  10  15 
triunghiului ABC. Inegalitatea este echivalentă cu: b + c − a + a + c − b + a + b − c ≤ a + b + c . Cum
x + y ≤ 2( x + y) , x, y ≥ 0, obŃinem inegalităŃi analoage. Prin sumarea acestora rezultă concluzia

problemei; 3. a) Din relaŃia de recurenŃă xn ≥ 0, pentru orice n ∈ N şi cum xn+1 – xn = 2xn + 8x 2n + 1 > 0 , rezultă

că xn+1 > xn, (∀) n ∈ N. Totodată x n +1 − 3x n = 8x 2n + 1 implică x 2n +1 − 6x n x n +1 + x 2n = 1 , pentru orice n ∈ N.

377
Deci: x 2n + 2 − 6 x n +1x n + 2 + x 2n +1 = 1 = x 2n +1 − 6x n x n +1 + x 2n , (∀) n ∈ N şi de aici (xn+2 – xn)(xn+2 – 6xn+1 + xn) = 0.
Cum xn+2 > xn rezultă xn+2 = 6xn+1 – xn. Deoarece x0 = 0; x1 = 1 şi xn+2 = 6xn+1 – xn, n ∈ N, prin inducŃie rezultă
xn ∈ N; b) x0, x1, x2, x3, ..., considerat modulo 6, este 0, 1, 0, 5, 0, 1, ... (xn+2 = –xn = xn–2, modulo 6, n ≥ 2).
Rexultă că (xn + 1) ⋮ 6 ⇔ n = 4k + 3, k ∈ N; 4. a) Numărăm triunghiurile isoscele BA1C cu A1B = A1C.
Bisectoarea unghiului 'BA1C este diametrul cercului circumscris poligonului. Punctul B îl putem alege în
 n − 1  n − 1
 2  moduri şi implicit C. Din cele  2  triunghiuri, dacă n este divizibil cu 3, trebuie eliminat triunghiul
   
 n − 1
echilateral de vârf A1. Astfel sunt n   triunghiuri isoscel (neechilaterale) dacă n nu se divide cu 3 şi
 2 
 n − 1
n  – n, dacă n se divide cu 3; b) Dacă n nu se divide cu 3, nu se formează triunghiuri echilaterale, nu
 2 
eliminăm niciun vârf. Dacă n se divide cu 3, n = 3k, triunghiurile echilaterale care se formează sunt A1Ak+1A2k+1,
A2Ak+2A2k+2, ..., AkA2kA3k şi trebuie eliminat câte un vârf din fiecare, deci eliminăm k vârfuri (de exemplu, A1,
n
A2, ..., Ak) deci vârfuri.
3
CLASA A X-
X-A
  
1. Dacă OA şi OB sunt coliniari, atunci OB ⋅ OA ± OA ⋅ OB = 0 şi luăm a = OB, b = ±OA, c = 0. Dacă OA şi
OB sunt necoliniari, atunci există a, b ∈ R astfel încât OC = a ⋅ AO + b ⋅ OB . Arătăm că a, b sunt raŃionale.
Avem sistemul OC ⋅ OA = a ⋅ OA 2 + b ⋅ OB ⋅ OA şi OC ⋅ OB = a ⋅ OA ⋅ OB + b ⋅ OB2 . Dar AB2 = AB ⋅ AB =
= (OB − OA) 2 = OB 2 + OA 2 − 2OB ⋅ OA , deci OA ⋅ OB ∈ Q şi analog OB ⋅ OC ∈ Q; OC ⋅ OA ∈ Q. Din
a 2 + b2 − c2
sistemul obŃinut rezultă că a, b ∈ Q; 2. Se deduce uşor x = şi relaŃii analoage pentru y şi z. ObŃinem
2ab
a 2 ( b + c) + b 2 (a + c) + c 2 (a + b ) − a 3 − b 3 − c 3
x+y+z= . Inegalitatea x + y + z ≥ 1 se reduce la inegalitatea
2abc
(a – b + c)(a + b – c)(–a + b – c) ≥ 0, adevărată, deoarece (a + b)2 = a2 + b2 + 2ab > c2 (din x > 0), deci a + b > c
3
şi analoagele. Inegalitatea x + y + z ≤ se reduce la a2(b + c – a) + b2(a + c – b) + c2(a + b – c) ≤ 3abc, care
2
rezultă, alegând x = min{a, b, c} din a (b + c – a) ≤ abc şi b2(a + c – b) + c2(a + b – c) ≤ 2abc.
2

Notă. Observând că a, b, c pot fi laturile unui triunghi, x = cos A, y = cos B, z = cos C, A, B, C măsurile
3
unghiurilor triunghiului, avem de arătat că 1 ≤ cos A + cos B + cos C ≤ , inegalităŃi cunoscute; 3. Dacă a1, a2,
2
..., an sunt în progresie aritmetică, atunci D = {(i – j)r | i, j = 1, n} = {0, ±r, ±2r, ..., ±(n – 1)r}, deci D are 2n – 1
elemente. Reciproc, fie a1 < a2 < ... < an. Numerele 0 = a1 – a1; ±(a2 – a1); ±(a3 – a1); ... ; ±(an – a1) sunt din D,
distincte şi în număr de 2n – 1. Prin urmare orice număr ai – aj va fi egal cu unul dintre acestea. Notăm b1 = a2 –
– a1, b2 = a3 – a1, ..., bn–1 = an – a1. Avem: 0 < a3 – a2 < a3 – a1 = b1, deci a3 – a2 = b1, apoi b1 = a3 – a2 < a4 – a2 <
< a4 – a1 = b3, deci a4 – a2 = b2; b2 = a4 – a2 < a5 – a2 < a5 – a1 = b4, deci a5 – a2 = b3; .....; bn–3 = an–1 – a2 < an –
– a2 < an – a1 = bn–1 ⇒ an – a2 = bn–2. Procedând analog cu numerele ai – a3, i > 3, găsim: a4 – a3 = d1, a5 – a3 = d2,
..., an – a3 = dn–3, continuăm procedeul găsind an – an–1 = d1. Din a2 – a1 = a3 – a2 = ... = an – an–1 = d1 rezultă că
(an) este o progresie aritmetică; 4. a) Considerăm sumele S1 = a1, S2 = a1 + a2, ..., Sm = a1 + a2 + ... + am. Dacă o
sumă este divizibilă cu m, soluŃia se încheie, în caz contrar există două sume Si şi Sj, i < j, egale modulo m,

378
atunci Sj – Si = ai+1 + ai+2 + ... + aj se divide cu m; b) Dacă A1, A2, ..., A 2 n −1 sunt submulŃimile nevide ale lui A şi
S1, S2, ..., S2 n −1 , sumele elementelor lor, deoarece m ≤ 2n – 1 sau una dintre sume se divide cu m (când problema
este rezolvată) sau două sume Si, Sj sunt egale modulo m. Dacă eliminăm din Ai, Aj elementele comune, rămân
două mulŃimi Ak, Ap astfel încât Sk = Sp (mod m), deci a− ∑
b = 0 (mod m).
a∈A k

b∈A p

CLASA A XI-
XI-A

1. Dacă (a1, a2, ..., a10) ∈ A atunci definim b1 = a1, b2 = a2 – 9, b3 = a3 – 2 ⋅ 9, ..., b10 = a10 – 9 ⋅ 9 şi atunci (b1, b2,
..., b10) ∈ B = {(b1, b2, ..., b10) | 1 ≤ b1 < b2 < ... < b10 ≤ 20 = 101 – 81}. Reciproc. Dacă (b1, b2, ..., b10) ∈ B atunci
(a1, a2, ..., a10) ∈ A, unde a1 = b1, a2 = b2 + 9, a3 = b3 + 2 ⋅ 9, ..., a10 = b10 + 9 ⋅ 9. MulŃimile A şi B au acelaşi
număr de elemente. Un element din B se alege luând 10 elemente distincte din mulŃimea {1, 2, ..., 20}, ceea ce
10
se poate face în C10
20 moduri, deci |A| = C 20 ; 2. Dacă a ≠ b, atunci xn are forma: xn = A ⋅ an + B ⋅ bn, n ∈ N. Din
1
x0 = 1 şi a ≠ b, obŃinem x n = [ x1 (a n − b n ) − ab(a n −1 − b n −1 )] , n ∈ N. Putem considera a < b. Dacă şirul
a−b
(xn)n conŃine o progresie aritmetică infinită, yk = x n k , y k = y 0 + kr , r ≠ 0, atunci şirul (xn)n este nemărginit, deci
b > 1. Pentru b > 1 se arată uşor că lim ( x n +1 − x n ) = +∞ sau –∞, în particular există N ∈ N astfel ca |xn+1 – xn| >
n →∞
> 2|r| pentru n > N şi atunci există intervale de lungime 2|r| în care nu se găsesc termeni din şir, deci nici termeni
din progresia (yk)k, contradicŃie. Dacă a = b, atunci xn = A ⋅ an + B ⋅ n ⋅ an–1 şi din x0 = 1 = A şi x1 = A ⋅ a + B
rezultă xn = an + (x1 – a) ⋅ n ⋅ an–1. Cu acelaşi argument şirul (xn)n trebuie să fie nemărginit, deci a ≥ 1. Dacă a > 1
atunci la fel lim ( x n +1 − x n ) = +∞ sau –∞, deci nu conŃine progresii infinite. Dacă a = 1, atunci xn = 1 + (x1 – 1) ⋅
n →∞
⋅ n, care este o progresie aritmetică infinită de raŃie r = x1 – 1 ≠ 0 (x1 ≠ a ⇒ x1 ≠ 1); 3. Vom arăta că σ(n) = n şi
prin inducŃie va rezulta σ(k) = k, k = 1, n . Dacă prin absurd σ(n) ≠ n, atunci există k < n astfel ca σ(k) = n.
Avem k + σ(k) < (k + 1) + σ(k + 1) ⇔ σ(k + 1) > n, deci σ(k + 1) = n, contradicŃie cu σ(k) = n;
k k
4. a) Avem relaŃia iCik = kCik−−11 şi atunci ∑i =1
iCik = k ∑C
i =1
i −1
k −1 = k ⋅ 2 k −1 ; b) Exprimăm C px ca un polinom în x:

x ( x − 1)...( x − p + 1) x x(x −1) x(x − 1)(x − 2)


C px = şi atunci polinomul Q(x) = C1x + 2C 2x + ... + nC nx ; = + 2 ⋅ + 3⋅ +
p! 1! 2! 3!
x ( x − 1)...( x − n + 1)
+ ... + n ⋅ are gradul n şi Q(k) = C1k + 2C 2n + ... + kC kk + 0 = k ⋅ 2 k −1 , deci Q(k) = P(k), k = 0,
n!
1, ..., n. P şi Q fiind egale în mai mult de n numere, rezultă că sunt identice, deci P = Q şi atunci: P(n + 1) =
n +1 (a )
= Q(n + 1) = C1n +1 + 2C 2n +1 + ... + nC nn +1 = ∑C
i =1
i
n +1 − (n + 1)C m +1 n n
n +1 = ( n + 1) ⋅ 2 − ( n + 1) = ( n + 1)(2 − 1) .

CLASA A XII-
XII-A

1  cos t sin t  a
1. Scriem pe Xa sub forma X n = ⋅  , unde t ∈ [0, 2π], astfel că cos t = , sin t =
1 + a 2 − sin t cos t  2
a +1

379
1 1  cos nt sin nt 
= . Avem (X a ) n = ⋅  şi condiŃiile devin cos nt > sin nt, pentru n = 1, 2, ...,
a2 +1 ( 1 + a 2 ) n − sin nt cos nt 
π π  π π 
2009 şi cos 2010t < sin 2010t. Alegem t > 0 astfel ca 2009 t < şi 2010 t > , adică t ∈  , ,
4 4  4 ⋅ 2010 4 ⋅ 2009 
π π 1 1− b
de exemplu luăm t = . Dacă notăm cu b = sin 2 , din relaŃia = sin t rezultă a = este
8038 8038 1+ a2 b
una din valorile lui a pentru care se verifică inegalităŃile cerute; 2. Deoarece mulŃimea A este finită şi mulŃimea
B = {(an, an–1) | n ≥ 1} este finită. Există m, n ∈ N, m ≠ n astfel ca (an, an–1) = (am, am–1). Dacă n = m + p, atunci
din relaŃia de recurenŃă obŃinem: aq+p = aq, (∀) q ≥ m, deci şirul (an)n este periodic de perioadă p de la rangul m.
Notăm Sn(x) = a0 + a1x + a2x2 + ... + anxn şi avem: Sm+kp(x) = Sm(x) + xmR(x) + xm+pR(x) + ... + xm+(k–1)pR(x) =
1 − x kp
= Sm ( x ) + x m ⋅ , unde R(x) = am+1x + am+2x2 + ... + am+pxp. Trecând la limită cu k → ∞ obŃinem:
1− xp
xm P(x)
lim Sm + kp ( x ) = Sm ( x ) + p
= , (∀) x ∈ (–1, 1), unde P, Q ∈ R[x]. Pentru numerele n care nu sunt de
k →∞ 1− x Q( x )
forma m + kp, k ∈ N, mai rămân în Sn(x) un număr finit de termeni a m + kp +1x m + kp +1 + ... + a m + kp + i x m + kp + i , şi

când k → ∞ fiecare termen tinde la zero ( lim x m + kp + i = 0 , (∀) x ∈ (–1, 1)); 3. a) În relaŃia f'(x) – 2f(x) ≥ 0
k →∞

înmulŃim cu e–2x şi obŃinem (f(x)e–2x)' ≥ 0, (∀) x ≥ 0, deci funcŃia f(x)e–2x este crescătoare pe [0, ∞) şi atunci
f(x)e–2x ≥ f(0) ⇔ f(x) ≥ e2x, (∀) x ≥ 0; b) Scriem a doua relaŃie sub forma (g'(x) – g(x))' – 2(g'(x) – g(x)) ≥ 0,
(∀) x ≥ 0. Notăm h(x) = g'(x) – g(x) şi verifică condiŃiile h'(x) – 2h(x) ≥ 0, (∀) x ≥ 0, h(0) = g'(0) – g(0) = 1. Din
a) rezultă h(x) ≥ e2x, (∀) x ≥ 0 ⇔ g'(x) – g(x) ≥ e2x, (∀) x ≥ 0. În ultima inegalitate înmulŃim cu e–x şi obŃinem:
(g(x)e–x – ex)' ≥ 0, (∀) x ≥ 0. FuncŃia g(x)e–x – ex este crescătoare pe [0, ∞) şi atunci g(x)e–x – ex ≥ g(0) – 1 = –1,
(∀) x ≥ 0. ObŃinem g(x) ≥ –ex + e2x, (∀) x ≥ 0; 4. a) Asociem matricei A = [aij], matricea B = [bij] în care bij = 1
a
dacă aij este număr par şi bij = –1 dacă aij este număr impar (bij = (−1) ij ). Observăm că matricea A are
proprietatea (P) dacă şi numai dacă produsul oricăror două linii Li' şi Lj' din matricea B conŃine n de 1 şi n de –1,
2n 2n
adică ∑
k =1
b ik b jk = 0 . Deoarece ∑ (b
k =1
ik )
2
= 2n , (∀) i = 1, 2n , rezultă că A are proprietatea (P) dacă şi numai

dacă B ⋅ Bt = 2n ⋅ I2n. Evident avem Bt ⋅ B = 2n ⋅ I2n, relaŃie care reinterpretată dă aceleaşi condiŃii asupra
 1 1
coloanelor matricei B, respectiv asupra coloanelor matricei A; b) Definim B 2 =   şi B 2 k +1 =
− 1 1
 B2 k B2 k  1 1 A 2 k A2k 
= k
 , (∀) k ≥ 1, respectiv A 2 =   şi A 2 k +1 =  k
 , (∀) k ≥ 1, unde [a ij ] =[a ij ] şi 1 = 0 ,
− B k B2  0 1 A k A 2 
 2  2
0 = 1.

380
Concursul interjudeŃean de matematică „Danubius”, Corabia, 2010

CLASA A IX-
IX-A

1. a) Sistemul este echivalent cu x + x = b − c , deci x > 0 (deoarece b ≠ c), rezultă b > c. Din a doua ecuaŃie:
y < c < b; din prima ecuaŃia, 0 < x < b – c; b) Din prima ecuaŃie x + x = a , a = b– c > 0, rezultă
2a + 1 − 4a + 1
x= , apoi y = c – x; 2. Fie a = sin x ∈ [0, 1]. Inegalitatea este echivalentă cu (1 + a)(1 – a2) ≤
2
32
≤ , care este echivalentă cu (3a – 1)2(3a + 5) ≥ 0. SoluŃie alternativă. (1 + a)(1 – a2) = (1 + a)(1 + a)(1 – a) =
27
3
 1+ a 1+ a 
1− a + + 
 1 + a  1 + a  2 2  = 32 ; 3. RelaŃia dată este echivalentă cu: OA + IO + OB +
= 4(1 – a) ⋅    ≤ 4
 2  2   3  27
 
 
2 1
+ GO + OC = OH ⇔ OH + IO + GO = 0 ⇔ IO = 0 ⇔ I = O ⇔ ∆ABC este echilateral; 4. Folosind
3 3
1 1
inegalitatea a 2 + b 2 + c 2 ≥ (a + b + c) , obŃinem: x 2 + [ y]2 + {z}2 ≥ ( x + [ y] + {z}) . Scriind încă două
3 3
inegalităŃi analoge şi sumându-le, obŃinem inegalitatea cerută.

CLASA A X-
X-A

1. Considerăm un sistem de axe cu originea în O. Notăm cu rX vectorul de poziŃie al punctului X, faŃă de O.


1 1
Avem z G 1 = (rA + rB + rP ) şi analoagele. Cum G1G2G3G4 este paralelogram, z O = (z A + z B + z C + z D +
3 6
1
+ 2z P ) . Dar z O = 0, rezultă z P = − (z A + z B + z C + z D ) , de unde unicitatea lui P; 2. Dacă z1 = r(cos 2a +
2
+ isin 2a), z2 = r(cos 2b + isin 2b), a, b ∈ R, numărul din enunŃ este egal cu r2(1 + 8cos acos bcos(a + b)), deci
este real şi cum 1 + cos acos bcos(a + b) = (2cos(a + b) + cos(a – b))2 + sin2(a – b) ≥ 0, numărul cerut este real
pozitiv; 3. Avem f(f(f(x))) = f([x]) = [f(x)], pentru orice x ∈ R, de aici f(k) ∈ Z, pentru orice k ∈ Z. Avem
a−b a−b a−b
|sin a – sin b| = 2 sin cos ≤ 2 sin ≤ |a – b|, pentru orice a, b ∈ R. Arătăm că există a, b ∈ R astfel
2 2 2
încât |f(a) – f(b)| ≥ |a – b|. Presupunem că pentru orice a, b ∈ R, |f(a) – f(b)| < |a – b|, a ≠ b. Rezultă |f(k + 1) –
– f(k)| < 1, pentru orice k ∈ Z, deci f este constantă pe Z, fals. Concluzia se impune; 4. În general: dacă ak > 0,
k = 1, n , n ≥ 3, avem (n − 1)n a1a 2 ...a n −1 + a n ≥ n n ( n −1 a1a 2 ...a n −1 ) n −1 a n = n n a1a 2 ...a n −1 . Egalitate dacă

a1a2...an–1 = a nn .

381
Concursul interjudeŃean de matematică pe echipe „Gheorghe łiŃeica”,
EdiŃia a XXXII-a, Drăgăşani, 21-22 mai 2010
Proba individuală
CLASA A IX-
IX-A

n −1
1. Fie n soluŃie, atunci [ n ] = = k ∈ N. Rezultă n = 3k + 1, [ 3k + 1] = k , k ∈ N sau încă 3k + 1 ∈ [k,
3
 3k + 1 ≥ k
k + 1). Deci  , k ∈ N, rezultă k ∈ {2, 3}. Prin urmare n = 7 sau n = 10; 2. Pentru x = 0 obŃinem
 3k + 1 < k + 1
f(f(y)) = f2(0) + y, (∀) y ∈ R, pentru y = –f2(0) rezultă f(f(–f2(0))) = 0, deci există a ∈ R cu f(a) = 0; x = a ⇒
⇒ f(f(y)) = y, (∀) y ∈ R. Înlocuind pe x cu f(x), obŃinem f(f(x)x + f(y)) = x2 + y, deducem f2(x) = x2 şi de aici
f(1) = 1 sau f(1) = –1. Dacă f(1) = 1, pentru x = 1, relaŃia dată impune f(1+f(y)) = 1 + y, (∀) y ∈ R. Rezultă
f2(1 + f(y)) = (1 + y)2 şi f2(1 + f(y)) = (1 + f(y))2, deci (1 + f(x))2 = (1 + y)2, de unde f(y) = y, pentru orice y ∈ R.
Dacă f(1) = –1, analog găsim f(y) = –y, (∀) y ∈ R. În concluzie f(x) = x, pentru orice x ∈ R, sau f(x) = –x,
k 3k (b + a )
pentru orice x ∈ R; 3. Avem a – b = k (m b − m a ) = ⋅ (m 2b − m a2 ) = ⋅ (a + b) . Dar 4(ma +
mb + ma 4(m a + m b )
 a b
+ mb) > 4 b − + a −  = 2(a + b) ≥ 3k(a + b). Deducem astfel a = b. Analog, din a – c = k(mc – ma), găsim
 2 2
a = c. Prin urmare triunghiul este echilateral.

CLASA A X-
X-A
x 1 1 a
1. Notăm 3 = a , 2 x = b , a, b > 0. Rezultă ecuaŃia 2
+ 2 2 − 3 2 2 = 3 sau încă b2(9a2 – 1) + 3(3a – 1) =
3a a b a b
1
= 0. Rezultă a = şi x = –2; 2. a) Se verifică uşor că, în condiŃiile date |λz1 + z2| = |z1 + λz2|, (∀) λ ∈ R, atunci
3
pentru λ ≥ 0, (λ + 1) ⋅ |z1 + z2| = |λz1 + z1 + λz2 + z2| ≤ |λz1 + z2| + |z1 + λz2| = 2|λz1 + z2|; b) Identificăm planul
(M1M2M3) cu planul complex iar O*, centrul de greutate al triunghiului cu originea planului. Notăm zi afixele
2009z 3 + z1 2009z 2 + z 3 2009z1 + z 2
punctelor Mi, atunci afixele punctelor Mij sunt z13 = ; z 32 = ; z 21 = .
2010 2010 2010
Observăm că z13 + z32 + z21 = 0, deci O* este centrul de greutate al triunghiului M13M32M21. Cum OO* ⊥
⊥ (M1M2M3) şi O*M13 = O*M32 = O*M21 (raza cercului circumscris ∆ M13M32M21), concluzia se impune;
c) Cercul de centru O* şi rază |z13| taie M1M3 în două puncte simetrice faŃă de mijlocul segmentului: M13 şi M13'.
2009z 3 + z1 z + 2009z1
Acestea au afixele , respectiv 3 . Valoarea comună a modulelor, adică |O*M13| este mai
2010 2010
| z + z3 |
mare sau egală cu distanŃa de la O* la M1M3, adică 1 . Inegalitatea de la a) devine: 2 ⋅ dist(O*, M1M3) ≤
2
≤ |O*M13| + |O*M13'|; 3. Căutăm n dintre numerele xi egale cu 1, n egale cu a şi unul egal cu b, b > 0, a ≠ b.
 1 1   n 1  1  1
Atunci E = (x1 + x2 + ... + x2n+1)  + ... +  = (n + na + b) n + +  = 1 + 2n2 + n 2  a +  + n b +  +
 x1 x 2 n +1   a b  a  b

382
a b  1 1 a b  1
+ n +  > 1 + 2n 2 + 4n + n 2  a +  (am folosit b + ≥ 2, + > 2 ). Căutăm a astfel încât n 2  a +  = k ,
b a  a b b a  a
2
 n ( n −1) 
 1   n  q
lucru posibil deoarece k = (q n +1 + ... + q + 1) q 2 + 2 n  ≥  n q 2  ⋅ 2 2 n = 2n 2 .
 q    q
 
SoluŃie alternativă.
1  2n
Alegem x2 = x3 = ... = x2n+1 = 1, x1 = a > 0; E = (a + 2n ) + 2n  = 1 + 2an + + 4n 2 > 1 + 2a + 4n 2 . Căutăm
a  a
k − 2n 2 + 4n
a > 0 astfel ca 1 + 2an + 4n2 ≥ 1 + 4n + 2n2 + k, evident putem face acest lucru, alegem a ≥ .
2n

CLASA A XII-
XII-A

1. Din 1 + xy = x + y = y + x = yx + 1, rezultă că xy = yx, (∀) x, y ∈ A – {0}. Deci xy = yx, (∀) x, y ∈ A. Fie


x ∈ A – {0} arbitrar. Dacă, în relaŃia din ipoteză, y = –x, rezultă x2 = 1, deci orice x ∈ A – {0} este inversabil, cu
x–1 = x. Rezultă că A este corp comutativ. Dacă, în relaŃia din ipoteză, y = x, rezultă x + x = x2 + 1, adică
(x – 1)(x – 1) = 0, deci x = 1. Rezultă A = {0, 1}, deci A ≅ Z2; 2. Deoarece funcŃia continuă f are limita zero la
1
infinit, rezultă că există M ∈ (0, 1) astfel încât f(x) < M, pentru orice x ∈ [0, ∞). Fie 0 < ε < şi un rang k ∈ N,
2
 1 2
astfel încât f(x) < ε, oricare ar fi x ∈ [k, ∞). Vom considera descompunerea z n =  f n ( x )dx + f n −1 ( x )dx +
 0 1 ∫ ∫
k   k +1 n 
+ ... + ∫k −1
f n +1− k ( x )dx  + 
  ∫
k
f n − k ( x )dx + ∫
n −1
f ( x )dx  = a n + b n . Pentru primul termen avem estimarea: an =

1 2 k 1 − M k M n +1− k
= ∫
0
f n ( x )dx + ∫
1
f n −1 ( x )dx + ... + ∫k −1
f n +1− k ( x )dx ≤ M n + M n −1 + ... + M n +1− k = M n +1− k ⋅
1− M
<
1− M
.
n +1− k 1 2
M
Cum limita lim
n →∞ 1 − M
= 0 , există un rang n1 ∈ N, astfel încât ∫0
f n ( x )dx + ∫
1
f n −1 ( x )dx + ... +
k k +1 n −1
+ ∫
k −1
f n +1− k ( x )dx < ε , pentru orice n ≥ n1. Pe de altă parte, avem: b n = ∫ k
f n − k ( x )dx + ... + ∫ n −2
f 2 ( x )dx +

n 1 − εn −k ε
+ ∫
n −1
<
f ( x )dx ≤ εn–k + ... + ε2 + ε = ε ⋅
1− ε 1− ε
< 2ε . În concluzie obŃinem zn < 3ε, pentru orice n ≥ n1 şi

 x   2x 
problema este rezolvată; 3. Din relaŃia de proprietatea b) avem: f ( x ) = f   + f   , (∀) x ∈ [0, ∞).
3  3 
 f (x)
 , x>0
Considerăm funcŃia G: [0, ∞) → R, G(x) =  x . FuncŃia G este continuă pe [0, ∞) şi G(x) =
f '(0), x = 0
1  x  2  2x 
= G   + G   , (∀) x ∈ [0, ∞), (1). Fie a > 0 arbitrar. Notăm M:= sup G ( x ) . Rezultă că există x ∈
3 3 3  3  x∈[ 0, a ]

383
x  x 
∈ [0, a], astfel încât G(x0) = M. Atunci, din relaŃia (1), obŃinem G  0  = M şi, analog, G  n0  = M, (∀) n ∈
 3  3 
∈ N, de unde deducem că f'(0) = M. Analog, f'(0) = m, unde m:= inf G ( x ) . Deci G(x) = f'(0), (∀) x ∈ [0, ∞),
x∈[ 0, a ]
iar f(x) = xf'(0), (∀) x ∈ [0, ∞).

Pentru echipaje de elevi


CLASELE A IX-
IX-A – A X-A

1. Notăm l1, l2, l3 lungimile laturilor unui triunghi şi presupunem că l1 ≤ l2 ≤ l3. Cum raportul lungimilor laturilor
se păstrează constant în cazul oricărui triunghi asemenea cu el, atunci putem presupune întotdeauna că avem
l2 = 1. Deci este suficient să ne referim doar la cazul l1 ≤ l2 ≤ l3. Această categorie de triunghiuri o împărŃim în
1 1
două clase: I) l1 ≤ ; II) l1 > .
l3 l3
l 1 l l
Cazul I. Observăm pentru început că raportul maxim mai mare sau egal cu 1 dintre 3 , , 3 este 3 . Apoi cum
1 l1 l1 l1
1 1 l 1
l1 ≤ rezultă l3 ≤ , deci 3 este raportul minim mai mare sau egal cu 1. astfel avem: l3 < l1 + 1 ≤ + 1 , de
l3 l1 1 l3
1− 5 1+ 5 
unde l3 <
1
+ 1 . Rezultă l3 ∈  ,  . Afirmăm că pentru această clasă α = 1 + 5 . Evident α ≤
l3  2  2
 2
1+ 5 1+ 5 2 1+ 5
≤ şi dacă α < , vom lua triunghiul în care l1 = , l 2 = 1, l3 = . (Se verifică uşor că
2 2 1+ 5 2
acestea pot fi lungimile laturilor unui triunghi). Raportul minim mai mare sau egal cu 1 în acest caz particular
este α, contradicŃie cu cerinŃa de a avea raportul respectiv strict mai mic decât α.
l 1 l 1 1 1 1
Cazul II. Raportul minim dintre 3 , , 3 este pentru că l1 > adică l3 > . Astfel găsim < l 3 < l1 + 1,
1 l1 l1 l1 l3 l1 l1
1 1 1+ 5 1 1+ 5
de unde − l1 − 1 < 0 . Notând t = , avem t 2 − t − 1 < 0 şi deducem că t < sau < . De aici
l1 l1 2 l1 2
1+ 5 1+ 5 1+ 5
rezultă că α = , iar dacă α < , cum α trebuie determinat pentru ambele cazuri, rezultă α = ;
2 2 2
2. Deoarece an+2 = an + an+1 – (an + an+1) ≥ 0 rezultă că an + an+1 ∈ [0, 1] şi cum an şi an+1 sunt numere naturale,
2

obŃinem an + an+1 ∈ {0, 1}, deci an+2 = 0, (∀) n ∈ N. Dacă a0 + a1 = 0, atunci a0 = a1 = 0 şi deci an = 0, (∀) n ∈ N.
Dacă a0 + a1 = 1, atunci obŃinem şirurile: 0, 1, 0, 0, 0, 0, ... şi 1, 0, 0, 0, 0, ... .

CLASELE A XI-
XI-A – A XII-
XII-A

1. Fie ε1, ε2, ..., εn rădăcinile ecuaŃiei xn – 1 = 0 şi A = (a ij )i , j∈1, n ∈ Mn(C), cu aii = εi, (∀) i ∈ 1, n . Deci A =

384
 ε1 a12 a13 ... a1n 
 
 a 21 ε 2 a 23 ... a 2 n 
= ∈ Mn(C). Pentru B, C ∈ Mn(C),cu Bn = In şi Cn = On, trebuie să avem det Bn =
... ... ... ... ... 
 
a 
 n1 a n 2 a n 3 ... ε n 
= det In = 1 sau (det B) = 1, deci det B = 1 sau det B = –1. De asemenea, detC n = det On = 0 sau (det C)n = 0,
n

 ε1 a 12 a13 ... a1n 


 
 0 ε 2 a 23 ... a 2 n 
deci det C = 0. Considerând matricele B, C ∈ Mn(C), astfel: B =  ;
... ... ... ... ... 
 
0 0 0 ... ε n 

 0 0 0 ... 0 
 
 21
a 0 0 ... 0 
C =  a 31 a 32 0 ... 0  , avem B + C = A, iar detB = ε1ε2... εn = (–1)n–1, (det B)n = (–1)n(n–1) = 1 şi det C =
 
 ... ... ... ... ... 
 
 a n1 a n 2 a n 3 ... 0 
= 0. Să mai arătăm că Bn = In şi Cn = On. Polinomul caracteristic al matricei B este: PB (X) = det(X ⋅ I n − B) =
X − ε1 − a12 − a13 ... − a1n
0 X − ε2 − a 23 ... − a 2 n
= = (X – ε1) (X – ε2) ... (X – εn) = Xn – 1. Matricea C are polinomul
... ... ... ... ...
0 0 0 ... X − ε n
 X 0 0 ... 0 
 
 − a 21 X 0 ... 0 
caracteristic: PC (X) = det(X ⋅ I n − C) =  − a 31 − a 32 X ... 0  = Xn. Conform teoremei Hamilton-
 
 ... ... ... ... ... 
 
 − a n1 − a n 2 − a n 3 ... X 
Cayley, avem PB(B) = On şi PC(C) = On, deci Bn – In = On şi Cn = On. Matricele considerate, B şi C, verifică deci
Bn = In şi Cn = On (evident, nu avem şi unicitatea lor); 2. InegalităŃile din ipoteză sunt echivalente cu şirul de
y + ... + y n
inegalităŃi: y1 ≤ αx1, y1 + y2 ≤ α(x1 + x2), ..., y1 + ... + yn ≤ α(x1 + ... + xn), unde α = 1 ∈ [0, 1]. Vom
x1 + ... + x n
n n
arăta în cele ce urmează că în aceste condiŃii are loc inegalitatea: ∑k =1
f (yk ) ≤ ∑ f (α x
k =1
k) . Pentru că funcŃia f este

conveză are loc inegalitatea f(y) ≥ f(x) + f'(x)(y – x), pentru orice x, y > 0. Folosind această caracterizare a
n n
convexităŃii obŃinem: ∑k =1
(f (αx k ) − f ( y k )) ≥ ∑ f ' (y
k =1
k )(αx k − y k ) = (f'(y1) – f'(y2)(αx1 – y1) + (f'(y2) – f'(y3) ⋅

⋅ (αx1 – y1 + αx2 – y2) + ... + (f'(yn–1) – f'(yn)(αx1 – y1 + ... + αxn–1 – yn–1) + f'(yn)(αx1 – y1 + ... + αxn – yn) ≥ 0.
łinând cont că f(αx) = f(αx + (1 – α)0) ≤ αf(x) + (1 – α)f(0) = αf(x), obŃinem inegalitatea de unde obŃinem
concluzia dorită.

385
Tabăra NaŃională de Matematică. Concursul NaŃional de Matematică
„RecreaŃii matematice”, Durău, 28 august 2009
CLASA A IX-
IX-A

1. Dacă x = 0 rezultă y = 0 sau y = 1. Dacă y = 0, atunci x = 0 sau x = 1. Fie x ≠ 0, y ≠ 0, notând y = tx, t ∈ Q*,
 1 + t 1 + t 2009 
2009
1 + t 2009 1 + t 2009
rezultă soluŃia: x = , y = t ⋅ , t ∈ Q* , t ≠ –1. În final avem soluŃiile:  , t ⋅ ,t ∈
1 + t 2010 1 + t 2010  1 + t 1 + t 2010 
2010

∈ Q} ∪ {(0, 1)}; 2. Fie a cel mai mare element din A şi b cel mai mic element din B. Deoarece la scrierea
elementelor unei mulŃimi ordinea este nesemnificativă, rezultă că aceste două numre nu se modifică, se permută
linii sau coloane. Prin urmare, putem face permutări de linii şi coloane care să aducă pe a şi b în careul 2 × 2 din
stânga sus. De asemenea putem face permutări de linii sau coloane astfel încât cel mai mic element din pătratul
2 × 2 să fie în stânga sus. Fără a afecta generalitatea, putem considera că cele patru numere din pătratul 2 × 2
1 2  1 2  1 3  1 3  1 4 1 4
sunt: 1, 2, 3, 4. Avem posibilităŃile:  ;  ;  ;   ;  ;   ;
3 4  4 3  2 4  4 2  2 3 3 2
a=3 a=3 a=2 a=2 a=2 a=2
b=3 b=3 b=2 b=3 b=2 b=3
4
Rezultă că probabilitatea cerută este egală cu ; 3. Fie F ∈ (BD) astfel încât m('BCF) = 30°. Cum m('CBF) =
6
= 75°, rezultă că m('CFB) = 75°, deci CF = CB = CA şi triunghiul CAF este dreptunghic isoscel. Rezultă
m('BAF) = 15° şi implicit F coincide cu E.

CLASA A X-
X-A

1. x = 3a, x + 2 = 3b, x + 8 = 3c, y = a + b + c. ObŃinem 3b – 3a = 2 şi 3c – 3a = 8. Se găseşte a = 0, b = 1, c = 2 şi


BC AC
x = 1, y = 2; 2. Folosim teorema sinusurilor în triunghiurile ABC şi APC obŃinând: 
=
sin 20 sin 80
AP AC sin 20 sin 10
şi 
= 
. Atunci AP = BC dacă şi numai dacă 
= care se verifică cu uşurinŃă;
sin 10 sin 150 sin 80 sin 150
3. Notăm an = f(n), ecuaŃia devine 2an+3an+2 = an+1 + an + 1, an ∈ N, (∀) n ∈ N şi implicit 2an+4an+3 = an+2 + an+1 +
+ 1, rezultând 2an+3 ⋅ |an+4 – an+2| = |an+2 – an|. Dar 2an+3an+2 = an+1 + an + 1 ≥ 1, deci an ≥ 1, (∀) n ≥ 2. Dacă există
n0 ∈ N cu a n 0 + 2 ≠ a n 0 , se obŃine a n 0 + 2 k + 2 ≠ a n 0 + 2 k , (∀) k ∈ N şi a n 0 + 2 − a n 0 > a n 0 + 4 − a n 0 + 2 > ... > 0 ,
contradicŃie. Deci an+2 = an, (∀) n ∈ N şi a1 = a3 = a5 = ... = a ∈ N; a0 = a2 = a4 = ... = b ∈ N. Se obŃine 2ab = a +
+ b + 1 şi apoi (a, b) = (1, 2) sau (a, b) = (2, 1).

CLASA A XI-
XI-A

 π   π 
1. FuncŃia f: − , 0 → R, f(t) = 2t – tg t este strict crescătoare. Prin inducŃie, dacă xn ∈  − , 0  , atunci xn+1 =
 4   4 
 π   π 
= f(xn) ∈  − + 1, 0  ⊂  − , 0  . Rezultă că (xn) este corect definit, strict crescător şi mărginit, deci convergent.
 2   4 

386
1 ln( − x n )
 π π
Deoarece tg x = x, x ∈  − ,  , dacă şi numai dacă x = 0, obŃinem lim x n = 0 . Apoi, (− x n ) n = e n ,
 2 2 n →∞

x 2x − tgx n 2 x − tgx
n ≥ 1, iar ln(–xn+1) – ln(–xn) = ln n +1 = ln n . Cum lim = 1 , cu Stolz-Cesaro se deduce că
xn xn x → 0 x
'
f ' (x)  −1  1
− x n = 1 ; 2. Avem + 1 = 0 sau  
 f 2 ( x ) + x  = 0 , (∀) x ∈ I, deci − f 2 ( x ) + x = c , c constantă reală.
lim n
n →∞ f 3 (x)  
1
Din f(0) = 1, găsim c = –1 şi atunci 2
= x + 1 . Rezultă x + 1 > 0, deci x > –1. Cum 0 ∈ I, rezultă 0 ∈ I ⊂
f (x )
1  π 
⊂ (–1, ∞) şi f: I → R, f ( x ) = ; 3. Considerăm funcŃia h: [0, 1] → R; h ( x ) = 1 − sin x g( x ) . Aplicăm
x +1  2 
π π    π 
teorema lui Rolle funcŃiei h pe [0, 1], există c ∈ (0, 1) cu h'(c) = 0. ObŃinem cos c g (c) = 1 − sin  c   ⋅
2 2    2 
⋅ g'(c) < g'(c) = f(c).

Concursul de matematică RMCS, 7 martie 2010, OŃelu-Roşu

CLASA A IX-
IX-A

4 16   210  8


1. Din   = 1 ∈ M ⇒ 4 ∈ M,   = 4 ∈ M ⇒ 16 ∈ M, inductiv 4n ∈ M, n ∈ N. Din  =2 ⇒2 ,2 +
10 10

 
4  
4  4 
+ 1, ..., 211 – 1 ∈ M. Cum 210 < 2000 + 2010 < 211 − 1 ⇒ 2000 + 2010 ∈ M. Prin inducŃie: 22000 < 22000 + 244 <
1 1
< 22000 + 2 2010 < 211 − 1 = 2 2000 + 2 2000 − 1 , de unde 2 2000 + 2 2010 ∈ M; 2. A n = 1 − n ; B n = 1 − ;
2 n +1
1 1 1 1
c) + 2 + ... + n − 2 + n −1 = 1 , putem considera ak = 2k, k = 1, n − 1 şi an = 2n–1; 3. a) Avem 13 pătrate negre şi
2 2 2 2
12 albe, deci 13 ⋅ 12 = 156 perechi de pătrăŃele colorate diferit. Sunt 156 vectori; b) Grupăm vectorii în 78 de
vectori opuşi. Suma vectorilor este nulă; 4. Coliniaritatea a trei centre de greutate atrage coliniaritatea a trei
dintre punctele A, B, C, contradicŃie.

CLASA A X-
X-A

1. a) Fie log 3 ( x + 2) = log 2 (2 x + 7) = t > 1 . Rezultă 0 < 2t = 2x + 7 < 3t = x + 2, fals, deoarece 2x + 5 > x, x ∈ N;
b) Pentru y ≥ 3, 2y + 7 ≡ 7(mod 8) pe când restul împărŃirii lui 3x la 8 este 1 sau 3. Rezultă y ∈ {0, 1, 2}.
− 1 + 13
ObŃinem soluŃia x = 2, y = 1; 2. a) x = 0, x = ; b) Deoarece z3 – 8 = (z – 2)(z2 + 2z + 4), rezultă α3 =
4
= β 3 = 8. Folosind şi α + β = –2, αβ = 4, găsim A = –2 şi B = 25; 3. Cum f este strict monotonă rezultă că f este
injectivă. Pentru x = y = 1 obŃinem f(1) = 1. Pentru x = 1 rezultă f(f(y)) = y, (∀) y ∈ R. Dacă a ∈ R şi f(a) > a ⇒

387
⇒ f(f(a)) > f(a) ⇒ a > f(a), fals. Dacă a ∈ R şi f(a) < a ⇒ f(f(a)) < f(a) ⇒ a < f(a), fals. Rezultă că f(x) = x,
(∀) x ∈ R; 4. Presupunem că cei 5 jucători au jucat m partide cu jucători care au jucat până la sfârşit (avem m
partide în care exact un jucător părăseşte turneul). Aşadar m ≤ 10 (când oricare dintre cei 5 nu au jucat între ei) şi
m ≥ 5 (când cei cinci joacă între ei). Dacă n este numărul de jucători care termină turneul, avem C 2n + m = 200,
sau 190 ≤ C 2n ≤ 195 , de unde n = 20. IniŃial au fost 25 de jucători.

CLASA A XI-
XI-A

0 1
1. a) A =   ; b) Se verifică: (AB)2008 = O2 şi atunci: I2 – (AB)2008 = (I2 - AB)(I2 + AB + (AB)2 + ... +
 − 1 − 1 
1
+ (AB)2007), de unde (I2 – AB)–1 = I2 + AB + (AB)2 + ... + (AB)2007; 2. b) Se arată că lim a n = ∞ , cum bn = ,
n →∞ an
 0 −1 0 
1  
rezultă lim b n = 0 ; c) c n = 1 − , deci lim c n = 1 ; 3. a) De exemplu A =  − 1 0 0  ; b) Dacă A2 =
n →∞ a1a 2 ...a n n →∞
0 0 − 1

= O3, atunci din inegalitatea lui Sylvester: 0 = rang(A2) ≥ 2rang(A) – 3, deci rang(A) ≤ 1. Deoarece A ≠ O3,
rezultă rang(A) = 1. Din A2 = O3 avem că valorile proprii ale lui A sunt egale cu zero, deci Tr(A) = 0. Reciproc,
deoarece rang(A) = 1, există B ∈ M3,1(R) şi C ∈ M1,3(R) astfel încât A = BC şi CB = Tr(A) = 0. Deducem A2 =
= (BC)2 = O3; 4. Limita cerută este egală cu zero.

CLASA A XII-
XII-A

1. Deducem a3 = b3 = c3, deci f(a) = f(b) = f(c), de unde a = b = c; 2. Pentru x = e, rezultă f(f(y)) = yf(e), (∀) y ∈
∈ G, deci f este injectivă. Pentru y = 0 rezultă f(xf(x)) = f(x2), deci xf(x) = x2. Rezultă f(x) = x, x ∈ G. RelaŃia
dată devine x2y = yx2, (∀) x, y ∈ G, de unde x2n+2y = yx2n+2, (∀) x, y ∈ G şi cum x2n+1 = e, obŃinem xy = yx,
(∀) x, y ∈ G; 3. Aplicăm teorema lui Rolle funcŃiei h: [0, 1] → R, h(x) = (F(x) – G(x))(1 – x) pe [0, 1]; 4. g(m) =
 1 − 4m 2  1
 , m ∈  0,   1
=  m  2  . FuncŃia g nu este integrabilă pe 0, 2  nefiind mărginită.
  
0, m=0

Concursul de matematică „Petre Sergescu”, 26 martie 2010,


Drobeta-Turnu Severin

CLASA A IX-
IX-A

1. Notăm cu ma, mg şi mh respectiv media aritmetică, media geometrică şi media armonică a numerelor a, b, c.
Trebuie să demonstrăm că 3ma + mh ≥ 4mg. Aplicând inegalitatea mediilor pentru numerele ma, ma, ma, mh,

388
obŃinem 3m a + m h ≥ 44 m 3a m h . Cum ma ≥ mg, avem 44 m 3a m h ≥ 44 m g m a2 m h , deci este suficient să demon-

străm că 44 m g m a2 m h ≥ m g . Această inegalitate se scrie echivalent: m a2 m h ≥ m 3g ⇔


2
a+b+c 3abc
⇔  ⋅ ≥ abc ⇔ (a – b)2 + (b – c)2 + (c – a)2 ≥ 0, ceea ce este evident; 2. Analizând
 3  ab + bc + ca
cazurile x = 3k, x = 3k + 1, x = 3k + 2, k ∈ Z, deducem că ecuaŃia nu are soluŃii în Z; 3. Notăm rX vectorul de
rA + k rB r +r r +r
poziŃie al punctului X în raport cu un punct oarecare O. Avem: rM = ; rN = B C ; rP = C D ;
1+ k 1+ k 1+ k
r +r r +r r +r r +r r + r + k rB + k rD r + r + k rC + k rA
rQ = D A , rE = A C ; rF = B D ; rG = M P = A C ; rH = B D .
1+ k 2 2 2 2(k + 1) 2(k + 1)
k k k
Deducem EG = rG − rE = ⋅ (rB + rD − rA − rC ) = ⋅ EF şi HF = rF − rH = ⋅ EF . Prin urmare EG ,
2(k + 1) k +1 k +1
k
HF şi EF sunt coliniari (deci E, F, G, H sunt coliniare) şi EG = HF = EF .
k +1

CLASA A X-
X-A

f ( b ) − f (a )
1. EcuaŃia dreptei AB este y – f(a) = ( x − a ) . Punând condiŃia că punctul de coordonate (0, a2b + ab2)
b−a
f (a ) + a 3
este pe dreaptă, rezultă a(b3 + f(b)) = b(f(a) + a3) pentru orice a, b ∈ (0, ∞), deci = constant pentru
a
orice a ∈ (0, ∞). Rezultă f: (0, ∞) → (0, ∞), f(x) = –x3 + kx, k constantă reală; 2. a) Notăm log 22 x = t , x ∈
∈ (0, ∞). EcuaŃia devine 2t + t2 – 2t = 1. Cum funcŃia f: [0, ∞) → R, f(t) = 2t + t2 – 2t – 1 este convexă, ecuaŃia
f(t) = 0 are cel mult două soluŃii. Observăm că f(0) = 0 şi f(1) = 0, deci singurele soluŃii sunt t = 0 şi t = 1.
 1
( 2
) 2
Rezultă x ∈ 1, 2,  ; b) EcuaŃia este echivalentă cu 2 tg x + 1 = 4 cos x . Fie x soluŃie, atunci 4cos x =
 2

(
= 2 tg
2
x
)
2
+ 1 ≥ 22 = 4 ≥ 4cos x. Rezultă că inegalităŃile sunt egalităŃi, deci cos x = 1 şi atunci x ∈ {2kπ, k ∈ Z};
z − zB zB − zC
3. Dacă triunghiul este echilateral, relaŃia se verifică. Reciproc: Din relaŃia dată rezultă: A + +
c a
z − zA z − zB z − zC z − zA
+ C = 0 . Dacă M1M2M3 este triunghiul cu afixele z1 = A ; z2 = B ; z3 = C , atunci |z1| =
b c a b
z −z z 2π
= |z2| = |z3| = 1 şi z1 + z2 + z3 = 0, deci ∆M1M2M3 este echilateral. Din arg B C = arg 2 = rezultă că
zA − zB z1 3
m('CBA) = 60° sau 120°. Analog pentru celelalte unghiuri ale ∆ABC. Rezultă că ∆ABC este echilateral.

CLASA A XI-
XI-A

1. a) Şirul este monoton descrescător şi mărginit; an ∈ (0, 1), deci este convergent. lim a n = 0 ; b) Folosind
n →∞

389
0 n 1 a k (1 − a kn ) k 1 xk
Stolz, cazul , lim na kn = lim = lim = lim n = , (deoarece lim =
0 n →∞ n →∞ 1 n→∞ 1 1 n → ∞ 1 − (1 − a k ) k k x → 0 1 − (1 − x k ) k
− n
a kn a kn +1 a kn
1 1
= ). Rezultă că limita cerută este egală cu ; 2. Fie x0 ∈ R, arbitrar fixat. Cum f este crescătoare, rezultă că
k k
k
 1  1
există lim f ( x ) = a şi lim f ( x ) = b şi a ≤ f(x0) ≤ b. Dar a = lim f  x 0 −  = lim f  x 0 +  = b , rezultă a = b =
x↑x0 x↓x0 n→∞  n  n →∞  n
= f(x0), deci f este continuă în x0, deci pe R; 3. a) Considerăm liniile i1 ≠ i2 şi coloanele j1 ≠ j2. Fie σ o permutare
cu σ(i1) = j1 şi σ(i2) = j2 şi τ o permutare cu τ(i1) = j2 şi τ(i2) = j1 şi σ(k) = τ(k) în rest. Rezultă a i1 j ⋅ a i1 j 2 − a i1 j 2 ⋅
1

⋅ a i 2 j 2 = 0 , adică toŃi minorii de ordin doi sunt nuli, rezultă că matricea dată are rangul egal cu 1; b) Fie f(k)
numărul matricelor de tip k × n, cu toate liniile nenule, care au rangul 1. Atunci rezultatul cerut este: N =
= C1n f (1) + C 2n f (2) + ... + C nn f (n ) ; f(k) = g(k) + h(k), unde g(k) este numărul matricelor de tip k × n, de rang 1,
care au pe primul rând 0, 1, 2 şi h(k) este numărul matricelor de tip k × n, de rang 1, care au pe primul rând 0, 1
sau 0, 2; g(k) = 3n – (2n – 1) – (2n – 1) – 1 (numărul de aşezări pe primul rând ale numerelor 0, 1, 2) – (numărul
de aşezări ale numerelor 0, 1) – (numărul de aşezări ale numerelor 0, 2) – toate 0) (celelalte rânduri trebuie să fie
la fel, pentru ca rang = 1) ⇒ g(k) = 3n – 2n+1 + 1; h(k) = [(2n – 1) + (2n – 1)] ⋅ 2k–1 (numărul aşezări pe primul
rând ale numerelor 0, 1 + numărul de aşezări pe primul rând ale numerelor 0, 2 şi apoi rândurile pot alterna, de
n
aceea înmulŃim cu 2k–1; N = ∑C
k =1
k n
n (3 − 2 n +1 + 1 + 2 n + k − 2 k ) = (3n − 2 n +1 + 1)(2 n − 1) + (2 n − 1)(3n − 1) = (2n –

– 1) ⋅ (3n – 2n+1 + 1 + 3n – 1) = 2(3n – 2n)(2n – 1).

CLASA A XII-
XII-A

 x y
1. b) FuncŃia f: (C, +, ⋅) → (M, +, ⋅), f ( x + iy) =   ; c) Folosind b) şi notând f(A) = z, z = x + iy, obŃinem
− y x
−3± 5
z4 + 3z3 + 2z2 + 3z + 1 = 0 cu soluŃiile z1, 2 = , z 3,4 = ±i şi corespunzător A = f–1(z); 2. I =
2
π
1 π
= ∫
π − sin( 2n
3
2 − 1)xdx = −
2n − 1
cos(2n − 1) ; 3. Pentru x = 0, u(0) = |a|. Ridicând la pătrat şi derivând relaŃia
3

găsită, rezultă u'(x) – u(x) = 0, pentru orice x, de unde (e − x u ( x ))' = 0 , prin urmare u(x) = kex, x ∈ R. ObŃinem
funcŃiile u(x) = aex pentru orice x ∈ R sau u(x) = –aex, pentru orice x ∈ R.

Concursul de matematică „Chindia”, 9 mai 2010, Târgovişte

CLASA A IX-
IX-A
1 5
1. Din ∆ = p2 – 4p ≥ 0 rezultă p ∈ (–∞, 0] ∪ [4, ∞) iar inegalitatea de demonstrat devine: |p – 2| + ≥ .
|p−2| 2
Cu substituŃia t = |p – 2| ∈ [2, ∞), aceasta este echivalentă cu (t – 2)(2t – 1) ≥ 0, adevărată în condiŃiile date;

390
mx + n + p(− x + 1), x < 1 ax + b, x < 1
2. f(x) = mx + n + p|x – 1| =  =  dacă şi numai dacă m – p = a; n + p = b;
mx + n + p( x − 1), x ≥ 1 cx + d, x ≥ 1
a+c b+d c−a b−d
m + p = c; n – p = d, dacă şi numai dacă m = ;n = ;p = = . Prin urmare există m, n, p în
2 2 2 2
     
condiŃiile date dacă şi numai dacă c + d = a + b; 3. Din relaŃia dată rezultă 4AD ⋅ BC + 19BE ⋅ BC + 25CF ⋅ BC = 0
deci 0 + 19BE ⋅ BC ⋅ cos(90° – C) – 25 ⋅ CF ⋅ BC ⋅ cos(90° – B) = 0, de unde 19c2 – 25b2 = 0. Analog 4c2 –
c 19 2c 1
– 25a2 = 0, deci b = şi a = . Găsim cos B = , deci m('B) = 60°.
5 5 2

CLASA A X-
X-A

1 1
1. log 22 (2x + y) + log 22 (2 y + z) + log 22 (2z + x ) ≥ [log 2 (2x + y) + log 2 (2 y + z) + log 2 (2z + x )]2 ≥ log 22 (2 2xy ⋅
3 3
1
⋅ 2 2 yz ⋅ 2 2zx ) = ; 2. z = x + iy ∈ A ⇔ y ≠ 0 şi x2 – y2 + ax + i(2xy + y) ∈ R ⇔ 2xy + y = 0, y ≠ 0 ⇔ x =
12
a  a   b−c 
= − . Prin urmare A = z ∈ C - R | z = − + yi . Analog B = z ∈ C - R | z = + yi şi A = B ⇔ a + b = c;
2  2   2 
1 1 1 2 2 n−2
3. a) Inegalitatea este echivalentă cu: 0 ≤ 2 + 3 + ... + n − 2 ≤ şi cum C n = C n este cel mai mare dintre
Cn Cn Cn n
1 1 1 1 2(n − 3) 2
numerele C 2n , C3n , ..., C nn − 2 , n ≥ 5, rezultă + + ... + n − 2 ≤ (n − 3) 2 = < ; b) Inegalitatea este
C 2n C3n Cn C n n (n − 1) n
1 1 1 1 2 1 1 1
echivalentă cu: 0 ≤ 2 2
+ 3 2 + ... + n − 2 2 ≤ 2 + − 2 . Avem: 2 2
+ 3 2 + ... + n − 2 2 ≤ (n –
(C n ) (C n ) (C n ) n n (C n ) (C n ) (C n )
1 4(n − 3) 1 2
– 3) ⋅ = < 2 + − 2 , n ≥ 5.
(C 2n ) 2 n 2 (n − 1) 2 n n

CLASA A XI-
XI-A

1. Dacă detA ≠ 0 sau detB ≠ 0, atunci din ABAB = O2, rezultă BAB = O2 sau ABA = O2. Dacă AB = O2 sau A =
= O2 sau B = O2, atunci ABA = BAB = O2. Presupunem că detA = 0, detB = 0, AB ≠ O2, A ≠ O2, B ≠ O2. Fie
 x y a b x z  ky y 
A =  ; B =   ; xt = zy; ad = bc. Dacă y, t ≠ 0, atunci = = k , deci A =   . Dacă b, d ≠ 0,
z t  c d y t  kt t 
a c  pb b 
atunci = = p , deci B =   . Deoarece (AB)2 = O2, rezultă det(AB) = 0 şi tr(AB) = 0, deci AB =
b d  pd d 
 u 2   2 
  ky y  pb b   u u  u2
= u v  . Din     =
 kt t  pd d   v  obŃinem pyα = u; yα = v ; ptα = –u, tα = –u, unde α =
− v − u    − v − u
   
u u2
= bk + d ≠ 0 (altfel AB = O2). Deci t = − ,y = . Prin calcul, ABA = O2. Analog tratăm celelalte cazuri;
α αv

391
 1 1 
 nk nk − 1 1 
x
a −1 3 − 1 1 5 1
2. Avem limita fundamentală lim = ln a , a > 0. Astfel a k = lim  ⋅ + ⋅  = (ln 3 +
x →0 x n → ∞ 1 k 1 k k
 
 nk nk 
1  3a k − 1 5a k − 1 
+ ln 5) = ln 15 şi L = lim  ⋅ ka k + ⋅ ka k  = ln 15(ln 3 + ln 5) = ln 2 15 ; 3. Pentru fiecare x ∈ R*,
k k →∞  
 ak ak 
fixat, ecuaŃia t – et = x – ex are două soluŃii, t1 ∈ (–∞, 0) şi t2 ∈ (0, ∞). Pentru x = 0, ecuaŃia are soluŃie unică t =
t1 ( x ), x ∈ A

= 0. Dacă A, B ⊂ R*, A ∪ B = R*, A ∩ B = ∅, definim f: R → R, f ( x ) = 0, x = 0 , care verifică ecuaŃia
t ( x ), x ∈ B
2
dată. Cum A poate fi aleasă într-o infinitate de moduri, există o infinitate de soluŃii.

CLASA A XII-
XII-A

1. n = 2k3, k ∈ Q*; 2. Vom arăta că f(x) = F(x), pentru orice x ∈ R. Fie că există x0 ∈ R, f(x0) ≠ F(x0).
Presupunem că f(x0) < F(x0). Fie ε > 0, pe care îl vom preciza. A = f(x0 – ε, x0 + ε) = {f(t), t ∈ (x0 – ε, x0 + ε};
B = F(x0 – ε, x0 + ε) = {F(t), t ∈ (x0 – ε, x0 + ε}. Cum f este continuă pentru ε > 0, există δ1 > 0 astfel încât
|x – x0| < δ1 ⇒ |f(x) – f(x0)| < ε, deci x ∈ (x0 – δ1, x0 + δ1) ⇒ f(x) ∈ (f(x0) – ε, f(x0) + ε). La fel din F continuă,
există δ2 > 0 astfel încât x ∈ (x0 – δ2, x0 + δ2) ⇒ F(x) ∈ (F(x0) – ε, F(x0) + ε). Alegem ε astfel încât f(x0) + ε <
F( x 0 ) − f ( x 0 )
< F(x0) – ε, deci 0 < ε < . Rezultă că pentru acest ε şi x ∈(x0 – δ1, x0 + δ1) ∩ (x0 – δ2, x0 + δ2),
2
A ∩ B = ∅, contradicŃie. Prin urmare f(x) = F(x), (∀) x ∈ R. Rezultă (F(x)e–x)' = 0, (∀) x ∈ R, deci F(x) = kex, k
sin x 2 cos x 2  1 1 
constantă reală fixată şi atunci f(x) = kex, x ∈ R; 3. Avem + ≤ (sin 2 x 2 + cos 2 x 2 ) 2 + 2  ,
a b a b 
π π
 sin x 2 cos x 2  1 1 π  1 1  π π
deci ∫ 0
2 
 a + b dx ≤



+
a 2 b2
⋅ ∫
0
2 dx = ⋅ +  . Din
2  a 2 b2  2
≥ x ≥ 0, rezultă:
2

π
π π
 sin x 2 cos x 2   sin x 2 cos x 2   2 2  2
11 1
dx = 1  sin x − cos x 
⋅ ∫
0
2 
 a + b dx ≥



∫0
2 x
 a
+
b 
 2  b
 a 
0
=  + .
2a b
π
 sin x 2 cos x 2  1 1 1
Deci ∫
0
2 
 a + b dx ≥



 + .
2π  a b 

392
Concursul interjudeŃean de matematică Memorialul „Traian Lalescu”,
EdiŃia a XXIV-a, 26-28 martie 2010, Timişoara

CLASA A IX-
IX-A

1. a) Din f(m) – m = f(n) – n = k, (∀) m, n ∈ A, rezultă f(n) = n + k, unde k ∈ N* (deoarece f ≠ 1A). Fie A finită
şi a = minA, b = maxA. Din f(a) = a + k ≥ 0 rezultă k ≥ 0, iar din f(b) = b + k ≤ b, rezultă k ≤ 0. Prin urmare k =
= 0, contradicŃie; b) Nu. Pentru orice A finită definim 1A: A → A, 1A(n) = n, n ∈ A, care verifică cerinŃele
2x  2  2x
problemei; 2. Deoarece 2 ∈ − , 2 , pentru x ∈ R, rezultă ∈ {–2, –1, 0, 1, 2}, de unde x ∈ {–2,
x − x + 1  3  2−x
2  2
0, ,1 . Prin verificare, găsim soluŃiile x = –2; 0; ; 1; 3. a) Notăm cu k valoarea comună a rapoartelor date.
3  3
1 k
ObŃinem SM = SA + SB şi analoagele. Dacă G este centrul de greutate al triunghiului ABC, rezultă
1+ k 1+ k
1 k
GM + GN + GP = 0 şi concluzia se impune; b) Avem: SM ≤ SA + SB (deoarece k ∈ (0, 1)). Rezultă
1+ k 1+ k
1 k2 2k 1 k2 k
SM 2 ≤ 2
SA 2 + 2
SB 2 + 2
SA ⋅ SB ≤ 2
SA 2 + SB 2 + (SA 2 + SB 2 ) .
(k + 1) (1 + k ) (1 + k ) (1 + k ) (1 + k ) 2 (1 + k ) 2
 1 k 2 
∑ SM ∑ SA = ∑ SA
2k
Rezultă 2
≤  + + 2 2
; 4. b) An ∩ A2010 = {–1, 1}; c) Alegem
 (1 + k )
2
(1 + k ) 2 (1 + k ) 2 
z0 ∈ (–1, 1) ∩ A2010. Rezultă z 0m ∈ A2010, (∀) m ∈ N, deci A2010 este infinită.

CLASA A X-
X-A

1. Fie f(0) = t. Pentru n = 0, rezultă f(m) + t = f(f(m)), (∀) m ∈ N. Deoarece f este surjectivă pentru orice p ∈ N
există mp ∈ N astfel încât f(mp) = p. Pentru m = mp, obŃinem f(p) = p + t, pentru orice p. Din surjectivitatea lui f
rezultă t = 0 (altfel f(p) > 0, (∀) p ∈ N). Astfel unica soluŃie este f: N → N, f(n) = n; 2. Fie x1 cel mai mare dintre
n
numerele x1, x2, ..., xn. Avem: ( x 2 + x 3 + ... + x n ) 2 = ∑x
i=2
2
i +2 ∑x x
2 ≤ i < j≤ n
i j . Cum x i2 ≤ x1x i < 2x1x i , rezultă

∑x x ∑x x ∑x x
2
+8y
( x 2 + x 3 + ... + x n ) 2 ≤ 2 1 i +2 i j =2 i j = 2 , de unde x 2 + x 3 + ... + x n ≤ 2 ; 3. a) a 8 x +
i=2 2 ≤ i < j≤ n 1≤ i < j≤ n
2 2 2
+ 8x
+ a 8y ≥ 2a 4 x + 4 y ⋅ a 4 y + 4 x ≥ 2a–2, deoarece 4x2 + 4y2 + 4x + 4y ≥ –2, (∀) x, y ∈ R, cu egalitate pentru x =
1 1
= y = − . Rezultă că toate inegalităŃile sunt egalităŃi. Prin urmare ecuaŃia are soluŃie unică x = y = − ;
2 2
8x 2 + 8 y 8y 2 +8y (8 x 2 + 8 y ) ln a (8 y 2 + 8 x ) ln b ( 4 x 2 + 4 y ) ln a + ( 4 y 2 + 4 x ) ln b 2
b) a +b =e +e ≥ 2e ≥ log a log b (deoarece (4x +
2

a b ⋅b a
2 2
ln b ln a
+ 4y)ln a + (4y2 + 4x)ln b ≥ − − ). Egalitatea se realizează dacă (9x2 + 8y)ln a = (8y2 + 8x)ln b şi
ln a ln b

393
ln b ln a AM
2 x ln a + = 2 y ln b + . ObŃinem a = b. În concluzie nu sunt soluŃii pentru a ≠ b; 4. a) Fie =
ln a ln b AD
BN
= α, = 1 – α, atunci zM = (1 – α)zA + αzD şi zN = αzB + (1 – α)zC. Deducem |zM – zN| ≤ (1 – α) ⋅ |zA – zC| +
BC
BN AM z + zN
+ α|zB – zD| şi de aici MN ≤ ⋅ AC + ⋅ BD , care este echivalentă cu relaŃia din enunŃ; b) Din M =
BC AD 2
z + zC z + zD
= (1 – α) A +α B rezultă concluzia.
2 2

CLASA A XI-
XI-A

1. (xn) este monoton crescător. Dacă ar fi mărginit superior, ar fi convergent. Din l = lim x n ⇒ l = l + l , fals.
n →∞

x n +1  1  x
Prin urmare lim x n = ∞; lim n x n = lim = lim 1 + = 1 . Folosind Lema Stolz-Cesaro, lim n2 =
n →∞ n →∞ n →∞ x n n → ∞ 
xn  n → ∞ n

x n +1 − x n xn x n +1 − x n 1 xn 1
= lim = lim = lim = lim = ; 2. Notăm Sn = x1 + x2 + ... + xn.
n →∞ 2n + 1 n → ∞ 2n + 1 n →∞ 2 2 n → ∞ x n +1 + x n 4
n n n −1
Prin inducŃie se arată că Sn > n, n ∈ N*. Rezultă 0 < xn+1 = < 1 . De asemenea xn+1 – xn = − =
Sn Sn Sn −1
nSn −1 − (n − 1)Sn Sn −1 − (n − 1) x n n − 1  Sn −1  n − 1 1 − x 2n
= = = ⋅ − xn  = ⋅ > 0 . Rezultă că (xn) este con-
Sn Sn −1 Sn Sn −1 Sn Sn −1  n − 1  Sn Sn −1 x n
n +1− n 1
vergent şi l = lim x n ∈ (0, 1]. Folosind Lema Stolz-Cesaro, lim x n = lim = . Rezultă l = 1.
n →∞ Sn +1 − Sn l
n →∞ n →∞

3. Din A–1 + B–1 = In ⇒ In + AB–1 = A şi B + A = AB, de aici (In – A)(In – B) = In. Rezultă (In – A) este
inversabilă cu inversa In – B, deci (In – B)(In – A) = In sau încă A + B = BA. Prin urmare AB = BA. Deoarece
AB = BA, obŃinem In – A3 – B3 – (AB)3 = In – A3 – B3 – A3B3 = (In – A3)(In – B3) = (In – A)(In – B) ⋅ (In + A +
+A2) ⋅ (In + B + B2) = (In + A + A2) ⋅ (In + B + B2). Cum det(In + X + X2) ≥ 0, rezultă concluzia; 4. detA = lg 300 ⋅
4
 25 4 
25 4  lg 300 + lg + lg + lg 3 
⋅ lg ⋅ ⋅ lg ⋅ lg 3 <  3 3  =1.
3 3  4 
 
 

CLASA A XII-
XII-A

0 1 + tg 2 x x x
1. Facem substituŃia t = x – y, rezultă f ( x ) = − ln ∫ x 1 + tgx + tgy
dy = ∫
0
ln(1 + tg 2 x )dx − ∫ ln(1 + tgx + tgy)dy ,
0

π π f (x)
deci 2f(x) = xln(1 + tg2 x) = –2xln cos x. Prin urmare f(x) = –xln cos x. Rezultă f   = ln 2 şi lim 3 =
4 8 x → 0 x
1
= ; 2. f este bijectivă. Rezultă că ϕ = f şi este derivabilă. Cu schimbarea de variabilă x = f(y), I =
–1
2

394
e f ' ( y)
= ∫
1 1+ y
dy = 1; 3. a) Pentru x = y = 0 rezultă f(0) = 0 iar y = –x. Conduce la f(–x) = f(x). Din ipoteză f(x + y)

sau f(x – y) este mai mic decât f(x) + f(y). Prin inducŃie rezultă concluzia; 4. a) Fie g ∈ G, fixat şi B = { ga −2 1 ,
a2 ∈ A}. Se verifică uşor că cardB = cardA (gx–1 = gy–1, x, y ∈ A ⊂ G ⇒ x = y). Dar din ipoteză A ∩ B ≠ ∅,
deci există a1 ∈ A ∩ B. Din a1 = ga −2 1 , rezultă g = a1a2; b) Considerăm C = {x2 | x ∈ K}, aceasta conŃine cel
1
puŃin un element din perechile (x, –x), x ∈ K* şi 0, de unde cardC > cardK . Folosind a) rezultă concluzia.
2

Concursul interjudeŃean de matematică „SperanŃe râmnicene”,


4 aprilie 2010, Râmnicu Sărat

CLASA
CLASA A IX-
IX-A

1. Se impune x ≥ 0, y ≥ 0. Fie x = a8, y = b8, a, b ≥ 0. Sistemul devine a6 – b6 = 63, a7 + b7 = 129. Dacă a > 2,
atunci b6 = a6 – 63 > 1, deci b > 1 şi b7 = 129 – a7 < 1, deci b < 1, contradicŃie. Analog nu se poate a < 2. Rezultă
a = 2, b = 1. Prin urmare x = 256, y = 1; 2. Din SMP1P2 + SMP2 P3 + ... + SMPn P1 = SP1P2 ...Pn = S , rezultă d1 + d2 +
2S
+ ... + dn = . Din inegalitatea (CBS) rezultă n (d12 + d 22 + ... + d 2n ) ≥ (d1 + d 2 + ... + d n ) 2 , cu egalitate pentru
a
4S2
d1 = d2 = ... = dn. Prin urmare min(d12 + d 22 + ... + d 2n ) = 2 , obŃinută penru d1 = d2 = ... = dn, deci M este centrul
na
cercului înscris poligonului; 3. Deoarece ∆ = 4(a2 + b2 + c2 – ab – ac – bc) ≥ 0, ecuaŃia are rădăcini reale. Notăm
f(x) = x2 – 2 x a 2 + b 2 + c 2 + ab + bc + ac . Cum f(0) ⋅ f(1) < 0, rezultă cerinŃa problemei.

CLASA
CLASA A X-
X-A

1. Din |z – zk| = |zk|, rezultă (z − z k )( z − z k ) = z k ⋅ z k , deci | z |2 −z ⋅ z k − z ⋅ z k = 0 , k = 1, 4 . Sumând aceste


relaŃii, avem 4 | z |2 −z(z1 + z 2 + z 3 + z 4 ) − z(z1 + z 2 + z 3 + z 4 ) = 0 , deci 4 | z |2 −z ⋅ z − z ⋅ z = 0 , rezultă 2|z|2 = 0,
de unde z = 0; b) Fie A, B, C, D punctele de afixe z1, z2, z3, z4. A, B, C, D sunt situate pe cercul C(O, R), R = |zk|,
k = 1, 4 . Putem presupune că A, B, C, D sunt în această ordine pe cerc. Considerăm M, N, P, Q mijloacele
laturilor (AB), (BC), (CD) respectiv (DA). Din z1 + z2 + z3 + z4 = 0 rezultă că centrul paralelogramului MNPQ
este O, deducem OM ⊥ AB, OP ⊥ CD, deci AB || CD. La fel BC || AD. Prin urmare ABCD este paralelogram
inscriptibil, deci dreptunghi; 2. Arătăm că log a (a + 1) < log a −1 a , pentru orice a ∈ (2, ∞). Avem log a (a + 1) ⋅
2 2
 log a (a + 1) + log a (a − 1) 
2
 log (a 2 − 1)   log a a 2 
⋅ log a (a − 1) ≤   =  a  ≤   = 1 , deci loga (a + 1) < loga–1 a, a > 2;
 2   2   2 
2 1
3. Se impune x ∉ [0, 1). Pentru x ≥ 1, [x] ≥ 1 şi [x] + > [ x ] ≥ 1 ≥ , deci nu sunt soluŃii în intervalul [1, ∞).
x [x]
2 1
Pentru x < 0, [x] ≤ –1 şi [ x ] + < [ x ] ≤ −1 ≤ , prin urmare ecuaŃia nu are soluŃii nici în intervalul (–∞, 0).
x [x ]
EcuaŃia nu are soluŃii.

395
CLASA A XI-
XI-A

1. Fie x0 ∈ R. Cum f este monotonă, aceasta are limite laterale în fiecare punct, fie ls = lim f ( x ) şi
x→x0
x<x0

 1  1
lim f ( x ) = = ld. Avem ls ≤ f(x) ≤ ld. Dar ls = lim f  x 0 −  = lim f  x 0 +  = l d . Deducem ls = ld = f(x0), prin
x →x 0 n →∞  n  n →∞  n
x>x0
urmare f este continuă în x0 ∈ R, deci pe R; 2. Considerăm funcŃia continuă f: (0, ∞) → R, f(x) = 1 +
+ c + (log a x ) 2 ⋅ log a x + bx (1 + c + b 2 x 2 ) . Cum f este strict monotonă (deoarece f'(x) > 0, x > 0) şi
lim f ( x ) = –∞ şi f(1) > 0, rezultă concluzia; 3. Folosind binomul lui Newton, (A + B)m+p–1 = On. Rezultă In = In –
x →0
x >0
– (A + B)m+p–1 = (In – A – B)(In + (A + B) + (A + B)2 + ... + (A + B)m+p–2) şi concluzia se impune.

CLASA A XII-
XII-A

x n −1
n n
1  1 2 2
1. a) Pentru x ∈  ,1 , n
≤ <   . Rezultă 0 ≤ nIn < n ⋅   şi de aici lim nI n = 0 ; b) Avem
 2  ( x + 1) ( x + 1) n  3  3 n →∞

1 1 1 1 1 1 
2
∫ 2

kI k = 1 ( x k )' ( x + 1) − k dx = x k (1 + x ) − k 1 + k 1 x k ( x + 1) − k −1 dx . Rezultă Ik =
2
 k − k  + I k +1 şi atunci
k2 3 
n n
1 1  4 x5 + x
∑ ∑ (I
1
 k − k
k =1  2
k 3
=
 k =1
k − I k +1 ) = I1 − I n +1 . Rezultă că limita cerută este egală cu I1 = ln
3
; 2. ∫ x3 + 1
dx =

x3 1 1 1 1 2x − 1
= − ln(x 3 + 1) − ln(x + 1) + ln(x 2 − x + 1) + arctg + C; 3. a) Din x1, x2, ..., xn = (λx1)(λx2)...
3 3 3 6 3 3
(λxn), xk ≠ 0, rezultă λn = 1; b) Fie Sk = ∑ x x ...x
1 2 k , k = 1, ..., n – 1, atunci Sk = λk Sk şi cum λk ≠ 1 pentru
k = 1, n − 1 , rezultă Sk = 0. Astfel P(x) = a 0 ( x n − S1x n −1 + S2 x n − 2 + ... + (−1) n Sn ) = a 0 ( x n + (−1) n x1x 2 ...x n ) =
= axn + b, a, b ∈ C*.

396
Concursul NaŃional de Matematică aplicată „Adolf Haimovici”,
Etapa judeŃeană, 13 martie 2010
Filiera tehnologică: profilul servicii, resurse naturale şi protecŃia mediului

CLASA A IX-
IX-A

1. Dacă x, y, z reprezintă numărul de cutii cu 6, 9 respectiv 20 bucăŃi, atunci: a) 6x + 9y + 20z = 33, cu soluŃiile
 4
(x, y, z) ∈ {(1, 3, 0); (4, 1, 0)}; b) 6x + 9y + 20z = 43, nu are soluŃii; 2. S = 45 r +  ≥ 180 (am folosit x + y ≥
 r
≥ 2 xy , x, y ≥ 0), deci Smin = 180 pentru r = 2, a1 = 9; 3. a2 + b2 + c2 = (x – 2y)2 + (y – 2z)2 + (z – 2x)2, de unde
5(x2 + y2 + z2) – 4(xy + yz + zx) = 1. Cum x2 + y2 + z2 ≥ xy + yz + zx, pentru orice x, y, z ∈ R, rezultă 5(x2 +
+ y2 + z2) = 1 + 4(xy + yz + zx) ≤ 1 + 4(x2 + y2 + z2), de unde x2 + y2 + z2 ≤ 1; 4. a) AM = AM = bc ; b) MD =

= MA + AD = −bAB + bAB + c AC = cAC = AN ; c) AMDN este romb, deci (AD este bisectoarea 'BAC.

CLASA A X-
X-A

1. Fie x, y, z numărul jetoanelor pe care sunt scrise numerele 5, 7, respectiv 11; a) 5x + 7y + 11z ≠ 13, x, y, z ∈
∈ N, deci 13 nu este norocos; b) 14 = 2 ⋅ 7; 15 = 3 ⋅ 5; 16 = 1 ⋅ 5 + 1 ⋅ 11; 17 = 2 ⋅ 5 + 1 ⋅ 7; 18 = 1 ⋅ 7 + 1 ⋅ 11,
deci 14, 15, 16, 17, 18 sunt norocoase; c) Dacă n este norocos, atunci n + 5 este norocos şi folosim inducŃia
1 1 1 1 1 1 1 1
matematică; 2. = log A (ab) şi analoagele, rezultă + + = logA (abc) 2 = logA (abc) = + +
α 2 α β γ 2 loga A logb A
1
+ ; 3. a) Din 7 − 2 x − x > 0 rezultă x ∈ [0,9 – 4 2 ) ; b) Din x ∈ [0, 9 − 4 2 ) şi x ∈ Z rezultă x ∈ {0, 1,
log c A
2, 3}. Convine x = 3 şi găsim punctul A(3, 2); 4. a) Inegalitatea este echivalentă cu 2 + 2|cosx| ≤ 4; b) Conform
a) rezultă 2x + 2–x ≤ 2, de unde (2x – 1)2 ≤ 0, deci x = 0, care verifică ecuaŃia dată.

CLASA A XI-
XI-A

a b
1. Fie A =   ∈ M2(R). Din condiŃiile date, ad – bc – 3(a + d) = –5 şi ad – bc + 2(a + d) = 5, de unde ad –
c d
– bc = 1, a + d = 2. Cum A2 – (a + d)A + (ad – bc)I2 = O2, rezultă A2 – 2A + I2 = O2; 2. Determinantul are n2
elemente, dintre care n2 – n + 2 sunt egale cu α, deci cel mult n – 2 sunt diferite de α. Rezultă că există două linii
1
(cel puŃin) cu toate elementele egale; 3. a) f (0 − 0) = − ; f(0 + 0) = 0; f(1 – 0) = ∞; f(1 + 0) = –∞; b) Pentru x
2

397
suficient de mare [e–x] = 0, deci l1 = 0; deoarece ex – 1 < [ex] ≤ ex, rezultă 1 – e–x < e–x[ex] ≤ 1, rezultă l2 = 1;
x2 +1 P( x )
4. Deoarece lim = 1 , rezultă lim 2 = 1 , de unde n = m + 2 şi a0 = b0, pentru aceste valori
x →∞ x x → ∞ x Q( x )
x2 +1 P( x )
obŃinem –1 = lim = lim 2 = 1 , contradicŃie.
x → −∞ x x → −∞ x Q( x )

CLASA A XII-
XII-A

1. b) x & x & ... & x = (x + 1)n – 1, n ∈ N*, obŃinem ecuaŃia (x + 1)n = 2, deci x = n 2 − 1 ; c) Dacă x, y ∈ H, x =
= a2 – 1, y = b2 – 1, a, b ∈ Q*, rezultă x & y = (x + 1)(y + 1) – 1 = (ab)2 – 1 ∈ H şi dacă x ∈ H, x = a2 – 1, a ∈
2
x 1− a2  1 
∈ Q*, atunci x ' = − = =   − 1 ∈ H. Rezultă că (H, &) este subgrup al lui (G, &); 2. b) Din a), avem
x +1 a2 a
1 1
1 π 1 1 1

1
arctgt x = arctgt 1x , de unde arctgx + arctg = , x > 0; c) Cu substituŃia x = rezultă I(a) = − a arctg dt =
x 2 t a t t
a
a1π  π π 2 2x − 1 2 2x − 1
= ∫
1
a
 − arctgt dt = ln t − I(a ) şi de aici I(a ) = ln a ; 3. a) I = 2
t2  2 1 2 ∫
0 ( x 2 − x + 1) 2
dx + ∫ (x
0 2
− x + 1) 3
dx =
a
2 1
 2 1  16 x+
=  − 2 − 2
 = ; b) I( x ) = xe x + C ; 4. Elevul poate colora toate punctele de coordo-

 x − x + 1 2( x − x + 1)  0 9
nate întregi situate pe dreptele x = 0, x = 1, y = 0 şi y = 1. Considerând punctele Mk(k, 0) şi Nk(k, 1), k ∈ Z,
elevul poate colora toate punctele de ordonată întreagă de pe dreptele x = k, k ∈ Z.

Concursul NaŃional de Matematică aplicată „Adolf Haimovici”,


Etapa naŃională, 22 mai 2010
Filiera tehnologică: profilul servicii, resurse naturale şi protecŃia mediului

CLASA A IX-
IX-A

1. Forma liniei frânte se repetă la fiecare 6 segmente, o secvenŃă are 12 cm. Sunt 334 secvenŃe de câte 12 cm şi o
b
secvenŃă de 11 cm. Lungimea liniei frânte este 4019 cm; 2. a) b n +1 = n , deci (bn) este progresie geometrică cu
3
n −1 n −1
1 1 n 2 1 n (n + 1)
b1 = 2 şi q = ; b) b n = 2 ⋅   , n ≥ 1. Rezultă a n = + ⋅  , n ≥ 1; c) 3Sn − = 3−
3 3 3 3 3 2
n −1
1 hx (a − x ) ah 1
−  < 3 ; 3. a) Din ∆AMN ~ ∆ABC rezultă hx + ay = ah; b) SMNPQ = ≤ = SABC (am folosit
3 a 4 2
a2 
x (a − x ) ≤  ; c) Triunghiul de laturi AB = 30 cm, AC = 40 cm şi BC = 50 cm este dreptunghic în A. Avem
4 

398
două posibilităŃi: 1) MN || BC, P, Q ∈ BC, MN = 25 cm; NP = 12 cm; 2) MN || AC, P ∈ AC, Q = A, MN =
3
= 20 cm, NP = 15 cm; 4. Fie A(x0, k) ∈ Gf, x0 ∈ Q –   , k ∈ Q, rezultă x 02 − 2x 0 (1 + k ) + 2 + 3k = 0 , deci ∆ =
2
= 4k2 – 4k – 4 = (2k – 1)2 – 5 = p2, p ∈ Z. Din (2k – 1 – p)(2k – 1 + p) = 5 găsim k = –1 sau k = 2. Pentru k = –1
rezultă x0 = ±1 iar pentru k = 2, rezultă x0 = 2 sau x0 = 4. Punctele căutate sunt (–1, –1); (1, –1); (2, 2); (4, 2).

CLASA A X-
X-A

1. a) x = 4; b) x = –1 sau x = 2; 2. a) f1, f2: R → R, f1(x) = 2x, f2(x) = x sunt strict crescătoare, deci f = f1 + f2 este
strict crescătoare; b) RelaŃia este echivalentă cu f(a) = f(b) şi cum f este injectivă, rezultă a = b; c) EcuaŃia este
2 2
echivalentă cu 2sin x − 2cos x = cos 2 x − sin 2 x , conform b) obŃinem cos2 x = sin2 x care are soluŃiile x ∈
 π kπ 
∈ + , k ∈ Z  ; 3. Capra poate să pască: o jumătate de cerc de rază 2 m; un sfert de cerc de 6 m; un sfert de
4 2 
cerc de 4 m şi un sfert de cerc de 2 m, adică o suprafaŃă egală cu 16π m2; 4. a) Observăm că un număr este bun
dacă şi numai dacă este impar, multiplu de 3; b) Dacă a = 3(2k + 1); b = 3(2m + 1), m, k ∈ N* sunt bune, atunci
a ⋅ b = 9(4km + 2k + 2m + 1) este bun (fiind impar, divizibil cu 3); c) Din a ⋅ b = 3(2n + 1), n ∈ N* rezultă că
ambele numere sunt impare, cel puŃin unul divizibil cu 3.

CLASA A XI-
XI-A

a b c
  1
1. b) Se deduce că A =  c a b  , a, b, c ∈ (0, ∞), c = şi detA = (a + b + c)(a2 + b2 + c2 – ab – ac – bc) ≥ 0;
b c a ab
 
1 1 − 1
   a b c
2. a) De exemplu A =  1 1 − 1 ; b) Fie B =   , cu proprietatea (P), atunci abc = –1; xyz = –1,
 − 1 − 1 − 1  x y z
 
ax = –1; by = –1; cz = –1. Rezultă abcxyz = 1 şi abcxyz = –1, fals; 3. a) f este descrescătoare pe (0, 2) şi
crescătoare pe (2, ∞); b) Notăm cu x latura bazei şi h înălŃimea rezervorului. Aria = A = x2 + 4xh şi cum
16
volumul = x2h = 4, rezultă că A = x 2 + şi este minimă pentru x = 2 şi h = 1; 4. a = 2, b = 1, c = 0.
x

CLASA A XII-
XII-A

1. b) Se observă că ultimul număr este x = (x1 – 1)(x2 – 1) ⋅ ... ⋅ (x4021 – 1) + 1, unde xi, i = 1, 4021 sunt numerele
a dx 1 1
date. Cum unul dintre numerele xi este egal cu 1, rezultă x = 1; 2. ∫ (x + 1)(2x
0 2
+ 1)
=  ln(x + 1) − ln(2x 2 +
 3 6
a
2   1 (a + 1) 2 2  1
+ 1) + arctg( x 2 )  , iar limita cu lim  ln 2 + arctg(a 2 )  = (π 2 − ln 2) ; 3. a) f este
3  a → ∞ 6 2a + 1 3  6
0  
1 a+b
continuă pe [0, 1] şi f(0) = 0; f(1) = 1, deci f ∈ F; ∫ f (x)dx = 1 −
0 2
; b) Conform a) alegem a, b ∈ [0, 1], a <

399
a+b 1
< b şi c, d ∈ [0, 1], c < d cu a + b = c + d; c) Alegem a, b ∈ [0, 1], a < b astfel încât 1 − = ;
2 2010
4. a) C 220 = 190 partide; b) În joc sunt 190 de puncte. Dacă x este numărul de puncte al ultimului clasat, cum
fiecare elev de pe locurile 1, 2, 3, .., 18 are cel puŃin 10 puncte, rezultă că suma tuturor punctelor S ≥ 10 ⋅ 18 +
+ 9,5 + x, rezultă x ≤ 0,5; c) Folosind b) avem posibilităŃile: x = 0 şi clasamentul 10,5; 10; 10; ...; 10; 9,5; 0 sau
x = 0,5 şi clasamentul 10; 10; ... ; 10; 9,5; 0,5.

Concursul NaŃional de Matematică aplicată „Adolf Haimovici”,


Etapa judeŃeană, 13 martie 2010
Profil real, specializarea ştiinŃele naturii

CLASA A IX-
IX-A

1. Avem că 3b2 = a ⋅ ab , prin urmare 10a2 + ab – 2b2 = 0. Rezultă că (5a + 3b)(2a – b) = 0. Cum 5a + 3b ≠ 0,
întrucât a, b sunt cifre şi a ≠ 0, obŃinem că b = 2a. Numerele căutate sunt 12, 24, 36 şi 48; 2. Cateta AB, opusă
AB 1
unghiului 'C de măsură 30°, este egală cu jumătate din ipotenuză, deci = . Din teorema bisectoarei,
BC 2
AE BA 1 AE CD BP 1 3 2
obŃinem că = = . Astfel, ⋅ ⋅ = ⋅ ⋅ = 1 şi concurenŃa dreptelor CP, AD şi BE urmează
EC BC 2 EC DB PA 2 1 3
B
conform reciprocei teoremei lui Ceva; D
3. Notăm cu AB şi CD înălŃimile celor doi brazi şi cu I poziŃia iepurelui.
Din triunghiurile dreptunghice BAI şi CDI, obŃinem că AI = 21 m, iar CI =
= 20 m. Dacă AS = x, IS = h, din triunghiurile dreptunghice SAI şi SCI, S
găsim că x2 + h2 = 441, respectiv (13 – x)2 + h2 = 400. Scăzând membru cu A C
105 252
membru, deducem că x = m, apoi h = ≅ 19,38 m;
13 13
I
4. Notăm cu x numărul numerelor 1,1 rămase şi cu y numărul numerelor 1,11 rămase. Atunci 1,1 ⋅ x + 1,11 ⋅ y =
= 19,93, adică 110x + 111y = 1993. Urmărind ultima cifră, deducem că y se termină în 3. Pe de altă parte y ≤ 20,
prin urmare y ∈ {3, 13}. Verificând, reŃinem doar valoarea y = 13, când x = 5. Astfel Lucică cel obraznic a şters
15 de 1,1 şi 7 de 1,11, în total 22 de numere.

CLASA A X-
X-A

 1 1 
1. a) Impunem condiŃiile x ≥ 0, 1 – x ≥ 0, x − 1 − x ≠ 0 şi x + 1 − x ≠ 0 . ObŃinem că x ∈ 0,  ∪  , 1 ;
 2 2 
( x + 1− x ) − ( x − 1− x ) 2 1− x  1 1 
b) E ( x ) = = ; c) Cum 2 1 − x ≥ 0, (∀) x ∈ 0,  ∪  , 1 , se impune
( x − 1 − x )( x + 1 − x ) 2x − 1  2 2 
 1
condiŃia 2x – 1 < 0. ObŃinem soluŃia x ∈ 0,  ; 2. Observăm că lg(tg 1°) + lg(tg 89°) = lg(tg 1° ⋅ tg 89°) =
 2
= lg(tg 1° ⋅ ctg 1°) = lg 1 = 0. Grupând câte doi termeni egal depărtaŃi de capete şi procedând analog, obŃinem 44

400
de perechi cu suma 0. Rămâne în mijloc termenul lg(tg 45°) = lg 1 = 0. În concluzie, S = 0; 3. a) De exemplu,
1 3 1 3
z1 = 1, z 2 = +i , z3 = − i ; b) (w1 + w2 + w3)2 = w12 + w 22 + w 32 + 2 w1w 2 + 2w1w 3 + 2 w 2 w 3 ⇒ (w1 +
2 2 2 2
 1 1 1  2
+ w2 + w3)2 = 2w1w2w3 ⋅  + +  ⇒ w 1 + w 2 + w 3 = 2 w 1 ⋅ w 2 ⋅ w 3 ⋅ w 1 + w 2 + w 3 ⇒ |w1 + w2 +
 w1 w 2 w 3 
+ w3|2 = 2 w 1 + w 2 + w 3 = 2 w1 + w 2 + w 3 ⇒ w1 + w 2 + w 3 = 2 (Am folosit w1 + w2 + w3 ≠ 0); 4. Colorăm
alternativ cu alb şi negru cele şase sectoare de disc. IniŃial, trei dintre pioni sunt pe negru şi trei sunt pe alb. La o
mutare, doi pioni îşi schimbă culoarea sectorului pe care se află. În urma unei mutări, rămâne un număr impar de
pioni pe negru şi un număr impar de pioni pe alb. Rezultă astfel că nu este posibil să adunăm toŃi cei şase pioni
pe o aceaşi culoare.

CLASA A XI-
XI-A

1. a) Dacă detA ≠ 0, atunci A este inversabilă în M2(Q). Presupunând că f(X1) = f(X2), unde X1, X2 ∈ M2,1(Z),
rezultă că AX1 = AX2, de unde A–1AX1 = A–1AX2, deci X1 = X2, ceea ce justifică injectivitatea funcŃiei f;
b) Când det A ∈ {–1, 1}, inversa A–1 a matricei A are tot elemente întregi. Pentru orice Y ∈ M2,1(Z) ecuaŃia
f(X) = Y are unica soluŃie X = A–1Y ∈ M2,1(Z), prin urmare f este bijectivă; 2. a) –Bt = –(A – At)t = –(At –
– (At)t) = –(At – A) = A – At = B; b) Cum detB = detBt, avem că detB = det(–Bt) = (–1)3 ⋅ detBt = –detB. Rezultă
n +1
1 1 
∑4
1 1
că 2detB = 0, deci detB = 0; 3. Observâm că Sn = = 1 −  , prin urmare, lim Sn = . Apoi, Pn =
i =1
i
3  4 n +1  n →∞ 3
n
 1  n +1

1 1
= (1 + 2 ) ⋅ i
= (1 + 2 ) 2 − n  şi atunci lim Pn = 2 + 2 2 ; 4. a) Avem că a n −1 − a n = − +
i =0 2  2  n →∞ n +1 2

n n (n + 1) − (n − 1)
+ = . Cum n(n + 1) > (n – 1)2, (∀) n ∈ N*, rezultă că a n +1 − a n > 0 , (∀) n ∈ N*, deci
2 2 n +1
1 n n
şirul (an)n∈N* este strict crescător; b) Observăm că a n > n ⋅ − = . Considerând, de exemplu, n =
n 2 2
4020 n
= 40202, obŃinem că a 4020 2 > = 2010 ; c) Cum a n > , rezultă că lim a n = +∞ , prin urmare şirul dat
2 2 n →∞
este divergent.

CLASA A XII-
XII-A

(e + 1) 2
1. a) F'(x) = f(x), (∀) x ∈ (0, ∞); b) Valoarea integralei este 2
(2 − ln(1 + e)) 2 − 2(1 − ln 2) 2 ; 2. a) Se
2e
verifică axiomele grupului; b) Grupurile nu sunt izomorfe; 3. (A, E, C) : (R, +); (A, E, C ) : (Sn, ⋅); (A, E, C) :
x+y
: (Z, &), unde x & y = 5xy + 5x + 5y + 4; (A, E, C) : (R+*, &), unde x & y = ; (A, E, C) : (M, &), unde M =
2
= {a, b, c, d} şi tabla este:

401
(A, E, C) : (M, &), unde M este mulŃimea cuvintelor cu opt litere alese dintr-un & a b c d
alfabet cu n litere, iar pentru x = a1a2....a8 şi y = b1b2...b8 definim x & y = a a b c d
= a 1a 2a 3a 4a 5b 6b 7b 8; b b d d d
c c c c c
d d b b b
9 ln x 3 2 ln t 3 ln t
4. Cu schimbarea de variabilă x = t2, obŃinem că ∫
1 x (x + 3)
dx = ∫
1 2
t(t + 3)
⋅ 2tdt = 4 ⋅ ∫
1 2
t +3
dt . Făcând

1 3 ln t 1 ln s 3 ln y − ln 3
schimbările de variabilă t = şi apoi 3s = y, avem că I =
s ∫1 2
t +3
dt = − ∫
1
3
3s 2 + 1
ds = − ∫
1 y2 + 3
dy = −I +

3
3 1 ln 3 y ln 3  1  π ln 3
+ ln 3∫ 1 2
y +3
dy , prin urmare, 2I =
3
arctg
31
= ⋅  arctg 3 − arctg
3 
 =
3 6 3
.

Concursul NaŃional de Matematică aplicată, „Adolf Haimovici”,


Etapa finală, 22 mai 2010
Profil real, specializarea ştiinŃele naturii

CLASA A IX-
IX-A

1. Dacă se notează cu x distanŃa dintre dreptele NP şi BC, se găseşte că MN = 5 – 2x, MQ = 6 – 3x şi x ∈ (0, 2).
Problema revine la rezolvarea ecuaŃiei (5 – 2x)(6 – 3x) = 3. Se obŃine în final MN = 2 m şi MQ = 1,5 m;
2. a) S(50) = 115; b) De exemplu, t1 = 15 şi t2 = 20; 3. Numerele sunt: 219, 220, ..., 228. A fost şters numărul
225; 4. a) Cum MA + MC ≥ AC (cu egalitate dacă punctele A, M, C sunt coliniare) şi MB + MD ≥ BD (cu
egalitate dacă punctele B, M, D sunt coliniare), suma dată este minimă atunci când M este punctul de intersecŃie
a diagonalelor; b) Notăm cu E, F, G mijloacele segmentelor AC, BD, respectiv EF. Dacă N este un punct
oarecare din plan, folosind teorema medianei în triunghiurile (eventual degenerate) NAC, NBD şi NEF, obŃinem
1 1
că NA2 + NB2 + NC2 + ND2 = 2(NE2 + NF2) + (AC2 + BD 2 ) = 4 NG 2 + EF 2 + (AC2 + BD 2 ) . Lungimile
2 2
AC, BD şi EF fiind constante, suma dată va fi minimă atunci când NG = 0, deci când N este mijlocul
segmentului determinat de mijloacele diagonalelor patrulaterului; c) Observăm că PA + PC = 2PE, PB + PD =
= 2PF , iar PE + PF = 2PG , prin urmare suma din enunŃ este egală cu 4PG . Modulul acestui vector este minim
când P = G.

CLASA A X-
X-A

1. SoluŃia 1: Fie z = a + bi, a, b ∈ R, a2 + b2 = 1. Inegalitatea de demonstrat revine la (2 – b)2 + a2 ≤ 9, deci la


(a2 + b2) + 4 – 4b ≤ 9, adică b ≥ –1. Însă, cum a şi b sunt reale, din a2 + b2 = 1 rezultă că b2 ≤ 1, de unde b ∈
∈ [–1, 1]. Egalitatea se atinge când b = –1. ObŃinem că a = 0, astfel că z = –i;
SoluŃia 2: Din inegalitatea modulului, |2 + iz| ≤ 2 + |i| ⋅ |z| = 3. Egalitatea se atinge când numerele complexe 2 şi

iz au acelaşi argument, deci când arg z = . Modulul lui z fiind 1, obŃinem că z = –i;
2

402
2. a) Considerăm un grup format din persoanele A, B, C, D, E şi F, în care perechile de cunoscuŃi sunt A – B,
A – C, B – C, D – E, D – F, E – F. Un astfel de grup este aproape unit, iar împărŃirea (A, B, C); (D, E, F)
îndeplineşte cerinŃele; b) Considerăm un grup format din persoanele A, B, C, D, E şi F, în care perechile de
cunoscuŃi sunt A – D, A – E, B – D, B – F, C – E, C – F. Un astfel de grup este aproape unit, iar împărŃirea
(A, B, C); (D, E, F) îndeplineşte cerinŃele; 3. Succesiunea operaŃiilor care se efectuează este: 2, 0, 1, 0 → 2, 2,
1, 0 → 2, 2, 1, 2 → 2, 2, 3, 2 → 4, 2, 3, 2 → 4, 4, 3, 2 → 4, 4, 3, 4 → 4, 4, 5, 4 → ... → 2008, 2008, 2007, 2008
→ 2008, 2008, 2009, 2008 → 2010, 2008, 2009, 2008. Suma numerelor aflate iniŃial pe tablă este 3, iar suma
finală este 8035, deci suma numerelor se măreşte cu 8032. Cum la fiecare pas suma se măreşte cu 2, sunt
necesari 8032 : 2 = 4016 paşi; 4. a) Cum fa(1) = 2, (∀) a ∈ R, punctul A(1, 2) aparŃine graficelor tuturor
funcŃiilor fa; b) Deoarece graficul lui fa este mediatoarea segmentului OMa, rezultă că OA = AMa, deci AMa =
= 5 , (∀) a ∈ R. Prin urmare, punctele Ma sunt toate situate pe un cerc de centru A şi rază 5 . DistanŃa
maximă dintre două astfel de puncte se atinge atunci când ele sunt diametral opuse şi este egală cu 2 5 .

CLASA A XI-
XI-A

 − 10 11 − 7 
 
1. Da, este posibil; A =  1 − 2 − 5  ; 2. a) FuncŃia este derivabilă, cu derivata f'(x) = 3x2 – 12x + 9.
 3 − 15 6 

Punctele critice sunt x1 = 1 şi x2 = 3. Avem că lim f ( x ) = −∞ , f(1) = a + 4, f(3) = a, lim f ( x ) = ∞ . Folosind
x → −∞ x →∞
şirul lui Rolle sau urmărind variaŃia funcŃiei, obŃinem că graficul intersectează axa Ox în trei puncte distincte
dacă şi numai dacă a + 4 > 0 şi a < 0, deci când a ∈ (–4, 0). Cum a este întreg, reŃinem valorile a ∈ {–3, –2, –1};
b) Abscisa punctului de intersecŃie cu Ox care aparŃine intervalului (1, 3) este număr întreg dacă şi numai dacă
este egală cu 2. CondiŃia f(2) = 0 conduce la a = –2, iar în acest caz f(x) = (x – 2)(x2 – 4x + 1). Cum zerourile
expresiei din paranteză sunt numere iraŃionale, răspunsul la întrebarea problemei este negativ; 3. Considerând, de
0 1
exemplu, A =   , avem că A2 = O2 = O 22 , deci f(A) = f(O2). Deducem că f nu este injectivă. Pentru orice
 0 0 
matrice A cu elemente reale, det(A2 + I2) = det(iA + I2) ⋅ det(–iA + I2) = det(iA + I2) ⋅ det(iA + I 2 ) =| det(iA +
+ I 2 |≥ 0 , deci funcŃia nu ia valori negative; 4. Gândim cele două drumuri ca formând un sistem cartezian,
orientarea axelor fiind dată de sensurile iniŃiale ale celor două mobile. Măsurând timpul din momentul
schimbării sensului de mers, la momentul t ∈ [0, 2], mobilul A se află în punctul (160 – 100t), iar B se află în
punctul (0, 200 – 80t). DistanŃa dintre ele este (160 − 100 t ) 2 + (200 − 80t ) 2 . Trebuie să studiem variaŃia
80
funcŃiei f: [0, 2] → R, f ( t ) = 20 41t 2 − 160 t + 164 . Se constată că f are un punct de minim pentru t = , deci
41
distanŃa minimă între cele două mobile se atinge după aproximativ 1,95 ore.

CLASA A XII-
XII-A

1. a) (Z19*, ⋅) este grup finit de ordin 18, prin urmare b̂18 = 1̂ , b ∈ Z19*; b) g, h au aceleaşi 18 rădăcini. Dacă f =
= g – h ∈ Z19[X], atunci f (b̂) = 0̂ , (∀) b ∈ Z19* şi gradf < 18, deci f ≡ 0 şi atunci g = h; c) Ambele sume sunt
egale cu 0̂ , lucru uşor de argumentat cu relaŃiile lui Viéte pentru polinomul g şi utilizând punctul b);

403
1 1 1 1
∫ ∫t
p −1
2. a) β(2, 2) = t (1 − t )dt = ; β(p,1) = dt = ; b) Cu schimbarea de variabilă t = 1 – u ⇒ β(p, q) =
0 6 0 p
' 1
1  1 t p p 1 p
  (1 − t ) q −1 dt = t (1 − t ) q −1 − t (q − 1)(1 − t ) q − 2 ⋅ (−1)dt =
= β(q, p); c) β(p, q) =
0∫ t p −1 (1 − t ) q −1 dt =
0 p 
 
∫ p
0
0 p ∫
q −1 1 p q −1 q −1
=
p 0∫
⋅ t (1 − t ) q − 2 dt =
p
⋅ β(p + 1, q − 1) ⇒ β(p, q) =
p
⋅ β(p + 1, q − 1) ; 3. Avem n(t) > 0, (∀) t ≥ 0 şi

n' (t)
= k ⇒ ln'(n(t)) = k ⇒ ln n(t) = kt + C ⇒ n(t) = ekt+C, (∀) t ≥ 0. Dar n(0) = b0 = eC ⇒ n(t) = b0 ⋅ ekt, (∀) t ≥
n (t)
t
1
≥ 0. Pentru b0 = 300, k = ⇒ n(t) = 300 ⋅ e 10 , (∀) t ≥ 0. Fie t ≥ 20 ⇒ n(t) ≥ n(20) = 300 ⋅ e2 > 300 ⋅ 2,72 =
10
= 2187 > 2010; 4. Se află înălŃimea trapezului h = asin α, α = m('BAD). Se află baza mare a trapezului de
(AD + BC) ⋅ h (2a + 2a cos α )
secŃiune AD = a + 2acos α. Aria trapezului este A = = = a 2 (1 + cos α )(sin α ) .
2 2
 π
Considerând funcŃia f:  0,  → R, f(α) = a2(1 + cos α)⋅ sin α, f'(α) = a2(2cos2 α + cos α – 1); f'(α) = 2a2 ⋅
 2
 1
⋅(cos α + 1) ⋅  cos α −  ;
 2
α π π
0
3 2 π
Aria(f) este maximă pentru α = .
f'(α) + + + + +0 – – – – 0 3
f(α)

Concursul NaŃional de Matematică aplicată „Adolf Haimovici”, Etapa finală,


22 mai 2010
Filiera tehnologică: profil tehnic

CLASA A IX-
IX-A

1. a) O(0, 0); A(4, 0); b) V(2, 4); c) S > SVOA = 8 şi S < aria pătratului de latură 4, deci S < 16; 2. a) k ≤
1
≤ min(2x2 – 3x + 1) = − ⇒ kmax = –1; b) Notăm 2x2 – 3x + 1 = y, rezultă y2 – 7y + 6 ≤ 0, deci y ∈ [1, 6], deci
8
2x2 – 3x + 1 ∈ [1, 6], x ∈ Z. Rezultă x ∈{–1, 0, 2}; 3. a) Notăm numărul de prune culese din prunul cu numărul
n cu xn, observăm că (xn)n≥1 este o progresie aritmetică cu x1 = 2 şi r = 3, deducem că xn = 3n – 1; b) Numărul
n (3n + 1) n (3n + 1)
prunelor culese este . Din ≥ 2010 rezultă n ≥ 37; 4. a) Deosebim două cazuri: 1) triunghiurile
2 2
au câte un vârf pe fiecare latură; rezultă 27 de triunghiuri; 2) două vârfuri sunt pe o latură şi al treilea pe altă
latură, avem 54 de triunghiuri; b) Dacă luăm două vârfuri roşii pe o latură, avem 12 triunghiuri roşii. Dacă luăm
câte un vârf roşu de pe fiecare latură, mai avem 8 triunghiuri. La acestea se adaugă şi un triunghi cu vârfurile
albastre. Sunt deci 21 de triunghiuri cu vârfurile de aceeaşi culoare.

404
CLASA A X-
X-A

1 1 1
1. a) z ∈ {cos t – isin t, cos t + isin t}; b) z 4 + = −2 şi z 8 + 8 = 2 ; c) z1024 + 1024 = 2 ; 2. a) x = 1; b) n =
z4 z z
 12 18 
= 10. Termenul căutat este T7; 3. a) MB: 5x + 3y – 6 = 0; b) Punctul de intersecŃie este P ,  ; 4. a) S-au
 19 19 
extras 5 bile verzi şi 5 bile roşii: B1 , B 2 , B3 , B 4 , B5 , B6 , B 7 , B8 , B9 , B10 . Dacă ultima bilă extrasă este verde,
verde rosie verde verde
a noua este tot verde, deoarece în caz contrar extragerea s-ar fi oprit după 8 bile extrase. A opta bilă extrasă este
roşie, deoarece în caz contrar, ultimele 3 bile ar avea aceeaşi culoare, în contradicŃie cu ipoteza. A şaptea bilă
extrasă este verde, pentru că altfel extragerea s-ar fi oprit după doar 6 bile extrase; b) Dacă B5 şi B6 sunt verzi,
s-ar obŃine trei bile extrase consecuitive, de aceeaşi culoare. Dacă B5 şi B6 sunt roşii, atunci extragerea s-ar fi
oprit după patru bile extrase, contradicŃie. Aşadar B5 şi B6 au culori diferite.

CLASA A XI-
XI-A

1 a 
1. b)   ∈ M, a ∈ M; c) Dacă A, B ∈ M, atunci detA, detB ∈ {–1, 1}. Rezultă det(AB) = detA ⋅ detB ∈
0 1
∈ {–1, 1}, deci A ⋅ B ∈ M; 2. a) f continuă ⇔ 3 + a = b = 1 + c; b) a = 0, b = 3, c = 2; 3. a) De exemplu, A =
 1 2 3 1 4 7 3 1 8
     
=  4 5 6  şi B =  2 5 8  ; b) 9!; c) De exemplu: 5 6 2 .
7 8 9 3 6 9 4 7 9
     

CLASA
CLASA A XII-
XII-A

1 1 2
 
 3 x x + C, x ∈ [0, 1]
0 4
2 2 
1. b) F = O3; c) E =  0
1
0 0  ; 2. b) F: R → R, F(x) =
1
 3
 x − x 2 + 2x − 2 , x ∈ 1, 4 
; c) V = π ∫0
3 f 2 ( x )dx =
2 0 
 2   3 3  3
4
1 436π 6+ 2 6− 2

= π xdx + π
0 ∫1
3 (x 2 − 2x + 2) 2 dx = π +
535
; 3. a) x1,2 = ± 2 + 3 = ±
2
; x 3,4 = ± 2 − 3 = ±
2
;

π
b) P = (x2 + x + 1)(x2 – x + 1); 4. a) f''(x) = –sin x ≤ 0, c ∈ [0, π]; b) l = ∫
0
1 + cos 2 x dx > OA + AB , unde

π  π2
A , 1 , B(π, 0). Cum OA = AB = + 1 , rezultă că l > π 2 + 4 .
2  4

405
Concursul NaŃional de Matematică aplicată „Adolf Haimovici”,
Etapa judeŃeană, 13 martie 2010
Filiera tehnologică: profil tehnic

CLASA A IX-
IX-A

1. a) Progresiile sunt de forma {a, a + r, a + 2r} ⊂ M. Găsim 20 de progresii; b) Progresiile sunt de forma {a, aq,
aq2} ⊂ M (deci a ∈ M dar q nu este neapărat din M). Găsim progresiile {1, 2, 4}; {1, 3, 9}; {2, 4, 8}; {4, 6, 9};
(L + l) 2 k 2
2. a) Dacă L, l sunt dimensiunile dreptunghiului şi L + l = k (constant), S = L ⋅ l ≤ = . Deci Smaxim =
4 4
k2 k L2 + l 2
= pentru L = l = ; b) Fie l, L dimensiunile dreptunghiului şi Ll ≤ = 2R 2 , rezultă Smax = 2R2 pentru
4 2 2
L = l; 3. b) Dacă a2 – 2b2 = 1, atunci (a2 + 2b2)2 – 2 ⋅ (2ab)2 = 1. Acum putem determina perechi ideale din
aproape în aproape: (3, 2) este ideală, rezultă (17, 12) este ideală; 4. Dacă sunt k bârne b1, b2, ..., bk, iar bârna bi a
fost tăiată de xi ori, obŃinem xi + 1 bucăŃi, i = 1, k . Deci x1 + x2 + ... + xk = 1510 şi (x1 + 1) + (x2 + 1) + ... + (xk +
+ 1) = 2010. Rezultă k = 500.

CLASA A X-
X-A

(a + b ) 2
1. a) 4; b) ab ≤ = 4; 2. b) Observăm că ε2 + ε + 1 = 0. Fie z1 = a1 + b1ε, z2 = a2 + b2ε. Atunci z1z2 =
4
= a1a2 + ε(a1b2 + a2b1) + b1b2ε2 = a1a2 + ε(a1b2 + a2b1) + b1b2(–ε – 1) = a + bε ∈ H; c) |a + bε| = 1 ⇒ (2a – b)2 +
+ 3b2 = 4 şi cum a, b ∈ Z rezultă b = –1, 0 sau 1. Găsim soluŃiile (a, b) ∈ {(1, –1); (0, –1); (1, 0); (–1, 0); (1, 1);
π 4π x
(0, 1)}; 3. a) x = 1; b) tgx = 3 , x ∈ [0, 2π]; x = , ; c) fn(x) = , n ∈ N*; 4. a) d6 = 9; b)
3 3 1 + nx 2
Adăugând un vârf unui poligon cu n laturi (deci dn diagonale) se adaugă n – 1 diagonale, deci dn+1 = dn + n – 1;
c) Putem folosi inducŃia matematică şi b).

CLASA A XI-
XI-A

1. a) detA = 0, deci A nu este inversabilă; b) Răspunsul este afirmativ, transpusa se obŃine în urma operaŃiilor de
adunare de 4 ori asupra liniei 1 şi de două ori asupra liniei 2, urmate de operaŃiile de scădere de 4 ori asupra
coloanei 1 şi de 2 ori asupra coloanei 2; 2. Aria triunghiului An–1AnAn+1 este egală cu 1, (∀) n ∈ N*, n ≥ 2;
1 1 1
3. a) 2; b) 2 + 4 + 6 + ... + 4020 = 2010 ⋅ 2011; 4. a) L(m) = e–1–|m|; b) L(m1) ⋅ L(m2) = ⋅ 1+ |m | + | m | ≤ ⋅
e e 1 2 e
1 1
⋅ 1+ | m + m | = L(m1 + m 2 ) .
e 1 2 e

406
CLASA A XII-
XII-A

x2
 + 3x + C, x < 1
3 1 2
1. b) x +
4
ln( x − 2) + ln( x + 2) + C ; 2. a) a = 1; b)
4 ∫ f ( x )dx = 
3 2
 x + x + 2 x + 2 + C, x ≥ 1
; 3. a) (Z5, +);

 3 2 3
b) G este ciclic de ordin 5; 4. A3 = I3 şi G = {I3, A, A2}.

Olimpiada NaŃională de Matematică,


Etapa judeŃeană şi a Municipiului Bucureşti, 13 martie 2010

CLASA A IX-
IX-A

IB + pIA
1. i) Avem IP = şi a IA + bIB + cIC = 0 , de unde concluzia; ii) Analog i), a (1 + q)IQ = (a − cq)IC −
1+ p
a − pb cp
− bq IB . Cum P, I, Q sunt coliniare, obŃinem = , de unde concluzia; iii) Conform ii) din a2 =
bq a − cq
AQ CD BP
= 4bcpq, rezultă (bp + cq)2 = 4bcpq sau (bp – cq)2 = 0, deci bp = cq. Fie {D} = AI ∩ BC. Cum ⋅ ⋅ =
QC DB PA
1 b
= ⋅ ⋅ p = 1 , rezultă concurenŃa cerută. ObservaŃie. Punctul ii) rezultă uşor din teorema transversalei: P, I, Q
q c
BP CQ DI p(p + 1)
coliniare, dacă şi numai dacă CD ⋅ + BD ⋅ = BC ⋅ ; 2. Avem Sp = 2 p +1 − − 1 . Deoarece 2p <
PA QA IA 2
< Sp < 2p+1, pentru p ≥ 3, iar S0 = 20; S1 = 21; S2 = 22, rezultă p ∈ {0, 1, 2}; 3. Dacă x ∈ Z, deoarece [kx] = k[x],
n ([ x ] + [nx ])
pentru orice k ∈ N, relaŃia rezultă imediat. Reciproc, observăm că, din ipoteză, rezultă + [(n +
2
(n + 1)([x ] + [(n + 1) x ]
+ 1)x] = sau încă: n[nx] = [x] + (n – 1)[(n + 1)x], pentru orice n ∈ N*. Înlocuind pe n cu
2
n + 1 şi scăzând relaŃiile găsite, rezultă: 2[(n + 1)x] = [(n + 2)x] + [nx], pentru orice n ∈ N*. Cum [x] = x – {x},
x ∈ R, deducem că 2{(n + 1)x} = {(n + 2)x} + {nx}, pentru orice n ∈ N*. Astfel şirul an = {nx} este progresie
aritmetică şi cum şirul (an) este mărginit, rezultă că r = 0, deci {nx} = {x} pentru orice n ∈ N*. Rezultă
{(n + 1)x} = {nx}, deci (n + 1)x – [(n + 1)x] = nx – [nx], adică x = [(n + 1)x] – [nx], deci x ∈ Z; 4. Din f(1) +
+ f(2) + f(f(1)) = 1, rezultă f(1) ∈ {1, 2}. Dacă f(1) = 1, atunci f(2) = 2 şi inductiv f(n) = n, pentru orice n ∈ N*.
 n + 1, n impar
Dacă f(1) = 2, atunci f(2) = 1 şi inductiv f(n) =  = n − (−1)n .
 n − 1, n par

CLASA A X-
X-A

1. i) Dacă log2[x] = [log2x] = m ∈ Z, atunci [x] > 0, deci [x] ≥ 1, rezultă m ∈ N, [x] = 2m şi log2x ∈ [m, m + 1),
deci x ∈ [2m, 2m + 1) şi x ∈ [2m, 2m+1) rezultă x ∈ [2m, 2m + 1). Prin urmare x ∈ A = {x ∈ R | log2[x] = [log2x]}

407
atunci x ∈ ∪ [2 m
, 2m + 1) = B . Reciproc: Fie x ∈ B, deci există m ∈ N cu x ∈ [2m, 2m + 1) deci log2[x] =
m∈N
= [log2x] = m, adică x ∈ A. În concluzie A = B; ii) Analog i): [2x] = 2[x] = t ∈ N implică log2 t ≤ x < log2 (t + 1)
şi [x] = log2 t' ∈ N, deci x ∈ [log2 t, log2 (t + 1)) = [m, log2 (2m + 1)), m ∈ N. Reciproc: x ∈
∈ ∪ [m, log (2
2
m
+ 1)) , atunci x ∈ [m, log2(2m + 1)), de unde [2x] = 2[x] ; 2. În condiŃiile date r2n + arn + 1 ≥ r2n –
m∈N
1
–2rn + 1. Este suficient să arătăm că (rn – 1)2 ≥ (r – 1)2n. Cum inegalitatea se păstrează dacă înlocuim pe r cu ,
r
putem presupune că r ∈ (0, 1) (cazurile r = 0, r = 1 sunt evidente). În acest caz inegalitatea este echivalentă cu 1
– rn ≥ (1 – r)n şi cum r şi 1 – r sunt din (0, 1), avem rn + (1 – r)n ≤ r + (1 – r) = 1; 3. Se arată uşor că f este
injectivă. Pentru n = 0, rezultă f(0) = 0. Inductiv, arătăm că f(n) = n. Presupunem că f(0) = 0, f(1) = 1, ..., f(n) = n.
Deoarece f este injectivă, f(n + 1) ≥ n + 1, f(f(n + 1)) ≥ n + 1, f(f(f(n + 1))) ≥ n + 1, deci 6n + 6 = f(n + 1) +
+2f(f(n + 1)) + 3f(f(f(n + 1))) ≥ ≥ 6(n + 1), rezultă f(n + 1) = n + 1;
1 1 1 1
4. a) a n + a n + 2 = z n + n + z n + 2 + n + 2 ≥ z n + n + z n + 2 + n + 2 =
z z z z
 1  1  1 1
=  z +  z n +1 + n +1  = z + ⋅ z n +1 + n +1 = a1a n +1 > 2a n +1 , pentru orice n ∈ N*; b) Din a) an+2 – an+1 >
 z  z  z z
> an+1 – an, n ∈ N*, rezultă că şirul (an+1 – an) este strict crescător, prin urmare an+1 – an > a2 – a1, pentru orice n ∈
2 2
1 1  1 1 1
∈ N*. Dar a 2 − a1 = z 2 + 2
− z + = z +  − 2 ≥ z + − z + − 2 = a12 − a1 − 2 = (a1 − 2)(a1 + 2) > 0 ,
z z  z  z z
dacă presupunem a1 > 2. Rezultă an+1 – an > 0, pentru orice n ∈ N*, deci ak > a1 > 2, contradicŃie. Astfel a1 ≤ 2.

CLASA A XI-
XI-A

1. Din continuitatea lui f rezultă a1 + b1 = a2 + b2. Dacă a1 = a2, rezultă b1 = b2. Putem considera m1 = a1 = a2, n1 =
= b1 = b2, m2 – n2 = 0, ε = ±1. Presupunem a1 ≠ a2, deci b1 ≠ b2. Trebuie să avem: m1x + n1 + ε|m2x + n2| = a1x +
+ b1, pentru orice x ≤ 1 şi m1x + n1 + ε|m2x + n2| = a2x + + b2, pentru orice x > 1. Penru m2 > 0, găsim m1x + n1 +
+ ε|–m2x – n2| = a1x + + b1, pentru orice x suficient de mic şi m1x + n1 + ε|m2x + n2| = a2x + b2, pentru orice x
a + a2
suficient de mare. Rezultă m1 – εm2 = a1, n1 – εn2 = b1; m1 + εm2 = a2, n1 + εn2 = b2, şi de aici m1 = 1 , n1 =
2
b + b2 a − a1 b − b1 a1 − a 2 a + a2 b + b2
= 1 ; εm 2 = 2 ; εn 2 = 2 = = −εm 2 . Rezultă f (x ) = 1 ⋅x+ 1 + εm 2 | x − 1 | =
2 2 2 2 2 2
a + a2 b + b 2 a 2 − a1 a + a2 b + b2 a − a1 a − a2
= 1 ⋅x + 1 + ⋅ | x −1|= 1 ⋅x+ 1 + sign (a 2 − a1 ) ⋅ 2 ⋅x + 1 . Procedăm
2 2 2 2 2 2 2
analog în cazul m2 < 0; 2. Evident, f(x) = ax3 + bx2 + cx + d, a, b, c, d ∈ R. Dar f(x) = det(A + xB) = det t(A +
+ xB) = det(At + xBt) = det(–A + xB) = –det(A – xB) = –f(–x), pentru orice x ∈ R. Din f(x) = –f(–x) pentru orice
x ∈ R, rezultă b = 0 şi d = 0, deci f(x) = ax3 + cx. Cum f are o rădăcină multiplă, rezultă c = 0 şi atunci f(x) = ax3.
Prin urmare det(A + xB) = x3a, pentru orice x ∈ R. Pentru x = 1 găsim det(A + B) = a. Cum
det( A + xB) 1 
a = lim = lim det A + B  = det B , rezultă det(A + B) = detB; 3. Cum f este monotonă, f are în
x →∞ x x →∞ x 
fiecare punct limite laterale finite. Fie acestea f(x0 –) şi f(x0 +). Deoarece f este crescătoare, în orice x0 ∈ R,
f(x0 –) ≤ f(x0) şi cum f este crescătoare, f(f(x0 –)) ≤ f(f(x0)). Pe de altă parte pentru x < x0, f(x) < f(x0 –) şi deci

408
f(f(x)) ≤ f(f(x0 –)), pentru x < x0. Deoarece f  f este continuă, prin trecere la limită f(f(x0)) ≤ f(f(x0 –)). Rezultă
f(f(x0–)) = f(f(x0)) şi cum f este injectivă rezultă f(x0–) = f(x0), adică f este continuă la stânga în x0. Analog se
arată că f este continuă la dreapta în x0; 4. Alegem a0 = –1. Pentru fiecare n ∈ N*, notăm kn numărul de termeni
1
egali cu +1 din secvenŃa a1, a2, ..., an, restul fiind egali cu –1. Şirul a cărui limită o vrem , fie el xn, devine
2
2k n k
x n = k n n + 1 − k n n − 1 . Avem x n = . Rămâne să construim şirul (an) astfel că lim n =
n +1 + n −1 n →∞ n

1
= . Alegem an = +1 dacă şi numai dacă n = (2m)2, m ∈ N*. Prin urmare, (2kn)2 ≤ n < (2(kn + 1))2. De aici
2
k 1
lim n = .
n →∞ n 2

CLASA A XII-
XII-A

1. Dacă 1 + 1 ≠ 0, atunci x ≠ –x, pentru orice x inversabil, deci S = 0. În caz contrar observăm că xS = S, pentru
orice x inversabil. Prin urmare S2 = kS, unde k este numărul elementelor inversabile. Atunci, pentru k impar S2 =
= S, iar pentru k par, S2 = 0; 2. i) Pentru a ∈ G, fie C(a) = {b ∈ G şi ab = ba}. Din ipoteză C(a2) ⊂ C(a).`
Deoarece C(a) ⊂ ` C(a2), obŃinem C(a) = C(a2), pentru orice a ∈ G. Prin urmare C(a) = C(a2) = ... =
 1̂ a b 
 
2n
= C(a ) = C(e) = G , pentru orice a ∈ G, deci G este abelian; ii) Grupul (G, ⋅) al matricelor de forma  0̂ 1̂ c  ,
 
 0̂ 0̂ 1̂ 
 
a, b, c ∈ Z3 are proprietatea cerută. Într-adevăr A3 = I3 şi oricare ar fi A ∈ G, dacă A2B = BA2, atunci A–1B = BA–1,
deci AB = BA; 3. f are primitive pe [a, b] dacă şi numai dacă funcŃia g = f + 1 – f(c) are primitive pe [a, b].
Putem presupune că f(c) = 1. Cum f este continuă în c, există [α, β] ⊂ [a, b], astfel încât c ∈ (α, β) şi 0 < f(x) < 2,
oricare ar fi x ∈ [α, β]. Fie Fa, respectiv Fb, o primitivă a lui f pe [a, c), respectiv (c, b]. Deoarece Fa, Fb sunt
crescătoare pe [α, c) şi (c, β], rezultă că limitele lim Fa ( x ) = p, lim Fb ( x ) = q există şi sunt finite. FuncŃia
x →c x →c

Fa ( x ), x ∈ [a , c)

F: [a, b] → R, F( x ) = p, x=c este primitivă a lui f pe [a, b]; 4. i) Deoarece f' are proprietatea lui
F ( x ) + p − q, x ∈ (c, b]
 b
Darboux, rezultă f'(x) < 1, oricare ar fi x ∈ [0, 1] sau f'(x) > 1, oricare ar fi x ∈ [0, 1]. Din teorema lui Rolle,
există c ∈ (0, 1) astfel încât f'(c) = 0. Rezultă f'(x) < 1, pentru orice x ∈ [0, 1]. Cum g'(x) = f'(x) – 1 < 0, oricare
x2
ar fi x ∈ [0, 1], rezultă că g este strict descrescătoare; ii) Fie G: [0, 1] → R, G(x) = F(x) – , F primitivă a lui
2
f. G'(x) = f(x) – x este strict descrescătoare, deoarece G''(x) = f'(x) – 1 < 0, pentru orice x ∈ [0, 1]. Aplicând
 k k + 1  k +1 k +1   k + 1   k  2k + 1 
teorema lui Lagrange pe  , , k = 0, n − 1 , rezultă că f  − < n  F  − F  − <
n n 

 n  n   n  n n 2 
n −1 n
k k  k +1 n +1  1
∑ ∑ f  n  −
k
< f   − . Prin sumarea acestor inegalităŃi, obŃinem f − < n F(1) − F(0) −  <
n n k =0 
n  n  2 k =0
n −1 1

2
, cum F(1) – F(0) = ∫ f (x)dx = 0 şi f(0) = f(1), rezultă inegalitatea cerută.
0

409
Olimpiada NaŃională de Matematică
CLASA A IX-
IX-A

1 1
1. a) '(AD, EF) = (AE + FD) = ('B + 'A + 'C) = 90°; b) Scriem condiŃia dată în forma OA + OB + OC =
2 2
= OD + OE + OF , O centrul cercului circumscris, deci triunghiurile ABC şi DEF au acelaşi ortocentru. Astfel, în
triunghiul ABC ortocentrul coincide cu centrul cercului înscris, prin urmare triunghiul este echilateral; 2. Dacă
ma mb mc 3
pătratele laturilor sunt în progresie aritmetică, de exemplu, 2b2 = a2 + c2, atunci = = = şi are loc
c b a 2
m m m
asemănarea cerută. Reciproc, fie a ≤ b ≤ c, atunci ma ≥ mb ≥ mc, deci a = b = c = k . Dar m a2 + m 2b +
c c a
3 3
+ m c2 = (a 2 + b 2 + c 2 ) . Rezultă k = şi 2b2 = a2 + c2; 3. a) A2 ∪ A3 = {–1, 0} ∪ {–5, –4, –3, –2, –1, 0} =
4 2
1 1 1
= A3; b) Pentru n ≥ 4, dacă x ∈ An, atunci x ≤  + + ... +  x , deci x ≥ 0. Apoi, pentru x ∈ An, n ≥ 4, x ≥
 2 3 n
 x   x   x  x −1 x − 2 x − 3  x  x − (n − 1)
≥ + + ≥ + + (deoarece pentru x, n ∈ N,   ≥ ). Rezultă x ≤ 23. Cum
2 3 4 2 3 4 n n
23 ∈ A4, rezultă că maxA = 23; 4. Pentru a = b rezultă f(0) = 0, apoi f(a2) = f2(a), pentru orice a ∈ N. Deci f(1) =
= f2(1), de unde f(1) ∈ {0, 1}. Cum pentru funcŃia f0 ≡ 0 şi f1 = 1N avem f0(0) = 0 şi f1(1) = 1 şi f0, f1 ∈ F, rezultă
că mulŃimea căutată este {0, 1}; b) Dacă a, b ∈ N, a ≥ b şi f ∈ F, atunci f2(a) – f2(b) = f(a2 – b2) ≥ 0, deci f(a) ≥
≥ f(b), adică f este crescătoare. În cazul f(1) = 0, obŃinem f(4) = f2(2) = f2(2) – f2(1) = f(3), de unde f(7) = f2(4) –
n
– f2(3) = 0. Inductiv, f (7 2 ) = 0 şi cum f este crescătoare, rezultă f = f0. În cazul f(1) = 1, notăm f(2) = x,
obŃinem f(4) = x2, f(3) = x2 – 1, f(5) = f2(3) – f2(2) = x4 – 3x2 + 1, iar din f(16) = f2(5) – f2(3), deducem x4 = (x4 –
n n
– 3x2 + 1)2 – (x2 – 1)2. Deducem x = 2, apoi f (2 2 ) = 2 2 şi, din stricta monotonie, f = f1.

CLASA A X-
X-A

1. Notăm a0 = a, obŃinem imediat a1 = 2a şi pentru n = 2, a 22 − 2aa 2 − 8a 2 = 0 , deci a2 = 4a. Prin inducŃie arătăm
n
că an = 2na, pentru orice n. Presupunem că ak = 2ka, 0 ≤ k ≤ n, atunci 2a ⋅ a n +1 + a 2 ⋅ 2 n +1 ⋅ ∑C
k =1
k
n +1 = a 2n +1 sau

2a ⋅ an+1 + a2 ⋅ 2n+1 ⋅ (2n+1 – 2) = an+1, rezultă an+1 = 2n+1 ⋅ a; 2. Dacă există k ∈ [–1, 1] astfel încât w = kv,
v v v
inegalitatea se verifică. Reciproc: Deoarece = 1 , relaŃia este adevărată pentru z = − , rezultă − w + w ≤
v v v
w w w
≤ 0, deci = , deci ∈ R. Notăm w = kv, k ∈ R. Inegalitatea devine |k| ⋅ zv + v ≤ zv + v , pentru orice
v v v
z ∈ C, |z| = 1. Rezultă |k| ≤ 1. Deci k ∈ [–1, 1], ceea ce încheie soluŃia; 3. a) Dacă grupele conŃin a1, a2, ..., a10
puncte, atunci numărul de triunghiuri formate este N = C3a1 + C3a 2 + ... + C 3a10 . Arătăm că acest număr este minim
3
atunci când a1 = a2 = ... = a10 = 10, valoarea minimă fiind 10C10 . Într-adevăr, dacă există o grupă cu m ≥ 11

410
puncte, va exista o altă grupă cu n ≤ 9 puncte. Mutând un punct din prima grupă în cea de a doua grupă, numărul de
triunghiuri scade, deoarece C 3m + C 3n > C 3m −1 + C 3n +1 ; b) Alegem în fiecare grupă câte 10 puncte şi în fiecare grupă
formăm câte două subgrupe de 5 puncte. Deoarece poziŃiile punctelor nu contează, putem presupune că fiecare
subgrupă de 5 puncte formează un pentagon convex. Colorăm laturile lor cu culoarea c1, diagonalele cu culoarea c2,
iar segmentele ce unesc punctele din subgrupe diferite cu culoarea c3. Astfel nu există niciun triunghi monocolor;
4. Considerăm triunghiul ABC orientat direct trigonometric. Notăm cu x afixul punctului X. Din ipoteză,
m−b n −c p−a
= = = k . Rezultă m = ka + (1 – k)b, n = kb + (1 – k)c, p = kc + (1 – k)a. Deoarece MNP este
a −b b−c c−a
2π 2π
echilateral, avem m + εn + ε2p = 0, ε = cos + i sin . Înlocuind 0 = k(a + bε + cε2) + (1 – k)(b + cε + aε2) =
3 3
 1− k  1
= (a + bε + cε2)  k +  , cum ABC nu este echilateral, rezultă k = şi din relaŃia m = ka + (1 – k)b rezultă
 ε  1− ε
2π π
m – a = ε(m – b), deci AMB este isoscel, cu un unghi egal cu şi celelalte două egale cu .
3 6

CLASA A XI-
XI-A

1. A2 – 2aA + bI2 = (A – aI2)2 + (b – a2)I2 = B2 + c2I2, B = A – aI2, c = b − a 2 > 0 . Rezultă det(B + icI2)(B –
m n
– icI2) = 0, deci det(B + icI2) ⋅ det(B – icI2) = 0. Fie B =   , m, n, p, q ∈ R. Din det(B ± icI2) = 0, rezultă
 p q
mq – np – c2 ± ic(m + q) = 0, deci m + q = 0, mq – np – c2 = 0, de unde q = –m, m2 + np + b – a2 = 0. Matricele
m + a n 
A căutate sunt A =   , pn = a2 – b – m2, m ∈ R; 2. Fie K ∈ Mn,1(C), L ∈ M1,n(C), astfel încât
 p − m + a 
 a1 
 
a 
K ⋅ L = B. Atunci On = (AK) ⋅ (LB) cu AK ∈ Mn,1(C), LB ∈ M1,n(C). Fie AK =  2  ; LB = (b1 b 2 ... b n ) ,
...
 
a 
 n
 a1b1 a1b 2 ... a1b n 
 
deci O n =  ... ... ... ...  . Dacă LB ≠ O1,n atunci există 1 ≤ i ≤ n cu ki ≠ 0, rezultă a1 = a2 = ... = an =
 a b a b ... a b 
 n 1 n 2 n n
= 0, deci AK = On,1. Rezultă că AK = On, sau LB = O1,n. În primul caz AB = On, iar în al doilea BC = On;
3. Fie g monoton crescătoare pe R, atunci f(x + y) = ex+y ⋅ g(x + y) ≥ ex(1 + y) ⋅ g(x) = (1 + y)f(x) (deoarece ey ≥
≥ 1 + y, (∀) y ∈ R) pentru orice x ∈ R şi orice y ≥ 0. Reciproc: Fie f(x + y) ≥ (1 + y)f(x), (∀) x ∈ R şi (∀) y ≥ 0.
Se arată inductiv f(x + nt) ≥ (1 + t)n ⋅ f(x), (∀) x ∈ R, t ≥ 0, n ∈ N. Fie x, z ∈ R, x < z, y = z – x.
n n
 y  y  y
Avem: g(z) = e–zf(z) = e–x–y ⋅ f  x + n  ≥ e − x − y 1 +  f ( x ) , deci g(z) ≥ e − y 1 +  g ( x ) . Rezultă g(z) ≥
 n  n  n
n
 y
≥ lim e − y 1 +  g( x ) = g( x ) , de unde g este crescătoare pe R; 4. Are loc proprietatea: pentru orice n ∈ N*,
n →∞  n

411
1 1
există k ∈ N*, k ≥ n, astfel încât x k ≤ . Altfel, dacă pentru un anumit n ∈ N*, avem xk ≥ , (∀) k ≥ n, rezultă
k k
m m

∑ ∑ k , (∀) m ≥ n, de unde
1
x m +1 = x n + ( x k +1 − x k ) = x n − lim x m = −∞ , contradicŃie. Fie ε > 0 şi N ∈ N*
n →∞
k =n k=n

1  1   1 
cu < ε , există N1, N1 > N astfel încât x N1 ∈ 0,  şi prin inducŃie x n ∈ 0,  ⊂ [0, ε) , (∀) n ≥ N1.
N  N1   N1 
Rezultă că (xn) converge la 0.

CLASA A XII-
XII-A

1. Fie f crescătoare şi x0 ∈ R. Atunci, f(x0 – 0) şi f(x0 + 0) ale lui f în x0 există, sunt reale şi f(x0 – 0) ≤ f(x0) ≤
x0
≤ f(x0 + 0). Fie x < x0. Întrucât f(t) ≤ f(x0 – 0), x ≤ t < x0, rezultă că ∫
x
f ( t )dt ≤ ( x 0 − x )f ( x 0 − 0) . Prin urmare,
x0
F( x ) − F( x 0 )
=

x
f ( t )dt
≤ f ( x 0 − 0) . Întrucât F este derivabilă în x0, rezultă că F'(x0) ≤ f(x0 – 0). Analog
x − x0 x0 − x
F'(x0) ≥ f(x0 + 0). Deci f(x0 – 0) = f(x0 + 0) = f(x0); 2. a) Dacă A este corp şi x ∈ A, x ≠ 0, atunci x este inversabil
şi x = x + 0: scriere unică deoarece 0 este singurul element neinversabil. Presupunem că A este corp. Fie x ∈ A,
x ≠ 0, un element neinversabil. Întrucât 1 + x = (1 + x) + 0 şi –1 + x = (–1 + x) + 0, elementele 1 + x şi –1 + x
sunt neinversabile (în caz contrar, unicitatea scrierii ar implica x = 0). Deoarece x = 1 + (–1 + x) = –1 + (1 + x),
rezultă 1 = –1, contradicŃie; b) Fie E o mulŃime nevidă şi A = (P(E), ∆, ∩) inelul părŃilor lui E. În acest inel E
este singurul element inversabil. Dacă X este o submulŃime nevidă a lui E, atunci E – X este neinversabil şi X =
= E∆(E – X). Scrierea este unică, deoarece E este singurul element inversabil; 3. a) Întrucât G este grup, rezultă
că |xH| = p. Deci, n = |G| ≥ |H ∪ xH| = |H| + |xH| – |H ∩ xH| = 2p – |H ∩ xH|, de unde |H ∩ xH| ≥ 2p – n; b) Fie
x ∈ H şi y ∈ H ∩ xH. Atunci y = y–1 şi y = xh, h ∈ H. Întrucât xy = x2h = h ∈ H, rezultă xy = (xy)–1 = y–1x–1 =
3n n
= yx. Deci x comută cu toate elementele din H ∩ xH. Întrucât |H ∩ xH| ≥ 2p – n > − n = , rezultă că
2 2
n
subgrupul elementelor care comută cu x are cel puŃin elemente. Din teorema lui Lagrange, rezultă că x comută
2
3n n
cu toate elementele lui G. Deci H ⊂ ` Z(G), centrul lui G. Prin urmare |Z(G)| ≥ |H| = p > > , deci Z(G) =
4 2
= G; c) Presupunem că G este comutativ. Dacă x, y ∈ H, atunci (xy) = x y = e, deci xy ∈ H. Întrucât H este
2 2 2

n 3n
finită, rezultă că H este subgrup al lui G. CondiŃia |H| > forŃează H = G, deci ≥| H | = | G | = n , contradicŃie;
2 4
I( h ) − 2 f ( 0 ) h
4. a) FuncŃia continuă ϕ: (0, 1] → R, ϕ(h ) = , poate fi prelungită prin continuitate în 0, deoarece
h2
lim ϕ(h ) = 0 ; M = sup{|ϕ(h)| | 0 < h ≤ 1}, atunci |I(h) – 2f(0)h| ≤ Mh2, oricare ar fi h ∈ [0, 1]; b) Deoarece
h →0

||I(h) – 2|f(0)|h| ≤ |I(h) – 2f(0)h| ≤ Mh2, rezultă că 2|f(0)h – Mh2| ≤ |I(h)| ≤ 2|f(0)h + Mh2|, 0 < h ≤ 1 şi de aici,

412
n n n
 1  2 | f (0) |
∑ ∑ ∑k
2 | f (0) | M M 1 1 1
− ≤ k ⋅ I  ≤ + , k ∈ N*. Dar ≥ = n , n ∈ N* şi =
k k k  
k k k k k =1 k k =1 n k =1 k
n n

1   1 
∑k ∑ 
1 1
= 1+ ≤ 1+ 2  = 1 + 21 −
−  < 3 , n ∈ N*. Rezultă că:
k=2 k
k=2 k  k −1  n
1) Dacă f(0) = 0, atunci an ≤ 3M, oricare ar fi n ∈ N*, cum (an)n∈N* este crescător, el este convergent;
2) Dacă f(0) ≠ 0, atunci an ≥ 2|f(0)| n – 3M, oricare ar fi n ∈ N*, deci (an)n∈N* este nemărginit.

413
CUPRINS

enunŃuri soluŃii

1. etapa locală .................................................................................................................. 5 .............. 174


Clasa a IX-a ......................................................................................................................... 5 .............. 174
Algebră ........................................................................................................................ 5 .............. 174
Geometrie .................................................................................................................32 .............. 212

Clasa a X-a .........................................................................................................................61 .............. 274


Algebră ......................................................................................................................61 .............. 274
Geometrie .................................................................................................................83 .............. 305

Clasa a XI-a ........................................................................................................................85 .............. 308


Algebră ......................................................................................................................85 .............. 308
Analiză matematică ................................................................................................93 .............. 316

Clasa a XII-a ....................................................................................................................102 .............. 326


Algebră ....................................................................................................................102 .............. 326
Analiză matematică ..............................................................................................109 .............. 333

2. concursuri interjudeŃene ................................................................................118 .............. 345

3. etapa naŃională .....................................................................................................155 .............. 397


COMENZI – CARTEA PRIN POŞTĂ

EDITURA PARALELA 45
Piteşti, jud. Argeş, cod 110174, str. FraŃii Goleşti 130
Tel./fax: 0248 214 533; 0248 631 439; 0248 631 492
Tel.: 0753 040 444;
0721 247 918.
E-mail: comenzi@edituraparalela45.ro
sau accesaŃi www.edituraparalela45.ro

Tiparul executat la Graficprint


www.graficprint.eu
e-mail: comenzi@graficprint.eu

S-ar putea să vă placă și